Download NEET PG 2016 Question Paper with Answers

Download NEET PG (The National Eligibility cum Entrance Test Postgraduate) 2016 Question Paper with Answers

1.Zygotewithzonapellucidareachesuterine
cavityby:
a)2days
b)4days
c)5days
d)6days
CorrectAnswer-B
4days

2.Inspermatogenesis,independent
assortmentofpaternalandmaternal
chromosomesoccursduring-

a)Primarytosecondaryspermatocyte
b)Spermatogoniatoprimaryspermatocyte
c)Secondaryspermatocytetospermatids
d)Spermatidstospermatozoa
CorrectAnswer-A
Answer-A.Primarytosecondaryspermatocyte
Inspermatogenesis,independentassortmentofpaternaland
maternalchromosomesoccursduringmeiosisI,inwhichprimary
spematocyte)to(2n)isconvertedintotwosecondaryspermatocytes
(In).

3.Tonguemusclesarederivedfrom
a)Lateralplatemesoderm
b)Occipitalmyotome
c)Intermediatemesoderm
d)Cervicalmyotome
CorrectAnswer-B
Ans:BOccipitalmyotome
Developmentofthetongue:-
I.Epithelium:
Ant2/3--lingualswellingsof1starchandtuberculumimpar.
Post1/3--largedorsalpartofhypobranchialeminence,i.e,3rd
arch.
Posteriormostpart--smalldorsalpartofthehypobranchial
eminence,i.e.4tharch.
II.Muscles:
Derivedfromoccipitalmyotomesexceptpalatoglossuswhich
isderivedfromthe6tharch.


4.Whichofthefollowingisremnantofdistal
umbilicalartery?
a)LigamentumTeres
b)SuperiorVescicalartery
c)MedialumbilicalLigament
d)Ligamentumarteriosum
CorrectAnswer-C
Ans.C.MedialumbilicalLigament
Remnantofumbilicalartery:-

1. Proximalpart:Superiorvesicalartery
2. Distalpart:Medialumbilicalligament

5.Allarederivedfromneuralcrestexcept?
a)Adrenalmedulla
b)Pigmentcellinskin
c)Cornealstroma
d)Retinalpigmentedepithelium
CorrectAnswer-D
Ans.is`d'i.e.,Retinalpigmentedepithelium
Derivativesofneuroectoderm
1.Fromneuraltube:CNS(brain,spinalcord),astrocytes,
oligodendrocytes,ependymalcells,retina,pinealgland,
neurohypophysis(posteriorpituitary),allcranialandspinalmotor
nerves.
2.Fromneuralcrest:Neuralcrestderivativesare:?
3.Neuralderivatives
*Sensoryneuronsof5th,7th,8th,9th,10thcranialnerveganglia
(trigeminal,geniculate,sphenopalatine,submandibular,cochlear,
vestibular,oticandvagalparasympatheticganglia).
*Sensoryneuronsofspinaldorsalrootganglia.
*Sympatheticchaingangliaandplexus(celiac/preaortic/renal
ganglia,entericplexusinGIT,i.e.AuerbachsandMeissner's)
*ParasympatheticgangliaandplexusofGIT.
*Schwanncellsofperipheralnerves,satellitecellsofallganglia.
*Adrenalmedulla,chromaffincells,parafollicularC-cellsofthyroid
gland.
*Melanocytesandmelanoblasts.

6.Opticvesicleisderivedfrom-
a)Endoderm
b)Mesoderm
c)Neuroectoderm
d)SurfaceEctoderm
CorrectAnswer-C
Answer-C.Neuroectoderm
Anoutgrowthfromprosencephalonformsopticvesicle
(neuroectodermalstructure).
Proximalpartofopticvesiclebecomesconstrictedandelongatedto
formopticstalk
Growingopticvesiclecomesincontactwithsurfaceectodermwhich
isthickenedtoformlensplacode.

7.Digeorgesyndromeischaracterizedbyall
except?
a)Congenitalthymichypoplasia
b)Abnormaldevelopmentofthirdandfourthpouches
c)Hypothyroidism
d)Hypocalcemictetany
CorrectAnswer-C
Ans.is'c'i.e.,Hypothyroidism

8.

Partofneuraltubefromwhichcorpus
callosumdevelops

a)Basallamina
b)Alarlamina
c)Laminaterminalis
d)Basalplate
CorrectAnswer-C
Ans.is'c'i.e.,Laminaterminalis
Thedevelopmentofthecorpuscallosumoccursbetweenthe
12thand16-20thweeksofgestation6.
Itbeginswiththegenuandthencontinuesposteriorlyalongthe
bodytothesplenium.Therostrumisthelastparttobeformed.
Myelinationofthecorpuscallosumoccursintheoppositedirection,
fromthespleniumforwards.
Theydevelopfromlaminaterminaliswhichiscranialpartofneural
tube
andlaterliesintheanteriorwallof3'dventricle.
Thecorpuscallosum,thelargestofcerebralcommissures,takesthe
formofanarchoverthethirdventricle.
Itconnectstheneocorticesofbothsides.

9.Epithelialliningofurinarybladder?
a)Squamous
b)Transitional
c)Cuboidal
d)Columnar
CorrectAnswer-B
Ans.is'b'i.e.,Transitional
Urothelium(transitionalepithelium)isfoundinrenalpelvis,calyces,
ureter,urinarybladder,proximalpartofurethra.

10.Femaleurethradevelopsfrom-
a)Urogenitalsinus
b)Mesonephricduct
c)Uretericbud
d)MetanephricBlastema
CorrectAnswer-A
Ans.A.Urogenitalsinus
Thefemaleurethraismainlyderivedfromtheurogenitalsinuswhile
theurethralplateformsthevestibuleandlabiaminora.

11.Allofthefollowinghelpinformationof
IVCexcept-
a)Theposteriorintercardinalanastomosis
b)Terminalportionofrightvitellinevein
c)Segmentofrightcardinalvein
d)Subcardinalsinus
CorrectAnswer-D
Ans.D.Subcardinalsinus
Theinferiorvenacavaiscomposedof(fromcaudaltocranial):

1. Posteriorintercardinalanastomosis.
2. Thecaudalportionoftherightsupracardinalvein.
3. Therightanastomosisbetweenthesupracardinalandthe
subcardinalveins.
4. Asegmentoftherightsubcardinalvein.
5. Theanastomosisbetweentherightsubcardinalandrightvitelline
veins.
6. Theterminalportionoftherightvitellinevein.

12.Kidneyparenchymaisderivedfrom-
a)Uretericbud
b)Mesonephros
c)Metanephros
d)Paramesonephros
CorrectAnswer-C
Ans.C.Metanephros
Metanephros:Thissystemwillformthenephronsandparenchyma
ofthedefinitivekidney.

13.Facialnerveisaderivativeofwhichofthefollowingbranchialarch?
a)Firstarch
b)Secondarch
c)Thirdarch
d)Fourtharch
CorrectAnswer-B
Facialnerveisaderivativeofsecondbranchialarch.Musclesoffacialexpressionderived
fromitarebuccinator,auricularis,frontalis,platysma,orbicularisorisandorbicularisoculi.
Additionalmusclessuppliedbyitarestapedius,stylohyoid,andposteriorbellyofdigastric.
Branchial
Cranialnerve
Muscles
arch
First
Trigeminal
Musclesofmastication:masseter,temporalis,medial
mandibular
andlateralpterygoid.Additionalmuscles:mylohyoid,
anteriorbellyofdigastric,tensortympani,tensorveli
palatini
Third
Glossopharyngeal
Stylopharyngeus
Fourthand
Superior
Pharyngealandlaryngealmuscles:cricothyroid,
sixth
laryngeal,
levatorvelipalatini,constrictorsofpharynx,intrinsic
recurrent
musclesoflarynx
laryngeal
branchesof
vagus
Ref:NeurosciencefortheStudyofCommunicativeDisordersBySubhashChandra
Bhatnagarpage279.

14.Leptoteneandpachytenearestages
ofwhichphasesofmeiosis-
a)ProphaseI
b)MetaphaseI
c)AnaphaseII
d)TelophaseII
CorrectAnswer-A
Ans.A.ProphaseI
Meiosis1isdividedintofollowingphases:-
l)Prophase1:Itisfurtherdividedintofollowingstages:-
1. Leptotene
2. zygotene
3. Pachytene
4. Diplotene
5. Diakinais
2)Metaphase1
3)Anaphase1
4)Telophase1

15.Pelvickidneysareduetoallexcept?
a)Inabilitytoascendduringfetallife
b)Fusionofthelowerpoles
c)Beingblockedbybranchesoftheaorta
d)p53mutation
CorrectAnswer-D
Ans.D.p53mutation
Pelvickidney
Afetalpelvickidneyisaconditionthatresultswhenthekidneysfail
toascendtotheirnormalpositionabovethewaistandremaininthe
pelvisbecausetheyareblockedbybloodvesselsintheaorta.
Developingkidneysmayalsofusetogethercausingwhatisknown
asa'horseshoekidney"
Afetalpelvickidneyorhorseshoekidneyisgenerallydiagnosedby
ultrasound(sonogram)examinationbeforebirth.
Evaluationofthekidneysispartoftheroutineultrasound
examinationdonebymanyobstetriciansaspartoftheirprenatal
carearoundthe20sweekofpregnancy.

16.Cryptamagnadevelopsfromwhich
pouch?
a)1st
b)2nd
c)3rd
d)4th
CorrectAnswer-B
Ans.B.2nd
Medialsurfaceofeachtonsilhas15-20crypts,thelargestofwhich
iscalledIntratonsillarcleftorcryptomagna(whichrepresents
persistenceoftheventralportionofthesecondpharyngealpouch).

17.Whichofthefollowingisatraction
epiphysis?
a)DistalRadius
b)Mastoidprocess
c)TibialCondyles
d)CoracoidProcess
CorrectAnswer-B
Ans.B.Mastoidprocess

18.Allofthefollowingaretrueabouttheliver
except?
a)ItiscoveredbyGlisson'scapsule
b)StellatecellsarepresentinthespaceofDisse
c)Kupfercellsarethedefensecells
d)Thelobulesintheliverarepentagonal
CorrectAnswer-D
Ans.D.Thelobulesintheliverarepentagonal
Liverhashexagonallobules

19.Hering'scanalispresentin?
a)Spleen
b)Liver
c)Kidney
d)Lung
CorrectAnswer-B
Ans,B.Liver
1hecanalofHeringorintrahepaticbileductulesarepartofoutflow
systemofexocrinebileproductfromtheliver.
Theyarefoundbetweenthebilecanaliculiandinterlobularbileducts
neartheouteredgeofliverlobule.

20.Herring'sbodiesarepresentin?
a)Parstuberalis
b)Parsintermedia
c)Neurohypophysis
d)Parsterminalis
CorrectAnswer-C
Ans.C.Neurohypophysis
Herringbodiesorneurosecretorybodiesarestructuresfoundinthe
posteriorpituitary(neurohypophysis).

21.Allofthefollowingaretrueaboutthymus
except?
a)Thecorticalportionismainlycomposedoflymphocytes
b)ThemedullacontainsHassall'sCorpuscles
c)ItisderivedfromthefourthPharyngealpouch
d)Itundergoesatrophypubertyonwards
CorrectAnswer-C
Ans.C.ItisderivedfromthefourthPharyngealpouch
Thethymusisaspecializedprimarylymphoidorganoftheimmune
system.Withinthethymus,TcellsorTlymphocytesmature.
Thethymusislargestandmostactiveduringtheneonatalandpre-
adolescentperiods.Bytheearlyteens,thethymusbeginstoatrophy
andthymicstromaismostlyreplacedbyadipose(fat)tissue
Thymusisderivedfromthethirdpharyngealpouch

22.Auerbachsplexusispresentinthe-
a)Colon
b)Esophagus
c)Stomach
d)Alloftheabove
CorrectAnswer-D
Alloftheabove

23.Whichofthefollowinglayerisabsentin
theesophagusa-
a)Adventitia
b)Serosa
c)Muscularispropria
d)Mucosa
CorrectAnswer-B
Ans.B.Serosa
Theesophagusalsohasanadventitia,butnotaserosa

24.Whichisthemostabundantcartilage-
a)Hyalinecartilage
b)Elasticcartilage
c)Fibrocartilage
d)None
CorrectAnswer-A
Ans.A.Hyalinecartilage

25.Haustrationsarepresentin-
a)Duodenum
b)Ileum
c)Jejunum
d)Colon
CorrectAnswer-D
Ans.is'd'i.e.,Colon[RelBDC#/eVol.2p.2661
1. Characteristicsfeaturesoflargeintestine(colon)are:-i)
3longitudinalbands,formedbylongitudinalmusclecoat,called
Taeniaecoli.
2. Sacculationorhaustration
3. Fatfilledperitonealpouchescalledappendicesepiploicae.
Thesearenotfoundinappendix,caecumandrectum.
4. Greaterpartisfixedexceptforappendix,transversecolonand
sigmoidcolon.
5. Pyere'spatches(presentinsmallintestine)arenotpresent.

26.Whatistheliningofthelacrimalgland
alveoli?
a)Ciliatedcolumnarcells
b)Pyramidalcells
c)Nonkeratinizingsquamousepithelium
d)None
CorrectAnswer-B
Ans.B.Pyramidalcells
Alveolioftheglandarelinedbypyramidalcells,whichshowlightly
stainedapicalsecretarygranules.

27.Whattypeofmusclesaremedialtwo
lumbricals?
a)Unipennate
b)Bipennate
c)Multipennate
d)None
CorrectAnswer-B
Ans.B.Bipennate

28.Ansanephroniislinedby?
a)Columnar
b)Squamousepithelium
c)Cuboidalandcolumnarepithelium
d)Stratifiedsquamousepithelium
CorrectAnswer-B
Bi.e.Squamousepithelium

29.Fourcarpalbonesarepresentatwhat
age?
a)3years
b)4years
c)5years
d)6years
CorrectAnswer-B
4yearsREF:Parikh6'editionpage2.9
Between2to6years,thenumberofcarpalbonespresentonXray
representstheapproximateageinyears,asforexample,fourcarpal
bones--4years.

30.Whatisthelevelofthespineofscapula?
a)T7
b)TI0
c)T4
d)T2
CorrectAnswer-C
Ans.C.T4
SpineofscapulaisatT3T4level

31.Whichofthefollowingmusclescarries
outshoulderabductionfrom15to90
degrees?

a)Suprapinatus
b)Trapezius
c)Deltoid
d)SerratusAnterior
CorrectAnswer-C
Ans.C.Deltoid

32.Whichmuscleactingonthethumbhas
dualnervesupply?
a)FlexorPollicisLongus
b)FlexorPollicisbrevis
c)AdductorPollicis
d)OpponensPollicis
CorrectAnswer-B
Ans.B.FlexorPollicisbrevis

33.WhatisWartenberg'ssign?
a)InabilitytomaintainIntrinsicplusposition
b)Inabilitytoadductsmallfingeragainsttheringfinger
c)Inabilitytograspabookbetweenthethumbandindexfinger
d)Inabilitytomovethemiddlefingersideways
CorrectAnswer-B
Ans,B.Inabilitytoadductsmallfingeragainsttheringfinger
Wartenberg'ssignisinabilitytoadductthesmallfingerinagainstthe
ringfngerduetoweaknessofpalmarinterosseousmuscles.

34.Sensoryregionoftheulnarnerveis?
a)Tipoflittlefinger
b)Tipofindexfinger
c)1stwebspace
d)Lateralupperaspectofarm
CorrectAnswer-A
Ans.A.Tipoflittlefinger

35.Pulpoftheindexfingerissuppliedby
a)Mediannerve
b)Radialnerve
c)Ulnarnerve
d)Axillarynerve
CorrectAnswer-A
Ans,A.Mediannerve

36.Lowradialnerve[justafterspiralgroove]
palsydoesnotproduce?
a)Lossofwristextention
b)Lossofelbowextention
c)Lossoffingerextention
d)Lossofthumbextention
CorrectAnswer-B
Ans.B.Lossofelbowextention
Lowradialnervepalsy
Injuryisafterthespirdgroove.
Lowradialnervepalsymaybeoftwotypes:
i)TypeI:-Injuryoccursbetweenthespiralgrooveandelbowjoint.
Musclesinvolvementis:-
1. Elbowertensors(Triceps,anconeus)arespared.
2. Wrist,elbowandfingerextensorsareparalysed.
3. Sensorylossinfirstwebspace(ondorsalside)
Ii)TypeII:-Injuryoccursbelowtheelbowjoint.
1. Elbowextensors(triceps,anconeus)andwristextensors(ECRL)are
spared.
2. Fingerextensors(extensordigitorum,extensordigitiminimi,
extensorindicis)andthumbextensors(extensorpollicisclongus&
brevis)areparalysed.
3. Sensorylossinfirstwebspace(ondorsalside).
Iflesionislow
a)Type1
Wristdrop,thumbdropandfingerdrop.
Elbowextensionispreserved.
Sensorylossoverthedorsumoffirstwebspace.

b)Type2
Thumbdropandfingerdrop
Elbowandwristextensionispreserved
Sensorylossoverthedorsumoffirstwebspace

37.Infraspinousfossaofscapulacontains
whichofthefollowingmuscles?
a)Subscapularis
b)Infraspinatus
c)Teresmajor
d)Supraspinatus
CorrectAnswer-B
Ans.B.Infraspinatus
Infraspinatusattachesmediallytotheinfraspinousfossaofthe
scapulaandlaterallytothemiddlefacetofthegreatertubercleofthe
humerus.

38.Structurepassingdeeptoflexor
retinaculumatwrist:
a)Ulnarnerve
b)Mediannerve
c)Radialnerve
d)Ulnarartery
CorrectAnswer-B
Theflexorretinaculumstretchesacrossthefrontofthewristand
convertstheconcaveanteriorsurfaceofthehandintoan
osteofascialtunnel,thecarpaltunnel,forthepassageof:
Themediannerve
Flexortendonsofthethumb(flexorpollicislongus)andfingers
(flexordigitorumsuperficialisandprofundus).
Radialandtheulnarbursa
Itisattachedmediallytothepisiformboneandthehookofthe
hamateandlaterallytothetubercleofthescaphoidandthe
trapeziumbones.
Theattachmenttothetrapeziumconsistsofsuperficialanddeep
partsandformsasynovial-linedtunnelforpassageofthetendonof
theflexorcarpiradialis.
Thelowerborderisattachedtothepalmaraponeurosis.

39.Coracoacromialligamentresistswhich
movements?
a)Upwarddisplacementofhumeralhead
b)Abductionofshoulder
c)Inferiordisplacementofhumerus
d)Externalrotation
CorrectAnswer-A
Ans.A.Upwarddisplacementofhumeralhead
Thecoracoacromialligamentisaflattriangularbandthatplaysa
supportiverolefortheshoulderjoint.
CoracoacromialpreventsUpwarddisplacementofhumeralhead.
Ithastwopart,conoid(medial)andtrapezoid(lateral).Theweightof
theupperlimbistransmittedtothemedialtwo-thirdoftheclavicle
andthencetotheaxialskeletonthroughthecoraco-clavicular
ligment.

40.Coracohumeralligamentinsertson?
a)Greatertuberosity
b)Lesserandgreatertuberosities
c)Anatomicalneckofhumerus
d)Bicepetalgroove
CorrectAnswer-B
Ans.B.Lesserandgreatertuberosities
Coracohumeralligament:
Anextraarticularligamentonthelateralsurfaceofcoracoidand
insertsintothegreaterandlessertuberosities,spanningthebicipital
groove.
Sectioningofcoracohumeralligamentsproducesanteroinferior
instability.
Representsfoldedthickeningofglenohumeralcapsuleinareaof
rotatorintervalbetweensubscapularis&supraspinatus.
W/bodyupright&armindependentposition,coracohumeral&
MGHLplayimportantrolesinresistinginftranslation.

41.Whatistheactionofanconeus?
a)Primaryelbowextensor
b)AssistsExtensionofelbow
c)Wristextension
d)ThumbAbduction
CorrectAnswer-B
Ans.B.AssistsExtensionofelbow
Anconeus=Itsroleinelbowextensionistrivialinhumans.Itassists
inextensionoftheelbow,wherethetricepsbrachiiistheprincipal
agonist,andsupportstheelbowinfullextension.

42.Whichmusclehelpsinclimbingatree?
a)LatissimusDorsi
b)Rhomboideus
c)Trapezius
d)Levatorscapulae
CorrectAnswer-A
Ans.A.LatissimusDorsi
Climbingoftreeishelpedby:

1. LatissimusDorsi
2. Pectoralismajor
LatissimusDorsiisalsoknownas"climber'smuscle"or"Tree
climbingmuscle".

43.Posteriorinterosseusarteryisabranchof
?
a)Commoninterosseusartery
b)Radialartery
c)Medianartery
d)Brachialartery
CorrectAnswer-A
Ans.A.Commoninterosseusartery
Thecommoninterosseousartery,about1cm.inlength,arises
immediatelybelowthetuberosityoftheradiusfromtheulnarartery.

44.Whichmuscleprotectsthebrachial
plexusincaseofclaviclefractures?
a)Subclavius
b)Supraspinatus
c)Subscapularius
d)TeresMinor
CorrectAnswer-A
Ans.A.Subclavius
Thesubclaviusprotectstheunderlyingbrachialplexusand
subclavianvesselsfromabrokenclavicle.

45.Whichofthefollowingistrueaboutdeep
palmararch?
a)Mainlyformedbytheradialartery
b)Ulnararteryhasnocontributiontoit
c)Itgivesoff5perforatingbranches
d)Itdoesnotanastomosewiththesuperficialpalmararch
CorrectAnswer-A
Ans.A.Mainlyformedbytheradialartery
Deeppalmararch
Itliesacrossthebaseofmetacarpalbones.Itisformedmainlyby
radialarteryandcompletedbyadeepbranchoftheulnarartery.
Itsbranchesare:-
Threepalmarmetacarpalarteries
Threeperforatingarteries
Recurrentbranches
Thedeeppalmararchliesdeeptotheobliqueheadofadductor
pollicis,longflexortendon,andlumbricalmusclesandpasses
acrossthebaseofmetacarpalandinterossei.

46.

Whichpartofscapulacanbepalpatedinthe
infraclavicularfossa?

a)Coracoidprocess
b)Spineofscapula
c)Inferiorangle
d)Supraspinousfossa
CorrectAnswer-A
Ans.A.Coracoidprocess
Thecoracoidprocessisathickcurvedprocessattachedbyabroad
basetotheupperpartoftheneckofthescapula;itrunsatfirst
upwardandmedialward;then,becomingsmaller,itchangesits
direction,andprojectsforwardandlateralward.
Itispalpablejustbelowtheclavicle.

47.Flexorcarpiradialisinsertsinto?
a)Baseof5thmetatarsal
b)Baseof2ndand3rdmetacarpal
c)Scaphoidandtrapezium
d)Capitateandhamate
CorrectAnswer-B
Ans.,B.Baseof2ndand3rdmetacarpal
Flexorcarpiradialis
Origin:Medialepicondyleofthehumerus.
Insertion:Baseofsecondandthirdmetacarpals.
Nervesupply:Mediannerve.
Action:Pronatoroftheforearm,weakflexorofelbow.

48.Whatistrueaboutlateraltibialcondyle?
a)Iliotibialtractisattachedtothelateralcondyleoftibia
b)Ligamentumpatellaeinsertsonit
c)Medialcollateralligamentisattachedtoit
d)Semimembranosusisattachedtoit
CorrectAnswer-A
Ans.A.Iliotibialtractisattachedtothelateralcondyleoftibia
*Tibiaisthesecondlongestbone(afterfemur).
*Proximalend(upperend)
-Proximal(upper)endoftibiaincludesmedialandlateralcondyles,
formingtibialplateau.Italsoincludestibialtuberosityand
intercondylararea(areabetweenmedialandlateralcondyle).
-Distalend
*Medialmalleolusgivesattachmenttodeltoidligament(medial
collateralligament)ofankle.

49.WeaknessofextensorHallucislongusis
duetowhichnerverootmainly?
a)L5
b)L4
c)S1
d)S2
CorrectAnswer-A
Ans.A.L5

50.Whichofthefollowingiscommon
betweenthemedialandlateralplantar
arch?

a)FlexorDigitorumBrevis
b)PlantarFascia
c)SpringLigamnet
d)DeltoidLigament
CorrectAnswer-B
Ans,B.PlantarFascia
Plantarfasciaactsasatiebeamforbothmedialandlateralplantar
arches.

51.Whichtendonislodgedinthegrooveon
posteriorsurfaceoflateralmalleolus?
a)Peroneuslongus
b)Tibialisanterior
c)Tibialisposterior
d)FlexorHallucisLongus
CorrectAnswer-A
Ans,A.Peroneuslongus
Peroneuslongusendsinalongtendon,whichrunsbehindthelateral
malleolus,inagroovecommontoit.

52.Allofthefollowingaretrueabouttibialis
anteriorexcept?
a)Itissuppliedbythesuperficialperonealnerve
b)Itdorsiflexesthefoot
c)Itiscloselyrelatedtotheanteriortibialvessels
d)Itinsertsonthemedialcuneiform
CorrectAnswer-A
Ans.A.Itissuppliedbythesuperficialperonealnerve
Tibialisanterior
Itissituatedonthelateralsideofthetibia;itisthickandfleshy
above,tendinousbelow.
Thetibialisanterioroverlapstheanteriortibialvesselsanddeep
peronealnerveintheupperpartoftheleg.

53.Attachmentonposteriorsurfaceof
sacrum?
a)MultifidusLumborum
b)Iliacus
c)Coccygeus
d)Piriformis
CorrectAnswer-A
Ans,A.MultifidusLumborum
Attachementsonsacrum
A)PosteriorSurface
Multifiduslumborum?
Thedeepestmusclearisingfromthesacrum.
SomeofitsfiberscovertheuPpertwosacralforamina.
Thismuscleattachestothetransverseprocessesofthesuperior
vertebraeandisthereforeabletohelpstabilizethespine.
Erectorspinae?
Partlyarisesfromtheposteriorsacrumandthesacrospinous
ligament.
Itisessentialinachievingextensionandlateralbendingofthehead
andvertebralcolumn

54.Structurespassingthroughsacralhiatus
are?
a)S4nerveroot
b)S2nerveroot
c)S3nerveroot
d)S5nerveroot
CorrectAnswer-D
Ans.D.S5nerveroot
Sacralhiatus
Thesacralhiatuscorrespondstotheposteriorcaudalopeningatthe
endofthesacralcanal,whichusuallyoccursatthefifthsacral
vertebra(S5),attheposteriorsurfaceofthesacrum.

55.Longestcutaneousnerveinbody?
a)Lateralcutaneousnerveofthigh
b)Medialcutaneousnerveofthigh
c)Saphenousnerve
d)Suralnerve
CorrectAnswer-C
Ans,C.Saphenousnerve
Thesaphenousbranchofthefemoralnerve(saphenousnerve)is
thelongestcutaneousnerve.Itrunswiththegreatsaphenousvein
infrontofmedialmalleolusandsuppliestheskinofanteromedial
aspectofthelegandmedialborderofthefoot.Thesaphenous
nervemaybedamagedinfrontofthemedialmalleolusduring
venesectionofthelongsaphenousvein.
Therefore,femoralnervedamagecancausesensorylossoverthe
areaofthegreatsaphenousveinintheleg.

56.Obliquepoplitealligamentisderivedfrom
?
a)Semitendinosus
b)Bicepsfemoris
c)Adductormagnus
d)Semimembranosus
CorrectAnswer-D
Ans,D.Semimembranosus
Obliquepoplitealligament
Itisanexpansionfromthetendonofsemimembranosusattachment
tointercondylarlineoffemur.
Itiscloselyrelatedtopoplitealarteryandispiercedbymiddle
genicularvesselsandnerveandtheterminalpartoftheposterior
divisionoftheobturatornerve.

57.Lateralborderofthefootreceivesits
sensorysupplyfrom?
a)Saphenousnerve
b)Suralnerve
c)Deepperonealnerve
d)Sciaticnerve
CorrectAnswer-B
Ans.B.Suralnerve

58.Allofthefollowingisincludedinchest
wallexcept?
a)Ribs
b)ThoracicVertebrae
c)Sternum
d)Lumbarvertebrae
CorrectAnswer-D
Ans,D.Lumbarvertebrae

59.Respiratorybronchiolesareformedfrom
?
a)Principalbronchus
b)Terminalbronchioles
c)TertiaryBronchus
d)Lobarbronchioles
CorrectAnswer-B
Ans,B.Terminalbronchioles
Terminalbronchioleemnateintorespiratorybronchioles.
Repiratorybronchiolesproceedintothealveolarducts,which
immediatelybranchintoalveolarsacs(alveoli).

60.Segmentofbronchidistaltoprimary
bifurcation?
a)Primarybronchi
b)Terminalbronchiole
c)Respiratorybronchiole
d)Sencondarybronchi
CorrectAnswer-A
Ans.A.Primarybronchi

61.Thecricopharyngealsphincterishowfar
fromthecentralincisor?
a)15cm
b)25cm
c)40cm
d)50cm
CorrectAnswer-A
Ans,A.15cm
AtthelevelofT1Overtebra,itspassagethroughesophagealhiatus
ofdiaphramloweresophagealsphincter-37.5-40cm(f5-f6inches)
fromincisor

62.Whichofthefollowingstructuresis
relatedtotheesophagus22.5cmfromthe
incisorteeth?

a)Archofaorta
b)Rightprincipalbroncus
c)ThoracicDuct
d)AzygousVein
CorrectAnswer-A
Ans,A.Archofaorta
2dconstrictionisatT4levelwherearchofaortacrossesesophagus.

63.Thoracicductopensintosystemic
circulationat?
a)junctionofSVCandleftbrachiocephalicvein
b)Junctionofleftinternaljugularandleftsubclavianvein
c)Directlyintocoronarysinus
d)Intoazygousvein
CorrectAnswer-B
Ans.B.Junctionofleftinternaljugularandleftsubclavianvein
Thoracicductbeginsasacontinuationoftheupperendofthe
cisternachylinearthelowerborderofT12vertebraandentersthe
thoraxthroughtheaorticopeningofdiaphragrn(atT12).Itthen
ascendsthroughtheposteriormediastinumandatT5levelcrosses
fromrightsidetotheleftsideandascendsalongleftmarginof
oesophagustoentertheneck.AtthelevelofC7vertebrae,arches
towardsleftsidetoopenintoleftbrachiocephalicveinattheangleof
unionofleftsubclavianandleftinternaljugularveins.

64.Sympatheticsupplyoftheheartisfrom?
a)Vagus
b)Thoracicsympatheticfibres[T1toT5]
c)Lumbarsympatheticfibres
d)Cervicalganglion
CorrectAnswer-B
Ans,B.Thoracicsympatheticfibres[T1toT5]

65.Whichofthefollowingarecuspsofthe
aorticvalves?
a)Left,rightandAnterior
b)Anterior,RightandPosterior
c)Posterior,LeftandRight
d)Anterior,PosteriorandLeft
CorrectAnswer-C
Ans,C.Posterior,LeftandRight
Theaorticvalveisasemilunarvalvewiththreecuspswhichinclude
left,rightandposterior.

66.Whichisthewidow'sarteryinmyocardial
infarction?
a)Leftanteriordescendingartery
b)Rightcoronaryartery
c)Posteriorinterventricularartery
d)Leftcircumflexartery
CorrectAnswer-A
Ans,A.Leftanteriordescendingartery
Theanteriorinterventricularbranchoftheleftcoronaryartery,(also
leftanteriordescendingartery(LAD),oranteriordescendingbranch)
isabranchoftheleftcoronaryartery.
Occlusionofthisarteryisoftencalledthewidow-makerinfarction
andhencethisarteryiscalledawidow'sartery.

67.Whichofthefollowingpassesposteriorto
thehilumofthelung?
a)Vagus
b)Phrenicnerve
c)SVC
d)Rightatrium
CorrectAnswer-A
Ans,A.Vagus

68.Whatisthelevelofthepulmonaryvalve?
a)3rdintercostalspace
b)4thcostalcartilage
c)3rdcostalcartilage
d)2ndintercostalspace
CorrectAnswer-C
Ans,C.3rdcostalcartilage

69.Lowerlimitoftheinferiorborderofthe
lunginthemidaxillarylineis?
a)6thrib
b)8thrib
c)10thrib
d)12thrib
CorrectAnswer-B
Ans,B.8thrib
Thelowerlimitoftheinferiorborderofthelungis2ribsabovethe
reflectionofthepleural.
Inthemidaxillarylinethepleurareflectsatthe10thribandhencethe
lowerlimitofthelungis8thrib.

70.Whichisatypicalintercostalnerve?
a)First
b)Second
c)Third
d)Seventh
CorrectAnswer-C
Third
"Typicalintercostalnervesaretheonesthatareconfinedtotheir
ownintercostalspacesinthethoracicwall.Thethird,fourth,fifthand
sixthintercostalnervesarethetypicalnerves"

71.Greatcardiacveinliesin?
a)Tricuspidvalve
b)Anteriorinterventricularsulcus
c)Posteriorinterventricularsulcus
d)None
CorrectAnswer-B
Ans.is'B'i.e.,Anteriorinterventricularsulcus[RefBDC4th/e
Vol.I,p.251-252;KeithMooreClinicalAnatomyhlth/ep.136-137;
Snell'sClinicalAnatomy9th/ep.121]
(Atrioventricular)sulcusGreatcardiacvein,coronarysinus,
Smallcardiacvein,RCA,LCX.
AnteriorinterventricularsulcusGreatcardiacvein,leftanterior
descending(interventricular)artery.
PosteriorinterventricularsulcusMiddlecardiacvein,Posterior
interventricularbranchofRCA.

72.Apexatofthelungliesatwhatlevel?
a)Abovetheclavicle
b)Belowtheclavicle
c)Attheleveloftheclavicle
d)None
CorrectAnswer-A
Ans.is'a'i.e.,Abovetheclavicle[RefBDC4thiep.222-228]
Apexliesintheinletofthorax,2-5cmabovetheclavicle.Itis
relatedanteriorlytosubclavianarteryandvein.Posteriorlyitis
separatedfromneckoffirstribby(frommedialtolateral)
sympathetictrunk,fistposteriorintercostalvein,superiorintercostal
artery,andascendingbranchofventralramusof1"thoracicnerve.

73.Diaphragmissuppliedby?
a)Phrenicnerve
b)C2,C3,C4Roots
c)Thoracodorsalnerve
d)Longthoracicnerve
CorrectAnswer-A
Ans.is'a'i.e.,Phrenicnerve[RefBDC6th/eVolIp.192,fig.12.12]
Nervesupply
Motor:-Phrenicnerve
(C3C4C5).
Sensory:-i)centrallybyphrenicnerve.
Peripherallybylower6intercostalnerves.

74.Midpointbetweensuprasternalnotchand
pubicsymphysespassesthroughwhich
plane?

a)Transpyloricplane
b)Transtubercularplane
c)Trnasxiphoidplane
d)None
CorrectAnswer-A
Ans.is'a'i.e.,Transpyloricplane[RefBDCVol-26th/ep.229]
Anteriorabdominalwallisdividedinto9regionswiththehelpoftwo
verticalandtwohorizontalplanes.
Thehorizontalplanesinclude:-
1. Transpyloricplane(ofAdison):-Itliesmidwaybetweenthe
suprasternalnotchandpubicsymphysis.It
passesanteriorlythroughtipsof9'hcostal
cartilageandposteriorlythroughlowerborderofL,
vertebra.
Organspresentatthislevelarehilumofkidney,pylorus
ofstomach,beginningofduodenum,neckofpancreas,fundusof
gallbladder
andoriginofSMA.
2. Transtubercularplane:-Itconnectsthetuberclesofiliaccrests
andpassthroughupperborderofL5vertebra.
Thetwoverticalplanesarerightandleftlateralplanespassing
throughmidinguinalpoint(alsocalledasmidinguinalplaneor
midclavicularplane).
Thenineregionsfromabovedownwadsare-

1. Inmiddle:-Epigastrium,umblical,hypogastrium.
2. Onrightside:-Righthypochondrium,rightlumbar,andrightinguinal

(iliac)regions.
3. Leftside:-Lefthypochordrium,leftlumbarandleftinguinal(iliac)
regions.

75.Musclelyingbetweenanteriorandmiddle
layerofthoracolumbarfasciais?
a)Psoasmajor
b)QuadratusLumborum
c)Obduratorinternus
d)Externaloblique
CorrectAnswer-B
Ans.is'b'i.e.,QuadratusLumborum[RefBDC6thleVol.2p.
343;Snell9"/ep.695]
Quadratuslomborumisenclosedbetweenanteriorandmiddle
layers.Erectorspinae(paraspinalmuscle)isenclosed
between
middleandposteriorlayer.

76.AnteriorRectusSheathjustabovepubic
symphysisisformedby?
a)ExternalObliqueAponeurosis
b)TheaponeurosisofthreemusclesincludingExternalOblique,
InternalOblique,andTransversusAbdominis
c)LineaAlba
d)InternalObliqueonly
CorrectAnswer-B
Ans.is'B'i.e.,AponeurosisofthreemusclesincludingExternal
Oblique,InternalOblique,andTransversusAbdominis

Theanteriorwalljustabovethesymphysispubis(areabelowthe
arcuateline)isformedbyaponeurosisofallthreemuscles
(externaloblique,internaloblique,transversusabdominis).
Threeaponeuroticlayersformingrectussheathofbothsides
interlacewitheachothertoformatendinousraphe,Lineaalba.It
extendsfromthexiphoidprocesstopubicsymphysis.
Lineaalbaisnarrowandindistinctbelowtheumbilicus,astworecti
lieinclosecontact.Lineaalbabroadensoutabovethelevelofthe
umbilicus.

77.Rightsuprarenalveindrainsinto?
a)Inferiorvenacava
b)Rightrenalvein
c)Leftrenalvein
d)AccessoryHemiazygousvein
CorrectAnswer-A
Ans.is'a'i.e.,Inferiorvenacava
Arterialsupplyofadrenalglandisbythreearteries:-
1.Superiorsuprarenalartery(branchoftheinferiorphrenicartery);
2.Middlesuprarenalartery(branchofabdominalaorta);and
3.inferiorsuprarenalartery(branchoftherenalartery).
Venousdrainageisthroughthesuprarenalveins.
Rightsuprarenal(adrenal)veindrainsintoIVCandleftsuprarenal
veindrainsintotheleftrenalveinandthenintoIVC.
Lymphaticsfromsuprarenalglandsdrainintolateralaortic(para-
aortic)nodes.

78.Whichofthefollowingisnotderivedfrom
theexternalobliqueaponeurosis?
a)InguinalLigament
b)Lacunarligament
c)LineSemilunaris
d)PectinealLigament
CorrectAnswer-C
Ans.is'c'i.e.,LineSemilunaris[RefBDC6th/eVol2p.343;
Snell91Vep.695]
ExternalobliqueInguinal(Pouparts)ligament
LacunarligamentMnemonic:IPL
Pectineal(cooper's)ligament
Superficialinguinalring
Externalspermaticfascia
InternalobliqueCremastericfascia&muscle
Alongwithtendonoftransversusabdominisformsconjointtendon

79.Stomachissuppliedby?
a)Coeliactrunk
b)Splenicartery
c)Gastroduodenalartery
d)Alloftheabove
CorrectAnswer-D
Ans.is'd'i.e.,Alloftheabove

80.Whatisthenumberoflayersingreateromentum?
a)1
b)2
c)3
d)4
CorrectAnswer-D
Thegreateromentumisfoldedbackonitselfandisthereforemadeupoffourlayersof
closelyappliedvisceralperitoneum,whichareseparatedbyvariableamountsofadipose
tissue.

81.Stomachwallismainlydrainedbyall
lymphnodesexcept?
a)Pyloricnodes
b)Shortgastricvesselnodalgroup
c)Rightgastroepiploicnodes
d)Inguinalnodes
CorrectAnswer-D
Ans.is'd'i.e.,Inguinalnodes[RefGray'sanatomy20thedition]
Thestomachisdrainedbyfourgroupsoflymphnodes:

1. Leftgastricarterialnodalgroup,whichfollowstheleftgastricartery
anddrainintotheceliacnodes.Theydrainthelessercurvatureof
thestomachtotheleft.
2. Shortgastricandleftgastroepiploicvesselsnodalgroup.The
lymphaticvesselswhichdraintheleftsideofthegreatercurvatureof
thestomachfollowsthesevesselsanddrainintothe
pancreaticosplenicgroupofnodes.
3. Rightgastroepiploicnodes,whichdraintherighthalfofthegreater
curvatureofstomachasfarasthepylorus
4. Pyloricnodeswhichdrainsthepyloricpartofstomachtothehepatic
,pyloricandleftgastricnodes.
Allthevesselsenterintotheceliacnode.Fromthesenodes
theypassintotheintestinallymphtrunks,whichthenenterthe
cisternachyliortheabdominalconfluenceoflymphtrunks.The
cisternachylidrainsintothethoracicduct.


82.Alllymphofstomachdrainsinto?
a)Pyloricnodes
b)Shortgastricvesselnodalgroup
c)Rightgastroepiploicnodes
d)Coeliacnodes
CorrectAnswer-D
Ans.'D'i.e.,Coeliacnodes
Allthevesselsenterintotheceliacnode.
Fromthesenodes,theypassintotheintestinallymphtrunks,which
thenenterthecisternachyliortheabdominalconfluenceoflymph
trunks.
Thecisternachylidrainsintothethoracicduct.

83.Gallbladderisrealtedtowhichsegment
oftheliver?
a)I
b)II
c)III
d)IV
CorrectAnswer-D
Ans.is'd'i.e.,IV[RefGray's40thlep.1163-1167;Sabiston
18th/ep.1584]
Thegallbladderliesontheinferiorsurfaceoftheliverclosely
relatedtosegmentIVorthequadratelobe.
Anatomicallyliverisdividedintoalargerightlobeandasmallleft
lobebylineofattachmentoffalciformligament(anterosuperiorly),
fissureforligamentumteres(inferiorly),andfissureforligamentum
venosum(posteriorly).
Rightlobeismuchlargerandformsfivesixthofliverandleftlobe
formsonlyonesixth.Caudatelobeandquadratelobearepartsof
anatomicalrightlobe.
Thephysiologicalleftlobeiscomposedof4segmentsdesignated
ItoIVandissuppliedbyleftbranchofhepaticartery,leftbranchof
portalveinanddrainedbylefthepaticduct.
ThephysiologicalrightlobeconsistsofsegmentV,VI,VIIandVIII
andissuppliedbyrighthepaticartery,rightbranchofportalveinand
drainedbyrighthepaticduct.

84.Whichsegmentofliverdrainsonboth
sides?
a)I
b)II
c)III
d)IV
CorrectAnswer-A
Ans.is'A'i.e.,I
Caudatelobe(segmentI)
Itissituatedontheposteriorsurfaceoftherightlobe.
ItisboundedonrightbyagrooveforIVC,onleftbyfissurefor
ligamentumvenosum,andinferiorlybyportahepatis(containing
hepaticartery,portalvein,hepaticductbileduct,nerveplexus,and
lymphatics).
Justbehindtheportahepatis,thecaudatelobeisconnectedtothe
restoftherightlobebythecaudateprocess.
Thereisasmallroundedelevationtotheleft,calledthepapillary
process.
Caudatelobeliesinthesuperiorrecessoflessersacandisrelated
tothecruraofthediaphragm,rightinferiorphrenicartery,and
coeliactrunk.
Caudatelobe(anatomicalpartoftherightlobe)belongs
physiologicallytobothrightandleftlobesbecauseitreceivesblood
fromtherightandlefthepaticarteries;rightandleftbranchesofthe
portalvein,anddrainsbileintoboththerightandlefthepaticduct.
Thusitisconsideredasthephysiologicallyindependentlobe.

85.Caudatelobeoftheliver-Trueis?
a)Itreceivesbloodsupplyfrombothrightandlefthepaticarteries
b)ItisSegmentIIoftheliver
c)Itissituatedontheanteriorsurfaceofliver
d)Itliesbetweentheaortaandligamentumvenosum
CorrectAnswer-A
Ans.is'a'i.e.,Itreceivesbloodsupplyfrombothrightandleft
hepaticarteries[RefRameshBabup.249]
Caudatetube(segmentI)issituatedonposteriorsurfaceofliver
betweenIVC&ligamentumvenosum.Itreceivesbloodsupplyfrom
right&leftarteries.

86.Superiorborderofepiploicforamen
formedby-
a)Caudatelobe
b)Hepaticartery
c)Bileduct
d)IVC
CorrectAnswer-A
Ans.is'a'i.e.,Caudatelobe
Epiploicforamen(foramenofWinsloworaditustolessersac)isa
slit-likeopeningthroughwhichlessersaccommunicateswithgreater
sac.ItissituatedatthelevelofT12vertebra.Itsboundariesare:-
Anterior:-Rightfreemarginoflesseromentum(containsportalvein,
hepaticarteryproperandbileduct).
Posterior:-IVC,rightsuprarenalglandandT12vertebra.
Superior:-Caudatelobeoftheliver.
Inferior:-1stpartoftheduodenumandhorizontalpartofthehepatic
artery.

87.InternalanalSphincterisformedby?
a)Puborectalis
b)Circularmusclesfromlowerrectum
c)LongitudinalInvoluntarymuscles
d)None
CorrectAnswer-B
Ans.is'b'i.e.,Circularmusclesfromlowerrectum[RefBDC
4th/eVol.Hp.383;Gray'sAnatomy40thieChapter67]
Externalsphincteriscontributedbyfibersfrompuborectalispartof
levatoranimuscle(inuppermostpart);superficialtransverse
perinealmusclesanteriorlyandanococcygealrapheposteriorly(in
upperthird)andanococcygealligament(inmiddlethird).

88.Allofthefollowingaretrueabout
duodenumexcept?
a)Fourthpartistheshortestpart
b)AmpullaofVateropensthroughthesecondpart
c)Minorduodenalpapillaisinthethirdpart
d)Firstpartappearslikeaduodenalcaponbariumstudies
CorrectAnswer-C
Ans.is'c'i.e.,Minorduodenalpapillaisinthethirdpart[Ref
BDC6'1*Vol.2p.259-262]
Thirdpart(Horizonalpart):
Itis10cm(4inches)long.Itbeginsatinferiorduodenalflexureand
passestowardstheleftinfrontofIVCbehindsuperiormesenteric
vesselsandrootofmesenterytomeet4thpartofduodenum.

89.Whichofthefollowingisabranchofthe
inferiormesentericartery?
a)Sigmoidartery
b)Middlecolicartery
c)Renalartery
d)RightColicartery
CorrectAnswer-A
Ans.is'a'i.e.,Sigmoidartery[RefBDC6th/eyoi.2p.276]
Inferiormesentricarterygivesfollowingbranches?

1. Leftcolicartery
2. Sigmoidarteries
3. Superiorrectalartery

90.Waldeyer'sfasciaconnects?
a)Rectumtosacrum
b)Rectumtouterus
c)Rectumtolateralwallofpelvis
d)Rectumtobladder
CorrectAnswer-A
Ans.is'a'i.e.,Rectumtosacrum[RefClinicalanotomy2"dlep.
786]
Supportofrectuminclude

1. Fasciaofwaldeyer:Itattachesthelowerpartofrectalampullato
thesacrum.Itisformedbycondensationofpelvicfasciabehindthe
rectumandenclosesthesuperiorrectalvesselsandlymphatics.
2. Lateralligamentsoftherectum:Itisformedbycondensationof
pelvicfasciaandenclosesmiddlerectalvessels,andbranchesof
pelvicplexuses.
3. Rectovesicalfasciaofdenonvilliers:Itextendsfromrectum(behind)
totheprostateandseminalvesicleinfront.
4. Pelvicperitoneumandrelatedvascularpedicles.
Perinea(bodywithitsmuscles.

91.ContentofAlcock'scanalis?
a)Internalpudendalartery
b)Internaliliacartery
c)Inferiorrectalvein
d)Inferiormesentericvein
CorrectAnswer-A
Ans.is'a'i.e.,Internalpudendalartery[RefBDC6th/eVol-2
p.362]
Pudendalcanal(Alcock'scanal)isafascialcanalinthelateralwall
ofischeorectal(ischeo-anal)fossa,enclosingpudendalnerveand
internalpudendalvessels(arteryandvein).Itisaspacebetween
obturatorfasciaandlunatefascia.Otherbelievethatitisformedby
splittingoftheobturatorfascia.

92.Allofthefollowingorgansareindirect
contactwiththespleenexcept?
a)Duodenum
b)Stomach
c)Leftkidney
d)Colon
CorrectAnswer-A
Ans.is'a'i.e.,Duodenum
Grossmorphologyofthespleen
Spleenhastwoends(anteriororlateralandposteriorormedial),
threeborders(superior,inferiorandintermediate),twosurfaces
(visceralanddiaphragmatic),twoangles(anterobasalangleand
posterobasalangle)andhilum.
Theanteriorendissupportedbythephrenicocolicligament.
Thesuperiorborderischaracteristicallynotchednearitsanterior
end.
Thevisceralsurfaceisrelatedtothefundusofstomach(atgastric
impression),leftkidney(atrenalimpression),splenicflexureofthe
colon(atcolicimpression)andtailofthepancreas(atpancreatic
impression).Itslowerendisrelatedtothephrenicocolic
ligament.Thediaphragmatic
surfaceisrelatedtothediaphragm.

93.Glanspenisisacontinuationof-
a)Corpusspongiosum
b)Ischiocavernosus
c)CorporaCavernosa
d)Puborectalis
CorrectAnswer-A
Ans.is'A'i.e.,Corpusspongiosum
Thepenisisthemaleorganofcopulation.Thepenishasarootand
abody.
Therootofthepenisissituatedinthesuperficialperinea!pouch,
attachedtotheinferiorsurfaceoftheperinealmembrane.Itconsists
ofthreemassesoferectiletissue:thebulbofthepenisandtwo
crura.Eachcruscontinuesforwardtobecomethecorpus
cavernusum(inthebody)andthebulbistheposteriorendofthe
corpusspongiosum(ofthebody).
Thebodyofthepenisisthefreeportionofthepenis.Itiscomposed
ofthreeelongatedmassesoferectiletissues:-rightandleftcorpora
cavernosa,andmediancorpusspongiosum.Corporacavernosae
areenvelopedbytunicaalbugineaandcorpusspongiosumisalso
surroundedbytunicaalbuginea.Thepenileurethrarunsthroughthe
wholelengthofthecorpusspongiosumfromthebulbatthebackto
theterminalexpandedpartofthecorpusspongiosum,calledthe
glanspenis.

94.Whichmusclecausesopeningofthe
upperendofesophagus?
a)Epiglottis
b)Thyropharungeus
c)Stylopharyngeus
d)Cricopharyngeusofinferiorconstrictor
CorrectAnswer-D
Ans.is'd'i.e.,Cricopharyngeusofinferiorconstrictor[RefHall,
ArthurC.Guyton,JohnE.(2005).Textbookofmedical
physiology(11thed.).Philadelphia:W.B.Saunders.p.782-784.]
Upperesophagealsphinctor
Theupperesophagealsphinctersurroundstheupperpartofthe
esophagus.
Itconsistsofskeletalmuscle,butisnotundervoluntarycontrol.
Openingoftheupperesophagealsphincteristriggeredbythe
swallowingreflex.
Theprimarymuscleoftheupperesophagealsphincteristhe
cricopharyngealpartoftheinferiorpharyngealconstrictor.

95.Posteriorperforationofstomach,
collectionofcontentsoccursinwhich
pouch?

a)Greatersac
b)Leftsubhepaticandhepatorenalspaces[pouchofMorrison]
c)Omentalbursa
d)Rightsubphrenicspace
CorrectAnswer-C
Ans.is'B'i.e.,Leftsubhepaticandhepatorenalspaces[pouch
ofMorrison]
Aposteriorgastriculcermayperforateintothelessersac(omental
bursa).
Theleakingfluidpassesoutthroughepiploicforamento
reachthehepatorenalpouch.Sometimesinthesecasestheepiploic
foramenisclosedbyadhesions.Thenthelessersacbecomes
distended,andcanbedrainedbyatubepassedthroughthelesser
omentum.

96.Cremastricmuscleisformedfrom?
a)Fasciafrominternaloblique
b)Fasciafromexternaloblique
c)Fasciafromrectusabdominis
d)Fasciafromtransversusabdominis
CorrectAnswer-A
Ans.is'a'i.e.,Fasciafrominternaloblique
Thelayersofscrotumfromoutsidetoinsideare:-

1. Skin
2. Dartosmuscle(smoothmusclelayer)continuouswithCollesfascia
ofperineumposteriorlyandScarpa'sfasciaandCamper'sfascia
anteriorly.
3. Theexternalspermaticfascia,extensionfromexternaloblique.
4. Thecremastericmuscle,continuouswithfasciafrominternal
oblique.
5. Theinternalspermaticfascia,continuouswithfasciafromfascia
transversalis.

97.Kidneyiscoveredbywhatfascia?
a)Sibson'sfascia
b)Buck'sFascia
c)Gerota'sFascia
d)None
CorrectAnswer-C
Ans.is'c'i.e.,Gerota'sFascia[RefFarlexPartnerMedical
DictionaryFarlex2012]
Renalfascia
Therearefourcoveringsaroundthekidney(fromwithinoutwords):-
Truecapsule(fibrouscapsule)
Itisformedbythecondensationoffibrousstromaofkidney.
Falsecapsule(renalfasciaorfasciaofGerota)
Itisformedbycondensationofextra-peritonealconnectivetissue
aroundkidneyandiscontinuouslaterallywithfasciatransversalis.
Falsecapsuleconsistsoftwolayers:anterior"fasciaofToldt"and
posterior"fasciaofZuckerkendl".

98.

Narrowestpartofureteris?
a)Brimofthepelvis
b)Crossingbygonadalvessels
c)Vesicouretricjunction
d)Crossingbyductusdeferens
CorrectAnswer-C
Ans.is'c'i.e.,Vesicouretricjunction[RefCampbellisurology
6thlep.2123;Gray'sAnatomyforstudents1"/ep.325]
Uretermeasuresabout3mmindiameter,butisconstrictedat
fiveplaces

1. Pelviuretericjunction
2. Brimoflesserpelvis(atthelevelofbifurcationofcommoniliac
arteryandcrossingofexternaliliacartery
3. Pointofcrossingofureterbyductusdeferensorbroadligamentl
4. Entryinbladderwall(thisvesicoureteraljunctionisthenarrowest
partofureter)
5. Openinginlateralangleoftrigone

99.Whatisthetotallengthofthecolon?
a)1metre
b)1.5metres
c)2metres
d)4metres
CorrectAnswer-B
Ans.is'b'i.e.,1.5metres[RefBDCVol.IIelep.269-273]
Thelargeintestineextendsfromtheileocaecaljunctiontoanus.
Itis1.5meterslongandisdividedintocaecum,ascendingcolon,
rightcolic(hepatic)flexure,transversecolon,leftcolic
(splenic)flexure,descendingcolon,sigmoidcolon,
therectumand
analcanal.
Transversecolonislongestpart(50cm)andanalcanal
shortest(3.8cm).
Caecum6cmSigmoidcolon37.5cm
Ascendingcolon->12.5cmRectum12cm
Transversecolon50cmAnalcanal3.8cmDescendingcolon
25cm

100.Submandibularlymphnodesdrainthe
followingareasofthefaceexcept?
a)Medialhalfofeyelids
b)Centralpartoflowerlip
c)Medialpartofcheek
d)Centralpartofforehead
CorrectAnswer-B
Ans.is'b'i.e.,Centralpartoflowerlip[RefBDCVol.III6th/ep.
73]
Lymphaticdrainageofface
Thefacepossessesthreeareasfromwhichlymphaticdrainage
isasfollows:?

1. Upperarea,comparisinggreaterpartofforehead,lateralViof
eyelids,conjunctiva,lateralpartofcheekandparotidarea,drains
intopreauricular(superficial)parotidnodes.
2. Middlearea,comprisingcentralpartofforehead,externalnose,
upperlip,lateralpartoflowerlip,medialhalvesofeyelids,medial
partofcheek,andgreaterpartoflowerjaw,drainsinto
submandibularnodes.
3. Lowerarea,includingcentralpartoflowerlipandthechin,drains
intosubmentalnodes.

101.Larynxbelowthevocalcordsdraininto
?
a)Pretracheallymphnodes
b)Occipitallymphnodes
c)Mediastinalnodes
d)Lymphaticsalongthesuperiorlaryngealvein
CorrectAnswer-A
Ans.is'a'i.e.,Pretracheallymphnodes
Supraglotticpart(Abovevocalcord)
Lymphaticsalongthesuperiorlaryngealveinandnodesadjacentto
thethyrohyoidmembrane
Infraglotticpart(Belowvocalcord)
Pretrachealandprelaryngealnodes
Vocalcords
Devoidoflymphaticsupply

102.Nervesupplytotheangleofthe
mandibleisby?
a)PosteriorprimaryramiofC2,C3
b)Greaterauricularnerve
c)Maxillarynerve
d)Mandibularnerve
CorrectAnswer-B
Ans.is'b'i.e.,Greaterauricularnerve
Theskinovertheangleofthejaw(mandible)issuppliedbythe
anteriordivisionofthegreaterauricularnerve.

103.Nervesupplytothetipofthenoseis
from?
a)Theophthalmicdivisionofthetrigeminalnerve
b)Greaterauricularnerve
c)Themaxillarydivisionofthetrigeminalnerve
d)Mandibularnerve
CorrectAnswer-A
Ans.is'a'i.e.,Ophthalmicdivisionofthetrigeminalnerve
Tipofthenoseandlowerpartofthedorsumofthenoseare
suppliedbytheexternalnasalbranchoftheophthalmicdivisionof
thetrigeminalnerve.

104.Dangerousspaceintheneckisfound
between?
a)Buccopharyngealfasciaandalarfascia
b)Prevertebralfasciaandalarfascia
c)BuccopharyngealfasciaandPrevertebralfascia
d)None
CorrectAnswer-B
Ans.is'b'i.e.,Prevertebralfasciaandalarfascia[Ref"Severe
softtissueinfectionsoftheheadandneck:aprimerforcritical
carephysicians".Lung.187(5):271-9.]
Thedangerspaceoralarspace,isaregionoftheneck.The
commonnameoriginatesfromtheriskthataninfectioninthisspace
canspreaddirectlytothethorax,and,duetobeingaspace
continuousontheleftandright,canfurthermoreallowinfectionto
spreadeasilytoeitherside.
Itisboundedsuperiorlybytheskullbase,anteriorlybythealar
fasciaandposteriorlybytheprevertebralfascia.Itcomestoanend
atthelevelofthediaphragm.
Theretropharyngealspaceisfoundanteriortothedangerzone,
betweenthealarfasciaandbuccopharyngealfascia

105.Whichmuscleisattachedtothediscof
thetemporomandibularjoint?
a)Buccinator
b)Lateralpterygoid
c)Masseter
d)Temporalis
CorrectAnswer-B
Ans.is'Bi.e.,Lateralpterygoid[RefBDC415/eVol.3p.145;
Last'sanatomy11'"/e

Lateralpterygoid
originUpperhead:Infratemporalsurface&crestofgreaterwing
ofsphenoid.Lowerhead:Lateralsurfaceoflateralpterygoidplate.
NervesupplyPterygoidfoveaonAnteriordivisionoftheneckof
manmandibularnerve.anaudible,Articulardiscandcapsule
oftemporomandibularjoint.
ActionDepressesthemandible.Protrusionandsidetoside
movement.

106.Maxillarytuberclegivesattachmentto?
a)Lateralpterygoid
b)Medialpterygoid
c)Temporalis
d)Masseter
CorrectAnswer-B
Ans.is'b'i.e.,Medialpterygoid[RefBDCVolIII6thlep.116]
MedialPterygoid
Origin:Superficialhead:Tuberosityofmaxilla.Deephead:Medial
surfaceoflateralpterygoidplate&pyramidalprocessofpalatine
bone.
Insertion:Medialsurfaceofanglemandible&adjoiningramus.
Nervesupply:Branchfromtrunkofmandibularnerve
Action:Elevatesthemandible,Protection&sidetosidemovement.

107.Vidiannerveisalsoknownas?
a)NerveofPterygoidcanal
b)GreaterPetrosalnerve
c)LesserPetrosalnerve
d)GreaterAuricularnerve
CorrectAnswer-A
Ans.is'a'i.e.,NerveofPterygoidcanal
Thenerveofthepterygoidcanal(Vidiannerve)isformedbythe
junctionofthegreaterpetrosalnerveandthedeeppetrosalnerve
withinthepterygoidcanalcontainingthecartilaginoussubstance,
whichfillstheforamenlacerum.
Itpassesforwardthroughthepterygoidcanalwithitscorresponding
artery(arteryofthepterygoidcanal)andisjoinedbyasmall
ascendingsphenoidalbranchfromtheoticganglion.
Itthenentersthepterygopalatinefossaandjoinstheposteriorangle
ofthepterygopalatineganglion.

108.Whichofthefollowingnucleibelongtothegeneralvisceralafferentcolumn?
a)Facialnervenucleus
b)Trigeminalnucleus
c)Dorsalnucleusofvagus
d)Nucleusambiguus
CorrectAnswer-C
Visceralafferentfibers,alsocalledgeneralvisceralafferent
fibers,
conveysensationfromthealimentarytract,heart,vessels,
andlungsbywayofnervesIXandX.Aspecializedvisceralafferent
componentisinvolvedwiththesenseoftaste;fiberscarrying
gustatoryimpulsesarepresentincranialnervesVII,IX,andX.The
generalvisceralafferentcolumnisrepresentedbypartofthedorsal
nucleusofthevagusnerve.

Ref:WaxmanS.G.(2010).Chapter8.CranialNervesand
Pathways.InS.G.Waxman(Ed),ClinicalNeuroanatomy,26e.

109.RightRecurrentlaryngealnerveloops
around?
a)Rightsubclavianartery
b)Rightaxillaryartery
c)RightExternalcarotidartery
d)RightSuperiorthyroidartery
CorrectAnswer-A
Ans.is'a'i.e.,Rightsubclavianartery[RefLarsen,WilliamJ.
(1993).Humanembryology]
Recurrentlaryngealnerve
Onrightsideitarisesintherootofneckandwindsaroundfirstpart
ofrightsubclavianartery.Itmaybeanterior(superficial)orposterior
(deep)toinferiorthyroidartery.
Onleftsideitarisesinthorax(superiormediastinum)andwind's
aroundthearchofaortaimmediatelybehindtheattachmentof
ligamentumarteriosum.Itisusuallyposterior(deep)toinferior
thyroidarteryorbetweenitsbranches.
Recurrentlaryngealnervesuppliesallintrinsicmusclesoflarynx
(exceptcricothyroid)andmucousmembraneoflarynxbelowvocal
fold.Italsogivesbranchestodeepcardiacplexus,trachea,
esophagusandinferiorconstrictior.
Inferiorthyroidarteryisligatedawayfromglandtoavoidinjuryto
nerve.Leftnerveismoreliabletodamage.

110.Leftrecurrentlaryngealpassesbetween
?
a)Trachea&larynx
b)Trachea&esophagus
c)Esophagusandbronchi
d)Esophagusandaorta
CorrectAnswer-B
Ans.is'b'i.e.,Trachea&esophagus[RefGray's38th/ep.786]
Thepathsoftheleftandrightrecurrentlaryngealnervesveryslightly
withtheleftrecurrentlaryngealnervedividingfromthemainvegus
nerveattheleveloftheaorticarch.
Theleftrecurrentlaryngealnervethandipsposteriorlyaroundthe
aroticarchtoassendthroughthesuperiormediastinumtoenterthe
groovebetweentheesophagusandtrachea.
Therightrecurrentlaryngealnervedividesfromthemainvagus
nerveattheleveloftherightsubclavianarterytoenterthesuperior
mediastinum.
Therightrecurrentlaryngealnervethendipsposteriorlyaroundthe
subclavianarterytoassendinthegrovebetweentheesophagus
andtrachea.

111.Structurespiercedbytheparotidduct
areallexcept?
a)Buccopharyngealfascia
b)Buccinatormuscle
c)Buccalfatpad
d)Investinglayerofdeepcervicalfascia
CorrectAnswer-D
Ans.'d'i.e.,Investinglayerofdeepcervicalfascia
Theparotidduct(Stenson'sduct)
Parotidductemergesfromtheanteriorborderoftheglandand
passesforwardoverthelateralsurfaceofthemasseterandcanbe
palpatedatthetenseanteriormarginofthemassetermuscle.
Initscourseductpiercesbuccalfatpad,buccopharyngealfascia
andbuccinatormuscle(obliquely)andopensonthemucous
membraneofcheekoppositetoseconduppermolartooth.
Whenintraoralpressureisraised(duringblowing)theductis
compressedbetweenthebuccinatorandmucousmembrane,
preventinginflationoftheduct.

112.Whichlayerofthescalpisvascular?
a)Pericranium
b)Superficialfascia
c)Skin
d)Aponeurosis
CorrectAnswer-B
Ans.is'b'i.e.,Superficialfascia
Thescalpisasofttissuethatcoversthecalvariaoftheskull.It
consistsoffivelayersandcanbememorizedbyamnemonic
usingtheinitiallettersoftheword.SCALP:?
Skin
Closenetworkofconnectivetissue(superficialfascia)
Aponeurosis(galeaaponeurotica)withoccipitofrontalismuscles
Looseareolar(subaponeurotic)tissue
Pericranium(outerperiosteumoftheskull)

113.Allofthefollowingpassthroughthe
Sinusofmorgagniexcept-
a)Auditorytube
b)Levatorvelipalatini
c)Ascendingpalatineartery
d)Stylopharyngeus
CorrectAnswer-D
Ans.'d'i.e.,Stylopharyngeus
SinusofMorgagniisthelargegapbetweentheupperconcave
borderofthesuperiorconstrictorandthebaseoftheskull.
Thestructurespassingthroughitare:
1.Auditorytube
2.Levatorvelipalatini
3.Ascendingpalatineartery
4.Palatinebranchofascendingpharyngealartery

114.Chordatympaniisabranchof?
a)Facialnerve
b)Trigeminalnerve
c)Greaterauricularnerve
d)Externallaryngealnerve
CorrectAnswer-A
Ans.is'a'i.e.,Facialnerve[Ref:BDC6thleVol3p.371]
Branchesoffacialnerve

1. Infallopian(facialcanal):-Greaterpetrosal(greatersuperficial
petrosal)nerve,nervetostapedius,chordatympani.
2. Atitsexitfromstylomastoidforamen:-Posteriorauricular,digastric
nerve,stylohyoidnerve.
3. Terminalbranches:-Temporal,zygomatic,buccal,marginal
mandibular,andcervical.

115.Chorda-tympanidoesnotcarrywhich
fibers?
a)Preganglionicparasympatheticfibersforsublingualglands
b)Preganglionicparasympatheticfibersforsubmandibulargland
c)Preganglionicparasympatheticfibersforparotidgland
d)Tastefibersfromanteriortwothirdoftongue
CorrectAnswer-C
Ans.is'c'i.e.,Preganglionicparasympatheticfibersforparotid
gland
Chordatympaniisabranchoffacialnervemainlycarryingtaste
sensationsfromtheanterior2/3rdofthetongue
Thechordatympanicarriestwotypesofnervefibersfromtheir
originwiththefacialnervetothelingualnervethatcarriesthemto
theirdestinations:
Specialsensoryfibersprovidingtastesensationfromtheanterior
two-thirdsofthetongue.
Preganglionicparasympatheticfiberstothesubmandibularganglion,
providingsecretomotorinnervationtotwosalivaryglands:the
submandibularglandandsublingualglandandtothevesselsofthe
tongue,whichwhenstimulated,causedilationofbloodvesselsof
thetongue.

116.Whichofthefollowingpassthroughthe
Hypoglossalcanal?
a)Hypoglossalnerve
b)Externaljugularvein
c)Facialnerve
d)Mandibularnerve
CorrectAnswer-A
Ans.is'a'i.e.,Hypoglossalnerve[RefBDC6th/eVol.3p.18-20]

117.Whichmuscleisantagonistto
orbicularisoculithatisnotsuppliedby
facialnerve?

a)LevatorPalpebraesuperioris
b)Orbicularisoris
c)Superioroblique
d)Inferioroblique
CorrectAnswer-A
Ans.is'a'i.e.,LevatorPalpebraesuperioris
Orbicularisoculiclosestheeyeandissuppliedbythefacialnerve.
LevatorPalpebraesuperiorisopenstheeyelidandissuppliedbythe
oculomotornerve.

118.Allofthefollowingaremainbranchesof
Trigeminalnerveexcept?
a)Mandibularnerve
b)Maxillarynerve
c)Ophthalmicnerve
d)Opticnerve
CorrectAnswer-D
Ans.is'd'i.e.,Opticnerve[RefBDC6th/eVol3p.369]
Divisionsoftrigeminalnerve

1. Ophthalmicdivision(Ophthalmicnerve:V1)
2. Maxillarydivision(Maxillarynerve:V2)
3. Mandibulardivision(Mandibularnerve:V3)

119.Extensionoftheretropharyngealspace
isbetween?
a)Alarfasciaandbuccopharyngealfascia
b)buccopharyngealfasciaandprevertebralfascia
c)AlarfasciaandPrevertebralfascia
d)None
CorrectAnswer-A
Ans.is'a'i.e.,Alarfasciaandbuccopharyngealfascia
Retropharyngealspace
Theretropharyngealspaceisapotentialspaceoftheheadand
neck,boundedbythebuccopharyngealfasciaanteriorlyandthealar
fasciaposteriorly.Togetherwiththelateralpharyngealspace,these
spacesaretermedtheparapharyngealspaces.
Itcontainstheretropharyngeallymphnodes.
Becauseseriousinfectionsofteethcanspreaddownthisspaceinto
theposteriormediastinum,itisoftenconfusedwiththedanger
space.Thedangerspaceisactuallybetweenthealarfasciaandthe
prevertebralfasciaandextendsfromthecranialbaseabovetothe
levelofthediaphragm.
Itislimitedabovebythebaseoftheskull,andbelowwherethealar
fasciafuseswiththebuccopharyngealfasciaataboutthelevelofT4
andthecarina.

120.Delphiannodesare?
a)Prelaryngealnodes
b)Occipitalnodes
c)Coeliacnodes
d)Noneoftheabove
CorrectAnswer-A
Ans.is'a'i.e.,Prelaryngealnodes
TheDelphiannode(prelaryngeal)alongwithparatrachealnodes,
pretrachealnodes,perithyroidalnodesmakeuplevelVIcervical
lymphnodes,andisnotroutinelyexcisedinradicalneck
dissections.Itreceiveslymphfromthethyroidandlarynx.
TheDelphiannodegainsitsnamefromtheOracleofDelphi,whose
prophecy,inthiscase,wouldbeofanunpleasantdeathsecondary
tolaryngealcancer.
Theinvolvementofthisnodecanbearesultofdiffusenodal
involvementinheadandnecksquamouscellcarcinomaorisolation
fromthedirectlymphaticspreadoflaryngealcancerthroughthe
anteriorcommissure.Thyroidcarcinomasmayalsoinvolvethis
node.

121.Straightsinusisformedby?
a)InferiorSagittalSinus
b)InternalJugularveins
c)SuperiorSagittalSinus
d)Tranversesinus
CorrectAnswer-A
Ans.is'a'i.e.,InferiorSagittalSinus
Thestraightsinus,alsoknownastentorialsinusorthesinusrectus,
isanareawithintheskullbeneaththebrainthatreceivesvenous
blood.
Straightsinusisformedbytheunionoftheinferiorsagittalsinuswith
thegreatcerebralvein.
Itisconsideredacontinuationoftheinferior
sagittalsinus.
Itdrainsintothetransversesinus,mostcommonlyintheleftone.
Thestraightsinusissituatedwithintheduramater,wherethefalx
cerebrimeetsthemidlineoftentoriumcerebelli.
Incross-section,itistriangular,containsafewtransversebands
acrossitsinterior,andincreasesinsizeasitproceedsbackward.

122.Whicharterysuppliestheparacentral
lobule?
a)MedialStriateartery
b)CallosoMarginalartery
c)Pericallosalartery
d)Frontopolarartery
CorrectAnswer-B
Ans.is'b'i.e.,CallosoMarginalartery[RefBDCVol.3
6th/e
p.461,462]
Callosomarginalarteryisabranchofanteriorcerebralartery
thatsuppliestheparacentrallobulewhichhasaroleincontrol
ofmicturition
Anteriorcerebralartery
Hasfollowingbranches:-

1. Medialstriateartery(recurrentarteryofHeubner):Itsupplies
caudatenucleus(ventralpart),putamen,andanteriorlimbandgenu
ofinternalcapsule.
2. Fronto-polarartery:Itsuppliesmedialandorbitalsurfacesoffrontal
lobe.
3. Orbitalbranches:Itsuppliesmedialandorbitalsurfacesoffrontal
lobe.
4. Calloso-marginalartery:Itsuppliestheparacentrallobuleandparts
ofgyruscinguli.
5. Pericallosalartery:Itsuppliesmedialsurfaceofparietallobeand
precuneous.

123.Nucleusofbasalganglia
a)Dentate
b)Thalamus
c)Caudate
d)Rednucleus
CorrectAnswer-C
Ans:Ci.e.Caudatenucleus
Thebasalgangliahavefivenucleioneachsideofthebrain.
-Caudatenucleus
-Putamen
-Globuspallidus
-Subthalamicnucleus
-Substantianigra
Thecaudatenucleusandputamencollectivelyformthestriatum.
Theputamenandglobuspalliduscollectivelyformthelentiform
nucleus.
Theglobuspallidusisdividedintoexternalandinternalsegments

124.Mostlateralnucleusofcerebellumis?
a)Dentate
b)Globose
c)Fastigial
d)Emboliform
CorrectAnswer-A
Ans.is'A'i.e.,Dentate
Therearefourdeepcerebellarnuclei(fromlateraltomedial):
dentate,emboliform,globose,andfastigial.
Thegloboseandtheemboliformnucleiaresometimeslumped
togetherastheinterpositusnucleus.

125.

Superiormarginalgyrusisapartof?
a)Parietallobe
b)Frontallobe
c)Temporallobe
d)Occipitallobe
CorrectAnswer-A
Ans.is'a'i.e.,Parietallobe
Thesuperiormarginalgyrusisaportionoftheparietallobe.
ThisareaofthebrainisalsoknownasBrodmannarea40basedon
theuniversallyusedbrainmapcreatedbyKorbinianBrodmannto
definethestructuresinthecerebralcortex.


126.

Parsdorsalisisapartof?
a)Cerebrum
b)Cerebellum
c)Pons
d)Thalamus
CorrectAnswer-C
Ans.is'c'i.e.,Pons[RefFarlexPartnerMedicalDictionary
Farlex2012]
ParsDorsalis
Thepartoftheponsboundedlaterallybythemiddlecerebellar
pedunclesandanteriorlybytheventralpartofpons;itiscontinuous
withthetegmentumofthemesencephalonandcontainslongtracts
suchasthemedialandlaterallemnisci,cranialnervenuclei,and
reticularformatin.

127.Arborvitaeareseenin?
a)Cerebrum
b)Cerebellum
c)Pons
d)Thalamus
CorrectAnswer-B
Ans.is'b'i.e.,Cerebellum
Thearborvitaeisthecerebellarwhitematter,so-calledforits
branched,tree-likeappearance.
Insomeways,itmoreresemblesafernandispresentinboththe
cerebellarhemispheres.
Itbringssensoryandmotorinformationtoandfromthecerebellum.
Thearborvitaeislocateddeepinthecerebellum.
Situatedwithinthearborvitaearethedeepcerebellarnuclei;the
dentate,globose,emboliformandthefastigialnuclei.
Thesefourdifferentstructuresleadtotheefferentprojectionsofthe
cerebellum.

128.TotalvolumeofCSFis?
a)150ml
b)500ml
c)50ml
d)800ml
CorrectAnswer-A
Ans.is'a'i.e.,150ml
ThemajorsourceofCSFisthechoroidalplexusofall4ventricles,
mainlyintwolateralventricles.OthersourcesofCSFareependymal
cellsoftheventriclesandthebrainitself,viaperivascularspaces.
ThetotalvolumeofCSFinanadultisabout125-150ml.Therateof
formationofCSFisabout500-550ml/day.ThustheCSFisreplaced
3-4timeseveryday.
ThewaterypartofCSFissecretedbytransductionbuteachofits
constitutionsisactivelytransported.Na.issecretedintotheCSF
withthehelpofNa+ATPase.GlucoseentersCSFthroughfacilitated
diffusionmediatedbyGLUT-1.HCO3issecretedwiththehelpof
carbonicanhydrase.

129.Laterallemniscusterminatesinto?
a)Lateralgeniculatebody
b)Superiorcolliculus
c)Inferiorcolliculus
d)Inferiorolivarycomplex
CorrectAnswer-C
Ansis'c'i.e.,Inferiorcolliculus[RefBDC6th/eVol..3p.374]

130.Internalcapsule-Allofthefollowingare
partsexcept?
a)Anteriorlimb
b)Sublentiformpart
c)Retrolentiform
d)Prelentiform
CorrectAnswer-D
Ans.is'd'i.e.,Prelentiform
Theinternalcapsuleisdividedfrombeforebackwardsinto
followingparts:
Anteriorlimb
Posteriorlimb
Retrolentiformpart
Genu
Sublentiformpart

131.Substantiaferrugineaisfoundin-
a)Fourthventricle
b)Thalamus
c)Midbrain
d)Thirdventricle
CorrectAnswer-A
Ans.is'a'i.e.,Fourthventricle[RefMedicalDictionary,2009
FarlexandPartners]
Itisashallowdepression,ofabluecolorinthefreshbrain,
lyinglaterallyinthemostrostralportionoftherhomboidal
fossanearthecerebralaqueduct;itliesnearthelateralwallof
thefourthventricleandconsistsofabout20,000melanin-
pigmentedneuronalcellbodiesthenorepinephrine-containing
axonsofwhichhavearemarkablywidedistributioninthe
cerebellumaswellasinthehypothalamusandcerebralcortex.
Alsocalledaslocuscinereus,locusferrugineus.


132.Infundibulardiverticulumisanextention
of?
a)1stand2thventricles
b)3rdventricle
c)4thventricle
d)None
CorrectAnswer-B
Ans.is'B'i.e.,3rdventricle
Thirdventricleisamidlinecavityofdiencephalon.Itisamediancleft
betweentwothalami.Anterosuperiorlyitcommunicateswithlateral
ventriclethroughtheinterventricularforamen(foramenofMonro).
Posteroinferiorlyitcommunicateswithfourthventriclethrough
cerebralaqueduct(DuctofSylvius).
Therearefourextensions(recesses)ofthirdventricle:(a)
Suprapinealrecess,(b)Pinealrecess,(c)Infundibularrecess,and
(d)Opticrecess.

133.Whichofthefollowingisacomplete
sulcusinthebrain?
a)Calcarinesulcus
b)Paracentralsulcus
c)Both
d)None
CorrectAnswer-A
Ans.is'a'i.e.,Calcarinesulcus
Thecalcarinesulcus(orcalcarinefissure)isananatomical
landmarklocatedatthecaudalendofthemedialsurfaceofthe
brainofhumansandotherprimates.Itsnamecomesfromthe
Latin"calcar"meaning"spur".Itisacompletesulcus.
Foraccommodationinalimitedspacewithintherigidcranialbox,
thecerebralcortexisfoldedintonumerousgyriorconvolutions
separatedbysulciorfissures.Eventuallythetotalsurfaceareaof
thecortexofhumanbrainisincreasedtoabout2200cm2,inwhich
onlyaboutonethirdofthecortexisexposedasgyriandtwothirdis
hiddeninthesulci.

134.Whichofthefollowingisderivedfrom
theneuraltubeexcept?
a)Retina
b)Brain
c)Dorsalrootganglia
d)Pinealgland
CorrectAnswer-C
Ans.is'c'i.e.,Dorsalrootganglia[RefTextbookofhuman
embryology-786]
Nervoussystemdevelopsfromectoderm(neuroectoderm).Nervous
systemdevelopsfromneuraltubewhichinturndevtlopsbyprocess
ofneurulation,i.e.formationofneuralplateanditsinfoldinginto
neuraltube.

135.Cerebellovestibularfibrespassthrough
?
a)Superiorcerebellarpeduncle
b)Middlecerebellarpeduncle
c)Inferiorcerebellarpeduncle
d)None
CorrectAnswer-C
Ans.is'c'i.e.,Inferiorcerebellarpeduncle[RefBDCVol.III6thle
p.405]
Inferiorcerebellarpeduncle

1. Posteriorspinocerebellar
2. Cuneocerebellar(posteriorexternalarcuatefibres)
3. Olivocerebellar
4. Parolivocerebellar
5. Reticulocerebellar
6. Vestibulocerebellar
7. Anteriorexternalarcuatefibres
8. Striaemedullares

136.Longspinousprocessisseenin?
a)Cervicalvertebrae
b)ThoracicVertebrae
c)LumbarVertebrae
d)Sacrum
CorrectAnswer-B
Ans.is'b'i.e.,ThoracicVertebrae[Ref:BDC5th/eVol.3p.40]

137.Movementoccuringatatlanto-axial
joint?
a)Flexion
b)Bending
c)Rotation
d)Nodding
CorrectAnswer-C
Ans.is'c'i.e.,Rotation[Ref:Clinicalanatomy3rdiep.786]
Movmentspermittedatatlanto-occipitaljointare:-
Flexionandextension(noddingofhead),and(ii)Lateralflexion
(bendingofneck).
Movementspermittedatatlanto-axialjointsareside-to-siderotation
ofhead(lookingtowardsright-or-left).

138.Spinalsegmentalarteryisabranchof?
a)Ascendingspinalartery
b)Basilarartery
c)Posteriorspinalartery
d)Anteriorspinalartery
CorrectAnswer-A
Ans.is'a'i.e.,Ascendingspinalartery[RefSpinalCord
Medicine.DemosMedicalPublishing.]
Arterialsupplyofspinalcord


139.Whichofthefollowingisnota
permanentmucosalfold?
a)Heister'svalves
b)Transverserectalfold
c)Plicaecircularis
d)Gastricrugae
CorrectAnswer-D
Ans.is'd'i.e.,Gastricrugae[RefInderbirSinghHistologyp.
240;
BDC4th/eVol.Hp.241,245,274,
378;Gray's40thle
p.1138,1120,1151,1132,1178]
Gastricrugaeofstomach,andlongitudinalfoldsinmucosaofupper
rectumandcolonaretemporarymucosalfoldsandareobliterated
bydistension.Whereas,plicacircularis(valvesofkerkring)ofsmall
intestine,crescenticmucosalfoldsofcysticduct(spiralvalveof
Heister),transverse(horizontal)rectalfolds(Houston'svalvesor
plicatransversalis)andpermanentlongitudanalrectalcolumnsor
folds(foundin

140.Lipidraftsareseenin?
a)Ribosomes
b)Mitochondria
c)Plasmamembrane
d)ER
CorrectAnswer-C
Ans.C.Plasmamembrane.
*Lipidraftsareregionsinplasmamembranethataccumulate
Cholesterolandglycolipids(glycosphingolipids).
-Sothereregionsareslightlythickerthanotherareasofplasma
membrane.
-Becausefdistinctmolecularcompositionoflipidrafts,theycan
actusmicrocompartmentwithincells,givingthecellanadditional
waytoorganizepathways.
-Involvedintheregulationofsignaltransduction.
Typesoflipidrafts:
*Planarlipidrafts(noncaveolarorglycolipidrafts)
-Continuouswithplaneofcellmembrane(theyarenotinvaginate).
Caveolae:
*Specializedtypesoflipidraftswhichareflasklikeinvaginationsof
plasmamembrane.
*Producedwherecaveolinproteinispresentinlipidbilayer
membrane.

141.Markerofendoplasmicreticulum?
a)Acidphosphatase
b)Glucose-6-phosphatase
c)Catalase
d)LDH
CorrectAnswer-B
Ans.B.Glucose-6-phosphatase.
OrganelleorfractionMarkers
Plasmamembrane-Adenylcyclase,Na+ICATPase
Lysosome-Acidphosphatase
Golgiapparatus-Galactosyltransferase,GolgimannosidaseII,
Sialyltransferase,G1cNActransferase.
Endoplasmicreticulum-Glucose-6-phosphatase
Peroxisome-Catalase,Urate(uricacid)oxidase
Cytosol-Lactatedehydrogenase
Nucleus-DNA
Ribosome-HighcontentofRNA

142.Followingisafeatureofsimple
diffusion?
a)Againstaconcentrationgradient
b)Easyfornon-polarsubstance
c)Moreinthickmembrane
d)Requirescarrierprotein
CorrectAnswer-B
Ans.B.Easyfornon-polarsubstance
Simplediffusion:
Referstodiffusionofmoleculeacrossthemembranefollowinga
concentrationgradientorchemicalgradient,butwithoutthehelpof
anycarrierprotein.
Factorsinfluencing:
A)Rateofdiffusionisdirectlyproportionate:-
Concentration(chemical)gradient
Cross-sectionalareaofthemembranethroughwhichdiffusiontakes
place
Lipidsolubilityofthesubstance
B)Rateofdiffusionisinverselyproportionate:-
Thicknessofdiffusionmembrane
Sizeoftheparticle
Chargeorpolarityofsubstance
So,simplediffusionisfavoredbysmallsize,lipidsolubilityand
absenceofpolarity(non-polarsubstance)andcharge(neutral
molecule)throughathin,largemembranewheretheconcentration
gradientismore.

143.Mostcommonmechanismfortransport
intothecell?
a)Diffusion
b)Primaryactivetransport
c)Antiport
d)Cotransport
CorrectAnswer-A
Ans.A.Diffusion.
(RefPrinciplesofmedicalphysiologyp.3.)
Mostimportantandmostcommonmechanismoftransportis
passivediffusion.

144.TrueaboutNernstequation?
a)Usedtocalculateequilibriumpotential
b)Calculatedfornon-ionicsolution
c)NernstpotentialforClis-90my
d)Allarecorrect
CorrectAnswer-A
Ans.A.Usedtocalculateequilibriumpotential.
[Ref:Ganong24thlep.9;Principlesofmedicalphysiologyp.8]
RMPvalueiscalculatedonbasisofNernstequation.
Alsoreferred"Nernstpotential/Equilibriumpotential/Diffusion
potentialofK+.


145.ECFconcentrationof1C.is150meq/L
andICFconcentrationofleis5meq/L.
WhatistheequilibriumpotentialforK+
is?

a)+60mV
b)-60mV
c)-90mV
d)+90mV
CorrectAnswer-C
Ans.'c'i.e.,-90mV
NernstEquation?
Canbeusedtofindthecellpotentialatanymomentinduringa
reactionoratconditionsotherthanstandard-state.
E=cellpotential(V)underspecificconditions
E=cellpotentialatstandard-stateconditions\
R=idealgasconstant=8.314J/mol-K
T=temperature(kelvin),whichisgenerally25C(298K)
n=numberofmolesofelectronstransferredinthebalanced
equation
F=Faraday'sconstant,thechargeonamoleofelectrons=
95,484.56C/mol
lnQc=thenaturallogofthereactionquotientatthemomentintime.
-61150

EK=log=90mV
+15

146.Nernnstequationrelatedtoequilibrium
potentialdoesnotdependupon?
a)Concentrationgradient
b)Electricgradient
c)Non-ionicsolution
d)Concentrationofionsintwosolution
CorrectAnswer-C
Ans.C.Non-ionicsolution
[RefGuyton12th/ep.50;Principlesofmedicalphysiologyp.8]
Nernstequationisforionicsolution.

147.DuetoDonnan-Gibbseffect?
a)ConcentrationofK.isgreaterinECF
b)ConcentrationofclisgreaterinECF
c)TotalionsaremoreinICF
d)Allaretrue
CorrectAnswer-C
Ans.C.TotalionsaremoreinICF
[RefPrinciplesofmedicalphysiologyp.7]
Gibbs-Donnanequilibrium:
Themammaliancells(intracellularfluid)containsnon-diffusible
anionlikeproteinsandorganicphosphatewhereasK'andCl-are
diffusiblecationandanion,respectively.
TheECFcontains1K+andCl-asdiffusiblecationandanion
respectively.
DuetoGibbs-Donnanequilibrium:-
ConcentraionofCl-isgreaterinICFthanECF(concentrationof
diffusiblecationisgreaterincompartmentwithnon-diffusibleanion).
ConcentrationofClisgreaterinECFthanICF.
TotalnumberofionsisgreaterinICFthanECF.
Alltheseeffectshelptomaintaintheshapeandvolumeofcellsby
distributingdiffusibleionsacrossthemembraneaccordingto
physiologicaldemand.

148.ATPaseiswhichtypeofpump?
a)Secondaryactive
b)Electrogenic
c)Symport
d)Alloftheabove
CorrectAnswer-B
Ans.B.Electrogenic
[RefGanong24th/ep.51,10;Guyton12th/ep.53]
Sodium-potassiumpump:
*Mostimportantpumpforprimaryactivetransportinbody.
*AnElectrogenicpump.
*ResponsibleformaintainingNa+&K+conc.differenceacrosscell
membrane.
Mechanism:
*ICF-HighK+concentration.
*ECF-HighNa+concentration.
*BothNa+&K+transportedagainstconcentrationgradient.
*UsesenergybyhydrolyzingATP;
Functions:
*PumpcontainsATPaseactivity.
*Na+-K+pumpextrudes-
-3Na+outfromcell.
-Pumps2K+intocell.
-CouplingratioofNa+-K+pump-3:2.

149.Diffusionrelatedto02transportacross
respiratorymembraneisanexampleof?
a)Simplediffusion
b)Facilitateddiffusion
c)Activediffusion
d)Osmoticdiffusion
CorrectAnswer-A
Ans.A.Simplediffusion.
[Ref.Ganong25thlep.51&24thlep.53]
Simplediffusion-Movementoffats,oxygen,CO,throughlipid
portionofmembrane.
Facilitateddiffusion-Movementofglucoseandsomeamino-acids.
Primaryactivetransport-IonsK+,Na*,
Secondaryactivetransport-Glucoseororamino-acidintothecell
alongNa`(Sympoftorcotransport)1-1+outofthecellagainstNa+
(Antiportorcountertransport).

150.Measurementofintracellularfluidina50
yearsoldmaleisdoneby?
a)Dilutionmethod
b)Evansblue
c)D20
d)Indirectly
CorrectAnswer-D
Ans.D.Indirectly.
[RefGanong23th/ep.3-6;Guyton12th/ep.286-288]
Mostofthefluidiscalculateddirectlybydilutionmethod,exceptfor
intracellularfluid(ICF)andinterstitialfluid.
Boththesearecalculatedindirectlybycalculatingotherbodyfluids.
ICF=Totalbodywatervolume-ECFvolumeInterstitialfluid=ECF
volume-Plasmavolume.

151.Calculationofinterstitialfluidina50
yearsoldisdoneby?
a)TBWminusECF
b)ECFminusplasmavolume
c)ICFminusECF
d)TBWminusICF
CorrectAnswer-B
Ans.B.ECFminusplasmavolume.
[RefGanong23th/ep.3-6;Guyton12th/ep.286-288]
Mostofthefluidiscalculateddirectlybydilutionmethod,exceptfor
intracellularfluid(ICF)andinterstitialfluid.
Boththesearecalculatedindirectlybycalculatingotherbodyfluids.
ICF=Totalbodywatervolume-ECFvolumeInterstitialfluid=ECF
volume-Plasmavolume.

152.Excitabilityofcellsismaximallyaffected
bychangeinconcentrationofwhich
ion?

a)IC+
b)Na*
c)a-
d)Ca+2
CorrectAnswer-D
Ans.D.Ca+2
[RefPrinciplesofmedicalphysiologyp.801]
Effectsofionconcentrationchangeonmembranepotential:
*Hypercalcemia
-AnincreaseinextracellularCa2+concentrationcanstabilizethe
membranebydecreasingexcitability.
*Hypocalcemia
-Calciumionismembranestabilizers.
-AdecreaseinextracellularCa2+concentrationincreasesthe
excitabilityofnervebydecreasingtheamountofdepolarization
necessarytoproducetheactionpotential.
-Hyperexcitabilityisseen.

153.Non-specificpainpathwayisfor?
a)Nociceptivepain
b)Neuropathicpain
c)Idiopathicpain
d)Inflammatorypain
CorrectAnswer-C
Ans.C.Idiopathicpain
[RefTextbookofpsychotherapyp.6]
Painmaybedividedinto:-
Nociceptivepain:
Itismusculoskeletalpainthatresultsfrominjuryorinflammatoryor
degenerativedisorders,e.g.bonefractureorarthritisorburn.
Neuropathicpain:
Itisduetodysfunction/damageofnerves,e.g.prolapse
intervertebraldisc.
Mixedpain:
Ithascomponentsofbothnociceptiveandneuropathicpains.
Idiopathicorunspecifiedpain:
Itispurelypsychologicalinnatureandisthereforecalled
psychogenicpain.

154.Dullvisceralpainiscarriedbywhich
typeofneurons?
a)Agamma
b)Aa
c)Cfibres
d)B
CorrectAnswer-C
Ans.C."C"fibers.
[RefGanong24th\ep.92&23rdlep.89]
Aviscusdoesnothaveanyothersensation(e.g.touch,temperature
etc)exceptpain.
Duetosparsedistributionofpainreceptors,visceralpainispoorly
localized.
Visceralpainsensationiscarriedbytype-Cafferentsinthe
sympatheticsystem(fromthoracicandabdominalviscera)and
parasympatheticsystem(frompelvicviscera).

155.'C'fiberscarrysensationsthroughwhich
pathway?
a)Posteriorcolumn
b)Anteriorspinothalamictract
c)Lateralspinothalamictract
d)Alloftheabove
CorrectAnswer-C
Ans.C.Lateralspinothalamictract.
[RefGanong23rdiep.648;Goyton12thlep.573].
CfiberscarryPain(slowpain)andtemperaturesensation.
Thesearecarriedbylateralspinothalamictract.

156.TypesCnervefibersare?
a)Sensory
b)Motor
c)Mixed
d)Anyoftheabove
CorrectAnswer-A
Ans.A.sensory.
TypeCfibers:
Postganglionicautonomicfibers.
Sensoryfunctionafferenttotemperature&pressure.
Nomotorfunction.

157.Warmthsensationiscarriedby?
a)Aafibers
b)A(3fibers
c)Ayfibers
d)ASfibers
CorrectAnswer-A
Ans.a.Aafibers
[RefGanong25"Yep.94&24"lep.92;Principlesofphysiologyp.
512]
Temperature(warmth/cold)sensationiscarriedbyA&Cfibers.

158.Trueaboutmyosin?
a)Thinfilament
b)Coversactivesiteofaction
c)HasATPaseactivity
d)Ca'bindingprotein
CorrectAnswer-C
Ans.C.HasATPaseactivity
[RefGanong241thlep.100,102]
Theshorteningofmusclefiberoccursduetoslidingofactin
filamentsonmyosinfilament.
However,therearefourmuscleproteinsinvolvedintheprocess:-
Actin,Myosin,Tropomyosin,andtroponin.
Myosin:
Myosinistheproteinthatconstitutesthethickfilaments.Myosinof
skeletalmuscleismyosin-II.
Myosinparticipatesinthecontractilemechanismandalsoactsasan
ATPase.

159.Incardiacmuscles,T-tubulesarepresent
at?
a)Zlines
b)Alines
c)Ilines
d)A-Ijunction
CorrectAnswer-A
Ans.A."Z"lines.
[RefPrinciplesofmedicalphysiologyp.794]
Thecardiacmusclecellcontainsactin,myosinthesarcotubular
systemandotherorganellesseeninskeletalmuscles.
However,theTsystemofcardiacmuscleislocatedatZlinesrather
thanatA-Ijunction,whereitislocatedinskeletalmuscles.

160.Inamusclefiberatrest,thelengthofthe
Ibandis1mmandAbandis1.5mm.
Whatisthelengthofthesarcomere

a)0.5mm
b)2.5mm
c)3.5mm
d)5mm
CorrectAnswer-B
Ans.B.2.5mm.
SarcomereistheportionofmyofibrilbetweentwoZlines.
Thus,lengthofsarcomereingivenquestion=0.5mm(1/2Iband)+
1.5mm(Aband)+0.5mm(1/2Iband)=2.5mm.

161.Whitefibersarepresentinwhich
muscle?
a)Calfmuscles
b)Backmuscles
c)Glutealmuscles
d)Handmuscles
CorrectAnswer-D
Ans.D.Handmuscles.
[RefPrinciplesofmedicalphysiologyp.118]
Typeofmusclefibers:
Type1(red)ortype2(white).
Mostofthemuscleinhumanbodycontainsbothtypesoffibers,i.e.
mostmusclesinthebodyhavebothtypesofmotorunits.
But,somemuscleshavemoretype1fibers(type1motorunits)than
type2,andvice-versa.
Accordingly,skeletalmusclescanbedividedinto:
Redmuscles
Thesemusclescontainmoretype1(red)fibers,thereforeareslow
twitchmusclesandcontainmoremyoglobincontent.
Thesearemuscleswhichrequireprolongedcontraction,for
example,muscleswhichhelpinmaintainingposture,i.e.Back
musclesgluteusmuscles(atbackofhip)andcalfmuscles.
White(pale)muscles
Thesemusclescontainmoretype2(white)fibersandarefasttwitch
muscles.
Thesearemuscleswhichhelprapidcontractionsandfiner
movements.
ExamplesareHandmusclesandextraocularmuscles.


162.Functionofmusclespindleis?
a)Movementofalimb
b)Muscletonemaintenance
c)Goalorientedmusclecontraction
d)Alloftheabove
CorrectAnswer-B
Ans.B.Muscletonemaintenance
[RefUnderstandingofmedicalphysiologyp.138]
Ifaskeletalmuscleisstretched,itrespondbycontracting.
Inotherwords,ifamuscleisstretched,ittendstobecomeshorter
andmorestiff,therebyresistingstretch.
Thisisbecauseofreceptorsensitivetostretchwithinthemuscle.
Stretchreceptorwithinamuscleiscalled"musclespindles".
Stretchreflexhastwoprincipalfunctions:
1)Tomaintainmuscletone
Toneisatendencyofamuscletoresistbeingstretched.
Muscletoneisnotonlyimportantformaintainingposturebutalso
facilitateslocomotionandmakesallvoluntarymovementsmooth.
2)Tomakemusclesrespondtostretchandrelease.

163.Afferentsforstretchreflexesarecarried
bywhichfibers?
a)Aa
b)Ay
c)TypeB
d)TypeC
CorrectAnswer-A
Ans.A.Aa
[RefGanong24th/ep.229]
Sensoryinnervation(afferent)ofmusclespindle:
Twotypesofsensorynervefibers(afferent)originatefromthe
intrafusalfibers:
Annulospiral(Primary)endingarewoundaroundthecentralregion
ofbothnuclearbagfiberandnuclearchainfiber.
TheseareA(orI)fibers.
Flower-spray(Secondary)endingsinnervatetheperipheralparts
(ends)ofnuclearchainfiber.
TheseareAor(orII)fibers.

164.Spinalcordhashowmanysynapsesin
golgitendonreflex?
a)1
b)2
c)3
d)4
CorrectAnswer-B
Ans.B.2
[Ref:Principlesofmedicalphysiologyp.786]
StrechreflexthroughmusclespindleMonosynaptic
GolgitendonreflexBisynaptic.

165.Inversestretchreflexisa?
a)Monosynapticreflex
b)Bisynapticreflex
c)Polysynapticreflex
d)Nonsynapticreflex
CorrectAnswer-B
Ans.B.Bisynapticreflex
[RefPrinciplesofmedicalphysiology3rd/ep.786]
Golgitendonreflex(inversestretchreflex)isbisynapticreflex.

166.Facilitatorypresynapticneurotransmitter
is?
a)GABA
b)Glycine
c)Glutamate
d)Aspartate
CorrectAnswer-C:D
Ans.is'c>d'i.e.,Glutamate>Aspartate
[RefUnderstandingofmedicalphysiology1"/ep.412]
Glutamateisthechiefexcitatoryneurotransmitterinthebrainand
spinalcord.
Aspartateseemstobethechiefexcitatoryneurotransmitterof
corticalpyramidalcells.

167.

Nissl'sgranulesarefoundinwhichpartof
nervecell-

a)Axonhillock
b)Axons
c)NodeofRanvier
d)Body
CorrectAnswer-D
Ans.D.Body
[RefChaudhri7h/ePrinciplesofmedicalphysiology-7]
NisslBodies(Nisslgranuleortigroidbody):
Nisslbodiesarelargegranularbodyfoundinneuron.
Presentalloverthesoma(body),exceptingaxonhillockandthey
extendtosomeextentinthedendrites,butnotwithintheaxon.
Thesegranulesareroughendoplasmicreticulumwithfree
ribosomesandarethesiteofproteinsynthesis.
Theyarethoughttobeinvolvedinthesynthesisofneurotransmitter
suchasacetylcholine.
Nisslbodiesarebasophilicgranules.
Chromatolysis(disappearanceofNisslbodies)isanimportant
histologicalsignofneuronalinjury.
Whenthedemandoftheproteinsynthesisisgreatthenisslgranules
overworkandmanyaltogetherdisappear(chromatolysis).

168.Stereociliaarefoundin?
a)Eye
b)Nose
c)Tongue
d)Epididymis
CorrectAnswer-D
Ans.D.Epididymis
*Stereociliaarefoundinthreeplaces
-Haircellsofinnerear
-Epididymis
-Ductusdeferens.

169.Stereociliaarepresentin?
a)Tastebuds
b)Haircells
c)Retina
d)Nose
CorrectAnswer-B
Ans.B.Haircells.
[RefGanong25th/ep.202]
Haircellshaveacommonstructure.
ThetallesthaircelliscalledasKinociliumandtheprogressively
shorterhaircellsarecalledasStereocilia.
Thesehaircellsareconnectedtogetherbytiplinkswhichhas
mechanicallysensitivecationchannels.

170.CSFpressureisincreasedinallexcept-
a)Forcedinspiration
b)Coughing
c)Valsalvamanoeuvre
d)Crying
CorrectAnswer-A
Ans.A.Forcedinspiration.
[RefEssentialofmedicalphysiologyp.950]
Eventslikecoughing,valsalvamanoeuvre,andcryingincreasethe
pressurebydecreasingabsorption.
CompressionofIJV(internaljugularvein)alsoraisestheCSF
pressure.

171.CSFispresentinwhichspace?
a)Centralcanalofspinalcord
b)Ventriclesofbrain
c)Subarachnoidspace
d)Alloftheabove
CorrectAnswer-D
Ans.D.Alloftheabove.
[RefGanong24th/ep.603;Principlesofmedicalphysiologyp.293]
*Cerebrospinalfluid(CSF)isaclear,colorless,almostproteinfree
filtrate(transduate)ofblood.
*Itispresent,
-Aroundthebrain(insubarachnoidspace)andinsidethebrain(in
itsventricles).
-Aroundthespinalcord(insubarachnoidspace)andinsidethe
spinalcord(initscentralcanal).

172.FunctionofGhrelin?
a)Stimulatewaterabsorption
b)Increaseappatite
c)Regulationoftemperature
d)Stimulatelipogenesis
CorrectAnswer-B
Ans.B.Increaseappatite
[RefClinicalendocrinologyp.48;Ganong24th/ep.487]
*Ghrelinisapeptidesecretedbyoxynticcellsingastricfundusthat
arecharacterizedbyround,compact,electron-densesecretory
granules.
*Ghrelinpromotesfoodintake,i.e.,stimulateappetite(orexigenic).
*Increaseswithanorexia.
*Inhuman,ghrelininduceslipolysis.
*Ghrelinisreleasedfromthestomachinfastingstate.
-Increaseshungerbyinhibitingtheventromedialhypothalamus
(satietycenter).
-StimulatesGHsecretions.

173.Locationofvisualcortex?
a)Precentralgyrus
b)Postcentralgyrus
c)Sylvianfissure
d)Calcarinesulcus
CorrectAnswer-D
Ans.D.Calcarinesulcus
[RefPrinciplesofmedicalphysiologyp.531]
Theoccipitallobeisthevisualprocessingcenterofbraincontaining
mostoftheanatomicalregionofvisualcortex.
Primaryvisualareaisbrodmannarea17(alsocalledVI)onmedial
sideofoccipitallobeincalcarinesulcus.
Itisalsocalledasstriateareaorstriatecortexbecauseitcanbe
identifiedbyalargestripeofmyelin,thestriaofgennari.

174.Parvocellularpathwayforvisionis
concernedwith?
a)Finedetailsofobject
b)Movementsofobject
c)Flickeringfeatures
d)Depthofvision
CorrectAnswer-A
Ans.A.Finedetailsofobject
[RefGanong24th/ep.190]
Parvocellularpathway(arisesfromlayer3,4,5,6ofLGB).
Axonsfromparvocellularterminateinlayer4ofthevisualcortex.
Parvocellularpathwaycarriessignalsforcolorvision,texture,shape
andfinerdetails.

175.Sweatglandsaresuppliedbyallexcept?
a)Cholinergicneurons
b)Sympatheticneurons
c)Adrenergicneurons
d)C-fibers
CorrectAnswer-C
Ans.C.Adrenergicneurons
[RefUnderstandingofmedicalphysiologyp.786]
Nervesupplyofsweatglandisuniqueinthatitissympatheticbut
cholinergic(mostothersympatheticsitesarenoradrenergic).
Post-ganglionicsympatheticfibersareType-Cfibers.

176.Gustatorypathwayinvolveswhich
nerve?
a)Facial
b)Glossopharyngeal
c)Vagus
d)Alloftheabove
CorrectAnswer-D
Ans.D.Alloftheabove
[RefPrinciplesofmedicalphysiologyp.87]
Fibersinnervatingtastebudsarebranchesofcranialnerves,i.e.,
branchesoffacial,glassopharyngeal,andvagusnerves.
Thetastebudsintheanteriortwo-thirdsofthetongueareinnervated
bylingualbranchesofthefacialnerve.
Thetastebudsintheposteriorthirdofthetongueareinnervatedby
glossopharyngealnerve.
Tastereceptorsinthepharyngealpartoftongueandonthehard
palate,softpalate,andepiglottisareinnervatedbyfibersofthe
vagusnerve.

177.Functionofcerebellum?
a)Regulationoftone
b)Coordinationofeyemovement
c)Planning&initiationofmovement
d)Alloftheabove
CorrectAnswer-D
Ans.D.Alloftheabove
Functionsofthecerebellum:
Regulationoftone,postureandequilibrium.
Smootheningandcoordinationofvoluntarymovements.Themost
importantfunctionandthebestknownfunctionofcerebellumis
coordinationofmovements.
Coordinationofeyemovements
Planningandinitiationofmovements
Learningoffrequentlyperformedvoluntarymovements.
Timingandcomparison:
Thecerebellumisbelievedtobeatimingdevicethattimesthe
durationofagonisticmuscleactivityandlatencyofantagonistic
activity,Sothatanymovementishaltedatthecorrectpoint.

178.Whichofthefollowingisnota
metabotropicreceptorforserotonin?
a)5HT,A
b)5HT,13
c)5HT2A
d)5HT3
CorrectAnswer-D
Ans.D.5HT3
[Ref:Ganong32p.139;Principlesofmedicalphysiologyp.786]
Metabotropicreceptors:
AreG-proteincoupledreceptorswhichactthroughsecond
messenger.
Serotoninincluded:
Withtheexceptionofthe5-HT3receptor,aligandgatedion
channel,allother5-HTreceptors.

179.Twopointdiscriminationismainlya
functionofwhichtouchreceptors?
a)Merkel'sdisc
b)Ruffini'sendorgan
c)Pacciniancorpuscle
d)Meissner'scorpuscle
CorrectAnswer-A
Ans.A.Merkel'sdisc
[RefPrinciplesofmedicalphysiologyp.647]Tactile(touch)
receptors
Fortouch(superficaltouch):-
Meissner'scorpuscle(detecttextureofsurface,i.e.roughor
smooth),Merkel'sdisc(detecttwopointdiscrimination).

180.Whichofthefollowingdefinesvital
capacity?
a)Airinlungafternormalexpiration
b)Maximumairthatcanbeexpiratedafternormalinspiration
c)Maximumairthatcanbeexpiratedaftermaximuminspiration
d)Maximumairinlungafterendofmaximalinspiration
CorrectAnswer-C
Ans.C.Maximumairthatcanbeexpiratedaftermaximum
inspiration
Vitalcapacity(VC):
4700ml.
Amountofairthatcanbeexhaledwithmaximumeffortafter
maximuminspiration(ERV+TV+IRV).
Usedtoassessstrengthofthoracicmusclesaswellaspulmonary
function.

181.Functionalresidualcapacityinnormal
adultis?
a)500ml
b)1200ml
c)2400ml
d)3200ml
CorrectAnswer-C
Ans.C.2400ml
[Ref:Ganong24th/ep.629-631]
Volumeofairinthelungsattheendofanormalexpiration.
Inotherwords,FRC=ERV+RV.
About2400ml.

182.Whatismaximumvoluntaryventilation?
a)Amountofairexpiredinonemunuteatrest
b)Maximumamountofairthatcanbeinspiredandexpiredinone
minute
c)Maximumamountofairthatcanbeinspiredperbreath
d)Maximumamountofairremaininginlungafterforcedexpiration
CorrectAnswer-B
Ans.B.Maximumamountofairthatcanbeinspiredand
expiredinoneminute
[RefGuyton12th/ep.472,473;Ganong24th/ep.633]
Maximumvoluntaryventilation(MVV):
Itisthemaximumamountofairthatcanbemovedintoandoutof
thelungsin1minutebyvoluntaryeffort.
About125-170L/min.

183.TrueaboutCarboxyhemoglobin?
a)Takeup02veryquickly
b)Causeshistotoxichypoxia
c)CausesleftshiftofHb-02dissociationcurve
d)Allaretrue
CorrectAnswer-C
Ans.C.CausesleftshiftofHb-02dissociationcurve
[RefUnderstandingofmedicalphysiologyp.173].
COHbcannottakeup02;liberatesCOveryslowlyandshifts
dissociationcurveofremainingHbO,toleft,decreasingtheamount
of02released.
COpoisoningcausesanemichypoxiabecausetheamountofHb
thatcancarryO2,isreducedbutthetotalHbamountofbloodis
unaffectedbyCO.

184.WhatisthedifferencebetweenHb-0
dissociationcurveandHb-COcurve?
a)COshiftsthecurvetoleft
b)COhasmoreaffinitytoHb
c)Co-Hbcurveissimilarto02-Hbcurve
d)Allaretrue
CorrectAnswer-D
Ans.D.Allaretrue
[RefUnderstandingsofmedicalphysiologyp.786]
COHbcannottakeup02;liberatesCOveryslowlyandshifts
dissociationcurveofremainingHb02toleft,decreasingtheamount
ofO2,released.
TheaffinityofHbforCOis200-250timesitsaffinityfor02.
SoCO-Hbdissociationcurveisalmostidenticalto02Hb
dissociationcurveexceptthatpartialpressureareatalevelof
1/250.

185.NottrueaboutBohreffect?
a)Decreaseaffinityof02byincreasePCO,
b)LeftshiftofHb-02dissociationcurve
c)ItisduetoH*
d)Allaretrue
CorrectAnswer-B
Ans.B.LeftshiftofHb-02dissociationcurve
[RefGanong24th/ep.644,Understandingofmedicalphysiology
p.789]
Bohreffect:
IncreaseinPCO2decreasesthe02affinitytohemoglobinandshifts
theoxygendissociationcurvetoright;itiscalledBohreffect.
TheeffectofraisedPCO2ismediatedbyincreaseinhydrogenion
concentration.
Hydrogenionsshiftthecurvebybindingwithhemoglobin.
Deoxygenatedhemoglobin(Deoxyhemoglobin)bindsII'more
activelythandoesoxygentatedhemoglobin(oxyhemoglobin).
Ionsbindtodeoxyhemoglobinandreducetheaccessibilityof
oxygentohaemgroups.
Thatiswhyinthepresenceofmorehydrogenions,lessoxygencan
combineswithhemoglobinatagivenP02andtheoxygen
dissociationcurveshiftsrightward.

186.Whichofthefollowingexplainsuptakeof
0,infetalcirculation?
a)Bohr'seffect
b)Halden'seffect
c)HigheraffinityofHbFfor0,
d)Noneoftheabove
CorrectAnswer-C
Ans.C.HigheraffinityofHbFfor0,
[RefSmith'sanaesthesia3rdlep.77]
Fetalhemoglobin'sgreateraffinityforoxygenimprovesoxygen
uptakeattheplacenta.
Agreateraffinityforoxygenisanadvantageforuptakeatthe
placenta.

187.Whichofthefollowingexplainsdelivery
of02infetalcirculation?
a)Bohr'seffect
b)Halden'seffect
c)HigheraffinityofHbFfor02
d)Noneoftheabove
CorrectAnswer-A
Ans.A.Bohr'seffect
Uptakeof0,atplacentaDuetohighaffinityofHbFfor02
Deliveryof02attissuelevelDuetoBohreffect.

188.Trueof02-Hbdissociationcurve?
a)Straightlinecurve
b)100%saturatedatP02of100mmHg
c)Cooperativebinding
d)Hbmoleculecancarry6moleculesof0,
CorrectAnswer-C
Ans.C.Cooperativebinding
Molecularbasis:
"Duetophenomenon"CooperativebindingofOxygento
hemoglobin".
Stepsinvolved:
Hemoglobin,atetramer
FourO2moleculesbinds1Hbmolecule.
1stO2moleculebondswithgreatestdifficulty.
IncreasesaffinitytonextO2molecule.

189.Whichincreasesaffinityofhemoglobin
for0,-
a)Acidosis
b)Hyperthermia
c)HighpH
d)HighPCO2
CorrectAnswer-C
Ans.C.HighpH
Conditionsassociatedwithincreasedaffinityofhemoglobinfor
oxygen:
HighpH.
DecreasedH+ionconcentration(alkalosis).
ReducedPCO2
Reducedbodytemperature.
Reduced2,3-bisphosphoglycerate(2,3-BPG)/2,3-
diphosphoglycerate(DPG)
Fetalhemoglobin.
COpoisoning.

190.Incomparisontohemoglobin,effectof
myoglobinonBohreffect?
a)Increased
b)Decreased
c)Same
d)NoBohreffect
CorrectAnswer-D
Ans.D.NoBohreffect
[RefPrinciplesofmedicalphysiologyp.711]
Myoglobin:
Singlepolypeptidechain.
Humanmyoglobincontains152aminoacidswithamolecularweight
of17,500.
Hemeisattachedto92"dhistidinresidue.
Onemoleculeofmyoglobincancombinewithonemoleculeof
oxygen.
MyoglobinhashigheraffinitytooxygenthanthatofHb.
MyoglobinhashighoxygenaffinitywhileBohreffect,cooperative
effectand2,3-diphosphoglycerateeffectcanabsent.

191.FunctionofchlorideshiftinRBCs?
a)RightshiftofHb-O,curve
b)LeftshiftofHb-02curve
c)TransportofCO2
d)Diffusionof02inalveoli
CorrectAnswer-C
Ans.C.TransportofCO2
[RefGanong24th/ep.644;Principlesofmedicalphysiologyp.819]
TransportofCO2:
*Carbondioxideistransportedinbloodasplasmabicarbonate.
*Redbloodcells(RBCs)playamajorroleinthemechanism
becauseRBCscontaintheenzymecarbonicanhydrasethat
catalyzesthereactionCO2+H20=HCO3+H.
*HencewhenCO2diffusesintotheRBC,itreactschemicallywith
watertogenerateHCO3.
*TheH+ionsaremoppedupbyhemoglobin,whichisanexcellent
buffer.
*Thisenablesthereactiontoproceedintheforwarddirection.
*TheHCO3-ionsgenerateddiffuseoutintotheplasmainexchange
forClionsthatdiffuseintoRBCssimultaneously.
*ThemovementofchlorideionsintoRBCiscalledChlorideshift.
-Theaboveeventsresultsinanincreaseintotalnumberions
insidetheRBC,whichincreasesitsosmolarity.
-Asaresult,waterenterstheRBCthroughosmosis.

192.Closingvolumeisrelatedtowhichofthe
following?
a)Tidalvolume
b)Residualvolume
c)Vitalcapacity
d)None
CorrectAnswer-B
Ans.B.Residualvolume
[RefPrinciplesofmedicalphysiologyp.240,241]
Theclosingvolumeisthepointatwhichdynamiccompressionofthe
airwaysbegins,especiallyduringforcedexpiration.
Lower(dependent)partsofthelungshaslessertransmural
pressure,thereforetheybegintocloseearly.
Therefore,closingvolumeisthelungvolumeaboveresidualvolume
atwhichairwayinthelower,dependentpartsofthelungsbeginto
closeoff.
Closingcapacityisthelungvolume(includingresidualvolume)at
whichairwaysinthelower,dependentpartsofthelungsbeginto
closeoff,i.e.,Closingcapacity=closingvolume+residualvolume.
Closingcapacityandvolumearetestedforsmallairwayfunction.
Criticalclosingvolumeistheminimumvolumeandpressureofgas
necessarytopreventsmallairwaycollapse.Itissomewhatnear
residualvolume.

193.Centralchemoreceptorsarenot
stimulatedby?
a)TPCO2
b)TEl'inCSF
c)Hypoxia
d)Allstimulate
CorrectAnswer-C
Ans.C.Hypoxia
[RefGanong25th/ep.658]
Centralchemoreceptors:
Locatedinachemosensitiveareaontheventralsurfaceofthe
medullaneartheexitoftheninthandtenthcranialnerves.
Theprimarystimulusforthecentralchemoreceptorsisanincrease
inthehydrogenionconcentration.
Stimulationofcentralchemoreceptorsbyincreasedhydrogenion
concentrationleadstoexcitationoftherespiratoryneurons,thereby
producinganincreaseintherateanddepthofrespiration.
CentralchemoreceptorsaredirectlystimulatedbyanincreaseinH*
concentrationinCSFandbraininerstitialtissue,whichisbrought
aboutbychangeinarterialPCO2(PaCO2).
Notstimulatedbyhypoxia;ratherlikeanyothercells,theyare
depressedbyhypoxia.

194.Chemicalregulationofrespirationisnot
affectedby?
a)P02
b)PCO2
c)pH
d)MeanBP
CorrectAnswer-D
Ans.D.MeanBP
[RefPrinciplesofmedicalphysiology;Ganong24th/ep.662-663]
ChemicalRegulation
Centralchemoreceptors:TFPConcentrationinCSF(1,pHofCSF);
TPCO,ofblood.
Peripheralchemoreceptors:LP02,TPACO2,acidosis(4pH)

195.Trueabouthighaltitudeacclimatization?
a)Leftshift0102-Hbcurve
b)DecreasedRBCcount
c)Hypoventilation
d)Increasederythropoietin
CorrectAnswer-D
Ans.D.Increasederythropoietin
[RefGanong24th/ep.650,651&23'/ep.617-619;Guyton12ip.
529,530]
Acclimatization:
Importalcompensatorymechanismsathighaltitudeare:-
Hyperventilation:-CausesCO2washout,.1.,PCO2andrespiratory
alkalosis.
Increased2,3DPG:-Rightwardshiftof02-Hbdissociationcurve.
PolycythemiaandincreasedHb:-Duetoincreasederythropoietin
releasewhichcausesabsolutepolycythemiawithincreasedredcell
mass.
Others:-Increasedrenalexcretionofalkali(HCO3-),increased
tissuevascularity,increasedoxidativemetabolism,Increased
diffusioncapacityoflung,increasedmyoglobin.

196.Respiratoryexchangeofgasesisstrated
from?
a)Branchi
b)Alveoli
c)Bronchiole
d)Tissuelevel
CorrectAnswer-B
Ans.B.Alveoli
[RefGanong25th/ep.639-640]
Externalrespiration:
Itconsistsofexchangeofgases(02andCO2)inthealveoli.
Thereisdiffusionof02fromalveolarairintopulmonarycapillary
bloodanddiffusionofCO2intooppositedirection.

197.Distendingcapacityoflungismaximum
at?
a)Apex
b)Base
c)Midregion
d)Posteriorlobe
CorrectAnswer-A
Ans.A.Apex
[RefUnderstandingsofmedicalphysiologyp.791]
"Theairspacesattheapicesofthelungstendtoberelatively
distended,duetoweightofthelungseffectivelydraggingitself
downwards,puttingtractionontheupperpartofthelungand
stretchingtheapicalairspacesopen."

198.Actionpotentialincardiacmusclesis
duetowhichions?
a)K*
b)Na*
c)Ca'
d)CI?
CorrectAnswer-A:B:C
Ans.b>a&c
Na+,K+andCa+,allareinvolvedinfullcycleofactionpotentialin
cardiacmuscle.
But,themainphase(phase0ordepolarization)isduetoNa+ions.
Phase0(phaseofrapiddepolarization)openingoffastsodium
channelswithNa*influx.
Phase1(initialphaseofrapidrepolarization)-4closureoffast
sodiumchannels.
Phase2(plateuphase)openingofvoltagegatedslowCa**
channelswithcalciuminflux.
Phase3(finalrepolarization)openingofK*channelswithefflux.
Phase4-4Restingmembranepotential.

199.Heartsoundoccuringjustbeforeclosure
ofAV?
a)S1
b)S2
c)S3
d)S4
CorrectAnswer-D
Ans.D.S4
[RefCECILp.212]
S4occursjustbefore1stheartsound(1stheartsoundisdueto
closureofAVvalve,i.e.mitral&tricuspidvalve).

200.Baroreceptorsarerelatedtowhich
vessels?
a)Internalcarotidartery
b)Externalcarotidartery
c)Subclavianartery
d)Brachiocephalictrunk
CorrectAnswer-A
Ans.A.Internalcarotidartery
[RefGanong24thlep.590,591;Principlesofmedicalphysiology
p.791]
Baroreceptorsaremechanoreceptorsthatarelocatedinthe
adventiaofcarotidarteryandaorta,atspecializedlocationscalled
sinuses.

201.EffectofpositiveG?
a)Increasedcerebralarterialpressure
b)Increasedvenousreturn
c)Decreasedcardiacoutput
d)Increasedpressureinlowerlimb
CorrectAnswer-C
Ans.C.Decreasedcardiacoutput
[RefGanong25thiep.576,24thlep.607-608,Principlesofmedical
physiologyp.632]
EffectsofpositiveG:
Throwingofbloodinthelowerpart
Increasedlowerlimbvenouspressure
Decreasedcerebralarterialpressure
Decreased,venousreturn
Decreased,cardiacoutput
Gray-outandBlack-out.

202.Whichofthefollowingcauseincreasein
pulmonaryarterialpressure?
a)Histamine
b)Hypoxia
c)ANP
d)PGI2
CorrectAnswer-B
Ans.B.Hypoxia
[RefGanong24th/ep.637]
Hypoxiacausesvasoconstrictioninpulmonarybloodvessels
causingincreaseinpulmonaryarterialpressure.
Effectofvariousstimulationsonpulmonaryvessels:
Vasoconstriction:
alpha-adrenergic
Thromboxane-A2
AngiotensinII
LTC4,LTD4
Endothelins
Hypoxia
Hypercapnia

203.Mostimportantcerebralvasodilator?
a)H+
b)Na+
c)Cal+
d)None
CorrectAnswer-A
Ans.A.H+
[RefR.K.Marya3rdiep.156]
Themainmetabolicfactorresponsibleforthevasodilatation
associatedwithcerebralactivityistheCO2producedbythe
activatedneurons.
IncreaseinbloodPCO2alsoproducescerebralvasodilatation.
ThevasodilatoryeffectofCO2isindirectandismediatedby
formationoflocalH+whichhasadirectvasodilatoryeffecton
cerebralbloodvessels.
AfallinbloodPO,producescerebralvasodilatationandanincrease
inbloodPO,producescerebralvasoconstriction.

204.Reninsecretionisdecreasedby?
a)Sympatheticstimulation
b)Prostacycline[PGI2]
c)Naclindistaltubules
d)Hypotension
CorrectAnswer-C
Ans.C.Naclindistaltubules
[RefGanong241thlep.674,670]
Principalregulatorsofreninsecretion:
Thejuxtaglomerularcellsthemselvesarethesensorsoftheafferent
arteriolarpressure.Loweredpressurestimulatereninrelease
IncreasedNaC1indistaltubulesissensedbymaculadensaandthe
signalistransmittedtoJGcells.
ThisresultsindecreasedReninrelease.
OppositeoccurswhendecreasedNaC1isdeliveredindistaltubule,
i.e.,increasedreninrelease.
Adenosineisprobablythemediatorofsignal.
TheJGcellsareinnervatedbysympatheticfibers.Theyrelease
renininresponsetosympatheticdischarge,andbycirculating
catecholamines.
Prostacycline(PGI2)stimulatesreninsecretionthroughadirect
action.

205.Trueaboutfunctionofdistalconvoluted
tubule?
a)ReabsorbeNa*byNal(+2c1-channel
b)ReabsorbeIC'byNei('channel
c)WaterreabsorptionbyADH
d)Allarecorrect
CorrectAnswer-C
Ans.C.WaterreabsorptionbyADH
Over-allimpactofreabsorptioninearlydistaltubuleistodiluteurine
byremovingsolutes.
Latesegmentofdistaltubuleisfunctionallysimilartocortical
collectingduct.
Principal(P)cellsreabsorbsodium&waterfromlumen(ByADH&
secretepotassiumintolumen.
Intercalated(I)cellsreabsorbpotassium&secretehydrogeninto
lumen.
Cl-isreabsorbedintolatedistaltubule.

206.FunctionofLaciscellsinnephron?
a)H+secretion
b)Na+reabsorption
c)Reninsecretion
d)Regulationofvasoconstriction/vasodilatationofarterioles
CorrectAnswer-D
Ans.D.Regulationofvasoconstriction/vasodilatationof
arterioles
[RefPrinciplesofmedicalphysiologyp.412]
Juxtaglomerular/Extraglomerularmesangialcells(Laciscells)
formingconnectionviaactinandmicrotubuleswhichallowfor
selectivevasoconstriction/vasodilationoftherenalafferentand
efferentarterioleswithmesangialcellcontraction.
Note:Laciscellsalsocontainsomerenin.
ButReninismainlysecretedbyjuxtaglomerularcells.

207.Plasmainulinofapersonis4mg/mland
urineflowrateis20ml/min.Whatwillbe
GFRifurineinulinis50mg/ml?

a)125ml/min
b)250ml/min
c)500ml/min
d)1000ml/min
CorrectAnswer-B
Ans.B.250ml/min
[RefGanong23rdlep.678,679]
GFR?UrineinulinxUrineflow
GFR=?250ml/min.

208.Trueaboutaquaporinsareallexcept?
a)Protein
b)Aquaporin-1inPCT
c)Aquaporin-2inloopofHenle
d)Aquaporin-2inCD
CorrectAnswer-C
Ans.C.Aquaporin-2inloopofHenle
[RefGanong24thiep.683-690]
Aquaporinsareproteinchannelswhichhelpinrapiddiffusionof
water
ProximaltublesandthindescendinglimbofloopofHenle
aquaporin-1,whichisindependentofADH.
Latedistaltubuleandcollectingductaquaporin-2,dependenton
ADH.

209.
MaximumfatabsorptioninGItract
occursin?
a)Duodenum
b)Jejunum
c)Ileum
d)Calcium
CorrectAnswer-B
Ans.B.Jejunum
Afterfatdigestion,fattyacidsandmonoglyceridesareabsorbedin
smallintestineespeciallyinthejejunumandsomeamountalsoin
ileum.
Insidetheenterocyte,fattyacidsandmonoglyceridesagainform
triglycerides.
Thesetriglyceridesareincorporatedintochylomicronsand
transportedtolymphaticsandfromtheretobloodvessels.

210.Iftheileumisexcised,whatwillincrease
instool?
a)Bilesalts
b)Bileacids
c)Iron
d)Calcium
CorrectAnswer-A
Ans.A.Bilesalts
Bile(Majorconstituentbilesalts)isabsorbedinterminalileum.
Ironandcalciumareabsorbedinduodenum.

211.
Whichofthefollowingactsas
"Gatekeeper"intheGIT?
a)Na+-aminoacidcotransporter
b)Na+K+ATPass
c)Calciumchannel
d)Ne-glucosecotransporter
CorrectAnswer-C
Ans.C.Calciumchannel
[Refwww.ncbi.nlm.nih.gov]
Epithelialcalciumchannels(ECaCs)actas'gatekeeper'for
transepithelialCa"transport.
PrimetargetforhormonalcontrolofactiveCa(2+)fluxfromtheurine
spaceorintestinallumentothebloodcompartment.
ThisreviewcoversthedistinctivepropertiesofthesehighlyCa(2+)-
selectivechannelsandhighlightstheimplicationsforour
understandingoftheprocessoftransepithelialCa(2+)transport.

212.TrueaboutbasicrhythmofGIT?
a)Fluctuatebetween-65and-40mV
b)Initiatedbyzymogencells
c)Pacemakercellsarepresentinproximalstomach
d)Alloftheabove
CorrectAnswer-A
Ans.A.Fluctuatebetween-65and-40mV
[RefPrinciplesofmedicalphysiologyp.915]
Thesmoothmusclecellsofgastrointestinaltracthasspontaneous
rhythmicfluctuationsinmembranepotentialbetweenabout-65and
-45mV.
Thisiscalledbasicelectricalrhythm(BER).
ThisBERisinitiatedbypacemakercellscalledintersititialcellsof
Cajal.

213.Dailyfecalurobilinogenexcretionin
healthyadults?
a)20-40gm
b)40-280gm
c)20-40mg
d)40-280mg
CorrectAnswer-D
Ans.D.40-280mg
[Ref:Principlesofmedicalphysiologyp.212]
Thenormaldailyexcretionofurobilinogeninthefecesrangesfrom
40to280mg,averaging150mg.
Atotaldailyexcretionof140mgofurobilinogenwouldrepresentthe
catabolismof3.99gramsofhemoglobin.

214.Maximumdailydegradationof
hemoglobininnormaladults?
a)2gm
b)4gm
c)6gm
d)8gm
CorrectAnswer-D
Ans.D.8gm
Atotaldailyexcretionof140mgofurobilinogenwouldrepresentthe
catabolismof3.99gramsofhemoglobin.
Normalmaximumurobilinogeninfecesis280mg,i.e.8gramsof
hemoglobincatabolism.

215.cAMPactivates?
a)Proteinkinase'A'
b)Proteinkinase'C'
c)Nucleartranscription
d)Phospholipare
CorrectAnswer-A
Ans.A.Proteinkinase'A'
Hormonesactingthroughadenylylcyclase(AC):
Corticotropinreleasinghormone(CRH),FSH,LH,TSH,ACTH
(corticotropin),ADH,Vasopressin(V2receptors),Parathormone,
Catecholamine)e.g.,adrenaline(mostactions),Glucagon,hCG,
Calcitonin,Somatostatin,acetylcholine(M2),Dopamine(1),
AngiotensinII(epithelialcells),GABA-B,Histamine(H2).

216.Mechanismofactionof5-areductase?
a)BreakageofC4C5doublebond
b)BreakageofC-Nbond
c)Breakageofamidebond
d)BreakageofN-Nbond
CorrectAnswer-A
Ans.A.BreakageofC4C5doublebond
5-alphareductasecausesreduction(breakge)ofC4-05doublebond
(A4.5)withthehelpofNADHasacofactor.
Itconverts(reduces)testosteronetodihdrotestosterone.

217.Glucosetransporteraffectedindiabetes
mellitus?
a)GLUT-2
b)GLUT-5
c)GLUT-4
d)SGLT-2
CorrectAnswer-C
Ans.C.GLUT-4
[RefPrinciplesofmedicalphysiologyp.790]
GLUT-4isresponsibleforfacilitatingthetransportofglucoseintothe
cellsinresponsetoinsulin.
Forthisreason,mutationinGLUT-4havebeenassociatedwithtype
2diabetes.
TheGLUT-4geneislocatedonshortarmofchromosome17
(17p13).

218.Geneforinsulinresponsiveglucose
transporterislocatedonchromosome?
a)7
b)21
c)17
d)13
CorrectAnswer-C
Ans.C.17
[RefPrinciplesofmedicalphysiologyp.790]
GLUT-4isresponsibleforfacilitatingthetransportofglucoseintothe
cellsinresponsetoinsulin.
Forthisreason,mutationinGLUT-4havebeenassociatedwithtype
2diabetes.
TheGLUT-4geneislocatedonshortarmofchromosome17
(17p13).

219.TrueaboutACTHandcortisol
[corticosteroid]secretion?
a)Maximumsecretionintheevening
b)ACTHhasnegativefeed-backcontrol
c)ACTHhasmajoreffectonmineralocorticoidsecretion
d)ACTHisderivedfromPOMC
CorrectAnswer-B:D
Ans.B&D.ACTHhasnegativefeed-backcontrol(D)ACTHis
derivedfromPOMC
[RefUnderstandingsofmedicalphysiologyp.539]
ACTHisderivedfromprecursormoleculepro-opiomelanocortin
(POMC).
ACTHstimulatestheadrenalcortextoincreasethesynthesisand
releaseofglucocorticoids.Atnormalphysiologicalconcentration,the
effectofACTHonsecretionofmineralocorticoid(aldosterone)and
androgenisminimal.However,athigherconcentrationsynthesis
andreleaseofthesehormonecanalsoincrease.
ThesecretionofACTHissubjectedtonegativefeedback(inhibition)
byglucocorticoids.
ACTHsecretionshowsdiurnal(circadian)rhythmwithminimum
secretionateveningandmaximumsecretionatearlymorning.

220.Secretionofcortisolishighestat?
a)Mid-night
b)Earlymorning
c)Afternoon
d)Evening
CorrectAnswer-B
Ans.B.Earlymorning
[RefTextbookofclinicalendocrinologyp.78]
ThesecretionofACTHandconsequentlythatofcortisolfollowsa
circadianrhythmduetohypothalamic(Suprachiasmaticnucleus)
control.
ACTHsecretionisminimumduringnightandmaximumearlyinthe
morning(6-8am).

221.Allaretrueregardingintracellular
receptors,except?
a)Actbyregulatinggeneexpression
b)Fastestactingreceptors
c)Glucocorticoidreceptors
d)DNAcontainshormoneresponsiveelements
CorrectAnswer-B
Ans.B.Fastestactingreceptors
[RefGanong24thlep.406;Harper28th/ep.4281]
Thisisslowestactingtranductionmechanismbecauseprotein
synthesistakessometime.

222.HormonewhichaffectsIC*ion
concentration?
a)GH
b)Thyroxine
c)Insulin
d)Estrogen
CorrectAnswer-C
Ans.C.Insulin
[RefGuytonIlth/ep.710]
InsulinlowersserumIC'concentrationi.e.,causeshypokalemia.The
hypokalemicactionofinsulinisduetostimulationofK+intakeby
thecellsmainlyinmuscleandadiposetissue.Insulinincreasesthe
activityofNa+-K.ATPaseincellmembrane,sothatmoreK+is
pumpedintocells.

223.Tissuefactoractivates?
a)Intrinsicpathway
b)Contactpathway
c)Invitropathway
d)Invivopathway
CorrectAnswer-D
Ans.D.Invivopathway
[RefGanong23thiep.5.31-53.5]
Bloodcoagulationpathwaysaredividedinto:-
Intrinsicpathway(contactpathway):
Itislargelyan'invitro'pathwayandisactivatedwhenfactorXII
(Hagemanfactororcontactfactor)comesincontactwithnegatively
chargedsurface,e.g.glass,kaolinetc.
Extrinsicpathway:
Itislargelyaninvivopathwayisactivatedbytissuefactor
(thromboplastin)atthesiteoftissueinjury.

224.Tissefactoractivates?
a)Preaccelerin
b)Hagemanfacter
c)Labilefacter
d)Prothrombin
CorrectAnswer-A
Ans.A.Preaccelerin
[RefGanong23rdlep.531-535]
Preaccelerin(factorVII)isactivatedtofactorVilabytissuefactor.
TissuefactoristhecofactorforbothfactorVIIandVila.

225.

Maximumstorageofmagnesiumoccursin
whichpartofbody?
a)Adiposetissue
b)Skeletalmuscles
c)Blood
d)Bone
CorrectAnswer-D
Ans.D.Bone
[RefPrinciplesofmedicalphysiologyp.114]
Anadultcontainsapproximately25gramsofmagnesium.
About60%ofthemagnesiumispresentinbone,ofwhich30%is
exchangeableandfunctionsasareservoirtostabilizetheserum
concentration.
About20%ispresentisskeletalmuscles,19%inothersofttissues
andlessthan1%inECF.

226.

Phosphate/phosphorusispresentinwhich
partofcell?

a)Cellmembrane
b)DNA
c)RNA
d)Alloftheabove
CorrectAnswer-D
Ans.D.Alloftheabove
[RefPrinciplesofmedicalphysiology/ep.116]
PhosphorusisacomponentofDNA,RNA,ATPandalsothe
phospholipidsthatformallcellmembranes.
Nearlyeverycellularprocessthatusesenergyobtainsitintheform
ofATP.
Thus,itisanessentialelementforalllivingcellsandimportantfor
energyutilizationinthebody.

227.Exercisemediatedincreaseinmuscular
bloodflowismediatedbywhich
sympatheticfibers

a)Adrenergic
b)Noradrenergic
c)Dopaminergic
d)Cholinergic
CorrectAnswer-D
Ans.D.Cholinergic
Anexerciseisaformofstress,andlikemoststresses,is
accompaniedbysympatheticoveractivity.
Skeletalmuscleshavebothsympatheticnoradrenergic
vasoconstrictorfibersandsympatheticcholinergicvasodilatorfibers.
Vasoconstrictornervefibersactmainlyonveins.
Venoconstrictionimprovesvenousreturnandhelpsinimproving
cardiacoutput.
Ontheotherhand,sympatheticcholinergicfibers,whichareunique
toskeletalmuscles,bringaboutarteriolardilatationandthereby
increasemusclebloodflow.

228. Prematureejaculationoccursinwhich
phaseofsexualcycle?
a)Excitementphase
b)Plateuphase
c)Orgasmicphase
d)Resolution
CorrectAnswer-C
Ans.C.Orgasmicphase
[RefOxfordtextbookofpsychiatryp.227]
Orgasmphase:
Prematureejaculationoccursinthisphase
Reflexivemusclecontractionoccursinpelvis
Itistheshortestphaseandlastsonlyafewseconds

229. Erectionofpenisoccursinwhichphase
ofsexualcycle?
a)Excitementphase
b)Plateuphase
c)Orgasmicphase
d)Resolution
CorrectAnswer-A
Ans.A.Excitementphase
RefOxfordtextbookofpsychiatryp.227]
Excitementphase
ThereisincreasedphysiologicalexcitementsuchashighBPand
heartrate.
Thereiserectionofpenis(inmales)andswellingofclitoris&labia
minora(infemales).
Testesswell,scrotumtightens
Thereisvaginallubrication

230.Shortestphaseofsexualcycle?
a)Excitementphase
b)Plateuphase
c)Orgasmicphase
d)Resolution
CorrectAnswer-C
Ans.C.Orgasmicphase
[RefOxfordtextbookofpsychiatryp.227]
Orgasmphase:
Prematureejaculationoccursinthisphase
Reflexivemusclecontractionoccursinpelvis
Itistheshortestphaseandlastsonlyafewseconds

231.InhibitionofNa*ATPaseleadsto?
a)DecreasedNa*inthecell
b)IncreasedCa'inthecell
c)IncreasedK*inthecell
d)IncreasedCl-inthecell
CorrectAnswer-B
Ans.B.IncreasedCa'inthecell
[RefPrinciplesofmedicalphysiology3rdlep.786]
3Na+/1Ca+exchanger(Sodium/Calciumexchangepump):
Itmoves3Na+intothecellinexchangeoneCa+goingout

232.Hypercoagulablefactoris?
a)ProteinC
b)ProteinS
c)FactorVLeiden
d)AntithrombinIII
CorrectAnswer-C
Ans.C.FactorVLeiden
[RefTextbookofclinicalhematologyp.786]
FactorVLeidenisthemostcommoninheritedhypercoagulable
state.
Occurswhenaspecificmutationinaproteinthatismoreresistantto
beturnedoff,leadingtoanincreasedriskofthrombosis.

233.Apnea-hypopneaindexisusedfor?
a)Emphysema
b)Asthma
c)Hyalinemembranedisease
d)Obstructivesleepapnea[OSAI
CorrectAnswer-D
Ans.D.Obstructivesleepapnea[OSAI
[RefHarvarded]ApneaHypopneaIndex(AHI)
TheAHIisthenumberofapneasorhypopneasrecordedduringthe
studyperhourofsleep.
Itisgenerallyexpressedasthenumberofeventsperhour.
BasedontheAHI,theseverityofOSAisclassifiedasfollows:
None/Minimal:AHI<5perhour
Mild:AHI5,but<15perhour
Moderate:AHI15,but<30perhour
Severe:AHI30perhour.

234.Trueaboutheterophilicreceptors?
a)InvolvedinbindingofGHtocellmembrane
b)Bindtosameligand/hormone
c)Involvedincelladhesion
d)Allarecorrect
CorrectAnswer-C
Ans.C.Involvedincelladhesion
[RefTextbookofclinicalpathologyp.1132]
Cell-cellandcell-tissue-interactionsoccurthroughfollowing
typesofreceptors:
Homophilic&heterophilicreceptors.
Heterophilicreceptors:
Thesereceptorsrecognizedistinctligands(calledadhesion
epitopes)ofopposingcellmembranesortissue(called"lockandkey
bonds"or"links").

235.Thirdorderneuronsofsensationsfrom
facearisein?
a)Medulla
b)Spinalnucleusoftrigeminal
c)Thalamus
d)Brainstem
CorrectAnswer-C
Ans.C.Thalamus
Generalsensationsfromthefacearecarriedbytrigeminalnerve.
FromVPMnucleusofthalamus,thirdorderneuronsprojectto
postcentralgyrus(primarysensorycortex).

236.Somaticefferentofwhicharisefrom
medulla?
a)Oculomotor
b)Trochlear
c)Abducent
d)Hypoglossal
CorrectAnswer-D
Ans.D.Hypoglossal
Generalsomaticefferent(motor)nucleiofHypoglossal
nucleus:
Itliesinmedullaandthroughhypoglossalnervesuppliesmusclesof
tongue,exceptpalatoglossus.

237.Actiniswhichtypeofprotein?
a)Fibrous
b)Globular
c)Both
d)None
CorrectAnswer-C
Ans.C.Both
[RefPrinciplesofmedicalphysiologyp.731]
Therearetwotypesofactinfilaments:
F-actinFibrousprotein
G-actinGlobularprotein

238.NAD+Actsasacoenzymefor?
a)Xanthineoxidase
b)L-aminoacidoxidase
c)Succinatedehydrogenase
d)Malatedehydrogenase
CorrectAnswer-D
Ans.'D'Malatedehydrogenase
NAD-linkeddehydrogenasesPyruvatedehydrogenase,isocitrate
dehydrogenase,malatedehydrogenase,a-ketoglutarate
dehydrogenase,glutamatedehydrogenase,glyceraldehyde-3-P
dehydrogenase,lactatedehydrogenase,13-hydroxyacylCoA
dehydrogenase,glycerol3-Pdehydrogenase(cytoplasmic).
NADP*-linkeddehydrogenasesGlucose-6-Pdehydrogenase,6-
Phosphogluconatedehydrogenase,3-ketoacylreductase,Enoyl
reductase,gulonatedehydrogenase.
FAD-linkeddehydrogenasesSuccinatedehydrogenase,fattyacyl
CoAdehydrogenase,glycerol-3Pdehydrogenase(mitochondrial).

239.Enzymeinvolvedinthetransferof
hydrogenionis
a)Hydratase
b)Oxidase
c)Peroxidase
d)Dehydrogenase
CorrectAnswer-B:D
Ans.is'b'i.e.,Oxidase&'d'i.e.,Dehydrogenase[RefHarper
30th/ep.198;Vasudevan5th/ep.210]
Enzymeinvolvedinoxidation-reactionare:?
Causeremovalofhydrogen
Dehydrogenases:UseNADorFADasacceptor
Oxidases:Useoxgenasacceptor,Addoxygen
Oxygenases

240.Whichofthefollowingisa
constitutiveenzyme?
a)Hexokinase
b)Glucokinase
c)pgalactosidase
d)Cyclooxygenase-2
CorrectAnswer-A
Ans.is'a'i.e.,Hexokinase

241.Sequenceofcomplexesintheelectron
transportchainis-
a)NADHdehydrogenaseQCytochromebc1
Cytochromeaa3O,
b)NADHdehydrogenaseQCytochromeaa3
CytochromebclO,
c)NADHdehydrogenaseCytochromeaa3Q
CytochromebclO,
d)NADHdehydrogenaseCytochromebclQ
Cytochromeaa3O,
CorrectAnswer-A
Ans.is'a'i.e.,NADHdehydrogenaseQCytochromebcl
Cytochromeaa302
Electrontransportchainismadeupof5stationarycomplexesand2
mobilecomplexes

242.Regardingenergyproductionbythe
electrontransportchain,whichistrue?
a)Thecomplexesarearrangedinadecreasingorderofredox
potential
b)Thecomplexesarearrangedinadecreasingorderofability
togetreduced
c)Thecomplexesarearrangedinadecreasingorderofstate
ofoxidation
d)Thecomplexesarearrangedinadecreasingorderof
energylevel
CorrectAnswer-D
Ans.is'd'i.e.,Thecomplexesarearrangedinadecreasing
orderofenergylevel[RefEssentialofbiochemistryp.712]
ETChelpinATPgeneration
ItisexplainedbyMitchell'schemiosmotictheory.Accordingtothis
theory,thecomplexesarearrangedinanincreasingorderofredox
potential.Redoxpotentialisameasureofabilitytogetreduced.So
thecomplexesarearrangedinanincreasingorderofabilitytoget
reduced.Asmoreasubstanceisoxidisedhigherwillbetheabilityto
getreduced,thecomplexesarearrangedinanincreasingorderof
stateofoxidation.Asstateofoxidationisinverselyproportionalto
energylevel,thecomplexesarearrangedinadecreasingorderof
energylevel.
So,whenelectronsmovefromonecomplextoanother,itmeans
electronsmovefromacomplexofhighenergytoacomplexoflow
energyandthatliberatesenergy.Thisenergyisusedforpumping
hydrogenionsfromthemitochondrialmatrixtojustoutsidetheinner
mitochondrialmembrane.Afterhydrogenionsaccumulateoutside

theinnermitochondrialmembrane,hydrogenionsgothroughFO
ComponentofATPsynthase.

243.Electrontransportchainallaretrue
except
a)Complexesarearrangedinanincreasingorderofredox
potential;
b)MitochondrialGlycerolphosphatedehydrogenasesendsits
electrondirectlytoQ
c)l0HydrogenionsaretranslocatedwhenNADHentersinto
anelectrontransportchain
d)7HydrogenionsaretranslocatedwhenFADH2electrons
getintoelectrontransportchain.
CorrectAnswer-D
Ans.is'd'i.e.,7HydrogenionsaretranslocatedwhenFADH2
electronsgetintoelectrontransportchain
NADHelectronsgetintoelectrontransportchainthroughcomplexI.
EnergydifferencebetweenNADHandQisinsuchawaythatwhen
electronsmovefromcomplexItoQ,4hydrogenionsget
translocated.
Similarly4ionsgettranslocatedwhenelectronsmovefromQto
complexIIIand21-1'ionsgettranslocatedwhenelectronsmove
fromcomplexIIItoIV.Sototally101-1+ionsgettranslocatedwhen
electronsfromNADHgetintoelectrontransportchain.ComplexVor
ATPsynthasecomplexworksinsuchawaythatwhen10H*ionsgo
throughF0component,1ATPisgenerated.Sowhen10Hydrogen
ionsaretranslocated,2.5ATPscanbegenerated.
FADH,electronsgetintoelectrontransportchainthrougheither
complexIIortheydirectlygetintoQ,ineithercase,noenergyis
liberated.Nohydrogenionsaretranslocated.Whenelectronsmove

fromQtoComplexIII,4hydrogenionsandwhenelectronsmove
fromIIItoIV2hydrogenionsaretranslocated.Sototally6hydrogen
ionsaretranslocatedwhenFADH2getsintoelectrontransport
chain.ATPsynthasecomplexgenerates1ATPforevery4hydrogen
ionstranslocatedthroughF,component.Sofor6hydrogenions,itis
1.5ATP

244.Atractilosideactas?
a)Uncoupler
b)Inhibitorofoxidativephosphorylation
c)InhibitorofcomplexIofETC
d)InhibitorofcomplexIIIofETC
CorrectAnswer-B
Ans.is'b'i.e.,Inhibitorofoxidativephosphorylation
Inhibitorsofelectrontransportchain?
Inhibitorsofrespiratorychainmaybedividedintothreegroups:?
1.Inhibitorsofelectrontransportchainproper
Theseinhibitorsinhibittheflowofelectronsthroughtherespiratory
chain.Thisoccursatfollowingsites.
ComplexI(NADHtoCoQ)isinhibitedby:-Barbiturates
(amobarbital),PiericidinA(anantibiotic),rotenone(aninsectiside),
chlorpromazine(atranquilizer),andguanethidine(an
ntihypertensive).Theseinhibitorsblockthetransferofreducing
equivalentsfromFeSproteintoCoQ.
ComplexIIisinhibitedby:-CarboxinandTTFAinhibittransferof
electonfromFADH2toCoQ,whereasmalanatecompetitivelyinhibit
fromsuccinatetocomplexII.ComplexIII(Cytochromebto
cytochromeCI)isinhibitedby:-Dimercaprol,antimycinA,BAL
(Britishantilewisite),Naphthyloquinone.Theseinhibitorsblockthe
transferofelectronsfromcytochromebtocytochrome
ComplexIV(cytochromeCoxidase)isinhibitedby:-Carbon
monoxide,CN-,H2Sandazide(N3-).Theseinhibitorsblockthe
transferofelectronsfromcytochromeaa3tomolecularoxygenand
thereforecantotallyarrestcellularrespiration.
2.Inhibitorsofoxidativephosphorylation

ThesecompoundsdirectlyinhibitphosphorylationofADPtoATP.
OligomycininhibitsFocomponentofF0F,ATPase.Atractiloside
inhibitstranslocase,atransportproteinthattransportsADPinto
mitochondriaforphosphorylationintoATP.
3.Uncouples
Asthenamesuggests,thesecompondsblockthecoupelingof
oxidationwithphosphorylation.Thesecompoundsallowthetransfer
ofreducingequivalentsinrespiratorychainbutpreventthe
phosphorylationofADPtoATPbyuncouplingthelinkagebetween
ETCandphosphorylation.Thustheenergyinsteadofbeingtrapped
byphosphorylationisdissipatedasheat.Uncouplersmaybe:-
1. Natural:-Thermogenin,thyroxine
2. Synthetic:-2,4-dinitrophenol(2,4-DNP),2,4-dinitrocresol(2,4-
DNC),andCCCP(chlorocarbonylcyanidephenylhydrazone).

245.Whichofthefollowingisthe
respiratorycentreofcell?
a)Mitochondria
b)Microsome
c)Lysosome
d)Nucleus
CorrectAnswer-A
Ans.is'a'i.e.,Mitochondria
Asmitochondriaharbourstheelectrontransportchain.Inelectron
transportchain,theelectronsfromNADHandFADH2are
transferredthroughthevariouscomplexestofinallyOxygen.
Oxygenisthenconvertedtowater.Thiswayallfuelsgetoxidisedto
Carbondioxide.Inotherwordsinmiotochondria,Oxygenisutilised
andCarbondioxidegenerationissupported.Henceitiscalledas
therespiratorycentreofthecell.

246.Celluloseisbiochemically-
a)(1,4)Lglucose
b)?,(14)Dglucose
c)(1,4)Dglucose
d)?(1,4)Lglucose
CorrectAnswer-A
Ans.is'a'i.e.,(1,4)Lglucose
Celluloseisacomponentofcellwall.
Celluloseisastructuralhomopolysaccharidemadeupofglucose
moleculeslinkedby13(1,4)linkages.
Humanscannotdigestcellulosebecausehumandigestiveenzymes
cannotbreak43(1,4)linkagespresentinoligosaccharidesand
polysaccharides.
Thisiswhyvegetariandietisconsideredtoprovidefibretothediet.

247.DandLisomerismis-
a)Opticalisomerism
b)Functionalisomerism
c)Epimerism
d)Enantiomerism
CorrectAnswer-D
Ans.isd.i.e.,Enantiomerism
Enantiomerismisatypeofstereoisomerisminwhichtwomolecules
havethesamemolecularformulaandthesamestructuralformula
buttheydifferinspatialorientationwithrespecttoallthecarbon
atomsandtheyarenamedbasedontheorientationinthe
penultimatecarbonatom.
Inthepenultimatecarbonatom,ifOHisontherightside,itisD
form,ifOHisontheleftside,itisLform.Theothernamefor
enantiomerismisRacemism.

248.Whichamongthefollowingglucosetransporterpresentinbetacells?
a)GLUT1
b)GLUT2
c)GLUT3
d)GLUT4
CorrectAnswer-B
GLUT2istheglucosetransporterpresentinthebetacellsand
livercells
.IthasahighKmforglucose.Henceentryofglucoseis
directlyproportionaltotheglucoselevel.Itisaninsul;inindependent
transport.
GLUT3ispresentinbrainandGLUT4meditesinsulindependent
transportofglucoseintomuscleandadiposetissue.
Ref:MurrayR.K.,GrannerD.K.(2011).Chapter40.Membranes:
Structure&Function.InD.A.Bender,K.M.Botham,P.A.Weil,P.J.
Kennelly,R.K.Murray,V.W.Rodwell(Eds),Harper'sIllustrated
Biochemistry,
29e.

249.Allofthefollowingareconvertedto
cr-ketoglutarateoncatabolismexcept-
a)Glutamate
b)Histidine
c)Proline
d)Glycine
CorrectAnswer-D
Ans.is'd'i.e.,Glycine[RefHarper30th/ep.162,250/ep.166,
167]


250.Glucogenicaminoacidsgiverisetoall
ofthefollowingintermediatesofcitric
acidcycleexcept-

a)Isocitrate
b)cketoglutarate
c)SuccinylCoA
d)Fumarates
CorrectAnswer-A
Ans.is'a'i.e.,Isocitrate

251.Whatistheprecursorofprolinein
Krebscycle?
a)Oxaloacetate
b)cketoglutarate
c)SuccinylCoA
d)Fumarates
CorrectAnswer-B
Ans.is'b'i.e.,cketoglutarate[RefEssentialsofBiochemistryp.
232;Harper29`'/ep.
Prolineisanalphaaminoacidwithapyrrolidinering
ItisanonpolariminoacidwithNHasoneofitsfunctionalgroups
Itdisruptsahelix
Itisanonessentialaminoacidandissynthesizedfromanon-
essentialaminoacidglutamate
Glutamateinthepresenceofyglutamatekinasegetsconvertedto
glutamate5phosphate,whichinthepresenceofyglutamate
dehydrogenasegetsconvertedtoyglutamatesemialdehyde.y
glutamatesemialdehydespontaneouslycyclisestoformypyrroline
carboxylatewhichinthepresenceofreductaseformsproline

252.usedincitricacidcycleareallexcept-
a)NAD
b)FAD
c)NADP
d)GDP
CorrectAnswer-C
Ans.C.NADP
Enzyme
Reducingequivalent ATP
Isocitratedehydrogenase
1NADH
2.5
alphaketoglutaratedehydrogenase 1NADH
2.5
SuccinylCoA
ATP/GTP
1
Succinatedehydrogenase
FADH2
1.5
Malatedehydrogenase
NADH
2.5
total
10

253.Allofthefollowingstepsactas
sourcesofenergyincitricacidcycle
except-

a)Citratesynthase
b)Isocitratedehydrogenase
c)SuccinylThiokinase
d)SuccinateDehydrogenase
CorrectAnswer-A
Ans.is'a'i.e.,Citratesynthase

254.Trueaboutglucokinaseis-
a)Itispresentinallcells
b)Itisaconstitutiveenzyme
c)Ithasahighkm
d)Itisinhibitedbyglucose6phosphate
CorrectAnswer-C
Ans.is'c'i.e.,Ithasahighkm
S.
Property
Hexokinase
Glucokinase
No.
Liverand
1
Location
Allcells
Pancreatic(3cells
2
Affinity
High
Low
3
Km
Low
High
Inhibitionbyglucose6
4
Yes
No
phosphate
No(Constitutive
Yes(Inducible
5
InductionbyInsulin
enzyme)
Enzyme)

255.Allofthefollowingaretrueabout
lactateutilisationinliverexcept-
a)TotalnetnumberofATPformedbecauseofcori'scycleis
6
b)Cori'scycleshiftsthemetabolicburdenfrommuscletoliver
c)Cori'scyclecannotbesustainedindefinitelybecauseitis
energeticallyunfavourable
d)Cori'scycleislinkedtoglycogensynthesisinmuscle
CorrectAnswer-A
Ans.is'a'i.e.,TotalnetnumberofATPformedbecauseofcoil's
cycleis6
CORI'SCYCLE
Muscleusesamoleculeofglucosethroughanaerobicglycolysisand
gets2ATPs.Inthisprocess,glucosebecomestwomoleculesof
lactate.The2lactatemoleculesthroughcirculationreachliver.In
liver,thetwomoleculesoflactateareutilisedthrough
gluconeogenesistoformaglucosemoleculeattheexpenseof6
ATPs.Theglucoseformedinliverreachesmuscleandisutilisedfor
againanaerobicglycolysisifthemuscleisstillexercising.Incase
muscleisdonewithexercising,theglucosewhichreachesthe
musclefromliverisusedforglycogensynthesis.

256.Whichofthefollowingistrueabout
effectofinsulinandglucagonon
gluconeogenesis?

a)Insulinfavourstheformationoffructose2,6bisphosphate
b)Fructose2,6bisphosphateisaninhibitorofglycolysis
c)Insulinactsthroughakinase
d)GlucagonstimulatesPFK.2activityofthetandemenzyme
CorrectAnswer-A
Ans.is'a'i.e.,Insulinfavourstheformationoffructose2,6
bisphosphate
Glycolysisandgluconeogenesisarereversalofeachother.Hence
thetwopathwaysshouldberegulatedinsuchawaythatwhenone
pathwayisactive,theotheronehastobeinactive.Otherwisethey
willendupinfutilecycles.

257.Keyenzymeofgluconeogenesisareall
except?
a)Pyruvatecarboxylase
b)PEPcarboxykinase
c)Pyruvatekinase
d)Glucose-6-phosphatase
CorrectAnswer-C
Ans.is'c'i.e.,Pyruvatekinase
Mitochondrialpyruvatecarboxylasecatalyzesthecarboxylationof
PyruvatetoOxaloacetate,ItisanATP-requiringreaction,Biotinis
thecoenzyme.
PhosphoenolpyruvateCarboxykinase:Catalyzesthe
decarboxylationandphosphorylationofoxaloacetateto
phosphoenolpyruvate(PEPCK)(Cytosol)usingGTPasthe
phosphatedonor.
Theconversionofglucose-6-phosphatetoglucoseiscatalyzedby
glucose6-phosphatase

258.Inglycogensynthesistheactiveform
ofglucoseusedis-
a)Glucose6phosphate
b)GlucoseIphosphate
c)UDPglucose
d)UTPglucose
CorrectAnswer-C
Ans.is'c'i.e.,UDPglucose
GlycogensynthesisoccursinliverandSkeletalMuscle
UDPglucoseistheactiveformofglucosewhichgetsaddedtothe
growingglycogen
Thenumberofhighenergyphosphatesrequiredforattachinga
glucosemoleculetogrowingglycogenis3
Theratelimitingenzymeofglycogensynthesisisglycogensynthase
Glycogensynthasegetsactivatedbydephosphorylation
Glycogensynthaseattachesglucoseresiduesonebyonealonga
straightchain,linkedbya(1,4)linkages.Thiscontinuesuntil11to13
residuesareattachedinastraightchain.
Atbranchpointsinglycogen,a(1,6)linkagesshouldbeformed.

259.UDPglucoseisnotusedin?
a)Uronicacidpathway
b)Glycogensynthesis
c)Galactosemetabolism
d)HMPshunt
CorrectAnswer-D
Ans.is'd'i.e.,HMPshunt
UDP-glucoseisderivedfromglucose-6-phosphateviaglucose-l-
phosphate.
ThemajorfateofUDP-glucoseisthesynthesisofglycogen.
OtherusesofUDP-glucoseare-
1.Inuronicacid(glucoronicacid)cycletogenerateUDP
glucuronate.
2.Galactosemetabolism
3.Glycosylationofproteins,lipidsandproteoglycans.

260.Neonatalhypoglycaemiawhichdoes
notrespondtocounterregulatory
hormoneadministrationisdiagnosticof
-

a)Her'sdisease
b)Cori'sdisease
c)Anderson'sdisease
d)VonGierke'sdisease
CorrectAnswer-D
Ans.is'd'i.e.,VonGierke'sdisease
GlycogenstoragedisorderspresentingwithhypoglycaemiaareType
I(VonGierke'sdisease),TypeIII(Cori'sdiseaseorForbe's
disease),TypeVI(Her'sdisease),TypeIX(FanconiBickel
syndrome)
TheonlyGlycogenstoragedisorderwhichpresentsas
hypoglycaemianotrespondingtocounterregulatoryhormone
administrationisVonGierke'sdisease
MuscleinvolvementisnotafeatureofTypeI(VonGierke'sdisease),
TypeIV(Andersoendisease),TypeVI(Her'sdisease),TypeIX
(FanconiBickelsyndrome)
AndersendiseaseorTypeIVistheonlyglycogenstoragedisease
whichpresentswithneitherhypoglycaemianorwithmuscle
involvement.Itpresentsashepatomegalyandcirrhosis

261.Whichofthefollowingisaserine
protease
a)Chymotrypsin
b)Pepsin
c)Carboxypeptidase
d)Caspases
CorrectAnswer-A
Ans.is'a'i.e.,ChymotrypsinlRef:Chatterjee7h/ep.4j5-361
*ThetermProteaseisusedtorepresentthegroupofenzymesthat
catalyzethecleavageofpeptidebondsinproteinsandpeptide
moleculeswiththeparticipationofwaterasco-reactant.In
simplewords,proteasescatalyzethecleavageofpeptide
bondsbyhydrolysis(additionofwater
Serineproteases:-
*Thesepossesacriticalserineresidueattheactivesite.
-Exampleofserineproteasesaretrypsin,chymotrypsin,elastase,
andthrombin.
*Serineproteasesareinhibitedbydisopropylphosphofluridatewhich
bindscovalentlytoserineresidue.
-Theactivesiteofserineproteasescontainthreecritical
aminoacidsserine,histidineandaspartate.Theseresidues
areoftenreferredtoascatalytictriad

262.Essentialfattyacidsareexcept:
a)Arachidonicacid
b)Linoleicacid
c)Palmiticacid
d)Linolenicacid
CorrectAnswer-A
Theessentialfattyacidsarepolyunsaturatedfattyacids,linoleic
acid(18:26)andlinolenicacid(18:33).

Arachidonicacid(20:46)isderivedfromdietarylinoleicacidandis
presentprimarilyinmembranephospholipids.
Importantderivativesoflinolenicacidareeicosapentaenoicacid
(20:63)anddocosahexaenoicacid(DHA,22:63)foundinhuman
milkandbrainlipids.Palmiticacidisacommonsaturatedfattyacid.
Arachidonicacid(20C:ro6)isnotnutritionallyessentialbecause
chainelongasesystemcanconvertlinoleicacid(l8C:<o6)into
Arachidonicacid(20C:o6).Soarachidonicacidisconsideredas
conditionallyessential,becauseithastobesupplied
inthedietiflinoleicacidisnotsupplemented.
Ref:BothamK.M.,MayesP.A.(2011).Chapter23.Biosynthesisof
FattyAcids&Eicosanoids.InD.A.Bender,K.M.Botham,P.A.Weil,
P.J.Kennelly,R.K.Murray,V.W.Rodwell(Eds),Harper'sIllustrated
Biochemistry
,29e.

263.Whichofthefollowingisatransfatty
acid?
a)Oleicacid
b)Elaidicacid
c)Stearicacid
d)Arachidonicacid
CorrectAnswer-B
Ans.is'b'i.e.,Elaidicacid[RefHarper30"/ep.213]
Therearetwotypesoffattyacids:
Saturated
Unsaturated
Cis-transisomerismisforunsaturatedfattyacids.
Stearicacidisasaturatedfattyacid(Nocis-transisomerism)
Onlyimportantunsaturatedtrans-fattyacidisElaidicacid(trans-9-
octadecenoic)

264.ActivatorsofAcetylCoAcarboxylase
areallexcept
a)AcylcoA
b)Citrate
c)Glutamate
d)Dicarboxylicacid
CorrectAnswer-A
Ans.is'a'i.e.,AcylCoA[RefHarper29thlep.217,220]
AllostericmodulationofacetylCoAcarboxylase

1. Activaters:Citrate(tricarboxylicacid);glutamate(dicarboxylicamino
acid)&otherdicarboxylicacids;ATP
2. Inhibitors:AcylCoA

265.Arachidonicacidoxidationinvolves
howmanycyclesofbetaoxidation?
a)l0
b)20
c)9
d)8
CorrectAnswer-C
Ans.is'c'i.e.,9
NumberofacetylCoAformedin(3-oxidationoffattyacids=Number
ofcarbonatoms/2.
Numberofcyclesof(beta-oxidationrequired-Noofcarbon/2(-1)
Hencearachidonicacidwith20carbonatomsundergoespoxidation
toform10acetylCoAbygoingthrough(20/2)-1cyclesi.e.,9
cycles.

266.Allaretrueaboutbetaoxidationof
fattyacidsexcept-
a)CarnitineacyltransferaseIistheratelimitingenzymeof
fattyacidoxidation
b)CarnitineacyltransferaseIisstimulatedbyAcylCoA
c)CarnitineAcyltransferaseIisstimulatedbymalonylCoA
d)CarnitineAcyltransferaseIdefectcausesadecreasein
acylcarnitinelevels
CorrectAnswer-C
Ans.is'c'i.e.,CarnitineAcyltransferaseIisstimulatedby
malonylCoA
AsCATIistheratelimitingenzymeoffattyacidoxidation,its
substrateisacylCoA.Asweknowthatallenzymesgetstimulated
bytheirsubstrates,acylCoAstimulatesCATI.
MalonylCoAisaproductofacetylCoAcarboxylase(ACC).ACCis
theratelimitingenzymeoffattyacidsynthesis,ananabolicpathway.
So,malonylCoAisanintermediateofanabolicpathway.Henceitis
asignalofhighenergy(Anabolismhappensonlyinhighenergy
status).Whentheenergyisalreadyhigh,wedonotwantfattyacids
tobefurtheroxidised.Wewantfattyacidstobeonlystored.So,
malonylCoA,asignalofhighenergyinhibitsCATI.
Inshort,anythingwhichsignalslowenergystimulatesfattyacid
oxidation(CATI).So,ADP,NAD,FAD,Glucagon,AcylCoA
stimulatefattyacidoxidation
Anythingwhichsignalhighenergyinhibitfattyacidoxidation.So,
ATP,NADH,FADH2,InsulinandmalonylCoAinhibitfattyacid
oxidation.
Fattyacidoxidationdefectspresentasnonketotichypoglycaemia,

hyperammonemia,dicarboxylicaciduria.Andallfattyacidoxidation
defectsalsopresentwithincreaseinacylcarnitinelevels.One
exceptionisCATIdefect.InCATIdefect,asacylCoAisnot
convertedtoacylCarnitine,freecarnitinelevelsarehighandacyl
carnitinelevelsarelow.

267.AIIaretrueaboutbetaoxidationoffats
acidsexcept-
a)Occursinmitochondria
b)Occursinperoxisome
c)Resultsinhydrogenperoxidegeneration
d)Fattyacidoxidationdefectspresentwithketosis
CorrectAnswer-D
Ans.is'd'i.e.,Fattyacidoxidationdefectspresentwithketosis
Fattyacidoxidationdefectcausesnon-ketotichypoglycemia.
Fattyacidoxidationhappensinmitochondriaandinperoxisomes.
Thedifferencebetweenmitochondrialoxidationandperoxisomal
oxidationisthatinmitochondria,whenthe13carbonatomis
oxidised,thehydrogenatomsareremoved.Thosehydrogenatoms
areusedforreducingNADandFADtoformNADHandFADH2.
NADHandFADH2enterintoelectrontransportchaintoformATP.
Inperoxisome,thehydrogenatomremovedfrom(3carbonatomis
usedtoreduce02formingF1202.Onlybecause11202isformedin
peroxisomeby(betaoxidationtheorganismiscalledso)

268.Whichofthefollowingistrueabout
Betaoxidationoffattyacids?
a)Stearicacidonoxidationprovides106ATPs
b)OddchainfattyacidoxidationprovidesonlypropionylcoA
c)Fattyacidoxidationdefectscausehypoglycemia
d)Ketonebodiesareformedbyincompleteoxidationoffatty
acidduringstarvationtoincreaseenergyproduction
CorrectAnswer-C
Ans.is'c'i.e.,Fattyacidoxidationdefectscause
hypoglycaemia
Defectinfattyacidoxidationcauseshypoglycemia.
Aboutotheroptions
Stearicacidoxidationproduces122ATPs.
Evenchainfattyacidsare(3-oxidizedtoacetylCoA.Oddchainfatty
acidsarealso(3-oxidizednormallybutthelaststepproducesa3-
carbonpropionylCoAalongwithanacetylCoA(insteadof2
moleculesacetylCoAthatoccursinevenchainfattyacids).
Ketonebodyformation(ketogenesis)occurswhenthereisahigh
rateoffattyacidoxidationinliverwhichprovides
excessiveacetylCoA,substrateforketogenesis.
Whenketonebodiesareformed,aswecan'texpectthe10ATPs
whichwegetfromeveryacetylCoAthroughcitricacidcycle,the
formulaforenergeticsofincompleteoxidationoffattyacidsis:

269.Whichofthefollowingistrueabout
propertiesofVLDL/LDL-
a)Inelectrophoresis,VLDLmigratesmorecathodalthanLDL
b)LDLisformedfromliver
c)LDLisformedfromChylomicron
d)VLDLremnantsreachextrahepatictissues
CorrectAnswer-B
Ans.is'b'i.e.,LDLisformedfromliver
Lipoproteinelectrophoresisofafastingsampleshowsthreebands-
HDL,VLDL,LDLinthatorderfromanodetocathode.
VLDLissynthesizedinliverthatcontainshightriglyceride,ChE,
cholesterol,phospholipidandApoB-100.(VLDLparticlesresemble
chylomicronesincompositionexceptthatVLDLcontainsApoB-100
insteadofApoB-48).
VLDLparticlesaresecretedintheplasmaandaswithchylomicron,
ApoEandApoCaretransferredfromHDLtoVLDL.NowVLDL
containsApoB-100,ApoEandApoC.
Inplasma,triglyceridesofVLDLarehydrolysedbysamelipoprotein
lipase(seeabove)andapoCistransferredtoHDLandthe
remmantsarecalledIDL.
40-60%ofIDLisremovedbyliverviaLDLreceptormediated
endocytosis,thisprocessrequireApoEwhichactsasligandforLDL
receptors.
RemainingIDLisremodeledbyhepatic(liver)lipasewhich
hydrolyzesmoretriglyceridetoformLDLthatcontainsmaximum
cholesterol.
70%ofLDLisremovedbyliverviaLDLreceptorand30%isutilized
byperipheraltissuesasasourceofcholesterol.


270.Reversecholesteroltransport-allare
trueexcept-
a)Transportofcholesterolfromextrahepatictissuestoliver
b)ATPBindingCassetteTransporterproteinisinvolvedinthe
conversionofHDL3toHDL2
c)LecithinCholesterolAcylTransferasehelpsinthe
conversionofSpheroidalHDLtoDiscoidalHDL
d)CholesterolEsterTransferProteinhelpsinincreasingHDL
level
CorrectAnswer-D
Ans.is'd'i.e.,CholesterolEsterTransferProteinhelpsin
increasingHDLlevel
ReverseCholesterolTransportisthetransportofCholesterolester
andphospholipidfromextrahepatictissuestoliver.
HDLisreleasedbybothliverandintestinalcells.
Inboththecases,theyarereleasedasdiscoidalHDL
ApoAlactivatesLecithinCholesterolAcylTransferaseandit
convertsdiscoidalHDLtoSpheroidalHDL(HDL3)
HDL3activatesABC1(ATPBindingCasetteTransporter1)tocollect
cholesterolandphospholipidsfromextrahepatictissuemembranes.
ThiswayHDL3sizeincreasesanddensitydecreases.Henceit
formsHDL2.
ThisHDL2reacheslivertoemptyitscontentsintoliver.
Onthewaytoliver,ifHDL2encountersIDL,CholesterolEster
TransferProtein(CETP)transfersChoelsterolesterfromHDL2to
IDL,convertingIDLtoLDL.HenceCETPdecreasesHDLleveland
increasesLDLlevel.

271.Progesteronesynthesisrequires-
a)LDL
b)VLDL
c)HDL
d)Chylomicron
CorrectAnswer-C
Ans.is'c'i.e.,HDL
GranulosacellsusefollicularfluidHDLasasourceofcholesterolfor
thesynthesisofprogesterone.
UndertheinfluenceofLH,corpuslutealcellstakeupcholesterol
fromfollicularfluidHDLandconvertcholesterolintoprogesterone.

272.Siteofsmallchainfattyacid
absorptionis-
a)Ileum
b)Duodenum
c)Ascendingcolon
d)Rectum
CorrectAnswer-C
Ans.is'c'i.e.,Ascendingcolon
Shortchainfattyacids(SCFA)arefattyacidswith2to6carbon
atoms.Theyarethemajorend-productsofthemicrobialdigestionof
carbohydratesinthealimentarycanal.Theseshortchainfattyacids,
butyrateparticularlyisimportantforcolonhealthbecauseitisthe
primaryenergysourceforcoloniccellsandhasanti-carcinogenicas
wellasanti-inflammatorypropertiesthatareimportantforkeeping
coloncellshealthy.Butyrateinhibitsthegrowthandproliferationof
tumorcelllinesinvitro,inducesdifferentiationoftumorcells,
producingaphenotypesimilartothatofthenormalmaturecell,and
inducesapoptosisorprogrammedcelldeathofhumancolorectal
cancercells
Thehighestconcentrationsareobservedinthelargeintestine
(caecumandcolon)ofallthemammals.
Humancaecumandproximalcolonhavehighluminal
concentrationsoforganicnutrients(non-starchpolysaccharidesfrom
plantcellwalls,andproteinsnotabsorbedbythesmallintestine)
whichmaintainhighbacterialgrowthrates.Againstthisfermentative
background,antiperistalsisensuresretentionandthoroughmixingof
faecesintheproximalcolon,whichisthesiteofmaximalSCFA
production.SCFAabsorptionisconcentrationdependentandoccurs

mostreadilyintheproximalcolon(Includescecum,ascendingcolon
andtransversecolon).

273.HMGCoAisprecursorofallexcept-
a)Ubiquinone
b)Dolichol
c)Bilepigments
d)Ketonebody
CorrectAnswer-C
Ans.is'c'i.e.,Bilepigments
3-Hydroxy3methylglutarylCoAorHMGCoAisformedfrom
acetylCoA.

274.Refsum'sdiseaseisduetodeficiencyof
whichofthefollowingenzyme?
a)Malonatedehydrogease
b)Thiophorase
c)Succinatethiokinase
d)Phytanicalphaoxidase
CorrectAnswer-D
Di.e.Phytanicalphaoxidase
Refsum'sdiseaseisarareautosomalrecessivedisordercausedby
deficiencyofphytanicaoxides(Nelson)/a?hydroxylase(Lippincot)/
PhytanoylCoAhydroxylase(Lehninger)resultsinaccumulationof
phytanicacidduetoitsdecreaseda-oxidation(i.e.hydroxnlationat
acarbon
byfattyacidahydroxylase)

275.Whatistheparameterthatisusedto
assesslipidperoxidation?
a)Malondialdehyde
b)CRP
c)hsCRP
d)Carboxymethyllysine
CorrectAnswer-A
Ans.is'a'i.e.,Malondialdehyde
ROScanbeproducedbyeitherbreakageofcovalentbond,addition
ofelectronstoamoleculeorremovalofhydrogenbyotherradicals.
Theyaregenerallyhighlyreactivespeciesandtypicallyactas
electrophilicspeciesoroxidantagents.Themostimportantradicals
orpro-oxidantmoleculesinvolvedindiseaseprocessesare
superoxide(02-),hydroxylradical(OH),hydrogen
peroxide(H202)andcertainoxidesofnitrogen,likenitricoxide(NO)
andperoxynitrite(ONOO-)2
Sinceitiscomplexmeasuringfreeradicalsdirectlyinvivo,itis
necessarytocarryoutthequantificationofcellularcomponents
whichcanreactwiththesefreeradicals,suchasproteins,DNAand
mainlylipids.Oncelipidperoxidesareunstablecompounds,they
tendtodegraderapidlyinavarietyofsubproducts.MDA
(Malondialdehyde)isoneofthemostknownsecondaryproductsof
lipidperoxidation,anditcanbeusedasamarkerofcellmembrane
injury.
MDAisathree-carbon,low-molecularweightaldehydeformedby
cyclizationofaldehydeswhichhaveunsaturationinaor13positions
SeveralmethodshavebeendevelopedtoassessMDA,including

quantitativemethodsusingspectrophotometryorfluorimetric
detection,highperformanceliquidchromatography(HPLC),gas
chromatographyandimmunologicaltechniques
Othermarkersofoxidativestressincludeconjugateddienes,ethane
andpentanegases,isoprostanesand4-HNE(4-hydroxy2-
nonenal)

276.Allaretrueaboutketonebodiesexcept?
a)Acetoacetateisprimaryketonebody
b)Synthesizedinmitochondria
c)Synthesizedinliver
d)HMGCoAreductaseistherate-limitingenzyme
CorrectAnswer-D
Ans.is'd'i.e.,HMGCoAreductaseistherate-limitingenzyme

277.AllarefeaturesofAbetalipoproteinemia,EXCEPT:
a)Plasmalevelsofcholesterolandtriglycerideareextremelylow
b)Manifestinearlychildhoodwithdiarrhea
c)Progressivepigmentedretinopathyseen
d)Neurologicalmanifestationasataxiainfirstdecade
CorrectAnswer-D
Plasmalevelsofcholesterolandtriglycerideareextremelylowin
thisdisorder,andchylomicrons.Abetalipoproteinemiausually
presentsinearlychildhoodwithdiarrheaandfailuretothrive.
Theneurologicalmanifestationslikedecreaseddistallower
extremityvibratoryandproprioceptivesense,dysmetria,ataxia,and
thedevelopmentofaspasticgait,oftenbythethirdorfourth
decade.
Patientsalsodevelopaprogressivepigmentedretinopathy
presentingwithdecreasednightandcolorvision.
Ref:HarrisonsPrinciplesofInternalMedicine,18thEdition,Page
3153

278.Aminoacidinsynthesisof
neurotransmitter
a)Glutamate
b)Proline
c)Cysteine
d)Alanine
CorrectAnswer-A
Ans.is'a'i.e.,Glutamate
Aminoacid
Neurotransmitter
Glutamate
Glutamate&GABA
Glycine
Glycine
Phenylalanine&Tyrosine Dopamine,Norepinephrine&Epinephrine
Tryptophan
Serotonin

279.Maximumbufferingcapacityofabuffer
ismaximumatpH
a)Lessthanpka
b)Morethanpka
c)Equaltopka
d)Hasnorelationwithpka
CorrectAnswer-C
Ans.is'c'i.e.,Equaltopka[RefHarper30th/ep.21&29'11ep.
20,21,18;Vasudevan6thie
p.22,23]
MaximalbufferingcapacityoccursatpHequaltopkaofbuffer.
Therefore,toworkasabestbufferatphysiologicalpHaminoacid
shouldhavepkavalueclosetophysiologicalpH(7.4).
Aminoacidscanhavebufferingactionduetothreeionizablegroups
:-
a-carboxylgroup:-Differentaminoacidshavepkavalueofa-
carboxylgroupbetween3.5-4.So,carboxylgroupofaminoacids
hasmaximumbufferingcapacitybetweenpH3.5-4.
a-aminogroup:-Differentaminoacidshavepkavalueofa-amino
groupbetween8.0-9.0.Thus,a-aminogrouphasmaximum
bufferingcapacitybetweenpH8.0-9.0.
Specialionizablegroup(insomeaminoacids):-Amongspecial
ionizablegroupofaminoacids,imidazolegroupofhistidinehaspka
value6.5-7.4,whichisclosesttophysiologicalpH.Hence,histidine
(duetoimidazolegroup)hasmaximumbufferingcapacityat
physiologialpH.

280.Essentialamino-aciddeficiencyaffect
nitrogenbalanceby
a)Increasingproteindegradation
b)Decreasingproteindegradation
c)Decreasingproteinsynthesis
d)Increasingproteinsynthesis
CorrectAnswer-C
Ans.is'c'i.e.,Decreasingproteinsynthesis[RefPrinciplesof
medicalBiochemistryp.464]
Nitrogenbalanceisthedifferencebetweeningestednitrogenand
excretednitrogen.
Nitrogenbalance=Ningested-Nexcreted
Becausedietaryproteinsareanimportantsourceofnitrogen,
nitrogenbalanceisanimportantindexofproteinandaminoacid
metabolism.
Inhealthyadults,nitrogenbalanceiszero,i.e.astateofnitrogen
equlibriumexists,wherenitrogenintakeisequaltonitrogen
excretion.
Negativenitrogenbalance(excretionexceedsintake)indietary
proteindeficiency
Inadult,evenofproteinstarved,atleast30-40gmofaminoacids
aredegradedeachday;thisamountdefinestheminimumdietary
requirment.Ifdietarysupplydropsbelowthislimit,anegative
nitrogenbalanceoccursandthebodyproteinislost.Essential
aminoaciddeficiencyhasthesameeffectbecauserelative
deficiency.

281.Creatinineisformedfrom:
a)Arginine
b)Lysine
c)Leucine
d)Histamine
CorrectAnswer-A
Glycine,arginineandmethionineallparticipateincreatine
biosynthesis


282.Acidicaminoacidsare-
a)Asparagine
b)Arginine
c)None
d)Lysine
CorrectAnswer-A
Prolineisauniqueaminoacidandhasanimminogroup(=NH)
insteadofanamino(NH2)groupfoundinotheraminoacid.
Methionine&Cysteinearesulfurcontainingaminoacids.
Thepropertyofphotochromicity(i.e.absorbanceofultravioletlightat
250-290nmesp280nm)isseenwitharomaticaminoacid
(tryptophan>tyrosine>phenylalanine).
Hydrophobic(nonpolar)aminoacidshavenochargeontheirR
grouporsidechain.Aliphatic(egmethyl,methylene,thioether&
imino)sidechainsandaromaticsidechainsarenonpolar.Somethyl
(CH3)sidechainofalanine;propyl(C3H7)sidechainofvaline;butyl
(C4H9)sidechainofleuicine&isoleucine;thioethersidechainof
methionine;andiminogroup/pyrrolidinecontainingsidechainof
proline
arenonpolar.
Methyl(CH3)sidechainofalanineisnonpolar.Serine,threonine,
tyrosinecontaininghydroxylgroupandcysteinecontainingsulfhydryl
group,arepolaraminoacidswithneutral/uncharged/nonionicside
chain.Positivelychargedbasicamino(NH +
3 )groupsidechainof
histidine,arginineandlysine;andnegativelyacidiccarboxyl(COO-)
sidechainofasparticacidandglutamicacidispolar.

283.Functionoftyrosinaseis
a)Synthesisofnorepinephrine
b)Synthesisofdopamine
c)Synthesisofmelanin
d)Alloftheabove
CorrectAnswer-C
Ans.is'c'i.e.,Synthesisofmelanin[RefHarper29th/ep.288-
290]
Note-Tyrosinehydroxylaseandtyrosinasecatalyzestheconversion
oftyrosinetoDopa,butbotharedefferentenzymes.Tyrosine
hydroxylaseisinvolvedincatecholaminessynthesisinadrenal
medullaandsympatheticganglia,whereastyrosinaseisinvolvedin
melaninsynthesisinmelanoblastsofskin.

284.Cofactorfordopaminehydroxylase?
a)Fe
b)Mg
c)Mn
d)Cu
CorrectAnswer-D
Dopamine8-hydroxylaseisa'copper'containingmonooxygenase
thatrequiresascorbicacidandmolecularoxygen.
Itcatalyzestheformationofnorepinephrine.

285.Ratelimitingenzymeincatecholamine
synthesis?
a)Dopadecarboxylase
b)N-methyltransferase
c)Dopaminehydroxylase
d)Tyrosinehydroxylase
CorrectAnswer-D
Ans.is'd'i.e.,Tyrosinehydroxylase

286.Enzymewhichactsonaspartate
a)SerumGlutamatePyruvateTransaminase(SGPT)
b)SerumGlutamateOxaloacetateTransaminase(SGOT)
c)Ornithinetranscarbmaylase(OTC)
d)Argininosuccinatelyase(ASL)
CorrectAnswer-B
Ans.is'b'i.e.,SerumGlutamateOxaloacetateTransaminase
(SGOT)
SGPTcatalaysesthetransaminationbetweenAlanineanda
Ketoglutarate.
Alanine+aKetoglutaratePyruvate+Glutamate
Itisinnowayrelatedtoaspartate.
SGOTcatalysesthetransaminationbetweenAspartateanda
Ketoglutarate

287.Trueaboutglutamatedehydrogenaseis
A/E
a)Livermitochondrialenzyme
b)UsebothNAD*orNADP+coenzyme
c)InhibitedbyADP&activatedbyGTP
d)Reversibleoxidativedeamination
CorrectAnswer-C
Ci.e.InhibitedbyADP&activatedbyGTP
Duringfirstfewdaysoffasting,thereisrapidbreakdownofmuscle
protein,providingaminoacids(alanine&glutaminemainly)thatare
usedbyliverforgluconeognesisQ.
Inthefastingstate,theoutputofalaninefromskeletonmuscleisin
farexcessofitsconcentrationinthemuscleproteinsthatarebeing
catabolized.Becauseitisalsoformedbytransaminationofpyruvate
producedbyglycolysisofmuscleglycogen.Alanineisexportedto
theliver,whereitistransaminatedbacktopyruvate,whichserves
asasubstrateforgluconeogenesisQ.


288.Allostericstimulatorofglutamate
dehydrogenaseis
a)ATP
b)GTP
c)PalmitoylCoA
d)Leucine
CorrectAnswer-D
Ans.is'd'i.e.,Leucine
GLDHisallostericallystimulatedbyADP,GDP,leucine,valineand
isoleucine.
ItisinhibitedbyATP,GTP,palmitoylCoAandZinc.

289.Whichofthefollowingisabiologically
importanttripeptide?
a)Thyrotropinreleasinghormone
b)Thyroidstimulatinghormone
c)Gonadotropinreleasinghormone
d)FollicleStimulatinghormone
CorrectAnswer-A
Ans.is'a'i.e.,Thyrotropinreleasinghormone
Tripeptideisapeptidewith3aminoacidsand2peptidelinkages.
Biologicallyimportanttripeptidesincludeglutathione,TRH
(ThyrotropinReleasingHormone)andmelanostatin.
TSHisaglycoproteinnotapeptide.Itismadeupoftwosubunits-a
and(Iasubunitisapolypeptidewithabout92aminoacids.Itisa
structuralanalogueofasubunitofFSH.LHandHCG.Bsubunitisa
polypeptidemadeupof118aminoacids.
GnRH(GonadotropinreleasingHormone)isadecapeptidewith10
aminocids.
FSHisaglycoproteinwithtwosubunits-aandp.asubunitisa
polypeptidewithabout96aminoacids.Itisastructuralanalogueofa
subunitofTSH.LHandHCG.psubunitisapolypeptidemadeupof
111aminoacids.

290.CarbamoylPhosphatesynthetaseI
[CPSI]trueis
a)Itispresentincytoplasm
b)Itisinvolvedinpyrimidinesynthesis
c)N-AcetylGlutamateisanallostericstimulatorofCPSI
d)GlutamineistheaminogroupdonorforCPSI
CorrectAnswer-C
Ans.is'c'i.e.,N-AcetylGlutamateisanallostericstimulatorof
CPSI

PROPERTY CPS-I
CPS-II
Pathway
Ureacycle
Pyrimidinesynthesis
Subcellular Mitochondria
Cytoplasm
location
Amino
Group
Ammonia
Glutamine
donor
Inhibitedbytheproducts-
Allosteric
StimulatedbyN-
pyrimidinenucleotides,Uridine,Cytidine
regulation
AcetylGlutamate(NAG) &Thymidine

291.CarbamoylphosphatesynthetaseIis:
a)Lysosolicenzyme
b)Cytosolicenzyme
c)Mitochondrialenzyme
d)Alloftheabove
CorrectAnswer-C
MitochondrialcarbamoylphosphatesynthetaseIisanenzymethat
catalyzesareactionthatproducescarbamoylphosphate.
ThisenzymecatalyzesthereactionofATPandbicarbonateto
producecarbonylphosphateandADP.Carbonylphosphatereacts
withammoniatogivecarbamate
CytosoliccarbamoylphosphatesynthetaseIIusesglutaminerather
thanammoniaasthenitrogendonorandfunctionsinpyrimidine
synthesis.

292.HHHsyndromeisduetodefectin?
a)Tryptophanmetabolism
b)Histidinetransporter
c)BranchedchainAAmetabolism
d)Ornithinetransporter
CorrectAnswer-D
Ans.is'd'i.e.,Ornithinetransporter[RefTextbookofclinical
paediatrics
p.496]
Hyperornithinaemia,hyperammonaemia,homocitrullinuria(HHH)
syndromeisanautosomalrecessivedisorderofornithinetransport
causedbymutationsingeneSLC25A15encodingtheornithine
transorterprotein(ORNT1).
Thereisdefectiveactivityoftheornithinetransporteracrossthe
mitochondrialmembrane,whichcausesafunctionaldeficiencyof
twomitochondrialenzymes:
Ornithinetranscarbomylase:Whichcatalysesthecondensationof
ornithineandcarbamoylphosphatetocitrulline.
Ornithine-8-aminotransferase(OAT):Whichmetabolizesthe
ornithineto.'-pyrroline-5-carboxylateandultimatelyglutamateand
proline.
Ornithineaccumulatesinthecytoplasmanditsdeficiencyin
mitochondriacausesasecondaryureacycledisorderand
hyperammonemia.
Carbamoylphosphateaccumulatesandundergoesallternate
metabolismtoform:

1. Homocitrulline-Excretedinurine
2. Oroticacid
3. Plasma


293.Allofthefollowingcandetermineprotein
structureexcept
a)Edman'sSequencing
b)Xraycrystallography
c)Opticalrotatorydispersion
d)Spectrophotometry
CorrectAnswer-D
Ans.is'd'i.e.,Spectrophotometry
Methodsusedforstudyingprimarystructure:
A)Sanger'ssequencing
Sanger'sreagentis(1fluro2,4Dinitrobenzene)
B)Edman'sSequencing
Edman'sreagentisPhenylisothiocyanate
C)ReverseSequencing
IthastobesupplementedbyMassSpectrometry
Methodsusedforstudyingsecondarystructure:
1. OpticalRotatoryDispersion
2. OcularDichorism
Methodsusedforstudyingtertiarystructure:
1. X-rayCrystallography
2. UVspectroscopy
3. NMRspectroscopy

294.Edman'sreagentisusedfor
a)DNAsequencing
b)Proteinsequencing
c)ProteinDenaturation
d)DNAdenaturation
CorrectAnswer-B
Ans.is'b'i.e.,Proteinsequencing
Edman'sreagentisPhenylisothiocyanate.
Phenylisothiocyanateisusedforsequencingproteins
Phenylisothiocyanatebindstoaaminogroups.Inaprotein,only
aminoterminalaminoacid'saaminogroupwillbefree.
HencewhenPhenylisothiocyanateisaddedtoapeptidewhichis
adsorbedontoaglassfibrecoatedwithapolymer,inthepresence
of12%trimethylamine,itreactswiththeaminegroupofNterminal
aminoacid.
Byacidhydrolysis,thefirstaminoacidiscleavedfromthe
polypeptidechainandtheaminoacidisidentifiedby
chromatography.
Thecycleiscontinued.Thisway50aminoacidscanbesequenced.

295.Mostabundantaminoacidinbrainis
a)Glutamate
b)Aspartate
c)Glutamine
d)Asparagine
CorrectAnswer-A
Ans.is'a'i.e.,Glutamate
Glutamateisthemostabundantfreealphaaminoacidfoundin
Brain.
Itisanacidicpolaraminoacid.
Itisthepredominantexcitatoryneurotransmitterofbrain.
ItissynthesisedinbrainfromglutamineandaKetoglutarate.
Glutamateisreleasedfrompresynapticexcitatoryneuronsina
calciumdependentmanner.
Glutamateactsonbothinotropicandmetabotropicreceptors.
Inotropicreceptorsofglutamateinclude:
Kainatereceptors
AMPAreceptors
NMDAreceptors

296.Cystinehashowmanymoleculesof
cysteine?
a)1
b)2
c)3
d)4
CorrectAnswer-B
Ans.is'b'i.e.,2
Cysteineisasulphurcontainingaminoacid
Itisapolarbutunchargedaminoacid
Cysteinewithasulfhydrylgroupcangetoxidisedandformsadimer
(2)calledascystine.
Cysteineacquiresitspolarnatureonlybyvirtueofitssulfhydryl
group.

297.Vitamingiveninhomocysteinuriaareall
except
a)VitaminB6
b)VitaminB12
c)Folate
d)Thiamine
CorrectAnswer-D
Ans.is'd'i.e.,Thiamine
Majorfateofhomocysteineisthatitgetsconvertedintocysteinein
thepresenceofcystathionineIsynthase.Cystathioninepsynthase
isdependentonpyridoxalphosphate.
Hencedefectofcystathioninepsynthasecanresultin
homocysteinuria.Thisiscalledasclassicalhomocysteinuria.This
conditionrespondstoB6administration,astheenzymecystathione
psynthaseisdependentonB6.
Minorfateofhomocysteineisthatitgetsconvertedintomethionine
inthepresenceofmethioninesynthase.Methioninesynthaseis
dependentonmethylcobalamine(coenzymeformofVitamin
B12).
MethylgroupdonorformethylcobalamineismethylTHFA.
Homocysteinuriaisalsocausedbydefectofmethioninesynthase.
AsthisenzymeisdependentonVitaminB12andTHFA,
homocysteinuriarespondstoB12andTHFAadministration

298.Glutathioneisusedtodetoxifywhich
freeradical?
a)Hydrogenperoxide
b)Superoxide
c)Peroxylradical
d)SingletOxygen
CorrectAnswer-A
Ans.is'a'i.e.,Hydrogenperoxide
Glutathioneisatripeptide.
Itisgammaglutamylcysteinylglycine
ItisdenotedasGSH-becauseithascysteinewithasulphydryl
group
Itisusedtodetoxifyhydrogenperoxideandlipidperoxidesinthe
presenceofglutathioneperoxidase.
Glutathionecanalsodetoxifyperoxylradical.Peroxylradicalcannot
getreducedbyenzymaticreactions.Theygetdetoxifiedby
antioxidantslikevitaminEandGlutathione
Singletoxygengetsdetoxifiedprincipallybypolyphenolantioxidants
SuperoxideradicalsaredetoxifiedbySuperoxidedismutase(SOD)
usingZincaselectronacceptorordonor(SomeclassesofSODs
useironorNickelaselectronacceptorordonor)

299.Matchenzymewiththediseasecaused
duetoitsdeficiency-
Enzyme
Disease
A.TyrosinemiaTypeII
Fumarylacetoacetate
hydroxylase
Tyrosinetransaminase
B.HomogentisateOxidase
Tyrosinase
C.TyrosinemiaTypeI
Alkaptonuria
D-Albinism
a)ID,2C,3A,4B
b)IA,2C,3D,4B
c)1C,2D,3A,4B
d)lC,2A,3D,4B
CorrectAnswer-D
Ans.is'd'i.e.,1-pC,2A,3D,4B[RefHarper
29th/e
p.289]

300.Whichofthefollowinghastwoamino
groups-
a)Glycine
b)Arginine
c)Lysine
d)Asparagine
CorrectAnswer-B
Histidinehastwoiminogroups.
Lysinehasoneaminogroup.
Argininehastwoaminogroups

301.Alternatefuelforbrainis
a)Glucose
b)Ketonebodies
c)Fattyacid
d)Aminoacid
CorrectAnswer-B
Ans.is'b'i.e.,Ketonebodies
There
isnostoredfuelinbrain,butitutilizes60%oftotalenergy
underrestingconditions.
Glucoseisvirtuallythesolefuelforthebrain,exceptinprolonged
starvingwhenketonebodiesarethemajorsource.
Fattyacidsdonotserveasfuelforthebrain,becausetheyare
boundtoalbumininplasma;hencecannotcrosstheblood-brain
barrier.

302.Pyruvatecanbeasubstrateofallexcept
a)LactateDehydrogenase
b)Malicenzyme
c)Aspartatetransaminase
d)Alaninetransaminase
CorrectAnswer-C
Ans.is'c'i.e.,Aspartatetransaminase
Thereareseveralpathwaysintowhichpyruvatecanenter.The
pathwaychoseninagiventissuedependsonitsstateof
oxygenationandprevailingmetabolicconditions,asdescribedbelow
:
OxidativedecarboxylationtoacetylCoA
Intissuesthatareadequatelyperfusedwithoxygen(i.e.,under
aerobicconditions),pyruvateundergoesoxidative
decarboxylationtoformacetylCoA,whichisfurthercatabolizedto
CO2andH20viacitricacidcycle(Krebscycle).Thisreactionserves
asabridgebetweenglycolysisandKreb'scycle.Thus,pyruvate
servesasthesourceofsubstrateoffirstreactionofTCA(Citricacid
cycle),i.e.,acetylCoA.

303.Gasreleasedfromoligosaccharide
metabolismbyintestinalbacteriais
a)Carbondioxide
b)Sulphurdioxide
c)Nitricoxide
d)Methane
CorrectAnswer-D
Ans.is'd'i.e.,Methane
Somefooditemsarehighinindigestibleoligosaccharides.Eg,
transgalactooligosaccharides,fructooligosaccharides(Inulin)
Sincetheseoligosaccharidesdonotgetdigestedinthesmall
intestine,theyreachthelargeintestine.
Largeintestinalmicroorganismsbreakdowntheseoligosaccharides
toformshortchainfattyacidslikepropionate,butyrate.Theseshort
chainfattyacidsarefoundtobetrophictointestinalmucosa.Ithas
anti-inflammatoryeffectstoo.Hencethesearefoundtoprotect
intestinalmalignancy.Thisisthebasisofprebioticsbeing
supplementedtopeoplewithintestinalinflammatorydisordersandto
maintainintestinalflora.
Butthedownsideoftheseindigestibleoligosaccharidesisthat,when
microorganismsactontheseoligiosaccharides,theyresultingas
production.Thegasesproducedincludestheusualhydrogen,
nitrogenandcarbondioxide.Apartfromthatwhenoligosaccharides
areacteduponbymicroorganisms,itresultsinmethaneproduction.
Thismethaneisfoundtohavethebloatingandflatulenceeffect.

304.Whichvitaminisrequiredforconversion
ofserinetoglycine?
a)VitC
b)B12
c)Pyridoxine
d)Thiamine
CorrectAnswer-C
Ans.is'c'i.e.,Pyridoxine[RefHarper290/ep.267-268]
Glycineisanonessentialaminoacidsynthesizedfromanother
nonessentialaminoacidserine
SerineinthepresenceofSerinehydroxylmethyltransferase(SHMT)
getsconvertedtoglycine.SHMTisapyridoxinedependentenzyme.
Thisstepusestetrahydrofolateascoenzymeanditgetsconverted
toN5,N10methyleneTHFA,whichthenactsas1carbondonorand
helpsintheconversionofuridinetothymidine.

305.Headacheandpapilledemaarefeatures
oftoxicityofwhichvitamin?
a)VitaminA
b)VitaminD
c)VitaminC
d)VitaminE
CorrectAnswer-A
Ans.is'a'i.e.,VitaminA
HypervitaminosisA
CausedbyconsumptionoffoodrichinvitaminAlikefishorliver(not
byexcessiveintakeofcarotenoidsastheconversionofcaroteneto
vitaminAisregulated).
Mechanismoftoxicity:
Suppressesosteoblastsandstimulatesosteoclasts.
Highretinolconcentrationsstimulatelysosomalenzymereleaseand
causelocaltissuedamage.
Featuresincludeanorexia,irritability,headache,skinpeeling,
vomiting.Headache,vomitingandpapilledemaarefoundtobe
causedbyincreaseinintracranialtension.Hencethisconditionis
calledaspseudotumourcerebri.AsvitaminAstimulatesosteoclasts,
hypercalcemia,bonyprojections,pathologicalfracturesarealso
observedasfeatures.

306.Whichofthefollowingisteratogenic:
a)Folate
b)Cyanocobalamin
c)VitaminA
d)VitaminC
CorrectAnswer-C
Ans.C:VitaminA
PharmacologicaldosesofvitaminAareteratogenicandin
pregnancythedailydosemustnotexceed6000-8000IU.
SyntheticanaloguesofvitaminA:
*Tretinoin
*Isotretinoin
*Etretinate
*Acetretin
*Teratogenicdrugs
Drugsandmedications:
*TobaccoCaffeine
*Drinkingalcohol(ethanol)(fetalalcoholspectrumdisorder),
*Isotretinoin(13-cis-retinoicacid)
*Temazepam
*Nitrazepam
*Aminopterinormethotrexate
*AndrogenichormonesBusulfan
*Captopril,enalapril
*Coumarin
*Cyclophosphamide
*Diethylstilbestrol
*Phenytoin(diphenylhydantoin)

Lithium
*Methimazole
*Penicillamine
*Tetracyclines
*Thalidomide
*Trimethadione
*Flusilazole
*Valproicacid
Environmentalchemicals:
*Polycyclicaromatichydrocarbons(polynucleararomatic
hydrocarbons)
*Polychlorinatedbiphenyls(PCBs)
*Polychlorinateddibenzodioxinsa.k.adioxin,
*Organicmercury
Ionizingradiation:
*Atomicweaponsfallout(Iodine-131,uranium)
*Backgroundradiation
Diagnosticx-rays
*Radiationtherapy
Infections:
*Cytomegalovirus
*Herpesvirus
*ParvovirusB19
*Rubellavirus(Germanmeasles)
*Syphilis
*ToxoplasmosisAneasywaytoremembermaternalinfectionsis
TORCH:Toxoplasmosis,Otheragents,Rubella,CMVandHSV.
Metabolicimbalance:
*Alcoholism
Diabetes
*Folicaciddeficiency
*Iodinedeficiency
*Hyperthermia

307.Alcoholismleadstodeficiencyofwhich
vitamin?
a)VitaminA
b)VitaminB1
c)VitaminD
d)VitaminB6
CorrectAnswer-B
Ans.is'b'i.e.,VitaminB1
Ofallthemicronutrients,thiaminewillbethefirstmicronutrientto
becomedeficient.Reasonis,apartfromthepersonmissinghis
mixedbalanceddiet,alcoholisalsofoundtointerferewiththiamine
absorption.Thethirdreasonrelatedtothiaminedeficiencyina
chronicalcoholicisthatalcoholinterfereswithmagnesium
absorption.Magnesiumisnecessaryforactivationofthiaminetoits
coenzymeformthiaminepyrophosphateinthepresenceofthiamine
kinase.

308.VitaminB12isrequiredforallofthe
followingexcept?
a)Conversionofhomocysteinetomethionine
b)Conversionofhomocysteinetocysteine
c)ConversionofpropionylcoAtosuccinylCoA
d)ConversionofmethylTHFAtoTHFA
CorrectAnswer-B
Ans.is'b'i.e.,Conversionofhomocysteinetocysteine
ActiveformofvitaminB12aremethylcobalamineand
deoxyadenosylcobalamine.FollowingreactionsrequirevitaminB12
coenzyme:
i)IsomerizationofmethylmalonylCoAtosuccinylCoA:
Inthisreaction,activeformofvitaminB12isdeoxyadenosyl
cobalamine.
Propionyl-CoAisproducedascatabolicendproductofsome
alipathicaminoacidsand(3-oxidationofoddchainfatty
acids.PropionylCoAisthenconvertedtosuccinylCoAthrough
methylmalonyl-CoA.
Thusmethylmalonyl-CoAisaccumulatedandexcretedinurineas
methylmalonicacid(methylmalonate)invitaminB12deficiency,i.e.
methylmalonicaciduria.
ii)Conversionofhomocysteintomethionine
Inthisreaction,activeformismethylcobalamine.
Thisistheonlyreactionwhichrequiresbothvitamin812(as
methylcobalamine)andfolicacid(asN5-methyl-114-folate).
Thereactioniscatalyzedbytheenzymecobalamin-dependent
methioninesynthasealsocalled5-methyltetrahydrofolate-

homocysteinemethyltransferase.

309.Whichofthefollowingvitaminsis
significantlysynthesisedingutby
intestinalflora?

a)Folate
b)B12
c)Biotin
d)B6
CorrectAnswer-A:B:C
Ans.is'c>'b'&'a'i.e.,Biotin>B12&Folate
Thoughvitaminsaresupposedtobeessentialmicronutrientswhich
aresupposedtobesuppliedinthediet,somevitaminslikeVitamin
DandNiacinareendogenouslysynthesised.
VitaminD3orcholecalciferolissynthesisedintheskinepidermis
whenUVlightactson7dehydrocholesterolwhichispresentinthe
malphigianlayer.UVlightopensuponeoftheringsofcholesterol,
convertingcholesterolintoasecosteroidwhichischolecalciferol.
Cholecalciferolreachesliver.Inliveritishydroxylatedat25th
positionby25ahydroxylasetoform25hydroxycholecalciferol
25hydroxycholecalciferolreachesthekidney.Itishydroxylatedat
1stpositionby1ahydroxylasetoform1,25
dihydroxycholecalciferol,theactiveformofvitaminD.
Niacin'sactivecoenzymeformsNADandNADParesynthesised
endogenouslyfromaminoacidtryptophan
Apartfromthesetwovitaminssynthesisedbyhumanmetabolic
pathways,therearevitaminslikeBiotin,VitaminKwhichare
synthesisedtosignificantamountinlargeintestinebyintestinal
microorganisms.ResearchhasidentifiedsynthesisoffolateandB12

aswellinlargeintestine.
Thoughtheyaresynthesisedinlargeintestine,asallwatersoluble
vitaminsgetabsorbedinsmallintestine,onlyendogenously
synthesisedBiotinandVitaminKarefoundtocontributesignificantly
tometabolicpathways.ThatiswhybiotinandvitaminKdeficiencies
arerelativelyrare.

310.Chainbreakingantioxidantsareall
except-
a)Tocopherol
b)Ascorbicacid
c)Polyphenolicflavinols
d)Superoxidedismutase
CorrectAnswer-D
Ans.is'd'i.e.,Superoxidedismutase
Chainbreakingantioxidantsaremoleculeswhichcandonatean
electronoracceptelectronfromunstableintermediatesoflipid
peroxidationconvertingthemintostableintermediates.
Theyareoftwotwotypes
Lipidphasechainbreakingantioxidant
Aqueousphasechainbreakingantioxidant
Lipidphasechainbreakingantioxidant
Themostimportantlipidphasechainbreakingantioxidantisa
tocopherol.atocopherolreactswithperoxylradicaltoform
tocopheroxylradicalwithexcesschargeassociatedwithextra
electronbeingdistributedalongthechromanering.
Aqueousphasechainbreakingantioxidant
Themostimportantchainbreakingantioxidantofthistypeis
ascorbicacidorvitaminC.Itcanscavengemanysuperoxide
radicals.Mostimportantlyithelpsbyregeneratingtocopherolafterit
isoxidisedduringtheprocessofreducingperoxylradicals.
ApartfromvitaminCwehaveagroupofpolyphenolflavinolslike
epitachingallatewhichcanreduceoxidantspeciesinaqueous
phase.Thesearepresentingreenteaandsomeantioxidant
supplements

Superoxidedismutaseisanantioxidantenzymewhichhelpsin
detoxifyingsuperoxidestoformmolecularoxygenandhydrogen
peroxide.Itisnotachainbreakingantioxidant

311.Whichofthefollowingelementsis
knowntoinfluencethebody'sabilityto
handleoxidativestress?

a)Fluride
b)Iron
c)Copper
d)Selenium
CorrectAnswer-D
Ans.is'd'i.e.,Selenium[Ref:PankajNaikp.382]
Theactivityoftheantioxidantenzymesdependsonsupplyof
minerals:?
*Manganese
*Zinc
*Copper
*Selenium
-Manganese,copperandzincarerequiredfortheactivityof
superoxidedismutase.
-Seleniumisrequiredfortheactivityofglutathioneperoxidase.

312.VitaminB12deficiencycausesallexcept
?
a)Homocysteinuria
b)Methylmalonicaciduria
c)Subacutecombineddegeneration
d)Epinephrineexcess
CorrectAnswer-D
Ans.is'd'i.e.,Epinephrineexcess[RefDineshPuri3rdlep.381]
DeficiencyofvitaminB12causesPerniciousanemia,megaloblastic
anemia(secondarytofunctionalfolatedeficiencyduetofolatetrap),
methylmalonicaciduriaduetoaccumulationofmethylmalonyl-CoA,
andneuropathy,likesubacutecombineddegeneration(SACD)and
demyelination.
Theremayhomocysteinuriaasmethioninesynthase,a
methylcobalaminedependentenzyme,isdefective.

313.Deficiencyofwhichvitaminduring
pregnancypredisposesto
meningomyelocele?

a)Folicacid
b)Biotin
c)Pyridoxine
d)Thiamine
CorrectAnswer-A
Ans.is'a'i.e.,Folicacid
FolicaciddeficiencyinpregnancypredisposedtoNTD(e.g.
myelomeningocele,meningocele,spinabifida).

314.RichestsourceofvitaminB12?
a)Meat
b)Greenleafyvegetables
c)Cornoil
d)Sunfloweroil
CorrectAnswer-A
Ans.is'a'i.e.,Meat
VitaminB12isnaturallyfoundinanimalproducts,includingfish,
meat,poultry,eggs,milk,andmilkproducts.
VitaminB12isgenerallynotpresentinplantfoods,butfortified
breakfastcerealsareareadilyavailablesourceofvitaminB12with
highbioavailabilityforvegetarians.
RichsourceofvitaminB12
Beef,liver,andchicken.
Fishandshellfishsuchastrout,salmon,tunafish,andclams.
Fortifiedbreakfastcereal.
Low-fatmilk,yogurt,andcheese.
Eggs.

315.RegardingNADandNADP,trueis?
a)Precursoristyrosine
b)MalicenzymeisanNADdependentenzyme
c)Highleucinecausesniacindeficiency
d)Niacindeficiencycausescutaneousvasodilatation
CorrectAnswer-C
Ans.is'c'i.e.,Highleucinecausesniacindeficiency
NADandNADParenicotinicacidderivativeswhichissynthesized
fromtryptophan(nottyrosin)
MalicenzymeisNADPdependentenzyme(notNADdependent)
Excessofleucineinhibitstheconversionoftryptophanintoniacin
andcausespellagra.
NiacindeficiencycausesPellagra.Pellagracharacterisedbythe
threeDs-Diarrhoea,DermatitisandDementiaiscausedbyniacin
orvitaminB3deficiency.

316.AntistressVitaminis?
a)VitaminB1
b)VitaminB2
c)VitaminB3
d)VitaminB5
CorrectAnswer-D
Ans.is'd'i.e.,VitaminB5
VitaminB5orpantothenicacidiscalledasanantistressvitaminasit
isfoundtohaveanimpactoncortisolreleaseandisfoundto
supportourimmunesystem.

317.TrueaboutPurinesynthesis?
a)GlutamineistheaminogroupdonorforN9
b)PRPPsynthetaseistheratelimitingenzymeofpurinesynthesis
c)THFAisnecessaryforformingC6ofpurinering
d)GMPisthefirstnucleotidetobeformedduringpurinesynthesis
CorrectAnswer-A
Ans.is'a'i.e.,GlutamineistheaminogroupdonorforN9
Inpurinenucleotidesynthesis,firstRibose5phosphateisactivated
byPRPPsynthetasetoformPRPP.
ThisPRPPgetsattachedtoN9(sourceisglutamine)inthe
presenceofPRPPglutamineamidotransferase.Thisistherate
limitingenzymeofpurinesynthesis.
PRPPsynthetaseisacommonenzymeofpyrimidinenucleotide
synthesisandofniacinadeninedinucleotidesynthesis.AsPRPP
synthetaseisnotcommittedforPurinenucleotidesynthesis,itcan
notbeconsideredastheratelimitingenzymeofpurinesynthesis.
Inpurinesynthesis,Inosinemonophosphate(IMP)isfirstformed
IMPonaminationwithAspartategivesrisetoAMPinthepresence
ofadenylosuccinatesynthetase.ThisstepneedsGTPasasource
ofenergy
IMPondehydrogenationbyIMPdehydrogenasefollowedby
aminationwithglutaminewillgiverisetoGMP.ThisstepusedATP
asasourceofenergy

318.Trueaboutpyrimidinecatabolismis?
a)Itisasourceofuricacid
b)aminoisobutyrateisgenerated
c)Unlikeothercatabolicpathways,itdoesnotgenerate
intermediatesofcitricacidcycle
d)Increasedpyrimidinecatabolismcausesdecreasedsynthesisof
carnosine
CorrectAnswer-B
Ans.is'b'i.e.,aminoisobutyrateisgenerated
Inpyrimidinecatabolism,firstcytidineandthymidineareconverted
touridinebydeaminationanddemethylationrespectively.
Uridineinthepresenceofphosphorylasegetsconvertedintouracil.
Uracilundergoeshydrogenationinthepresenceofdihydrouracil
dehydrogenasetoformdihydrouracil
Dihydrouracilhydrataseopenstheringofdihydrouraciltoforma
linearstructure

319.RegardingFISHallaretrueexcept?
a)Usedtodetectcopynumbervariations
b)Usedtodetectbalancedtranslocations
c)Requiresoligonucleotides
d)RequiresDNApolymerase
CorrectAnswer-D
Ans.is'd'i.e.,RequiresDNApolymerase
FluoroscentinSituHybridisation
isacytogenetictechniqueused
todetectchromosomalabnormalities.
ThistechniqueusesfluorescentlylabelledoligonucleotidesorDNA
probes.Theseprobesbindtospecificsequencesofachromosome.
Attachedtotheprobesarereporterfluorescentdyeswhichunder
fluorescencemicroscopyconfirmthepresenceorabsenceofa
particularchromosomalaberration.
InFISH,thetargetisthenuclearDNAofeitherinterphasecellsorof
metaphasechromosomesaffixedtoamicroscopeslide.Whena
specificprobeisadded,itannealstoitscomplementarysequencein
theaffixedDNA.Astheprobeislabelledwithareportermoleculeit
isvisualizedbyfluorescencemicroscopy

320.Allofthefollowingaretrueabout
NucleicAcidSequenceBased
Amplificationexcept?

a)ItisaspecificamplificationofRNA
b)ItisareplacementforreversetranscriptasePCR
c)Denaturationiscarriedoutat94?C
d)ItrequiresReversetranscriptase.
CorrectAnswer-C
Ans.is'c'i.e.,Denaturationiscarriedoutat94?C
NucleicAcidSequenceBasedAmplification(NASBA)orIsothermal
RNAamplificationisareplacementforReversetranscriptasePCR
(RT-PCR).
BothNASBAandRT-PCRareusedforamplifyingdesiredor
specificRNAfragments.
InRT-PCR,firstthedesiredRNAisconvertedtodsDNAusing
reversetranscriptaseandthenthedsDNAisamplifiedusingPCR
technique

321.Thelinkagewhichlinksindividual
nucleotidesinapolynucleotidechainis-
a)pN-Glycosidiclinkage
b)aN-glycosidiclinkage
c)3'5'Phosphodiesterlinkage
d)5'3'Phosphodiesterlinkages
CorrectAnswer-C
Ans.is'c'i.e.,3'5'Phosphodiesterlinkage
Nucleoside
N-glycosidicbondbetweenpentosesugar
andnitrogenousbase.
Nucleotide
Posphoesterlinkage(notphosphodiester)
(monophasphate
betweennucleosideandphosphategrp.
nucleatide)
Diphosphate&
Acidanhydridelinkagebetween
polyphosphate
monophosphate&otherphosphategrp
nucleotides
Polynucleotidechain
3'5'phosphodiesterlinkagebetween3'
hydroxylgroupnucleotidewith5'
phosphategroupofnextnucleotide.

322.Bondformationbetweenribosesugar
andnitrogenis?
a)Acidanhydridelinkage
b)Phosphodiesterlinkage
c)Phosphoesterlinkage
d)Glycosidiclinkage
CorrectAnswer-D
Ans.is'd'i.e.,Glycosidiclinkage
Pentosesugar(riboseordeoxyribose)islinkedtoanitrogenous
base(purineorpyrimidine)viacovalentN-glycosidicbondtoform
nucleoside

323.Ataxiatelangiectasiaiscausedbya
defectof?
a)BaseExcisionRepair
b)NucleotideExcisionrepair
c)Mismatchrepair
d)dsDNAbreakrepair
CorrectAnswer-D
Ans.is'd'i.e.,dsDNAbreakrepair
DNArepair
Defectassociated
Mismatchrepair
HereditaryNonpolyposisColonCancer
NucleotideExcision
Xerodermapigmentosa,Cockayne
Repair
syndrome
DoubleStrandedDNA
AtaxiaTelangiectasia,Bloom's
Break
syndrome,
Repair
Fanconi'sanemia


324.Jumpinggenesare?
a)Moderatelyrepetitivesequences
b)Highlyrepetitivesequences
c)Satellitesequences
d)Simplesegmentrepeatsequences
CorrectAnswer-A
Ans.is'a'i.e.,Moderatelyrepetitivesequences

325.Molecularmimicryisestablishedinthe
presenceof?
a)Cysteine
b)Alanine
c)Glycine
d)Tryptophan
CorrectAnswer-A
Ans.is'a'i.e.,Cysteine
Molecularmimicryiscausedbystructuralhomologyorsimilarity
betweenforeignantigensandself-antigens.
AsaresultofstructuralsimilarityaTcellorBcellactivatedagainsta
foreignantigencrossreactswithself-antigenandcauses
autoimmunity.
Molecularmimicrydoesnotalwaysexpectasequencesimilarityto
getinitiated.Itisfoundtobeinducedevenwhentheselfantigens
andexogenousantigenssharethesamebindingmotifwithMHC
(MajorHistocompatibilityComplex).
Thisbindingmotifsimilarityismoreoftenfoundinthepresenceof
cysteine(formsdisulphidebridges),arginineorlysine(form
hydrogenbonds)inthebindingsiteofantigens.

326.DNAfragmentsareseparatedby?
a)Ultracentrifugation
b)Agarosegelelectrophoresis
c)Paperchromatography
d)Highpressureliquidchromatography
CorrectAnswer-B
Ans.is'b'i.e.,Agarosegelelectrophoresis[RefEssentialsof
biochemistryp.
756]
DNAiscutintolargefragments,usingrestrictionenzymes.These
fragmentsarethenseparatedbygelelectrophoresis(eitheragarose
orpolyacrylamidegelelectrophoresis).

327.Lefthandedhelixisseenin-
a)BDNA
b)ADNA
c)ZDNA
d)FDNA
CorrectAnswer-C
Ans.is'c'i.e.,ZDNA
BasedonquartenarystructureofDNAmanyconformationsare
possibleoutofthecommonthreeformsare?BDNA,ADNAandZ
DNA.
ZDNA
ItisfoundinthoseregionsofchromosomeswhicharerichinGC
sequences
Itisalefthandedhelix
Everyfullturnhas12basepairing.

328.AminoacyltRNAgetsattachedtowhich
siteofribosome?
a)Psiteof40sribosome
b)Asiteof60sribosome
c)Asiteof40sribosome
d)Psiteof60sribosome
CorrectAnswer-B
Ans.is'b'i.e.,Asiteof60sribosome
TranslationisaprocessbywhichnucleotidebasesofmRNAget
translatedasaminoacidsequencesofpolypeptidechain.Ittakes
partinfreeribosome.
FreeribosomereadsthemRNAfrom5'endto3'end.Itreadsthe
codonsofmRNAonebyone.Dependinguponthecodonthatis
presentinmRNA,ribosomeiscapableofattachinga
complementaryanticodoncontainingtRNA.
ThistRNAcarriesacorrespondingaminoacid.Thiswaythe
polypeptidechaingrowsfromaminoterminalendtocarboxy
terminalend.
Eukaryoticribosomeisa80sunit.Itdissociatesinto40sand60s
subunits.60ssubunitcontainsPsiteandAsite.
Whenribosomeentersintochainelongation,onthePsiteof60s,is
attachedtheAUGcodonofmRNA,towhichinitiationmethionine
tRNAisattachedandAsiteisfree

329.Okazakifragmentsareformedduring?
a)Transcription
b)Translation
c)DNAreplication
d)None
CorrectAnswer-C
Ans.is'c'i.e.,DNAreplication

330.Splicingisaprocessof?
a)Activationofprotein
b)Removalofintrons
c)Synthesisofprotein
d)ReplicationofDNA
CorrectAnswer-B
Inmolecularbiologyandgenetics,splicingisamodificationofan
RNAaftertranscription,inwhichintronsareremovedandexonsare
joined.
ThisisneededforthetypicaleukaryoticmessengerRNAbeforeit
canbeusedtoproduceacorrectproteinthroughtranslation.
Formanyeukaryoticintrons,splicingisdoneinaseriesofreactions
whicharecatalyzedbythespliceosomeacomplexofsmallnuclear
ribonucleoproteins(snRNAs),buttherearealsoself-splicingintrons.

331.Proteinsarestoredas?
a)Structuralproteins
b)Functionalproteins
c)Fats
d)Lysosomalenzymes
CorrectAnswer-C
Ans.is'c'i.e.,Fats
Thereisnostorageformofprotein
Hencetheaminoacidsformedbybreakdownofproteinsundergo
catabolism
Duringcatabolism,mostoftheaminoacidsgiveofftheiramino
groupsandthenthecarbonskeletonundergoescatabolism
Oncatabolism,thecarbonskeletonofaminoacidsgiveriseto
glycolyticintermediateorcitricacidcycleintermediates.

332.Flowcytometryisdoneon
a)Polycythemia
b)Thrombocytosis
c)Basophil
d)Lymphocytes
CorrectAnswer-D
Ans.d.Lymphocytes
"Flowcytometrycanrapidlyandquantitativelymeasureseveral
individualcellcharacteristics,suchasmembraneantigensandthe
DNAcontentoftumorcells.Flowcytometryhasalsoproveduseful
intheidentificationandclassificationoftumorsarisingfromTandB
lymphocytesandfrommononuclear-phagocyticcells."-Robbins8/e
p324


333.TrueabouttRNA?
a)80%oftotalRNA
b)Contains50-60nucleotides
c)CCAsequenceistranscribed
d)LongestRNA
CorrectAnswer-C
"TheCCAtailisaCCAsequenceat3'endofthetRNAmolecule.In
prokaryotes,CCAsequenceistranscribed.Ineukaryotes,theCCA
sequenceisaddedduringprocessing".
"tRNAisthesmallestofthreemajorspeciesofRNAs"--Dinesh
puri
tRNAcomprises15%oftotalRNAinthecell.Itcontains73-93
nucleotideresidue.

334.MostcommonlyusedvectorforDNA
cloning?
a)Plasmid
b)Virus
c)Cosmid
d)Phage
CorrectAnswer-A
Ans:A.Plasmid
AcloningvectorisacarrierDNAmoleculetowhichhumanDNA
fragmentisattached.Normally,foreignDNAfragmentscannotself-
replicatewithinhostcell.Therefore,theyarejoinedtoavectorDNA,
thatcanreplicatewithinhostcell.
Thefivemajortypesofcloningvectorsusedare-
*Plasmids
*Viralvectors/Bacteriophages
*Cosmids
*BacterialArtificialChromosomes(BACs)
*Yeastartificialchromosomes(YACs)

335.ChimericDNAtrueareallexcept?
a)FormedbylinkingDNAfragmentsofunrelatedgenome
b)Stickyendproducingrestrictionendonucleasesfavour
formationofchimericDNA
c)Theydon'trequireDNAligases
d)TheorganismharbouringachimericDNAhasfeaturesof
themselvesandthepropertiesoftheinsert
CorrectAnswer-C
Ans.is'c'i.e.,Theydon'trequireDNAligases
ChimericDNAorrecombinantDNAisformedbylinkingDNA
fragmentsoftwounrelatedgenome.
ItisastepinvolvedinrecombinantDNAtechnology.
Itisdonetointroduceafavourablequalityintoanorganismlike
abilitytoproduceinsulinbyE.Coli(DonebylinkinginsulincDNAinto
avectorandintroducingtherecombinantvectororchimericDNA
intoE.Coli)orinsectresistanceincrops(donemyintroducingthe
genefragmentcapableofproducingBacillusthurengiensis)intoTi
plasmidandintroducingthechimericDNAorrecombinantDNAinto
growingplants).

336.Whatistheroleofcataboliteactivatorprotein(CAP)inLACoperon?
a)Positiveregulator
b)Negativeregulator
c)Attenuation
d)Constitutiveexpression
CorrectAnswer-A
Cataboliteactivatorprotein,CAPisanactivatorrequiredforhigh
leveltranscriptionofthelacoperon.CAPproteinexertspositive
controlandlacrepressorexertsnegativecontrol.
WhatisanOperon?

Inprokaryotes,thegenescodingforproteinsinvolvedinaparticular
metabolicpathwayareoftensequentiallyarranged-togetheronthe
chromosomealongwithasinglepromoterorregulatoryregion.This
entireclusteristermedasanoperon,foreg,thelacoperon(coding
proteinsformetabolismoflactose)ortrpoperon(codingproteins
neededforthesynthesisoftryptophan).
Lacoperon:

LacoperoncontainslacZ,lacYandlacAgenesencodingbeta
galactosidase,galactosepermeaseandthiogalactoside
transacetylaseandisprecedbyanoperator(O)siteandpromoter
(P)site.TheoperonistranscribedbyRNApolymerasetoproducea
singlepolycistronicmRNAthatistranslatedtoproduceallthree
enzymeswhichareinvolvedinlactosemetabolism.

337.LacYinLacOperoncodesfor?
a)BGalactosidase
b)GalactosidePermease
c)ThiogalactosideTransacetylase
d)Repressor
CorrectAnswer-B
Ans.is'b'i.e.,GalactosidePermease
Thesecodefor3proteinsthatareinvolvedincatabolismoflactose.
Thesegenesare'Z'gene(codesforP-galactosidase),'Y'gene
(codesforgalactosidepermease),and'A'gene(codesfor
thiogalactosidetransacetylase).

338.UbiquitinProteasomepathwayisused
fordegradationof?
a)Extracellularlonglivedproteins
b)Intracellularlonglivedproteins
c)Intracellularshortlivedproteins
d)Extracellularshortlivedproteins
CorrectAnswer-C
Ans.is'c'i.e.,Intracellularshortlivedproteins
Extracellularproteinsandintracellularlonglivedproteinsget
degradedinlysosomeswiththehelpofcathepsin
Intracellularshortlivedproteinsgetlabelledwithubiquitinoncetheir
lifespangetsover.

339.SmallRNAsare?
a)Between200and500bpsinlength
b)Codedbysmallexons
c)Amodeofregulationofgeneexpression
d)Alwaysendogenouslysynthesised
CorrectAnswer-C
Ans.is'c'i.e.,Amodeofregulationofgeneexpression
SmallRNAsarelessthan200nucleotidesinlength.
Theyarecodedbyintronicsequencesofgenes.
TheyhelpinregulationofgeneexpressionthroughGENE
SILENCINGmechanism
ThemiRNAgenesequenceslocatedwithinintronicsequencesare
transcribedbyRNApolymeraseIIItoformPri-miRNA.
Pri-miRNAisprocessedbymicroprocessorcomplextoformastem
loopstructure,Pre-miRNA.

340.Allofthefollowingaretrueabout
collagenstructureexcept-
a)Collagenissecretedbyfibroblastsasprocollagen
b)LysyloxidaseisdependentonVitaminC
c)Hydroxylysineundergoesglycosylation
d)Glycineisthemostabundantaminoacidofcollagen
CorrectAnswer-B
Ans.is'b'i.e.,LysyloxidaseisdependentonVitaminC
Collagenisthemostabundantproteininthebody.
Collagenisatriplehelix.Itismadeupof3polypeptidechains-
Eachpolypeptidechainhasabout1000aminoacids.Itismadeup
ofrepetitiveunitsof(Gly-X-Y),whereXandYaremostcommonly
prolineandhydroxyproline.Hence33%ofaminoacidresiduesof
collagenisglycine-themostabundantaminoacidofcollagen.
Collagenissynthesisedasprocollagenbyfibroblastsintracellularly.
AftertranslationofmRNAofcollageninribosomes,thepolypeptide
chainsundergoposttranslationalmodificationsintheformof
hydroxylationandglycosylation.

341.Themostabundantaminoacidof
collagenis-
a)Glycine
b)Proline
c)Lysine
d)Tryptophan
CorrectAnswer-A
Ans.is'a'i.e.,Glycine

342.Foldsincollagenaredueto-
a)Glycine
b)Proline
c)Hydroxyproline
d)Lysine
CorrectAnswer-A:B
Ans.is`b>a'i.e.,Proline>Glycine[RefEssentialsof
biochemistyp.868]
Twoaminoacidsareinvolvedinproducingfoldsincollagen:proline
andglycine.
Inordertoformatriple-helixapolypeptidechain(a-chain)must
containglycineaseverythirdresidueinthesequence.
Thisisbecauseonlytheglycineissmallenoughtobe
accommodatedinthelimitedspaceavailabledownthecentralcore
ofthetriplehelix.Eachturnofpolypeptidechain(a-chain)contains
threeaminoacidresidues,andglycine(Gly)ispresentateverythird
position.
Thusglycineconstitutes33%ofthetotalaminoacidresidues.The
repeatingaminoacidresidues,representedas(Gly-X-Y)n,isan
absoluterequirementforformationoftriplehelix.XandYcanbe
anyaminoacids,butmostofthetimeXisproline(10%ofthetotal
aminoacidresidues)andmostofthetimeYishydroxyproline.Other
importantaminoacidsfoundincollagenarelysineand
hydroxylysine.

343.Hemesynthesisrequires
a)Ferrousiron
b)Glycine
c)SucchylCoA
d)All
CorrectAnswer-D

344.Hemeiswhichporphyrin?
a)TypeI
b)TypeII
c)TypeIII
d)TypeIV
CorrectAnswer-C
Ans.is'c'i.e.,TypeIII

345.Gerhardt'stestisusedtodetect?
a)Reducingsugar
b)Ketonebody
c)Protein
d)Blood
CorrectAnswer-B
Ans.is'b'i.e.,Ketonebody
Gerhardt'stestisatestusedtodetectacetoacetateinurine
(acetoneand(3hydroxybutyratedonotanswerthistest).
ReagentsusedforGerhardt'stestincludeConcentratednitricacid
and10%Ferricchloride

346.GeneticallymediatedVLDL
overproductionisafeatureofallexcept
?

a)Familialcombinedhyperlipidemia
b)Hypoapobetalipoproteinemia
c)Familialdyslipidemichypertension
d)LDLsubclassB
CorrectAnswer-B
Ans.is'b'i.e.,Hypoapobetalipoproteinemia[RefEssentialsof
Biochemistry
p.712]
FamilialcombinedhyperlipidemiaorTypelibFamilial
hyperlipoproteinemiaischaracterisedbygeneticallydetermined
overproductionoftriglyceriderichVLDL(VLDL1)andsmalldense
LDLparticles.
Itisthemostcommonlyinheriteddisorder.Pleaseremember
Acquiredcombinedhyperlipidemiawithsamefeaturesiscausedby
metabolicsyndromeorinsulinresistance.
Hyperapobetalipoproteinemiaischaracterizedbyanincreased
numberofsmall,denseLDLparticlesandanelevatedLDL-Blevel
withnormalorborderlinehighLDL-Clevels.Patientswith
Hyperapobetalipoproteinemiamayhavenormalorhightriglycerides.
Hyperapobetalipoproteinemiahasbeenfoundtobethemost
commonphenotype(34%)associatedwithprematureCAD
;Hyperapobetalipoproteinemiawithhypertriglyceridemiaisfoundto
beevenmorestronglyassociatedwithCADthan
Hyperapobetalipoproteinemiawithnormaltriglycerides.

347.Abetalipoproteinemiaaffects?
a)Retinalpigmentepithelium
b)Opticnerve
c)Occipitalcortex
d)Bipolarneurons
CorrectAnswer-A
Ans.is'a'i.e.,Retinalpigmentepithelium
AbetalipoproteinemiaorBassen-Kornzweigsyndromeiscausedby
adefectofMicrosomalTriglycerideTransferProtein(MTTP).
Inabetalipoproteinemia,bothchylomicronandVLDLformationget
affected.
Asaresult,fatabsorptionisaffectedandtheaffectedchildpresents
withsteatorrheainthefirstfewmonthsoflife.
Allfatsolublevitaminabsorptionalsogetaffected.HenceVitaminA,
D,EandKdeficiencyisexpected,howeverVitaminEdeficiencyis
foundtohaveprofoundeffects.
VitaminEdeficiencypresentsasretinitispigmentosaandsubacute
combineddegeneration.
Otherfeaturesofabetalipoproteinemiaareacanthocytes(star
shapedRBCsinperipheralsmearcausedbecausethelipid
compositionofRBCmembranegetsaffected),hypocholesterolemia.
TreatmentinvolvesvitaminEadministration.

348.Syntheticoxygencarrieris?
a)2,4dinitrophenol
b)Chloflurocarbon
c)Perflurocarbon
d)1fluro2,4dinitrophenol
CorrectAnswer-B
Ans.is'b'i.e.,Chloflurocarbon
Blooddopingisaformoffraudulentincreaseintheoxygencarrying
capacityofaperson,widelyusedtoimprovetheaerobiccapacityof
athletes
Thisisdonebyvariousmethods
Autologousbloodtransfusion
Homologousbloodtransfusion
ErythropoietinorContinuousErythropoietinReceptorActivator

349.Pepsinogenisactivatedby?
a)Enterokinase
b)Enteropeptidase
c)H+
d)Trypsin
CorrectAnswer-C
Ans.isci.e.,H+
Pepsinissecretedbychiefcellsofstomachinaninactive
(zymogen)formcalledpepsinogen.
Acid(IF)inlumenofstomachconvertspepsinogentoactivepepsin.
Pepsinonceformedalsoattackspepsinogenproducingmorepepsin
moleculesbyautocatalysis.

350.Whichofthefollowingaccumulatesin
maplesyrupurinedisease
a)Leucine
b)Valine
c)Isoleucine
d)All
CorrectAnswer-D
Ans.is`d'i.e.,All
Maplesyrupurinedisease(MSUD)orbranched-chain
ketoaciduria
*Itisaninbornerrorofmetabolismofbranched-chainamino
acidsvaline,leucineandisoleucine.
*Itisduetodeficiencyofanenzymethatcatalyzesthesecond
reactionintheseaminoacidsmetabolismi.e.branchedchain-a
keto
aciddehydrogenasewhichcatalyzesthedecarboxylationof
branched-chainaminoacids.
*Asaresult,thebranched-chainaminoacids,leucine,isoleucine
andvaline,andtheira-ketoacidsaccumulateintheblood,urine,
andCSF.
*Thereisacharacteristicmaplesyrupodourtotheurine.
*Inmaplesyrupurinediseasethereistheexcretionofbranched-
chainaminoacids(isoleucine,leucine,valine)andtheirketoacids
(-keto-methylvalerate,-ketoisocaproate,-ketoisovalerate)in
theurine.

351.Hemeissynthesizedfrom?
a)Lysine+succinylCoA
b)Glycine+succinylCoA
c)Arginine+MalonylCoA
d)Glycine+MalonylCoA
CorrectAnswer-B
Ans.B.Glycine+succinylCoA
Organelle:Partlycytoplasmicandpartlymitochondrial
Startingmaterials:SuccinylCoAandGlycine


352.Glycemicindexisdefinedas:
a)Glucosecontrolinlast3months
b)Measureofthechangeinthebloodglucosefollowingingestion
ofproteins
c)Measureofthechangeinthebloodglucosefollowingingestion
ofcarbohydrate
d)Measureofthechangeinthebloodglucosefollowingingestion
offats.
CorrectAnswer-C
TheGlycemicindex(GI)ofacarbohydratecontainingfoodisa
measureofthechangeinthebloodglucosefollowingitsingestion

353.TrueaboutcoproporphyrinIand
coproporphyrinIIIis-
a)CoproporphyrinIisexcretedinurine
b)CoproporphyrinIIIisexcretedinbile
c)InDubinJohnsonSyndrome,CoproporphyrinIinurineis80%
ofthetotalcoproporphyrin
d)InDubinJohnsonSyndrome,totalcoproporphyrinlevelsis
elevated
CorrectAnswer-C
Ans.is'c'i.e.,InDubinJohnsonSyndrome,CoproporphyrinIin
urineis80%ofthetotalcoproporphyrin
Coproporphyrinsareintermediatesofhemesynthesis
NormallycoproporphyrinIisexcretedinbileandislostinfeces
CoproporphyrinIIIisexcretedinurine
HenceinnormalurineCoproporphyrinIis25%oftotal
coproporphyrinlevelsinurine
DubinJohsonsyndrome,isaformofconjugatedhyperbilirubinemia
andiscausedbyadefectofMultiDrugResistantProtein2(MRP-2),
whichisresponsibleforsecretingbilecomponents(conjugatedbile
pigments)fromhepatocytesintobiliarycanaliculi.
Inthisdisorder,theratioofCoproporphyrinI:CoproporphyrinIIIis
reversedandcoproporhyrinIismorethan80%ofthetotal
Coproporphyrinlevels.Butthetotalcoproporphyrinlevelsisnormal.
PossiblemechanismisCoproporphyrinIissecretedintobilethrough
MRP-2.

354.VitaminAdeficiencyleadstometaplasia
of?
a)Squamousepithelium
b)Columnarepithelium
c)Both
d)None
CorrectAnswer-A
Answer-A.Squamousmetaplasia
Itoccursmostfrequentlyinrespiratorytractasaresultofchronic
irritationbycigarettesmoking.Thecolumnarepitheliumisreplaced
bysquamousepithelium.Thissquamousmetaplasiarepresentsan
attemptbythehosttorepairorprepareanepithelialtissuethathas
beendamagedbyenvironmentaltoxicantswithamoreresistant
tissue.
Squamousmetaplasiaisalsoencounteredinotherlocationsuchas
pancreaticduct(VitAdeficiency,stonesin
pancreaticduct),gallbladder(gallstones),urinarybladder(bladder
calculi,Schistosomiasis),andinendocervix.
Thoughthemetaplasiaisreversible,iftheinfluencesthatpredispose
tometaplasiaispersistent,malignanttransformationmayoccurin
metaplasticepithelium.

355.Physiologicalhyperplasiaand
hypertrophyareseensimultaneouslyin-
a)Uterusinpregnancy
b)Skeletalmuscleinatheleteas
c)Breastatpuberty
d)aandc
CorrectAnswer-D
Ans.is'a>c'i.e.,Uterusinpregnancy>Breastatpuberty
Physiologicalhypertrophyandhyperplasia
*Examplesofphysiologicallyincreasedgrowthoftissuesinclude:-
-Skeletalmusclehypertrophyinatheletes,bothintheskeletal
muscleoflimbs(asaresponsetoincreasedmuscleactivity)andin
leftventricleofheart(asaresponsetosustainedoutflow
resistance).
-Hyperplasiaofbonemarrowcellsproducingredbloodcellsin
individualslivingathighaltitude;thisisstimulatedbyincreased
productionoftheerythropoietin.
-Hyperplasiaofbreastatpuberty,andinpregnancyandlactation,
undertheinfluenceofestrogen,progesterone,prolactinandhuman
placentallactogen.
-Hypertrophyandhyperplasiaofuterinesmoothmusclein
pregnancy,stimulatedbyestrogen.
-Thyroidhyperplasiaasaconsequenceoftheincreasedmetabolic
demandofpubertyandpregnancy.Foroption'c'
-"Hormonalhyperplasiaiswellillustratedbytheproliferationofthe
glandularepitheliumofthefemalebreastatpubertyandduring
pregnancy,usuallyaccompaniedbyenlargement(hypertrophy)of
theglandularepithelialcell"-
*However,IamnotsureaboutthisstatementofRobbin's,asno

*However,IamnotsureaboutthisstatementofRobbin's,asno
othertextbookhasmentionedthatbreastatpubertyundergoboth
hyperplasiaandhypertrophy.

356.Exampleofcoagulativenecrosisis-
a)Acutetubularnecrosis
b)Stroke
c)Malignanthypertension
d)Acutepancreatitis
CorrectAnswer-A
Answer-A.Acutetubularnecrosis
Thecausesofcoagulativenecrosisare:

1. Coagulativenecrosisismostfrequentlycausedbysuddencessation
ofbloodflow(ischemia)insolidorganssuchasheart(MI),Kidney
(ATN),Liver,adrenalgland,spleen.Amongstsolidorgansbrainis
theonlyexception,i.e.,itistheonlysolidorganinwhichischemia
leadstoliquefactivenecrosisandnotcoagulativenecrosis.
2. Toxicproductsofcertainbacteria,asincalfdiphtheria,necrophorus
enteritisandotherformsnecrobacillosis.
3. Certainlocallyactingpoisons,e.g.,mercuricchloride.
4. Mildburns(thermalinjury),whetherproducedbyheat,electricity,or
x-rays.
5. Zenker'sdegenerationnecrosisofmuscle.Itoccursintyphoid.
Rectusanddiaphragmaretheusualmusclesaffected

357.Dystrophiccalcificationisseenin?
a)Milkalkalisyndrome
b)Atheromatousplaque
c)Hyperparathyroidism
d)VitaminAintoxication
CorrectAnswer-B
Ans.is'b'i.e.,Atheromatousplaque
Dystrophiccalcification
*
Whenpathologicalcalcificationtakesplaceindead,dyingor
degeneratedtissue,
itiscalleddystrophiccalcification.oCalcium
metabolismisnotalteredandserumcalciumlevelisnormal.
Dystrophiccalcificationin
Dystrophiccalcificationin
deadtissues
degeneratedtissues
1.Incaseousnecrosisof
1. Atheromatousplague
tuberculosis
2. Monkeberg'ssclerosis
(mostcommonwhichmaybe
3. Psommamabodies
inlymphnodes)
4. Densoldscars
2.Chronicabscessin
5. Seniledegenratedchangessuch
liquifactivenecrosis
asincostalcartilage,tracheal,
3.Fungalgranuloma
bronchialrings,Pinealglandin
4.Infarct
brain.
5.Thrombi
6. Heartvalvesdamagedby
6.Haematomas
rheumaticfever.
7.Deadparasites-
Cystecercosis/Toxoplasma
Hydatid/Schistosoma
8.Infatnecrosisofbreast&
othertissues

Howdoescalcificationoccursinthesesitewithnormalserum
calcium?
*Calcificationofdeadanddyingcellsandtissuesisacommon
findinginhumanpathologicconditions.
*Denaturedproteinsindeadorirreversibledamagedtissues
preferentiallybindphosphateions.
*Phosphateionsreactwithcalciumionstoformaprecipitateor
calciumphosphate.
*Thus,necrotictissueservesasacalciumsink.

358.Whatisthefirstsignofinjury?
a)Mitochondrialdysfunction
b)Membranedamage
c)DiminishedATP
d)Releaseoflysosomalenzymes
CorrectAnswer-A
Answer-A.Mitochondrialdysfunction
hypoxiaisthemostcommoncauseofcellinjury.
Oxygenisanimportantrequirementofmitochondriaforthe
formationofATP;therefore,hypoxiawillresultintheearliest
involvementofmitochondriaresultingindecreasedformationof
ATP.
DiminishedATPconstitutesthecriticalmechanismofthecellular
injurywhichleadsto:
a.IntracellularaccumulationofCa+2
b.Mitochondrialdysfunction
c.Membranedamage
d.Releaseoflysosomal(hydrolytic)enzymes

359.Blebsarefoundinwhichtypeofinjury?
a)Reversible
b)Irreversible
c)Both
d)None
CorrectAnswer-A
Answer-A.Reversible
Pathologicalfeaturesofreversiblecellinjuryare:Cellularswelling
(earliest);lossofmicrovilli;cytoplasmicmembraneblebs;ER
swelling;Myelinefigures;detachmentofribosomefromER;
cytoplasmic(lipid)vacuole;clumpingofchromatin.

360.Hydropicchangeisdueto-
a)Accumulationofwaterintracellularly
b)Fataccumulationintracellularly
c)Lysozymedegeneration
d)Glycogenaccumulationintracellularly
CorrectAnswer-A
Answer-A>Accumulationofwaterintracellulary
Hydropicchange(abominoiddegeneration,hydropicdegeneration,
parenchymatousdegeneration,cloudyswelling)ofacellis
degenerativechange,inwhichthecellsswellduetoinjurytothe
membraneaffectingIonictransfer,causingthecytoplasmtoappear
cloudyandwatertoaccumulatebetweenthecells,withresultant
Swellingofthetissue.
Cellularswelling(hydropicchange)istheearliestchangeincell
injury(reversible).

361.Nucleiarearrangedatthecellperiphery
inwhichtypeofcell?
a)Langhansgiantcell
b)Merkelcells
c)NKcells
d)Neutrophils
CorrectAnswer-A
Answer-A.Langhansgiantcell
PeripheralnucleiareseeninLanghanstypegiantcells
Langhansgiantcellsarelargecellsfound
ingranulomatousconditions.
Theyareformedbythefusionofepithelioidcells(macrophages)and
containnucleiarrangedinahorseshoe-shapedpatterninthecell
periphery.
Thesecellscontain3-5nuclei.Nucleiarearrangedaroundthe
peripheryintheformofahorseshoe.Thesecellsmayalsoactas
precursorsofforeignbodygiantcells.ThesecellsareseeninTB&
sarcoidosis.

362.NottrueaboutVEGFis-
a)Highlyspecificforendothelium
b)Hypoxiapotentiatesitsexpression
c)Inhibitsangiogenesis
d)Helpsintumormetastasis
CorrectAnswer-C
Answer-C.Inhibitsangiogenesis
Vascularendothelialgrowthfactor(VEGF)isapotentangiogenic
factorandwasfirstdescribedasanessentialgrowthfactorfor
vascularendothelialcells.
VEGFisup-regulatedinmanytumorsanditscontributiontotumor
angiogenesisiswelldefined.
Inadditiontoendothelialcells,VEGFreceptorsareexpressedon
numerousnon-endothelialcellsincludingtumorcells.
Bindstoendo-specificreceptorsFlt-1andFlk-1(ofthetyrosine
kinasefamily).
ExpressionofVEGFpotentiatedbyhypoxiaandinactivationofp53.
ThetwomostimportantangiogenicfactorsinatumorareVEGFand
basicfibroblastgrowthfactor(bFGF),thushelpingintumor
metastasis.

363.Endothelialmoleculeinvolvedinrolling
is-
a)ICAM1
b)VCAM1
c)CD31
d)CD34
CorrectAnswer-D
Answer-D.CD34
Endothelialmolecule-MajorRole
P-selectin-Rolling
E-selectin-Rollingandadhesion
GlyCam-1,CD34-Rolling
ICAM-1(Immunoglobulinfamily)-Adhesion,arrest,transmigration
VCAM-1(Immunoglobulinfamily)-Adhesion
PECAM-1(CD-31)-Diapedesis(transmigration)

364.Acutephasereactantsarestimulatedby
-
a)IL-6
b)IL-1
c)TNF-a
d)Alloftheabove
CorrectAnswer-D
Answer-D.Alloftheabove
Inresponsetoinjury,localinflammatorycells(neutrophil
granulocytesandmacrophages)secreteanumberofcytokinesinto
thebloodstream,mostnotableofwhicharetheinterleukinsIL6and
IL8,andTNFa.Theliverrespondsbyproducingalargenumberof
acute-phasereactants.

365.Vasodilatationfollowingendothelial
damageisdueto?
a)Histamine
b)IL-2
c)TGF-3
d)FGF
CorrectAnswer-A
Answer-A.Histamine
Vasodilatation:-Histamine,prostaglandins(PGD2,PGE2),
prostacyclins,serotonin,Nitricoxide,bradykinin

366.Lymphotactiniswhichtypeof
chemokine?
a)C
b)C-C
c)C-X-C
d)C-X3-C
CorrectAnswer-C
Answer-C.C-X-C
C-chemokines:-Theyarespecificforlymphocytes,e.g.
lymphotactin.

367.Primarygranulescontainwhichenzyme?
a)Myeloperoxidase
b)Hydrolases
c)Lactoferrin
d)PhospholipaseA2
CorrectAnswer-A
Answer-A.Myeloperoxidase
Myeloperoxidase
Lysozyme
Acidhydrolases
Elasases
Nonspecificcollegenase
Bacterialpermeabilityprotein
Defensin(BPI)
CathepsinG
PhospholipaseA2

368.Activecompoundsecretedby
neutrophillsagainstpathogenisknown
as?

a)Majorbasicprotein
b)Myeloperoxidase
c)Hydrolase
d)Proteoglycans
CorrectAnswer-B
Answer-B.Myeloperoxidase
Theazurophilicgranulesofneutrophilscontaintheenzyme
myeloperoxidase(MPO),whichInthepresenceofahalidesuchas
Cl-,convertesH202tohypochlorite(HoC1).

369.Peroxidaseenzymefoundinneutrophills
isusedfor-
a)Macrophageactivation
b)Chemotacticagent
c)Microbialkilling
d)Vasodilatation
CorrectAnswer-C
Answer-C.Microbialkilling
Peroxidase(myeloperoxidase)isinvolvedinphagocytosis(microbial
killing).

370.Whichoneofthefollowingstatementsis
correctregardingchronicgranulomatous
disease?

a)Itisanautosomaldominantdisease
b)Itischaracterizedbyabnormalbacterialphagocytosis
c)Recurrentstreptococcalinfectionsareusualinthisdisease
d)Nitrobluetetrazoliumtestisusefulforscreening
CorrectAnswer-D
Ans.is'd'i.e.,Nitrobluetetrazoliumtestisusefulforscreening
Leukocytesfromthepatientsofchronicgranulomatousdiseasefail
toreducenitrobluetetrazolium(NBT)duringphagocytosis.This
propertyhasbeenusedasascreeningmethod(NBTtest)forthe
diagnosisofchronicgranulomatousdisease.
Chronicgranulomatousdisease(CGD)
Itisagroupofdisordersofgranulocyteandmonocyteoxidative
metabolism.
Inheritance
X-linked(Mostcommon)
Autosomalrecessive
Phagocytosisisnormalbutleukocytesdonotundergodegranulation
followingphagocytosis.
ThediminishedH202productionappearstobethemajorreasonfor
defectivebactericidaldefect.
Mutationinthegenesforthefourproteinsthatassembleatthe
plasmamembraneaccountforallpatientswithCGD:
Twoproteinsformtheheterodimercytochromeb-558intheplasma
membrane.

Twootherproteinsinteractwithcytochromeaftercellactivationto
formNADPHoxidaserequiredforHydrogenperoxideproduction.
Thebacteriainvolvedinrecurrentinfectionsarecatalasepositive
pyogenicpathogenssuchasstaphylococciandcoliforms.
Catalase
negativepathogenssuchasstreptococciandpneumococciare
handlednormally.
Thereisexcessiveinflammationwithgranulomasformation.

371.Mostcommoncauseofchronic
granulomatousdiseaseinchildrenis:
a)Myeloperoxidasedeficiency
b)Defectivephagocytosis
c)DefectiveH2O2production
d)Job'sdisease
CorrectAnswer-C
AnswerisC(DefectiveH2O2production)
Chronicgranulomatousdiseaseisa'disorder'ofmicrobialkilling'
characterizedbydecreasedabilityofneutrophilstoproduceH4)-Q

Chronicgranulomatousdiseaseisa'disorder'ofmicrobialkilling'characterizedby
decreasedabilityofneutrophilstoproduceH,O2.QPatients,herebecome
susceptibletodiseasecausedbyorganismsthatproduce`catalase',whichdestroys
thesmallamountofH2O,presentinthesecellsandleadstofailureofbacterial
killing.
Thediseaseiscalledso,becausegranulomas,areformedinvarioustissues,asa
secondlinedefenceagainstorganisms,thatnormallywouldberemovedbythe
acuteinflammatoryresponse.
Manifestations:
Recurrentinfectionswithwhichcatalase+vePyogenicbacteriae.g.staph/coliforms.
Q
-catalasenegativebacteriae.g.streptococcus,Pneumococcusareoftenhandled
normally.Q
Normalhumoralimmuneresponse?but
Defectivekillingprocess:Leucocytesareunabletokillbacteriaafterphagocytosis
themajorreasonbeingdecreasedproductionofH-O2.Q

Screeningmethod:
NBTtest:Nitroblueretrazoliumdyeisnotreducedbyneutrophilsinvitro.

372.Delayedumbilicalcorddetachment
leukocytosisisseenin?
a)Leukocyteadhesiondeficiency
b)Chronicgranulomatousdisease
c)Severecombinedimmunodeficiency
d)Noneoftheabove
CorrectAnswer-A
Answer-A.Leukocyteadhesiondeficiency
Leukocyteadhesiondeficiencytype1isadisorderthatcausesthe
immunesystemtomalfunction,resultinginaformof
immunodeficiency
Oneofthefirstsignsofleukocyteadhesiondeficiencytype1isa
delayinthedetachmentoftheumbilicalcordstumpafterbirth.In
newborns,thestumpnormallyfallsoffwithinthefirsttwoweeksof
life;but,ininfantswithleukocyteadhesiondeficiencytype1,this
separationusuallyoccursthreeweeksorlater.Also,affectedinfants
oftenhaveinflammationoftheumbilicalcordstump(omphalitis)due
toabacterialinfection.
It'sduetotheabsenceofCD18antigenthat'snecessaryforthe
formationofintegrins,thesubstancesthatareresponsibleforthe
secondstepofleukocytemigration:adhesion.Becausethe
leukocytescannotefficientlyreachthesiteofinsult,theycannot
engulftheoffendingsubstance,andnopuswillbeevident.Also,as
acompensatorymechanism,theremaybeexcessiveleukocytosis.

373.Decreaseinplasmaosmoticpressureis
causeofedemain?
a)CHF
b)DVT
c)Nephroticsyndrome
d)None
CorrectAnswer-C
Answer-C.Nephroticsyndrome
Decreasedplasmaosmoticpressureofcapillaries:Itcauses
decreasesininwarddrivingforce.ItresultsfromHypoproteinemia.
Whentotalplasmaproteinisbelow5gm/dl(normal6-8gm/d1)or
albuminisbelow2.5gm/dl(normal3.5-5gm/dl)edematakesplace.
Conditionscausinghypoproteinemiaarenephroticsyndrome,liver
cirrhosis,proteinlosingenteropathy,andPEM.

374.Normaltimetakenforscarformation
afterinjuryis?
a)2weeks
b)3weeks
c)4weeks
d)5weeks
CorrectAnswer-C
Answer-C.4weeks
Immediatelyafterincision:Incisionisfilledwithbloodclotcontaining
fibinandbloodcells(including),dehydrationofsurfaceclotforms
scabonsurface.
Within24hours:Neutrophilsappearatthemarginofwound,which
isfilledwithbloodclot.
24-48hours:Epithelialcellsmovefromwoundedgesandfusein
themidlinebeneaththesurfacescab,producingacontinuousbut
thinepitheliumlayerthatclosesthewound.
Day3:Neutrophilsarereplacedbymacrophages,granulationtissue
appears,collagenfibersarepresentinthemarginbutdonotbridge
theincision.
Day5:Abundantgranulationtissue,maximumneovascularization,
abundantcollagenwhichbridgestheincision,theepidermis
recoversitsnormalthickness.
2ndweek:Diappearanceofleukocytesandedema,presenceof
fibroblasts(derivedfromlocalmesenchyme)andmaximum
collagen.
Endof1month:Thereisscarwhichismadeupofcellular
connectivetissuedevoidofinflammatoryinfiltratecoverednowby
intactepidermis.ThereisreplacementofcollagentypeIIIwith
collagentypeI.

collagentypeI.

375.Whichtypeofhealingoccursinan
incisionalwoundwithinfection?
a)Primary
b)Delayedprimary
c)Secondary
d)Tertiary
CorrectAnswer-C
Answer-C.Secondary
Itsproducedbysharpcuttinginstruments-
knife,razor,blade,swords,chopper,axeetc.
Secondaryhealingoccursinanincisionalwoundwithinfection.

376.Allofthefollowingcelltypesundergocelldivision,EXCEPT:
a)Pericyte
b)Cardiacmusclecell
c)Smoothmusclecell
d)Satellitecellofskeletalmuscle
CorrectAnswer-B
Myocytecelldivisioninthehumanheartceasesafewweeksafter
birth.
Thereafter,enlargementoftheheartisasaresultofcell
hypertrophyorthelayingdownofcollagenintheextracellularspace.
DNAturnoverisalmostundetectableexceptinpathologicstates.
Approximately20%ofmyocytesinthehumanhearthavetwonuclei,
sothatcellseparation,ratherthanmitosis,couldbringaboutasmall
increaseinthetotalcellnumber.
Ref:FrancisG.S.,TangW.,WalshR.A.(2011).Chapter26.
PathophysiologyofHeartFailure.InV.Fuster,R.A.Walsh,R.A.
Harrington(Eds),Hurst'sTheHeart,13e.

377.Geneforembryogenicstemcellis-
a)Oct-4
b)Nanog
c)GJAI
d)Alloftheabove
CorrectAnswer-D
Answer-D.Alloftheabove
Acommonsubsetof92geneswasidentifiedthatincludedNanog,
GTCM-1,connexin43(GJA1),oct-4,andTDGF1(cripto)."

378.Complementmediatedhypersensitivity
reactionis?
a)Type-1hypersensitivity
b)Type-2hypersensitivity
c)Type-4hypersensitivity
d)None
CorrectAnswer-B
Answer-B.Type-2hypersensitivity
TypeIIreactions(Antibodymediated)InvolveimmunoglobulinG
orImmunoglobulinMboundtocellsurfaceantigen,withsubsequent
complementfixation.

379.T-cellmediateddiseaseis?
a)Asthma
b)Myastheniagravis
c)SLE
d)Sarcoidosis
CorrectAnswer-D
Answer-D.Sarcoidosis
TypeI(IgEmediated)
Eczema
Hayfever
Asthma
Atopy
Urticaria
Anaphylacticshock
Acutedermatitis
Theobaldsmithphenomenon
PrausnitzKusnter(PK)reaction
Casonistest
Schultz-Dalephenomenon
TypeII(IgG,IgMandcomplementmediated)
Bloodtransfusionreactions
Erythroblastosisfetalis
Autoimmunehemolyticanemiaorthrombocytopeniaor
agrnulocytosis4
Pemphigusvulgaris
Goodpasturesyndrome
Bullouspemphigoid
Perniciousanemia

Acuterheumaticfever
Diabetesmellitus
Gravesdisease
Myastheniagravis
TypeIII(IgG,IgM,complementandleucocytemediated)
Local-Arthusreaction
Systemic-serumsickness
Schicktest
Polyarteritisnodosa(PAN)
Rheumatoidarthritis
SLE
Acuteviralhepatitis
Penicillaminetoxicity
Hyperacutegraftrejection

380.Type4hypersensitivityreactiontoTB
antigenissimilartowhichofthe
following?

a)Serumsickness
b)Asthma
c)Myastheniagravis
d)Temporalarteritis
CorrectAnswer-D
Answer-D.Temporalarteritis
TypeIV(T-Cellmediated)
Tuberculintest
Lepromintest
Sarcoidosis
Tuberculosis
Contactdermatitis
Granulomatousinflammation
TypeIleprareaction
Patchtest
Temporalarteritis
Jonesmotereaction(cutaneousbasophilicHSN)
Graftrejection
Fairleystest
Frie'stest

381.NonIgEmediatedanaphylacticreaction
includes-
a)IgG
b)IgM
c)Compliments
d)Alloftheabove
CorrectAnswer-D
Answer-D.Alloftheabove
Cytotoxicreactionscanalsocauseanaphylaxis,viacomplement
activation.Antibodies(IgGandIgM)againstredbloodcells,as
occursinamismatchedbloodtransfusionreaction,activate
complement.Thisreactioncausesagglutinationandlysisofred
bloodcellsandperturbationofmastcellsresultinginanaphylaxis

382.TrueaboutMHC-classII-
a)Notinvolvedininnateimmunity
b)CytotoxicT-cellinvolved
c)Presentinnucleatedcells
d)All
CorrectAnswer-A
Ans.is'a'i.e.,NOTinvolvedininnateimmunity
oMHCIandIIarepartofadaptiveimmunity.Amajorfunctionofthe
MHCmoleculeistopresenttheantigentoTcellthatispartof
adaptiveimmunity.
CytotoxicTcellsareMHC-classIrestricted.
MHCclassIIcomplexisfoundonlyoncellsoftheimmunesystem
(classIisfoundonallnucleatedcellsandplatelets.)
TheantigensbindingwithMHCIImoleculearepresentedtoCD4+T
cells.Asdiscussedearlier,helperTcells/CD4TcellsareMHCII-
restricted.
BcellshavebothMHCclassIandIIcomplexes.

383.MHC1isinvolvedin?
a)Tumorlysis
b)Mixedleukocytereaction
c)Autoimmunediseasesusceptibility
d)Alloftheabove
CorrectAnswer-A
Answer-A.Tumorlysis
MHCclassImoleculesarepresentonallnucleatedcells,allvirus-
infectedcellscanbedetectedandeliminatedbyCD8+CytotoxicT
lymphocytes.
MHCclassIisresponsibleforgraftrejectionandcell-mediated
cytolysisofviralinfectedortumorcells.

384.CD4isassociatedwithHLA?
a)HLA1
b)HLA2
c)HLA3
d)Alloftheabove
CorrectAnswer-B
Answer-B.HLA2
MHC-IIrecognizesexogenousantigen(extracellularmicrobes,
solubleproteins)andpresentittoCD4helperTcells(MHC-II
restricted).
MHCclassIpresentantigentocytotoxicCD-8Tcells(MHC-I
restricted).

385.Memorycellsare:
September2004
a)Basophils
b)Eosinophils
c)Lymphocytes
d)Neutrophils
CorrectAnswer-C
Ans.Ci.e.Lymphocytes

386.Whichofthefollowinginterleukinis
secretedbyThelper2cells?
a)IL11
b)IL7
c)IL1
d)IL13
CorrectAnswer-D
Ans.is'd'i.e.,IL13
A)Thelper-1(TF,')secretes4IL-2andinterferon-y
B)Thelper-2(T 2

0 )secretes->IL-4,IL-5,IL-6,IL-13

387.AllaretrueaboutTolllikereceptors
except?
a)Recognizemicrobialmolecules
b)Activationofimmunesystem
c)Regulationofcalciumchannel
d)Activationoftranscriptionfactors
CorrectAnswer-C
Answer-C.Regulationofcalciumchannel
Toll-likereceptors(TLRs)aremicrobialsensorsthatarenamedfor
thefoundingmembercalledToll,whichwasdiscovered
inDrosophila.
TherearetenmammalianTLRs,whichrecognizeproductsof
bacteria(suchasendotoxinandbacterialDNA),viruses(suchas
double-strandedRNA),andotherpathogens(Fig.2?3,A).
TLRsarelocatedinplasmamembranesandendosomes,sothey
candetectextracellularandingestedmicrobes.
Theyarecomplementedbycytoplasmicandmembranemolecules,
fromseveralotherfamilies,thatalsorecognizemicrobialproducts.
TLRsandtheotherreceptorsrecognizeproductsofdifferenttypes
ofmicrobesandthusprovidedefenseagainstessentiallyallclasses
ofinfectiouspathogens.
Recognitionofmicrobesbythesereceptorsactivatestranscription
factorsthatstimulatetheproductionofseveralsecretedand
membraneproteins.
Theseproteinsincludemediatorsofinflammation,antiviralcytokines
(interferons),andproteinsthatpromotelymphocyteactivationand
evenmorepotentimmuneresponses.

388.Tolllikereceptorsareseenon?
a)Macrophages
b)Naturalkillercells
c)Endothelialcells
d)Alloftheabove
CorrectAnswer-D
Answer-D.Alloftheabove
TheToll-likereceptorsarernembraneproteinsthatrecognize
avarietyofmicrobe-derivedmoleculesandstimulateinnateimmune
responsesagainstthemicrobes.
TheToll-likereceptorsareexpressedonmanydifferentcelltypes
thatparticipateininnateimmuneresponsesincluding
,macrophages,dendriticcells,neutrophils,NKscells,mucosal
epithelialcellsandendothelialcells.

389.Themostcommonorgantobeinvolved
intheacutephaseofGraftvshost
diseaseis?

a)Bonemarrow
b)Skin
c)Liver
d)Gut
CorrectAnswer-B
Answer-B.Skin
AcuteGraftvshostdisease(occurringdaystoweeksafter
transplantation)causesepithelialcellnecrosisinthreeprincipal
targetorgans:liver,skin,andgut.Destructionofsmallbileducts
givesrisetojaundice,andmucosalulcerationofthegutresultsin
bloodydiarrhea.Cutaneousinvolvementismanifestedbya
generalized(maculopapular)rash.
Histologicalfindingsare-
1. Perivascularmononuclearinfiltrates.
2. Vacuolardegradationofdermo-epidermaljunction.
3. Dyskeratoticcellsoreosinophilicbodiesintheepidermis.
4. Epidermolysis.
5. Denudationofepidermis(separationoftheepidermisfromthe
dermis).

390.NormalvalueofCD4:CD8ratiois-
a)0.5
b)1
c)1.5
d)2
CorrectAnswer-D
Answer-D.2
AnormalCD4/CD8ratiois2.0,withCD4lymphocytesequaltoor
greaterthan400/mm3andCD8lymphocytesequalto200to
800/mm3.

391.Morethan90%patientsofCRESTsyndromewiththelimitedcutaneousform
ofthisdisordermakewhichofthefollowingautoantibodies?
a)Anti-centromere
b)Anti-DNAtopoisomerasel
c)Anti-double-strandedDNA
d)Anti-Golgi
CorrectAnswer-A
Allformsofsclerodermaarethoughttohaveastrongautoimmunecomponent,and
glucocorticoidsandazathioprineareusedtosuppresstheinflammatorycomplicationsof
scleroderma.
(Otherdrugsthatcanbeusedintherapyincludepenicillamine,whichinhibitscollagen
cross-linking,NSAIDSforpain,andACEinhibitorstoprotectthekidneyifhypertensionor
renaldamageoccurs).
Theanti-centromereantibodyisquitespecificforCRESTsyndrome(96%ofcases),
andisonlyseeninaminorityofpatientswithdiffusescleroderma(mainlythosewith
Raynaud'sphenomenon)andrarelyinsystemiclupuserythematosusandmixedconnective
tissuedisease.
Alsoknow:
Anti-DNAtopoisomeraseI,alsocalledanti-Scl-70occurscommonly(64-75%)indiffuse
scleroderma,butonlyrarelyinCRESTsyndrome.
Anti-double-strandedDNAisfairlyspecificforsystemiclupuserythematosus,althoughit
onlyoccursin50-60%oflupuscases.
Anti-GolgiantibodiesareseenmostofteninsystemiclupuserythematosusandSj?gren
syndrome.
Ref:HellmannD.B.,ImbodenJr.J.B.(2013).Chapter20.Musculoskeletal&Immunologic
Disorders.InM.A.Papadakis,S.J.McPhee,M.W.Rabow(Eds),CURRENTMedical
Diagnosis&Treatment2013.

392.AntiJo-1antibodiesareafeature
associatedwith?
a)SLE
b)Systemicsclerosis
c)Polymyositis
d)Rheumatoidarthritis
CorrectAnswer-C
Answer-C.Polymyositis
Anti-Jo-1antibodyisamyositisspecificautoantibodymost
commonlyfoundinpatientswithidiopathicinflammatorymyopathies
(UM).

393.HLAmarkerassociatedwithdiabetes
mellitustype1is-
a)B7
b)DR4
c)DQ3
d)DQ4
CorrectAnswer-B
Answer-B.DR4
HLAassociatedwithDM1areDR3,DR4,DR8,DQ8.

394.Expressionofageneisknownas?
a)Genotype
b)Phenotype
c)Genome
d)Morphology
CorrectAnswer-B
Answer-B.Phenotype
Thegenotype(Thegeneticmakeupofanindividual)ofanorganism
istheinheritedmapitcarrieswithinitsgeneticcode.
Thegenotypeisthepart(DNAsequence)ofthegeneticmakeupof
acell.Genotypeisoneofthreefactorsthatdeterminephenotype,
theothertwobeinginheritedepigeneticfactors,andnon-inherited
environmentalfactors.

395.Group-Dchromosomeis?
a)Chromosome3
b)Chromosome6
c)Chromosome12
d)Chromosome15
CorrectAnswer-D
Answer-D.Chromosome15
13-15-SixchromosomesslightlysmallerthantheCgroupwiththe
kinetochoresinanearterminalposition.

396.Acuteintermittentporphyriais
associatedwithwhichtypeof
inheritance?

a)Autosomaldominant
b)Autosomalrecessive
c)X-linkeddominant
d)X-linkedrecessive
CorrectAnswer-A
Answer-A.Autosomaldominant
Metabolic
Familialhypercho-lesterolemia
Acuteintermittentporphyria

397.Inheritanceassociatedwithcongenital
adrenalhyperplasia-
a)AR
b)AD
c)XR
d)XD
CorrectAnswer-A
Answer-A.AR
Endocrine-
Congenitaladrenalhyperplasia
Albinism

398.InheritanceassociatedwithfragileX-
syndromeis-
a)Autosomaldominant
b)Autosomalrecessive
c)X-linkeddominant
d)X-linkedrecessive
CorrectAnswer-C
Answer-C.X-linkeddominant
'None>c'i.e.,X-linkeddominant
TheunstabletripletistransmittedinanX-linkedinheritancebutthe
probabilitiesofdifferentphenotypesarefarfromtraditionalX-linked
inheritance.

399.HOXgeneisassociatedwith
a)Cranio-caudaldevelopment
b)Braindevelopment
c)Teethformation
d)Alloftheabove
CorrectAnswer-A
Answer-A.Cranio-caudaldevelopment
HOXgenes(asubsetofhomeoticgenes)areagroupofrelated
genesthatcontrolthebodyplanofanembryoalongthe
cranio?caudal(head-tail)axis.

400.ChromosomeforABOgeneis?
a)9q
b)10p
c)11q
d)12p
CorrectAnswer-A
Answer-A.9q
TheABOgene(Histo-bloodgroupABOsystemtransferaseenzyme)
residesonchromosome9attheband9q34.2.
Histo-bloodgroupABOsystemtransferaseis
anenzymewithglycosyltransferaseactivity,whichisencodedby
theABOgeneinhumans.Itisubiquitouslyexpressedinmany
tissuesandcelltypes.

401.Barrbodyisfoundinthefollowingphase
ofthecellcycle:
a)Interphase
b)Metaphase
c)GIphase
d)Telophase
CorrectAnswer-A
Ai.e.Interphase
TheinactiveXcanbeseenintheinterphasenucleusasadarkly
stainingsmallmassincontactwiththenuclearmembraneknownas
theBarrbodyorXchromatin.Barrbodyistheinactivated
X
chromosome.Innondividinginterphasecellsitremainstightlycoiled
andcanbeseenasadarkstainingbodywithinthenucleus.


402.Trinucleotidesequenceassociatedwith
spino-cerebellarataxiais?
a)CTG
b)CUG
c)GGG
d)CAG
CorrectAnswer-D
Answer-D.CAG
Huntington'sdisease(CAGrepeat),
Spinocerebellarataxia(CAGrepeat),
Friedreichataxia(GAArepeat),
Fragile-X-syndrome(GGGorGCCrepeat),
Dystrophiamyotonica(CTG/CUGrepeat)

403.CA-125is?
a)Mucinglycoprotein
b)Carcinomatransmigrationantigen
c)Mucopolysaccharide
d)Lipid
CorrectAnswer-A
Answer-A.Mucinglycoprotein
*CA-125,whichstandsfor"CancerAntigen125"alsoknownas
mucin16orMUC16isaproteinthatinhumansisencodedbythe
MUC16whichisamemberofthemucinfamilyglycoproteins.
*Itisfoundinhighamountsinthebloodofpatientswithovarian
cancer.
*CA-125isproducedonthesurfaceofcellsandisreleasedinthe
bloodstream.
*Itisamongthebloodteststhatmaybeorderedbyadoctorif
ovariancancerissuspected.
*CA-125isalsoelevatedincancersofendometrium,cervix,
fallopiantubes,pancreas,breast,lungandcolon.
*Non-neoplasticconditionscausingelevationofCA-125are
pregnancy,menstruation,endometriosis,PID,abdominalTB,
peritonitisanduterinefibroid.
*referencerange:CA125is0-35units/mL(0-35kU/L).

404.ElevatedCA-125areseenin-
a)AbdominalTB
b)Cacervix
c)Endometriosis
d)All
CorrectAnswer-D
Ans.is'a'i.e.,AbdominalTB;'b'i.e.,Cacervix;'c'i.e.,
Endometriosis
ElevatedlevelofCA-125isseenin
*EpithelialovarianCancer
*Nonovariantumors:-Cancersofendometrium,cervix,fallopian
tubes,pancreas,breast,lungandcolon.
*Nonmalignantconditions:-Pregnancy,mensturation,peritonitis,
endometriosis,pelvicinflammatorydisease(abdominalTB)and
uterinefibroids.

405.Li-fraumanisyndromeoccursdueto
mutationingene?
a)p53
b)p16
c)p41
d)p12
CorrectAnswer-A
Answer-A.p53
Li-Fraumenisyndromeisduetogermlinemutationinp-53gene.

406.Followinggeneticcounsellinginafamily
forFamilialpolyposiscoli(FPC)next
screeningtestis

a)Flexiblesigmoidoscopy
b)Colonoscopy
c)Occultbloodinstools
d)APCgene
CorrectAnswer-D
Ans.is'd'i.e.APCgene
Schwartzwrites-"Flexiblesigmoidoscopyoffirst-degreerelativesof
FAP(Familialadenomatouspolyposis)patientsbeginningatage10
to15yearshasbeenthetraditionalmainstayofscreening.Today
followinggeneticcounselling,APCgenetestingmaybeusedto
screenfamilymembersprovidinganAPCmutationhasbeen
identifiedinafamilymember."
Familialadenomatouspolyposis(FAP)isadominantlyinherited
coloncancersyndromeduetogermlinemutationsinthe
adenomatouspolyposiscoli(APC)tumorsuppressorgeneon
chromosome5.
Patientswiththissyndromedevelophundredstothousandsof
adenomasinthecolonwhichifleftuntreatedwilleventuallydevelop
intocoloncancer.
EachoftheFAPadenomashaslostthenormalremainingalleleof
APCgenebuthasnotyetaccumulatedtherequiredadditional
mutationstogeneratefullymalignantcells.Induecourseoftime
manyoftheseadenomasacquirefurthergeneticabnormalitiesand
developintofullymalignantcancers.APCgeneisthusconsideredto

beagatekeeperforcolontumorigenesis.
ThedetectionofAPCgenemutationinfamilymembersofaFAP
patienthelpsinmakingadefinitivediagnosisbeforethe
developmentofpolyps.


407.Mostcommontumorsuppressorgene
involvedincancergenesisis?
a)Rb
b)p53
c)p16
d)p73
CorrectAnswer-B
Answer-B.p53
p-53isthemostcommontargetforgeneticalterationinhuman
tumors;Alittleover50%ofhumantumorscontainmutationinthis
gene.

408.Allarepre-malignantconditionsexcept-
a)Anaplasia
b)Metaplasia
c)Hamartoma
d)Dysplasia
CorrectAnswer-C
Aprecancerousconditionisaconditionorlesioninvolving
abnormalcellsthatareassociatedwithanincreasedriskof
developingintocancer.Clinically,precancerousconditions
encompassavarietyofconditionsorlesionswithanincreasedrisk
ofdevelopingintocancer.
Dysplasiaisabroadtermthatreferstotheabnormaldevelopment
ofcellswithintissuesororgans.Itcanleadtoawiderangeof
conditionsthatinvolveenlargedtissueorpre-cancerouscells.
Anaplasiaisaconditionofcellswithpoorcellulardifferentiation,
losingthemorphologicalcharacteristicsofmaturecellsandtheir
orientationconcerningeachotherandtoendothelialcells.Theterm
alsoreferstoagroupofmorphologicalchangesinacell
(nuclearpleomorphism,alterednuclear-cytoplasmicratio,presence
ofnucleoli,highproliferationindex)thatpointtoapossiblemalignant
transformation.Suchlossofstructuraldifferentiationisespecially
seeninmost,butnotall,malignantneoplasms.
Ahamartomaisabenign(noncancerous)tumorlikemalformation
madeupofanabnormalmixtureofcellsandtissuesfoundinareas
ofthebodywheregrowthoccurs.Itisconsideredadevelopmental
errorandcanoccuratseveralsites.
consideringtheabovetermshamartomaisnotaprecancerous
condition


409.Softwoodexposureleadsto-
a)Nasaladenocarcinoma
b)Skincancer
c)Livercancer
d)Bladdercancer
CorrectAnswer-A
Answer-A.Nasaladenocarcinoma
"Thereislittledoubtthatexposuretohardwooddusts(especially
thatofbeech6,oak)inthefurnitureindustry,aswellassome
domesticsoftwoodoutsidetheindustry,isassociatedwithsino-
nasalcancers.Amongtheneoplasias,nasaladenocarcinomaisthe
cancermostcommonlyfoundinwoodworkers."

410.Atwhichcellcyclecheckpointisthecell
cyclehaltedifthecell'sDNAis
damaged?

a)GI-S
b)S-G2
c)G2-M
d)GO-GI
CorrectAnswer-A:C
Answer-(A)GI-S(C)G2-M
TheSphaseisthepointofnoreturninthecellcycle,andbeforethe
cellmakesthefinalcommitmenttoreplicate,theG,/Scheckpoint
checksforDNAdamage.IfDNAdamageispresent,theDNArepair
machineryandmechanismsthatarrestthecellcycleareputin
motion.ThiscausesdelayatGiScheckpoint.
ThischeckpointinusedfortherepairoftheDNAwhichisdamaged
afteritsreplication

411.Malignancyassociatedwith
hypercalcemia:
a)Breastcancer
b)Smallcelllungcancer
c)Non-smalllungcancer
d)Prostatecancer
e)Multiplemyeloma
CorrectAnswer-A:C:D:E
Answer-A,BreastcancerC,Non-smalllungcancerD,Prostate
cancerE,Multiplemyeloma
Lungcarcinoma,breastcarcinoma,andmultlplemyelomaaccount
formorethan50%ofallcasesofmalignancy-associated
hypercalcemia.
Gastrointestinaltumarsandprostatecarcinomaarelesscommon
causesofhypercalcemia.

412.AFPisraisedin-
a)Yolksactumor
b)Seminoma
c)Teratoma
d)aandc
CorrectAnswer-D
Ans.is'a'i.e.,Yolksactumor;'c'i.e.,teratoma
AFPisaglycoproteinsynthesizednormallyearlyinfetallifebyyolk
sac,fetalliverandfetalGIT.Itisstructurallyandgeneticallyrelated
toalbumin.
AFPisraisedinlivercancer(hepatocellularcarcinoma),lung
carcinoma,pancreaticcarcinoma,coloncarcinoma,andnon
seminomagermcelltumoroftestis/ovary
(yolksac
tumor/endodermalsinustumor,embryonalcarcinoma,teratoma).

413.Immunohistochemicalmarkerfor
sentinelnodebiopsyforbreast
carcinomais?

a)Cytokeratin
b)Vimentin
c)Calretinin
d)CD45
CorrectAnswer-A
Answer-A.Cytokeratin
Cytokeratin-Carcinoma

414.Scavengerreceptorsarefor?
a)OxidizedLDL
b)ReducedLDL
c)HDL
d)VLDL
CorrectAnswer-A
Answer-A.OxidizedLDL
OxidizedLDLhasfollowingeffects

1. Ingestedbymacrophagesthroughthescavengerreceptor,distinct
fromLDLreceptors,thusformingfoamcells.
2. Increasesmonocyteaccumulationinlesions.
3. Stimulatesreleaseofgrowthfactorsandcytokines.
4. Iscytotoxictosmoothmusclecellsandendothelialcells

415.Causesofconstrictivepericarditisis/are
?
a)T.B.
b)SLE
c)Brucellosis
d)aandb
CorrectAnswer-D
Ans.is'a'i.e.,T.B.;b'i.e.,SLE
Causesofconstrictivepericarditis
*Tuberculosis(mostcommoncause)
*Pericardialmalignancies(especiallybreast,lung)
*Post-viralpericarditis(especiallyhemorrhagic)
*Drugs(e.g.Hydralazine,Procainamide,minoxidil,phenytoin)
*Bacterial(purulent)pericarditis
*Trauma/post-cardiacsurgeryhaemopericardium
*Fungalpericarditis(Histoplasmosis)
*Connectivetissuedisorders(e.g.SLE,RA,Sarcoidosis)
*Parasiticpericarditis(toxoplasmosis,trichonosis)
*Dresslersyndrome
*Mediastinalradiotherapy
*Renalfailure

416.Characteristichistopathologicalfeature
ofrheumaticheartfailureis-
a)Aschoff'snodule
b)McCallumnpatch
c)Bread&butterpericarditis
d)Shaggyvegetation
CorrectAnswer-A
Answer-A.Aschoff'snodule
Microscopically,Aschoffbodiesareareasofinflammationof
theconnectivetissueoftheheart,orfocalinterstitialinflammation.
FullydevelopedAschoffbodiesaregranulomatousstructures
consistingoffibrinoidchange,lymphocyticinfiltration,
occasionalplasmacells,andcharacteristically
abnormalmacrophagessurroundingnecroticcenters.
Someofthesemacrophagesmayfusetoformmultinucleatedgiant
cells.OthersmaybecomeAnitschkowcellsor"caterpillarcells,"so
namedbecauseoftheappearanceoftheirchromatin.
MyocardiuminvolvementresultsinmyocarditiswithAschoffbodies.
Endocardiuminvolvementleadstotheformationofsmallwarty
projections(verrucae)alongthelineofclosureofvalvularleaflets,
mostlyonthemitralvalve.
Theyarepathognomicfocioffibrinoidnecrosisfoundinmanysites,
mostoftenthemyocardium.Initially,theyaresurroundedby
lymphocytes,macrophages,andafewplasmacells,buttheyare
slowlyreplacedbyafibrousscar.Aschoffbodiesarefoundinallthe
threelayersoftheheart,theleastchanceinthepericardium.

417.Prostheticvalvesarepredisposedto-
a)Thromboembolism
b)Infection
c)Hemolysis
d)Alloftheabove
CorrectAnswer-D
Answer-D.Alloftheabove
Thrombo-embolicevents
Bleeding
Prostheticheartvalveobstructionorstenosis
Prostheticheartvalveregurgitation(valvularandparavalvular)
Infectiveendocarditis
Hemolyticanemia
Patient-prosthesismismatch.

418.Mostfrequenttimeperiodbetween
myocardialinfarctionandsubsequent
myocardialruptureis-

a)3-4days
b)4-8days
c)1-3weeks
d)3-6weeks
CorrectAnswer-A
Answer-A.3-4days
Myocardialrupturemayoccuralmostanytimeduring3weeksafter
acutemyocardialinfarction,butismostcommonbetweenthefirst
andfourthdays,whentheinfractedwallisweakest.

419.Infarctsinvolvingwhichportionofthe
myocardiumcauseaneurysmasapost-
MIcomplication-

a)Subendocardial
b)Anteriortransmural
c)Posteriortransmural
d)Inferiorwall
CorrectAnswer-D
Answer.D.Inferiorwall
Leftventricularaneurysmformation:
Leftventricularapicalaneurysmformationusuallyoccursfollowing
antero-apicalmyocardialinfarction,afterLADocclusion.
Thisweakeningoftheapicalwallresultsinanoutpouchingor
"dyskinesis"oftheapexoftheheartduringsystole.

420.Inpatientswithhypertrophic
cardiomyopathymaximummutationsare
foundinwhichgene:

a).-myosinheavychain
b)Elastin
c).-tropomyosin
d)TroponinT
CorrectAnswer-A
-Myosinheavychain
Mutationsingenefor-Mysoinheavychainareassociatedwith
40%ofthefamilieswithhypertrophiccardiomyopathy.
TroponinTmutations-15%ofthefamilies-tropomyosin
mutations~5%ofthefamilies

421.Mutationinproteinassociatedwith
restrictivecardiomyopathy?
a)Myosinregulatoryproteins
b)Myosinbindingprotein-C
c)TroponinI
d)Tropomyosin
CorrectAnswer-C
Answer-C.TroponinI
RCM-associatedmutationshavebeenreportedinfourgenesthat
encodekeysarcomericproteins/myofilaments
1. MYH7gene-13-Myosinheavychain
2. TNNI3gene-CardiactroponinItype3
3. TNNT2gene-CardiactroponinTtype2
4. ACTCgene-a-actin
MutationsintheTNNI3geneareoneofthemajorcausesofthis
condition.TheTNNI3geneprovidesinstructionsformakinga
proteincalledcardiactroponinI,whichisfoundsolelyintheheart.
CardiactroponinIisoneofthreeproteinsthatmakeupthetroponin
proteincomplex,whichhelpsregulatetensing(contraction)and
relaxationoftheheartmuscle.
TNNI3genemutationsassociatedwithfamilialrestrictive
cardiomyopathyresultintheproductionofadefectivecardiac
troponinIprotein.
Thealteredproteindisruptsthefunctionofthetroponinprotein
complexanddoesnotallowtheheartmuscletofullyrelax.Asa
result,notenoughbloodenterstheventricles,leadingtoabuildupin
theatriaandlungs.Theabnormalheartrelaxationandbloodflowis
responsibleformanyofthesignsandsymptomsoffamilial

restrictivecardiomyopathy.

422.Takayasuarteritismainlyaffects?
a)Pulmonaryartery
b)Celiacartery
c)Subclavianartery
d)SMA
CorrectAnswer-C
AnswerisC(SubclavianArtery):
Subclavianarteryisthesinglemostcommonarteryinvolvedin
Takayasuarteritis.
Takayasuarteritis:Mostcommonsitesaffected
Takayasuarteritistypicallyinvolvesmediumandlargesizedarteries
Ithasastrongpredilectionfortheaorticarchanditsbranches.
Theinvolvementofthemajorbranchesoftheaortaismuchmore
markedattheiroriginthandistally
Themostcommonlyaffectedarteriesasseenbyarteriography
inorderoffrequency

Coronary(<10%)
Vertebral(35%)
Coeliacaxis(18%)
Pulmonary(10-40%)
SuperiorMesenteric(18%)
Iliac(17%)
Subclavian(93%)
CommonCarotid(58%)
AbdominalAorta(47%)
Renal(38%)

423.Strawberrygumsareseenin
a)Goodpasteur'ssyndrome
b)Classicpolyarteritisnodosa
c)Wegener'sgranulomatosis
d)Kawasakisyndrome
CorrectAnswer-C
Answer-C.Wegener'sgranulomatosis
StrawberrygumsareseeninWegner'sgranulomatosis.
StrawberrytongueisseeninKawasakidisease.

424.Strawberrygingivitisseenin
a)Myelocyticinfiltration
b)Phenytointoxicity
c)Wegnergranulomatosis
d)Klipelrenaunaysyndrome
CorrectAnswer-C
Wegener'sgranulomatosis[Ref:IADVLTextbookofDermatology
3r9ep.695]
Wegener'sgranulomatosisisknowntocauseoralmucosallesions.
"Strawberrygingivitisischaracteristicorallesionassociatedwith
wegener'sgranulomatosis".
Straberrygingivitisclinicallypresentswith:?
'Swollenerythematousgumsclinicallyresemblingoverripe
strawberries"
Histologicallystrawberrygingivitisischaracterizedby

425.Heinzbodiesareremovedby?
a)Macrophages
b)Lymphocytes
c)Neutrophils
d)Fibroblasts
CorrectAnswer-A
Answer-A.Macrophages
*Theunstablehemoglobinsarethosehemoglobinvariantsthat
undergodenaturationandprecipitationwithintheredcellsasHeinz
bodies.
*Theygiverisetowhatisknownascongenitalnon-spherocytic
hemolyticanemiaorcongenitalHeinzbodyhemolyticanemia.
*Thesedisordershaveeitherautosomaldominantinheritanceor
developfromspontaneousmutations.
*Theunstablehemoglobinsarisefromeitherasingleaminoacid
substitutionintheglobinchainorduetodeletionofoneormore
aminoacidswithinthe-globinchainsothatthefirmbondingofthe
haemgroupwithinthemoleculeisdisturbedleadingtotheformation
ofmethemoglobinandprecipitationofglobinchainsasHeinz
bodies.
*Heinzbodiesarenotseenafterthefirstoneortwodayssincethey
areremovedbythespleen,leadingtotheformationof`bitecells'
andfragmentedredcells.
*Macrophagesdetecttheantigenandremovethedamaged
portionsofthecell,itsdamagedmembrane,andthedenatured
hemoglobin.
*Theseareassociatedclinicallywithcongenitalhemolyticanemia,
G6PDdeficiency,hemolyticanemiasecondarytodrugssuchas

phenacetin,somehemoglobinopathies(thalassemia),andafter
splenectomy.

426.NotafeatureofG-6PDdeficiency?
a)Intravascularhemolysis
b)Oxidativestress
c)Membranedefect
d)Bitecells
CorrectAnswer-C
Answer-C.Membranedefect
*Amongthedefectsinhexosemonophosphateshunt,themost
commonisG6PDdeficiency.
*G6PDgeneislocatedontheXchromosomeanditsdeficiencyis,
therefore,asex(X)-linkedtraitaffectingmales,whilethefemales
arecarriersandareasymptomatic.
*PATHOGENESIS:Normally,redbloodcellsarewellprotected
againstoxidantstressbecauseofanadequategenerationof
reducedglutathioneviathehexosemonophosphateshunt.
*IndividualswithinheriteddeficiencyofG6PD,anenzymerequired
forhexosemonophosphateshuntforglucosemetabolism,failto
developadequatelevelsofreducedglutathioneintheirredcells.
*Theclinicalmanifestationsarethoseofacutehemolyticanemia
withinhoursofexposuretooxidantstress.
*Thehemolysisis,however,self-limitingeveniftheexposuretothe
oxidantiscontinuedsinceitaffectstheolderredcellsonly.

427.Gallstonesinhemolyticanaemiaare-
a)Pigment
b)Mixed
c)Cholesterol
d)Anytype
CorrectAnswer-A
Ans.is'a'i.e.,Pigment

428.Hypercoagulabilityduetodefective
factorVgeneiscalled:
a)Lisbonmutation
b)Leidenmutation
c)Antiphospholipidsyndrome
d)Induciblethrombocytopeniasyndrome
CorrectAnswer-B
AnswerisB(Leidenmutation)
HypercoagulabilityduetodefectiveFactorVgeneiscalled'Leiden
mutation'andisnamedafterthecityinwhichitwasdescribed.
FactorVLeiden
FactorVLeidenisavariant(mutated)ofnormalclottingfactorVand
differsfromnormalclottingfactorVbyasinglenucleotide.
WhileFactorVLeideniscompletelynormalintermsofitsabilityto
preventbleeding,theoneaminoaciddifferencemakesFactorV
LeidenresistanttobeingdegradedbyactivatedproteinCand
proteinS.
ConsequentlyfactorVLeidenpersistsinthecirculationlongerand
contributestoformationofbloodclots.
FactorVLeidenmutationisthemostcommonunderlyinggenetic
causeofthrombophilia(venousthrombosis)
FactorVLeidenmutationposesalifelongriskofdeepvenous
thrombosis.

429.Cryoprecipitatecontains:
March2009
a)FactorII
b)FactorV
c)FactorVIII
d)FactorIX
CorrectAnswer-C
Ans.C:FactorVIII
Cryoprecipitateispreparedfromplasmaandcontainsfibrinogen,
vonWillebrandfactor,factorVIII,factorXIIIandfibronectin.
Cryoprecipitateistheonlyadequatefibrinogenconcentrateavailable
forintravenoususe.
Cryoprecipitateisindicatedforbleedingorimmediatelypriortoan
invasiveprocedureinpatientswithsignificanthypofibrinogenemia(
CryoprecipitateshouldnotbeusedforpatientswithvonWillebrand
diseaseorHemophiliaA(FactorVIIIdeficiency).Itisnotusually
givenforFactorXIIIdeficiency,astherearevirus-inactivated
concentratesofthisproteinavailable.Cryoprecipitateissometimes
usefulifplateletdysfunctionassociatedwithrenalfailuredoesnot
respondtodialysis.

430.Whichofthefollowingisapro-
coagulant?
a)Thrombomodulin
b)ProteinC
c)ProteinS
d)Thrombin
CorrectAnswer-D
Answer-D.Thrombin
VariousfactorsinvolvedincoagulationcascodesarefactorI
(fibrinogen);factorII(prothrombin);factorIII(thromboplastinor
tissuefactor);factorIV(calciumionsorCa");factorV(labilefactor);
factorVI(accclerin);factorVII(stablefactororpreaccelerin);factor
VIII(antihemophilicfactor);factorIX(christmasfactor);factorX
(stuart-prowerfactor);factorXI(plasmathromboplastinantecedent
orPTA);factorXII(Hagemanfactor);factorXIII(fibrinstabilizing
factororLakiLorandfactor);HMWK(highmolecularweight
kinogen);prekallikrein;kallikrein;andplateletphospholipids.

431.Whichtestisusedforbothintrinsicand
commonclottingpathways?
a)Thrombintime
b)Partialthromboplastintime
c)Ristocetinagglutinationtest
d)FDPs
CorrectAnswer-B
Answer-B.Partialthromboplastintime
1)Partialthromboplastintime(PTT):Itteststheintrinsicand
commoncoagulationpathways.So,aprolongedPTTcanresults
fromdeficiencyoffactorV,VIII(factorVIIIc,Vonwillebrandfactor),
IX,X,XI,XII,prothrombinorfibrinogen.

432.MaximumlifeoftransfusedRBCsis
a)110-120days
b)80-100days
c)60-80days
d)50-60days
CorrectAnswer-B
Answer-B.80-100days
Normalaveragelifespanofredbloodcellsinadultsisabout120
days,whereasthatoftransfusedRBCsisabout50-60days"
AfteranRBCtransfusion,themeanpotentiallifespanofRBCis85
dayswithameanhalflifeof43?11days.

433.
HemolyticUremicSyndromeis
characterizedby?
a)Microangiopathichaemolyticanaemia
b)PositiveCoomb'stest
c)Thrombocytopenia
d)aandc
CorrectAnswer-D
Ansis'a'i.e.,Microangiopathichemolyticanemia;'c'i.e.,
Thromhocytopenia
Hemolyticuremicsyndrome
*Hemolyticuremicsyndromeischaracterizedbythetriadof:
-Anemia(microangiopathichemolyticanemia).
-Renalfailure(microangiopathyofkidneyinvolvingglomerular
capillariesandarterioles).
-Thrombocytopenia(duetoplateletconsumption).
*Hemolyticuremicsyndromeismostcommonischildrenunder2
yearsofage.
*Itusuallyfollowsanepisodeofacutegastroenterities,often
triggeredbyEcoli.
*Theprodromeisusuallyofabdominalpain,diarrhoeaand
vomiting.
*Shortlythereafter,signsandsymptomsofacutehemolyticanemia,
thrombocytopeniaandacuterenalfailureensue.oSometimes
neurologicalfindingsalsooccur.(Butusuallyabsentand
differentiateHUSfromTTP.)oRarelyHUSmayfollowrespiratory
tractinfection.
Etiology
*Gastrointestinalinfectionininfantswiththefollowingorganism:

*Ecoli,Shigelladysenteriae,Streptococouuspneumoniae
Hematologicalfindingsinacaseofmicroangiopathichemolytic
anemia:
*Presenceofschistocytes(fragmentedredcells).Thisisthe
hallmarkofmicroangiopathichemolyticanemia.oNeutrophil
leukocytosis.
*Thrombocytopenia.
*Hemoglobinuriaismildtomoderatewithhemosiderinuria.
*Bloodureaandserumcreatininelevelsarehigh.
*PTandAPTTnormal.
*ElevatedserumLDH

434.Allaretrueaboutthrombotic
thrombocytopenicpurpuraexcept?
a)Microangiopathichemolyticanemia
b)Thrombocytopenia
c)Normalcomplementlevel
d)Grosslyabnormalcoagulationtests
CorrectAnswer-D
Ans:D.Grosslyabnormalcoagulationtest[Ref:Robbin's7m/e
p.1010]
*Thromboticthrombocytopenicpurpura(TTP)ischaracterizedby
thepresenceofwidespreadthrombosisthroughoutthe
microcirculation.
*Theuniquecharacteristicofthesethrombiisthattheyareprimarily
composedofplateletsowithverylittle.fibrininit.
*Theconsumptionofplateletleadstothrombocytopeniaandthe
presenceofintravascularthrombicausesmicroangiopathic
hemolyticanemiaQandwidespreadorgandysfunction.
*YoumusthavenoticedthatTTPandDICsharesomanysimilar
featuressuchas4icrovascularocculsionandmicroangiopathic
hemolyticanemia,
Buttheyarepathologicallydistinct.InTTP
*Activationofcoagulationcascadeisnotofprimaryimportance.The
thrombipresentareessentiallyplateletthrombi,henceresultsof
laboratorytestsofcoagulationsuchasPTandPTTareusually
normal.
InD.I.C.
*ThemaindefectisactivationofcoagulationsystemthereforePT

andPTTareabnormal.
*PatientswithTTParedeficientinanenzymecalledADAMTSB.
ThisenzymeisalsoknownasvWfinetalloprotease.
*Thisenzymenormallydegradesveryhighmolecularweight
multimersofvon-Willebrand.factor.Intheabsenceofthisenzyme,
thesemultimersofvWFaccumulateinplasmaandundersome
circumstancespromoteplateletmacroaggregate.formation
throughoutthemicrocirculationleadingtosymptomsofTTP.
*Thromboticthrombocytopenicpurpuraisathrombotic
microangiopathy.
*TheotherdisorderswhichisincludedinthisgroupincludeH.U.S.
*Thediseasesarecharacterizedbywidespreadthrombosisin
microcirculation.
*ThevesselswhichareinvolvedareterminalarteriolesQand
capillaries.
*Thesevesselshavethrombi.
*Thecharacteristicofthisthrombusisthatitisprimarilyaplatelet
thrombusQi.e.thethrombiiscomposedmainlyofplateletsandvon
Wilebrandfactorwithverylittlefibrininit.
Pathogenesis
*PatientswithTTParedeficientinanenzymecalledADAMTSBQ.
(ThisenzymeisalsoknownasvWfnielalloproteaSe.)
*Thisenzymenormallydegradesveryhighmolecularweight
multimersofvon-Willebrand.factor.Intheabsenceofthisenzyme,
thesemultimersofvWFaccumulateinplasmaandundersome
circumstancespromoteplateletmacroaggregateformation
throughoutthemicrocirculationleadingtosymptomsofTTP.
*Plateletassociatedimmunoglobulin(IgG)andcomplementlevels
arenormalinT.T.P.

435.LeftsideshiftinArneth'scountisseen
in?
a)Megaloblasticanemia
b)Septicemia
c)TB
d)Liverdisease
CorrectAnswer-C
Answer-C.TB
Shifttoleft(hyperactivebonemarrow)
Individualswhohavealargerpercentageofneutrophillswithfewer
lobes(mainlyNI&N2)havealeftshiftwhichcanbeindicativeof
diseaseprocessessuchasinfections(pyogenicandTB),malignant
tumors,hemolyticcrises,myocardialinfarction,acidosis,irradiation
etc.

436.Mostcommonheavychaindiseaseis
a)Franklindisease
b)Seligmanndisease
c)Muheavychaindisease
d)Waldenstromcryoglobulinemia
CorrectAnswer-B
Ans.is'b'i.e.,SeligmannDisease(Alphaheavychaindisease)
Therearefourforms:
*Alphachaindisease(Seligmann'sdisease)-mostcommontype
*Gammachaindisease(Franklin'sdisease)
*Muchaindisease
*Deltachaindisease

437.Dutcherbodiesareseenin?
a)Brain
b)Liver
c)Spleen
d)Bonemarrow
CorrectAnswer-D
Answer-D.Bonemarrow
Dutcherbodies,whichareattributedtoimmunoglobulinfilled
cytoplasminvaginatingintothenucleuscreatingtheappearanceof
anintranuclearinclusion.
Dutcherbodiesaredescribedasintranuclearinclusionsinpatients
withWaldenstrom'smacroglobulinemia.
Theinclusionsarepositiveonaperiodicacid-Schiffreactionand
werepresentinthecytoplasmaswellasthenucleus.
Theyidentifiedtheinclusionsasglycoproteinandpostulatedthat
theymightbechemicallyidenticaltothemacroglobulininthe
plasma.

438.Gammagandybodiesareseeninall
except?
a)Cirrhosiswithportalhypertension
b)Chronicmyeloidleukemia
c)Sicklecellanemia
d)Thalassemia
CorrectAnswer-D
Ans.is'd'i.e.,Thalassemia
Gandy-Gammabodiesarefocioffibrosiscontainingironand
calciumsaltsdepositsonconnectivetissueandelasticfibres.
Gandy-Gammabodiesareseenincongestivesplenomegalye.g.,
insicklecellanemia,CMLandlivercirrhosis.


439.Allofthefollowingstatementsabout
Burkitt'slymphomaaretrue,Except:
a)Bcelllymphoma
b)8,14translocation
c)Canpresentasanabdominalmass
d)Radiotherapyisthetreatmentofchoice
CorrectAnswer-D
AnswerisD(Radiotherapyisthetreatmentofchoice)
ThetreatmentofchoiceforBurkitt'sLymphomaischemotherapy
andnotRadiotherapy.
'Burkitt'slymphomarespondswelltoshorttermhighdose
chemotherapy.TreatmentofBurkitt'slymphomainbothchildrenand
adultsshouldbeginwithin48hoursofdiagnosisandinvolvesthe
useofintensivecombinationchemotherapyregimens'?Harrisons
Burkitt'sLymphomaisaBcelllymphoma
Burkitt'sLymphomaaretumorsofmatureBcells?Robbins7th/677
t(8;14)translocationisthemostcommontranslocationinBurkitt's
LymphomaBurkitt'slymphomasareassociatedwithtranslocationof
thec-MYCgeneonchromosome8.Translocation
t(8;14)
t(8;22)
t(2;8)
Burkitt'sLymphomamaypresentwithanabdominalmass
MostBurkitt'sLymphomapresentsatextranodalsitesbutmay
presentwithlymphadenopathy

Extranodalsitesofinvolvementincludethemandibleandabdominal
viscera
`Burkitt'sLymphomasmaypresentwithperipherallymphadenopathy

oranintraabdominalmass'?Harrison
CNSinvolvementisfrequent
Thediseaseisrapidlyprogressiveandhasapropensityto
metastasizetoCNS,prophylactictherapytoCNSistherefore
mandatory.
Burkitt'sLymphomaisthemostrapidlyprogressivehumantumor

440.Oncogeneassociatedwithburkitt'slymphomais:
a)BCL-1,IgH
b)BCL-2,IgH
c)C-MYC
d)ALK
CorrectAnswer-C
Disease CytogeneticAbnormality Oncogene
Mantlecelllymphoma
t(11;14)(q13;q32)
BCL-1,IgH
Follicularlymphoma
t(14;18)(q32;q21)
BCL-2,IgH
Diffuselargecelllymphoma
t(3;-)(q27;-)
BCL-6
t(17;-)(p13;-)
p53
Burkitt'slymphoma,Burkitt's
t(8;-)(q24;-)
C-MYC
leukemia
CD30+Anaplasticlargecell
t(2;5)(p23;q35)
ALK
lymphoma
Lymphoplasmacytoidlymphoma
t(9;14)(p13;q32)
PAX5,IgH
Ref:Harrison,E-18,P-921

441.CagAgeneisassociatedwith
a)Hepatocellularcarcinoma
b)Esophagealcarcinoma
c)MALTlymphoma
d)Lungcarcinoma
CorrectAnswer-C
Answer-C.MALTlymphoma
CagA,Agenethatcodesforanimmunodominantantigen,ispresent
onlyinHelicobacterpyloristrainsthatareassociatedwithsevere
formsofthegastroduodenaldisease(typeIstrains).
Helicobacterpyloriisamicroaerophilicspiral-shapedlophotrichous
Gram-negativebacteriumthatcolonizesthegastriclumenof
primates,includinghumans.
Hpyloriwasidentifiedasthecauseofchronicactivegastritisand
pepticulcerdiseaseinhumansandisconsideredtobeariskfactor
forthedevelopmentofgastricadenocarcinomaandMALT
lymphoma

442.Richter'ssyndromereferstowhichofthefollowingmalignant
transformation?
a)CLLevolvingintoaggressivelymphoma
b)HairycellleukemiaevolvingtoAML
c)BlastcrisisinCML
d)SplenicinfiltrationinNHL
CorrectAnswer-A
Ans.A.CLLevolvingintoaggressivelymphoma
Richter'stransformationorRichter'ssyndrome
isacomplication
ofBcellchroniclymphocyticleukemia(CLL)orhairycellleukemia
(HCL)inwhichtheleukemiachangesintoafast-growingdiffuse
largeBcelllymphoma.

443.Basophillicleucocytosisoccursin-
a)AML
b)ALL
c)CML
d)CLL
CorrectAnswer-C
Ans.is'c'i.e.,CML

444.TEL-AML1fusionisassociatedwith?
a)CLL
b)CML
c)ALL
d)AML
CorrectAnswer-C
Answer-C.ALL
TEL-AML1genefusionisthemostcommongeneticalterationin
childhoodacutelymphoblasticleukemia.Itiscreatedbybalanced
translocationt(12:21).
CytogeneticabnormalitiesassociatedwithALLaregainoffunction
mutationinNOTCH-1geneinT-cellALL.

445.Granulomatouslungdiseaseiscaused
by?
a)Hypersensitivitypneumonitis
b)Sarcoma
c)Bronchogeniccarcinoma
d)Bronchogeniccyst
CorrectAnswer-A
Answer-A.Hypersensitivitypneumonitis
Granulomatouslungdiseasereferstoabroadgroupofinfectious
andwellasnoninfectiousconditionscharacterizedbytheformation
ofgranulomas.
Infectious:Mycobacterial(TB&Non-TB);Fungalinfection
(cryptococcus,Histoplasma,Blastomycosis,Coccidioidomycosis,
Aspergillosis);ParasiticInfections.
Non-infectious:Sarcoidosis;Wegnergranulomatosis;ChurgStrauss
syndrome;Chronicgranulomatousdisease;Hypersensitivity
pneumonitis;RA;ChronicAspirationPneumonitis;LangerhansCell
Histiocytosis;substances(TalcGranulomatosis,Berylliosis)

446.Terminalstageofpneumoniais
a)Congestion
b)Redhepatization
c)Grayhepatization
d)Resolution
CorrectAnswer-D
Ans.is'd'i.e.,Resolution
Intheusualcourseofpneumonia,finalstageisresolution.
However,insomeneglectedcasesfollowingcomplicationsmay
develop-
AbscessformationPleuraleffusion,pleuritis
EmpyemaBecteremicdisseminationBrainabscess,
endocarditis,meningitis,
Organizationsupurrativearthritis.
Pathologicalchangesofbacterialpneumonia
A.Lobarpneumonia
Largeconfluentareaofthelungorentirelobesareconsolidated.
Thelowerlobesareaffectedmostcommonly.
Therearefourstagesoftheinflammatoryresponse(Laennec's
stages)?
1.Stageofcongestion(initialphase)
Theaffectedlobeisenlarged,heavy,darkredandcongested.
Cutsurfaceexudesblood-stainedfrothyfluid.
Thereisdilatationandcongestionofalveolarcapillaries.
Therearefewneutrophilsandnumerousbacteriainthealveolar
fluid.
2.Stageofredhepatization(earlyconsolidation)
Thetermhepatizationreferstoliver-likeconsistencyoftheaffected

lobeoncutsection.
Theaffectedlobeisredandfirm.
Theedemafluidofprecedingstageisreplacedbystrandsoffibrin.
Thereismarkedcellularexudateofneutrophilswithextravasationof
redcells.
3.Stageofgrayhepatizatioa(lateconsolidation)
Theaffectedlobeisgrayishbrown,firmanddry.
Thefibrinstrandaredenseandmorenumerous.
Thereisprogressivedisintigrationofredcellsandneutrophils.
Themacrophagesbegintoappearintheexudate.
Theorganismsarelessnumerousandappearasdegenerated
forms.
4.Resolution
Thepreviouslysolidandfibrinousconstituentisliquefiedby
enzymaticaction.
Granularandfragmentedstrandsoffibrininthealveolarspacesare
seenduetoprogressiveenzymaticdigestion.
Thereisprogressiveremovaloffluidcontentaswellascellular
exudatefromtheairspaces,resultinginrestorationofnormallung
parenchymawithareation.
B.Bronchopneumonia
Patchyareasofredorgreyconsolidation,moreoftenmultilobarand
frequentlybilateralandbasal(lowerzones)becauseoftendencyof
secretionstogravitateintolowerlobes..
Thereissuppurativeexudate,consistingchieflyneutrophils,filling
bronchi,bronchiolesandadjacentalveolarspaces.
Alveolarseptathickenduetocongestedcapillariesandleucocytic
infiltration.

447.Pinkpuffersareassociatedwith?
a)Emphysema
b)Chronicbronchitis
c)Pneumonia
d)Bronchiectasis
CorrectAnswer-A
Answer-A.Emphysema
A"pinkpuffer"isapersonwhereemphysemaistheprimary
underlyingpathology
Inemphysema,notonlythereislesssurfaceareaforgasexchange,
thereisalsolessvascularbedforgasexchange..Thebodythen
hastocompensatebyhyperventilation(the"puffer").
Havelesshypoxemia(comparedtobluebloaters)andappearto
havea"pink"complexionandhence"pinkpuffer".

448.Mostcommoncancerfoundincoalmine
workers
a)Analcanal
b)Testicular
c)Lung
d)Liver
CorrectAnswer-C
Answer-C.Lung
Amongcoalworkerpneumoconiosispatientsstandardincidence
ratioofvariousimportantcarcinomas:ovary(2.0)>esophagus
(1.76)Lung>(1.45)>Stomach(1.42)>Liver(1.18)>kidney(1.06)
>prostate(1.02)>colorectal(1.00)>urinarybladder(0.91)>head
&neck(0.87)

449.Inallergicbronchopulmonary
aspergillosis,thelocationoflesionsis
a)Pleura
b)Bronchiandbronchioles
c)Alveoli
d)Alloftheabove
CorrectAnswer-B
Answer-B.Bronchiandbronchioles
Allergicbronchopulmonaryaspergillosis(ABPA)isacondition
characterizedbyahypersensitivityresponsetotheAspergillus(most
commonlyAspergillusfumigatus).
Anallergicbronchopulmonaryaspergillosisisaformoflungdisease
thatoccursinsomepeoplewhoareallergictoAspergillus.
WithABPA,thisallergicreactioncausestheimmunesystemto
overreacttoAspergillusleadingtolunginflammation.
ABPAcausesbronchospasm(tighteningofairwaymuscles)and
mucusbuildupresultingincoughing,breathingdifficultyandairway
obstruction.
italsocausesairwayinflammation,leadingtobronchiectasis--a
conditionmarkedbyabnormaldilationofthebronchiand
bronchioles.

450.Deficiencyofp53mutationismaximum
inwhichlungcarcinoma?
a)Smallcellcarcinoma
b)Squamouscellcarcinoma
c)Adenocarcinoma
d)Lymphoma
CorrectAnswer-B
Answer-B.Squamouscellcarcinoma
-MYCandRB-smallcellcancer
RASandp16-non-smallcellcancer
p53-bothsmallandnon-smallcell,buthighestinsquamouscell
carcinoma
K-RAS?adenocarcinoma

451.Metastasistolungscomemost
commonlyfrom
a)Breastcarcinoma
b)Braincarcinoma
c)Bonecarcinoma
d)Melanoma
CorrectAnswer-A
Answer-A.Breastcarcinoma
Metastatictumortolungoriginatefromcarcinomainthebreast,
colon,stomach,pancreas,kidney,prostate,liver,thyroid,adrenal,
malegenitaltractandfemalegenitaltract.

452.Whichtypeofparaneoplasticsyndrome
ismostcommonlyassociatedwithlung
carcinoma?

a)SIADH
b)Gynaecomastia
c)Acanthosisnigricans
d)Hypocalcaemia
CorrectAnswer-A
Answer-A.SIADH
SIADHisthemostcommonparaneoplasticsyndromeassociated
withlungcarcinomaasSmallcellcarcinomaisthemostcommon
typeoflungcancerassociatedwithectopichormoneproductionand
paraneoplasticsyndromeandADH(CausingSIADH),ACTH
(Causingcushingsyndrome)productionarepredominantly
associatedsmallcellcarcinoma.

453.Mostcommoncauseofchylothoraxis?
a)Trauma
b)Lymphoma
c)Left-sidedheartfailure
d)Infections
CorrectAnswer-B
Answer-B.Lymphoma
Chylothoraxisapleuralcollectionofamilkylymphaticfluid
containingmicroglobulesoflipid.
Itresultsfromlymphformedinthedigestivesystemcalledchyle
accumulatinginthepleuralcavityduetoeitherdisruptionor
obstructionofthethoracicduct.
Thetotalvolumeoffluidmaynotbelarge,butchylothoraxisalways
significantbecauseitimpliesobstructionofthemajorlymphducts,
usuallybyintrathoraciccancerlikeprimaryorsecondarymediastinal
neoplasm,suchaslymphoma.

454.TrueStatementaboutsilicosis-
a)Producespleuralplaque
b)Associatedwithtuberculosis
c)Lowerlobeinfiltration
d)All
CorrectAnswer-B
Ans.is'b'i.e.,Associatedwithtuberculosis
Silicosis
3Silicosisisalungdiseasecausedbyinhalationofcrystalline
silicondioxide(silica).
*Currently,itisthemostprevalentoccupationaldiseaseinthe
world.
*Silicosisisaslowlyprogressivedisease,usuallypresentingafter
decadesofexposureasslowlyprogressivenodularfibrosing
pneumoconiosis.
*Silicaoccursinbothcrystallineandamorphousformsbut
crystallineformsaremuchmorefibrogenic.Thecrystallineformare
quartz,crystobalite.
*Silicosisischaracterizedinitsearlystagesbynodulesintheupper
zonesofthelung.Asthediseaseprogressesthesenodulesmay
coalesceintohardcollagenousscars.
*Fibroticlesionsoccurinthehilarlymphnodeandpleura.
Sometimesthinsheetsofcalcificationoccurinthelymphnodesand
areseenradiographicallyaseegshellcalcificationi.e.(calcium
surroundingazonelackingcalcification.
*Itthediseasecontinuestoprogress,expansionandcoalescence
oflesionsproduceprogressivemassivefibrosis.oHistologicallythe
lesionsofsilicosisconsistsofconcentriclayersofhyalinized

collagensurroundedbyadensecapsuleofmorecondensed
collagen.
*Examinationofthenodulesbypolarizedmicroscopyrevealsthe
birefringentsilicaparticles.
*SilicosisisassociatedwithanincreasedsusceptibilitytoT.B.Itis
postulatedthatsilicosisresultsindepressionofcellmediated
immunityandthecrystallinesilicamayinhibittheabilityof
pulmonarymacrophagestokillphagocytosedmycobacteria.
*Nodulesofsilicotuberculosisoftendisplayacentralzoneof
caseation.

455.Mostcommontypeofpneumoconiosis,
associatedwithTBis?
a)Silicosis
b)Bysinosis
c)Asbestosis
d)Baggassosis
CorrectAnswer-A
Answer-A.Silicosis
SilicosisisassociatedwithincreasedsusceptibilitytoT.B.
Itispostulatedthatsilicosisresultsinadepressionofcell-mediated
immunity,andcrystallinesilicamayinhibittheabilityofpulmonary
macrophagestokillphagocytosedmycobacteria.
Nodulesofsilicotuberculosisoftencontainacentralzoneof
caseation.

456.Onpolarisingmicroscopy,refractile
bodyinthecentreofgranulomaisseen
in?

a)Sarcoidosis
b)Silicosis
c)Tuberculosis
d)Asbestosis
CorrectAnswer-B
Answer-B.Silicosis
Histologicallythelesionsofsilicosisconsistofconcentriclayersof
hyalinizedcollagensurroundedbyadensecapsuleofmore
condensedcollagen.Examinationofthenodulesbypolarized
microscopyrevealsthebirefringentsilicaparticles.

457.Anthracosisisduetoinhalationof-
a)Coaldust
b)Asbestos
c)Silicadust
d)Beryliumdust
CorrectAnswer-A
Ans.is'a'i.e.,Coaldust
Coalworker'spneumoconiosis(Blacklung)
Blacklungdisease,alsoknownascoalworker'spneumoconiosis
(CWP),iscausedbylongexposuretocoaldust.oCoalworker's
pneumoconiosishasfollowingspectrum.
1.Asymptomaticanthracosis
*Inhaledcarbanpigmentisengulfedbyalveolarorinterstitial
macrophages,whichthenaccumulateintheconnectivetissuealong
thelymphatics.
*Patientiscompletelyasymptomatic.
2.Simplecoalworker'spneumoconiosis
*Characterizedbycoalmacules(composedofcarbonladen
macrophages)andcoalnodules(carbonladenmacrophages+
collegen).
*Theupperlobesandupperzonesofthelowerlobesareinvolved.
*Theyarelocatedprimarilyadjacenttorespiratorybronchioles,the
siteofinitialdustaccumulation.
*Centriacinaremphysemamaydevelop.
3.Complicatedcoalworker'spneumoconiosis
*Requiresmanyyearstodevelop.
*Characterizedbyintenselyblackenedmultiplescarlargerthan2
cms.oThisisalsoknownasProgressivemassivefibrosis.

Remember
*InCWPandsilicosisupperzonesoflungareinvolvedinitially.oIn
asbestosislowerlungfieldsareinvolvedinitially.

458.Chronicbronchitisisassociatedwith?
a)Increaseinnumberofglandswithoutanychangeinsize
b)Bronchiolitisobliterans
c)DecreaseinReidindex
d)Onlylargeairwaysareinvolved
CorrectAnswer-B
Answer-B.Bronchiolitisobliterans
Theearliestfeatureofchronicbronchitisishypersecretionofmucus
inthelargerairways,associatedwithhypertrophyofsubmucosal
glandoftracheaandbronchi.
Later,smallairways(smallbronchiandbronchioles)arealso
involvedandshowgobletcellhyperplasia,andintraluminaland
peribronchialfibrosis.
Inmostseverecases,theremaybeoblitrationoflumendueto
fibrosisBronchiolitisobliterans.

459.Histopathologyshowinglargecellswith
plantlikeapperancewithperinuclear
haloisseeninwhichtypeofrenalcell
carcinoma?

a)Onchocytoma
b)Granularcellcarcinoma
c)Angiosarcoma
d)Chromophobic
CorrectAnswer-D
Ans.is'd'i.e.,Chromophobic
Histopathologicalfindingsofperinuclearhaloandplantcell
appearanceareseeninchromophobecellcarcinomaofRCC.
Electronmicroscopicfindingconsistingofnumerous150-300nm
microvesiclesisthesinglemostdistinctiveanddefiningfeatureof
chromophobecellca.

460.TrueaboutRCCis?
a)Mostcommonsiteislowerlobeofkidney
b)Mostcommonvarietyispapillarytype
c)Invasionofrenalveinismorecommonthanrenalartery
d)Mostcommonsiteofmetastasisislymphnodes
CorrectAnswer-C
Answer-C.Invasionofrenalveinismorecommonthanrenal
artery
Renalcellcarcinomaisthemostcommonmalignanttumorof
kidney.Itoccursusuallyin6thand7thdecadeoflife.Thereismale
preponderance.
ImportantriskfactorsforRCCaresmoking(mostsignificant),
obesity,hypertension,asbestosexposure,estrogentherapy,CRF,
tuberoussclerosisandfamilialconditions(VonHippel-Lindau
syndrome).
RCCisanadenocarcinomaandmostcommonlyarisesfromupper
pole.
TheoneofthestrikingcharacteristicsofRCCistoinvaderenalvein
andisassociatedwithpoorprognosis.

461.Strawberrygallbladderisseenin
a)Cholesterosis
b)Primarysclerosingcholangitis
c)Cholestasis
d)Primarybilliarycirrhosis
CorrectAnswer-A
Answer-A.Cholesterosis
InCholesterosis,therearedepositsofcholesterolintheepithelial
cellsandinthesubmucosalmacrophages.Thisgivesappearanceof
multiplesmallyellowspotsonaredhyperplasticmuscosa,the
straberrygallbladder.

462.Whatisresponsibleforregenerationof
livercells?
a)HGF
b)VEGF
c)TGF-13
d)IFN-y
CorrectAnswer-A
Answer-A.HGF
Hepatocytegrowthfactor(HGF/scatterfactor)levelsrisetohigh
levelssoonafterpartialhepatectomy.Thisistheonlyfactortested
thatactsbyitselfasapotentmitogenforisolatedhepatocytes
culturedinvitro.Thisfactorisalsoofcriticalimportancein
developmentoftheliver,astargetdeletionsofitsgeneleadtofetal
deathduetohepaticinsufficiency.

463.Bridgingnecrosisischaracteristicallyseenin:
a)Acutehepatitis
b)Chronichepatitis
c)Bothoftheabove
d)Noneoftheabove
CorrectAnswer-C
Ans:C.Bothoftheabove
Bridgingnecrosisisamoresevereformofhepatocellularinjuryin
acuteviralhepatitisandmayprogresstofulminanthepatitisor
chronichepatitis.Bridgingnecrosisischaracterizedbybandsof
necrosislinkingportaltractstocentralhepaticveins,onecentral
hepaticveintoanother,oraportaltracttoanothertract.
Inseverecasesofacutehepatitis,confluentnecrosisofhepatocytes
mayleadtobridgingnecrosisconnectingportal-to-portal,central-to-
central,orportal-to-centralregionsofadjacentlobules.
Hepatocyteswellingandregenerationcompresssinusoids,andthe
moreorlessradialarrayofhepatocyteplatesaroundterminal
hepaticveinsarelost.
Bridgingfibrosisisanimportantcomponentofsevereexamplesof
chronicviralhepatitisandsteatohepatitis.

464.Thezonalnecrosismostcommonly
affectedinchronicpassivehepatic
congestionis?

a)Central
b)Peripheral
c)Midzonal
d)None
CorrectAnswer-A
Answer-A.Central
Centrilobularhemorrhagicnecrosisiscausedbychronicpassive
congestion(CPC)duetorightheartfailure.

465.Normallysquamo-columnarjunctionis
usuallylocatedat
a)Distal2-3cmsofesophagus
b)Proximal2-3cmsofstomach
c)Inesophagusmorethan3cmsproximaltoGEJ
d)Noneoftheabove
CorrectAnswer-B
Answer-A.Distal2-3cmsofesophagus
Proximalextensionofthesquamocolumnarjunctionbeyondthe
distal2-3cmsofoesphagusisabnormalandissuggestiveof
Barrett'sesophagus.

466.Carcinomaassociatedmostcommonly
withupperonethirdofesophagusis
a)Adenocarcinoma
b)SquamouscellCarcinoma
c)Adeno-squamousCarcinoma
d)Leiomyosarcoma
CorrectAnswer-B
Answer-B.SquamouscellCarcinoma
Squamouscellcarcinomaisthemostcommontypeofesophageal
carcinomaworldwideandinIndia.Itusuallyoccursinmiddle1/3rd
(motcommon)andupper1/3'ofesophagus.Somemayalsoarisein
lower1/3

467.Autoimmunegastritisisassociatedwith
deficiencyofvitamin?
a)A
b)B12
c)C
d)D
CorrectAnswer-B
Answer-B.B12
Inautoimmunegastritis,twoimportantautoantibodiescausing
damageareanti-parietalcellantibodies(mostcommon)and
antibodiesagainstintrinsicfactor;anti-IF(mostspecific).
VitaminB-12deficiencyandperniciousanemia:Duetodeficiencyof
IFcells(secretedbyparietalcells)aswellasdamageofIFbyanti-IF
antibodies.

468.Bloodgroupmostcommonlyassociated
withgastriccarcinomais?
a)BloodGroup0
b)BloodgroupA
c)BloodgroupAB
d)BloodgroupB
CorrectAnswer-B
Answer-B.BloodgroupA
Geneticfactors:BloodgroupA,Hereditarynonpolyposiscolon
cancersyndrome(HNPCC)andFamilialgastriccancersyndrome
(E-cadherinmutation).

469.Whichdiseaseisdiagnosedbyjejunal
biopsy?
a)Celiacdisease
b)Intestinallymphoma
c)Argentaffinomaofintestine
d)Tropicalsprue
CorrectAnswer-B
Answer-B.Intestinallymphoma
thejejunalbiopsyisusefulinthediagnosisofthefollowing
disorders:
Intestinallymphoma
Intestinallymphangiectasia
Eosinophilicgastroenteritis
Amyloidosis
Crohn'sdisease
Infectionbyoneormore
microorganisms
Mastocytosis
Whipple'sdisease

470.commonestbenigntumorofthe
esophagus?
a)Leiomyoma
b)Papilloma
c)Adenoma
d)Hemangioma
CorrectAnswer-A
Ansisa.i.e.Leiomyoma
"Leiomyomasconstitutemorethan50%ofbenignesophageal
tumors"-Schwartz


471.Totalcolonicaganglionosisisavariant
of?
a)Crohn'sdisease
b)Ulcerativecolitis
c)Hirschsprung'sdisease
d)Tropicalsprue
CorrectAnswer-C
Answer-C.Hirschsprung'sdisease
Hirschprungdisease(Congenitalaganglonicmegacolon)iscaused
bydefectivemigrationofneuralcrestintothemesodermallayerof
gut.Thereisaganglionosis(absenceofganglions)inaportionof
intestinaltract.IntestinalsegmentlacksbothMeissnersubmucosal
andAuerbachmyentericplexuses.

472.APCgeneisinvolvedin?
a)Colorectalcarcinoma
b)Gastriccarcinoma
c)Gastriclymphoma
d)Esophagealadenocarcinoma
CorrectAnswer-A
Answer-A.Colorectalcarcinoma
TheAPCproteinisanegativeregulatorthatcontrolsbeta-
cateninconcentrationsandinteractswithE-cadherin,whichare
involvedincelladhesion.
MutationsintheAPCgenemayresultincolorectalcancer

473.Vitamindeficiencyassociatedwithcystic
fibrosisis
a)K
b)B6
c)C
d)B12
CorrectAnswer-D
Answer-D.B12
Patientswithcysticfibrosis(CF)areatriskofdeveloping
deficienciesoffat-solublevitamins(A,D,E,andK)becauseof
pancreaticinsufficiency,hepatobiliarydisease,orboth.

474.FalseaboutPatterson-Kelly-Brown
syndromeis?
a)Anemia
b)Esophagealwebs
c)Glossitis
d)Riskfactorforadenocarcinoma
CorrectAnswer-D
Answer-D.Riskfactorforadenocarcinoma
Plummer-Vinsonsyndrome(PVS)(Paterson-Brown-Kellysyndrome
orsideropenicdysphagia),ischaracterizedbydifficultyin
swallowing,irondeficiencyanemia,glossitis,chielosis,and
esophagealwebs.

475.Mostcommonsiteforcarcinoma
pharynxinfemalessufferingfrom
plummervinsonsyndromeis

a)Postcricoidregion
b)Posteriorwall
c)Lateralwall
d)Pyriformisfossa
CorrectAnswer-A
Ans.is'a'i.e.,Postcricoid
Plummer-Vinsonsyndrome
Plummer-Vinsonsyndrome,alsoknownasBrown-Kelly-Paterson
syndromeorsederopenicdysphagia,seeninmiddleaged
edentulouswomen.
TheplummerVinsionPatersonBrownKellySyndromeis
characterizedby:-
Dysphagia
Chronicirondeficiencyanemia
Atrophicoralmucosaandglossitis
Brittle,spoon-shapedfingernails(Koilonychia)
Thecauseofdysphagiaisusuallyacervicalesophagealweb,but
abnormalpharyngealandesophagealmotilitymayplayarole.
Thesyndromecharactersticallyoccursinmiddleagededentulous
(withoutteeth)women.
Itisapremalignantlesion.Approximately10%ofpatientdevelop
squamouscellCaofesophagus,oralcavityorthehypopharynx.
Asiron-deficiencyanemiaisacommonfinding,itisalsoknownas
sideropenicdysphagia.

Carcinomadevelopsinpost-cricoidregion.

476.Commonestcarcinomathatcancausesplenicmetastasisiswhichofthe
following?
a)Ca.Pancreas
b)Ca.Stomach
c)Ca.Ovary
d)Ca.Cervix
CorrectAnswer-C
Althoughisolatedmetastasistospleenisrare,studiesfoundthemostcommonprimary
neoplasmswithsplenicmetastasistobegynecologic(61%),withmajoritybeingovarian,
colorectal(15%),lung(9%),andstomach(4%).
Ref:CTandMRIoftheAbdomenandPelvis:ATeachingFileeditedbyPabloR.Ros,
KoenraadJ.Mortele,2006,Page218.

477.Phlegmonousgastritisoccursdueto?
a)H.pylori
b)E.coli
c)C.jejuni
d)Cl.Tetani
CorrectAnswer-B
Answer-B.E.coli
Mostcasesofphlegmonousgastritisareduetoalpha-hemolytic
streptococci,althoughpneumococci,staphylococci,Escherichiacoli,
andrarely,ProteusVulgarisandClostridiumwelchiicanbethe
causativeorganisms.

478.Collarbuttonulcerisfoundin?
a)Ulcerativecolitis
b)Crohn'sdisease
c)Shigella
d)Alloftheabove
CorrectAnswer-D
Answer-D.Alloftheabove
"Collarbuttonulcers",aradiologicalsign,aremanifestationsof
inflammatoryprocesseswithinthebowel.
Collarbuttonulcershavealsobeenobservedinthesettingofother
inflammatorybowelprocesses,suchasCrohn'sdisease,ischemic
colitis,andshigellosis.

479.Whichofthefollowingwouldbethebestmorphologicalfeatureto
distinguishulcerativecolitisfromCrohn'sdisease?
a)Diffusedistributionsofpseudopolyps
b)Mucosaledema
c)Cryptabscesses
d)Lymphoidaggregatesinthemucosa
CorrectAnswer-A
PseudopolypsaremorecommonlyfoundinulcerativecolitisthanCrohn'sdisease.
Thesearediscreteareasresultingfromsurvivingislandsofmucosaorheapedup
granulationtissue.
Sinceinulcerativecolitisthereisdiffusemucosalinflammationthesepseudopolypsare
diffuselydistributed.
DistinguishingfeaturesbetweenUlcerativecolitisandCrohn'sdisease:

Ulcerativecolitis
Crohn'sdisease
Rectalinvolvement
Yes
Variable
Distribution
Diffuse
Segmentalordiffuse
Terminalileum
Backwashileitis
Thickenedandstenosis
Serosa
Normal
Creepingfat
Mucosa
Hemorrhagic
Cobblestoneandlinearulcers
Pseudopolyps
Frequent
Lesscommon
Strictures
No
Common
Fistulas
No
Common
Lymphoidhyperplasia
Infrequent
Common
Cryptabscess
Extensive
Focal

Ref:PediatricInflammatoryBowelDiseaseByPetarMamulapage227.

480.Penilecarcinomaisusually?
a)Squamouscellcarcinoma
b)Basalcellcarcinoma
c)Adenocarcinoma
d)Smallcellcarcinoma
CorrectAnswer-A
Answer-A.Squamouscellcarcinoma
Mostofpenilecancersaresquamouscellcarcinomaandariseon
glansorinnersurfaceofprepuce.

481.Stainusedforstainingthenucleusis?
a)Safranin
b)Fastgreen
c)Hematoxylin
d)Erythrosine
CorrectAnswer-C
Answer-C.Hematoxylin
Hematoxylinstainsthecellnucleusandotheracidicstructures(such
asRNA-richportionsofthecytoplasmandthematrixofhyaline
cartilage)blue,whileeosinstainscytoplasm,connectivetissueand
otherextracellularsubstancespinkorred.

482.Allareassociatedwithdiabeticgangrene
except?
a)Wetgangrene
b)Drygangrene
c)Gasgangrene
d)Fournier'sgangrene
CorrectAnswer-C
Answer-C.Gasgangrene
Itisabacterialinfectionthatproducesgaswithintissues.Itcanbe
causedbyClostridium,mostcommonlyalphatoxinproducing
Clostridiumperfringens,orvariousnon-clostridialspecies.

483.Choroidmetastasisisassociatedmost
commonlywith?
a)Gastriccarcinoma
b)Renalcarcinoma
c)Braintumor
d)Bonetumor
CorrectAnswer-B
Answer-B.Renalcarcinoma
Tumorsmostlikelytometastasizetothechoroidplexusarerenal
cellcarcinomaandlungcancer.Othertumorswithdocumented
spreadtothechoroidplexusincludecolon,gastric,breast,thyroid,
andbladdercancers,melanomaandlymphoma.

484.Whichofthefollowingstatementsabout
thepathologyinAlzheimer'sdiseaseis
nottrue:

a)NeuriticPlaquesareformedofamyloidprotein
b)Neurofibrillarytangles(NFT)aremadeoftauprotein
c)NFTsappearextracellularlybeforeintracellularappearance
d)NumberofNFTscorrelateswithdementia
CorrectAnswer-C
AnswerisC(NFTsappearextracellularlybeforeintracellular
appearance):
NFTsaretypicallyseenintracellularlywithinthesomaandproximal
dendritesofneurons.
NeurofibrillaryTangles(NFTs)areintracellularaccumulationsof
hyperphosphorylated'tau'proteins.
NeurofibrillaryTanglesareIntracellularAccumulations
NeurofibrillaryTanglesareintracellularaccumulationsof
hyperphosphorylatedmicrotubulebindingprotein'tau'.
Pairedhelicalfilamentsoftauprotein(NFTs)formintracellularly
withinthesomaandproximaldendritesofneurons.
Thesecytoskeletalproteintangles(NFTs),initiallyimpedecellular
metabolismandaxosplasmictransportleadingtoimpairedsynaptic
functionandeventuallytoneuronaldeath.
Theseneurofibrillatytanglesmaybeseenasextracellulartangles
afterdegenerationoftheneuronasevidenceoftheneuronalcell's
demise
NeurofibrillaryTanglesareintracellularaccumulationsthatmay
appearextracellularlyalterdegenerationofneuron(neuronaldeath)


HistopathologicalHallmarksofAlzheimer'sDisease
AmyloidPlaques(Extracellular)
AmyloidNeuriticPlaquesareformedbyextracellularaccumulation
ofbetaamyloid
depositswithintheneutropil
'Neuritic'or'Senile'I3-amyloidplaquesareanearlyhistopathological
signofAlzheimer'sdisease(thatoccurrarelyinhealthysubjects)
Theamyloid13-proteinaccumulatedinsingleneuriticplaquesis
toxictosurroundingstructuresandadjacentneurons.
Clinicopathologicalstudieshaveshownthatamy/aidburdendoes
notdirectlycorrelatewithseverityordurationofdementia.

NeurofibrillaryTangles(Intracellular)
Neurofibrillarytanglesarcformedbyintracellularaccumulationof
hyperphosphorylatedmicrotubulebindingprotein'tau'.
NFT'soccurinmanyneurodegenerativediseasesand/oragroupof
diseasescalledlaupathies'.
TheseincludeFrontotemporaldementia,Pick'sdiseaseetc.The
cooccuranceoffi-amyloidplaqueswithNFT'ssuggestsadiagnosis
ofAD.
TheNFT'saretoxictotheneuronsandneuronswithNFT's
eventuallydieanddegenerateleavingaresidual`ghosttangle',in
theextracellularspaceremindingofthepyramidalcellbodyinwhich
itwasinitiallyformed.
Clinicopathologicalstudieshaveshownthatdementiacorrelates
morestronglywithNFT'sthanwithsenileplaques(3-amyloid)


485.Metastasistothyroidcomesfromwhich
primarysiteofmalignancy?
a)Liver
b)Testis
c)Prostate
d)Kidney
CorrectAnswer-D
Answer-D.Kidney
Mostcommonprimarysitesarekidney,breast,lung,uterus,and
melanoma.

486.Whichofthefollowingisthemost
commonmutationinEwing'ssarcoma-
a)TranslocationX:18
b)Translocation11;22
c)ActivativemutationofG5a,p,surfaceprotein
d)MissensemutationinEXTI
CorrectAnswer-B
Ans.is'b'i.e.,Translocation11;22
Ewing'ssarcomaistypicallycharachterizedbyatranslocationt(11;
22)(q24;q12)inupto90%ofpatients.

487.Leastdesmoplasticbreastcarcinomais
a)Ductal
b)Lobular
c)Tubular
d)Medullary
CorrectAnswer-B
Answer-B.Lobular
"Invasivelobularcarcinomahasatendencytospreaddiffuselyor
betweenthecollagenfibersofthebreastandproduces
desmoplasticresponse"

488.Markerforneuroblastomaamongthe
followingis?
a)NMP22
b)ChromograninA
c)LDH
d)32microglobulin
CorrectAnswer-B
Answer-B.ChromograninA
Neuroendocrinemarkersofneuroblastoma:-i)CD-56ii)
Chromogranin-Aiii)Synaptophysin

489.Thymichyperplasiaisseenin?
a)Thymoma
b)Thymiclymphoma
c)Myastheniagravis
d)Scleroderma
CorrectAnswer-C
Ans.is'c'i.e.,Mystheniagravis
Thymichvperplasia
ThetermthymichyperplasiausuallyappliestotheappearanceofB-
cellgerminalcenterswithinthethymus,afindingthatisreferredto
asthymicfollicularhyperplasia.
SuchB-cellfolliclesarepresentinonlysmallnumbersinthenormal
thymus.
Itcanoccurinanumberofchronicinflammatoryandimmunologic
states,butitismostfrequentlyencounteredinmyastheniagravis
(65%to75%ofcases).
SimilarthymicchangesaresometimesencounteredinGraves
disease,systemiclupuserythematosus,scleroderma,rheumatoid
arthritis,andotherautoimmunedisorders.

490.InEndometrialcarcinoma,whichofthe
followingtumorsuppressorgene
occurs?

a)P53
b)Rb
c)PTEN
d)APC
CorrectAnswer-C
PTENisatumorsuppressorgenewhichisimplicatedinthe
causationofendometrialandprostatecarcinoma.

491.Foldingdefectisassociatedwithwhich
disease
a)Parkinson'sdisease
b)Marfansyndrome
c)Acuteintermittentporphyria
d)Wermersyndrome
CorrectAnswer-A
Answer-A.Parkinson'sdisease
Defectiveproteinfoldingdisorders(DPFDs)areagroupofdiverse
neurologicalandsystemicdiseasesinwhichthehallmark
pathologicaleventisthemisfolding,aggregationandaccumulation
ofaproteinindifferentorgans,inducingcellularapoptosis,tissue
damageandorgandysfunction.
IncludesAlzheimer'sdisease,transmissiblespongiform
encephalopathies,serpin-deficiencydisorders,sicklecellanemia,
Huntington'sdisease,diabetestypeII,amyotrophiclateralsclerosis,
Parkinson'sdisease,dialysis-relatedamyloidosis,spinocerebellar
ataxias,secondaryorreactiveamyloidosis,cysticfibrosisandprion
diseases.

492.Stainusedfortubulinis
a)Lunastain
b)Cajalstain
c)SiRstain
d)Masson'strichrome
CorrectAnswer-C
Answer-C.SiRstain
fluorescentstains(SiR-ActinandSiR-Tubulin)arecellpermeable
compoundswhichstainF-actinandmicrotubules,respectively.

493.X-linkedadrenoleukodystrophyis
a)Fattyaciddisorder
b)Lysosomalstoragedisorder
c)Mucoplysaccharridoses
d)Glycogendefectdisorder
CorrectAnswer-A
Answer-A.Fattyaciddisorder
X-linkedadrenoleukodystrophy(ALD/X-ALD)isadiseaseiscaused
bymutationsinABCD1,agenelocatedontheXchromosome,that
codesforALD,aperoxisomalmembranetransporterprotein.

494.ChromosomeforMEN2geneis
a)11q13
b)13q11
c)10811.2
d)11q10-2
CorrectAnswer-C
Answer-C.10811.2
Multipleendocrineneoplasiatype1MEN111813
Multipleendocrineneoplasiatype2aRET10811.2

495.Hobnailappearanceisseenin
a)Clearcellcarcinoma
b)Endodermalsinustumor
c)HCC
d)Choriocarcinoma
CorrectAnswer-A
Answer-A.Clearcellcarcinoma
Hobnailcellisacellwithacharacteristicappearance,includinga
bulbousnucleusandnuclearprojectionsintothecytoplasm.Hobnail
cellsarefoundinclearcellovarianadenocarcinoma,collectingduct
carcinoma,andinend-stagecirrhosis

496.Sphenoiddysplasiaisseenin?
a)NF-1
b)Tuberoussclerosis
c)Sturge-Webersyndrome
d)Creutzfeldt-Jakobdisease
CorrectAnswer-A
Answer-A.NF-1
NF-1isdiagnosediftwooffollowingsevensarepresent:

1. Sixormorecafe-au-laitmacules:>5mminprepubertalageand>
15mminpostpubertalage.
2. Axillaryoringuinalfreckling.
3. TwoormoreLischnodules(hamartomasoniris).
4. Twoormoreneurofibromaoroneplexiformneurofibroma.
5. Adistinctiveosseouslesion:Sphenoiddysplasiaorcorticalthinning
oflongbones.
6. Opticglioma.
7. AfirstdegreerelativewithNF-1.

497.InacaseofDysgerminomaofovaryone
ofthefollowingtumormarkersislikely
toberaised:

a)SerumHCG
b)Serumalphafetoprotein
c)Serumlacticdehydrogenase
d)Seruminhibin
CorrectAnswer-C
Ans.isci.e.Serumlacticdehydrogenase
howeverplacentalalkalinephosphateandlactate
dehydrogenasearecommonlyproducedbydysgerminomas
andmaybeusefulinmonitoringthedisease."


498.Mostcommonsiteforectopicthyroid
tissueis?
a)Ovaries
b)Lingual
c)Infrontofhyoidbone
d)Stomach
CorrectAnswer-B
Answer-B.Lingual
Byfarthemostcommonlocationisnearitsembryologicaloriginat
theforamencaecum,resultinginalingualthyroid.Thisaccountsfor
90%ofallcasesofectopicthyroids.

499.Salivaryschintigraphyisusefulin?
a)Monomorphicadenoma
b)Pleomorphicadenoma
c)Sialidinitis
d)Sjogrensyndrome
CorrectAnswer-D
Answer-D.Sjogrensyndrome
Technitiumpertechnetatescans(Salivaryscintigraphy)teststhe
size,shapeandfunctionofthesalivaryglands.Itisuseful.
1. Tohelpdeterminethecauseofsalivaryglandswelling(e.g.bacteria
orvirus).
2. Todetectablockageofthesalivaryducts.
3. Todetectagrowthinthesalivaryglands(e.g.Warthin'sTumour).
4. TohelpdiagnoseabnormalmouthdrynessasSjogren'sSyndrome.

500.Trilateralretinoblastomais?
a)BilateralRetinoblastomaplusmedulloblastoma
b)Bilateralretinoblastomapluspineoblastoma
c)Bilateralretinoblastomaplusneuroblastoma
d)Bilateralretinoblastomaplusependymoma
CorrectAnswer-B
Answer-B.Bilateralretinoblastomapluspineoblastoma
Trilateralretinoblastoma(TRb)referstothecombinationof
retinoblastoma(usuallybilateral)andpineoblastoma.This
relationshiphighlightsthecloserelationshipbetweenthesehighly
aggressivesmallroundbluecelltumors.

501.Mostcommoncancertooccurinorgan
transplantpatientis?
a)Squamouscellcarcinoma
b)Melanoma
c)Lungcancer
d)Colorectalcancer
CorrectAnswer-A
Answer-A.Squamouscellcarcinoma
mostcommontumorsaftertransplant
Nonmelanomaskincancer(SCC>BCC)(mostcommon)
NHL(2ndmostcommon)
Lungcarcinoma
Kaposi'ssarcoma
HCC
Cervicalcarcinoma

502.Hydroxylionsaredestroyedinthebody
by?
a)VitaminC
b)VitaminA
c)VitaminK
d)VitaminD
CorrectAnswer-A
Answer-A.VitaminC
"VitaminCandotherwatersolublecompounds,suchasuricacid,
thiolsincludingglutathioneanddihydrolipoicacidandpossiblyother
substancessuchasmetallothionein,servetodefendagainst
hydroxylradicals.VitaminE,incontrast,islesseffectivein
eliminatinghydroxylradicals.

503.Whichofthefollowingdrugsisnotan
inhibitorofP-glycoprotein?
a)Quinidine
b)Erythromycin
c)Verapamil
d)Phenobarbitone
CorrectAnswer-D
Ans.D.Phenobarbitone
[RefKDT7Vep.15]
P-glycoprotein:
Productofmultidrugresistance1gene(ABCB1).
Importantroleinpharmacokineticsofdrugs.
AnATP-bindingcassette(ABC)transporterandisanimportant
factortolimitmembranepermeabilityinseveraltissuesand/or
eliminationpathwaysintourine(renaltubles)andbile(liver).
SomedrugsaresubstrateforbothCYP3A4andP-gp.Examples
:
CCBs:Verapamil,diltiazam
Anticancerdrugs:Etoposide,daunorubicin,doxorubicin,paclitaxel,
vincrisitineAntimicrobials:HIVprotoeaseinhibitors(indinavir,
ritonavir),erythromycin,ketoconazole
Immunosuppressants:Cyclosporine,tacrolimus,sirolimus.
Other:Digoxin,fexofenadine,loperamide.

504.Liposomedrugdeliverysystemisused
forallexcept?
a)Vincristine
b)AmphotericinB
c)Hyoscine
d)Amikacin
CorrectAnswer-C
Ans.C.Hyoscine
Importantdrugswithliposomedeliverysystems:
AnticancerdrugsDonorubicin,Daunorubicin,vincristine,
camptothecin,methotrexate,cisplatin,mitoxantrane
AntifungalAmphotericinB,Nystatin
AntibioticsAmikacin,Ampicillin,ciprofloxacin,Ribavirin,
Ganciclovir,chloroquine
OthersIL-2,cyclosporin

505.Tachyphylaxisisseenwithwhichofthe
followingdrugs?
a)Pethidine
b)Ephedrine
c)Phenoxybenzamine
d)Phentolamine
CorrectAnswer-B
Ans.B.Ephedrine
[RefKDT7th/ep.70&6th/ep.68;Laurence50/ep.448;Katzung
11`"/ep.32]
Tachyphylaxis:
*Rapidlydiminishingresponsetorepeatedadministrationofadrug.
Tachyphylaxismayoccurdueto:
*Down-regulationofreceptors
-Whentissuesarecontinuouslyexposedtoanagonist,thenumber
ofreceptorsdecreases(down-regulation).
-Itoccursinasthmaticswhouse132-agonistbronchodilators
excessively.
*Depletionofstoredneurotransmitter
-Itisparticularlycommonwithindirectlyactingsympathomimetics
drugs,e.g.amphetamine,tyramineandephedrine.
-Itisduetodepletionofreleasablepoolofnoradrenalinefrom
adrenergicnerveterminals.

506.Synergesticactionisshownbythe
followingdrugcombinationsexcept?
a)Glibenclamideandmetformin
b)Enalaprilandhydrochlorthiazide
c)Levodopaandcarbidopa
d)Hydrochlorthiazideandtriamterene
CorrectAnswer-D
Ans.D.Hydrochlorthiazideandtriamterene
[RefKDTp.56]
SYNERGISM:
Whentheactionofonedrugisfacilitatedorincreasedbytheother,
theyaresaidtobesynergistic.
Inasynergisticpair,boththedrugscanhaveactioninthesame
directionorgivenaloneonemaybeinactivebutstillenhancethe
actionoftheotherwhengiventogether.
Additivedrugcombination:
Aspirin+paracetamol-Asanalgesic/antipyretic
Nitrousoxide+halothane-Asgeneralanesthetic
Amlodipine+atenolol-Asantihypertensive
Ephedrine+theophylline-Asbronchodilator

507.Oxidationinbiotransformationis?
a)Functionalizationreaction
b)Conjugationreaction
c)Syntheticreaction
d)Felsonreaction
CorrectAnswer-A
Ans.A.Functionalizationreaction
[Ref:KDT7th/ep.22,23]
Biotransformationincludes2typesofreaction:
PhaseI/Nonsynthetic/functionalizationreaction:Oxidation,
reduction,hydrolysis,cyclization,decyclization.
PhaseH/synthetic/conjugationreactions:Acetylation,glutathione
conjugation,glucoronideconjugation(glucuranization),glycine
conjugation,methylation,sulfateconjugation(sulfuranation),
nucleotidesynthesis.

508.Oxybutyninactsby?
a)Adrenergicreceptorantagonist
b)Muscarinicrece8ptorantagonist
c)Histaminicantagonist
d)Serotonergicantagonist
CorrectAnswer-B
Ans.B.Muscarinicrece8ptorantagonist
[RefKDT7thlep.113,117]
Oxybutynin:
Thisrecentlyintroducedantimuscarinic(muscarinicreceptor
antagonist)hashighaffinityforreceptorsurinarybladderand
salivaryglandswithadditionalsmoothmusclerelaxantandlocal
anestheticproperties.
ItisrelativelyselectiveforM1/M3subtypesthanforM2.

509.Whichofthefollowingdrugcrosses
BBB?
a)Glycopyrrolate
b)Neostigmine
c)Physostigmine
d)Alloftheabove
CorrectAnswer-C
Ans.C.Physostigmine
[RefKDTp.07,117]
Physostigmine-RapidlyabsorbedfromGITandparenteralsites,
penetratescorneafreelyandcrossesBBB(bloodbrainbarrier).
Neostigmine-Itisaquarternaryammoniumcompoundwhichis
poorlyabsorbedorallywithpoorcornealpenetrationanddoesn't
crossBBB.
Glycopyrrolate-Itisapotentandrapidlyactinganti-muscarinic
lackingcentraleffectsandisusedasapre-anaestheticmedication.

510.Whichofthefollowingisnottrueabout
theactionofanticholinergicdrugs?
a)AtropineisaCNSdepressant
b)Atropinecausesmydriasis,abolitionoflightreflexand
cycloplegia
c)Atropinecausesbronchoconstriction
d)Atropinecanincreasethechancesofhyperthermiainchildren
CorrectAnswer-A:C
Ans.A.AtropineisaCNSdepressant&C.Atropinecauses
bronchoconstriction
[RefKDT7iVep.113,114]
Actionsofanticholinergicdrugs:
Theseareoppositeofparasympathetic(cholinergic)system.
AtropinehasanoverallCNSstimulantaction.
Stimulatesmedullarycentres-vagal,respiratory,vasomotor.
Depressesvestibularexcitationandhasantimotionsickness
property.
AbbrevatesrefractoryperiodofA-VnodeandfacilitatesA-V
conduction,PRintervalisshortened.
DoesnothaveanyconsistentormarkedeffectonBP.
UsedinarrhythmiaslikeAVblockanddigitalisinducedarrhythmia.
Causesmydriasisduetocontractionofcircularmuscles(constrictor
pupillae),abolitionoflightreflexandcycloplegia(paralysisof
accomodation).
IncreasesintraoculartensionContraindicatedinglaucoma.
Causesbronchodilatationandreducedairwayresistance,especially
inCOPDandasthmapatients.
Relaxesurinarybladder,urinaryretentionmay

occurcontraindicatedinbenignprostatehypertrophy.
Decreasessweat,salivary,tracheobronchialandlacrimalsecretion.
Decreasessecretionofacid,pepsinandmucusinthestomach.

511.Advantageofglycopyrolateover
atropineis?
a)Itisanaturalalkaloid
b)ItlacksCNSpenetration
c)CanbeusedinOPCpoisoning
d)Ismorepotent
CorrectAnswer-B
Ans.B.ItlacksCNSpenetration
[RefKDT7thiep.117]
Glycopyrrolate:
Quarternarysyntheticcompound,whichispotentandrapidlyacting
antimuscariniclackingCNSpenetrationandcentraleffects.
Almostexclusivelyusedinpreanaestheticmedication.

512.Oximesareineffectiveinwhichofthe
followingpoisoning:?
a)Organophosphatepoisoning
b)Amanitaphylloidespoisoning
c)Carbamatepoisoning
d)Dhaturapoisoning
CorrectAnswer-C
Ans.C.Carbamatepoisoning
[RefKDT7th/ep.111&6t/ep.105;Katzung11thiep.121]
Oximes[Pralidoxime2-PAM,obidoximeanddiacetyl-monoxime
(DAM)]areusedinorganophosphatepoisoning.
Oximesactsbyreactivatingcholinesteraseenzyme.
IneffectiveinCarbamatespoisoning.
Pralidoximeiscontraindicatedincarbamatespoisoning,becausenot
onlyitdoesnotreactivatecarbamylatedenzyme,ithasweakanti-
chEactivityofitsown.
Mostcommonlyusedcholinesterasereactivater.

513.Whichofthefollowingisanexampleof
irreversiblecarbamate?
a)Ambenonium
b)Galantamine
c)Propoxur
d)Rivastigmine
CorrectAnswer-C
Ans.C.Propoxur
Irreversiblecarbamate:
Carbaryl
Propoxur

514.38yroldpatientwithhighriskof
coronaryarterydiseaseriskhas
hypertention,whichofthefollowing
antihypertensivedrugswillbesuitable
asafirstlinetreatmentforthispatient?

a)ACEinhibitors
b)Calciumchannelblockers
c)Betaadrenergicblockers
d)Diuretics
CorrectAnswer-A
Ans.A.ACEinhibitors
Patientisrelativelyyounghypertensive(38yrs)withhighriskof
coronaryarterydisease,ACEinhibitors/Angiotensinreceptor
blockeristhesuitablefirstlinetherapyformanagementof
hypertentioninsuchpatients.

515.WhichofthefollowingisNOTaside
effectofamiodarone?
a)Pulmonaryfibrosis
b)Cornealmicrodeposits
c)Photosensitivity
d)Tachycardia
CorrectAnswer-D
Ans.D.Tachycardia
[Ref:KDT7"Vep.534]
Amiodaroneisabroadspectrumanti-arrhythmicdrugwhich
belongstoclassIIIoftheanti-arrhythmicdrugs.
Followingaretheadverseeffects-
FallinBP,bradycardiaandmyocardialdepressionoccursoni.v.
injection.

516.Asideeffectofloopdiureticsisusedin
?
a)Post-surgerycare
b)Chronicanemia
c)Bloodtransfusion
d)Oncology/cancer
CorrectAnswer-D
Ans.D.Oncology/cancer
Furosemideandotherloopdiureticscausehypocalcemiaby
increasingCa2+excretion.
Forthesamereasontheyareusedintumorinducedhypercalcemia
toreduceserumcalciumlevel.

517.Whyadenosinehasashorthalflife?
a)Spontaneoushydrolysis
b)Uptakeinsubcutaneoustissue
c)UptakeinRBCandendothelialcells
d)Renalexcretion
CorrectAnswer-C
Ans.C.UptakeinRBCandendothelialcells
[RefKDT6thIep.518;Katzung11thiep.244]
AdenosineistheDOCforP.S.V.T.
Administeredbyrapidi.v.injectioneitherasfreebaseorasATP.
Actionisveryrapid-terminatesmorethan90%episodesofPSVT
within30sec.
Adenosineisveryshortacting(t.inblood-10sec)duetouptake
intoRBCsandendothelialcells.

518.Mechanismofactionofnicorandilis?
a)K*channelblocker
b)I('channelopener
c)Na'channelblocker
d)Cl-channelblocker
CorrectAnswer-B
Ans.B.I('channelopener
[Ref:KDT7th/ep.540,552]
Nicorandil
Thisdualmechanismanti-anginadrugthatactivatesATPsensitive
K+channels(potassiumchannelopener)therebyhyperpolarizing
vascularsmoothmuscle.

519.Whichofthefollowingantiarrhythmic
drugscandevelopLongQTsyndrome?
a)Ibutilide
b)Dofetilide
c)Sotalol
d)Alltheabove
CorrectAnswer-D
Ans.D.Alltheabove
ProarrhythmicManifestationsofMostFrequentlyUsed
AntiarrhythmicAgents:
Amiodarrone
Digoxin
Disopyramide
Dofetilide
Dronedarone
Flecainides
Propafenone
Quinidine
Sotalol

520.Whichofthefollowingisnottrueabout
themechanismofactionofdigitalis?
a)ItbindstotheintracellularfaceofNa'ATPaseenzyme
b)ThereisriseinintracellularNa'
c)Ithaspositiveionotropicaction
d)Digitalisactionisindependentofcardiacinnervation
CorrectAnswer-A
Ans.A.ItbindstotheintracellularfaceofNa'ATPaseenzyme
[RefKDT7th/ep.496]
Digitalismechanismofaction:
Digitalisincreasestheforceofcontractionbyadirectaction
independentoftheinnervation.
BindstotheextracellularfaceoftheNa+K+ATPaseandinhibitsthe
enzymecausingriseintheintracellularlevelsofNa+.
TheraisedNa+inturninhibitstheNa+Ca2+exchangerandcauses
riseinintracellularCa+.
ThisraisedintracellularCa2+isresponsibleforthepositive
ionotropiceffect.
Thus,digitalisincreasesthecardiaccontractabilityandforceof
contraction.

521.Whichdruginhibitsboth
cyclooxygenaseandlipooxygenase?
a)Aspirin
b)Indomethacin
c)Imidazole
d)BW755
CorrectAnswer-D
Ans.D.BW755
[RefBiochemistryofeyebyElaineR.Bermanp.165]
Drug-BW755
Enzymeinhibited-Cyclooxygenaseandlipoxygenase

522.Drugactingon5HT4receptoris?
a)Loxiglumide
b)Renzapride
c)Atractiloside
d)Metoclopromide
CorrectAnswer-B
Ans.B.Renzapride
[RefKDT7th/ep.174]
Renzapride:
Renzaprideisacisapridecongenerwhichisaprokineticdrugandit
increasesthegastrointestinalmotilitybyactingon5HT4receptors.
Renzaprideisstillmoreselectivefor5HT4thancisapride.
Italsohaslesserpropensitytocausecardiacarrhythmias.

523.Drugofchoiceforabortingtheacute
attackofmigraineis?
a)NSAIDslikeindomethacin
b)Opioidslikemorphine
c)Triptanslikesumatriptan
d)Glucocorticoids
CorrectAnswer-C
Ans.C.Triptanslikesumatriptan
[RefKDT7th/eP.179]
Treatmentandprophylaxisofmigraine:
*Forabortinganacuteattackofmigraine,sumatriptan(oranyother
triptan)isthedrugofchoice.
-OtherdrugsusedfortreatmentareNSAIDs,ergofamineand
dihydroergotamine,andintranasalbutorphanol.
*ForProphylaxis,Beta-blocker(propranolol)isthedrugofchoice.
-Otherdrugsusedforprophylaxisaretricyclicantidepressants
(amitriptyline),calciumchannelblockers(cinnarizine,verapamil),
serotonineantagonists(methysergide,cyproheptadine),MAO
inhibitors,andanticonvulsants(valproate,topiramate,gabapentine),
fluxetin,onabotulinumtoxineA,pepaverineandphenalzine.

524.Rasburicaseisananalogueof?
a)Xanthineoxidase
b)IMPdehydrogenase
c)AdenosineDeaminase
d)UrateOxidase
CorrectAnswer-D
Ans.D.UrateOxidase
Rasburicaseisarecombinantversionofurateoxidase.
ThecDNAofUrateoxidaseisobtainedfromAspergillusflavusand
isintroducedintoSachhromycescervicae.
Itisusedforreducinguratelevels.

525.Adverseeffectofmethysergideis?
a)Metabolicsyndrome
b)Endocardialfibrosis
c)Peyronie'ssyndrome
d)Drymouth
CorrectAnswer-B
Ans.B.Endocardialfibrosis
[RefKDT7th/ep.174]
Methysergide
Chemicallyrelatedtoergotalkaloidswhichantagonizesactionof
5HTonsmoothmusclesincludingthatofbloodvessels.
Mechanismofaction
Potent5HT2A/2CantagonistwithNoaadrenergicordopaminergic
action.
Uses:
Migraineprophylaxis,carcinoidsyndromeandpostgastrectomy
dumpingsyndrome.
Adverseeffects
Abdominal,pulmonaryorendocardialfibrosisisproducedwith
prolongeduse.

526.Contraindicationforthetriptansiswhich
ofthefollowing?
a)Ischemicheartdisease
b)Epilepsy
c)Hepaticfailure
d)Alloftheabove
CorrectAnswer-D
Ans.D.Alloftheabove
[RefKDT7th/ep.179]
Triptans
Theseare5-HTIDABreceptoragonists.
Alltriptanscancausecoronaryvasospasmandarecontraindiatedin
IHD.Othercontraindicationsarehypertension,epilepsy,hepaticor
renalimpairmentandduringpregnancy.
Themostimportantadverseeffectsarefeelingofchestpressure,
tightnessandpainwhichmaybeaccompaniedbyarrhythmiaandMI
andappeartobeduetocoronaryvasospasm.

527.Advantageofformoteroloversalmeterol
is?
a)Itcanbeusedforprophylaxisinasthmatics
b)Ithasgotafasteronsetofaction
c)Itisashortactingbeta2agonist
d)Italsohasbeta1agonisticaction
CorrectAnswer-B
Ans.B.Ithasgotafasteronsetofaction
[RefKDT7Vep.224;GoodmanGillman11thiep.720]
Formoterol
Itisalongactingselective132agonistwhichacts12hourswhen
inhaled.Incomparisontosaletetrolithasfasteronsetofactionand
isusedonaregularmorning-eveningscheduleforroundtheclock
bronchodilatation.
Dose:12-24pginhalationtwicedaily.

528.UseofPGF2aanaloguesis
contraindicatedin?
a)Postpartumharmorrhage
b)Glaucoma
c)Bronchialasthma
d)Priapism
CorrectAnswer-C
Ans.C.Bronchialasthma
[RefKDT7th/ep.185,189]ProstaglandinF2a]
PGF2aanaloguesaresmoothmuscleconstrictorwhichbringabout
contractionofsmoothmuscles
PGF2aanaloguelikecarboprostisusedforpostpartum
haemorrhage
PGF2aanalogueslikelatanoprosthasbeenusedforglaucomaand
isafirstlinedrugforopenangleglaucoma.
Itiscontraindicatedinbronchialasthmaasitcausesbronchial
musclecontraction.

529.MechanismofactionofTeriparatideis?
a)RecombinantPTH[rPTH]
b)Recombinantcalcitonin
c)Recombinantinsulin
d)Recombinantprolactin
CorrectAnswer-A
Ans.A.RecombinantPTH[rPTH]
Teriparatide
Thisisrecombinantpreparationof1-34moleculesofaminoterminal
ofhumanPTH.
Itduplicatesalltheactionsoflong(1-84)PTH.
Diagnosticuse:Todifferentiatepseudofromtrue
hypoparathyroidism:teriparatideisgiveniv:ifplasmacalciumlevel
failstorise,thenitispseudohypoparathyroidism.

530.Anti-inflammatoryactionsof
corticosteroidsaremediatedby?
a)Byinhibitingangiogenesis
b)Byinhibitingbreakdownofphospholipids
c)Byincreasingvascularity
d)Byincreasinggranulationtissueformation
CorrectAnswer-B
Ans.B.Byinhibitingbreakdownofphospholipids
[RefKDT7thiep.279]
Glucocorticoidsinterfereatseveralstepsintheinflammatory
responsebutthemostimportantoverallmechanismappearstobe
limitationofrecruitmentofinflammatorycellsatthelocalsiteand
productionofproinflammatorymediatorslikePCs,LTs,PAFthrough
inhibitionofphospholipaseA.

531.Whichofthefollowingisnotan
inhalationalsteroids?
a)Beclomethasone
b)Betamethasone
c)Budesonide
d)Fluticasoneacetonide
CorrectAnswer-B
Ans.B.Betamethasone
[RefKDT7Vep.290]
Corticosteroidmaybeusedasinhaledorsystemicdrugsinasthma.
Inhaledsteroids
Theseareglucocorticoidswithhightopicalandlowsystemicactivity.
Commonlyuseddrugsarebeclomethasome,budesonide,
fluticasoneandciclesonide.
Inhaledsteroidsarethemosteffectiveanti-inflammatoryagents
usedinasthma.

532.Whichofthefollowingantithyroid
medicationshadthemaximumchances
ofcausingagranulocytosis?

a)Carbimazole
b)Clotrimazole
c)Propylthiouracil
d)Methimazole
CorrectAnswer-C
Ans.C.Propylthiouracil
[RefKDT7h/ep.253]
Propylthiouracilhasgotmaximumchanceofcausing
agranulocytosis.Itismostlyreversible.

533.Mifepristoneactsonwhichreceptor?
a)TypeAprogesteronereceptor
b)Estrogenreceptor
c)LHreceptor
d)Thyroidreceptor
CorrectAnswer-A
Ans.A.TypeAprogesteronereceptor
[RefKDT7hlep.319,320]
Mifepristone
Itisapotent,19-norsteroid.
Ithasanti-progestational,significantanti-glucocorticoidandanti-
androgenicactivity.
MifepristoneisapartialagonistandcompetitiveantagonistatbothA
andBformsofprogesteronereceptor(PR)Intheabsenceof
progesterone(anovulatorycyclesaftermenopause)itexertsweak
progestionalactivityandinducespredecidualchanges.

534.Pegvisomantis?
a)Somatostatinantagonist
b)Somatotropinantagonist
c)GHreceptorantagonist
d)GHreceptoragonist
CorrectAnswer-C
Ans.C.GHreceptorantagonist
[RefKDT7th/ep.238]
Pegvisomant
ThispolyethyleneglycolcomplexedmutantGH.
ItbindstoGHreceptorbutdoesnottriggersignaltransductionand
actsasaGHreceptorantagonist.

535.Drugusedformedicalmanagementof
acromegalyduetosmallpituitarytumors
is?

a)Fulvestrant
b)Pegvisomant
c)Vigabatrin
d)Cabergoline
CorrectAnswer-B
Ans.B.Pegvisomant
[RefKDT7h/ep.238]
Pegvisomant:
ThispolyethyleneglycolcomplexedmutantGH.
ItbindstoGHreceptorbutdoesnottriggersignaltransductionand
actsasaGHreceptorantagonist.
Approvedfortreatmentofacromegalyduetosmallpituitary
adenomas.

536.Tiboloneisa?
a)Naturalsteroidalestrogen
b)Naturalnon-steroidalestrogen
c)Syntheticsteroidalestrogen
d)Syntheticnon-steroidalestrogen
CorrectAnswer-C
Ans.C.Syntheticsteroidalestrogen
[RefKDT7Vep.306,311]
Syntheticestrogens
SteroidalEthinylestradiol,mestranol,tibolone.
NonsteroidalDiethylstilbestrol,hexestrol,dienestrol.

537.Tiboloneisusedfor?
a)Fibroids
b)Endometriosis
c)Hormonereplacementtherapy
d)Anovulatoryinfertility
CorrectAnswer-C
Ans.C.Hormonereplacementtherapy
[RefKDT7thiep.306,311]
Tibolone
Itisa19-norsteroiddevelopedspecificallytobeusedforhormone
replacementtherapy,whichcombinesestrogenicandprogestational
propertieswithweakandrogenicactivity.
Inadoseof2.5mgdaily,itsuppressesmenopausalsymptomsand
loweretheraisedGnlevels.Noendometrialstimulationhasbeen
noted.
Urogenitalatrophy,psychologicalsymptoms,libidoandosteoporosis
areimprovedsimilartootherformsofHRT.
Increaseinbreastcancerriskappearstobelessthanwithcombined
HRT.
Weightgain,increasedfacialhair,andoccasionalvaginalspotting
maybenoted.

538.Danazolactsthrough:?
a)IncreasesreleaseofGn
b)Increasesinsulinrelease
c)InhibitionofreleaseofGn
d)Inhibitionofinsulinrelease
CorrectAnswer-C
Ans.C.InhibitionofreleaseofGn
[RefKDT7th/ep.301]
Danazole
Ithasweakandrogenic,anabolicandprogestationalactivity.
Themostprominentactionissuppressionofgonadotropin(FSH/LH)
frompituitary.

539.MechanismofactionofVogliboseis:?
a)pgalactosidaseinhibitor
b)3lactaseinhibitor
c)aglucosidaseinhibitor
d)3glucosidaseinhibitor
CorrectAnswer-C
Ans.C.aglucosidaseinhibitor
[RefKDT7th/ep.277]
Voglibose
Itisaglucosidaseinhibitorwhichpreventsbreakdownofcomplex
carbohydratesintosimplersugarslikeglucose.

540.Metyrosineactsbyinhibiting?
a)PhenolethanolamineNmethylTransferase
b)PhenylalanineHydroxylase
c)TyrosineHydroxylase
d)Tyrosinase
CorrectAnswer-C
Ans.C.TyrosineHydroxylase
MetyrosineisaMethylL-tyrosine
ItisacompetitiveinhibitorofTyrosineHydroxylase
TyrosinehydroxylaseistheenzymewhichconvertsTyrosineto
Dihydroxyphenylalanine(DOPA).Itistheratelimitingenzymeof
catecholaminesynthesis.

541.Whichofthefollowingoral
hypoglycaemicdrugshasthelongestt
1/2?

a)Gliclazide
b)Glimepiride
c)Chlorpropamide
d)Tolbutamide
CorrectAnswer-C
Ans.C.Chlorpropamide
[RefKDT6thlep.268]
Tolbutamideisshortestactingsulfonylurea.
Chlorpropamideislongestactingsulfonylureaandlongestacting
oralhypoglycemic.
Nateglinideisshortestactingoralhypoglycemic.
Tolbutamide,becauseofshorterdurationofaction,issaferinelderly
andinthosepronetohypoglycemia.

542.Allofthefollowingareusesofoctreotide
except:?
a)Secretorydiarrhea
b)Acromegaly
c)Hepaticencephalopathy
d)Bleedingesophagealvarices
CorrectAnswer-C
Ans.C.Hepaticencephalopathy
[RefKDT7tVep.238]
Octreotideisasyntheticoctapeptidesurrogateofsomatostatin
whichis40timesmorepotentinsuppressingGHandprolactin
secretion.
Itisalsoaweakinhibitorofinsulinsecretion.
Uses:
Acromegaly
Secretorydiarrheaassociatedwithcarcinoids,AIDS,cancer
chemotherapyordiabetes.

543.Whichofthefollowingpairsiscorrect?
a)Glibenclamide-Na'ATPblocker
b)Biguanides-AMPKinaseactivation
c)Vildagliptin-SGLT2inhibitor
d)Voglibose-DPP4inhibitor
CorrectAnswer-B
Ans.B.Biguanides-AMPKinaseactivation
[RefKatzung11th/ep.741]
BiguanideactsasanAMP-activatedproteinkinase(AMPK)
activator.
ActivationofAMPKhasahighnumberofpotentially
antiatheroscleroticeffects,includingreducinginflammatorycell
adhesiontobloodvesselendothelium&reducinglipidaccumulation.

544.Mechanismofactionofsulfonylureasis
?
a)NaATPchannelblocker
b)KATPchannelblocker
c)CIATPchannelblocker
d)CaATPchannelblocker
CorrectAnswer-B
Ans.B.KATPchannelblocker
[RefKDT7th/ep.270,274]
Sulfonylureasprovokeabriskreleaseofinsulinfrompancreas.
Theyactonthesocalled"Sulfonylureareceptors"(SUR1)on
pancreatic(beta-cellmembrane-causedepolarizationbyreducing
conductanceofATPsensitiveK.channels.
ThisenhancesinfluxofCa'2--degranulation.
Theydonotcausehypoglycemiainpancreatectomizedanimalsand
type1diabetes(Presenceofatleast30%offunctional[3-cellsin
essentialfortheiraction).
Aminoractionreducingglucagonsecretionbyincreasinginsulinand
somatostatinreleasehasbeendemonstrated.

545.NasallyactingGnRHanalogueis?
a)Goserelin
b)Triptorelin
c)Nafarelin
d)Leuprolide
CorrectAnswer-C
Ans.C.Nafarelin
[RefKDT7thiep.242]
GnRHanaloguesareusedbySCroute,howevernafarilinand
busurelincanbeusedasintranasalspray.
GnRHanaloguescancausehotflushes,lossoflibidoand
osteoporosis.

546.Whichofthefollowingis/areside
effect/sofgrowthhormone
administration?

a)Painatinjectionsite
b)Glucoseintolerance
c)Hypothyroidism
d)Alltheabove
CorrectAnswer-D
Ans.D.Alltheabove
[RefKDT&hiep.234]Adverseeffectsofgrowthhormone
SomatropinandsomatremarerecombinantGHanalogues.
Somatremhasanadditionalmethionineresidueandismore
immunogenicthansomatropin,butallergicreactionsorresistanceto
treatmentarenotaproblem.
Painatinjectionsiteandlipodystrophycanoccur.
Glucoseintolerance,hypothyroidism(duetounmaskingofTSH
deficiency),saltandwaterretention,handstiffiress,myalgias,
headachearethepossibleadverseeffects
Riseinintracranialtensionoccursinfewcases.

547.Whichofthefollowingdrugshalts
macrovascularaswellasmicrovascular
effectsofDM?

a)Acarbose
b)Biguanides
c)Meglitinide
d)Algaliptin
CorrectAnswer-B
Ans.B.Biguanides
[RefKDTTh/ep.276]Biguanides]
*Biguanideslikemetforminarethe1stlinedrugsforthetreatment
ofType2DM,whichactsbyAMPKactivation.
*AdvantagesofBiguanidesareasfollows,
-Nonhypoglycemic.
-Promotesweightloss.
-PreventsmacroaswellasmicrovascularcomplicationsofDM.

548.Whichofthefollowingarenaturally
occurringopioid?
a)Diacetylmorphine
b)Ethylmorphine
c)Morphine
d)Pholcodeine
CorrectAnswer-C
Ans.C.Morphine
[RefKDT7"/ep.474]
Classificationofopioids:
Naturallyoccurringopiumalkaloids:-Morphine,codeine
Semisyntheticopiates:-Diacetylmorphine(heroin),pholcodeine
andethylmorphine.
Syntheticopioids:-Pethidine(Meperidine),fentanyl,methadone,
dextropropoxyphene,tramadol.

549.Mostpotentopioidis?
a)Butorphanol
b)Pentazocine
c)Sulfentanyl
d)Hdrocodone
CorrectAnswer-C
Ans.C.Sulfentanyl
[RefKDTp.476]
Theorderofpotencyofopioids:
Sufentanil>Fentanyl>Buprenorphine>Hydromorphone,
Oxymorphone>Butorphanol>Levorphenol>Oxycodone>
Hydrocodone>Nalbuphine,morphine,Methadone>Pentazocine>
codeine>Mepridine(Pethidine)>Propoxyphane.

550.Allofthefollowingpairsarecorrect
except?
a)Peripheraldecarboxylaseinhibitor-Benserazide
b)MAO-Binhibitor-Clorgyline
c)COMTinhibitor-Entacapone
d)Dopaminefacilitation-Amantadine
CorrectAnswer-B
Ans.B.MAO-Binhibitor-Clorgyline
[Ref:KDTThlep.439&&lep.439;Katzung11th/ep.513]
Note:ClorgylineisaMAOAinhibitornotMAOB.

551.Whichofthefollowingisusedforthe
patientonantiparkinsonianmedication
levodopa+carbidopa,butpatient
showingmarkedon-offeffect?

a)Bromocriptine
b)Amatidine
c)Selegeline
d)Rimonabant
CorrectAnswer-C
Ans.C.Selegeline
[Ref:Harrison18th/ep.3326,3327]
Ifthereiswearingoff(on-offeffect)COMTinhibitororMAO-B
inhibitor(selegiline)isadded

552.Whichofthefollowingisnottrueabout
benzodiazepines?
a)Canproduceataxia
b)HasGABAfacilitatorybutnoGABAmimeticaction
c)REM,andStage3and4sleepisincreased
d)Producesmusclerelaxationbyactiononmedulla
CorrectAnswer-C
Ans.C.REM,andStage3and4sleepisincreased
Ref:KDT6th/ep.394&5th/ep.317,363]
Mechanismofactionofbenzodiazepines(BZDs):
Musclerelaxationisproducedbyactiononmedulla.
Ataxiaisduetoactiononcerebellum.
BZDsactsonGABAAreceptors.
EffectonCNS:
Incontrasttobarbiturates,BZDsarenotgeneraldepressant,but
exertrelativelyselectiveanxiolytic,hypnotic,musclerelaxantand
anticonvulsanteffects.
TheantianxietyactionofBZDsisnotdependentontheirsedative
property-withchronicadministrationreliefofanxietyismaintained,
butdrowsinesswanesoffduetodevelopmentoftolerance.
Stage2sleepisincreased,whileREM,Stage3&4sleepare
decreased.

553.Whichofthefollowingisnottrueabout
barbiturate?
a)ShowsGABAmimeticaction
b)ShowsGABAfacilitatoryaction
c)ItcandepressvoltagegatedNa.-andIC'channelsathigh
concentrations
d)Limbicsystemismostsensitivetothedepressiveactionof
barbituratestoCNS
CorrectAnswer-D
Ans.D.Limbicsystemismostsensitivetothedepressive
actionofbarbituratestoCNS
[RefKDT7th/ep.399&6th/ep.391]
ActiononCNS:
BarbituratesactprimarilyattheGABA-BZDreceptor-Clchannel
complexandpotentiateGABAergicinhibitionbyincreasingthe
lifetimeofCl-channelopeningcausedbyGABA(Contrast
benzodiazepineswhichenhancefrequencyofCIchannelopening)-
GABAfacilitatoryaction.
BarbituratesdepressallareasofCNS,butthereticularactivating
systemismostsensitive.

554.Whichofthefollowingcanbeusedinthe
managementoftardivedyskinesia?
a)Cessationofantipsychoticmedication
b)Baclofen
c)Tetrabenazine
d)Alltheabove
CorrectAnswer-D
Ans.D.Alltheabove
[RefKDT7th/ep.445,Harrisons18th/ep.3333]
TardiveDyskinesia:
Commonestofthetardivesyndromesandistypicallycomposedof
choreformmovementsinvolvingthemouth,lips,andtongue.
Atypicalantipsychotics(e.g.,clozapine,risperidone,olanzapine,
quetiapine,ziprasidone,andaripiprazole)areassociatedwitha
significantlylowerriskofTDincomparisontotraditional
antipsychotics.
Treatmentprimarilyconsistsofstoppingtheoffendingagent.
Ifthepatientisreceivingatraditionalantipsychoticandwithdrawalis
notpossible,replacementwithanatypicalantipsychoticshouldbe
tried.
Inrefractorycases,catecholaminedepleterssuchastetrabenazine
maybehelpful.Tetrabenazinecanbeassociatedwithdose
dependentsedationandorthostatichypotension.
Otherapproachesincludebaclofen(40-80mg/d),clonazepam(1-8
mg/d),orvalproicacid(750-3000mg/d).

555.Temazepamissuperiortodiazepamin?
a)Longerdurationofaction
b)Safelyusedinliverfailure
c)Noactivemetaboliterequired.
d)Highhepaticmetabolism
CorrectAnswer-B:C
Ans.B.Safelyusedinliverfailure&C.Noactivemetabolite
required.
[RefKDT7'/ep.405]
Temazepam
Intermediateactingbenzodiazepine.
Itsmetabolismisindependentofliverandhence,canbesafely
giveninpatientswithhepaticfailure.
Temazepamdoesnothaveanactivemetabolitelikediazepam
[Note:Diazepamisconvertedtoanactivemetabolite`desmethyl
diazepam'(oxazepam)].

556.Drugcontraindicatedinabsence
seizuresis
a)Lamotrigine
b)Clonazepam
c)Tiagabine
d)Ethosuximide
CorrectAnswer-C
Ans.C.Tiagabine
Carbamazepine,vigabatrin,andtiagabinearecontraindicatedinthe
treatmentofabsenceseizures,irrespectiveofcauseandseverity.

557.Mechanismofactionoftianeptinis:?
a)Increase5HTuptake
b)Decrease5HTuptake
c)IncreaseDAuptake
d)DecreaseDAuptake
CorrectAnswer-A
Ans.A.Increase5HTuptake
[RefKDT7th/ep.447]
Tianeptin
Atypicalanti-depressant
Increases5HTuptakeandisneithersedativenorstimulant.
Shownefficacyinanxio-depressivestates,particularlywith
psychosomaticsymptoms,aswellasendogenousdepression.
Sideeffectsinclude:drymouth,epigastricpain,flatulence,
drowsiness/insomnia,tremorandthebodyache.

558.VenalafaxineisanFDAapproveddrug
forthetreatmentof?
a)Majordepression
b)Generalisedanxietydisorder
c)Panicdisorder
d)Alltheabove
CorrectAnswer-D
Ans.D.Alltheabove
[RefMedicalbasisofpsychiatarybySHossein,p.79]
Venlafaxine
Itisserotoninandnoradrenalinereuptakeinhibitors(SNRI),butin
contrasttoolderTCAsitdoesnotinteractwithcholinergic,
adrenergic,histaminergicreceptorsanddoesnothavesedative
property.
Itisfasteracting(otherfasteractingantidepressantsarebupropion
andmirtazapine).
ItraisestheBP(allotherantidepressantscausehypotension).
ItisFDAapprovedforuseinmajordepression,generalizedanxiety
disorder,andpanicdisorder.

559.FDAapproveddrugforrefractory
schizophrenia?
a)Amoxapine
b)Haloperidol
c)Clozapine
d)Penfluridol
CorrectAnswer-C
Ans.C.Clozapine
[RefKDTriep.441]
ClozapineisFDAapproveddrugforresistantschizophrenia
Itreducestheriskofsucidalbehaviorinyoungerpatientswith
schizophrenia.

560.Trueaboutanti-Parkinsondrug
levodopais:?
a)Levodopaisanactivemetaboliteofdopamine.
b)About50%ofadministeredlevodopaisperipherallyconverted
tocarbidopa.
c)About2%oftheadministeredlevodopacrossesbloodbrain
barrier.
d)Levodopahasnoroleinhepaticcoma.
CorrectAnswer-C
Ans.C.About2%oftheadministeredlevodopacrossesblood
brainbarrier.
[RefKDT7th/ep.426]
Levodopa:
Levodopaisinactivebyitself,butistheimmediateprecursorofthe
neurotransmitterdopamine.
95%oftheoraldoseisdecarboxylatedintheperipheraltissues.
About1-2%oftheadministeredlevodopacrossesthebloodbrain
barriertoreachthebrain,whichistakenupbythesurviving
dopaminergicneuronsandconvertedtodopamine.
ThestimulationofexcitatoryDIaswellastheinhibitoryD2receptors
instriatumachievesthesameneteffectofsmootheningmovements
andreducingmuscletone.
Itproducesanon-specific'awakening'effectinthehepaticcoma.

561.Whichofthefollowingistrueabout
ziprasidone?
a)Profoundextrapyramidalsymptoms
b)Causesweightloss
c)Hasanti-depressantproperties
d)Safeincardiacpatients.
CorrectAnswer-C
Ans.C.Hasanti-depressantproperties
[RefKDT7th/ep.442]
Ziprasidone:
Atypicalanti-psychotic
Ithasa5H1+5HT2+D2blockingactivitywithantagonismat
5HT1Dandagonistat5HT1A.
Beingatypicalitisfreefromextrapyramidalsideeffects(EPS)and
hasgotgoodanxiolyticandantidepressantproperties.
ItcausesmildweightgainandhyperglycemiaandcausesQTc
prolongationandseriouscardiacarrhythmias.

562.Whichofthefollowingnephrotoxic
drugsshouldbecompletelyavoidedin
renalfailure?

a)Doxycycline
b)Talampicillin
c)Nitrofurantoin
d)Nalidixicacid
CorrectAnswer-B:C:D
Ans.B,TalampicillinC.Nitrofurantoin&D.Nalidixicacid
Drugstobeavoidedinrenalpatients:
Cephalothin
Talampicillin
Nalidixicacid
Tetracyclines(exceptdoxycyline)
Nitrofurantoin

563.Thetypicalmaintencedoseof
Leveteracetamis?
a)10-20mg/Kg/day
b)20-30mg/Kg/day
c)30-40mg/Kg/day
d)40-50mg/Kg/day
CorrectAnswer-C
Ans.C.30-40mg/Kg/day
[RefKDT7hlep.420]
Levetiracetam
Uniqueanti-convulsantwhichhasshownefficacyinrefractory
partialseizureswithorwithoutgeneralization.
Dosage:
Therecommendedstartingdoseis10mg/Kg/daygiventwicedaily.
Thedosagecanbeincreasedweeklyorbiweeklyby10mg/Kg/day.
Thetypicalmaintenancedoseis30-40mg/Kg/day.

564.Botulinumtoxinisusedintreatmentof?
a)Axillaryhyperhidrosis
b)Blepharospasm
c)Cervicaldystonia
d)Alloftheabove
CorrectAnswer-D
Ans.D.Alloftheabove
[RefKDT7th/ep.99]
BotulinumtoxinAandBarehighlypotentexotoxinsproducedby
Clostridiumbotulinumwhichcauses`botulinum'byinhibitingAch
release.
Itisusedintreatmentofblepharospasm,spasticcerebralpalsy,
strabismus,spasmodictorticollis,nystagmus,hemifacialspasm,
axillaryhyperhidrosis,spasticcerebralpalsywhichisdueto
cholinergicexcess.
Ithasalsobeenusedforfacialwrinkles.

565.Whichofthefollowingantipsychotics
showpartialD2agonistactivity?
a)Aripiprazole
b)Clozapine
c)Quetiapine
d)Ziprasidone
CorrectAnswer-A
Ans.A.Aripiprazole
[RefGoodman6Gillman1p.464,465,466;Katzung11"'/ep.495]
Aripiprazole:
OnlyantipsychoticwithD2agonisticactivity.(allothersareD2
antagonists).
Longestacting
Italsohas5HT1Aagonisticand5HT2antagonisticactivity-Also
knownasdopamine-serotininestabilizer.
Itisleastsedatingantipsychoticcancauseinsomnia.

566.Whichdrugisgiveninthepaindueto
diabeticneuropathy?
a)Lamotrigine
b)Navalproate
c)Gabapentin
d)Morphine
CorrectAnswer-C
Ans.C.Gabapentin
[RefKDT7th/ep.409]
Gabapentin
ThisislipophilicGABAderivatve.
GabapentinenhancesGABAreleasebutitselfdoesnotactasa
agonistattheGABAAreceptor.
ItcrossestothebrainandenhancesGABAreleasebutdoesnotact
asagonistatGABAAreceptor.
Gabapentinisconsideredtobeafirstlinedrugforpaindueto
diabeticneuropathyandpostherpeticneuralgia.
Uses
GTCSandpartialseizures
Painduetodiabeticneuropathy
Postherpeticneuralgia
Prophalaxisofmigraine

567.Topicalantifungalofchoicefor
aspergillusinfectionofeyeis?
a)Miconazole
b)Clotrimazole
c)Econazole
d)Fluconazole
CorrectAnswer-B
Ans.B.Clotrimazole
[RefKhuranaOphthalmology4th/ep.422]
Clotrimazole:
FungistaticandiseffectiveagainstCandida,Aspergillusandmany
others.
Its1percentsuspensioniseffectivetopicallyandisthetreatmentof
choiceinAspergillusinfectionsoftheeye.

568.Whichofthefollowingisatopical
antifungalagent?
a)Benzylbenzoate
b)Brimetenide
c)Butenafine
d)Posconazole
CorrectAnswer-C
Ans.C.Butenafine
[RefKDT7thiep.796]
Butenafine:
Butenafineisabenzylaminecongeneroftheterbinafine,whichacts
byinhibitingtheenzymesqualeneepoxidase.
Usedonlytopicallyfortreatmentofdermatophytosis.
Efficacyintineacruris/corporis/pedisissimilartothatofterbenafine.

569.Fastestactingantimalarialdrugis?
a)Chloroquine
b)Quinine
c)Mefloquine
d)Artether
CorrectAnswer-D
Ans.D.Artether
[RefHarrisons18th/ep.26-2]
Artemisininderivatives:
Artesunate,artemether,arteether,andtheparentcompound
artemisininaresesquiterpenelactonesderivedfromthewormwood
plantArtemisiaannua.
Thesearethefastestactingerythrocyticschizontocide.

570.Whichofthefollowingincreases
AmphoterecinBinducednephrotoxicity?
a)Vancomycin
b)Cyclosporin
c)Acyclovir
d)Alltheabove
CorrectAnswer-D
Ans.D.Alltheabove
[RefKatzung11th/ep.836]
"Aminoglycosides.vancomycin,cyclosporineerothernephrotoxic
drugsenhancerenalimpairmentcausedbyAMB"
RiskfactorsforamphotericinBinducednephrotoxicity
Followingriskfactorsincreasethechancesofnephrotoxicity
causedbyamphotericin:
Concomitantuseofdiuretics.
Abnormalbaselinerenalfunction(kidneydiseases)
Dehydration(volumedepletion):-Itisakeyfactorfortherenal
toleranceofallthepotentialnephrotoxicdrugs.Therefore,all
patientsshouldreceive1-2litersofisotonicsalinepriorto
amphotericinBinfusion.
Concomitantuseofnephrotoxicdrugs:-Aminoglycosides,
vancomycin,cyclosporine,tacrolimus(FK-506),acyclovir,NSAIDS,
radio-contrastagents
HigheraveragedailydoseofamphotericinB.

571.Erythromycinisusedinthetreatmentof
whichGITdisorder?
a)Bacillarydysentery
b)Amoebicdysentery
c)Diabeticgastroparesis
d)Ulcerativecolitis
CorrectAnswer-C
Ans.C.Diabeticgastroparesis
[RefKDT7h/ep.753]
Erythromycin:
Macrolideantibiotic.
Erythromycinstimulatesmotilin(anuppergastrointestinalpeptide
hormone)receptorsintheGITwhichinducesgastriccontractions,
hastensgastricemptyingandpromotesintestinalmotilitywithout
significanteffectoncolonicmotility.
Ithasbeenoccasionallyusedtoaffordshorttermsymptomaticrelief
indiabeticgastroparesis,however,undesirablealterationofgutflora
limitsitsuse.

572.Whichmacrolideisactiveagainst
Mycobateriumleprae?
a)Azithromycin
b)Roxithromycin
c)Clarithromycin
d)Framycetin
CorrectAnswer-C
Ans.C.Clarithromycin
[RefKDT7th/ep.754,780,Harrisons18thiep.1364-65]
Clarithromycinisanewermacrolide,whichiseffectiveagainstMAC
(Mycobacteriumaviumcomplex),atypicalmycobacteriaand
Mycobacteriumleprae.

573.Ivermectinisthedrugofchoicefor
whichofthefollowinginfections?
a)Trichuriasis
b)Onchocerciasis
c)Loiasis
d)Trichinosis
CorrectAnswer-B
Ans.B.Onchocerciasis
[RefKDT7h/ep.853]
Ivermectin:
Extremelypotentsemisyntheticderivativeofnematodalprincipal
obtainedfromStreptomycesavermitilis.
DOCforsingledosetreatmentofonchocerciasisand
strongyloidosis.
Onlyoraldrugeffectiveagainstscabiesandpediculosis.
ActsbyglutamategatedCI-channelwhichcausestonicparalysisin
nematodes.

574.Whichofthefollowingcausesretinal
pigmentation?
a)Quinine
b)Chloroquine
c)Mefloquine
d)Atovaquone
CorrectAnswer-B
Ans.B.Chloroquine
[RefKDT7thiep.823]
Chloroquine:
1stlineantimalarialdrug,whichisanerythrocyticschizonticide.
However,itsprolongeduseofhighdoses(asinDLE,rheumatoid
arthritis)maycauselossofvisionduetoretinaldamageintheform
ofretinalpigmentation.
Causecornealdepositsandaffectvisionandarereversibleon
discontinuation.
Lossofhearing,rashes,photoallergy,myopathy,grayingofhairmay
occur.

575.MechanismofactionofLinezolidis:?
a)Inhibits30Sribosomesubunitof50Sribosome
b)Inhibits23Sribosomesubunitof50Sribosome
c)Inhibits5Sribosomesubunitof50Sribosome
d)Inhibits5PS.ribosomesubunitof50Sribosome
CorrectAnswer-B
Ans.B.Inhibits23Sribosomesubunitof50Sribosome
[RefKDT7111ep.758]
Linezolid:
An`oxazoledianones'usefulintreatmentofMRSAandsomeVRSA
strains
Itactsbyinhibitingbacterialproteinsynthesisbybindingto23S
fractionof50Sribosome.
BindingofthelinezoliddistortsthetRNAbindingsiteoverlapping
both50Sand30Sribosomalsubunitsandstopstheprotein
synthesis.
Itisapredominantlybacteriostaticdrug.

576.Whichofthefollowingisnottrueabout
levamisole?
a)Itisthelevoisomeroftetramisole
b)Ithasimmunomodulatoraction
c)Itcankillstrongyloideslarvaeandadultworms
d)Itisusedagainstascariasisandancyclostomiasis
CorrectAnswer-C
Ans.C.Itcankillstrongyloideslarvaeandadultworms
[RefKDT7ffilep.852]
Levamisole,Tetramisole:
Tetramisolewasdevelopedinthelate1960s.
Itisrecemic;itslevoisomer(levamisole)wasfoundtobemore
activeandispreferrednow.
Bothareactiveagainstmanynematodes,butuseisrestrictedto
ascariasisandancylostomiasis,becauseactiononotherwormsis
poor.
Strongyloideslarvaearekilled,butadultwormsarenotsensitive.
Thegangliainwormsarestimulatedcausingtonicparalysisand
expulsionofliveworms.
Interferencewithcarbohydratemetabolism(inhibitionoffumarate
reductase)mayalsobecontributing.
Levamisoleisanimmunomodulatoraswell.Levamisolerestores
depressedTcellfunction.
Itwasonceusedasadiseasemodifyingdruginrheumatoidarthritis
andasanadjunctinmalignancies,aphthousulcers,recurrent
herpesbutrepeateddosesproducereactionsandnowithasbeen
withdrawn.

577.Whichofthefollowingisasideeffectof
clofazimine?
a)Reddishblackskindiscoloration
b)Hemolyticanaemia
c)Flulikesyndrome
d)Axillaryfreckling
CorrectAnswer-A
Ans.A.Reddishblackskindiscoloration
[RefKDT7thlep.781,782]
Clofazamine:
1stlineanti-leproticdrug
Dyewithleprostaticandanti-inflammatoryproperties.
Getsaccumulatedinmacrophagesandgetsdepositedinmany
tissues.Includingsubcutaneousfatasneedleshapedcrystals
responsibleforreddishblackskindiscoloration.
Discolorationofskin,hairandbodysecretionsmayoccur.
Conjunctivalpigmentationmayalsooccur.

578.MechanismofactionofNiclosamideis:?
a)Inhibitionofsubstratelevelphosphorylation
b)Inhibitionofoxidativephosphorylation
c)Inhibitionofprotoneffluxpumps
d)Increaseproductionoffreeradicals
CorrectAnswer-B
Ans.B.Inhibitionofoxidativephosphorylation
[RefKDT7th/ep.854]
Niclosamideactsbyinhibitingoxidativephosphorylationin
mitochondriaandbyinterferingwithanaerobicgenerationofATP.
Itismainlyactiveagainstcestodes-T.solium,T.saginata,
DiphyllobothriumlatumandH.nana.

579.Forsystemicmycosisfluconazoleis
preferredoverketoconazolebecauseof-
a)Greaterefficacy
b)Longert1/2
c)Lessersideeffects
d)Alltheabove
CorrectAnswer-D
Ans.D.Alltheabove
[RefKDT&lep.761]
ImidazolesandTriazoles:
Thesearepresentlythemostextensivelyusedantifungaldrugs.
Fourimidazolesareentirelytopical,whileketoconazoleisusedboth
orallyandtopically.
Twotriazolesfluconazoleanditraconazolehavelargelyreplaced
ketoconazoleforsystemicmycosisbecauseofgreaterefficacy,
Longert1/2,fewersideeffectsanddruginteractions.
Theimidazolesandtriazoleshavebroadspectrumantifungalactivity
coveringdermatophytes,Candida,otherfungiinvolvedindeep
mycosis(exceptmucor),Nocardia,somegrampositiveand
anaerobicbacteria,e.g.Staph.aureus,Strep.faecalis,Bac.fragilis
andLeishmania.
Themechanismofactionofimidazolesandtriazolesisthesame.
TheyinhibitthefungalcytochromeP450enzymelanosterol14-
demethylase'andthusimpairergosterolsynthesisleadingtoa
cascadeofmembraneabnormalitiesinthefungus.
Thelowerhosttoxicityoftriazolescomparedtoimidazoleshas
correlatedwiththeirloweraffinityformammalianCYP450enzymes
andlesserpropensitytoinhibitmammaliansterolsynthesis.

However,becausetheyareactiveagainstcertainbacteriaaswell
(whichdonothaveergosterol),othermechanismsofactionalso
appeartobeinvolved.

580.INHhepatotoxicityisduetowhich
compound?
a)INHacetylhydrazine
b)INHsulfhydrazine
c)INHmethylhydrazine
d)Alloftheabove
CorrectAnswer-A
Ans.A.INHacetylhydrazine
[RefKDT7thlep.767]
INHisextensivelymetabolizedinliver,mostimportantpathway
beingN-acetylationbyNATZ.
TherateofINHacetylationshowsgeneticvariationwithsomebeing
Fastacetylators-30to40%patients.
Slowacetylators-60to70%patients.
Isoniazidinducedperipheralneuritisismorecommoninslow
acetylators.
HepatotoxicminormetaboliteisproducedbyCYP2E1from
acetylhydrazine.

581.Whatistheeffectofcoadministrationof
rifampicinandritonavirinpatients
sufferingfromAIDS?

a)AreaUnderCurvedecreasedby15%
b)AreaUnderCurvedecreasedby35%
c)AreaUnderCurveincreasedby15%
d)AreaUnderCurveincreasedby35%
CorrectAnswer-B
Ans.B.AreaUnderCurvedecreasedby35%
[RefKucers:theuseofantibiotics6thednbyMLindsayGrayson
p.1598]
Coadministrationofritonavirwithrifampicin:
Recommendationistousethecombinationwithcaution.
Areaundercurvedecreasedby35%
Thereisnochangeinrifampicinconcentration
Itisrecommendedtomonitortheantiretroviralactivityofritonavir

582.Drugofchoiceforsurgicalprophylaxis
is:?
a)Cefaclor
b)Ceftizoxime
c)Cefazolin
d)Cefoperazone
CorrectAnswer-C
Ans.C.Cefazolin
[RefKDT7th/ep.726]
Cefazolinistheprototype1stgenerationcephalosporinthatisactive
againstPnGsensitiveorganismsiestreptowcce,gonococciand
group.
Itisthepreferredparenteralfirstgenerationcephalosporinfor
surgicalpeophylaxis.

583.Vapiprostisa?
a)Thromboxanereceptorantagonist
b)Thromboxanesynthetaseantagonist
c)PGE1analogue
d)PGI2analogue
CorrectAnswer-A
Ans.A.Thromboxanereceptorantagonist
[RefInternetncbiarticle]
Vapiprost:
Novelcongener,whichisarecentlydevelopedthromboxane
receptorantagonist.
Itpreventsplateletaggregation,preventsthrombusformationand
therebypreventingvesselocclusion.
ItisusuallyusedalongwithrtPAlikealteplase,renecteplase,
tenecteplase.

584.Treatmentofclopidogreltoxicitycanbe
donewith?
a)Wholehumanblood
b)Platelettransfusion
c)vWftransfusion
d)rFVIIainfusion
CorrectAnswer-D
Ans.D.rFVIIainfusion
rFVIIa(RecombinantfactorVila)hasbeenshowntorestore
thrombingenerationinclopidogreltreatedbloodsamples,and
shortenthrombingenerationlagtimeinpatientswhohadbeen
treatedwithaspirinandclopidogrel,andinbloodsamplestreated
withclopidogrel'sactivemetabolite.
Thustheycanbeusedforthemanagementofclopidogreltoxicity
inducedbleedingi.e.canbeusedtoreversetheeffectsof
clopidogrel.

585.Whyisclopidogrelpreferredover
ticlopidine?
a)Lowerincidenceofneutropeniaandthrombocytopenia
b)Lowerincidenceofdyslipidemia
c)Lowerincidenceofhyperglycemia
d)Lowerincidenceofposturalhypotention
CorrectAnswer-A
Ans.A.Lowerincidenceofneutropeniaandthrombocytopenia
[RefKDT6thiep.610]
Clopidogrel:
Thisnewercongenerofticlopidinehassimilarmechanismofaction,
abilitytoinhibitplateletfunctionandtherapeuticefficacy,but
appearstobesaferandbettertolerated(CLASSICSstudy).
Clopidogrelissaferthanticlopidineasitislessassociatedwith
hematologicaldyscrasiasthanuseofticlopidine.
Alowerfrequencyofneutropenia,thrombocytopeniaandotherbone
marrowtoxicitycomparedtoticlopidinehasbeenrecorded.
Theclopidogrelasaspirininpatientsatriskofischaemicevents
(CAPRIE)trialhasfoundclopidogrelrecipientstohaveaslightly
lowerannualriskofprimaryischaemiceventsthanaspirin
recipients.

586.Whichofthefollowingantilipidemicdrug
isasterolabsorptioninhibitor?
a)Gemfibrozil
b)Simvastatin
c)Nicotinicacid
d)Ezetimibe
CorrectAnswer-D
Ans.D.Ezetimibe
[RefKDT7th/ep.635]3
Classificationofantihyperlipidemicdrugs
HMGCoAreductaseinhibitor:
Statins:Lovastatin,Simvstatin,Pravastatin,Atorvastatin,
Rosuvastatin
Bileacidsequestrants
Resins:Colestipol,Cholestyramine
LPLactivator/PPARalphaactivator-Clofibrate,gemfibrozil,
fenofibrate.
Sterolabsorptioninhibitor-Ezetimibe
Lipolysis&TGsynthesisinhibitor-Nicotinicacid

587.MechanismofactionofTorcetrapribis?
a)Bileacidsequestrant
b)Sterolabsorptioninhibitor
c)Lipoproteinlipaseactivator
d)CETPinhibitors
CorrectAnswer-D
Ans.D.CETPinhibitors
[Ref:KDT7hlep.641]
CETPInhibitors:
CETP,i.e.,Cholesterylestertransferproteinsinhibitorsareclassof
cholesterolloweringagents.
CETPfacilitatesexchangeofCHEwithT4betweenHDLand
chylomicrons,VLDLandLDLwhichplaysanimportantroleinthe
disposalofHDLassociatedCH.
ExamplesofCETPinhibitors:Torcetrapib&Anacetrapib
Clinicaltrialshavebeenperformedandtorcetrapibwasfoundto
increaseoccurrenceofcardiovasculareventslikeangina,MI,heart
failureanddeath.

588.MostpotentH2antagonistis:?
a)Ranitidine
b)Cimetidine
c)Famotidine
d)Nizatidine
CorrectAnswer-C
Ans.C.Famotidine
[RefKDT7h/ep.650&6thlep.629;Katzung1lthlep.1070]
H2-receptorantagonists
Drugsinthisgrouparecimetidine,ranitidine,famotidine,roxatidine,
nizatidineandloxatidine.
FamotidineisthemostpotentH2blocker.Famotidinehassome
inverseagonisticactiononH2receptors.
AllH2-blockersarecompetitiveblockersexceptfamotidine
(competitive-noncompetitive)andloxatidine(noncompetitive).
H2blockersareusuallygivenfor6-8weeks.

589.Notanadverseeffectofcimetidine?
a)Confusionalstate,restlessness
b)Gynecomastia
c)Drymouth
d)Decreasedprolactinlevels
CorrectAnswer-D
Ans.D.Decreasedprolactinlevels
[Ref:KDT7h/ep.650]
Cimetidine:
H2blockerwhichhasbeenreplacedbynewcongenersduetoits
manyadverseeffects.Followingareitsadverseeffects:
Welltoleratedbymostpatients:adverseeffectsoccurin<5%.
Thesearegenerallymild.
Headache,dizziness,bowelupset,drymouth,rashes.
CNSeffectslikeconfusionalstate,restlessness,convulsionsand
comahaveoccurredinfrequentlyinelderlypatients,inthosewith
renalimpairment,especiallywithlargedosesinfused

590.Useofmetoclopramidebeyondweeks
increasesthechancesoftardive
dyskinesia?

a)8
b)12
c)16
d)20
CorrectAnswer-B
Ans.B.12
[RefPharmacologyandphysiologyforanesthesiabyHughE.
Hemmingsp.511]
FDAhasissuedablackboxwarningformetoclopramide,giventhe
highiskofdevelopingtardivedyskinesiaifmetoclopramideuse
extendsbeyond12weeks.

591.ThetwomoleculesofAminosalicylate
coupledviaazobondform?
a)Mesalazine
b)Olsalazine
c)Balsalazine
d)Sulfasalazine
CorrectAnswer-B
Ans.B.Olsalazine
[RefKDT7h/ep.684]
Olsalazine?2moleculesofaminosalicylate(ASA)coupledbyazo
bonds.
Balsalzine5ASAlinkedto4?aminobenzoyl-(3alanine.
Sulfasalazine5ASAlinkedtosulfapyridinebyazobond.
Mesalazine:-pHcoatedtabletof5ASA

592.Naturalanticancerdrugis?
a)Paclitaxel
b)Methotrexate
c)Cyclophosphomide
d)Alloftheabove
CorrectAnswer-A
Ans.A.Paclitaxel
[RefGoodman&Gilman11th/ep.1350-1354;Katzung1p.949]
Antimitoticdrugsofnatural(plant)sources.
Vincristine
Vindesine
Paclitaxel
Vinblastine
Vinorelbine
Dacetaxel
Othernaturalanticancerdrugsare
Epipodophyllotoxins(Etoposide)
Bleomycin
Anthracyclins(Doxorubicin,daunorubicin)
Camptothecins
Mitomycin'C'
Actinomycin'D'

593.Drugactingon'S'phaseofcycle?
a)Chlorambucil
b)Methotrexate
c)Vincristine
d)Paclitaxel
CorrectAnswer-B
Ans.B.Methotrexate[RefKDT7"lep.823]
Cellcyclespecific
Theykillactivelycyclingcellandtheirsiteofactionisconfinedto
onephaseofthecellcycle.
Drugscausingcellcyclespecificinhibitionaregivenbelow:
G,phase:Vinblastine
Sphase:Mtx,cytarabine,6-TG,6-MP,Hydroxyurea,mitomycin
G2phase:Bleomycin,etoposide,Daunorubicin,Topotecan
Mphase:Vincristine,vinblastine,paclitaxel,dacetaxel,
Ixabepilone,Estramustine.

594.Mechanismofactionofaprepitantis?
a)RANKligandinhibitor
b)MMDAantagonist
c)NK1receptorantagonist
d)5HT3antagonist
CorrectAnswer-C
Ans.C.NK1receptorantagonist
[RefGoodman&Gillman's11`"/ep.1005]
Aprepitant
Aprepitantisanantiemeticsubstancethatbelongstoaclassof
drugscalledsubstancePantagonists.
Thecompoundmediatesitseffectbyblockingtheneurokinin1
(NK1)receptor.
Aprepitantcrossesthebloodbrainbarrier.
Aprepitantisusedforchemotherapyinducednausea&vomiting
(CINV),andpost-operativenausea&vomiting(PONY).
Afterabsorptionaprepitantisboundextensivelytoplasmaproteins
(>95%);itismetabolizedavidly,primarilybyhepaticCYP3A4andis
excretedinthestools.

595.UseofHAARTisassociatedwith
hepatotoxicityapproximatelywhat
percentageofpatients?

a)10%
b)20%
c)30%
d)40%
CorrectAnswer-A
Ans.A.10%
[RefHarrisons18th/ep.2566]
HighlyActiveAntiretroviralTherapy(HAART):
Combinationantiretroviraltherapy(cART),alsoreferredtoashighly
activeantiretroviraltherapy(HAART),isthecornerstoneof
managementofpatientswithHIVinfection.
Indirecthyperbilirubinemia,resultingfromdirectinhibitionof
bilirubinconjugatingactivitybyUDP-glucuronosyltransferase,usually
withoutelevationofaminotransferaseoralkalinephosphatase
activities,occursin-10%ofpatientstreatedwiththeprotease
inhibitorindinavir.

596.Bendamustineisusefulforthe
managementof?
a)Chroniclymphoidleukemia
b)Coloncarcinoma
c)Breastcarcinoma
d)Renalcarcinoma
CorrectAnswer-A
Ans.A.Chroniclymphoidleukemia
[RefHematologyBasicprinciplesandpracticebyRonaldHoffman,
p.819]
Bendamustine
Itisachemotherapymedicationusedinthetreatmentofchronic
lymphocyticleukemia(CLL),multiplemyeloma,andnon-Hodgkin's
lymphoma.
ItworksbyinterferingwiththefunctionofDNAandRNA

597.Abraxaneisa:?
a)Albuminbounddocetaxel
b)Globulinbounddocetaxel
c)Albuminboundpaclitaxel
d)Globulinboundpaclitaxel
CorrectAnswer-C
Ans.C.Albuminboundpaclitaxel
[RefKatzung11thlep.1235]
Abraxaneisanovelalbuminboundpaclitaxelwhichhasgotlesser
propensitytocausehypersensitivityreactionsthanpaclitaxel.
Abraxaneisuseinthetreatmentoftheadvancedmetastaticbreast
carcinomapatients.
Paclitaxelactsbyinhibitingmicrotubulinpolymerizationandinhibits
divisionofmetastaticcells.

598.AbataceptbindstoonTcellsurface
a)CDII
b)CD20
c)CD22
d)CD28
CorrectAnswer-D
Ans.D.CD28
[RefKatzung11th/ep.634]
Abatacept
Itisafusionproteinthatcombinestheextracellulardomainofthe
moleculeCTLA4(CD154)withtheFcportionofahuman
immunoglobulin.
Itinterferewiththeinteractionsbetweenantigenpresentingcellsand
Tlymphocytes.
Therefore,itaffectsearlystagesinthepathogeniccascadeofevent
inRA.
CTLA4hashighaffinityforCD28,whenabataceptbindstoCD28
onTcellsurface,itpreventsthesecondsignalfrombeingdelivered,
thusturningdowntheTcellresponse.

599.Cardiotmdcityisthesideeffectof:?
a)Bleomycin
b)Topotecan
c)Rubidomycin
d)Procarbazine
CorrectAnswer-C
Ans.C.Rubidomycin
[RefKDT7th/ep.826]
Anthracyclineinducedcardiotoxicity
Rubidomycinanddoxorubicinaretheanthracyclineanti-tumor
antibiotics
Theyhavecardiotoxicityasaadverseeffect.
ThiscanmanifesteitheracutelywithECGchanges,arrhythmiasand
hypotension,whicharereversible,orbedelayedlikeCHF.

600.Tocilizumabactsasanantagonistat
whichreeptor-
a)ILI
b)IL2
c)IL6
d)TNF
CorrectAnswer-C
Ans.C.IL6
[RefHarrisons18thiednp.2750]
Tocilizumabisahumanizedmonoclonalantibodydirectedagainst
themembraneandsolubleformsoftheIL-6receptor.
IL-6isaproinflammatorycytokineimplicatedinthepathogenesisof
RA,withdetrimentaleffectsonbothjointinflammationanddamage.
IL-6bindingtoitsreceptoractivatesintracellularsignalingpathways
thataffecttheacutephaseresponse,cytokineproduction,and
osteoclastactivation.

601.MechanismofactionofBasiliximabis?
a)TNFainhibitor
b)IL1antagonist
c)IL2antagonist
d)IL6antagonist
CorrectAnswer-C
Ans.C.IL2antagonist
[RefKDT7`/ep.878,884]
BasiliximabandDaclizumab
TheyarehighlyhumanizedchimericmonoclonalantiCD25
antibodywhichbindstoandactsasIL2receptorantagonist.
Combinedwithotherimmunosuppressantslikeazathioprineand
MMFtopreventrenalandothertransplantrejectionreactions.
Plasmat1/2ofDaclizumabisaround3weekswhichismuchlonger
thanBasiliximab.

602.Estramustineisacombinationof?
a)Estradiol+normustine
b)Estriol+normustine
c)Estriol+mechloroethamine
d)Estriol+cyclophosphamide
CorrectAnswer-A
Ans.A.Estradiol+normustine
[RefKDT7thiep.858,866]
Estramustine
Complexofestradiolandnitrogenmustardnormustinewhichhas
weakestrogenicbutnoalkalytingproperty.
Itbindstoptubulinandinterfereswithitsorganizationof
microtubulesexertinganti-mitoticaction.

603.Pemetrexedisindicatedforuseinwhich
ofthefollowing?
a)Mesoepithelioma
b)Nonsmallcelllungcarcinoma
c)Ewingssarcoma
d)Osteosarcoma
CorrectAnswer-A:B
Ans.A.Mesoepithelioma&B.Nonsmallcelllungcarcinoma
[Ref:KDT7th/ep.858,863]
Pemetrexedisanewercongenerofmethotrexatewhichprimarily
targetstheenzymethymidylatesynthase.
Uses:Incombinationwithcisplatin,pemetrexedisapprovedfor
treatmentofmesoepitheliomaandnonsmallcelllungcarcinoma.

604.Whichofthefollowingisnottrueabout
purineantagonists?
a)Azathioprineisusedasimmunosuppresant
b)Drugsareactivatedbyhypoxanthineguaninephosphoribosyl
transferase[HGPRTase]
c)FludoribineisthedrugofchoiceforCLL
d)Cladarabineisdegradedbyadenosinedeaminase
CorrectAnswer-D
Ans.D.Cladarabineisdegradedbyadenosinedeaminase
[RefKatzungIlth/ep.948]
Purineantagonists
6-mercaptopurine(6-MP),6-thioguanine,fludarabineand
cladarabinearepurineanaloguesthatareusedincancer
chemotherapy.
CladarabineistheDOCforHairycellleukemiaasitisresistantto
degradationbyadenosinederminase.

605.Panitumumabisusedforwhichcancer?
a)Coloncancer
b)Lungcancer
c)Breastcancer
d)Osteoclastoma
CorrectAnswer-A
Ans.A.Coloncancer
[RefHarrisons18thiep.677]
Panitumumab
ItisamonoclonalantibodyagainstEGFreceptor.Itisusedfor
managementof(Colorectal)coloncancer.

606.Cetuximabisusedtotreat?
a)Adamantinoma
b)BasalcellCa
c)ColorectalCa
d)Crohn'sdisease
CorrectAnswer-C
Ans.C.ColorectalCa
[RefKDT7th/ep.870,Harrisons18thiep.677]
Cetuximab
CetuximabisEGF(Epithelialgrowthfactor)receptorantibody,which
preventscellgrowth,proliferationandmetastasis.
ItisapprovedforHeadneckandfacesquamouscellCaasadjuvant
withcisplatin.
ItisalsousedforEGFreceptor+vecolorectalCa.

607.Whichofthefollowingisnotanadverse
effectofcyclophosphamide?
a)Hemorrhagiccystitis
b)Infertility
c)Bonemarrowsuppression
d)Diabetesinsipidus
CorrectAnswer-D
Ans.D.Diabetesinsipidus
[RefHarrison17e/ep.521;Goodman&Gilman11th/ep.1326;
Katzung11tVep.941]
ToxicityofCyclophosphamide
Bonemarrowsuppression(relativeplateletsparing)
Pulmonarytoxicity
Cardiac(athigherdoses)
Cystitis
Infertility
GIToxicity
Alopecia
Teratogenesis

608.Adverseeffectoftheimatinibareall
except?
a)Periorbitaledema
b)Myalgia
c)Pleuraleffusion
d)Arthralgia
CorrectAnswer-D
Ans.D.Arthralgia
[RefKDT7th/ep.870]
Imatinibisatyrosinekinaseinhibitor,whichinhibitsPDGF(Platelet
derivedgrowthfactor)receptoraswell.
Uses:
1. CML-Chronicmyeloidleukemia
2. DOCforc-kit+veGIST(GastrointestinalStromalTumor)
Adverseeffects:
Abdominalpain,vomiting,fluidretention,pleuraleffusion,periorbital
edema,myalgia,liverdamageandCHF.

609.Defectindiscriminatingbluegreen
visionisduetowhichdrug:?
a)Alprostadil
b)Primaquine
c)Sildenafil
d)Primaquine
CorrectAnswer-C
Ans.C.Sildenafil
[RefKDT7thlep.303-304]
Sildenafil(Viagra)actstoincreasecGMPbyinhibitingitsbreakdown
byphosphodiesteraseisoform5(PDE-5).
Recommendedthatatleast6hourspassbetweenuseofanitrate
andtheingestionofsildenafil.
Sildenafilalsohaseffectsoncolorvision,causingdifficultyinblue-
greendiscrimination.
TwosimilarPDE-5inhibitors,tadalafilandvardenafil,areavailable.

610.Racecadotrilisusedfor?
a)Chronicdiarrhea
b)Acutesecretorydiarrhea
c)Chronicconstipation
d)Diabeticgastroparesis
CorrectAnswer-B
Ans.B.Acutesecretorydiarrhea
[Ref:KDT7th/ep.686]
Racecadotrilisrapidlyconvertedtothiorphan,anenkephalinase
inhibitorwhichpreventsthedegradationofencephalin(ENK)which
aremainly8opioidreceptoragonists.
Itdecreasesintestinalhypersecretionwithoutaffectingmotilityand
usedforshorttermtreatmentofacutesecretorydiarrhea.

611.Trueabouttrienteneis:
a)Itisthedrugoffirstchoiceinwilsonsdisease
b)Itismorepotentcuriureticagentthanpenicillamine
c)Trientinetherapycancauseirondeficiency
d)Trientinecannotbegivenorally
CorrectAnswer-C
Ans.C.Trientinetherapycancauseirondeficiency
[RefHarrisons18thiep.3189]
PenicillamineVstrientineinwilsondisease
Penicillamineisthedrugofchoicefortreatmentofwilson'sdisease.
However,thedrugproducesundesirablesideeffectsandinsome
patientsbecomeintolerable.
Trientineisindicatedespeciallyinpatientswhoareintolerantto
penicillamine.
Trientineislesspotentcupriureticagentthanpenicillamine.
Trientineisorallyeffectiveandshortacting.
Trientinemaycauseirondeficiency;thiscanbeovercomewithshort
courseofirontherapybutironandtrientineshouldnotbeingested
within2hoursofeachother.

612.Whichofthefollowingdrugsis
associatedwithpriapism?
a)Hydralazine
b)Prazocin
c)Resperidone
d)Alltheabove
CorrectAnswer-D
Ans.D.Alltheabove
Drugsthatmaycausepriapism:
Anticoagulants
Haloperidol
hydralazine
Nifedipine
Olanzapine
Papaverine
Phenothiazines
Phentolamine
Prazosin
Risperidone
Trazodone

613.WhichofthefollowingisaCl-channel
activator?
a)Lubriprostone
b)Nefazodone
c)Varenioline
d)Valethamate
CorrectAnswer-A
Ans.A.Lubriprostone
[RefHarrisons18th/ep.2500]
ChlorideChannelActivators
Lubiprostoneisabicyclicfattyacidthatstimulateschloridechannels
intheapicalmembraneofintestinalepithelialcells.
Chloridesecretioninducespassivemovementofsodiumandwater
intothebowellumenandimprovesbowelfunction.
Lubiprostoneisanewclassofcompoundsfortreatmentofchronic
constipationwithorwithoutIBS.

614.Whichdrugisusedinthetreatmentof
TypeItyrosinemia?
a)Nitisinone
b)Alogliptin
c)Pemoline
d)Milrinone
CorrectAnswer-A
Ans.A.Nitisinone
[RefNelson20'Vep.641]
Adietlowinphenylalanineandtyrosinecanslowbutdoesnothalt
theprogressionofthecondition.
Thetreatmentofchoiceisnitisinone,whichinhibitstyrosine
degradationat4-HPPD.Thistreatmentpreventsacutehepaticand
neurologiccrises.
Althoughnitisinonestopsorgreatlyslowsdiseaseprogression,
somepretreatmentliverdamageisnotreversible.

615.Modafinilisadrugusedinwhichofthe
followingconditions?
a)Prematureejaculation
b)Premenstrualsyndrome
c)Shiftworkdisorder
d)Erectiledysfunction
CorrectAnswer-C
Ans.C.Shiftworkdisorder
[RefKDT7th/ep.487]
Modafinil
Itisanewerpsychostimulantpopularwithnightshift(callcentre)
workersandpeoplewhowanttoimprovealertnessandkeepawake.
Itisclaimedtoincreaseattentionspan,andimproveaccuracy
compromisedbyfatigueandsleepiness.
Theapprovedindicationsarenarcolepsy,sleepapneasyndrome
andshiftworkdisorder.

616.Acamprostateisusedfor?
a)Alcoholabstinence
b)Nicotineabstinence
c)Opioidabstinence
d)Cocaineabstinence
CorrectAnswer-A
Ans.A.Alcoholabstinence
[RefKDT7th/ep.393]
AcamprosateisaweakNMDAreceptorantagonistwithmodest
GABAAreceptoragonisticactivityusedformaintenanceofalcohol
abstinence.
Ithasalsobeenfoundtoreducerelapseofdrinkingbehavior.
Itisstartedimmediatelyafteralcoholwithdrawalandgivenatadose
of666mg2-3timesaday.
Loosemotionisthemostcommonsideeffect.

617.Ifawomanisassaultedbyherhusband
thenheischargedunder:
FMGE11

a)Sec.498-AIPC
b)Sec.304-AIPC
c)Sec.304-BIPC
d)Sec.504IPC
CorrectAnswer-A
Ans.Sec.498-AIPC

618.

Punishmentforcriminalabortionforthe
womenwhogivesconsentandtheperformer
arecoveredunderIPCsection?

a)312
b)313
c)314
d)315
CorrectAnswer-A
Ans.is'a'i.e.,312
Punishmentforperformingcriminalabortionwiththeconsentof
women;bothforthewomenandperformerisincludedinIPC
Section312.

619.

Adulteryisdescribedunderwhichsectionof
IPC?

a)314
b)375
c)497
d)504
CorrectAnswer-C
Ans.isc'i.e.,497[RefWomenandtheLawp.167]
Section497ofIPCsays,"Whoeverhassexualintercoursewitha
personwhoisandwhomheknowsorhasreasontobelievetobe
thewifeofanotherman,withouttheconsentorconnivanceofthat
man,suchsexualintercoursenotamountingtheoffenceofrape,is
guiltyoftheoffenceofadulteryandshallbepunished.

620.

Useofdangerousweaponisrelatedtowhich
sectionofIPC?

a)304
b)319
c)322
d)324
CorrectAnswer-D
Ans.is'D'i.e.,324
319IPC:Defineshurt.
320IPC:Definesgrievoushurt.
321IPC:Voluntarilycausinghurt.
322IPC:Voluntarilycausinggrievoushurt.
323IPC:Punishmentforvoluntarilycausinghurt(noprovocation,
nodangerousweapon)(1yearsimprisonment).
324IPC:Punishmentforvoluntarilycausinghurtbydangerous
weapon
(3yearsimprisonment?fine).
325IPC:Punishmentforvoluntarilycausinggrievoushurt(no
provocation,nodangerousweapon)(7yearsimprisonment?fine).

621.

Grievoushurtcomesunderwhichsectionof
IPC?

a)Section319
b)Section320
c)Section324
d)Section326
CorrectAnswer-B
Section320
REF:TextbookofForensicMedicineandToxicologybyNagesh
KumarRaoPage259
Section320:Grievoushurt
Thefollowingkindsofhurtonlyaredesignatedas"grievous"
1. Emasculation
2. Permanentprivationofthesightofeithereye
3. Permanentprivationofthehearingofeitherear
4. Privationofanymemberorjoint
5. Destructionorpermanentimpairingofthepowersofanymemberor
joint
6. Permanentdisfigurationoftheheadorface
7. Fractureordislocationofaboneortooth
8. Anyhurtwhichendangerslifeorwhichcausesthesufferertobe
duringthespaceoftwentydaysinseverebodilypain,orunableto
followhisordinarypursuits.

622.

Outragingmodestyofwomeniswhich
sectionofIPC?

a)375
b)354
c)195
d)304
CorrectAnswer-B
Ans.is'b'i.e.,354[RefReddy26th/ep.365]
Rapeandothersexualoffencesandassault
228IPC:Prohibitsdisclosureofidentityofrapevictim.
375IPC:Definingrape.
376IPC:Punishmentforrape(7yearstolifeimprisonment?fine).
376-AIPC:Punishmentformaritalrape(2yearsimprisonment?
fine).
377IPC:Unnaturalsexualoffences(10yearstolifeimprisonment?
fine).
354IPC:Assaultorcriminalforcetowomanwithintentto
outragehermodesty.
366-A
IPC:Procurationofminorgirlforillicitintercourse.
351IPC:Definesassault.
352-358IPC:Punishmentforcausingassault.
509IPC:Word,gestureoractintendedtoinsultthemodestyofa
women

623.

Section89IPCisfor?
a)Criminalresponsibilityofinsane
b)Criminalresponsibilityofachild
c)Criminalresponsibilityofdrunkenperson
d)Ageforconsent
CorrectAnswer-B
Ans.is'i.e.,Criminalresponsibilityofachild
89IPC:Achildunder12yearsofagecannotbegivenvalidconsent
tosufferanyharmwhichcanoccurfromanactdoneingoodfaith
forthebenefitofthechild,e.g.,consentforsurgery.Theonly
guardiancangivesuchconsent.
Anactisdoneingoodfaithforthebenefitofachildorinsane
person,byorbyconsentoftheguardian.
Nothingwhichisdoneingoodfaithforthebenefitofapersonunder
twelveyearsofage,orofunsoundmind,byorbyconsent,either
expressorimplied,oftheguardianorotherpersonhavinglawful
chargeofthatperson,isanoffencebecauseofanyharmwhichit
maycause,orbeintendedbythedoertocauseorbeknownbythe
doertobelikelytocausetothatperson

624.Apersonisdeclareddeadifhe/sheis
notfoundfor
a)7years
b)10years
c)14years
d)21years
CorrectAnswer-A
Ans.is'a'i.e.,7years[RefCivilLaw,Section108MA]
TheIndianEvidenceAct,undersection108provides7yearsfrom
thedatewhenceapersonisdeclaredtobemissingandhis

625.

Adulterationofdrugisunderwhichsection
ofIPC

a)271
b)272
c)273
d)274
CorrectAnswer-D
Ans.is'd'i.e.,274[RefSafetyManagementofHospitalsp.315;
PrinciplesofFMT,Bardalep.18]
LawrelatedtoAdulteration:-IndianPenalCode:?
Section272AdulterationofFoodorDrinkIntendedforSale
Imprisonmentfor6months,Fine-Rs.1000Section273Saleof
NoxiousFoodorDrink
Imprisonemntfor6months,fine-Rs.1000Section
274AdulterationofDrugs
Section275Saleofaduletrateddrugs
Section276Saleofdrugsasadifferentdrugorpreparation
Section277Foulingwaterofpublicspringorreservoir

626.Minimumageforgivingconsentfor
organdonationinIndiais?
a)16years
b)18years
c)21years
d)Noagelimit
CorrectAnswer-B
Alegalageof18yearshasbeensettoconsentforterminationofpregnancy(MTPAct
1971),donationofbloodanddonationoforgans(TransplantationofHumanOrganAct
1994).
AsperTransplantationofHumanOrgansAct1994"donor"meansanyperson,notless
thaneighteenyearsofage,whovoluntarilyauthorizestheremovalofanyofhisorgansfor
therapeuticpurposesundersubsection(1)orsub-section(2)ofsection3.
ParentsorGuardiansconsentwillberequiredforanyindividualbelow18yearswishingto
donateanorgan.

627.Firstcarpalbonetoappearis?
a)Trapezium
b)Capitate
c)Pisiform
d)Lunate
CorrectAnswer-B
Ans.is'B'i.e.,Capitate[RefReddy'sEssentials26th/ep.63]
Theossificationcentresincarpalbonesappearasfollows:

nCapitate
1year
nHamate
2years
nTriquitrum
3years nLunate
4years
nScaphoidandtrapezoid 5years nTrapezium 6years
nPisiform
11years

628.Fourcarpalbonesarepresentatwhat
age:
DNB09

a)3years
b)4years
c)5years
d)6years
CorrectAnswer-B
Ans.4years

629.Ossificationcentreofscaphoidappears
at
a)1-6months
b)1to2years
c)2to4years
d)4to6years
CorrectAnswer-D
Answer-D.4to6years

630.Holographwillisdefinedas
a)Doctordictatesandwrittenbyrelatives
b)Testatoriswritteninhisownhandwriting
c)Testatorisdictatedandwrittenbyrelatives
d)Testatorisdictatedandwrittenbydoctor
CorrectAnswer-B
Ans.is'b'i.e.,Testatoriswritteninhisownhandwriting[RefSK
Singhal4th/e
p.314]
Holographwillisonewhichiswrittenbythetestatorinhisown
handwritting.
Testamentarycapacityisdefinedasthecapacityofapersonto
makeavalidwill.

631.Subpubicangleis:
a)<65?
b)65-75?
c)85?
d)110-120?
CorrectAnswer-C
85?

632.Obtuseangleofmandibleisseenin
a)Infancy
b)Adulthood
c)AdultMale
d)AdultFemale
CorrectAnswer-A
Ans.is'a'i.e.,Infancy[RefParikh6thlep.2.30]
Feature
Infancy Adult
Oldage
Angleof
Obtuse Right
Obtuseangle
mandible
angle
angle
Mental
Near
Midway Looksapparentlynearertheofbody
foramen
lower
b/w
&lowerborderupperborderd/tlossof
opening
border
upper
teeth

633.Sizeofthefetusis20mm,thenbythe
ruleofHasse's,whatisthegestational
ageofthefetus?

a)2weeks
b)4weeks
c)6weeks
d)8weeks
CorrectAnswer-C
Ans.is'c'i.e.,6weeks
Hess'srule(Haase'srule):Itisaroughmethodforcalculatingthe
ageoffetusbymeasuringthelengthfromcrowntoheel.Upto
5thmonthofgestation,lengthofthefoetusincmissquareofthe
monthofgestationandbeyond5months,lengthincmis5timesthe
monthofgestation.
Lengthincm=(monthofpregnancy)upto5months
Lengthincm=monthofpregnancyafter5months
Inthisquestion:
Lengthincm=(monthofpregnancy)2
2=(monthofpregnancy)(Note:20mm=2cm)
Monthofpregnancy=14monthsor6weeks.

634.Firstincisortoeruptis
a)Lowercentral
b)Uppercentral
c)Lowerlateral
d)Upperlateral
CorrectAnswer-A
Ans.is'a'i.e.,Lowercentral[RefReddy30thiep.60]
Firsttemporarytoothtoupper(inprimarydentition)islowercentral
incisorsandlasttemporarytoothtoeruptis2ndmolar.The
sequenceoferuptionislowercentralincisor>uppercentralincisor
>upperlateralincisor>lowerlateralincisor>I"molar>Canine>
molar.Thereforeeurptionoftemporaryteethiscompletedby
eruptionof2"?molarat25months(2years).
Firstpermanenttoothtoerupt(insecondarydentition)is1"molar
andlasttoeruptis3rdmolar.Thesequenceoferuptionis1"molar>
centralincisor>lateralincisor>1"premolar>2"premolar>canine
>2"molar>3rdmolar.Eruptionofpermanentteethiscompletedby
eruptionof3rdmolarbetween17-25years.

635.Ababyhasstatedtogethisfirstmilkteeth.Hisageisapproximately:
a)3months
b)6months
c)9months
d)12months
CorrectAnswer-B
Mostbabieshavetheirfirstprimary(milk)teetheruptatage6monthofageandfirst
secondaryteetheruptatage6years.

Theteethintheupperjaweruptearlierthanthoseinthelowerjaw,exceptforlowercentral
incisors.Thelowercentralincisorsappear,commonly,betweentheagesof5and8
months.Theuppercentralincisorsappearamonthlaterandthelateralincisorsusually
withinthenextthreemonths.Thefirstmolarteethappeararoundtheageof12-15months,
precedingtheeruptionofcanineteethby6months,whichappearbetweentheageof18
and21months.Thesecondmolarsareoutattheageof21to24months.
Note:Permanentteetheruptionisinthefollowingorder:1stmolar-6years;centraland
lateralincisors-6-8years;caninesandpremolars-9-12years;secondmolars-12years;
thirdmolars-18yearsorlater.
Ref:GhaiEssentialPediatricsbyOPGhai,6thedition,Page6;NelsonTextbookof
Paediatrics17theditionpage18&37

636.X-rayofchoiceforagedetectionat
around16-17yearsofage
a)Elbow
b)Wrist
c)Shoulder
d)Pelvis
CorrectAnswer-B
Ans.is'b'i.e.,Wrist[RefParikh6th/ep.2.10]
Site
Age(inyears)
Female Male
Elbow
13-14
15-16
Wrist
16-17
18-19
Shoulder
17-18
19-20
Crestofilium 18-19
20-21

637.Rigormortisoccursdueto:
September2008
a)Muscleofthebodybegantorelax
b)Capillo-venousdistensioninthemostdependentbodyparts
c)Muscleofthebodybegantostiffen
d)Mummificationofthebodytissues
CorrectAnswer-C
Ans.C:Muscleofthebodybegantostiffen
Afewhoursafterthedeath,thejointsofthebodystiffenand
becomelockedinplace.Thisstiffeningiscalledrigormortis.
Dependingontemperatureandotherconditions,rigormortislasts
approximately24-48hoursinwinterand18-36hoursinsummer.
Thephenomenoniscausedbytheskeletalmusclespartially
contracting.Themusclesareunabletorelax,sothejointsbecome
fixedinplaceRigormortiscanbeusedtohelpestimatetimeof
death.
Theonsetofrigormortismayrangefrom1-2hours,dependingon
factorsincludingtemperature(rapidcoolingofabodycaninhibit
rigormortis,butitoccursuponthawing).Maximumstiffnessis
reachedaround12-24hourspostmortem.Itfirststartsininvoluntary
muscles(myocardium)
Eyelids,neckandjawmusclesareaffectedfirst,withtherigorthen
spreadingtootherpartsofthebody.
Thejointsarestiffbutaftersometimegeneraltissuedecayand
leakingoflysosomalintracellulardigestiveenzymeswillcausethe
musclestorelax.

638.Postmortemlividityisnotseenin?
a)Drowninginwell
b)Drowninginafastflowingriver
c)Postmortemsubmersion
d)Drowninginchlorinatedswimmingpool
CorrectAnswer-B
Ans.is'b'i.e.,Drowninginafastflowingriver[RefParikh6th/e
p.3.10]


639.Postmortemstaininggetsfixedafter:
a)2-3hrs.
b)3-4hrs.
c)5-6hrs..
d)7-8hours
CorrectAnswer-D
Di.e.7-8hours

640.Suggilationisduetowhichofthe
following?
a)Gravityleadingtopoolingofblood
b)Escapeofbloodfrommicrovasculature
c)Moleculardeathleadingtostiffeningofmuscles
d)Autolysisofenzymes
CorrectAnswer-A
Ans.is'a'i.e.,Gravityleadingtopoolingofblood[RefReddy
30th/ep.141]
GravityleadingtopoolingofbloodSuggilation
MoleculardeathleadingtostiffeningofmusclesRigorMortis
AutolysisofenzymesPutrefaction

641.Earliestsignoffetaldeath-
a)Spaldingsign
b)Robert'ssign
c)Ballsign
d)Adipocereformation
CorrectAnswer-B
Ans.is'b'i.e.,Robert'ssign[RefParikh6thlep.5.75,5.76&
4th/ep.268]
Robertsign(gasingreatvessels)12h
Spaldingsign(overlappingofskullbones)1week
Blair-Hartley/Ballsign(hyperflexion/hyperextension3-4weeksof
spinewithovercrowdingofribs

642.Firstsignofmacerationis
a)Robert'ssign
b)Skinslippage
c)Spaldingsign
d)Greenishdiscolorationofbody
CorrectAnswer-B
Ans.is'b'i.e.,Skinslippage[RefParikh6"Vep.5.75;SKParikh
4th/ep.268]
Macerationisaprocessofasepticautolysis.
Itoccurswhenadeadfetusremainsintheuterusfor3-4days
surroundedbyliquoramniibutwithexclusionofair.
Skinslippageistheearliestsign(occurswithin12hours).

643.Whichisthebesttemperaturefor
putrefaction?
a)-10degreesC
b)0degreesC
c)30degreesC
d)50degreesC
CorrectAnswer-C
Ans.is'c'i.e.,30degreesC[RefReddy30th/ep.150]
Putrefactionisthelaststageinresolutionofbodyfromtheinorganic
toorganicstateandisacertainsignofdeath.
Theputrefactionisdueto:
Autolysis:
Theenzymesofthebody,breakdownthedeadbodycausing
proteolysis,lipolysisandglycolysis.
Bacterialenzymes:
Aerobicandanaerobicbacteriapresentinsmallintestine(e.g.C.
welchii,staphylococcus,E.colietc.)releaseenzymes(especilly
lipaseandlecithinase),whichactonbodytocausebreakdown.
Threeconditionsarenecessoryforputrefaction:(i)warmth(10-45?),
(ii)moisture(humidity)and(iii)air.Ifairisabsent,adipocere
formation(saponification)occurs,andintheabsenceofmoisture,
mummificationoccurs.

644.AccordingtoCasper'sdictum,
decompositionisthefastestwhenthe
bodyisinwhichofthefollowing?

a)Air
b)Water
c)Earth
d)Soil
CorrectAnswer-A
Ans.is'a'i.e.,Air[RefReddy30thiep.155]
Itistheeffectofmedium(inwhichbodylies)onrateofputrefaction.
Casper'sdictumstatesthatabodydecomposesinairtwiceas
rapidlyasinwaterandeighttimesasrapidlyinearth.

645.Enzymeresponsibleforpostmortem
hemolysisis
a)Hemolysin
b)Lecithinase
c)Lipoproteinase
d)Protease
CorrectAnswer-B
Ans.is'b'i.e.,Lecithinase[RefTextbookofFMTbyVij5th/ep.
89]
Bacteriaproducealargevarietyofenzymesthatacton
carbohydrates,proteinsandfatsandbreakdownthevarious
tissues.
Oneofthemostimportantenzymesisthelecithinase'producedby
theClostridiumwelchii,whichhydrolysesthelecithin
presentinallthecellmembranesincludingbloodcellsandthusis
responsibleforproducinghemolysisofbloodpostmortem.
Thisenzymealsohelpsinpostmortemhydrolysisandhydrogenation
ofbodyfat.

646.Mummificationisenhancedby?
a)Moistandhotair
b)Moistandcoolair
c)Dryandhotair
d)Dryandcoolair
CorrectAnswer-C
Ans.is'c'i.e.,Dryandhotair
Mummification
Itisamodificationofputrefaction,whichoccursintheabsenceof
moisture.
Thatiswhenthereisexcessairandwarmthbutno
moisture(humidity),
i.e.hotdryandwindyclimate,mummification
takesplaceinplaceofnormalputrefaction.Thusmummification
occursindeserts,especiallyinsummerandalsoinbodiesburiedin
shallowgraveinsandysoil.
Mummificationischaracterizedbydessicationordryingofthedead
body.Thereisdrying,dehydrationandshrivelingofdeadbody.It
proceedsfromexteriortointerior.Thereforefirsttobeinvolvedis
skin,
especiallyofexposedbodypartslikelips,nosetip,hands
(fingers)andfeet(toes).
Theskinisshrunken,contracted,dry,
brittle,leathery,strechedacrossbonyprominencesandrustybrown
toblackincolor.Internalvisceraalsodryup,darkenincolorand
blendwitheachothertoformasinglemass.Bodyemitssmelllike
rottencheese.Facialfeaturesandinjuriesarewellpreserved,thus
identificationofbodyandcauseofdeathcanbedetermined(like
adipocereformation).
Timerequiredformummificationvariesbetween3months-2years.
Ifproperlypreserved,amummifiedbodycanremainforyears.
Chronicarsenicorantimonypoisoningfavormummification.

Medicolegalimportance:(i)Identificationofbody(facialfeaturesare
preserved),(ii)causeofdeath(injurymarksarepreserved),(iii)time
sincedeathcanbeestimated.

647.Postmortemwoundbestdiffersfromthe
antemortemwoundby:
a)Gapsonincising
b)Noclots
c)Absenceoferythemaandcellularchanges
d)All
CorrectAnswer-C
Ci.e.Absenceoferythema&cellularchange

648.Pugilisticattitudeischaracteristically
seenin?
a)Burns
b)Drowning
c)Electrocution
d)Hanging
CorrectAnswer-A
Ans.is'a'i.e.,Burns

649.Treatmentofchoiceforstabinjury
caecum?
a)Caecostomy
b)Ileo-transverseanastomosis
c)Transversecolostomy
d)Sigmoidcolostomy
CorrectAnswer-B
Ans.is'b'i.e.,ileo-transverseanastomosis
Stabandlow-velocityinjuriestothecolonwithminimal
contaminationandhemodynamicalstabilitycanbemanagedby
primaryrepairmechanismslikeileotransverseanastomosis.

650.Gunpowderonbodyandclothingcanbe
visualizedby?
a)HarrisonandGilroytest
b)Neutronactivationanalysis
c)Dermalnitratetest
d)Alloftheabove
CorrectAnswer-D
Ans.is'd'i.e.,Alloftheabove[RefParikhelep.4.39]
Testsusedinfirearmdischarge:
Basedondetectionofnitrate:Dermalnitrite(Paraffin)test.
Basedondetectionofheavymetal:HarrisonandGilroytest,
neutronactivationanalysis,Atomicobsorptionspectroscopy,and
flamelessatomicabsorptionspectroscopy.

651.Bulletfixedfromagunisnotreleased.It
isejectedoutwithsubsequentshot.
Suchabulletiscalledas

a)Tandembullet
b)Dum-dumbullet
c)Inceridiarybullet
d)Tumblingbullet
CorrectAnswer-A
Ans.is'a'i.e.,Tandembullet[RefSKSinghal4thlep.188]
Varietiesofbulletsare:-
i)Incendiary(igniting)bullet:
Thetipofbulletcontainsselfignitingmateriale.g.bariumnitrateand
powderedaluminiumandmagnesium(inthepast,phosphoruswas
used),sothatitcatchesfireonhittingthetarget.Itisusedtocause
fireinusuallyinflammabletargetslikefueltanks(ofaircraftsetc).
ii)Explosivebullet:
Thetipcontainsadetonatororleadazide,sothatthebullet
explodesonhittingthetarget.
iii)Dum-dumbullet(expandingbullet):
ItisajacketedbulletwithisnosetipchiseledorcutoffItisdesigned
toincreaseindiameterandexpanduponstrikingthetarget,thus
producinglargerdiameterwoundsoflimitedpenetration.
iv)Tandembullet(Piggeytailbullet):
Itiscalledone-behind-otherbulletbecausetwobulletsareejected
oneaftertheother,whenfirstbulletfailedtoleavetheburrelandis
ejectedbysubsequentlyfiredbullet.
Therefore,bothenterbodythroughsameenterancewound(some

times,theymayenterthroughdifferententries),butthewoundsof
exitarealwaystwo.

652.Whichofthefollowingcausesmaximum
damage?
a)Tandem
b)Dumdum
c)Souvenir
d)Piggy
CorrectAnswer-B
Ans.is'b'i.e.,Dumdum[RefSKSinghale/ep.188]
Expandingbullets,alsoknownashollow-pointbulletsor"dum-
dums",aredesignedto"mushroom"uponenteringatargetinorder
tostopitfromleavingthebody.Comparedtoregularbullets,orfull
metaljackammunition,expandingbulletsareintendedtocause
maximumtissuedamage.

653.Abulletpackedwithjacketandopensat
abaseiscalled
a)Dumdumbullet
b)Tandembullet
c)Duplexbullet
d)Souvenirbullet
CorrectAnswer-A
Ans.is'a'i.e.,Dumdumbullet[RefParikhp.4.33]

654.Ricochetbulletis
a)Bulletproducingakeyholeentrywound
b)Bulletwithnosetipchiseledoff
c)Deviationinthedirectionorpathofbullet
d)Bulletcontainingignitingmaterial
CorrectAnswer-C
Ans.is'c'i.e.,Deviationinthedirectionorpathofbullet[Ref
Reddy30th/ep.211]
Ricochettingofbulletisdefinedasdeviationinthedirectionorpath
ofbulletbecauseofstrikingofbullettoanintermediaryobject.

655.Compoundusedasprimerinweaponis
a)Potassiumchlorate
b)Sulphur
c)Potassiumnitrate
d)Nitrocellulose
CorrectAnswer-A
Ans.is'a'i.e.,Potassiumchlorate[RefParikh6th/ep.
4.29]Cartridge(round)
Cartridgeistheammunitionusedinfirearms.Cartridgeconsistsof
(i)Cartridgecase,(ii)Propellant(gunpowder),(iii)Projectile
(missile)and(iv)Wads(onlyinshotguns.).
1)Cartridgecase
Itistheoutershellorcoveringofthecartridge.Inshotgunitis
cylinderical.Baseisofbrassandrestofitisofcardboardorplastic.
Inrifledweapons,itistaperingandwholeofitismadeofbrassor
steel.
Centerofbasehaspercussioncap(detonatorcap)whichhas
sensitivecomposition,i.e.primer,consistingofpotassiumchlorate,
antimonysulphide,mercuryfulminate,bariumnitrite,leadperoxide
ortetrazene.
Inallweapons,afterfiring,thecartridgecaseremainsinthebarrel
anditistobetakenoutmanually(shotgun,revolvers,rifle)oritis
ejectedoutautomatically,e.g.inpistals.
2)Propellent(gunpowder)
Itpropelstheprojectile(missile)forward.Itiscomposedofblack
powderorsmokelesspowderintheformofgrains,pellets,thin
cylindricalcordsorflakes.Itsignitionresultsinformationof
expandinghotgasesunderpressurewhich(gaspressure)propels

theprojectile(missile)byprovidingitnecessarymuzzlevelocityand
strikingenergy.
Theclassicalgunpowderisknownasblackpowder,consistsof
charcol(15%),Sulphur(10%),andpotassiumnitrate(75%).
Dependingonfineness,theblackgunpowderisdesignatedasFG,
FFG,FFFG.etc.(F=fineness).Pyrodexisanothergunpowderwith
samecomponents,butwithdifferentratios.
Blackgunpowderproducessmoke,i.e.Itissmokeproducing
powder.Smokelesspowder,inadditiontoblackpowder,has
nitrocellulose(singlebase),ornitrocelluloseplusnitroglycerine
(doublebase),ornitrocelluloseplusnitroglycerineplus
nitroguanidine(triplebase).Semismokelesspowderhas80%black
powderand20%smokeless

656.Destructivepowerofbulletisdetermined
byallexcept
a)Velocityofbullet
b)Weightofbullet
c)Shapeofbullet
d)Kineticenergy
CorrectAnswer-C
Ans.is'c'i.e.,Shapeofbullet[RefReddy29thlep.194]
Tissuedamage(ordestructivepowerofbullet)isdependenton
striking(kinetic)energyofbullet,whichisproportionaltoitsmass
andsquareofvelocity(KE='/2mv2).Somodernbulletsare
designedtohavesmallermassallowingtransportationoflarge
amountofammunitionandhighvelocity,asvelocityismore
importantdeterminantindestructivepowerofbullet.

657.HeatRuptureischaracterizedby:
a)Regularmargins
b)Irregularmargins
c)Rupturedbloodvessels
d)RupturedRedBloodcells/clottedblood
CorrectAnswer-B
Bi.e.Irregularmargins

658.Sexualstimulationobtainedthrough
someinanimateobjectisknownas:
September2003

a)Transvestism
b)Fetichism
c)Voyeurism
d)Zoophilia
CorrectAnswer-B
Ans.Bi.e.Fetichism

659.Transvestismis:
a)Wearingclothesofoppositesex
b)Touchingonesownprivatepartstoothers
c)Desireforsexualintercoursewithdeadbodies
d)Orgasmfromvisualisationpartofthebodyofawoman
CorrectAnswer-A
Ai.e.Wearingclothesofoppositesex

660.Whichofthefollowingismeantbythe
term`SinofGomorrah'?
a)Oralsex
b)Analsex
c)Bestiality
d)Lesbianism
CorrectAnswer-A
OralsexisalsocalledcoitusperosorSinofGomorrahorbuccal
coitus.
AccordingtoBiblethesinwascommoninatowncalled:Gomorrah,
sothename.
ItisincludedinUnnaturalsexualoffencesAct(S.377IPC).
Ref:TheEssentialsofForensicMedicineandToxicologybyDr.K.
S.NarayanReddy,27thedition,Page365.

661.Voyeurismis?
a)Sexualgratificationbywatchingtheactofsexualintercourse
b)Useofsuchobjectsforsexualgratification
c)Sexualgratificationisbyselfpain
d)Sexualgratificationbyrubbingprivatepart
CorrectAnswer-A
Ans.is'a'i.e.,Sexualgratificationbywatchingtheactofsexual
intercourse[RefReddy30th/ep.395]
Voyeurism(Scoptophilia,Peepingtom):
Sexualgratificationis
obtainedbylookingatthesexualorgansofotherpersons,watching
theactofsexualintercourse,orwitnessingundressingbyawoman.

662.Whichofthefollowingisthetestdoneto
confirmlivebornfetusafterfetaldeath?
a)Mirrortest
b)Breslau'stest
c)Magnustest
d)Paraffintest
CorrectAnswer-B
Ans.is'b'i.e.,Breslau'stest[RefSKSinghal4thlep.271]
Testsusedininfanticide
Ploucquet'stest:
Weightoflungismeasuredinrelationtobodywieght.Beforebirth
weightoflungis1/70ofbodyweightandafterrespirationitbecomes
1/35ofbodyweightduetoincreasedbloodflowinlungbeds.
StatictestorFodere'stest:
Theaverageweightofbothlungsbeforerespirationis30-40gmand
afterrespirationis60-70cm.
Hydrostatictest(Raygat'stest,1"lifetest):
Thegravityofanon-respiredlungis1040-1050andofarespired
lungis940-950,so,afterrespirationlungfloatsonwater(specific
gravityofwateris1000).
Breslau'ssecondlifetest:
Itassumesthatalivebornchildwouldrespireandtherefore,would
alsoswallowsomeairintothestomachandbowel.
Hencetheyfloatonwater.Thistestisfalselypositiveinputrefaction
(duetoputrefiedgases)orincasesofattemptedartificialrespiration.
Werdin'stest:
Beforebirthmiddleearcontainsgelatinousembryonictissuewhich
isreplacedbyairafterrespiration.


663.Bansdolaisaformofstrangulation
by:
NEET14

a)Ligature
b)Hands
c)Woodensticks
d)Bendofelbow
CorrectAnswer-C
Ans.C.Woodensticks
Dependinguponthemethodusedtoconstricttheneck,
strangulationcanbedividedinto:
Ligaturestrangulation:
*Neckiscompressedbyaligatureofwhichusuallymultiplerounds
aregivenandnoknotistied.
Throttling(manualstrangulation):
*Neckiscompressedbyoneorbothhands.
*Whenneckiscompressedbytwopalms,itisknownaspalmar
strangulation.
Bansdola:
*Compressionofneckwithoneortwowoodensticksorbamboo.
Garrotting:
*Itiscompressionofneckbyaropethrownfrombehind.Spanish
windlassisatypeofgarrotting,whichusedtobethe
officialmodeofexecutioninspain.Inthis,anironcoloraroundthe
neckwastightenedbyascrewforstrangulation.
Mugging(chokehold):
*Itiscompressionofneckbyforearmorinthebendofelbow.

Strangulationbyknee/foot:
*Inthis,neckiscompressedbykneeorfoot.

664.Followingismostsuggestiveof
antemortemhanging:
DNB09

a)Salivarydribbling
b)Congestionoflungs
c)Ligaturemarks
d)Petechialhemorrhages
CorrectAnswer-A
Ans.Salivarydribbling

665.Acouplewaslockedupinaroomwith
firearound.Allofthefollowingarethe
probableimmediatecausesofdeathin
suchcircumstanceexcept:-

a)Cyanideintoxication
b)Suffocation
c)Sepsis
d)Fatembolism
CorrectAnswer-C
Ans.is'c'i.e.,Sepsis[RefReddy29'/ep.287;Parikh6thlep.
4.153]
Causesofdeathinburns
Causesofdeathinburnsmaybedividedinto:
A)Causesofimmediatedeath:
Theseare(1)neurogenicshock(primaryshock),(2)hypovolaemic
shock(secondaryshock),(3)suffocation,duetoinhaledCO,CO2,
(4)cyanideintoxication,(5)fatembolism,(6)cerebralorpulmonary
edema,and(7)accidentalinjuriesduringburn.
B)Causesofdelayeddeath:
Theseare:(1)renalfailure(acutetubularnecrosis),(2)infections
(sepsis,gangrene,tetanus)and(3)centrilobularnecrosisofliver.

666.Puremotorpalsyseeninpoisoningof
a)Leadpoisoning
b)Arsenicpoisoning
c)Cocainepoisoning
d)Cannabispoisoning
CorrectAnswer-A
Ans.is'A'i.e.,Leadpoisoning
Theperipheralneuropathyofleadtoxicityisapuremotor
neuropathyaffectingtheupperlimbsmorethanthelowerlimbs,
presentingassymmetricorasymmetricwristdrop.Theweakness
mayalsoinvolveothermusclegroupsofthedistalupperextremities,
theinvolvementoflowerextremities,includingisolatedfootdrop,
alsomayoccur.

667.Trueaboutstrychninepoisoningis:
a)Allmusclesaffectedatthesametime
b)Shouldergirdleaffectedfirst
c)Pelvicgirdleaffectedfirst
d)Noneoftheabove
CorrectAnswer-A
Ai.e.Allmusclesaffectedatsametime

668.Wrongaboutdhaturaseedsis?
a)Kidneyshaped
b)Odourless
c)Yellowbrown
d)Convexsmoothsurface
CorrectAnswer-D
Ans.is'd'i.e.,Convexsmoothsurface[RefPilley4th/ep.207;
GautamBiswas2"d/e
p.496]
FeaturesofDhaturaseesare:-

1. Large&thick
2. Odourlers
3. Kidney-shaped
4. Laterallycompressedanddoubleedgedatconvexborder(not
smooth)
5. Yellowiskbrown
6. Bitter

669.Trueaboutacidpoisoningis
a)Greatestdamageisalongthelessercurvature
b)Corrosivescausevaporizationoftissues
c)Vitriolagemeansingestionofacidinemptystomach
d)Highestchanceofperforationiswithnitricacid
CorrectAnswer-A
Ans.is'a'i.e.,Greatestdamageisalongthelessercurvature
Magenstrasseisthetermappliedtothepathwayacidicagents
followinstomach.Thepathwayofacidsandalkalisinfoodfilled
stomachstartsalongthelessercurvatureofthestomachandleads
tothepylorus,whichexplainsthelocationofgreatestdamagein
foodfilledstomach.Stomachwithoutfoodhavesignificantinjuryin
thelowerhalfoftwothirdsandmayhavesparingoffundus.
Mechanismofaction:-Corrosivesfix,destroyanderodethesurface
withwhichtheycomeincontact.Theymainlyactlocallyby:
Hygroscopicextractionofwaterfromtissues
Coagulationofproteinsand
Conversionofhaemoglobinintohematin.
Titriolage'isthrowingofanycorrosiveonanotherperson.Eyesare
affectedmostcommonly.Itcomesundersector320IPS.
Therearehighestchancesofperforationofstomachwith11,SO4,
amongallmineralacids.

670.

Highestcontentofcannabisisfoundinwhich
partoftheplant?

a)Root
b)Resin
c)Seed
d)Stem
CorrectAnswer-B
Ans.is`b'i.e.,Resin[RefReddy26th/ep.528]
Cannabis,alsoknownasmarijuana,isobtainedfromtheIndian
Hempplantorcannabisindica(Cannabissativa).
TheactiveprinciplesofCannabisarecontainedinitsresin.
Allpartsoftheplant,maleorfemale,containtheactivematerial,
exceptstem,rootandseeds.
Theprincipalconstituentoftheresinarecannabinol,whichhasno
action.
Cannabidiolisalsoinert,butonexposuretoheat,itispartly
convertedtotheveryactiveisomerictetrahydrocannabinols(THC).

671.AllofthefollowingareCNSstimulants
except?
a)Amphetamines
b)Benzodiazepines
c)Cocaine
d)Methylphenidate
CorrectAnswer-B
Ans.is'b'i.e.,Benzodiazepines
CNSstimulantsareamphetamine,methylphenidate,atomoxetine,
modafinil,cocaine,pemolineandcaffeine.


672.Nuxvomicaseedscontain2alkaloids,
strychnineand:
a)Hyoscine
b)Hyoscyamine
c)Brucine
d)Atropine
CorrectAnswer-C
Ci.e.Brucine

673.Aseasnakeis:
a)Neurotoxic
b)Hemotoxic
c)Musculotoxic
d)Alloftheabove
CorrectAnswer-C
Ci.e.Musculotoxic

674.'ColdTurkey'termisusedtodenote
whichofthefollowing?
a)Consumptionofheroin
b)Abruptcessationofheroin
c)Gradualwithdrawalofheroin
d)Placetowithdrawadruginagroup
CorrectAnswer-B
Ans.is'b'i.e.,Abruptcessationofheroin[RefQuittingSmoking
forLifep.18]
'ColdTurkey'meansabruptandcompletecessationoftakingadrug
towhichoneisaddiced.
Alcoholisnottheonlydrugwherebyquittingcoldturkeyposesa
dangertothepatient.Peoplewhoareaddictedtoopiates(heroin,
methadone,morphine,Suboxone,andevenprescription
benzodiazepines)areatimmenseriskiftheytrytostoptheirdrug
intakewithoutaplan.

675.Allaretrueofopioidwithdrawal,except:
NIMHANS10;PGI14
a)Yawning
b)Hallucinations
c)Lacrimation
d)Piloerection
CorrectAnswer-B
Ans.Hallucinations

676.Smokystoolisseeninwhichpoisoning?
a)Phosphorus
b)Arsenic
c)Lead
d)Zinc
CorrectAnswer-A
Ans.is'a'i.e.,Phosphorus[RefModernMedicalToxicologyby
Pillayp.68]
Acutepoisoningofphosphorus:?
Breathsmellsofgarlic.
Vomitusandstoolsmaybeluminousinthedark.
Smokystoolsyndrome:-Faintfumesmayemanatefromthestools.
Manifestationsofliverdamage-tenderhepatomegaly,jaundice
whichmayprogresstoanolivegreenhue,flappingtremorofhands
(asterixis),mousyodourtothebreath(foetorhepaticus).

677.Gigantinisactiveprincipleof
a)PlumbagoRosea
b)Calotropis
c)RicinusCommunis
d)P.Zeylanica
CorrectAnswer-B
Ans.is'b'i.e.,Calotropis[RefConciseTextbookofFMT-
Sharmap.250]
Calotropisgigantea-4containsactiveprinciplenamedgigantin.
Whencut,stemandleavesoftheplantexrudeanacridmilkyjuice.
Whenthisjuiceisallowedtostandorheated,serumisexuded
leavingbehindwhiteclot.
Theserumcontainstheactiveprinciplenamedgigantinwhichis
verytoxic.

678.Whichofthefollowingistrueabout
cocainemetabolitetest
a)Measurebenzoylecgoninelevels
b)Usedtodiagnosecocaineuse
c)Treatmentofcocaineoverdose
d)Alloftheabove
CorrectAnswer-D
Ans.is'd'i.e.,Alloftheabove[RefCodeofFederalRegulations
p.174]
Cocaineandcocainemetabolitetestsystem:
Acocaineandcocainemetabolitetestsystemisadeviceintended
tomeasurecocaineandacocainemetabolite(benzoylecgoonine)in
serum,plasma,andurine.
Measurementsobtainedbythisdeviceareusedinthediagnosisand
treatmentofcocaineuseoroverdose.

679.Activeingredientofmarkingnutis?
a)Semecarpol
b)Crotin
c)Abrin
d)None
CorrectAnswer-A
Ans.is'a'i.e.,Semecarpol
Semicarpusanacardiumormarkingnutorbhilawan
Itsseedareheartshaped,conicalandblackwithacridoilyjuice
whichisbrownishbutturnsblackonexposuretoair.
Activeprinciplesaresemecarpolandbhilawanol.
Juiceappliedtoskinproducesirritation,painfulblistersfollowedby
itchingandeczema.Thereforeitisusedtoproduceartificialbruises.
Itisalsousedbywashermantomarkclothes.

680.Whoisthefatherofmodern
Microbiology?
a)Metchnikoff
b)LordLister
c)LouisPasteur
d)RobertKoch
CorrectAnswer-C
FatherofModernMicrobiology:LouisPasteur.
FatherofMedicalMicrobiology:RobertKoch.
FatherofPathology:RudolphVirchow.

681.WhichofthefollowingstatementregardingcelldivisionisNOTTRUE?
a)Produceshaploidnumberofchromosomes
b)Producessamenumberofchromosomes
c)Produces2cells
d)Noneoftheabove
CorrectAnswer-A
Mitosisisanucleardivisioninwhichdaughtercellsreceivethesamenumberof
chromosomesasthatofparentcell.Thedaughtercellsresultingfrommitosisareidentical
toeachotherandalsototheparentcellinthequantityandqualityofgeneticmaterial.
Mitosis:
Itistheprocessthatfacilitatesequalpartitioningofreplicatedchromosomesintotwo
identicalgroups.
Asaresultofthistwonewdaughtercellsarisefromoneoriginalcell.
Allthecellscreatedthroughmitosisaregeneticallyidenticaltooneanotherandtothecell
fromwheretheycame.
Themainpurposeofmitosisineukaryoticcellsaregrowthoftheindividual,repairoftissue
andasexualreproduction.

682.Allofthefollowingareimportant
mechanismsofgenetransferinbacteria,
except?

a)Lateralgenetransfer
b)Conjugation
c)Verticalgenetransfer
d)Horizontalgenetransfer
CorrectAnswer-C
Ans.is'c'i.e.,Verticalgenetransfer
Genetransfer
Genetransferreferstotheprocessofgeneticmaterial(e.g.DNA)
beingsentandreceivedamongtwoorganisms-*Donorsendsand
recipientreceivesthegeneticmaterial.
Therearetwoprocessesofgenetransfer:?
1.Horizontalgenetransfer(HGT)orlateralgenetransfer
Horizontalgenetransferistheprocessbywhichgeneticmaterialis
passedbetweentwodifferentorganism,i.e.organismofdifferent
species.
Therecipientisnottheoffspringofdonor.
Themostimportantexampleisgenetransferbetweenthebacteria.
Theprocessesofhorizontalgenetransferinbacteriaare:-
1. Transduction
2. Transformation
3. Conjugation
2.Verticalgenetransfer
Verticalgenetransferistheprocessoftransferringgeneticmaterial
toorganismofsamespecies,i.e.donorrecievesgeneticmaterial

fromitsancestor,e.g.itsparentoraspeciesfromwhichitevolved.
Therefore,thedonorwillhavethesamegeneralmakeupasthe
parents.
Verticalgenetransferis"amixoftwoparents",i.e.whentwo
organismsmate,theirgenesareverticallytransferedtotheirspawn.
Thisprocessisnotimportantinbacteria.

683.Naturalmethodofhorizontalgene
transferamongbacteriaincludes-
a)Electroporation
b)Transduction
c)Transformation
d)bandc
CorrectAnswer-D
Ans.is'b'i.e.,Transduction;'c'i.e.,Transformation
Genetransfer
Genetransferreferstotheprocessofgeneticmaterial(e.g.DNA)
beingsentandreceivedamongtwoorganisms--->Donorsends
andrecipientreceivesthegeneticmaterial.
Therearetwoprocessesofgenetransfer:?
1)Horizontalgenetransfer(HGT)orlateralgenetransfer
Horizontalgenetransferistheprocessbywhichgeneticmaterialis
passedbetweentwodifferentorganism,i.e.organismofdifferent
species.
Therecipientisnottheoffspringofdonor.
Themostimportantexampleisgenetransferbetweenthebacteria.
Theprocessesofhorizontalgenetransferinbacteriaare:-
1. Transduction
2. Transformation
3. Conjugation
2)Verticalgenetransfer
Verticalgenetransferistheprocessoftransferringgeneticmaterial
toorganismofsamespecies,i.e.donorrecievesgeneticmaterial
fromitsancestor,e.g.itsparentoraspeciesfromwhichitevolved.
Therefore,thedonorwillhavethesamegeneralmakeupasthe

parents.
Verticalgenetransferis"amixoftwoparents",i.e.whentwo
organismsmate,theirgenesareverticallytransferedtotheirspawn.
Thisprocessisnotimportantinbacteria.

684.Allculturemediaareusedforantibiotic
susceptibilityexcept-
a)Tetrathionate-F
b)Bloodagar
c)Chocolateagar
d)Muller-Hintonagar
CorrectAnswer-A
Ans.is'a'i.e.,Tetrathionate-F
*TheKirby-Bauerdiskdiffusionmethodisoneofthemostwidely
practicedantimicrobialsusceptibilitytests(AST).
*Itisaffectedbymanyfactorsamongwhicharethemediaused.
*Mueller-Hintonagar(MHA)isthestandardmediumrecommended
inguidelines.
*Mueller-Hintonhasafewpropertiesthatmakeitexcellent
forantibioticuse....
-Starchisknowntoabsorbtoxinsreleasedfrombacteria,sothat
theycannotinterferewiththeantibiotics.
-Second,itisalooseagar.Thisallowsforbetterdiffusionof
theantibioticsthanmostotherplates.

685.Loeffer'sserumisanexampleof
a)Basalmedium
b)Simplemedium
c)Complexmedium
d)Enrichmentmedium
CorrectAnswer-C
Ans.is'c'i.e.,Complexmedium[RefAnanthanarayan8th/e.p.
140]
Loeffler'smediumisanenrichedmediumAtypeofspecial
(complex)medium.

686.Tocreateanaerobiosiswhichorganism
isused
a)Micrococcus
b)Clostridium
c)B.anthracis
d)Corynebacterium
CorrectAnswer-A
Ans.is'a'i.e.,Micrococcus
Obligateaerobicmicrococcusisusedtocreateanaerobiccondition
(anaerobiosis).
Themicrococcusandtheanaerobicorganismtobecultivatedare
bothinoculatedintothesameliquidmedium.
DuringincubationtheMicrococcusgraduallyutilizesthefreeoxygen
creatingconditionsfavorableforthegrowthofanaerobe.
Afteranaerobiosisisachieved,themicrococcusdies(duetolackof
oxygen)leavingtheanaerobeinpureculture.

687.Savloncontains
a)Cetrimide+Chlorhexidine
b)Cetrimide+Chlorhexidine+butylalcohol
c)Cetrimide+butylalcohol
d)Cetrimide+Cetavlon
CorrectAnswer-A
Ans:A.Cetrimide+chlorhexidine[RefKDTpharmacology6/e,
p861,860;Park20/e,p117]
Savloncontains-Chlorhexidinegluconate(hibitane)+Cetrimide
(cetavlon)invariouspercentagesSavlonliquidantiseptic-
chlorhexidinegluconate1.5%+cetrimide3%
Savloncream-chlorhexidineHC10.1%+cetrimide0.5%
Savlonhospitalconcentrate-chlorhexidinegluconate7.5%+
cetrimide15%
Otherantisepticanddisinfectantsofimportance:
Betadine-povidone(polyvinylpyrrolidone)iodine
Dettol-chloroxylenol4.8%in9%terpinol&13%alcohol
Dakin'ssolution-Dilutedsodiumhypochloritesolutionbufferedwith
boricacid.Availablechlorineis0.5%.
Eusol-Solutionofchlorinatedlime(1.25%)+boricacid(L25%)
0.4%availablechlorine
Dakin'ssolution&Eusoldissolvepus&necrotictissueinadditionto
beinggermicidal.Sousedincleaningofinfectedwound.

688.Whichofthefollowingisamethodof
Pasteurization
a)Vatmethod
b)Pasteurmethod
c)Billingmethod
d)Flashmethod
CorrectAnswer-A:D
Ans.is'a&d'i.e.,Vatmethod&Flashmethod[RefPark23ra/e
p.655]
Pasteurizationisdonetodestroythepathogensinmilk.Itkillsnearly
90%ofbacteriainmilk,includingmoreheatresistanttubercular
bacilliandQ-feverorganism.However,thermoduricbacteriaand
sporesarenotkilled.
Therearefollowingmethodsofpasteurization:?
A)Holdermethod(Vatmethod)
Milkiskeptat63-66?Cfor30minutesandthenrapidlycooledto
5?C.
B)Hightemperatureshorttime(HTST)method
ItisalsocalledFlashmethod.Milkisheatedto72?Cfor15seconds
andthenrapidlycooledto4?C.
Itisnowthemostwidelyusedmethod.
C)Ultra-hightemperature(UHT)method
Milkisrapidlyheatedintwostagesto125?Cforfewseconds,
2"dstagebeingunderpressure.Itisthenrapidlycooled.
Method
Remarks
Holder/Vat
Forsmallandrural
Method
communities
HTST(flash)
Mostwidelyused,forlarge

HTST(flash)
Mostwidelyused,forlarge
Method
quantities
Heatingin2stages
UHTMethod
2'dstageunderpressure

689.TemperatureusedinTyndallizaton
a)40?C
b)60?C
c)80?C
d)100?C
CorrectAnswer-D
Ans.is'd'i.e.,100?C[RefAnanthanarayan9thiep.31&
trie
p.33]
Tyndallization(intermittentsterilization)
Mediacontainingsugarorgelatinaresterilizedbyheatingat100?C
for20minutesonthreesuccessivedays.
Firstexposurekillsallvegetativebacteria.
Sporesgerminateandarekilledonsubsequentexposures

690.Laproscopeissterilizedby?
a)2%formalin
b)2%glutaraldehyde
c)Autoclaving
d)Boiling
CorrectAnswer-B
Ans.is'b'i.e.,2%glutaraldehyde
Allendoscopes(e.g.laproscope)aresterilizedby2%glutaraldehyde
(cidex).

691.Sterilizationofculturemediacontaining
serumisby:
a)Autoclaving
b)Microporefilter
c)Gammaradiation
d)Gammaradiation
CorrectAnswer-A
Ans.(a)Autoclaving

692.Nutrientagarheatedat80?Cusedfor
a)Sporegermination
b)Togrowmesophilicbacteria
c)Togrowthermophilicbacteria
d)Forclostridiumisolation
CorrectAnswer-A:D
Ans.is'a'>'d'i.e.,Sporegermination>Forclostridium
isolation
Heatingfor20minutesat80degreescentigradedestroysvegetative
cellsandactivatesthesporesforgermination.
Thismethodcanbeusedtocultivateanaerobicspore-forming
organisms(e.g.clostridium)
Aboutoptionsa&c
Microorganismscanbegroupedintobroad(butnotveryprecise)
categories,accordingtotheirtemperaturerangesforgrowth.
Psychrophiles(cold-loving)cangrowat0?C,andsomeevenaslow
as-10?C;theirupperlimitisoftenabout25?C.
Mesophllesgrowinthemoderatetemperaturerange,fromabout
20?C(orlower)to45?C.
Thermophilesareheat-loving,withanoptimumgrowthtemperature
of50?ormore,amaximumofupto70?Cormore,andaminimumof
about20?C.
Hyperthermophileshaveanoptimumabove75?Candthuscangrow
atthehighesttemperaturestoleratedbyanyorganism.Anextreme
exampleisthegenusPyrodictium,foundongeothermallyheated
areasoftheseabed.Ithasatemperatureminimumof82?,an
optimumof105?andagrowthmaximumof110?C.

693.Involutionalformareseeninwhich
phaseofbacterialgrowth
a)Lagphase
b)Logphase
c)Stationaryphase
d)Deathphase
CorrectAnswer-D
Ans.is'd'i.e.,Deathphase[RefAnanthanarayan94Vep.22]
Bacterialgrowthisthedivisionofonebacteriumintotwodaughter
cellsinaprocesscalledbinaryfission.Providingnomutationalevent
occurs,theresultingdaughtercellsaregeneticallyidenticaltothe
originalcell.
Hence"Localdoubling"ofthebacterialpopulationoccurs.Bacteria
haveadistinctpatternofgrowthwhenabacteriumisseededintoa
suitableliquidmediumandincubated,itsgrowthfollowsadefinite
course.

694.Partialacidfastorganismis
a)M.tuberculosis
b)M.Bovis
c)Nocardia
d)None
CorrectAnswer-C
Ans.is'c'i.e.,Nocardia[RefConciseReviewofmicrobiologyp.
13]
Portialacidfastmeans,organismwhichisshowslessaffinityfor
primarystain,thuslessconcentratedH2So4(insteadof20%H2So4)
isusedfordecolorizationofprimarystain(Carbolfuschin).
PartialacidfastbacteriaareMleprae(5%H,SO4),andNocardia
(0.5%H2SO4)

695.Resolvingpowerofelectronmicroscope
a)1-5mm
b)1-5um
c)1-5nm
d)1-5A?
CorrectAnswer-D
Ans.is'd'i.e.,1-5Ae[RefEssentialsofmedicalmicrobiology]

696.Classificationofstaphylococcusis
basedon-
a)Catalasetest
b)Coagulasetest
c)Mannitolfermentation
d)Optochinsensitivity
CorrectAnswer-B
Ans.is'b'i.e.,Coagulasetest
Medicallyimportantstaphylococcithatcausehumandisease
aredividedintotwogroups:?
Coagulasepositive:Staphylococcusaureus.
Coagulase-negative:Staphylococcusepidermidis,Staphylococcus
haemolyticus,Staphylococcussaprophyticus.


697.Ironhelpsinvirulenceofwhich
organism
a)Streptococcuspyogenes
b)Pneumococcus
c)Staphylococcusaureus
d)Pseudomonas
CorrectAnswer-C
Ans.is'c'i.e.,Staphylococcusaureus
Staphylococcusaureuscausesasignificantamountofhuman
morbidityandmortality.
TheabilityofS.aureustocausediseaseisdependentuponits
acquisitionofironfromthehost.
S.aureuscanobtainironfromvarioussourcesduringinfection,
includinghaemandtransferrin.
Themostabundantironsourceinhumansishaemironboundby
haemoglobincontainedwithinerythrocytes.
S.aureusisknowntolyreerythrocytesthroughsecretionofpore-
formingtoxins,providingaccesstohosthaemoglobin.
Proteinsoftheiron-regulatedsurfacedeterminant(Isd)systembind
hosthemoproteins,removethehaemcofactor,andshuttlehaem
intothecytoplasmforuseasanutrientironsource.
DeletionofIsdsystemcomponentsdecreasesstaphylococcal
virulence,underscoringtheimportanceofhaemironacquisition
duringinfection.
Inadditiontohaem,S.aureuscanutilizetransferrinironthroughthe
secretionofsiderophores.
Severalstaphylococcalsiderophoreshavebeendescribed,someof
whichhavedefinedrolesduringthepathogenesisofstaphylococcal

infections.

698.Allaretrueaboutlisteriaexcept:
a)Grampositive
b)PALCAMagarisusedforisolation
c)Characteristictumblingmotilityat37?C
d)Umbrellashapedgrowth
CorrectAnswer-C
Ans.is.'c'i.e.,Characteristictumblingmotilityat37?C
L.monocytogenesisagram-positive'
coccobacillus'(coccoidrod)withatendencytooccurinchains.
Peritrichousflagellaareproducedbythebacillusoptimallyat20-
30?Cbutonlyscantilyornotatallat37?C
Culturemediausedforisolationarebloodagar,chocolateagar,
andPALCAMagar.
Itgrowsonordinarymediawithinatemperaturerangeof1?to45?C.
Mostcasesofhumandiseasearecausedbyserotypes1/2a,1/2b
and4b.
Theorganismcanbefoundasapartofthegastrointestinalflorain
healthyindividuals.
HumandiseaseduetoL.monocytogenesgenerallyoccursinthe
settingofpregnancyorimmunosuppression.

699.DOCforlisteriameningitis-
a)Ampicillin
b)Cefotaxime
c)Cefotriaxone
d)Ciprofloxacin
CorrectAnswer-A
Answer-A.Ampicillin
Theantibioticofchoiceforlisteriainfectionisampicillinorpenicillin
G.

700.Whichofthefollowingisnotmechanism
forresistancetoMRSA-
a)Resistanceischromosomallymediated
b)ProducedmainlybyalterationinPBPs
c)MRSAresistanceisabsolutelybeta-lactamaseindependent
d)Intrinsicresistanceisknown
CorrectAnswer-C
Ans.is'c'i.e.,MRSAresistanceisabsolutelybeta-lactamhhase
independent

701.Wool-Sorterdiseaseiscausedby
a)Pseudomonas
b)Bacillusanthracis
c)Vibrioparahemolyticus
d)Spirillusminor
CorrectAnswer-B
Ans.is'b'i.e.,Bacillusanthracis[RefHarrison19(h/ep.261;
Ananthnarayan/ep.246]
BacillusAnthracisisthecausativeorganismofanthrax.
Inhuman,anthraxoccursinfollowingforms?
Cutaneousanthrax(Hideporter'sdisease)
Itisthemostcommonformofanthrax.Itisapainlesslesionandis
calledcharbonormalignantpustule.Itgenerallyresolves
spontaneously,but10-20%ofuntreatedpatientsmaydevelopfatal
septicemia.
Pulmonaryanthrax(Woolsorter'sdisease)
Itfollowsinhalationofdustfrominfectedwool.Itpresentsas
hemorrhagicpneumonia.
Intestinalanthraxisrarestform.

702.Allaretrueaboutcutaneousanthrax
except?
a)Extremelypainful
b)Thewholeareaiscongestedandedematous
c)Centralcrustationwithblackeschar
d)Satellitenodulearoundinguinalregion
CorrectAnswer-A
Ans.is'a'i.e.,Extremelypainful
Cutaneousanthraxispainless.

703.CapsuleofBacillusanthracisisformed
of:
a)Polysaccharide
b)Lipopolysaccharide
c)Polypeptide
d)Longchainfattyacids
CorrectAnswer-C
Ans.is.'c'i.e.Polypeptide

704.Fishyodourisfoundongrowthofwhich
organism
a)Proteus
b)Pseudomonas
c)Yersiniapseudotuberculosis
d)Yersiniapestis
CorrectAnswer-A
Ans.is'a'i.e.,Proteus
Proteus
ispartofthenormalfloraofthehumangastrointestinal
tract.Itcanalsobefoundfree-livinginwaterandsoil.Whenthis
organism,however,enterstheurinarytract,wounds,orthelungs
itcanbecomepathogenic
Cultureofproteusbacillihasacharacteristicputrefactiveodor
describedas'fishyorseminal".
Rottencookedfishyodor:Proteusmirabilisproducesavery
distinctfishyodor.OnSalmonella-Shigella(SS)
agar
,Proteususuallysmellslike"rottencookedfish".


705.ConfirmatorytestforSyphilisis:
September2010March2013
a)VDRL
b)Rapidplasmareagintest
c)FT-ABS
d)Alloftheabove
CorrectAnswer-C
Ans.C:FT-ABS
T.pallidumcannotbegrowninvitro
Diagnostictestsforsyphilis:Testsincludeserologictestsforsyphilis
(STS),whichconsistofscreening(reaginic)andconfirmatory
(treponemal)tests,anddarkfieldmicroscopy.
Reaginictestsuselipidantigens(cardiolipinfrombovinehearts)to
detectreagin(humanantibodiesthatbindtolipids).TheVenereal
DiseaseResearchLaboratory(VDRL)andrapidplasmareagin
(RPR)testsaresensitive,simple,andinexpensivereaginicteststhat
areusedforscreeningbutarenotspecificforsyphilis.Resultsmay
bepresentedqualitatively(e.g.,reactive,weaklyreactive,borderline,
ornonreactive)andquantitativelyastiters(e.g.,positiveat1:16
dilution).
Manydisordersotherthantreponemalinfections(e.g.,SLE,
antiphospholipidantibodysyndromes)canproduceapositive
(biologicallyfalse-positive)reagintestresult.CSF
reaginictestsarereasonablysensitiveforearlydiseasebutlessso
forlateneurosyphilis.CSFreagintestscanbeusedtodiagnose
neurosyphilisortomonitorresponsetotreatmentbymeasuring
antibodytiters.
Treponemaltestsdetectantitreponemalantibodiesqualitatively

andareveryspecificforsyphilis.Theyincludethefollowing:
Fluorescenttreponemalantibodyabsorption(FTA-ABS)test
MicrohemagglutinationassayforantibodiestoT.pallidum(MHA-TP)
T.pallidumhemagglutinationassay(TPHA)

706.Rapiddetectionofmeningococal
meningitisis
a)Bloodculture
b)CSFculture
c)PCR
d)None
CorrectAnswer-C
Ans.is'c'i.e.,PCR[RefBasicinmicrobiologyp.719]
"PCRassaycanbeperformedrapidlywithaturnaroundtimeof2
hourfromintiationofDNAextractiontotheissuingof
reports".Laboratorydiagnosisofmeningococci
SpecimensusedareCSF(forcases),nasopharyngealswab(for
carrier),blood(inmeningococcemia&earlymeningitis),and
petechiallesions(inmeningococcemia).
Bestspecimenforcase4CSF(bylumbarpuncture).
Bestspecimenforcarrier4Naspharyngealswab.

707.Presumptivediagnosisof
meningococcalmeningitisismade
earliestby-

a)CSFculture
b)PCR
c)Latexagglutination
d)CFT
CorrectAnswer-C
Ans.is'c'i.e.,Latexagglutination[RefEssentialsofmedical
microbiology
r3d/ep.412]
IncombinationwithaclinicalpictureCSFexaminationconsistent
withbacterialmeningitis,apresumptivediagnosisofbacterial
meningitiscausedbyN.meningitidis,S.pneumoniae,orH.
influenzaecanbymadeafterperformingaGramstainoftheCSF
sedimentorbydetectionofspecificantigensintheCSFbyalatex
agglutinationtestorusingRDTs.
Positiveresultsforanyofthesetestscanrapidlyprovideevidenceof
infectionevenifculturesfailtogrow.PCRandCSFcultureareused
fordefinitivediagnosis(notpresumptivediagnosis)

708.Legionnairediseaseiscausedby?
a)Motilegrampositive
b)Motilegramnegative
c)Non-motilegrampositive
d)Non-motilegramnegative
CorrectAnswer-B
Ans.is'b'i.e.,Motilegram-negative
Legionellaisagram-negative,non-capsulatedcoccobacilluswhich
ismotilebypolarorsubpolarFlagella.
Legionellacausesthefollowinginfections:
A)Pulmonaryinfections
Therearetwotypesofpulmonaryinfections:-
1.PontiacFever:Itisself-limitingflu-likeillnesswithincubation
period24-48hours.Thereisnopneumonia.
2.LegionnairesDisease:
Itisatypicalpneumoniawithanincubationperiodof2-l0days.
Itischaracterizedbycough.chestpain,hemoptysis,high-grade
fever,diarrhoea,confusion,'relativebradycardiaandhyponatremia'.
Iftheonsetofsymptomsoccurswithin10daysofdischargefromthe
hospital,nosocomiallegionnaire'sdiseaseshouldbesuspected.
Extrapulmonaryinfections
Themostcommonsiteofinfectionisthe
heart(myocarditis/pericarditis/endocarditis).

709.Pontiacfeveriscausedby:
a)Legionella
b)Listeria
c)Scrubtyphus
d)Leptospira
e)Rickettsia
CorrectAnswer-A
Ans.(a)Legionella
Pontiacfeverisamildnonfatalinfluenzalikeillnesscausedby
Legionellapneumophila.
Pontiacfexc,
AnacuteselflimitingfluelikeillnesswithIPof24-48hours
Malaise,fatigueandmyalgiaarethemostfrequentpresenting
symptoms
Pneumoniadoesn'tdevelop.
Completerecoverytakesplace,withoutantibiotictherapy.
Diagnosisisestablishedbyantibodydetection.

710.Sewerswabsaretakentodetect
a)Typhoidcasesincommunity
b)Choleracasesincommunity
c)Typhoidcarriersincommunity
d)Choleracarriersincommunity
CorrectAnswer-C
Ans.is'c'i.e.,Typhoidcarriersincommunity[RefTextbookof
practicalMicrobiology
p.721]
TyphoidcarriersaredetectedbySewer-swabmethod'.
Theuseofsewerswabsenablespremisestobescreenedforthe
possibilityoftheexistenceofasalmonellaproblemmorequicklyand
moreeasilythanbytheexaminationofmanyend-of-linesamples.
Intheeventofsalmonellaebeingfound,areturnvisitcanbepaid
anddetailedsamplestakentoattempttodefectthesourceof
contamination.

711.Speciesofshigellacausingarthritis
a)Shdysenteriae-1
b)Shsonnei
c)Shflexneri
d)Shboydii
CorrectAnswer-C
Ans.is'c'i.e.,Shflexneri[RefHarrison18th/ep.945]
Shigellaishighlycommunicable.Theinfectivedoseforshigellais
less.Itcanbeaslowas10-100bacillibecausetheysurvivegastric
aciditybetterthanotherenterobacteriae.
Shigellaeproducefollwingclinicalfeatures.
1)Intestinal:Theseare:-
Dysentery:MostcommoncauseisShdyenteriaetypel.
Diarrhea:UsuallybyShSonnei.
2)Extraintestinal:Thesearehemolyticuremicsyndrome(causedby
Shdysenteriae-I),arthritis(Shflexneri),seizures(Shflexneri),
pneumoniaandReitersyndrome(inHLAB27association).

712.Griffthclassificationisbaseon
a)'C'-carbohydrate
b)M,T,Rantigens
c)Typeofhemolysis
d)0,requirment
CorrectAnswer-B
Ans.is'b'i.e.,M,T,Rantigens[RefAnanthanarayan
9th/e
p.209,210]
Group'A'strep.arefurthersubdividedintotypesbasedonthe
protein(M,TandR)antigenspresentonthecellsurface(Griffith
typing).About80typesofstr.pyogeneshavebeenrecognized.

713.Howdoeschlamydiadifferfromother
usualbacteria?
a)Lackcellwall
b)Cannotgrowincellfreeculturemedia
c)Containsinclusionbody
d)Noneoftheabove
CorrectAnswer-C
Ans.is'c'i.e.,Containsinclusionbody
Chlamydiaproducesbasophilic(intracytoplasmic)inclusionbodiesin
infectedcellsincontrasttoeosinophilicinclusionbodiesproduced
bymostvirusesandhencetheyaresometimesreferredtoas
Basophilicviruses.
Uniquepropertiesofchlamydiaeare
Chlamydiaisanobligateintracellularparasite.Thismeanstheycan
surviveonlybyestablishigresidenceinsideanimalcells
Theyneedtheirhost'sATPasanenergysourcefortheirown
cellularactivity.Theyareenergyparasitesusingacellmembrane
transportsystemthatusesATPfromthehostsystemandgivesout
ADP.
Thisobligateintracellularexistencemakesitimpossibletoculture
theseorganismsonnonlivingartificialmedia.Duetotheirsmallsize
andfailuretogrowincell-freemediatheywereconsideredtobe
viruses.
ChlamydiaegrowsinculturesofavarietyofeukaryoticcelllinesMc
CoyorHeLacells.Itmaybenecessarytotreatcellswithpolyanionic
compoundssuchasDEAD-dextrantoreducetheelectrostatic
barriertoinfection.Antimetabolitesuchascycloheximideisaddedto
favourcompetitionforhostcellaminoacidpools.Alltypesof

chlamydiaeproliferateinembryonatedeggsparticularyintheyolk
sac.
Thespecialfeaturesinstructureandchemicalcompositionof
chlamydiaeare:
1. Theoutercellwallresemblesthecellwallofgramnegativebacteria
2. Ithasarelativelyhighlipidcontent
3. Itisrigidbutitdoesnotcontaintypicalbacterialpeptidoglycan;
perhapsitcontainatetrapeptidelinkedmatrix.
4. NAcetylmuramicacidalsoappearstobeabsentfromchlamydiae
cellwall.

714.Similaritybetweenchlamydiaandvirus
is
a)Filterablethroughfilter
b)Abilitytogrowincellfreemedia
c)ContainsbothDNAandRNA
d)Alloftheabove
CorrectAnswer-A
Ans.is'a'i.e.,Filterablethroughfilter[RefTextbookof
microbiologybyParijap.418]
Chlamydiaewerethoughttobevirusesbecause(likeviruses)
they:
Passthrough0.45ymfilters.
Areobligateintracellularparasite-cannotbegrownincellfree
media.

715.Chlamydiatrachomatisinfection
commonlycauses:
March2004

a)Infertility
b)Postcoitalbleeding
c)Amenorrhoea
d)Malignancy
CorrectAnswer-A
Ans.Ai.e.Infertility

716.Whichtestcannotdifferentiateendemic
andepidemictyphus
a)Weil-Felixreaction
b)Complementfixationtest
c)Immunofluorescence
d)Radioprecipitation
CorrectAnswer-A
Ans.is'a'i.e.,Weil-Felixreaction
BothepidemictyphusandendemictyphusarepositiveforOX-19
antigenThusWeilFelixreactioncannotdifferentiatebetweenthe
two.
Weil-Felixreaction
Thisreactionisanagglutinationtestinwhichseraaretestedfor
agglutininsto0antigensofcertainnonmotileproteusstrainsOX-19,
OX-2andOX-K.
Thebasisofthetestisthesharingofanalkali-stablecarbohydrate
antigenbysomerickettsiaeandbycertainstrainsofproteus,P.
vulgarisOX-19andOX-2andP.mirabilisOX-K.
Thetestisusuallydoneasatubeagglutination,thoughrapidslide
agglutinationmethodshavebeenemployedforscreening.

717.E.colisubtypesaredividedonthebasis
of
a)Lactosefermentation
b)Virulenceproperties
c)Somatic0antigen
d)Maltosefermentation
CorrectAnswer-B
Ans.is'b'i.e.,Virulenceproperties[Refenwikipedia.org]
EntericE.coli(EC)areclassifiedonthebasisofserological
characteristicsandvirulenceproperties:?
EnteropathogenicE.coli(EnteroadherentEcoli)
EnterohemorrhagicE.coliorverotoxigenicE.coli
EnterotoxigenicE.coli
EnteroaggregativeE.coli
EnteroinvasiveE.coli

718.A20yearoldmanpresentedwith
abdominalpain,vomitingandbloody
diarrhea,hisstoolsamplegrew
Escherichiacoliinpureculture.Whichof
thefollowingserotypeofE.coliisthe
causativeagentofhemorrhagiccolitis?

a)O157:H7
b)O159:H7
c)O107:H7
d)O55:H7
CorrectAnswer-A
EnterohemorrhagicE.coliO157:H7istheserotypeofE.colicausing
hemorrhagiccolitis.
Itisassociatedwiththeingestionofundercookedhamburger,
sprouts,unpasteurizedmilkorjuice.
EHECproducesashigatoxinandcancausecolitisafteran
incubationperiodof3-5days.
Ittypicallyproduceswaterydiarrheathatprogresstobloodydiarrhea
afterafewhourstofewdays.
Fatigue,abdominalpain,nauseaandvomitingareassociated
complaints.
Mechanismofenterohemorrhagiccolitisappearstobevascular
endothelialdamagethatleadstoplateletaggregationandinitiation
ofthecoagulationcascade.Thisinturn,leadstoischemiaofthe

colonandresultsinhemorrhagiccolitis.
Ref:MayoClinicGastroenterologyandHepatologyBoardReview
ByStephenHauser,4thEdition,Page197

719.ELISAtestforvirulenceantigenisused
forwhichtypeofEcoli
a)ETEC
b)EIEC
c)EHEC
d)EAEC
CorrectAnswer-B
Ans.is'b'i.e.,EIEC[RefAnanthnarayan9th/ep.279]
ForlaboratorydiagnosisofEIEC,theSerenytestusedtobe
employed(thatis,instillationofsuspensionoffreshlyisolatedEIEC
orshigellaintotheeyesofguineapigsleadstomucopurulent
conjunctivitiesandseverekeratitis).
Micemaybeusedinsteadofguineapigs.CellpenetrationofHeLa
orHEP-2cellsincultureisamorehumanediagnostictest.
Thisabilitytopenetratecellsisdeterminedbyalargeplasmid,
detectionofwhichcanalsobeadiagnostictest.Theplasmidcodes
foroutermembraneantigenscalledthe'virulencemarkerantigens'
(VMA)whichcanbedetectedbytheELISA(VMAELISA)test.

720.E.coliinfectionoccurinwhichenzyme
defect
a)Lactase
b)Pyruvatekinase
c)Pepsin
d)Trypsin
CorrectAnswer-B
Ans.is'b'i.e.,Pyruvatekinase[RefHardcoremicrobiology&
immunologybyBenjaminWSearsp.133]
Opportunisticinfectionsisimmunodeficiencyduetodefectin
myeloidcellsare:-
Staphaureus
Klebsiella
Pneumococcus
E.coli
Neisseria
Candida
Conditionscausingdefectinmyeloidcellsare:?
Chronicgranulomatousdisease
Chediak-Higashisyndrome
G6PDandpyruvatekinasedeficiency
Myeloperoxidasedeficiency

721.Culturemediacontainingpotassium
tellurite
a)TCBSmedium
b)Monsurmedium
c)BYCEmedium
d)MullerHintonagar
CorrectAnswer-B
Ans.is'b'i.e.,Monsurmedium[RefFundamentalprinciplesof
bacteriologyp.129]
Monsur'sGTTAmediumisMonsur'sgelatintourocholate
trypticasetelluriteagar.Selectivemediaforvibriocholarae
TCBSmedium(pH8.6):Thismediumcontainsthiosulfate,citrate,
bilesalts,sucrose,andbromothymolblue(indicator).V.cholerae
produceslarge,yellowconvexcoloniesonthismedium.
Thisisduetofermentationofsucrosebythebacteria,leadingto
productionofacid.AccumulationofacidreducespHofthemedium,
andsothecolorofthebromothymolblueindicatorbecomesyellow,
thusmakingV.choleraecoloniesyellow.Nonsucrose-fermentingV.
parahaemolyricusproducesbluegreencolonies.
Monsur'sGTTAmedium(pH8.5):HighpHofthemediumand
presenceofpotassiumtelluriteinthismediuminhibitsmostofGram
positivebacteriaandentericbacteriawithexceptionofproteus
species.
Hence,theGTTAmediumisusedforisolationofV.choleraeand
othervibriosfromfeces.V.choleraeproducessmalltranslucent
colonieswithgrayishblackcencerandaturbidhaloafter24hoursof
incubation.Thecoloniesbecomelarger(3-4mminsize)aftera
prolongedincubationof48hours.

AlkalineBSA(pH8.2):ThisisanotherselectivemediumusedforV.
cholerae.ThecoloniesonBSAaresimilartothoseonnutrientagar.

722.Borreliacauseswhichofthefollowing
a)Weil'sdisease
b)Bejels
c)Vincentangina
d)Yaws
CorrectAnswer-C
Ans.is'c'i.e.,Vincentangina
ThreeimportantdiseasescausedbyBorreliaeare:?
Lymedisease:CausedbyB.burgdorferi
Relapsingfever:CausedbyB.recurrentis,Bduttoni,B.hermsii,B.
Parkeri,B.turicatae,B.persica,B.hispanica.
Vincent'sangina:CausedbyB.vincenti.

723.Whichofthefollowingismostactive
againstslowlydividingtubercularbacilli
?

a)Isoniazid
b)Rifampicin
c)Streptomycin
d)Ethambutol
CorrectAnswer-B
Ans.is'b'i.e.,Rifampicin

724.GenerationtimeforMtuberculosis
a)10-15min
b)10-15hours
c)10-15days
d)5-10days
CorrectAnswer-B
Ans.is'b'i.e.,10-15hours[RefAnanthanarayan8th/ep.365]
Generationtimeofleprabacillus412-13days
Generationtimeoftuberclebacillus14hours
Generationtimeofcoliformbacilli20minutes

725.Gonococcihasaffinityfor:
a)Columnarepithelium
b)Glandularepithelium
c)Stratifiedsqamousepithelium
d)Squamousepithelium
CorrectAnswer-A
Columnarepithelium

726.Whichofthefollowingagentsismostcommonlyassociatedwithrecurrent
meningitisduetoCSFleaks?
a)Meningococci
b)Pneumococci
c)HemophilusInfluenza
d)E.Coli
CorrectAnswer-B
IntracranialCSFleakscausebacterialmeningitis,about80%arecausedbyS.
Pneumoniae.Othercausativeorganismsaremeningococcus,Hemophilusspeciesand
S.aurues.
Ref:ClinicalPediatricNeurologyByRonaldB.David,Page217

727.Trueaboutvibriocholeraeis-
a)Diseasemorecommoninwoman
b)Classicalvibrioprotectagainstdevelopmentof0-139Toris
milderthanclassical
c)El-Torismilderthanclassical
d)All
CorrectAnswer-C
El-Torismilderthanclassical

728.Diphtheriatoxin'smechanismofaction
is:
September2009

a)Inhibitingglucosesynthesis
b)Inhibitingproteinsynthesis
c)Promotingacetylcholinerelease
d)AlteringcyclicGMPlevels
CorrectAnswer-B
Ans.B:Inhibitingproteinsynthesis
Thediphtheriatoxincausesthedeatheucaryoticcellsandtissues
byinhibitionofproteinsynthesisinthecells.Althoughthetoxinis
responsibleforthelethalsymptomsofthedisease,thevirulenceof
C.diphtheriaecannotbeattributedtotoxigenicityalone,sincea
distinctinvasivephaseapparentlyprecedestoxigenesis.

729.Stalactitegrowthingheebrothisdueto
thefollowingorganism?
a)Y.pestis
b)T.palladium
c)H.influenzae
d)C.diphtheriae
CorrectAnswer-A
CulturalcharacteristicsofYersiniapestis:
1.Whengrowninaflaskofbrothwithoil/gheefloatedontop(ghee
broth)acharacteristicgrowthoccurswhichhangsdownintothe
brothfromthesurface,resemblingstalactite.
2.Onnutrientagar,coloniesaresmall,delicate,transparentdiscs,
becomingopaqueoncontinuedincubation.
3.Onbloodagarcoloniesaredarkbrownduetoabsorptionofthe
heminpigment.
4.OnMacConkeyagarcolourlesscoloniesareformed.
5.Inbrothaflocculentgrowthoccursatthebottomandalongthe
sidesofthetube.
Ref:TextbookofMicrobiologyAnanthanarayanan,8thedition

730.Cystwithscolexandhooksisseenin
a)T.saginatu
b)Fishtapeworm
c)Echinococcus
d)H.diminuta
CorrectAnswer-C
Ans.is'c'i.e.,Echinococcus[Ref:Clinicalparasitology
3rd/e
p.315]
Firstonemustunderstandthemeaningofscolex.Scolexispear-
shapedorknoblikeheadofcystodes(tapeworms).
Scolexiscoveredwithsuckersandhooks:-
1. Taeniasolium:Scolexcontainsfoursuckersandrostellarhooks
(rostellumwithhooks).
2. Taeniasaginata:Scolexcontainsfoursuckersbutno
hooks/rostellum(hookless).
3. H.Nana:Scolexcontainsfoursuckersandrostellumofhooks.
4. H.diminuta:Scolexcontainsfoursuckersbutnohooks(hookless).
5. Echinococcus:Scolexcontainssuckersandrostellumofhooks.
6. Diphyllobothriumlotus:Scolexhastwoelongatedsuckinggrooves
(insteadofsuckers).Therearenohooks(hookless).

731.E.coliisdifferentiatedbyEhistolytica
bypresenceof
a)Veryactivemovement
b)Thinnuclearmembrane
c)Cystwith1-4nuclei
d)Bluntpseudopodia
CorrectAnswer-D
Ans.is'd'i.e.,Bluntpseudopodia[Ref:Essentialsofmedical
parasitology
p.37]


732.Pulmonaryeosinophiliaisfoundin
infectionwith?
a)Babesia
b)Malaria
c)Strongyloides
d)Trypanosoma
CorrectAnswer-C
Ans.is'c'i.e.,Strongyloides[RefHarrison18th/ep.2120&17th/e
p.1610]
Ascaris
Strongyloidesstercoralis
Ancylostoma(hookworm)
WuchereriabancroftiorW.malayi
Toxocara

733.AdherenceofE.histolyticatocolonic
mucosaismediatedby?
a)Fibronectin
b)Lectin
c)Collagen
d)Fucose
CorrectAnswer-B
Ans.is'b'i.e.,Lectin[Refwww.pathologyoutlines.com]
PathophysiologyofE.histolyticainfection
Cystareingestedfromfecallycontaminatedfoodorwater,sexual
transmissionalsooccurs.
Excystationto8motiletrophozoitesoccursinthesmallintestine
Thecystsareresistanttogastricacid(andchlorineinwater
supplies).
Trophozoitesarepotentiallyinvasiveandmultiplybybinaryfission.
Inanestimated20%ofinfectionsinvasionintothewallofthecolon
withtissuedestructionoccurs.
AdherencetocolonicmucosaismediatedbyalectinonE.
histolytics'ssurface.
Theparasitetheninducesapoptosisofepithelialcellsthrougha
channelformingporeprotein
E.histolyticaingestsremnantcells.
Sometrophozoitesundergoencystationthroughsignallingpathways
completingthecycle.

734.Calabarswellingisproducedby?
a)Onchocercavolvulus
b)Loaloa
c)Burgiamalayi
d)Wuchereriabancrofti
CorrectAnswer-B
Ans.is'b'i.e.,Loa-Loa
Loiasis
LoiasisiscausedbyL.Loa(theAfricaneyeworm)
Habitatofadultwormsissubcutaneousconnectivetissueofman;
ofteninthesub-conjuctivaltissueoftheeye.
Thewormpassesitslifecycleintwohosts:
1)Man&2)Chrysops(Mangoordeerflies)
C/FsAsymptomaticmicrofilaremia,Calabar(fugitive)swelling-
subcutaneousswelling,Nephropathy,Encephalopathy(rare),
Cardiomyopathy
Calabarswellingisduetohypersensitivityreactiontotheadult
worm.

735.

PromastigoteformofLeishmaniaisfoundin
whichpartofsandfly:
March2005

a)Lymphnode
b)GIT
c)Spleen
d)Bonemarrow
CorrectAnswer-B
Ans.B:GIT

736.WhichstageofLeishmaniaisfoundin
spleenaspirateofpatient?
a)Amastigote
b)Promastigote
c)Epimastigote
d)Trypomastigote
CorrectAnswer-A
Ans.is'a'i.e.,Amastigote[RefEssentialsof
parasitology
p.122]
Amastigoteforms,alsoknownasLeishmaniadonovanibodies,are
foundintracellularlyintissueslikeliver,spleenandbonemarrow.A
positivebonemarroworspleenaspirationforL.donovanibodies
providesconfirmationofdiagnosis.

737.Maurer'sdotsareseeninwhichspecies
ofplasmodium?
a)Povale
b)Pvivax
c)Pfalciparum
d)Pmalariae
CorrectAnswer-C
Ans.is'c'i.e.,P.falciparum[RefChatterjee12th/ep.79-80]
P.vivax
Schuffner'sdot
P.malariae Ziemann'sstippling,
P.falciparum Maurer'sdot
P.ovale
Schuffner'sdot

738.Cholangiocarcinomaiscausedby:
a)Giardialamblia
b)Clonorchisinfestation
c)Paragonimusinfestation
d)Ascarisinfestation
CorrectAnswer-B
Ans.b.Clonorchisinfestation

739.Intermediatehostisnotrequiredfor
whichparasite-
a)Toxoplasma
b)Schistosoma
c)Ancylostoma
d)Fishtapeworm
CorrectAnswer-C
Ans.is'c'i.e.,Ancylostoma[RefRajeshKarykartep.7]
Thesequentialstagesingrowth,developmentandmultiplication
constituteitslifecycle

740.Capsidofviralstructureis:
a)Extracellularinfectiousparticle
b)Proteincoataroundnucleicacid
c)Enveloparoundavirus
d)Noneoftheabove
CorrectAnswer-B
Ans.b.Proteincoataroundnucleicacid

741.SmallestDNAvirusis?
a)Herpesvirus
b)Adenovirus
c)Parvovirus
d)Poxvirus
CorrectAnswer-C
Ans.is'c'i.e.Parvovirus
Smallestvirus(alsosmallestDNAvirus)Parvovirus.
Largestvirus(alsolargestDNAvirus)Poxvirus.
SmallestRNAvirusPicornavirus
LargestRNAvirusParamyxoviridae.

742.Allvirusesareassociatedwithspecific
inclusionbody,except?
a)CMV
b)Malluscumcontagiosum
c)EBV
d)Yellowfever
CorrectAnswer-C
Ans.is'c'i.e.,EBV[RefEssentialsofmedicalmicrobiologyp.791]

743.Handfootmouthdiseaseiscausedby?
a)Enterovirus-70
b)Coxsackie-Avirus
c)Coxsackie-Bvirus
d)Enterovirus
CorrectAnswer-B
Ans.is'b'i.e.,Coxsackie-Avirus[RefGreenwoodle/ep.459;
Ananthanarayan9th/e
p.491]
TherearetwotypesofCoxsackieviruses:
1. CoxsackieA(Seotypes1to24):Theycauseasepticmeningitis
(especiallyA7andA9),Herpangina,febrileillness,acute
hemorrhagicconjucntivitis(byA24),and'Hand-foot-mouthdisease'.
2. CoxsackieB(Sterotypes1to6):Theycauseasepticmeningitis(all
serotypes),neonataldisease,Bornholmdisease(plurodyniaor
epidemicmyalgia),myocarditis,hepatitis,pancreatitis&DM
(serotypeB4),andpneumonia.

744.Rashofchickenpoxcanbedifferentiated
fromtherashofsmallpoxbyallexcept?
a)Pleomorphic
b)Centripetal
c)Deep-seated
d)Unilocular
CorrectAnswer-C
Ans.is'c'i.e.,Deep-seated
Rashofchickenpoxissuperficial(notdeepseated).

745.HSV-2(Herpessimplex)causes?
a)Oralulcers
b)Genitalulcers
c)U.T.I.
d)Pharyngitis
CorrectAnswer-B
Ans.is'b'i.e.,Genitalulcer

746.HSV-2causeslatentinfectioninwhich
nerveplexus/ganglia?
a)Trigeminalganglion
b)Oticganglion
c)Sacralganglion
d)Ciliaryganglion
CorrectAnswer-C
Ans.is'c'i.e.,Sacralganglion[RefEssentialsofmedical
microbiology3'/ep.1215]
Siteoflatency
HSV-1Trigeminalganglion.
HSV-2Sacralganglion.

747.WhichofthefollowingisHepadnavirus?
a)HAV
b)HBV
c)HCV
d)HDV
CorrectAnswer-B
Ans.is'b'i.e.,HBV[RefAnanthnarayan9th/ep.549&8thlep.
545]


748.Coreantigen[HBOinHBVisencloded
bywhichgene?
a)S
b)C
c)P
d)X
CorrectAnswer-B
Ans.is`b'i.e.,C[RefAnanthanarayan9th/ep.544]
ThegenomeofHBVismadeofcircularDNA,butitisunusual
becausetheDNAisnotfullydoublestranded->oneofthestrands
isincompleteandotheriscomplete4partiallydoublestrandedDNA.
Therearefourknowngenesencodedbygenome-'C',X','P','S'.
Pgeneisthelargestgene..X-genecodesforHBxAg,whichcan
transactivatethetranscriptionofcellularandviralgenesandmay
contributetocarcinogenesis.HBxAganditsantibodyarepresentin
patientswithseverechronichepatitisandhepatocellular
carcinoma.


749.TrueaboutCMVareallexcept?
a)Mostcommoncauseofpost-transpinatationinfection
b)Mostcommoncauseoftransplacentalinfection
c)Anon-envelopedDNAvirus
d)Producesintranuclearinclusions
CorrectAnswer-C
Ans.is'c'i.e.,Anon-envelopedDNAvirus
CMVisanenvelopedDNAvirus,belongstoHerpesviridae.

750.RespiratorySyncytialVirus(RSV)causes
all,EXCEPT:
a)Coryzainkids
b)ARDS
c)Bronchitis
d)Commoncold
CorrectAnswer-B
RSVinfectionleadstoawidespectrumofrespiratoryillnesses.
Ininfantsitcancausepneumonia,bronchiolitis,and
tracheobronchitis.Inthisagegroup,illnessbeginsmostfrequently
withrhinorrhea,low-gradefever,oftenaccompaniedbycoughand
wheezing.
Inadults,themostcommonsymptomsarecommoncold,with
rhinorrhea,sorethroat,andcough.Itcauseseverepneumoniain
elderly.
Sinusitis,otitismedia,andworseningofchronicobstructiveand
reactiveairwaydiseasearealsoassociatedwithRSVinfection.
Ref:Harrison'sPrinciplesofInternalMedicine,18thEdition,Chapter
186

751.Rota-teqoralvaccineforrotavirus
contains?
a)2reassortedrotaviruses
b)3reassortedrotaviruses
c)4reassortedrotaviruses
d)5reassortedrotaviruses
CorrectAnswer-D
Ans.is'd'i.e.,5reassortedrotaviruses[RefEssentialofmedical
microbiologyp.932]
Rota-virusvaccine
TheRotavirusvaccinesarelive-attenuatedvaccinesgivenorally.
Twonewvaccinesarenowinuse-
1. RV5oralpentavalentvaccine(Rotateq)containsfivereassortant
rotavirusesdevelopedfrom5humanstrainsonbovinerotavirus
background[ThesefivestrainsareGI,G2,G3,G4andP(8)]
2. RV1monovalentvaccine(Rotarix)containsoneliveattenuated
rotavirusstrain[thestrainisP1A(8)GI]
TheadministrationofRotarix(2doses)andRotaTeq(3doses)
needstobecompletedby32weeksofagetominimizeanypotential
riskofintussusception.
Thefirstlicensedrotavirusvaccine,aRhesusmonkeyrotavirus-
basedtetravalenthumanreassortantvaccine(Rotashield),was
withdrawnafterthisliveoralvaccinewasassociatedwith
developmentofintestinalintussusception.

752.Whatissimilarbetweenrotavirusand
Norwalkvirus?
a)Bothbelongtosamefamily
b)Bothhavesegmentedgenome
c)BothhavesinglestrandedRNA
d)Botharecausesofviralgastroenteritis
CorrectAnswer-D
Ans.is'd'i.e.,Botharecausesofviralgastroenteritis[Ref
EssentialsofMicrobiologyp.497]
Rotavirusbelongstoreoviridaedoublestrandedsegmented
RNA.
NorwalkvirusbelongstocalciviridaeSinglestrandednon
segmentedRNA.
BothRotavirusandNorwalkvirusarecausesofviralgastroenteritis

753.Zikaviruscauses?
a)Hepatitis
b)Myocarditis
c)Conjunctivitis
d)Noneofthese
CorrectAnswer-C
Ans.is'c'i.e.,Conjunctivitis
Zikavirus(ZIKV)isaflavivirusrelatedtoyellowfevervirus.Itis
transmittedbyAedesmosquito.
Zikavirusdisease(Zika)isadiseasecausedbytheZikavirus,
whichisspreadtopeopleprimarilythroughthebiteofaninfected

754.Ebolavirusbelongsto?
a)Picornaviridae
b)Togaviridae
c)Flaviviridae
d)Filoviridae
CorrectAnswer-D
Ans.is'd'i.e.,Filoviridae

755.Poliovirusisshedinstoolupto-
a)6weeks
b)8weeks
c)10weeks
d)12weeks
CorrectAnswer-D
Ans.is'd'i.e.,12weeks
Inthefaces,thevirusisexcretedcommonlyfor2to3weeks,
sometimesaslongas3to4months.

756.Trueaboutrhabdoviridaeareallexcept?
a)Includesvesculostomatitisvirus
b)Rabiesvirusisinactivatedbyformalin
c)RabiesvirusisnegativesensedoublestrandedRNAvirus
d)Alloftheabovecorrect
CorrectAnswer-C
Ans.is'c'i.e.,Rabiesvirusisnegativesensedoublestranded
RNAvirus
Rhabdoviridaecontainstwogenera:?

1. Vesiculoviruses:Vesculostomatitisvirus,chandipuravirus.
2. Lyssavirus:Rabiesvirus.
Rabiesvirus
Itisaneneveloped,RNA(negativesensessRNA)virus.
IthasRNAdependentRNApolymerase.
Itisabulletshapedvirus.
Rabiesvirusissensitivetoethanol,iodinepreparations,soap,
quaternaryammoniumcompound,detergentsandlipidsolvents(like
ether,chloroform).Itisinactivatedbyphenol,beta-propiolactone
(BPL),formalin,Sunlight,UVirradiation,andbyheat.
Rabiesisprimarilyazoonoticdiseaseofwarm-bloodedanimals,
particularlycarnivoroussuchasdogs,Cats,Jackalsandwolves.
Itistransmittedtomanusuallybybitesorlicksofrabidanimals.
Itisthecommunicablediseasewhichisalwaysfatal.
Itisdeadendinfectioninman.

757.Negribodiesarecharacteristicof:
September2008,March2013
a)Tetanus
b)Rabies
c)Polio
d)AIDS
CorrectAnswer-B
Ans.B:Rabies
SincefirstdescribedbyNegriin1903,thepresenceoftheNegri
bodiesarepracticallypathognomicforrabiesandareanimportant
diagnosticfinding.
Negribodiesarediscrete,intracytoplasmic,deeplyeosinophilic
inclusionsthatmeasureseveralmicronsindiameter.Inabout75%
ofcasesofrabiesthesecanbeseenonhematoxylinandeosin
stainedsections.
Theyoccurinneuronsofthebrainstem,particularlythoseinthe
hippocampus,andinthePurkinjecellsofthecerebellum.
UltrastructuralstudieshaveshownthatNegribodyconsistsofa
massofnucleocapsidssurroundedbyviralparticlesbuddingfrom
intracytoplasmicmembranes.
Thosebodiescanbeseeninaxons,anditisinthiswaythatvirus
spreadsfromthecentralnervoussystemtomanyorgansofthe
body.
BecauseNegribodiesareusuallyseeninintactneurons,theyare
foundawayfromtheinflammatory,nonspecificlesions.Rabiesviral
antigenscanbedemonstratedininfectedcellsbymeansof
fluorescentantibodytechnique.
AntigenscanbeshowntobepresentincellsintheabsenceofNegri
bodies,andhencethistechniqueismuchmoresensitivethanthe

bodies,andhencethistechniqueismuchmoresensitivethanthe
searchofsectionsofbrainforthepathognomoniccytoplasmic
inclusions.

758.CCR5mutationisrelatedtowhich
condition?
a)ResistancetoHIVinfection
b)SusceptibilitytoHIVinfection
c)ResistancetoHBVinfection
d)SusceptibilitytoHBVinfection
CorrectAnswer-A
Ans.is'a'i.e.,ResistancetoHIVinfection[Ref
www.lumenlearning.com]
Inrecentyears,scientificinteresthasbeenpiquedbythediscovery
ofafewindividualsfromnorthernEuropewhoareresistanttoHIV
infectionin1998,AmericangeneticistStephenJ.O'Brienatthe
NationalInstitutesofHealth(NIH)andcolleaguespublishedthe
resultoftheirgeneticanalysisofmorethan4,000individuals.
TheseindicatedthatmanyindividualsofEurasiandescent(upto
14%insomeethnicgroups)haveadeletionmutation,calledCCR5
delta32,inthegeneencodingCCR5isacoreceptorfoundonthe
surfaceofTcellsthatisnecessaryformanystrainsofthevirusto
enterthehostcell.
ThemutationleadstotheproductionofareceptortowhichHIV
cannoteffectivelybindandthusblocksviralpeoplehomozygousfor
thismutationhavegreatlyreducedsusceptibilitytoHIVinfection,
andthosewhoareheterozygoushavesomeprotectionfrom
infectionaswell.


759.A52-year-oldmalewithHIVpresents
withprofuse,waterydiarrheaof5days'
duration.Abiopsyofthesmallintestine
isshownhere.Whatisthemostlikely
causeofthispatient'ssymptoms?

a)Cryptosporidium.
b)Giardia.
c)Acanthamoeba.
d)Histoplasma.
CorrectAnswer-A
Thecausativeorganisminthiscaseiscryptosporidium,ateeny
tinyprotozoan.
Cryptosporidiummainlyaffectschildren,inwhomitcauseseither
self-limitedorpersistentdiarrhea.ItalsoaffectspatientswithAIDS,
inwhomitcausessevere,chronicdiarrhea.Theorganismishighly
infectious,andisspreadbyfecal-oraltransmission.

Diagnosismaybemadeonacid-fastexaminationofthestool,in
whichcryptosporidialoocystsarevisibleasround,red-staining
structures.Histologicexamination,whichisusuallynotnecessaryfor
diagnosis,showstinyroundorganismsprojectingfromthe
brushborder,asseeninthephotomicrographabove
.Treatment
involvesantiparasitictherapyandnutritionalsupport.

760.BloodydiarrheainHIVinfectedpatientis
mostlydueto?
a)Cryptosporidium
b)Isospora
c)CMV
d)Salmonella
CorrectAnswer-C
Ans.is'c'i.e.,CMV[Refwww.medscape.com]
Themostcommonoftheopportunisticinfectionsthatcausediarrhea
inpatientswithAIDSareCMVinfection,cryptosporidiosis,
microsporidiosis,andMACinfection.CMVinfectionisthemost
commonopportunisticviralinfectionintheGItractofHIV-infected
patientsandcancauseproblemsfromthemouthtotheanus.It
mostfrequentlycausesacolitisassociatedwithfever,crampy
abdominalpain,andfrequent(oftenbloody)stools.

761.Sabouraud'sdextrose[glucose]agaris
usedforisolationof-
a)Pseudomonas
b)B.Antracis
c)Fungi
d)Ancylostome
CorrectAnswer-C
Ans.is'c'i.e.,Fungi[RefGreenwood10/ep.570;Ananthanarayan
8th/ep.601]
Culturemediausedinmycologyare:
1. Sabouraud'sglucoseagar(mostcommon)
2. Czapek-Doxmedium
3. Cornmealagar

762.Neurotropicfungusis/are-
a)Cryptococcusneoformans
b)Histoplasmosis
c)Trichophyton
d)aandb
CorrectAnswer-D
Ans.is'a'i.e.,Cryptococcusneoformans;'b'i.e.,
Histoplasmosis
Fungiinfectingthebrainare
Cryptococcusneoformans
Blastomycesdermatitidis
Coccidioidesimmitis
Aspergillussp.
Candidasp.
Sporothrixschenckii
Histoplasmacapsulatum

763.Hairperforationtestispositivein
infectionwith?
a)Trichophyton
b)Microsporum
c)Epidermophyton
d)Alloftheabove
CorrectAnswer-A
Ans.is'a'i.e.,Trichophyton
Hairperforationtest
.Thehairperforationtestconsistsofinoculatingcoloniesofan
organismintoadishcontainingasmallamountofwater,afewdrops
ofyeastextractsolutionandsomehumanhair.Thedishis
incubatedat300Candafter7days,someofthehairaretakenand
arekeptonaslidewithacoverslipandareobservedundera
microscope.Thetestisconsideredpositiveifthehairhasdeep,
narrowwedgeshapedperforationinit.
.Hairperforationtestisdonetodistinguishbetweenisolatesof
dermatophytes,particularlytrichophytonmentagrophytesandits
variants.

764.Ectothrixiscausedby?
a)Ttansurans
b)Tviolaceum
c)Microsporumcanis
d)Alloftheabove
CorrectAnswer-C
Ans.is'c'i.e.,Microsporumcanis[Ref:Greenwood16thiep.574]
Tineacapitisisdermatophyticinfectionofscalpandscalphair.
Occurscommonlyinchildren.Itisuncommoninadults(incontrast,
T.crusis,T.manuumandT.unguiumarecommoninadults).Caused
bygeneratrichophytonormicrosporum.Asepidermophytondoes
notaffectthehair,EpidermophytondoesnotcauseT.capitis.Two
mostcommonspeciescausingT-CapitisareM.canis&T.
tonsurans.Tineacapitismaybetwotypes.
1. EndothrixInvasionofhairshaftbyfungus.Itiscausedby
T.tonsurans(MC),T.violaceum,T.schoenleinii.
2. EctothrixFungalinvasionisrestrictedtotheoutermostcuticleof
hair.ItiscausedbyM.canis(MC),M.audouini,T.mentagrophytes.
AsectothrixisthecommonesttypeofT.capitis,Microsporumcanis
isthemostcommoncauseofT.Capitis

765.Aspergillusfumigatusisdifferentiatedby
otherfungiby?
a)Showingseptatehyphae
b)Growat45?C
c)Causerespiratorytractinfection
d)Mostcommonendemicmycosis
CorrectAnswer-B
Ans.is'b'i.e.,Growat45?C[RefHarrison19thiep.1346;
Ananthanarayan8th/e
p.613]
Aspergillusisamouldwithseptatebranchinghyphae.
AbilityofA.fumigatustogrowat45?Chelpstodistinguishitfrom
otherspecies.
Modeoftransmission-InhalationofAspergillusspores(Conidia)
intolungs.
Thecommonesthumandiseasecausedbyaspergillusis
otomycosis.
Aspergillusinfectioninneutropenicpatientischaracterizedby
hyphalinvasionofbloodvessels,thrombosis,necrosisand
hemorrhagicinfarction

766.Chlamydosporeisformedby?
a)Candidaalbicans
b)Candidapscitasi
c)Histoplasma
d)Cryptococcus
CorrectAnswer-A
Ans.is'a'i.e.,Candidaalbicans[RefAnanthanarayan8thlep.607-
608]
Allcandidaspeciespathogenicforhumansarealsoencounteredas
commensalsofhumans,particularlyinthemouth,stoolandvagina.
Theygrowrapidlyonsimplemediaasovalbuddingcellsat25?to
37?C.
Intissue,bothyeastsandpseudohyphaearepresent.
Candidaalbicansisdifferentiatedbyothercandida:
Itformstruehyphae(mycelia)orgermtubeswhengrowninserum.
Itformsthickwalledlargesporescalledchlamydosporeswhen
grownincornmealagar.
Itisdimorphic.

767.Trueaboutcryptococcusareallexcept
a)Primarilyinfectslung
b)Ureasenegative
c)India-inkisused
d)Allaretrue
CorrectAnswer-B
Ans.is`bi.e.,Ureasenegative
CRYPTOCOCCUSNEOFORMANS
Theonlypathogenicyeast
Fourcapsularserotypes-A,B,CandD
Ithaspolysaccharidecapsule
Mostinfectionsinimmunocompromizedpatientsarecausedby
serotypeA.
Pigeondroppings
commonlycontainsserotypeAandD.
Eucalyptustree
containserotypeB.
Itisureasepositive.
Modeoftransmission
Byinhalationofthefungusintothelung(mostcommon)
Throughskinormucosa(sometimes).

768.Pneumocystisjeroviciis:
a)AssociatedwithCMV
b)Diagnosisisbysputummicroscopy
c)Seenonlyinimmunocompromisedpatients
d)Alwaysassociatedwithpneumatocele
CorrectAnswer-B
Answer:B.Diagnosisisbysputummicroscopy
Humanisolateof.Pneumocystiswhichisassociatedwithsevere
pneumoniainimmunocompromisedstateparticularlyAIDS.
P.jeroviciisanextracellularpathogen.Growthinthelungis
limitedtosurfactantlayerabovealveolarepithelium.
Serologicevidencesuggestthatmostindividualsareinfectedin
earlychildhood(thusoption"c"iswrong)butthepneumoniais
seenonlyinimmunocompromisedstate.
Diagnosisismadebydetectionoforganisminproperspecimen
Sputum:Quickandnoninvasive.
Broncho-alaeolarlaoage(BAL)fluid:Mainstayofpneumocystis
diagnosis.
Transbronchialbiopsy:IfdiagnosiscannotbemadebyBAL.

769.Pneumocystiscariniiisdiagnosedby-
a)Sputumexaminationfortrophozoitesandcystunder
microscope
b)Culture
c)Positiveserology
d)Growthonartificialmedia
CorrectAnswer-A
Ans.is'a'i.e.,Sputumexaminationfortrophozoitesandcystunder
microscope

770.A60yearsoldfarmerhasdeveloped
swellingonthesoleoffootwith
dischargingyellowgranules.The
diagnosisis-

a)Fungalmycetoma
b)Eumycetoma
c)Actinomycosis
d)Candidiasis
CorrectAnswer-C
Ans.is'c'i.e.,Actinomycosis
Mycetoma
Isalocalizedchronicgranulomatousinvolvementofthe
subcutaneousanddeepertissues,commonlyaffectingthefootand
lessoftenthehandandotherparts.
Presentingasasubcutaneousswellingwithmultipledischarging
sinuses.
Sinusesdischargeseropurulentfluidcontaininggranules.
Thesegranulesaremicrocoloniesoftheetiologicalagents.
Mycetomacanbecausedbybothfungusandbacteria

771.Lipophilicfungusis?
a)Malasseziafurfur
b)Candida
c)Cryptococcus
d)Histoplasma
CorrectAnswer-A
Ans.is'a'i.e.,Malasseziafurfur[RefPrinciplesofmedical
microbiologyp.781]
Malasseziafurfur(Pityrosporumovale)isalipopphilicfungusthatis
foundinareasofthebodythatarerichinsebaceousglands.
Thisfunguscausestineaversicolor(pityriasisversicolor).

772.NottrueaboutsporothrixSchenckii?
a)Dimorphicfungus
b)Asteroidbodies
c)Copperpennybodies
d)Commoningardners
CorrectAnswer-C
Ans.is'c'i.e.,Copperpennybodies[RefHarrison19th/ep.1353]
Copperpennybodiesareseeninchromoblastomycosis.
SporothrixSchenckii
SporothrixSchenckiiisadimorphicfungus.
Theorganism(sporothrixschenckii)areusuallydescribedastiny,
cigar-shapedbodies,3-5micronsinlength,whichbearfromoneto
threesmallovalbudsateitherorbothpales.
Occasionallyalargerasteroidbodymaybeseen.
BecauseS.schenckiinaturallyfoundinsoil,hay,sphagnummoss,
andplants,itusuallyaffectsfarmers,gardeners,andagricultural
workers.
Thisfungaldiseaseusuallyaffectstheskinalthoughrareformscan
affectthelungs,joints,bonesandCNS.
Fungusentersthroughsmallcutsandabrasionsintheskintocause
theinfection.
Becauserosescanspreadthedisease,itisoneofafewdiseases
referredtoasrose-thornorrosegardener'sdisease.

773.Antibodyspecificityisdueto?
a)AminoacidsequenceatHchain
b)AminoacidsequenceatLchain
c)Aminoacidatcarboxyterminal
d)Aminoacidsequenceattheaminoterminal
CorrectAnswer-A:B:D
Ans.is'd>a&b'i.e.,Aminoacidsequenceataminoterminal>
AminoacidsequenceatHchain&AminoacidsequenceatL
chain
[RefReadbelow]
Antibodyspecificityisduetovariabilityoftheaminoacidsequences
atthevariableunit(ataminoterminal)ofbothHandLchains(not
onlyHorLchains).

774.Antibodydiversityisdueto-
a)Generearrangement
b)Genetranslocation
c)Antigenicvariation
d)aandc
CorrectAnswer-D
Ans.is'a'i.e.,Generearrangement;'c'i.e.,Antigenicvariation

775.Specificityofantibodyisdependenton?
a)Fcportion
b)Fabregion
c)Carboxyterminal
d)Alloftheabove
CorrectAnswer-B
Ans.is'b'i.e.,Fabregion[Ref:Harrison19th/ep.372;
Ananthanarayan9thie
p.100]
Theinfiniterangeoftheantibodyspecificityofimmunoglobulins
dependsonthevariabilityoftheaminoacidsequencesatthe
variableunitsofHandLchains.
VariableregionispresentonFabregion.

776.Mosteffectiveantibodyforprecipitation
?
a)IgM
b)IgG
c)IgA
d)IgD
CorrectAnswer-B
Ans.is'b'i.e.,IgG[RefHarrison19th/ep.372&18th/ep.2674;
Ananthanarayan9thiep.98&8th/ep.98]
IgMismoreeffectivethanIgGfor:
1. Immunehemolysis
2. Opsonization
3. Complementfixationbyclassicalpathway
4. Bacterialagglutination
IgGismoreeffectivethanIgMfor:
1. Neutralizationoftoxins&viruses
2. Precipitationreactions.

777.ComplementFixationtestis:
September2005
a)VIDAL
b)Coombstest
c)Wassermannreaction
d)VDRL
CorrectAnswer-C
Ans.C:Wassermannreaction
Thecomplementfixationtest(CFT)wasextensivelyusedinsyphilis
serologyafterbeingintroducedbyWassermanin1906.However,
thereisnowatrendtoreplacetheCFTwiththesimpleflocculation
tests.
AlthoughCFTisconsideredtobearelativelysimpletest,itisavery
exactingprocedurebecause5variablesareinvolved.Inessencethe
testconsistsoftwoantigen-antibodyreactions,oneofwhichisthe
indicatorsystem.
Thefirstreaction,betweenaknownvirusantigenandaspecific
antibodytakesplaceinthepresenceofapredeterminedamountof
complement.Thecomplementisremovedor"fixed"bytheantigen-
antibodycomplex.
Thesecondantigen-antibodyreactionconsistsofreactingsheep
RBCwithhaemolysin.Whenthisindicatorsystemisaddedtothe
reactants,thesensitizedRBCswillonlylyseinthepresenceoffree
complement.TheantigensusedforCFTtendtobegroupantigens
ratherthantype-specificantigens.InorderfortheCFTtobesetup
correctly,theoptimalconcentrationofhaemolyticserum,
complement,andantigenshouldbedeterminedbytitration.
TheWassermanntestisnolongerinuse.


778.Apatientispresentingwithrecurrent
staphylococcalinfection,kyphoscoliosis
andtypicalfaces.Thepatientissuffering
from?

a)IgAdeficiency
b)HyperIgEsyndrome
c)Commonvariableimmunodeficiency
d)Burton'sAgammaglobulinemia
CorrectAnswer-B
Ans.is'b'i.e.,HyperIgEsyndromeJob'ssyndrome(Hyper-IgE
syndrome)
Job'ssyndrome,alsocalledHyper-IgEsyndromeor
HyperimmunoglobulinEsyndrome,isanautosomaldominant
disorderduetomutationsinSignalTransducerandActivatorof
Transcription-3(STAT-3).
Thereisdefectinphagocytosis.IgElevelsareelevated.Other
immunoglobulinsarenormal.
Clinicalmanifestationsincludestaphylococcalcoldabscess,otitis
media,recurrentsinopulmonaryandskininfections,andcavitary
pneumoniawithpneumatoceles.
Besideinfections,otherfindingsarecharacteristicfacieswithbrood
nose,osteoporosis,kyphoscoliosis,cerebral.

779.18yearsoldgirlpresentswithwatery
diarrhea.Mostlikelycausativeagent-
a)Rotavirus
b)V.cholerae
c)Salmonella
d)Shigella
CorrectAnswer-B
Ans.is'b'i.e.,V.cholerae
AmongstthegivenoptionsRotavirusandV.choleraecausewatery
diarrhea.
Acutewaterydiarrhainchildrenisusuallybacterialinorigin,most
commonlyduetoenterotoxigenicE.coli(ETEC).Vcholeraeisalso
acommoncause.
Rotavirusisthemostcommoncauseofdiarrheaininfantand
children(thepatientinquestionisadult).

780.Swimmingpoolconjunctivitisiscaused
by
a)Chlamydiatrachomatis
b)Adenovirustype8
c)Adenovirustype8
d)Gonococcus
CorrectAnswer-A
Ans.,Chlamydiatrachomatis

781.A30yearsoldmaleishavingprpductive
coughwithdysnea.Bloodgasanalysis
showslowpa02.Chestx-rayisshowing
reticulonodularpattern.Thecausative
agentis?

a)Staphaureus
b)Pneumococcus
c)P.jerovecii
d)Pseudomonas
CorrectAnswer-C
Ans.is'c'i.e.,P.jerovecii[RefReadbelow]
Reticulo-nodularshadowisseenininterstitialpneumonia.
AmongthegivenoptionsPjeroveciiisacauseofinterstitial
pneumonia.
Otherthreeoptionsarecausesofairspacepneumonia.
"PneumocystisCariniipneumoniaproducesadiffuse,symmetric,
fine-to-mediumreticulonodularpattern".

782.Mostcommonorganismgrowninurine
cultureofpregnantwomanwith
asymptomaticbacteriuria?

a)Proteus
b)E.coli
c)Staphaureus
d)Pseudomonas
CorrectAnswer-B
Ans.is'b'i.e.,E.coli
Asymptomaticbacteriuriaisthepresenceofbacteriainavoided
urinesampleandiscausedbybacterialcolonizationoftheurinary
tract.Itaffectsabout5to10percentofbothsexuallyactiveand
pregnantwomen.Asymptomaticbacteriuriaislessprevalentinmen.
Asthenameindicates,asymptomaticbacteriuriadoesnotcause
symptoms.Theconditionsimplyreferstothedetectionofbacteriain
aurinesample.Nonetheless,thereisgoodreasontobeconcerned
aboutthisinfection,particularlyifyouarepregnant,becauseitcan
leadtoasymptomaticupperurinarytractinfection(namely,
pyelonephritis),whichcancomplicatepregnancy.
Bacteriaaretypicallyintroducedintotheurinarytractduring
intercourseorwhenwipingafterabowelmovement.Thebacterium
E.coliisresponsibleforatleast75to80percentofasymptomatic
bacteriuria.Klebsiellapneumoniea,Proteusspecies,staphylococcal
species,enterococci,andgroupBstreptococcicanalsoestablish
colonization.

783.

Apaitentoperatedfortransurethralresection
ofprostate[TURP]developsUTI.The
organismmostcommonlygrownonculture
willbe-

a)Proteus
b)E.coli
c)Pseudomonas
d)Listeria
CorrectAnswer-B
Ans.is'b'i.e.,E.coli[Refwww.jcam.com]
MostcommonorganismcausingUTIaftertransurethralprostectomy
isEcoli.

784.Cellulitissurroundingdiabeticulceris
mostlycausedby?
a)Streptococcuspyogenes
b)Staphylococcus
c)Mixedorganisms
d)Pseudomonas
CorrectAnswer-C
Ans.is'c'i.e.,Mixedorganisms[RefEssentialsofmedical
microbilogyp.712]
"Amixtureofgram-positivecocciandgram-negativeacrobesand
anaerobesisoftenimplicatedincellulitissurroundingdiabeticand
decubitusulcers".

785.Candidiasofpenisis?
a)Thrush
b)Leukoplakia
c)Balanitis
d)None
CorrectAnswer-C
Ans.is'c'i.e.,Balanitis[RefEssentialsofmedical
microbiology
p.712]
Candidalinfectionsareidentifiedbytheirlocationonthebodyas
follows:
Axillae,underpendulousbreasts,groin,interglutealfolds:intertrigo.
Glanspenis:balanitis
Follicularpustules:candidalfolliculitis
Nailfolds:candidalparonychia
Mouthandtongue:oralcnadidiasis(thrush)
Areaincludedunderdiaper-diaperdermatitis

786.Non-parasiticeosinophiliaiscausedby
infectionwith-
a)Staphylococcus
b)Ehrlischia
c)Coccidioidomycosis
d)Candidiasis
CorrectAnswer-C
Ans.is'c'i.e.,Coccidioidomycosis[RefDiagnostichematologyp.
332]
"Oneoftheveryfewnon-parasiticinfectionsthatregularlycause
eosinophiliaisCoccidioidomycosis,afungalinfection.
atypicalpneumonia

787.Atypicalpneumoniacanbecausedby
thefollowingmicrobialagentsexcept?
a)Mycoplasmapneumoniae.
b)Legionellapemmophila
c)HumanCoronavirus
d)Klebsiellapneumoniae
CorrectAnswer-D
AnswerisD(Kleibsellapneumonia):
CausesofAtypicalpneumonias.
1.Mycoplasmapneumonias
2.Viralpneumonias-Influenza
-RSV
-Adenovirus
-Rhinovirus
-Rubeola
-Varicilla
-Coronavirus
3.Chlamydiapneumonia
4.Coxiellabumetti
5.Pneumocystiscarinii
6.Legionella
Coronavirusisaninfrequentcauseofpneumonia.
SARSassociatedcoronavirus(SARS-CoV)causedepidemicof
pneumoniafromNov2002toJuly2003-Harrison


788.Healthindexcharacteristicsareall
except?
a)Validity
b)Reliability
c)Affordability
d)Feasibility
CorrectAnswer-C
Ans.is'c'i.e.,Affordability
Therehasbeensomeconfusionoverterminology:healthindicator
ascomparedtohealthindex(plural:indicesorindexes).
Ithasbeensuggestedthatinrelationtohealthtrends,theterm
indicatoristobepreferredtoindex,whereashealthindexis
generallyconsideredtobeanamalgamationofhealthindicators
(56).
Characteristicsofindicators
Indicatorshavebeengivenscientificrespectability:forexampleideal
indicators.
Shouldbevalid,i.e.,theyshouldactuallymeasurewhattheyare
supposedtomeasure;
Shouldbereliableandobjective,i.e.,theanswersshouldbethe
sameifmeasuredbydifferentpeopleinsimilarcircumstances.
Shouldbesensitive,i.e.,theyshouldbesensitivetochangesinthe
situationconcerned.
Shouldbespecific,i.e.,theyshouldreflectchangesonlyinthe
situationconcerned,
Shouldbefeasible,i.e.,theyshouldhavetheabilitytoobtaindata
needed,and;
Shouldberelevant,i.e.,theyshouldcontributetotheunderstanding

ofthephenomenonofinterest.

789.Trueforepidemiologicaltriad
a)Time,place,person
b)Agent,host,environment
c)Disease,prevention,treatment
d)Agent,man,disease
CorrectAnswer-B
Ans:Bi.e.Agent,host,environment
Epidemiologicaltriad
Theoccurrenceandmanifestationsofanydisease,whether
communicableornoncommunicable,aredeterminedbythe
interactionofthefollowingthreefactors.
1. Theagent
2. Thehost
3. Theenvironment
Thesethreefactorstogetherconstitutetheepidemiologicaltriad.
Merepresenceofthesefactorsdoesnotcausedisease,the
interactionofthesethreeisrequiredforthecausationofadisease.

790.PQLIis
a)Objectivecomponentoflevelofliving
b)Subjectivecomponentoflevelofliving
c)Objectivecomponentofqualityoflife
d)Subjectivecomponentofqualityoflife
CorrectAnswer-D
Ans.is'd'i.e.,Subjectivecomponentofqualityoflife
Subjectivecomponent
Thesubjectivecomponentofwellbeingisqualityoflife.
Theleveloflivingandstandardoflivingareobjectivecriteriaofwell
being,whilethequalityoflifecomprisestheindividual'ssubjective
evaluationofthese.
Therecentdefinitionofqualityoflifeisasfollows"acomposite
measureofphysicalmentalandsocialwellbeingasperceivedby
eachindividualorgroupofindividuals."
WHOdefinitionisasfollows"theconditionofliferesultingfromthe
combinationoftheeffectsofthecompleterangeoffactorssuchas
thosedetermininghealth,happiness,education,etc."
Theindexforqualityoflifeis"Physicalqualityoflifeindex(PQLI)".
ThePQLIisanattempttomeasurethequalityoflifeorwell-beingof
acountry.
Physicalqualityoflifeindexconsolidatesthreeindicators:-
Literacyrate
Infantmortalityrate
Lifeexpectancyatage1year(LEI)
PQLIrangesfrom0to100.
PQLIinIndiais65.

791.Onlydiseasewhichiseradicated
worldwide?
a)Smallpox
b)Polio
c)Diphtheria
d)Measles
CorrectAnswer-A
Ans.is'a'i.e.,Smallpox
Eradicationimpliesterminationofalltransmissionofinfectionby
exterminationofinfectiousagent.
Itisanabsoluteprocess,i.e.allornonephenomenon.
Itliterallymeans"tearingoutbyroots".
Eradicationisa"globalterm"usedonlycessationofinfectionfrom
thewholeworld.
Smallpoxistheonlydiseasethathasbeeneradicated.
OtherdiseaseswhicharecandidatesforglobaleradicationPolio,
measles,dracunculiasis,diphtheria.

792.Endemicdiseaseisdefinedas-
a)Diseaseoccuringregularlyinexpectedfrequency
b)Diseaseoccuringirregularly
c)Diseaseoccuringinexcessofexpectedfreqency
d)Diseaseaffectingalargepopulation
CorrectAnswer-A
Ans.is'a'i.e.,Diseaseoccuringregularlyinexpected
frequency[RefPark23"/ep.93&22"/ep.89]
SporadicDiseaseoccuringirregularlyfromtimetotime.
EndemicDiseaseoccuringregularlyinexpectedfrequency.
EpidemicDiseaseoccuringinexcessofexpectedfrequency.
PandemicEpidemicaffectingalargeproportionofpopulation
overawidegeographicarea.

793.Medianincubationperiodis?
a)Maximumtimefromexposuretodevelopmentofsymptomsin
allcases
b)Minimumtimefromexposuretodevelopmentofsymptomsinall
cases
c)Timefromexposuretodevelopmentin50%ofcases
d)Noneoftheabove
CorrectAnswer-C
Ans.is'c'i.e.,Timefromexposuretodevelopmentin50%of
cases
Incubationperiodisthetimeintervalbetweeninvasionbyan
infectiousagentandappearnaceofthefirstsignandsymptom.
'Medianincubationperiod'isthetimerequiredfor50%casesto
occurfollowingexposure.

794.Standpipeinruraloreasisanexampleof
whichprincipleofprimaryhealthcare?
a)Equitabledistribution
b)Communityporticipation
c)Intersectoralcoordination
d)Appropriatetechnology
CorrectAnswer-C
Ans.is'c'i.e.,Inter-sectoralcoordination[RefPrimaryheath
careonline;Park23'/ep.742]
Publicworks,eg.ensuringanadequatesupplyofsafewater(Stand
pipe)andbasicsanitation,comesunderinter-sectoralcoordination
(multi-sectoralapproach).
Intersectoralcoordination
Theprimaryhealthcareisnotprovidedbyhealthsectoralone.
Itinvolvesallotherrelatedsectorsofnationalandcommunity
development,inparticularagriculture,animalhusbandry,food,
industry,education,housing,publicworks(e.g.anadequatesupply
ofsafewaterandbasicsanitation),communicationandother
sectors.

795.Secondaryattackrateisameasureof-
a)Communicability
b)Lethality
c)Strengthofassociation
d)None
CorrectAnswer-A
Ans.is'a'i.e.,Communicability
SARisanimportantmeasureofcommunicability.
Highersecondaryattackratemeans,morenumbersofsusceptible
contactsaredevelopingthediseaseafterexposuretoprimarycase.
So,highertheSARhigheristhecommunicability(infectiousness)of
disease.
Onetermrelatedtosecondaryattackrateisattackrateandrequires
specificmentionhere(Hasbeenexplainedinbriefpreviously)
AttackRate:
Whennewcasesoccurrapidlyoverashortperiodoftimeinawell
definedpopulationtheattackrateisused.oItisusuallyexpressed
asapercentage.
newcasesoccuringduringashortperiodof
time
AttackRate=
________________________________________________________________x
100
populationatriskatthebeginningofthetime
perion
Theattackrateisalsocalledcumulativeincidencerate.Itdiffers
fromtheconventionalincidencerateinthatittendstodescribe
diseaseoreventsthataffectalargerproportionofthepopulationof
interest.

interest.
Attackrateisusedwhen"populationisexposedtoriskforalimited
periodoftime,Suchasepidemic."
Attackrateeffectstheextentof
epidemic.
Attackratevssecondaryattackrate
Attackrateincludesnewcasesduringaspecificperiodoftime
(whichisusuallyshort)anditincludesallthecasesduringthat
period,i.e.primaryaswellassecondarycases.
Ontheotherhandsecondaryattackrateincludesnewcaseswhich
developwithintherangeofincubationperiodafterexposureto
primarycase,i.e.itincludesonlysecondarycase(notprimary
cases).

796.Apopulationof50childrenishaving10
immunizedagainstchickenpox.5
childrendevelopedchickenpoxonmarch
2017.Other28childrendeveloped
chickenpoxwithinnext2weekwhatis
theSARofchickenpox?

a)60%
b)70%
c)80%
d)90%
CorrectAnswer-C
Ans.is'c'i.e.,80%[Ref:ParkMllep.105-107&23ra/ep.100]
Primarycasesinthequestion5(developingchickenpoxonsame
day)
Immunechildren10
SusceptiblecontactsTotalchildren-(Primarycases+immunized
children)=50-(5+10)=35
No.ofsusceptibledevelopingdesease=28
Noofsusceptibledevelopingdiseasex100=80%
Totalnumberofsusceptiblesx

797.Denominatorinunder-5proportional
mortalityrate?
a)Numberofdeathunder5yearsofage
b)Mid-yearunder-5population
c)Totaldeaths
d)Mid-yearpopulation
CorrectAnswer-C
Ans.is'c'i.e.,Totaldeaths

798.Berkesonianbiasisatypeof?
a)Admissionratebias
b)Intervieverbias
c)Informationbias
d)Recallbias
CorrectAnswer-A
Ans.is'a'i.e.,Admissionratebias[Ref:Park24thlep.78-79]
Berkesonianbias
Berksonianbiasresultsfromthegreaterprobabilityofhospital
admissionforpeoplewithtwoormorediseasethanforpeoplewith
onedisease.So,itisalsoknownasadmissionratebias.

799.Trueabouticebergofdisease?
a)Clinicianisconcernedwithhiddenportionoficeberg
b)Tipoftheicebergrepresentclinicalcases
c)Tetanusisclassicalexample
d)ScreeningisdoneforTipoftheiceberg
CorrectAnswer-B
Ans.is'b'i.e.,Tipoftheicebergrepresentclinicalcases
Icebergofdisease
*Diseaseinacommunitymaybecomparedwithaniceberg.
*Thefloatingtipoftheicebergrepresentswhatthephysiciansees
inthecommunity,i.e.clinicalcases(Diagnosedcase,symptomatic
caseoftheclinicallyapparentcase).
*Thevastsubmergedportionoftheicebergrepresentsthehidden
massofdisease,i.e.latent,inapparent,presymptomaticand
undiagnosedcasesandcarriersinthecommunity.
-The"waterline"representsthedemarcationbetweenapparent
andinapparentdisease.
-Anepidemiologistisconcernedwiththehiddenportionofthe
icebergwhereastheclinicianisconcernedwiththetipofthe
iceberg.
-ScreeningisdoneforaHiddenportionoftheicebergwhereas
diagnosisisdoneforthetipoftheiceberg.
*TheicebergphenomenonofdiseaseisnotshownbyRabies,
Tetanus,Rubella,andMeasles.
*Theclinicianconcernedonlywiththetipoficeberg,i.e
symptomaticcasesthatareseeninclinicaltreatment,thiscanresult
ininaccurateviewofthenatureandcausesofadiseaseresults
becausetheminorityofthecasesarestudied(hiddencases:-

submergedportionoficebergisnotstudied)-->Clinician'sFallacy.

800.Liveinfluenzavaccineisgivenbywhich
route?
a)Intradermal
b)Subcutaneous
c)Intramuscular
d)Intranasal
CorrectAnswer-D
Ans.is'd'i.e.,Intranasal[RefPark24thiep.168]
Theroutesofimportantvaccinesare:?
Subcutaneous:Measles,rubella,killedinfluenza,killedcholera,
IPV,yellowfever.
Intramuscular:Mumps,killedinfluenza,typhoidVi-polysaccharide,
DPT(deepintramuscular),rabies,IPV.iii)Intradermal:BCG,rabies.
Nasal:Liveinfluenza.
Oral:OPV,oralcholera,oraltyphoid(typhoral).

801.Whichofthefollowingistrue?
a)Twolivevaccinesshouldnotbegiventogether
b)Liveandkilledvaccineshouldnotbegiventogether
c)Immunoglobolinshouldnotbegivenforatleast6weekswhen
alivevaccineisadministered
d)Livevaccineshouldnotbegivenfor12weeksifimmunoglobin
hasbeengiven
CorrectAnswer-D
Ans.is'd'i.e.,Livevaccineshouldnotbegivenfor12weeksif
immunoglobinhasbeengiven.
Twolivevaccinescanbegivensimultaneously,buttheyshouldbe
givenatdifferentsites.Otherwisetheyshouldbegivenataninterval
ofatleast3weeks(ifadministeredatsamesite).
Liveandkilledvaccinecanbegiventogether.
Livevaccinesshouldnotnormallybegivenfor12weeksafteran
injectionofnormalhumanIgandifalivevaccinehasalreadybeen
given,HumanIginjectionshouldbedeferredfor2weeks.

802.HALEisusedtomeasure
a)Disabilityadjustedlifeexpectancy
b)Healthylifeexpectancy
c)Qualityadjustedlifeexpectancy
d)Expectancyfreeofdisability
CorrectAnswer-B
Ans.is'b'i.e.,Healthylifeexpectancy
Health-Adjustedlifeexpectancy(HALE)
HALEistheindicatorusedtomeasureahealthylifeexpectancy.
HALEisbasedonthelifeexpectancyatbirthbutincludesan
adjustmentfortimespentinpoorhealth.
Itistheequivalentnumberofyearsinfullhealththatanewborncan
expecttolivebasedoncurrentratesofillhealthandmortality.

803.Duringinvestigationofanepidemic,the
areaisdeclaredfreeofepidemicwhen?
a)Twicetheincubationperiodofthediseasesinceoccurrenceof
thelastcase
b)Thricetheincubationperiodofthediseasesinceoccurrenceof
thelastcase
c)Thelongestincubationperiodforthedisease
d)Incubationperiodforthediseaseplustwostandarddeviations
CorrectAnswer-A
Ans.is'a'i.e.,Twicetheincubationperiodofthediseasesince
occurrenceofthelastcase[RefPark22"d/ep.123]
Therearefollowingstepsintheinvestigationofanepidemic:?

1. Verificationofdiagnosis:Thisisthefirststepininvestigationofan
epidemic.
2. Confirmationofexistenceofanepidemic:Bycomparingwith
diseasefrequenciesduringsameperiodinpreviousyears.
3. Definingthepopulationatrisk.
4. Rapidsearchforallcasesandtheircharacteristics:Searchfornew
casesiscarriedouteveryday,tilltheareaisdeclaredfreeof
epidemic;thisperiodisusuallytakenas"twicetheincubationperiod
ofthediseasesincetheoccuranceoflastcase".
5. Dataanalysis.
6. Formulationofhypothesis.

804.Specificcontentinmalariavaccineis?
a)Gametocyticprotein
b)Polysaccharidesheath
c)Sporozoiteprotein
d)Lipoproteinenvelop
CorrectAnswer-C
Ans.is'c'i.e.,Sporozoiteprotein[RefInternet]
Circumsporozoiteprotein(CSP)isasecretedproteinofthe
sporozoitestageofthemalariaparasite(plasmodiumsp)andthe
antigenictargetofRTS,S,apre-erythrocyticmalariavaccine
currentlyundergoingclinicaltrails.Theamino-acidsequenceof
CSPconsistsofanimmunodominantcentralrepeatregionflanked
byconservedmotifsattheN-andC-terminithatareimplicatedin
proteinprocessingastheparasitetravelsfromthemosquitotothe
mammalianvector.
ThestructureandfunctionofCSPishighlyconservedacrossthe
variousstrainsofmalariathatinfecthumansnon-humanprimates
androdents.
Itcanfirstbedetectedinlargequantitiesassporozoitesareforming
withinoocystsresidingthemidgutwallsofinfectedmosquitoesUpon
egressionfrommatureoocysts,sporozoitsbeginmigratingtothe
salivaryglands,andCSPisknowntobeanimportantmediatorof
thisprocess.AdditionallyCSPisinvolvedinhepatocytebindingin
themammalianhost.
HeretheN-terminusandcentralrepeatregioninitiallyfacilate
parasitebinding.Oncethehepatocytesurfaceproteolyticcleavage
atregion1oftheN-terminusexposestheadhesivedomianoftheC-
terminus,therebyprimingtheparasitesforinvasionoftheliver.


805.Roleofmagnesium[Mg]inOPV?
a)Adjuvant
b)Preservative
c)Stabilizar
d)Antiinfective
CorrectAnswer-C
Ans.is'c'i.e.,Stabilizer[Refwww.who.int]
"PolioSabin(oral)vaccineisamagnesiumchloridestabilized
preparationofliveattenuatedpoliovirusesofsabinstrainstype1,2,
er3"Stabilizers
Theseareusedtoconfirmproductqualityorstability.Examplesare
potassiumorsodiumsalts,lactose,humanserumalbumin,gelatin
andbovineserumalbumin

806.Thedifferencebetweendescriptiveand
analyticstudies?
a)Descriptivestudiesareusedtotesthypothesis
b)Analyticstudiesareusedtoformulateahypothesis
c)Descriptivestudiesarefirstphaseinepidemiology
d)Analyticstudiesobservedistributionofdisease
CorrectAnswer-C
Ans.is'c'i.e.,Descriptivestudiesarefirstphasein
epidemiology[RefPark24thiep.67-75]
Descriptivestudiesarefirstphaseofanepidemiological
investigation.Thesestudiesareconcernedwithobservingthe
distrubutionofdiseaseintime,placeandperson.Descriptivestudies
areusedtoformulateeitologicalhypothesis.
Analyticalstudiesarethesecondmajortypeofepidemiological
studies(afterdescriptivestudies).Itcontrasttodescriptivestudies
thatlookatentirepopulation,inanalyticstudies,thesubjectof
interestistheindividualwithinthepopulation(exceptinecological
study).Theobjectisnottoformulate,buttotesthypothesis.
Inexperimentalstudies,epidemiologistactivelyintervenetochange
adiseasedeterminantorprogressionofdisease.Experimental
studiesareusedfortestinghypothesis.

807.Strengthofassociationofoutcomeand
riskfactorismeasuredby?
a)Relativerisk
b)Attributablerisk
c)Populationattributablerisk
d)Noneoftheabove
CorrectAnswer-A
Ans.is'a'i.e.,Relativerisk[Ref:Park24`Vep.83]
Relativeriskisadirectmeasureofthestrengthofassociation
betweensuspectedcauseandeffect.Forexamplearelativeriskof
2meansthattheincidencerateis2timeshigherintheexposed
groupascomparedwithunexposed->i.e.,a100%increaseinrisk.
Ontheotherhand,attributableriskindicatestheextentwhichis
attributedbyriskfactor(exposure)todisease.Forexample
attributableriskof90%means90percentofdiseaseamong
exposedisduetoexposuretoriskfactor.
Insimplewords:?
Relativeriskindicatestheincreasedpercentageofriskofdeveloping
adisease,ifpersonisexposedtoriskfactor.
Attributableriskindicatesthepercentageofdiseasewhichis
attributedbyriskfactoramongtheexposed.
Relativeriskisabetterindexthanisattributableriskforassessing
theetiologicalroleofafactorindisease.

808.ImportantmeasureforNationalhealth
policy?
a)Relativerisk
b)Oddsratio
c)Incidence
d)Attributablerisk
CorrectAnswer-D
Ans.is'd'i.e.,Attributablerisk[RefPark24th/ep.83]
RelativeriskVsAttributablerisk
Relativeriskisadirectmeasureofthestrengthofassociation
betweensuspectedcauseandeffect.Forexamplearelativeriskof
2meansthattheincidencerateis2timeshigherintheexposed
groupascomparedwithunexposed-i.e.,a100%increaseinrisk.
Ontheotherhand,attributableriskindicatestheextentwhichis
attributedbyriskfactor(exposure)todisease.Forexample
attributableriskof90%means90percentofdiseaseamong
exposedisduetoexposuretoriskfactor.
Insimplewords:-
Relativeriskindicatestheincreasedpercentageofriskofdeveloping
adisease,ifpersonisexposedtoriskfactor.
Attributableriskindicatesthepercentageofdiseasewhichis
attributedbyriskfactoramongtheexposed.
Relativeriskisabetterindexthanisattributableriskforassessing
theetiologicalroleofafactorindisease.
Ontheotherhand,attributableriskgivesabetterideathandoes
relativeriskoftheimpactofsuccessfulpreventiveorpublichealth
programmemighthaveinreducingtheproblem.Thatmeans
attributableriskreflectthepublichealthimportancebetterthan

relativerisk.

809.Astudythatgivestheprevalenceof
delusionintheelderlyatagivenpointof
time?

a)Case-controlstudy
b)Cohortstudy
c)Cross-sectionalstudy
d)Ecologicalstudy
CorrectAnswer-C
Ans.is'c'i.e.,Cross-sectionalstudy
Cross-sectionalstudies
Across-sectionalstudyisthesimplestformofanobservational
study.
Itisalsoknownasaprevalencestudy.
Itisbasedonasingleexaminationofacross-sectionofthe
populationatonepointoftime.
Theresultsofthisexaminationcanbeprojectedonthewhole
population.
Thecross-sectionalstudytellsaboutthedistributionofadisease
ratherthanitsetiology.
Cross-sectionalstudiescanbethoughtofasprovidingasnapshotof
thefrequencyandcharacteristicofadiseaseinapopulationata
particularpointintime.
Across-sectionalstudyismoreusefulforchronicdisease,asthe
populationisstudiedatonce,nofollow-upisrequired.

810.100individualsarediagnosedwithlung
cancerinapopulationof100000.Outof
100patients,80weresmokersand20000
weresmokersintotalspopulation.What
isPAR?

a)60
b)75
c)80
d)90
CorrectAnswer-B
Ans.is'b'i.e.,75[RefPark241thlep.83]
Inthegivenquestion:
Incidenceintotalpopulation=100per1lac
Numberofexposed=20000(Allexposed)
Numberofnon-exposed=80000(100000-20000)
Non-exposedhavinglungcancer=20(outof100patients80were
smoker.Thus20arenoneexposed)
Incidenceamongnon-exposed=20per80000or25per100000

811.
Recallbiasismostcommonly
associatedwithwhichstudydesign-
a)Casecontrolstudy
b)Cohortstudy
c)Cohortcasecontrolstudy
d)Cross-sectionalstudy
CorrectAnswer-A
Ans.is'a'i.e.,CasecontrolStudy
Recallbias(Memorybias):
Thistypeofbiasmayoccurwhencasesandcontrolsareaskedto
recallcertainevents,andsubjectsinonegrouparemorelikelyto
remembertheeventthanthoseintheothergroup.
Forexamplepeopletakeaspirincommonlyandformanyreasons,
butpatientsdiagnosedashavingpepticulcerdiseasemayrecallthe
ingestionofaspiriningreateraccuracythanthosewithoutG1
problems.AlsopatientswhohavehadanMIaremorelikelytorecall
andremembercertainhabits(likeeatinghabit)withgreateraccuracy
thanthosewhohavenothadanMI.

812.Trueaboutstandardizationareall
except?
a)Mostcommonlyusedforagedifferences
b)Directstandardizationisusedwhenpopulationislarge
c)Agespecificratesarerequiredinindirectstandardization
d)Allarecorrect
CorrectAnswer-C
Ans.is'c'i.e.,Agespecificratesarerequiredinindirect
standardization[Ref:Park23"/ep.58]
Indirectstandardization
Whenthepopulationissmall(oroutcomeisrare)thenumberof
eventsobservedcanbesmall.
Inthatcircumstance,indirectstandardizedmethodscanbeusedto
produceastandardizedmortalityrate(SMR)orastandardized
incidencerate(SIR).
Inindirectstandardization,onecomputesthenumberofevents
(mortality)thatwouldhavebeenexpectediftheeventrates
(mortalityrate)fromthestandardpopulationhadappliedinthestudy
population,i.e.agespecificratesofstandardpopulationareapplied
tostudypopulation(oppositetodirectstandardization).
Studypopulationisusedtoprovideagespecificdeathrates.
Withineachagestratum,onemultipliestheagespecificrateof
standardpopulationbythenumberofpeopleinthestudypopulation
todeterminethenumberofcasesthatwouldhaveexpectedifthat
weretherateinthestudygroup
Theseexpectednumbersareaddedupacrossallagegroupsand
dividedintotheobservednumbertoyieldtheSMR.
Advantageofindirectstandardizationisthatagespecificratesof

studypopulationarenotrequired.

813.Standardizationismostimportantfor?
a)Sexdistribution
b)Agedistribution
c)Diseasedistribution
d)Noneoftheabove
CorrectAnswer-B
Ans.is'b'i.e.,Agedistribution[RefPark23ralep.58]
Standardizationismostcommonlyusedforage.
Astandardizateddeathrate(ASDR)isthebestmortalityindicator.
'Crudedeathrate'istobestandardizedforageforcomparisonof
twopopulation,asagecompositionisdifferent.

814.Standardizeddeathratesareused
becausehealth?
a)Forvalidcomparisomoftwogroupsofdifferenthealth
determinants
b)Calculationsaremoreaccurate
c)Toavoidselectionbias
d)Alloftheabove
CorrectAnswer-A
Ans.is'a'i.e.,Forvalidcomparisonoftwogroupsofdifferent
healthdeterminants[RefPark23rd/ep.58]
Standardization(oradjustment)ofratesisusedtounablethevalid
comparisonofgroupsthatdifferregardinganimportanthealth
determinant(mostcommonlyage).Forexample,ifwewantto
comparethedeathratesoftwodifferentpopulationswithdifferent
agecomposition,thecrudedeathrateisnotrightyardstickbecause
ratesareonlycomparableifthepopulationsuponwhichtheyare
basedarecomparable.

815.Trueaboutcohortstudy
a)Descriptivestudy
b)Incidencestudy
c)Proceedsfromeffecttocause
d)Allarecorrect
CorrectAnswer-B
Acohortstudyisatypeofanalyticobservationalstudy
Cohortstudyproceedsforwardfromcausetoeffect,i.e.,thedisease
hasnotoccurredinsubjects(Incontrasttocase-controlstudywhich
proceedsbackwardfromeffecttocause).
ItisalsoknownasProspectivestudy,longitudinalstudy,Incidence
study,forward-lookingstudy.
Thefeaturesofcohortstudiesare:
Thecohortsareidentifiedbeforetheappearanceofthedisease
underinvestigation.
Thestudygroups,sodefined,areobservedoversometimeto
determinethefrequencyofdiseaseamongthem.
Thestudyproceedsforwardfromcausetoeffect.

816.Allaretrueaboutnaturalexperiments,
except?
a)Researcherhasnocontrolovertheallocationofsubjects
b)Jameslindexperimentisanexample
c)IncludesRandomizedcontrolledtrials[RCTs]
d)Allarecorrect
CorrectAnswer-C
Ans.is'c'i.e.,IncludesRandomizedcontrolledtrials
[RCTs][RefModernepidemiologyp.397]
Naturalexperimentsarethoseinwhichexposuretotheeventor
interventionofinteresthasnotbeenmanipulatedbyresearcher.The
individualsexposedtotheexperimentalandcontrolconditionsare
determinedbynatureorbyotherfactorsoutsidethecontrolofthe
investigators.
Whenanaturallyoccurringeventorsituationisexploitedbya
researchertohelpansweraresearchquestion,itiscalledanatural
experiment.Theresearcherhaslittleornocontroloverthesituation
thatisbeingobserved.
AgoodexampleofnaturalexperimentisonebyJamesLindin1747
onthepreventionofscurvyamongsailors.Hecomparedtheeffects
ofdifferentacidicsubstances,rangingfromvinegartocidr,ongrups
ofafflictedsailors,andfoundthatthegroupwhofromvinegartocidr,
ongroupsofafflictedsailors,andfoundthatthegroupwhowere
givenorangesandlemonshadlargelyrecoveredfromscurvyafter6
days.
OtherimportantexampleisJohnSnow'snaturalexperimenton
choleralinkedwithcontaminatedwater.
RCTisnotnaturalexperimentasresearcherallocatetheindividuals
instudyandcontrolgroupbyrandomization.

instudyandcontrolgroupbyrandomization.

817.Inastudyapatientdoesnotknowthe
natureofdrug[whetheraplaceboor
curativedrug]heistaking.The
researcherknowsthedrugtypetobe
giventotheindividualsinstudy.Types
ofblindinginthisstudyis?

a)Single
b)Double
c)Triple
d)Combineddouble/triple
CorrectAnswer-A
Ans.is'a'i.e.,Single[RefPark24'1*p.78]
Single
Studysubjectsarenotawareof
blinding
thetreatmenttheyarereceiving
Studysubjectsaswellas
Double
investigatorarenotawareofthe
blinding
treatmentstudysubjectsare
Tripleblinding receiving
Studysubjects,investigatoras
wellasanalyzerarenotawareof
thetreatmcstudysubjectsare
receiving

818.WhichPassiveimmunityisalsoprovidedthroughcolostrumandbreast
milk?
a)IgG
b)IgA
c)IgE
d)IgM
CorrectAnswer-B
Ans.B.IgA
Passiveimmunityisalsoprovidedthroughcolostrumandbreast
milk,
whichcontainIgAantibodiesthataretransferredtothegut
oftheinfant,providinglocalprotectionagainstdisease-
causingbacteriaandvirusesuntilthenewborncansynthesizeits
ownantibodies.ProtectionmediatedbyIgAisdependentonthe
lengthoftimethataninfantisbreastfed,whichisoneofthereasons
theWorldHealthOrganizationrecommendsbreastfeedingforat
leastthefirsttwoyearsoflife.

819.Midyearpopulationisestimatedon?
a)1stMarch
b)1stJuly
c)1stApril
d)15thAugust
CorrectAnswer-B
Ans.is'b'i.e.,1StJuly[RefPark22"/ep.58&21"/ep.52]
Denominatorincrudedeathrateismidyearpopulation,whichis
estimatedonfirstofJulyofanyear.

820.Meningococcalvaccinecontains?
a)50mcgofpolysccharideofeachstrain
b)100mcgofpolysccharideofeachstrain
c)1000mcgofpolysccharideofeachstrain
d)5000mcgofpolysccharideofeachstrain
CorrectAnswer-A
Ans.is'a'i.e.,50mcgofpolysaccharideofeachstrain[Ref
Park24th/ep.176]
VaccinesareavailableforgroupA,C,YandW-125.Thereisno
groupBvaccineavailableatpresent.Vaccinesarepreparedfrom
capsularpolysaccharide.
Bivalent(A,C),trivalent(A,C,W135),andtetravalent(A,C,W135,
Y)vaccinesareavailable.
Thevaccinescontain50mcgofpolysaccharideofeachindividual
strain.

821.Howmanydosesofmonovalent
meningococcal'C'vaccineisgivenin
Infants?

a)One
b)Two
c)Three
d)Four
CorrectAnswer-B
Ans.is'b'i.e.,Two[RefPark24th/ep.176]
MonovalentMenAconjugatevaccineshouldbegivenasasingle
dosetoindividuals1-29yearsofage.
FormonovalentMenCconjugatevaccine,onesingleintramuscular
doseisrecommendedforchildrenaged>12months,teenagersand
adults.
Children2-11monthsofagerequire2doseadministrationatan
intervalofatleast2monthsandaboosterabout1yearthereather.
Quadrivalentvaccinesareadministeredasasingledoseto
individualsaged2years.

822.Secondaryattackrateofmumps:?
a)75%
b)85%
c)95%
d)<50%
CorrectAnswer-B
Ans.is'b'i.e.,85%[RefPark23'/ep.147&22"d/ep.139-
140]
SARofsomeimportantinfectiousdiseases

Measles
80%
Rubella
90-95%
Chickenpox 90%
Pertussis

90%
Mumps
86%

823.IsolationperiodofTB?
a)2daysaftertreatment
b)1weekaftertreatment
c)2weeksaftertreatment
d)3weeksaftertreatment
CorrectAnswer-D
Ans.is'd'i.e.,3weeksaftertreatment[RefPark24th/ep.129]
Periodofisolationrecommended
Tuberculosis(sputum+)2weeksadult,6weekspaediatric.

824.Sensitivityofascreeningtesttellsabout
a)Percentageofdiseasepeopleamongthosewithapositivetest
b)Percentageofdiseasepeopleamongthosewithanegativetest
c)Percentageofhealthypeopleamongthosewithanegativetest
d)Percentageofhealthypeopleamongthosewithapositivetest
CorrectAnswer-A
Ans.is'a'i.e.,Percentageofdiseasepeopleamongthosewith
apositivetest

Sensitivity:Abilityofascreeningtesttoidentifycorrectlyallthose
whohavethedisease(Cases).
Specificity:Abilityofascreeningtesttoidentifycorrectlyallthose
whodon'thavethedisease(healthy).
Positivepredictivevalue(PPV):Abilityofascreeningtesttoidentify
correctlyallthosewhohavethedisease,outofallthosewhotest
positiveonascreeningtest.
Negativepredictivevalue(NPV):Abilityofascreeningtesttoidentify
correctlyallthosewhodon'thavethedisease,outofallthosewho
testnegativeonascreeningtest.
Whenascreeningtestisusedtodiagnoseadisease,thetest
outcomecanbepositive(diseased)ornegative(healthy),while
theactualhealthstatusofthepersonmaybedifferent.Inthat
setting:
TruepositiveDiseasedpeoplecorrectlydiagnosedasdiseased.
FalsepositiveHealthypeoplewronglyidentifiedasdiseased.
TruenegativeHealthypeoplecorrectlyidentifiedashealthy.
FalsenegativeDiseasedpeoplewronglyidentifiedashealthy.

825.Ifeffectivetreatmentforadiseaseis
introducedinacommunity,whatwillbe
theeffectonincidence[I]andprevalence
[P]?

a)NochangeinP&I
b)BothP&Iwilldecrease
c)Pwilldecrease&Iwillincrease
d)Pwilldecrease&Iwillremainsthesame
CorrectAnswer-D
Ans.is'd'i.e.,Pwilldecrease&Iwillremainsthesame[RefPark
24th/ep.66&23rdiep.62]
Neweffectivetreatmentwillcurethepatientandtherebydecrease
thedurationofdisease.
So,neweffectivetreatmentwillaffectthedurationofdisease.
Incidencemeasurestherateatwhichnewcasesareoccurringina
population,Itisnotinfluencedbyduration.So,neweffective
treatmentwillhavenoeffectonincidence.
Ontheotherhand,prevalencewilldecreaseduetodecreasein
durationofdisease.

826.Ascreeningtesthassensitivityof90%
andspecificityof99%.Theprevalenceof
diseaseunderinvestigationis5per1000
population.WhatisthePPVofthegiven
screeningtest?

a)10
b)70
c)33
d)99
CorrectAnswer-C
Ans.is'c'i.e.,33[RefPark23rd/ep.139&22"dlep.132;
Hopefieldbiostatistics4thlep.49]
Positivepredictivevalueisrelatedtosensitivityspecificityand
prevalence.
ThisrelationshipisrepresentedbyBaye'stheorem:-

827.BestindicatorforspreadofTBina
community?
a)Annualinfectionrate
b)Prevalenceofinfection
c)Caserate
d)Incidenceofnewcases
CorrectAnswer-A
Ans.is'a'i.e.,Annualinfectionrate[RefPark24thlep.191-195&
23'/ep.177]
Thefollowingepidemiological-indicesareusedintuberculosis
problemmeasurementandprogrammestrategy
1.Prevalenceofinfection
Itisthepercentageofindividualsshowingpositivetuberculintest.
2.Incidenceofinfection(Annualinfectionrate)
Itisthepercentageofpopulationunderstudywhowillbenewly
infectedbyM.tuberculosisamongthenon-infectedofthepreceding
surveyduringthecourseofoneyear.
Itexpressestheattackingforceoftuberculosisandisalsoknownas
tuberculinconversionindexi.e.percentegeofnewpeoplebecoming
tuberculinpositive.
Indevelopingcountries,every1%ofannualinfectionrateissaidto
correspondto50newcases
ofsmearpositivepulmonaryTB,per
year,for100000generalpopulation.
ItisthebestindicatorforevalutionofTBproblemanditstrend.
InIndia,annualinfectionrate/tuberculinconversionindexis1.7%.
3.Prevalenceofdiseaseorcaserate
Itisthepercentageofindividualswhosesputumispositivefor
tuberclebacilli
onmicroscopicexamination.

Itisthebestavailablepracticalindextoestimatethenumberof
infectiouscasesorcaseloadinacommunity.
4.Incidenceofnewcases
ItisthepercentageofnewTBcases(confirmedbybacteriological
examination)per1000populationoccurringduringoneyear.
5.Prevalenceofsuspectedcases
ThisisbasedonX-rayexaminationofchest.
6.Prevalenceofdrugresistantcases
Itistheprevalenceofpatientexcretingtuberclebacilliresistantto
anti-tuberculardrugs.
7.Mortalityrate
Thenumberofdeathsfromtuberculosiseveryyearper1,000
population.

828.FordiagnosisofTB,Sputummicroscopy
has?
a)Highsensitivity&highspecificity
b)Highsensitivity&lowspecificity
c)Lowsensitivity&highspecificity
d)Lowsensitivity&lowspecificity
CorrectAnswer-D
Ans.is'd'i.e.,Lowsensitivity&lowspecificity
MostrapidmethodofdiagnosisforTBSputummicroscopy.
But,sputummicroscopyhaslowsensitivityandspecificity.
MostreliablemethodfordiagnosisofTB->Cultureoftubercular
bacilli.

829.Varicellazostervirusinfectionismore
likelytooccurinwhichofthefollowing
month?

a)March
b)August
c)October
d)November
CorrectAnswer-A
Ans.is'a'i.e.,March[Ref:CECILVol.1,p.1840]
"Varicellaoccursmostcommonlyduringthelatewinterandspring
months,thepeakbeingaboutinMarch"

830.Mostcommoninfluenzaviruscausing
disease?
a)TypeA
b)TypeB
c)TypeC
d)TypeD
CorrectAnswer-A
Ans.is'a'i.e.,TypeA[RefPark24th/ep.163-166]
Therearethreeviralsubtypes:TypeA(causesallpandemicsand
mostepidemics);typeB;andtypeC(notcirculatingcurrently).
Currentlytheinfluenzavirusescirculatingintheworldare:H,N,of
typeA(causesswineflu);H2N2oftypeA;H3N2oftypeA;H5N,of
typeA(causesbirdfluoravianinfluenza);117N9oftypeA(caused
epidemicofavianinfluenzainChinain2013);andtypeB.
Influenzashowscyclictrendwithepidemicoccuringevery2-3years
incaseofinfluenza-Aandevery4-7yearsincaseofinfluenza-B.
Pandemicsarecausedbyonlyinfluenza-Aevery10-15years.
Influenzaaffectsallagesandbothsexes.
Sourceofinfectionofinfluenzaisaclinicalcaseorsubclinicalcase.
Majorreservoirofinfluenzavirusexistsinanimalandbirds.

831.MammalianreservoirforR.prowazekii?
a)Rodents
b)Dog
c)Cattle
d)Humans
CorrectAnswer-D
Ans.is'd'i.e.,Humans[RefPark24th/ep.316,319]
Insect
Mammalian
Disease
Agent
vector
reservoir
Typhusgroup
R.prowazekii
a)Epidemictyphus
Louse
Humans
R.typhi
b)Murinetyphus
Flea
Rodents
R.
(Endemictyphus)
Mite
Rodents
Tsutsugamushi
c)Scrubtyphus

832.Rotavirusvaccinedosesshouldnotbeinitiatedbeyondwhichagetoprevent
complications:
a)6weeks
b)10weeks
c)12weeks
d)32weeks
CorrectAnswer-C
Twoliveattenuatedoralrotavirusvaccineshavebeenlicensedforuserotateqandrotarix.
Thefirstdoseofthesevaccinesshouldbegivennolaterthan12weeks.
Incaseofrotarix,vaccinationmustbecompletedby24weeks.
Incaseofrotate,lastdoseofvaccineshouldbeadministeredby32weeks.
Riskofintussusceptionsincreaseiffirstdoseisadministeredbeyond12weeks.
Ref:Park21stedition,page205.

833.Minimumacceptedintervalbetweentwo
dosesofDPTvaccine?
a)2weeks
b)4weeks
c)6weeks
d)8weeks
CorrectAnswer-B
Ans.is'b'i.e.,4weeks[RefPark24th/ep.172]
Totalthreedosesaregiveninprimarlyimmunizationwithaninterval
of4weeksbetweenthreedoses.Firstboosterisgivenat16-24
monthswithsecondboosterat5-6years.
DPT,6weeksofage
DPT,10weeksofage
DPT,14weeksofage
DPTim,,16-24monthsofage
terDPTB5yearsofage

834.Post-exposureprophylaxisisindicated
in?
a)HBV
b)Rabies
c)Diphtheria
d)All
CorrectAnswer-D
Ans.is'a'i.e.,HBV;'b'i.e.,Rabies;'c'i.e.,Diphtheria
Readthequestioncarefully,examinarisaskingaboutpostexposure
prophylaxis(notpost-exposureimmunization).oInprevious
explanation,Ihaveexplainedthediseasesforwhichpost-exposure
prophylaxisisdonebyimmunization
(vaccineorimmunoglobulinorboth).
Insomediseasespost-exposureprophylaxisisdonebydrugs,
i.e.,post-exposurechemoprophylaxis:-
HIVAntiretroviraltherapy
HerpesFamciclovir
Diphtheria
Miningococcalmeningitis

835.Healthycarrierisseenin?
a)Measles
b)Rubella
c)Meningococcalmeningits
d)Influanza
CorrectAnswer-C
Ans.is'c'i.e.,Meningococcalmeningitis[RefPark24th/ep.103-
106&23"1/ep.95,96]
Temporarycarrier
Temporarycarriersshedtheinfectiousagentforshortperiodoftime.
Thiscategorymayinclude
Incubatory
Measles,mumps,polio,pertussis
carriers:
(whoopingcough),influenza,diphtheria,
HepatitisB.
Convalescent Typhoid,cholera,diphtheria,Pertussis
carriers:
(whoopingcough),dysentery.
Healthy
Polio,Cholera,meningococcal
carriers:
meningitis,Salmonellosis,diphtheria
Chroniccarrier
Chorniccarriersexcretetheinfectiousagentforindefiniteperiod.
Examples:Typhoid,hepatitisB,dysentery,malaria,gonorrhoea,
cerebrospinalmenigitis,Diphtheria.

836.Infectivityofconvalescentcarrierof
choleralastsfor?
a)1-5days
b)1-2weeks
c)2-3weeks
d)4-5weeks
CorrectAnswer-C
Ans.is'c'i.e.,2-3weeks
Therearefollowingtypesofcarrierincholera:
Incubatory:Shedvibriosonlyinthebriefincubationperiodof1-5
days.
Convalescent:Shedvibriosfor2-3weeks.
Healthyorcontactcarrier:Hashadsubclinicalinfectionandshed
vibriosforlessthan10days.
Chroniccarriers:Canshedvibriosformonthsoryearsandmay
havepersistentinfectioningallbladder

837.Diagnosisoffilariasisisconfirmedmost
commonly?
a)Clinicalfeatures
b)Detectionofmicrofilariae
c)PCR
d)Seralogicaltest
CorrectAnswer-B
Ans.is'b'i.e.,Detectionofmicrofilariae[RefEssentialsof
clinicalmicrobiology-188]
Detectionofmicrofilariae(MO
Themostcommonlyusedmethodfordiagnosisoffilariasisis
detectionofmicrofilariaeinbloodsmear.Thebloodcollectionshould
bedoneatnightbecauseofnocturnalperiodicityofmicrofilariae
ThemicrofilariaeofW.bancroftiandB.malayioccuringinIndia
displayanocturnalperiodicity,i.e.,theyappearinlargenumberat
nightandretreatfromthebloodstreamduringtheday.Thisisa
biologicaladaptationtothenocturnalbitinghabitsofvector
mosquitoes.Themaximumdensityofmicrofilariaeinbloodis
reportedbetween10pmand2am.Whenthesleepinghabitsofthe
hostarealtered,areversalinperiodicityhasbeenobserved.
Thickfilmismostcommonlyusedmethodfordetectionof
microfilariae.
Concentrationtechniquebymembranefilterconcentration(MFC)
methodisthemostsensitivemethodwhichcandetectlowdensityof
microfilariaeinblood.

838.Maximumspreadofmalariaoccursin
whichmonth?
a)March-April
b)January-February
c)April-May
d)September-October
CorrectAnswer-D
Ans.is'd'i.e.,September-October[RefPark24th/ep.272-276]
Malariaisaprotozoaldiseasecausedbyinfectionwithparasiteof
genusplasmodiumandtransmittedtomanbycertainspeciesof
infectedfemaleAnophelinemosquito.
Definitivehost-Mosquito(sexuallifecycle).
IntermediatehostMan(Asexualcycle).
SeasonInIndiamaximumprevalenceisfromJulytoNovember.
Reservoir-Withpossibleexceptionofchimpanzeesintropical
Africa,whichmaycarrytheinfectionwithP.malariae,noother
animalreservoirisknowntoexist.Manharbouringsexualforms
(gametocytes)istheonlyreservoir.
Extrinsicincubationperiod(inmosquito)10to20days.Itisthe
periodoftimerequiredforthedevelopmentofparasitefrom
gametocytetosporozoitestage(infectivestagetoman)inthebody
ofmosquito.

839.Nottrueaboutdiphtheriavaccine?
a)Canbegivenaspentavalentvaccine
b)ForinfantDPTisthevaccineofchoice
c)Firstdoseisgivenat6weeksofage
d)allofthese
CorrectAnswer-D
Ans.is'DAllofthese,[RefPark24thiep.172]
Diphtheriavaccineisatoxoid.
ItisgivenastrivalentvaccineDPT-Preparationofchoicefor
immunizationofinfant
Firstdoseisgivenat6weeksofage
Pentavalentvaccineprovidesprotectiontoachildfrom5life
threateningdisease-diphtheria,pertussis,tetanus,hepatitisBand
haemophilusinfluenzatypeb(Hib).Givingpentavalentvaccine
reducesthenumberofprickstoachild.Whenused,itreplaces
HepatitisBandDPTprimaryvaccinationscheduleat6,10and14
weeksintheimmunizationprogramme,exceptthatthebirthdoseof
hepatitisBandboosterdosesofDPTarecontinued.

840.Whichvaccineisusedtopreventdeath
frompneumoniainchildren?
a)Measlesvaccine
b)Rubellavaccine
c)Chickenpoxvaccine
d)Influenzaviralvaccine
CorrectAnswer-A
Ans.is'a'i.e.,Measlesvaccine[RefPark24th/ep.182]
Threevaccineshavepotentialofreducingdeathfrom
pneumonia:-
Measlesvaccine
HIBvaccine(HaemophilusinfluenzeetypeB)
Pneumococcalvaccine
Thesevaccinesworktoreducetheincidenceofbacterial
pneumonia.

841.Antiboioticofchoiceforsevere
pneumoniain1yearallchild?
a)Cotrimoxazole
b)Ciprofloxacin
c)Benzylpenicillin
d)Tetracycline
CorrectAnswer-C
Ans.is'c'i.e.,Benzylpenicillin[RefPark24`h/ep.181,182]
Antibioticsofchoicefortreatmentofacuterespiratory
infectionsinchildrenaged2months-5years

1. Nopneumonia(Coughorcold)Noantibiotic
2. Pneumonia(notsevere)-scotrimoxazole
3. Severepneumonia-Benzylpenicillinorampicillinor
chloramphenicol
4. Veryseveredisease4Chloramphenicol
Antibioticsofchoicefortreatmentofacuterespiratory
infectionsininfantsyoungerthan2months

1. Nopneumonia(Coughorcold)4Noantibiotic
2. Allpneumonia(severeornotsevere)4(Benzylpenicillinor
ampicillin)plusgentamycin.

842. Reconstitutedmeaslesvaccineshould
beusedwithin-
a)1hour
b)3hour
c)6hour
d)12hour
CorrectAnswer-A
Ans.is'a'i.e.,1hour
"Thereconstitutedvaccineshouldbekeptoniceandusedwithin
onehour".--Park
Measlesvaccine:
Type:Liveattenuated,lyophilized(Freezedried)vaccine,
Measlesvaccineisliveattenuated,lypholized(Freezdried)vaccine.
StrainsofvirususedtopreparevaccineareEdmonstonZagreb
strain(mostcommon),SchwartzstrainandMoratenstrain.
Itisgivensubcutaneouslyintomiddleone-thirdofantero-lateral
aspectofthigh.
Itisgivenattheageof9months(agecanbelowereredto6
months
inepidemics&malnutrition)andisrepeatedat16-24
monthsofage.
Ithasprotectiveefficacy(sero-conversion)of95%.Vaccination
providelifelongimmunity.
Incubationperiodofvaccineinducedmeaslesis7days.
Inpost-exposureprophylaxis,measlesvaccineshouldbegiven
within2-3daysofexposure.Incubationperiodofmeaslesvirusis10
days.Incubationperiodofliveattenuatedmeaslesvirusoflive
vaccineis7days.Thus,ifthevaccineisgivenwithin2-3daysof
exposure,thereplicationofvaccinevirustakespreferenceover
replicationofwildvirus.

Diluentusedformeaslesvaccinereconstitutionisdistilledwateror
sterilewater.
Reconstitutedvaccineshouldbeusedwithin1hour.
Usualtemperatureforcoldchainstorageis+2to+8?C.

843.Colorofboxcontainingdrugsfor
treatmentofcategoryIofTB-
a)Red
b)Blue
c)Yellow
d)Green
CorrectAnswer-A
Ans.is'a'i.e.,Red[RefPark24th/ep.199]

844.MajorreservoirofKFD?
a)Human
b)Squirrels
c)Cattle
d)Monkey
CorrectAnswer-B
Ans.is'b'i.e.,Squirrels
KFD,alsoknownas'monkeydisease'isahemorrhagicfever
causedbyflavivirusbelongingtogroup-Barbovirus.Diseaseis
commoninfourdistrictsofKarnataka:Shimoga,NorthKannda,
SouthKanadaandChikamagaloor.KFDwasfirstrecognizedin
1957inShimogadistrictofKarnataka.
MajorvectorfortransmissionofKFDishardtick(Haemophysalis
spinigeraandH.turtura).But,softtickcanalsotransmitthedisease,
especiallyoutsidetheIndia.
Ratsandsquirrelsarethemajorreservoir.Monkeyactsas
amplifyinghostandmanisincidentaldead-endhost,thereisno
man-to-mantransmission.

845.Thresholdlevelofherdimmunityfor
Pertussisis?
a)80%
b)70%
c)90%
d)50%
CorrectAnswer-C
Ans.is'c'i.e.,90%
Herdimmunity
Itisthelevelofresistanceofacommunityorgroupofpeopletoa
particulardisease.
Itoccurswhenthevaccinationofaportionofthepopulation(or
herd)providesprotectiontounprotected(non?vaccinated)
individuals.
Advantageofherdimmunity
Itisnotnecessarytoachieve100%immunizationtocontrola
diseasebyprovidingherdimmunity.
Whenacertainpercentageofpopulation,isvaccinated,thespread
ofdiseaseiseffectivelystopped.
Thiscriticalpercentageisreferredtoasherdimmunitythreshold.
DiseaseHerdimmunitythreshod
Diphtheria85%
Measles83-94%
Mumps75-86%
Pertussis92-94%
Polio80-86%
Rubella80-85%
Smallpox83-85%


846.Whatisthecommonestformofplague?
a)Bubonicplague
b)Pneumonicplague
c)Septicaemicplague
d)Hemorrhagicplague
CorrectAnswer-A
Thecommonestformofplagueisbubonicplague.
Pneumonicplagueoccursinlessthan5%ofpatients.
Septicaemicplagueoccursrarelyexceptforaccidentallaboratoryinfections.
Ref:Park21stedition,page270.

847.MostcommonsourceofDiphtheria
a)Case
b)Carrier
c)Both
d)None
CorrectAnswer-B
Ans.is'b'i.e.,Carrier
Diphtheriaisanacuteinfectiousdiseasecausedbytoxigenicstrains
ofcorynebacteriumdiphtheriae.
Sourceofinfectioncasesorcarriers;carriersarecommonsources
ofinfection,theirratioisestimatedtobe95carriersfor5clinical
cases.
Infectiveperiod14-28daysfromtheonsetofdisease.
Agegroup1to5years
SexBothsexes
Incubationperiod2-6days

848.WHOVISION2020initiativeincludes?
a)Cornealulcer
b)Trachomablindness
c)Diabeticretinopathy
d)VernalkeratoConjunctivitis
CorrectAnswer-B
Ans.is'b'i.e.,Trachomablindness
Aftertherealizationthatunlessblindnesscontroleffortsare
intensified,theprevalenceofblindnesswilldoubleby2020AD,the
WHOalongwithanInternationalPartnershipcommitteelaunched
theVision2020Initiativein1995.
Thediseasesidentifiedforglobaleliminationinclude:?
1. Cataractblindness
2. Trachomablindnessandtransmission
3. Onchocerciasis
4. Avoidablecausesofchildhoodblindness
5. Refractiveerrorsandlowvision
Indianvisionof2020includesthefollowingsevendiseases.
1. Cataractblindness
2. Glaucoma
3. Trachomablindnessandtransmission
4. Diabeticretinopathy
5. Childhoodblindness
6. Cornealblindness
7. Refractiveerrorsandlowvision

849.PrevalenceofRHDinIndiain5-15years
agegroup?
a)1-2per1000
b)5-7per1000
c)10-12per1000
d)13-15per1000
CorrectAnswer-B
Ans.is'b'i.e.,5-7per1000[RefPark24th/ep.397]
InIndia,RHDisprevalentintherangeof5-7perthousandin5-15
yearsagegroupandthereareabout1millionRHDcasesinIndia.
RHDconstitutes20-30%ofhospitaladmissionsduetoCVDinIndia.
Streptococcalinfectionsareverycommonespeciallyinchildren
livinginunder-privilegedconditions,andRFisreportedtooccurin
1-3percentofthoseinfections.

850.JaiVigyanMissionmodeprojectinIndia
isfor?
a)Measles
b)TB
c)Rheumaticfever
d)STD
CorrectAnswer-C
Ans.is'c'i.e.,Rheumaticfever[RefPark24tVep.397]
JaiVigyanMissionModeprojectonCommunityControlofRF/RHD
inIndiaisbeingcarriedoutwithfourmaincomponents,viz.tostudy
theepidemiologyofstreptococcalsorethroats,establishregistries
forRFandRHD,vaccinedevelopmentforstreptococcalinfection
andconductingadvancedstudiesonpathologicalaspectsofRFand
RHD.

851.WhichdoesnothaveLivebirthsas
denominator?
a)Infantmortalityrate
b)Neonatalmortalityrate
c)Childmortalityrate
d)Childdeathrate
CorrectAnswer-D
Ans.is'd'i.e.,Childdeathrate[RefPark24thlep.608-612]
CDR=No.ofdeathsofchildrenaged1-4years/TotalNo.of
childrenaged1-4years.

852.Dualrecordsystemisusefulfor
estimationof?
a)Literacy
b)Fertility
c)Populationdensity
d)Sexratio
CorrectAnswer-B
Ans.is'b'i.e.,Fertility[RefPark24th/ep.878]
Sampleregistrationsystem(SRS),initiatedinmid1960sprovides
reliableestimatesofbirth(fertility)anddeath(mortality)ratesat
stateandnationallevels.
Itisadualrecordsystem,consistingofcontinuousenumerationof
birthanddeathbyanenumeratorandanindependentsurveyevery
6monthsbyaninvestigatorsupervisor.
MainobjectiveofSRSistoprovidereliableestimatesofbirthrate,
deathrateandinfantmortalityrateatthenaturaldivisionlevelfor
ruralareasandatstatelevelforurbanareas.
Infantmortalityrateisthedecisiveindicatorforestimationofsample
sizeatnaturaldivision.
SampledesignforSRSisuni-stagestratifiedsimplerandom
sample.SRSnowcoversentirecountry.

853.Whichofthefollowingisfalseabout
intra-uterinedevices(IUDs)?
a)MultiloadCu-375isathirdgenerationintra-uterinedevice(IUD)
b)Copperdevicesareeffectiveaspost-coitalcontraceptives
c)LNG-20(Mirena)hasaneffectivelifeof5years
d)PregnancyratesofLippesLoopandTCu-200aresimilar
CorrectAnswer-A
MultioadCu-375isanewercopperintra-uterinedevice(IUD).
ThecopperdevicescomprisethesecondgenerationIUDs.
Thenon-medicatedorinertdevicesarethefirstgenerationIUDsand
thehormone-releasingdevicesarethethirdgenerationIUDs
Ref:Park'sTextbookOfPreventiveAndSocialMedicine,ByK.
Park,19thEdition,Pages393-395.

854.Perinatalmortalityrateincludewhichof
thefollowing?
a)Abortions+Stillbirthearlyneonataldeaths
b)Stillbirth+earlyneonataldeaths
c)Abortions+earlyneonataldeaths
d)Deathsupto42dysafterbirth
CorrectAnswer-B
Ans.isb'i.e.,Stillbirth+earlyneonataldeath

855.Poorman'sironsourceis?
a)Almond
b)Grapes
c)Soya
d)Jaggery
CorrectAnswer-D
Ans.is'd'i.e.,Jaggery[RefPark24th/ep.661&23rd/ep.623]
Therearetwomajorformsofiron:?
1)Haem-iron
ItisbetterabsorbedbutislessimportantsourceofironinIndian
diet.Itismainlyfoundinfoodsofanimalorigin,e.g.liver,meat,
poultryandfish.
2)Non-haemiron
ItispoorlyabsorbedbutistheimportantsourceofironinIndiandiet.
Itismainlyfoundinfoodsofvegetableorigin,e.g.cereales,green
leafyvetetables,legumes,nuts,oilseeds,juggery,anddriedfruits.
Amongdryfruits,cashewnuthasmaximumiron(9%)followedby
almonds(7%),andpistachos(7%).
Ironofmilkislowinallmammalianspecies.
Jaggeryisconsideredaspoorman'sironsource.Itcontainsagood
amountofironalongwithvitamin-A.

856.An70kgfarmerisconsuming56grams
proteins,275gramscarbohydrateand60
gramslipids.Heconsuming?

a)Lesscalories
b)Morecalories
c)Adequatecalories
d)Connotbecommented
CorrectAnswer-A
Ans.is'a'i.e.,Lesscalories[RefReadbelow]
Proteinandcarbohydratebothprovide4calpergramandfat
provides9calpergram.
Thusthefarmeristaking(calorKcalperday)=(56x4)+(275x4)
+(60x9)=1864

857.Forevery100kilocalories,vitaminB,
requiredis-
a)0.05mg
b)0.5mg
c)5.0mg
d)1.0gn
CorrectAnswer-A
Ans.is`a'i.e.,0.05rug
Thiamineisrequired0.5mgper1000K.calofenergyintake,
i.e.,0.05mgper100Kcal.


858.Gomezclassificationisbasedon?
a)Weightretardation
b)Heightretardation
c)Midarmcircumference
d)Stunting
CorrectAnswer-A
Ans.is'a'i.e.,Weightretardation

859.Kanawatiindexisusedfor?
a)Airpopulation
b)PEM
c)Obesitydefinition
d)Infectivity
CorrectAnswer-B
Ans.is'b'i.e.,PEM
Kanawatiindexisusedtoclassifyproteinenergymalnutrition(PEM).

860.Humanmilkwithrespecttocowmilkhas
-
a)Lessfat
b)Lessprotein
c)Lesscarbohydrate
d)aandb
CorrectAnswer-D
Ans.is'a'i.e.,Lessfat;`b'Lessprotein
Humanmilkhaslessfat,lessprotein,morecarbohydratesandless
calcium,incomparisontocowmilk.
Humanmilkhaslesssodium,potassiumandchloride.However,
thesesubstancesareincorrectamount(thoughless)inhumanmilk.
Salts(meq/L) Cow'smilk
Humanmilk
Sodium
25(toomuch) 6.5(correctamount)
Chloride
29(toomuch) 12(correctamount)
Potassium
35(toomuch) 14(correctamount)

861.Mostcommonnutritionalproblemin
India?
a)Lowbirthweight
b)Fluorosis
c)Irandeficiencyanemia
d)VitaminAdeficiency
CorrectAnswer-C
Ans.is'c'i.e.,Irondeficiencyanemia[RefPark24thlep.677,661-
667]
ThemajornutritionalprobleminIndiaare:

1. Lowbirthweight
2. Iodinedeficiencydisorders(IDD)
3. Proteinenergymalnutrition
4. Endemicfluorosis
5. VitaminAdeficiency
6. Lathyrism
7. Irondeficiencyanemia
Irondeficiencyanemiaisthemostwidspreadamongthese.

862.Pulsesaredeficientin?
a)Methionirfe
b)Lysine
c)Threonine
d)All
CorrectAnswer-A
Ans.is'a'i.e.,Methionine[Ref:Park23'/ep.628&21"/ep.578]
Someaminoacidsaredeficientinaparticularfood,calledlimiting
aminoacids.Forexample,cerealsandwheataredeficientin
threonineandlysine,pulsesaremainlydeficientinmethionineand
cysteine,andmaizeisdeficientintryptophanandlysine.
Supplementaryactionofproteins:Iftwoormorefooditemseaten
together,theirproteinssupplementthedeficientaminoacidofeach
other.Forexamplecerealsaredeficientinthreonineandlysine,
whereaspulsesaredeficientinmethionineandcystein.Ifbothare
takentogether,theirproteinscomplementeachotherandprovidea
morebalancedandcompleteproteinintake.

863.DailyironrequirementinhealthyIndian
maleis-
a)35mg
b)17mg
c)10mg
d)5mg
CorrectAnswer-B
Ans.is'bi.e.,17mg
Iron
Recommended
Group
absorbed/day(mg) intake
Adultmale
0.84
17
Adultfemale
1.65
21
(mensturating)
35(extra14
Pregnantwoman
2.80(extra1.15)
mg/day)
Lactatingwoman(0-6
1.65
21
months)
Infant(6-12months)
0.7
S
Adolescentboys(13-15
1.6
32
years)
Adolescentgirls(13-15
1.36
27
years)

864.Notaprimaryairpollutant?
a)SO2
b)CO,
c)Ozon
d)VOCs
CorrectAnswer-C
Ans.is'c'i.e.,Ozone[RefPark24tVep.770]
Airpollutionistheintroductionofchemicalsparticulatematter,or
biologicalmaterialintotheatmospherethatcauseharmor
discomforttohumansorotherlivingorganisms,ordamagesthe
naturalenvironment.Anairpollutantisknownasasubstanceinthe
airthatcancauseharmtohumansandthe-environment.

865.Numberofholesinmosquitonet[persq.
inch]?
a)50
b)150
c)100
d)200
CorrectAnswer-B
Ans.is'b'i.e.,150[RefPark24th/ep.810&23'/ep.773]
Thebestpatternofmosquitonetistherectangularnet.
Thereshouldnotbeasinglerentinthenet.
TheSizeofopeningsinthenetisofutmostimportance,thesize
shouldnotexceed0.0475inchinanydiameter.
Thenumberofholesinonesquareinchinusually150.

866.Inmalariacontrol,insecticideusedfor
insectisidetreatedbednets(ITBN)-
a)Deltamethrin
b)Malathion
c)Lindone
d)Fenitrothion
CorrectAnswer-A
Ans.is'a'i.e.,Deltamethrin
InsecticideTreatedBedNets(ITBN)Programme(esp.deltamethrin)
hasresultedinsignificantdeclineinmalariaincidenceandAPI
1. Averagedeclineinanophelinemosquitodensity-68%
2. Averagedeclineincuicinemosquitodensity-50%
3. ChemicalsusedinITBNProgramme:Syntheticpyretheroids
Deltamethrin:2.5%indosageof25mg/m2
1. Cyfluthrin:5%indosageof50mg/m2
2. Otherinsecticidesused:Permethrin,Lambdacyhalothrin,
Etofenprox,Cypermethrin
Effectivenessofpyrethroids:For6-12months(Retreatmentevery6
months)
Long-lastinginsecticidalmosquitonets(LLINs):Alsousepyrethroid
insecticides,andachemicalbinderthatallowsthenetstobe
washed>20times,allowingusefor>3years.

867.Testforcoliformcount?
a)Eijkmantest
b)Casoni'stest
c)Nitratetest
d)Ureasetest
CorrectAnswer-A
Ans.is'a'i.e.,Eijkmantest
Adleranalysingpresumptivecoliformcount,Ecolicountiscofirmed
byothertestslikeEijkmanttestandindoleproduction.

868.Notacontactpoison:
MAHE14
a)Pyrethrum
b)Parisgreen
c)Rotenone
d)Eucalyptusoil
CorrectAnswer-A
Ans.is.A.Pyrethrum

869.Whichofthefollowingisnotaindoorair
pollutant?
a)Carbonmonoxide
b)Nitrousoxide
c)Radon
d)Mercury
CorrectAnswer-D
Ans.is'di.e.,Mercury
Indoorairpollutionreferstochemical.biologicalandphysical
contaminationofindoorair,i.e.thepollutionofairwithinandaround
buildingandstructures.
Asmostofthepeoplework,study,eat,drinkandsleepinenclosed
environments(i.e.indoor)whereaircirculationmayberestricted-->
Mostpeoplespendlargeportionoftimeindoors,asmuchas80-
90%oftheirlives.Therefore,morepeoplesufferfromindoorair
pollutionthanoutdoorpollution.

BasicInformationonPollutantsandSources
ofIndoorAirPollution
Asbestos
BiologicalPollutants
CarbonMonoxide(CO)
Formaldehyde/PressedWoodProducts
Lead(Pb)
NitrogenDioxide(NO2)
Pesticides
Radon(Rn)
IndoorParticulateMatter
SecondhandSmoke/EnvironmentalTobaccoSmoke
Stoves,Heaters,FireplacesandChimneys
VolatileOrganicCompounds(VOCs)

870.Thesourceofendogenousradiationis
a)Radon
b)Potassium
c)Thorium
d)Uranium
CorrectAnswer-B
Ans.is'b'i.e..Potassium
oRadiationispartofman'senvironment.


871.RangeofflightofAedesmosquitois?
a)1km
b)Lessthan100m
c)400m
d)10kms
CorrectAnswer-B
Ans.is'b'i.e.,Lessthan100m[RefPark23'/ep.771&
22"d/ep.715]
Aedesdonotflyoverlongdistances;usuallylessthan100metres
(110yards).Anopheles3-5Kms
Culex11Kms
Aedes100m

872.Inpositivelyskeweddeviation?
a)Mean=Median=Mode
b)Mean>Medians>Mode
c)Mode>Median>Mean
d)Noneoftheabove
CorrectAnswer-B
Ans.is'b'i.e.,Mean>Medians>Mode

873.Chi-squaretestisfor?
a)StandarderrorofMean
b)StandarderrorofProportion
c)Standarderrorofdifferencebetween2Means
d)StandarderrorofdifferencebetweenProportions
CorrectAnswer-D
Ans.is'd'i.e.,StandarderrorofdifferencebetweenProportions
Chi-squaretestmeasuresthesignificanceofdifferencebetweentwo
proportionsbytesting,whethertheobservedfrequenciesdiffer
significantlyfromtheexpectedfrequencies.


874.Whichofthefollowingdefines
movementacrossscioeconomicstatus.
a)Socialequality
b)Socialupliftment
c)Socialmobility
d)Socialinsurance
CorrectAnswer-C
Ans.is'c'i.e.,SocialMobility[RefPark23'lep.688&22"d/ep.
639]
Socialmobilityisthedegreetowhichanindividual'sfamilyor
group'ssocialstatuscanchangethroughoutthecourseoftheirlife
throughasystemofsocialhierarechy,i.e.Socialmobilityrefersto
movementofindividuals/familiesacrossdifferentsocioeconomic
levels.

875.Allarenon-parametrictestsexcept-
a)Chi-squaretest
b)Signtest
c)Fisherexacttest
d)Studentt-test
CorrectAnswer-D
Ans.is'd'i.e.,Studentt-test

876.ThenumberofAnganwadiworkers
supervisedbyaMukhyasevikais:
a)10
b)15
c)25
d)30
CorrectAnswer-C
MukhyaSevikaisamiddlelevelsupervisor.Shesupervises20to25
Anganwadiworkers.Sheisrequiredtobeagraduateinsocialwork
orhomescienceorarelatedfield.Sheistrainedforthreemonths.
Ref:HealthpoliciesandprogrammesinIndia,D.K.Taneja11th
editionpage:312

877.Whichofthefollowinghasresponsibility
ofdatacollectionforactivemalaria
surveillanceatPHClevel?

a)DHO[DistrictHealthOfficer]
b)MPW[Multipurposeworker]
c)MO-PHC[MedicalOfficer-PHC]
d)DMO[DistrictMedicalOfficer]
CorrectAnswer-C
Ans.is'c'i.e.,MO-PHC[MedicalOfficer-PHC]
[RefPark24"/ep.433]
"TheMedicalOfficer-PHChastheoverallresponsibilityfor
surveillanceandlaboratoryservices,andalsosupervicesthe
spray".

878.Whichofthefollowingconditionsmust
befulfilledforaPHCtobecomeafirst
referralunit?

a)4-6beds
b)15workers
c)Emergencyobstericcare
d)Basiclaboratoryservices
CorrectAnswer-C
Ans.is'c'i.e.,Emergencyobstericcare[RefTextbookofIndian
Healthcare-728]
Criticaldeterminantsofafirstfeferralunit
24-hourdeliveryservicesincludingnormalandassisteddeliveries
Emergencyobstetriccareincludingsurgicalinterventionslike
caesareansectionsandothermedicalinterventions,New-borncare,
Emergencycareofsickchildren,Fullrangeoffamilyplanning
servicesincludinglaproscopicservices,Safeabortionservices,
TreatmentofSTI/RTI,Bloodstoragefacility,Essentiallaboratory
services,Referral(transport)services.

879.Acculturationis?
a)Traige
b)Culturalchangesduetosocialisation
c)Attitude
d)Belief
CorrectAnswer-B
Ans.is'b'i.e.,Culturalchangesduetosocialisation
Acculturationisaprocessofsocial,psychological,andcultural
changethatstemsfromthebalancingoftwocultureswhile
adaptingtotheprevailingcultureofthesociety.Acculturation
isaprocessinwhichanindividualadopts,acquiresand
adjuststoanewculturalenvironment.


880.TheICDSschemeissponsoredby
a)Ministryofhealth&familywelfare
b)MinistryofSocialwelfare
c)Ministryofeducation
d)None
CorrectAnswer-B
Ans.is'b'i.e.,Ministryofsocialwelfare

881.TrueaboutNPCDCSisall,EXCEPT:
a)Separatecentreforstroke,DM
b)Implementationinsome5statesover10district
c)CHChasfacilitiesfordiagnosisandtreatmentofCVD,Diabetes
d)Daycarefacilitiesareavailableatsubcentre
CorrectAnswer-C
TheNPCDCSprogramhastwocomponentsviz.(i)Cancer&(ii)
Diabetes,CVDs&stroke.
Thesetwocomponentshavebeenintegratedatdifferentlevelsas
faraspossibleforoptimalutilizationoftheresource.
TheactivitiesatState,District,CHCandSubCentrelevelhave
beenplannedundertheprogrammeandwillbecloselymonitored
throughNCDcellatdifferentlevels.
Thestrategiesproposedwillbeimplementedin20,000SubCentres
and700CommunityHealthCentrein100Districtsacross21States
during2010-12
Earlydiagnosisofdiabetes,CVDs,StrokeandCancerisdone
atDistrictHospital,notatCHC.

Ref:NPCDCSOperationalGuidelines,DGHS,GOI,Page6;
http:/health.bih.nic.in/Docs/Guidelines-NPCDCS.pdf.

882.InternationalRedCrosswasfoundedby:
a)HenryDunant
b)JohnDRockfeller
c)MarieCurie
d)Noneoftheabove
CorrectAnswer-A
TheRedCrossisanon-politicalinternationalhumanitarianorganisationfoundedbySwiss
businessmanHenryDunant.
Ref:Park21steditionpage:858.

883.JaiVigyanNationalMissionisfor?
a)Adolescentgirlshealth
b)Mother&childhealth[MCH]
c)Science&technology
d)Childlabourprevention
CorrectAnswer-C
Ans.is'c'i.e.,Science&technology[RefInformationp.1997-
98]
TheUnionMinisterforhumanresourceDevelopment,Dr.Murli
ManoharJoshi,hassaidthatscientificInstitutionswouldtakeup21
importantdevelopmentprojectsaspartoftheJaiVigyanNational
Missionwithfocusonscienceandtechnology.Theseprojectswould
beinareassuchasfoodsecurity,energyconservations,health
care,disastermanagementandbio-diversity.ScientificandRandD
institutionswouldtakeuponeprojectin1999.Alltheprojectswould
begivengreenchanneltreatmentwithproceduresrelaxedhesaid
whileparticipatingattheannualsessionsofthe86t5IndianScience
CongressinChennalonJan.3.
TheministersaidhehaddirectedthedepartmentofBio-technology
toinstitute10awardseveryyearforoutstandingyoungscientistsin
biosciences.Thiswouldencouragehighqualityresearchof
excellenceandrelevance.Asachievingexcellencewasdependent
ininnovativetalentswhichinturnneededidentificationandnurturing
fromanearlyage,anewschemetoselectunder18youngstersof
outstandingtalentandtoprovidethemwithnecessaryambience
andopportunitiesforharnessingtheirtalentandbeeninstituted.

884.ColoredkitforSTDtreatmentiswhich
typeofapproach?
a)Preventive
b)Symptomatic
c)Syndromic
d)Rehabilitative
CorrectAnswer-C
Ans.is'c'i.e.,Syndromic[RefPark23'/e.p.332-336]
In1988,WorldHealthOrganizationintroducedtheconcept
ofSyndromicmanagement'.
Insyndromicmanagement,diagnosisandtreatmentisnotbasedon
specificdiseasesidentifiedbytestingbutratheronsyndromes,
whichisagroupofclinicalfindings.Treatmentisgenerallygivenfor
alloratleastmostcommonlyseendiseasesororganismsthatcould
causethatsyndrome.
Pre-packedcolourcodedSTI/RTIkitshavebeenprovidedforfree
supplytoalldesignatedSTI/RTIclinics.
Kit1Grey,forurethraldischarge,ano-rectaldischarge,cervicitis.
Kit2Green,forvaginitis
Kit3White,forgenitalulcers
Kit4Blue,forgenitalulcers
Kit5Red,forgenitalulcers
Kit6Yellow,forlowerabdominalpain
Kit7Black,forscortalswelling.

885.InRNTCPmicroscopiccenteris
recommendedforhowmuchpopulation
?

a)5000
b)10000
c)50000
d)100000
CorrectAnswer-D
Ans.is'd'i.e.,100000
TheRNTCPdesignated'Microscopycentre'isestablishedfor
100000populationinplanes(50000inhillyandmoutainareas).For
every5microscopycenters(500000population),thereisoneSenior
TBlaboratorysupervisor(STLS).STLSrechecksallpositiveslides
and10%ofallnegativeslides.Sputummicroscopicexamination
duringcasefindingisdoneindesignatedmiroscopycenters.
Onetuberculosisunitisestablishedfor500000populationinplanes
(250000populationinhilly/tribalareas).
Thereisonestatedrugstore(SDS)forevery50millionpopulation.

886.PERTiswhichtypeofmanagement
technique?
a)Basedonbehavioralscience
b)Qualitative
c)Quantitative
d)Noneoftheabove
CorrectAnswer-C
Ans.is'c'i.e.,Quantitative
Managementinhumanorganizationactivityissimplytheactof
gettingpeopletogethertoaccomplishdesiredgoalsandobjectives.
Therearetwomajortypesofmethodsofmanagement.
1. Methodsbasedonbehaviouralsciences.
2. Quantitativemethods.

887.WhichofthefollowingisSocratic
methodofcommunication?
a)Lectures
b)Groupdiscussion
c)Groupdiscussion
d)Massmedia
CorrectAnswer-B
Ans.is'b'i.e.,Groupdiscussion[RefPark24thlep.892
Two-waycommunication(socraticmethod)
Itismethodofcommunicationinwhichboththecommunicatorand
theaudiencetakepartandtheinformationistransferredinboth
direction.
ExamplesGroupdiscussion,Paneldiscussion,symposium,
workshop,conferences.

888.Worldheartdayiscelebratedon?
a)29thSeptember
b)28thSeptember
c)8thSeptember
d)1stDecember
CorrectAnswer-A
Ans.is'a'i.e.,29thSeptember[RefInternet]
29thSeptemberWorldheartday
28thSeptemberWorldrabiesday
8thSeptemberWorldliteracyday
PhDecemberWorldAIDSday

889.Minimumfloorspacerecommendedfor
workeraccordingtoFactoriesAct?
a)1000Cuft
b)500Cuft
c)200Cuft
d)100Cuft
CorrectAnswer-B
Ans.is'b'i.e.,500Cuft[RefPark24th/ep.852]
ThefirstIndianFactoriesactdatesasforbackas1881.
Theactwasrevisedandamendedseveraltimes,thelatesbeingthe
factories(Amendment)act,1987.
Followingstandardsarerecommended
1. Aminimumof500Cuftofspaceforeachworker(nottakinginto
accountspacemorethan14feetabovethegrouplevel).
2. Forfactoriesinstalledbeforethe1948Act,aminimumof350Cuft
ofspaceperworker.
3. Asafetyofficerineveryfactorywherein1000orworkersare
employed
4. Awelfareofficerineveryfactorywherein500ormoreworkersare
employed.
5. Acanteenwhereinmorethan250ormoreworkersareemployed.
6. Crecheswhereinmorethan30womenworkersareemployed.

890.Safetyofficerisrecommendedwhere
factoryhashowmanyworkers[In
factoryAct]?

a)250ormore
b)500ormore
c)1000ormore
d)2000ormore
CorrectAnswer-C
Ans.is'c'i.e.,1000ormore[RefPark24th/ep.852]
Followingstandardsarerecommended:

1. Aminimumof500Cuftofspaceforeachworker(nottakinginto
accountspacemorethan14feetabovethegrouplevel).
2. Forfactoriesinstalledbeforethe1948Act,aminimumof350Cuft
ofspaceperworker.
3. Asafetyofficerineveryfactorywherein1000orworkersare
employed
4. Awelfareofficerineveryfactorywherein500ormoreworkersare
employed.
5. Acanteenwhereinmorethan250ormoreworkersareemployed.
6. Crecheswhereinmorethan30womenworkersareemployed

891.Category4biomedicalwasteinclude?
a)Humananatomicalwaste
b)Animalwaste
c)Cytotoxicdrug
d)Wastesharps
CorrectAnswer-D
Ans.is'd'i.e.,Wastesharps[RefPark23"1/ep.793-794]
BMWsinIndiaaremanagedunder'BiomedicalWasteManagement
andHandlingRules1998'withexercisingpowers6,8,25of
EnvironmentalProtectionAct1986(underMinistryofEnvironment
andForests).
Schedulesare:-
1. ScheduleI:CategoriesofBMW,theirtreatmentanddisposal.
2. ScheduleII:ColorcodingandtypeofcontainerforBMWdisposal.
3. ScheduleIII:LabelsforBMWcontainers/bags.
4. ScheduleIV:LabelsfortransportofBMWcontainers/bags.
5. ScheduleV:StandardsfortreatmentanddisposalofBMW.
6. CategoryNo.7Solid(wastegeneratedfromdisposalitemsother
thanthewastesharpssuchastubings,catheters,intravenoussets
etc.)

892.Oneofthefollowingisnottrueof
InternationalClassificationofDisease-
a)Itisrevisedoncein10years
b)ItwasdevisedbyUNICEF
c)The10threvisionconsistsof21majorchapters
d)ItisacceptedforNationalandInternationaluse
CorrectAnswer-B
Ans.is'b'i.e.,ItwasdevisedbyUNICEF
ICDwasdevisedbyWHO.

893.Tensortympaniisattachedat?
a)Malleus
b)Incus
c)Stapes
d)Tympanicmembrane
CorrectAnswer-A
Ans.is'a'i.e.,Malleus[RefGray'sAnatomy38thlep.485]
Tympaniccavityhastwomuscles:

894.Themaxillarysinusopensintomiddlemeatusatthelevelof:
a)Hiatussemilunaris
b)Bullaethmoidalis
c)Infundibulum
d)Noneoftheabove
CorrectAnswer-A
Themaxillarysinusisthelargestoftheparanasalsinusesandislocatedinthemaxilla,
lateraltothenasalcavityandinferiortotheorbit.
Themaxillarysinusopensintotheposterioraspectofthehiatussemilunarisinthemiddle
meatus.
Theinfraorbitalnerve(CNV-2)primarilyinnervatesthemaxillarysinus.

895.Vesiclesonexternalearareseenin
a)Otitisexterna
b)Malignantotitisexterna
c)Herpeszoster
d)Clearcellcarcinoma
CorrectAnswer-C
Ans.is'c'i.e.,Herpeszoster[RefHeadandNeckSurgical
Pathologyp.53]
RamsayHuntsyndrome
ItislowermotorneurontypeoffacialpalsyduetoVaricella(herpes)
zoster.Painisoftenaprominentfeatureandvesiclesare
seenintheipsilateralear,onthehardpalateand/oronanteriortwo
thirdoftongue.Itmayinvolveothercranialnerves-V,
VIII,IXandXandcervicalbranches(C2,C3&C4)thathave
anastomoticcommunicationswithfacialnerve.Thisresults
infeatureslike:
Anaesthesiaofface
Giddiness
HearingimpairmentalongwithVIInervepalsy.

896.Firstlinetreatmentformildretraction
pocketintheearis
a)Observation
b)Antibiotics
c)Tympanostomytube
d)Surgicalexcision
CorrectAnswer-B
Ans.is'b'i.e.,Antibiotics[RefConqueringOtitismediaby
CharlesBluestonep.95]
Retractionpocket
Itmustbetreatedbyanotolaryngologist.
1)Antibiotics
Amildretractionpocketthatispresentinafluid-filledmiddleear
canfirstbetreatedwithantibiotics.
2)Tympanostomy
Ifantibioticsdoesnotwork,atympanostomytubeisusually
inserted,andinmostcases,theeardrumwillreturntonormal.
Iftheretractionpocketisverydeep,atubeshouldbeinserted,
bypassingtheantibiotictreatment.
3)Surgicalexcision
Iftheretractionpocketstilldoesnotgoaway,thedeformedeardrum
shouldbeoperatedontopreventacholesteatomafromdeveloping.
Onceacholesteatomadevelops,surgeryistheonlywaytoremove
it.

897.Cauliflowerearseenin:
a)Hematomaoftheauricle
b)Carcinomaoftheauricle
c)Fungalinfectionoftheauricle
d)Congenitaldeformity
CorrectAnswer-A
Cauliflowerear(boxer'sear,wrestler'sear)isanacquireddeformity
oftheouterear.
Inthisinjury,theearcanshrivelupandfoldinonitselfandappear
pale,givingitacauliflower-likeappearance,hencetheterm
cauliflowerear.
Wrestlers,boxersandmartialartistsinparticulararesusceptibleto
thistypeofinjury.Whentheearisstruckandabloodclotdevelops
undertheskin,ortheskinisshearedfromthecartilage,the
connectionoftheskintothecartilageisdisrupted.

898.Inelectrocochleography:
a)Itmeasuresmiddleearlatency
b)Outerhaircellsaremainlyresponsibleforcochlearmi
crophonicsandsummationpotential
c)Summationpotentialisacompoundofsynchronusauditory
nervepotential
d)TotalAPrepresentsendocochlearreceptorpotentialtoan
externalauditorystimulus
CorrectAnswer-B
Ans.is.B.Outerhaircellsaremainlyresponsibleforcochlear
microphonicsandsummationpotential


899.Whichfractureofthepetrousbonewill
causefacialnervepalsy:
a)Longitudinalfractures
b)Transversefractures
c)Mastoid
d)Facialnerveinjuryisalwayscomplete
CorrectAnswer-B
Ans.is.B.Transversefractures

900.OssicleM/CinvolvedinCSOM:
a)Stapes
b)Longprocessofincus
c)Headofmalleus
d)Handleofmalleus
CorrectAnswer-B
Ans.is.B.Longprocessofincus

901.MacEwan'striangleisthelandmarkfor:
a)Maxillarysinus
b)Mastoidantrum
c)Frontalsinus
d)None
CorrectAnswer-B
Ans.is.B.Mastoidantrum

902.Cristaeareseenin?
a)Utricle
b)Saccule
c)Semicircularcanal
d)Otolithmembrane
CorrectAnswer-C
Ans.is'c'i.e.,Semicircularcanal[RefDhingraSth/ep.16]
Vestibularapparatus
Thevestibularapparatuswithintheinnereardetectsheadmotion
andpositionandtransducesthisinformationtoaneuralsignal.

903.Functionofsacculeis?
a)Linearacceleration
b)Angularacceleration
c)Sensespositionofhead
d)Rotationalmovement
CorrectAnswer-A:C
Ans.is'a'i.e.,Linearacceleration&'c'i.e.,Sensespositionof
head


904.EarliestagefordoingBERAis?
a)Inutero-beforebirth
b)Atbirth
c)3months
d)6months
CorrectAnswer-B
Ans.is'b'i.e.,Atbirth[RefLoganTurner10thlep.254;PL
Dhingra4th/ep.28]
"Worldwidescreeningislargelyperformedinnewbornnurserywith
thefirstscreeningtestperformedfrombirthuntil10daysofage"

905.Mostcommonmalignancyofmiddleear
is
a)Glomustumor
b)Squamouscellcarcinoma
c)Adenocarcinoma
d)Sarcoma
CorrectAnswer-B
Ans.is'b'i.e.,Squamouscellcarcinoma[RefTextbookofENT
byRakeshShrivastav2"/ep.67]
Squamouscellcarcinomaisthemostcommonmalignanttumorof
themiddleear.
Otherformsofmalignancylikeadenocarcinomaandsarcomaare
rare.

906.Mostcommonbenigntumorofearcanal
is
a)Osteomas
b)Sebaceousadenoma
c)Papilloma
d)Ceruminoma
CorrectAnswer-A
Ans.is'a'i.e.,Osteomas[RefDhingra6thlep.107;BansalENT
p.160;EncyclopediaofImaging,Baertp.1318]
Osteomas/exostosesmostcommonbenigntumorsofthe
externalauditorycanal.
SquamouscellcarcinomasMostcommonmalignanttumorsof
theexternalauditorycanal.

907.Diplacusisis
a)Hearingsoundwithdiminishedintensity
b)Hearingsoundsoftwodifferenttones
c)Hearingextremelyloudsound
d)Perceivinglightonproductionofsound
CorrectAnswer-B
Ans.is'b'i.e.,Hearingsoundsoftwodifferenttones[RefTuli
lst/ep.114]
Monauraldiplacusis:-Inmonauraldiplacusis,alistenerhearstwo
toneswhenasingletoneispresentedtoasingleear,i.e.oneear
hearstwodifferenttoneswhenpresentedone.
Binauraldiplacusis:-Inbinauraldiplacusis,alistenerhearstwo
differenttonesinright&leftearwhenasingletoneispresentedto
bothears.
Bothmonoauralandbinauraldiplacusisarecausedby
inhomogenitiesinthecochleathatalsogiverisetospontaneous
otoacousticemissions.

908.Whichsemicircularcanalismost
commonlyinvolvedinBPPV?
a)Horizontal
b)Posterior
c)Superior
d)Alloftheabove
CorrectAnswer-B
Ans.is'b'i.e.,Posterior[RefDhingraSth/ep.51]
BPPVisthoughttobecausedbydisplacementofotoconia(mineral
crystals)fromthevestibuleofinnerearintothesemicircularcanals.
Theposteriorsemicircularcanalismostcommonlyinvolved,though
superiorandhorizontalcanalscanalcanalsobeaffected.
Otoconiaorearrocksaresmallcrystalsofcalciumcarbonate
derivedstructureintheutriclewhichmigrateintosemicircularcanals
andcauseBPPV.Whilesacculesalsocontainsotoconia,theyare
notabletomigrateintothesemicircularcanals.

909.Mostcommonossicleaffecteddueto
trauma-
a)Malleus
b)Incus
c)Stapes
d)Allaffectedsimilarly
CorrectAnswer-B
Ans.is'b'i.e,Incus[RefHearing:PracticalGuidebyTysomep.
96]
"Significantheadinjuryordirecttraumatothemiddleearcanresult
insubluxationofoneormoreoftheossicles,theincusbeingthe
mostcommonlyaffected

910.Thresholdofhearinginayoungnormal
adultis?
a)0dB
b)10dB
c)20dB
d)30dB
CorrectAnswer-A
Ans.is'a'i.e.,0dB
[RdDhingra4n/ep.21]
Audiometriczero
Thresholdofhearing,i.e.Thefaintestintensitywhichanormal
healthypersoncanhearwillvaryfrompersontoperson.
TheInternationalStandardsOrganisation(ISO)adoptedastandard
forthis,whichisrepresentedasthezerolevelontheaudiometer(0
dB).
AccordingtoISO,audiometriczeroisthemeanvalueofminimal
audibleintensityinagroupofnormallyhearinghealthyyoungadults.

911.Gradenigo'ssyndromeinvolvesallofthe
followingcranialnerves,EXCEPT:
a)IV
b)V
c)VI
d)VII
CorrectAnswer-A
Ans.A.IV
Gradenigo'ssyndromeischaracterizedbyfacialpain,particularlyin
thefirstdivisionofthetrigeminalnerveanddiplopiaduetosixth
cranialnervepalsy.Itisassociatedwithdiseaseattheapexofthe
petroustemporalbonewheretheabducensnerveiscloselyrelated
tothetrigeminalnerve.Facialnervepalsyanddeafness(VIIInerve
palsy)isalsoconsideredtobeapartofthissyndrome.
Causesincludes:
Inflammation(petrositis,possiblyspreadingfromalocalinfection
suchasotitisormastoiditis)
Tumors(cholesteatoma,chordoma,meningioma,nasopharyngeal
carcinoma,metastaticdisease)
Skullbasefracture

912.Singaporeearis
a)Hypertrophyofsweatglands
b)Hypertrophyofsebaceousglands
c)Otitisexterna
d)Excoriationofexternalearskin
CorrectAnswer-C
Ans.is'c'i.e,Otitisexterna[RefClinicalENT5th/ep.223]
Excessivemoistureisanimportantpredisposingfactorforotitis
externaasexcessivemoistureelevatesthepHandremoves
protectivecerumen.Therefore,humidityandhotclimatepredispose
tootitisexterna.Henceotitisexternaisalsoknownas:-
1. Singaporeear(whereclimateishotandhumid)
2. Swimmer'sear
3. Telephonist'sear(telephonistswhorequireinsertsintheirearhave
excessivemoistureduetosweating).

913.Allofthefollowingaretrueabout
malignantotitisexternaexcept:
a)ESRisusedforfollowupaftertreatment
b)Granulationtissuesareseenonsuperiorwalloftheexternal
auditorycanal
c)Severehearinglossisthechiefpresentingcomplaint
d)Pseudomonasisthemostcommoncause
CorrectAnswer-C
Ans.c.Severehearinglossisthechiefpresentingcomplaint
Severehearinglossisnotthechiefpresentingcomplaintmalignant
otitisexterna.
MalignantOtitisExterna:
Characterizedbygranulationtissueinexternalauditorycanalatthe
junctionofboneandcartilage.
MCorganismzPseudomonasaeruginosa
ESRisraised,usedforfollowupoftreatment

914.Adiabeticpatientpresentswithfoul
smellingeardischarge,feverandsevere
painintheear.Onexaminationthereis
thickyellowcoloureddischargefromthe
earandgranulationtissueinthecanal.
Whichofthefollowingistheappropriate
managementforthispatient?

a)Surgicaldebridement
b)Antibiotictherapy
c)Cryotherapy
d)Laserremovalofgranulationtissue
CorrectAnswer-B
Ans.is'b'i.e.,Antibiotictherapy[RefDhingra6th/ep.52]
Thepatientdescribedinthequestionmostlikelyhasmalignantotitis
externa.
Sothetreatmentofchoiceisanti-pseudomonalantibiotics.
Extensivesurgicaldebridementonceanimportantpartofthe
treatmentisnowrarelyneeded

915.Topodiagnosisoffacialnervehasallthe
testsexcept
a)Schirmertest
b)Bingtest
c)Tastetest
d)Salivaryflowtest
CorrectAnswer-B
Ans.is'b'i.e.,Bingtest[RefDhingra6th/ep.98]
Thefollowingtestsareusefulinfindingthesiteoflesioninparalysis
oflowermotorneuron.
1. Schirmertest:Itcompareslacrimationofthetwosides.Astripof
filterpaperishookedinthelowerfornixofeacheyeandtheamount
ofwettingofstripmeasured.Decreasedlacrimationindicateslesion
proximaltothegeniculategan-glionasthesecretomotorfibresto
lacrimalglandleaveatthegeniculateganglionviagreatersuperficial
petrosalnerve.
2. Stapedialreflex:Stapedialreflexislostinlesionsabovethenerve
tostapedius.Itistestedbytympanometry.
3. Tastetest:Itcanbemeasuredbyadropofsaltorsugarsolution
placedononesideoftheprotrudedtongue,orbyelectrogustometry.
Impairmentoftasteindicateslesionabovethechordatympani.
4. Submandibularsalivaryflowtest:Italsomeasuresfunc-tionof
chordatympani.PolythenetubesarepassedintobothWharton
ductsanddropsofsalivacountedduringoneminuteperiod.
Decreasedsalivationshowsinjuryabovethechorda.

916.Whichofthefollowingisthefunctionof
tensortympanimuscle?
a)Dampenveryloudsound
b)Tensestympanicmembrane
c)Tensespharyngotympanictube
d)Preventnoisetraumatotheinnerear
CorrectAnswer-B
Ans.is'b'i.e.,Tensestympanicmembrane[RefDhingra6"/ep.
8]
Normalopeningoftheeustachiantubeequalizesatmospheric
pressureinthemiddleear;closingoftheEustachiantubeprotects
themiddleearfromunwantedpressurefluctuationsandloud
sounds.
Themusclesoftheeustachiantubesystemhelptoopenandclose
thetube,thusallowingittoperformitsfunction.
Themusclesare:?
1. Tensorvelipalatini(tensorpalatini):-Contractionofthismuscle
duringswallowing,yawningandsneezingopensthetube.Defective
functionofthismuscleincleftpalateresultsineustachiantube
dysfunction.
2. Tensortympani:-Tensonisesthetympanicmembrane.
3. Levatorvelipalatini:-Sometimes,italsohelpstoopenthetube,
howeveritisusuallyconsideredasavelopharyngealvalvemuscle
only.
4. Salpingopharyngeus:-Itsfunctionalsignificanceisquestionable.

917.Trueaboutserousotitismediaareall
except?
a)Alsocalledglueear
b)Affectschoolgoingchildren
c)TypeCtympanogram
d)Fluidinmiddleear
CorrectAnswer-C
Ans.is`c'i.e.,TypeCtympanogram
Serousotitismedia
Serousotitismedia(SOM)hasmanysynonyms:Serousotitis
media,otitismediawitheffusion,glueear,non-suppurativeotitis
media,mucoidotitismedia,silentotitismedia.SOMisaninsidious
conditioninwhichthereisthickorstickynon-purulentfluidbehind
theeardruminthemiddleear,butthereisnoearinfection,i.e.,
effusionofmiddleearwithoutinfection.Fluidinthemiddleearis
sterile.SOMoccursmostcommonlyinschoolgoingchildrenand
SOMisthecommonestcauseofchildhoodhearingloss.
Etiopathogenesis
1.Eustachiantubedysfunction
Eustachiantubedysfunction,coupledwithrecurrentupper
respiratorytractinfectionisthemostimportantfactorinthe
developmentofSOM.Normallyeustachiantubehelpstodrainfluids
topreventthemfrombuildingupintheear.InEustachiantube
dysfunction,itisunabletodrainthefluid.Followingcancause
Eustachiantubeblock:-
1. Respiratorytractinfection:-Adenoid,rhinitis,tonsillitis,sinusitis.
2. Allergies
3. Benignandmalignanttumorofnasopharynx.

2.Unresolvedotitismedia
Inadequateantibiotictherapyinacutesuppurativeotitismediamay
inactivateinfectionbutfailstoresolveitcompletely.Lowgrade
infectionlingersonandactsasstimulusformucosatosecretemore
fluid.
Clinicalfeatures
Unlikechildrenwithanearinfection(ASOM),childrenwithSOMdo
notactsick.oTheonlypresentingsymptommaybehearingloss
withfullnessinear.
OtoscopicfindingofSOM
Airbubblesonthesurfaceofeardrum
Fluidbehindtheeardrum.
Dullnessoftheeardrumwhenalightisused,withlossoflightreflex.
oEardrummayappearyellow,greyorbluishincolour.
Retractedeardrumwithdecreasedmobility
TympanometryshowstypeBtympanogram.

918.Pinkreflexthroughintacttympanic
membraneinactiveotosclerosisis
knownas

a)Schwabach'ssign
b)Schwartzsign
c)Lyre'ssign
d)Chvostek'ssign
CorrectAnswer-B
Ans.is'b'i.e.,Schwartzsign[RefDhingra5th/ep.98;Current
otolaryngology2nd/ep.674]
Areddishhue/Flammingopinkmaybeseenonthe
promontoryandovalwindownicheowingtotheprominent
vascularityassociatedwithanotospongioticfocus.Thisisk/a
Schwartzsign.
Schwartzsignisapinkreflex,seenthroughintacttympanic
membrane,intheareaofovalwindow.Itindicatesactive
otosclerosis
usuallyduringpregnancy.
Lyre'ssignissplayingapartofinternalandexternalcarotidarteries
onangiogramincasesofcarotidbodytumouroftheneck.
Chvostek'ssignseeninhypocalcaemiaasaftertotalthyroidectomy
whereparathyroidshavealsobeenremoved.Tappingoverthe
distributionoffacialnerveproducesatwitch.

919.SADEclassificationclassifies
a)Retractionoftympanicmembrane
b)ExtensionofGlomustumor
c)Mortalityafterheartdiseaseduringpregnancy
d)ExtentofCSFrhinorrhea
CorrectAnswer-A
Ans.is'a'i.e.,Retractionoftympanicmembrane[RefBasic
ClinicalRadiobiology5th/ep.942]
Sade'sclassificationofretractionoftheparstensaofthetympanic
membranedefinedtwotypesofretractionandclassifiedeachonan
ordinalscale:?
1.Atelectasis,definedasdiffuse'retractionofthetympanic
membranetowardsthepromontorium'.
2.Retractionpocket,definedasfocal'retractionoftheparstensa
towardsorintotheattic'

920.A10yearoldchildpresentswithnon
foulpurulentsmellingdischarge,which
ispainless.Patientreportsthatheisable
tohearbetterinthepresenceof
dischargethanwhentheearisdry.The
mostprobablediagnosisis

a)CSOM
b)Serousotitismedia
c)Cholesteatoma
d)Mastoiditis
CorrectAnswer-A
Ans.is'a'i.e.,CSOM[RefDhingra6th/ep.68]
ClinicalfeaturesofCSOM
Profusemucopurulantdischargewhichisnotfoulsmelling,i.e.,non
foulsmellingdischarge.
Hearingloss(conductivetype).Ifsensorineuralcomponentalso
occurs(i.e.,mixedtype),itarousesthesuspicianoftoxicdeafness.
Sometimes,patientreportsaparadoxicaleffect,i.e.,hearsbetterin
thepresenceofdischargethanwhentheearisdry.Thisisdueto
roundwindowshieldingeffectproducedbydischargewhichhelpsto
maintainphasedifferential.
Thereisnopain,ifitoccursitisduetoassociatedotitisexternanot
duetootitismedia.
Sincetheinfectedareaisopenatbothends,dischargedoesnot
accumulateinthemiddleearcavity.
Ossicularchainismostlyuninvolved,ifinvolvedonlylongprocessof

incusisinvolved.

921.Treatmentofcentralsafeperforationof
tympanicmembraneincludesallexcept
a)Auraltoilet
b)Eardrops
c)Avulsionofauralpolyp
d)Myringoplasty
CorrectAnswer-C
Ans.is'c'i.e.,Avulsionofauralpolyp[RefDhingra6thlep.72]
Anauralpolypshouldneverbeavulsedasitmaybearisingfromthe
stapes,facialnerveorhorizontalcanalandthusleadtofacial
paralysisorlabyrinthitis.
Treatment
Theaimistocontrolinfectionandeliminateeardischargeandata
laterstagetocorrectthehearinglossbysurgicalmeans.
1. Auraltoilet:Removealldischargeanddebrisfromtheear.Itcanbe
donebydrymoppingwithabsorbentcottonbuds,suctionclearance
undermicroscopeorirrigation(notforcefulsyringing)withsterile
normalsaline.Earmustbedriedafterirrigation.
2. Eardrops:Antibioticeardropscontainingneomycin,polymyxin,
chloromycetinorgentamicinareused.Theyarecombinedwith
steroids,whichhavelocalantiinflammatoryeffect.
3. Systemicantibiotics:Theyareusefulinacuteexacerbationof
chronicallyinfectedear,otherwiseroleofsystemicantibioticsinthe
treatmentofCSOMislimited.
4. Precautions:Patientsareinstructedtokeepwateroutoftheear
duringbathing,swimmingandhairwash.Rubberinsertscanbe
used.Hardnoseblowingcanalsopushtheinfectionfrom
nasopharynxtomiddleearandshouldbeavoided.

5. Treatmentofcontributorycauses:Attentionshouldbepaidtotreat
concomitantlyinfectedtonsils,adenoids,maxillaryantrumandnasal
allergy.
6. Surgicaltreatment:Auralpolyporgranulations,ifpresent,shouldbe
removedbeforelocaltreatmentwithantibiotics.Itwillfacilitateear
toiletandpermiteardropstobeusedeffectively.
7. Reconstructivesurgery:Onceearisdry,myringoplastywithor
withoutossicularreconstructioncanbedonetorestorehearing.
Closureofperforationwillalsocheckrepeatedinfectionfromthe
externalcanal.

922.Korner'sseptumisseenin?
a)Petrosquamoussuture
b)Temporosquamoussuture
c)Petromastoidsuture
d)Frontozygomaticsuture
CorrectAnswer-A
Mastoiddevelopsfromsquamousandpetrousbone.
Korner'sseptumispersistenceofpetrosquamoussutureintheform
ofabonyplate.
Korner'sseptumissurgicallyimportantasitmaycausedifficultyin
locatingtheantrumandthedeeper(ells,andthusleadtoincomplete
removalofdiseaseatmastoidectomy.Mastoidantrumcannotbe
reachedunlessthe
Kornersseptumhasbeenremoved.

923.Aggernasiis
a)Mucosalflapcoveringthenasolacrimalduct
b)Openingofthesinuses
c)Depressioninfrontofmiddleturbinate
d)Elevationanteriortomiddleturbinate
CorrectAnswer-D
Ans.is'd'i.e.,Elevationanteriortomiddleturbinate[Ref
Dhingra5thiep.150;Tuli1"/e
p.135]
Atriumisashallowdepressioninfrontofthemiddleturbinate.Agger
nasiisanelevationjustanteriortotheattachementofmiddle
turbinate.

924.Quadrungularseptumisseeninwhichof
thefollowing?
a)Larynx
b)Nose
c)Cranium
d)Palate
CorrectAnswer-B
Ans.is'b'i.e.,Nose[RefDhingra5th/ep.150]
"Cartilageoftheseptum,alsoknownasthequandragularcartilage
becauseitisroughlyquadrilateralinshape-separatesthenostrils".

925.Nasalvestibuleis
a)Lateralpartofnasalcavity
b)Antero-inferiorpartofnasalcavity
c)Supero-medialpartofnose
d)Posteriorapertureofnose
CorrectAnswer-B
Ans.is'b'i.e.,Antero-inferiorpartofnasalcavity[Ref
OtolaryngologyBasicScience&ClinicalReviewbyVanDe
Waterp.462]
Internalnosehasfollowingparts:
Nasalcavityproper:-Internalnoseisdividedintorightandleftnasal
cavitiesbynasalseptum.Eachnasalcavitycommunicateswiththe
exteriorthroughnarisornostrilsandwithnasopharynxthrough
posteriornasalapertureorposteriornaresorchoana.
Vestibuleofnose:-Anteriorandinferiorpartofthenasalcavityis
linedbyskinandiscalledvestibuleofnose.Itcontainssebaceous
glands,hairfolliclesandthehaircalledvibrissae.

926.Whichofthefollowingtermsisusedto
describethemostprominentpointof
nasaltip?

a)Pronasale
b)Alare
c)Nasion
d)Columellaapex
CorrectAnswer-A
Ans.is'a'i.e.,Pronasale[RefHandbookofanthropometry:
Physicalmeasuresofhumanforminhealthanddiseases,by
VictorR.Preedy,p.922]
Pronasale:Mostprominentpointonthenasaltip.
Alare:Thepointwherethenasalblade(Alanasi)extendsthe
farthestout.
Columellaapex:Themostanteriororthehighestpointonthe
columellacrestattheapexofthenostril.
Nasion:Thepointinthemidlineofboththeanatomicnoseandthe
nasoanterioralsuture

927.Epistaxisafterligatingexternalcarotid
arteryisduetowhichvessel?
a)Anteriorethmoidalartery
b)Superiorlabialartery
c)Sphenopalatineartery
d)Greaterpalatineartery
CorrectAnswer-A
Ans.is'a'i.e.,Anteriorethmoidalartery[RefDhingra5thiep.
190]
Sinceexternalcarotidarteryisligated,thebleedingcomesfrom
branchesoftheinternalcarotidartery.
Anteriorethmoidalartery,abranchofophthalmicartery,whichisa
branchofinternalcarotidartery,isaconstituentofthebloodsupply
oftheLittle'sareaofthenasalseptum.
Bloodsupplyofnasalseptum
i)Internalcarotidsystem
Anteriorethmoidalartery
Branchesofophthalmicartery
Posteriorethmoidalartery
ii)ExternalCarotidSystem
Sphenopalatineartery(branchofmaxillaryartery)gives
nasopalatineandposteriormedialnasalbranches.
Septalbranchofgreaterpalatineartery(Br.ofmaxillaryartery).
Septalbranchofsuperiorlabialartery(Br.offacialartery)

928.Killian'spolypisa/an
a)Antrochoanalpolyp
b)Ethmoidalpolyp
c)Frontalpolyp
d)Maxillarypolyp
CorrectAnswer-A
Ans.is'a'i.e.,Antrochoanalpolyp[RefDhingra5th/ep.186;
Essentialotolaryngology2"1/ep.
660]
Nasalpolypsareoftwotypes:?
Antrochoanal:-Thisisusuallysolitaryandarisesfrommaxillary
sinusandgrowsbackwardinthenosetowardsthechoana(in
contrasttoethmoidalpolypswhichtendtogrowforward).The
AntrochoanalpolypwasfirstdescribedbyGustainkillians,therefore
thenamegiventoitwaskillian'spolyp.
Ethmoidal:-Thesearemultiple,bilateralandarisefrometmoidal
sinusesandtendtoprotrudeforwards.

929.Whichofthefollowingisthe
predisposingfactorforethmoidal
carcinoma

a)Smoking
b)Alcohol
c)Chronicinfection
d)Alloftheabove
CorrectAnswer-A
Ans.is'a'i.e.,Smoking[RefDiseasesofENTbyBansalp.363]
Riskfactorsforsquamouscellcarcinomaoftheparanasal
sinuses:-
Smoking,Mustardgas,Nickelandchromiumplatingindustry,
Isopropyloil
Leatherindustry,Wooddustexposure(adenocarcinomaof
ethmoid),Polycyclicvolatilehydrocarbons

930.Mostcommonsinuspredisposedto
malignancywhichofthefollowing?
a)Ethmoid
b)Maxillary
c)Frontal
d)Sphenoid
CorrectAnswer-B
Ans.is'b'i.e.,Maxillary[RefDhingra5thiep.219]
Themajorityofparanasalsinusmalignancies(50-80%)originate
withinthemaxillarysinusantrum.
Sinusesandvariousconditionsinwhichsinusesareaffectedin
descendingorderoffrequency

931.Mostcommonbenigntumorofparanasal
sinuses?
a)Papilloma
b)Osteoma
c)Warts
d)Fibroma
CorrectAnswer-B
Ans.is'b'i.e.,Osteoma[RefLoganTurner8`h/ep.89]
Osteomaofthefrontalsinus-Mostcommonbenigntumourarising
inthenasalsinuses.
Theosteomaarisesfromthefloorofthefrontalsinusnearthe
midline.

932.Rhinolithcancause
a)Nasalobstruction
b)Epistaxis
c)Epiphora
d)Alloftheabove
CorrectAnswer-D
Arhinolithisacalculuspresentinthenasalcavity.
Thewordisderivedfromtherootsrhino-and-lith,literallymeaning
"nosestone".
Arhinolithusuallyformsaroundthenucleusofasmallexogenous
foreignbody,bloodclotorsecretionbyslowdepositionofcalcium
andmagnesiumsalts.
Overaperiodoftime,theygrowintolargeirregularmassesthatfill
thenasalcavity.
Theymaycausepressurenecrosisofthenasalseptumor
lateralwallofnoseleadingtonasalobstruction,epistaxis,
headache,sinusitisandepiphora.


933.WhatisaRhinolith:
a)Foreignbodyinnose
b)Stoneinnose
c)Depositionofcalciumaroundforeignbodyinnose
d)Misnomer
CorrectAnswer-C
Rhinolithsarecalcareousmasseswhichresultduetodepositionof
salts-likecalciumandmagnesiumcarbonatesandphosphates
aroundthenucleusofaforeignbody.

934.Treatmentofnasalbonefracture
includesallexcept
a)Hematomadrainage
b)Topicalvasoconstrictor
c)Closedreduction
d)Immediaterhinoplasty
CorrectAnswer-D
Ans.is'd'i.e.,Immediaterhinoplasty[RefScottBrown7th/ep.
1612]
Treatmentofnasalinjuries
Afterensuringpatencyofairway,adequateventilationandoverall
stabilityofpatient,attentiontothenasalinjuriesshouldbegiven.
Standardtherapyistoperformclosedreductionoropenreducation
between3and7days,andupto2weeks.Thisisbecause:
Mostofthepatients(-70-80%)donotrequireanyactivetreatment,
asmanydonothaveanasalfractureandthosethatdo,thefracture
isnotdisplaced.Softtissueswellingcanproducethemisleading
appearanceofadeformitywhichdisapearsastheswellingsubsides.
Suchpatientsrequireonlyreassurance&topicalvasoconstrictorsto
alleviatecongestionandobstructivesymptoms.Are-examination
shouldbecarriedoutafter5days,ifthereisuncertainityaboutthe
needforreduction.
Alargenumberofpatientswillhaveapreexistingnasaldeformity
causedbyapreviousincident.Patientsthatfallinthiscategoryare
advisedtoconsideraformalrhinoplastywheneverythinghassettled
downsomemonthslater.
Immediatesurgicalinterventioninacutephaseisindicated:?
Severedeformity:-Septalhematomacausingnasalobstruction


935.Allofthefollowingaretrueabout
parapharyngealabscessexcept?
a)Mastoidprocessdividesthespaceintoanteriorandposterior
b)Alsoknownaspharyngomaxillaryspaceinfection
c)Tonsilispushedmedially
d)Occursaftertoothextraction
CorrectAnswer-A
Ans.is'a'i.e.,Mastoidprocessdividesthespaceintoanterior
andposterior[RefDhingraV/ep.281]
Styloidprocessandthemusclesattachedtoitdividethe
parapharyngealspaceintoanteriorandposteriorcompartments,not
mastoidprocess.

936.Trismusinparapharyngealabscessis
duetospasmof:
a)Massetermuscle
b)Medialpterygoid
c)Lateralpterygoid
d)Temporalis
CorrectAnswer-B
Styloidprocessdividesthepharynxintoanteriorandposterior
compartment.
Trismusoccursininfectionofanteriorcompartmentwhereas
torticollis(duetospasmofparavertebralmuscles)occursinthe
infectionofposteriorcompartment.

937.Regardingadenoidstrueis/are:
a)Thereisfailuretothrive
b)Mouthbreathingisseen
c)CTscanshouldbedonetoassesssize
d)aandb
CorrectAnswer-D
Higharchedpalateandmouthbreathingarefeaturesof
hypertrophiedadenoidswhichleadstoadenoidfacies
Inadenoidsasaconsequenceofrecurrentnasalobstructionand
URTI,childdevelopsfailuretothrive
Sizeofadenoidsmaywellbeassessedusinglateralradiographof
nasopharynx,andCTscanisnotnecessary
Adenoids,alsoknownasnasopharyngealtonsils,aresubepithelial
collectionoflymphoidtissueatthejunctionofroofandposteriorwall
ofnasopharynx.

938.Liningepitheliumofvocalcordis
a)Stratifiedsquamousepithelium
b)Nonstratifiedsquamousepithelium
c)Ciliatedcolumnarepithelium
d)Nonciliatedcolumnarepithelium
CorrectAnswer-A
Ans.is'a'i.e.,Stratifiedsquamousepithelium[RefDhingraVie
p.300]
Vocalcordsalongwiththeupperpartofthevestibuleistheonlypart
oflaryngealmucousmembranewhichislinedbyepitheliumof
stratifiedsquamoustype.Elsewhereinthelarynxepitheliumofthe
mucousmembraneisciliatedcolumnartype.

939.SelectcorrectstatementsaboutCa
larynx:
a)GlotticCaisthemostcommon
b)Supraglotticcahasbestprognosis
c)LymphaticspreadisthemostcommoninsubglotticCa
d)All
CorrectAnswer-A
Ans.is.A.GlotticCaisthemostcommon

940.WhichofthefollowingsitesofCalarynx
hasthebestprognosis?
a)Glottic
b)Supraglottis
c)Subgottis
d)Allhavepoorprognosis
CorrectAnswer-A
Ans.is'a'i.e.,Glottic[RefDhingra5th/ep.326]
Laryngealcarcinoma:-

Cancer
Prognosis
Glottis
Good
Supraglottis Poor
Subglottis
Worst

941.Laserusedintrachealneoplasmis
a)Argon
b)KTP-532
c)CO2
d)Nd-YAG
CorrectAnswer-C
Ans.is'c'i.e.,CO2[RefDhingra5th/ep.315]
FourtypesoflaseraregenerallyusedinENTsurgery:
Argon
KTP-532(Potassiumtitanylphosphate);
Nd:YAG(Neodyminum-yttriumaluminiumgarnet);
CO2
Thecarbondioxide(CO)laseristhemostcommonlaserusedfor
trachealneoplasm.

942.Mostcommonsiteofdistantmetastasis
fromCalarynx
a)Lymphnodes
b)Lung
c)Brain
d)Bone
CorrectAnswer-B
Ans.is'b'i.e.,Lung[RefClinicalOtorhinolaryngology
yd/el).931]
Distantmetastasisisseenmuchlessfrequently.Themost
commonlyaffectedsitesfordistantmetastasesarelungs(66%);
bone(22%),liver(10%),mediastinumandbonemarrow.

943.AllthefollowingaretrueaboutLaryngeal
carcinomaexcept:
a)Morecommoninfemales
b)Commoninpatientsover40yearsofage
c)Afterlaryngectomy,esophagealvoicecanbeused
d)bandc
CorrectAnswer-A
CancerLarynx
MostcommonhistologicaltypeoflaryngealCa-Squamouscell
carcinoma(seenin90%cases)
Itismorecommoninmales.
Male:Femaleratiois4:1
Mostcommonage=60-70years.

944.ApatientpresentswithCalarynx
involvingleftfalsecord,leftarytenoid
andleftaryepiglotticfoldwithbilateral
mobiletruecords.Thetreatmentof
choiceinthispatientiswhichofthe
following?

a)Verticalhemilarygectomy
b)Horizontalpartialhemilaryngectomy
c)Totallaryngectomy
d)Radiotherapyfollowedbychemotherapy
CorrectAnswer-B
Ans.is'b'i.e.,Horizontalpartialhemilaryngectomy[RefDhingra
4th/e
p.284]
Involvementofunilateralfalsecord,aryepiglotticfoldsand
arytenoidswithmobilecordsuggestssupraglotticcancerinT2stage
(morethanonesubsitesofsupraglottisareinvolved).

945.Inrecurrentlaryngealnervepalsywhich
musclekeepsvocalcordinmedian
position?

a)Posteriorcricoarytenoid
b)Cricothyroid
c)Vocalis
d)Alloftheabove
CorrectAnswer-B
Ans.is'b'i.e.,Cricothyroid[RefDhingra5th/ep.300]
Recurrentlaryngealnerveparalysis
Recurrentlaryngealnervesupplies:?

1. Allintrinsicmusclesofthelarynxexceptcricothyroid.
2. Sensorysupplytolarynxbelowvocalcords.
So,paralysisofrecurrentlaryngealnervecauses:?
1. Paralysisofallintrinsicmusclesincludingalladductors(exceptfor
cricothyroid)andallabductors.
2. Anaesthesiabelowthelevelofvocalcord.
Though,thereisparalysisofbothadductors(exceptcricothyroid)
andabductors,themanifestationsaremainlyduetoparalysisof
abductors.

946.

Esophagusisconstrictedat4anatomiclocations.Narrowestpartofesophaguslies
atwhichofthefollowingconstriction?

a)Atthelevelofcricopharyngealsphincter
b)Atthecrossingofaorticarch
c)Atthecrossingofleftbronchus
d)Atthelevelofopeninginthediaphragm
CorrectAnswer-A
Narrowestpartofesophagusisatitscommencementatthecricopharyngealsphincter
whichis15cmfromtheincisorteeth.
Othersitesofesophaguswhereitisslightlyconstrictedincludes:
Crossingbytheaorticarchwhichis22cmfromtheteeth.
Crossingbytheleftprincipalbronchuswhichis27cmfromtheteeth
Attheopeninginthediaphragmwhichis38cmfromtheteeth

947.

Foodparticlesmostlygetobstructedinwhich
partofesophagus-

a)Cricopharyngealsphincter
b)Crossingofarchofaorta
c)Cardiacend
d)Noneoftheabove
CorrectAnswer-A
Ans.is'a'i.e,Cricopharyngealsphincter[Ref:Dhingra6thiep.
349]
Byfarthecommonestsiteisatorjustbelowthecricopharyngeal
sphincter.Flatobjectslikecoinsareheldupatthesphincterwhile
othersareheldintheupperoesophagusjustbelowthesphincter
duetopoorperistalsis.

948.Trueregardingtractiondiverticuliof
esophagusisallexcept
a)Doesnotemptycompletely
b)Triangularappearance
c)Containsalllayers
d)Maintainselasticrecoil
CorrectAnswer-A
Ans.is'a'i.e.,Doesnotemptycompletely[RefTheEsophagus
byRichterp.143]

PulsionDiverticula
Falsediverticula
TractionDiverticula
Ususallyonlymucosal
Truediverticula
outpouchingsDistal
Type
Linedbyalllayers
esophagus
Layers
Midesophagus
Associatedwith
Location
ScarringfromTBor
dysmotility
Associated histoplasmosisinvolvingperihilar Rounded
Shape
orsubcarinallymphnodes
outpouchings
Emptying Triangularortented
Donotempty
Tendstoemptycontents
completely
Epiphrenicdiverticula,
Zenker'sdiverticula

949.Simplemastoidectomyisdonein:
a)Acutemastoiditis
b)Cholesteatoma
c)Coalescentmastoiditis
d)Localizedchronicotitismedia
CorrectAnswer-C
Ans.is.C.Coalescentmastoiditis

950.Radicalmastoidectomyisdonefor:
a)ASOM
b)CSOM
c)Atticoantralcholesteotoma
d)Acutemastoiditis
CorrectAnswer-C
Ans.is.C.Atticoantralcholesteotoma

951.Sluder'sneuralgiaisalsocalledas
a)Anteriorethmoidalsyndrome
b)Posteriorethmoidalsyndrome
c)Trottersyndrome
d)Lermoyezsyndrome
CorrectAnswer-A
Ans.is'a'i.e.,Anteriorethmoidalsyndrome[RefDhingra6th/e
p.450]
Sluder'sneuralgia
Itisalsocalledanteriorethmoidalsyndrome.
Itissaidtooriginatefromthemiddleturbinatepressingonthenasal
septum.
Itischaracterizedbypainaroundthebridgeofthenoseradiating
intoforehead.
TreatmentAnatomicalcorrectionrelievesthepain.

952.Styloidprocessisderivedfrom?
a)1starch
b)2ndarch
c)3rdarch
d)4tharch
CorrectAnswer-B
Styloidprocessisderivedfrom2ndpharyngealarch.

953.Mostcommoncauseofsinger'snodule
is?
a)Infection
b)Allergy
c)Vocalabuse
d)None
CorrectAnswer-C
Ans.is'c'i.e.,Vocalabuse
Vocalnodule(singer'sorscreamer'snodule)
Vocalnodulearebenignnoneoplasticgrowthonfreeedgeofboth
thevocalcordsatthejunctionofanterior1/3withposterior2/3.
Thisareaisparticularlyvulnerabletotraumaasthisistheareaof
maximumvibrationofthecord.
Themajorcauseisvoiceabuse,thereforeitismostcommonlyseen
insingers,actors,teachers,andhowkers.
Hoarsenessisthemostcommonsymptom.Vocalfatigueandpainin
neckonprolongedphonationareothersymptoms.
Treatment
Earlycasesofvocalnodulescanbetreatedconservativelyby
educatingthepatientinproperuseofvoice.Manynodules
especiallyinchildren,disappearwiththistreatment.
Surgeryisrequiredforlargenodulesorlongstandingnodulesin
adults.Microscopic(micorlaryngoscopic)excisionisthetreatmentof
choice.

954.Mostcommontumoroforopharynxis
a)Squamouscellcarcinoma
b)Adenocarcinoma
c)Melanoma
d)Salivaryglandtumors
CorrectAnswer-A
Ans.is'a'i.e.,Squamouscellcarcinoma[RefAbeloff'sClinical
Oncology6"lep.1059]
90?95%oftumorsintheoropharynxaresquamouscell
carcinomas,whereasothersareminorsalivaryglandtumors,
melanomas.
Themostcommonmanifestingsymptomsareanontenderneck
mass,dysphagia,otalgia,ora"hotpotato"voice.

955.Investigationofchoicefordiagnosing
submandibularglandductstonesis
a)Ultrasound
b)X-rayoffloorofmouth
c)Sialography
d)Sialoendoscopy
CorrectAnswer-A
Ans.is'a'i.e.,Ultrasound[RefChurchill'spocketbookof
surgeryp.133]
Investigationsforsalivaryglandstones
Ultrasoundistheinvestigationofchoice?asitpermitsassessment
ofthegland,ductsystemandcalculuswhichusuallyhasanacoustic
shadow.
'Floorofmouth'radiographicviewforsubmandibularcalculiin
Wharton'sductwhereultrasoundfindingsareequivocalor
unavailable.
Ifnostoneisseen,considersialography.
Sialoendoscopyisincreasinglyusedinthediagnosisandtreatment
ofsalivaryglandoutflowobstructiveconditions.

956.DeafnessinacaseofPaget'sdiseaseis
dueto
a)Retractionpockets
b)Otitis
c)Eightnerveinvolvement
d)Endolymphatichydrops
CorrectAnswer-C
Ans.is'c'i.e.,Eightnerveinvolvement[ReferNeurology
consult5"/ep.866]
Cranialnervecompressionistheusualcauseofdeafness.
ComplicationsofPaget'sdisease
FollowingcomplicationscanoccurinPaget'sdisease:-
1. Fracture:Arecommoninweightbearingbones
2. Cranialnervecompression:-Maycauseimpairedvision,facial
palsy,trigeminalneuralgiaordeafness.
3. Otosclerosis:-AnothercauseofdeafnessinPaget'sdisease.
4. Spinalcanalstenosisandnerverootcompression
5. Highoutputcardiacfailure
6. Osteoarthritis:ofHipandknee
7. Rarelyosteosarcoma

957.Membraneincisedduring
hemilaryngectomyis
a)Thyrohyoid
b)Cricothyroid
c)Aryepiglottic
d)Infralaryngeal
CorrectAnswer-B
Ans.is'b'i.e.,Cricothyroid[RefDhingraelep.310]
Indicationsandcontaindications
Idealforbulkylesionsofthemembranoustruevocalcord
Normalorslightlyimpairedvocalcordmobility
Noinvolvementofthesupraglottis

958.Snellen'schartisusedtotest:
a)Vision
b)Refraction
c)Presbyopia
d)Colourblindness
CorrectAnswer-A
Ans.Vision

959.InSnellen'schart,eyesubtendsanangle
ofhowmanyminuteswithletterson
Snellen'schart?

a)1minofarc
b)5minofarc
c)10minofarc
d)15minofarc
CorrectAnswer-B
Ans.is'b'i.e.,5minofarc
Indications:
Toprovideabaselinerecordingofvisualacuity(VA)
Toaidexaminationanddiagnosisofeyediseaseorrefractiveerror
Formedico-legalreasons
Equipment:
Multi-letterSnellenchart
EorCSnellenchartorachartwithillustrationsforpatientswho
cannotreadorspeak
Plainoccluder(notessential)
Pinholeoccluder
Torchorflashlight
Patient'sdocumentation
Procedure:
Atthegivendistance,eachlettersubtendsanangleof5minatthe
nodalpointoftheeye
Snellenchartisusedfordistantvision.
Thepatientshouldreadthechartatadistanceof6m.
Principle:



ltconsistsoflettersarrangedinlines,withprogressivelydiminishing
size.
Eachlettersubtendsanangleof5minutesatthenodalpointofan
eyewhenviewedfromitsrespectivedistance
Eachletterissoconstructedthatthewidth(ofeachstroke)subtends
anangleof1minute=MAR.
Interpretation:
Normalvisualacuityforfaris6/5
Bestvisualacuityforfaris6/3
MinimumrecordableVAonSnellen'schartis1/60.
Snellen'schart:

960.Placidodiscisusedfordiagnosingwhichofthefollowingcondition?
a)Uveitis
b)Keratoconus
c)Retinoblastoma
d)Retinaldetachment
CorrectAnswer-B
Placidodiscexaminationisusedtodiagnosekeratoconus.Inacaseofkeratoconus,
placidodiscexaminationshowsirregularitiesofthecircles.
Placido'skeratoscopicdisc:Itisadiscpaintedwithalternatingblackandwhitecircles.It
maybeusedtoassessthesmoothnessandcurvatureofcornealsurface.Normally,on
lookingthroughtheholeinthecenterofdiscauniformsharpimageofthecircleisseenon
thecornea.Distortionofthecirclesoccurwhenirregularitiesarepresentonthecorneal
surface.
Ref:ComprehensiveOphthalmologyByAKKhurana,4thedn,page119

961.Listersperimeterisusedfor
a)KineticVisualfieldtesting
b)StaticVisualfieldtesting
c)Bothkineticandstaticvisualtesting
d)Noneofthese
CorrectAnswer-A
Ans.is'a'i.e.,KineticVisualfieldtesting[RefKanski's8th/e
chapter10,p.325]
Staticperimetry:Amethodofassessingfields,usuallyautomated,in
whichthelocationofastimulusremainsfixed,withintensity
increaseduntilitisseenbythesubject(thresholdisreached)or
decreaseduntilitisnolongerdetected.
Kinetic(dynamic)perimetryisnowmuchlesscommonlyperformed
thanstaticperimetry.Astimulusofconstantintensityismovedfrom
anon-seeingareatoaseeingareaatastandardizedspeeduntilitis
perceived,andthepointofperceptionisrecordedonachart;points
fromdifferentmeridiaarejoinedtoplotanisopterforthatstimulus
intensity.
Stimuliofdifferentintensitiesareusedtoproduceacontourmapof
thevisualfield.Kineticperimetrycanbeperformedbymeansofa
manual(Goldmann)oranautomatedperimeterifthelatteris
equippedwithanappropriatesoftwareprogram.

962.WhatisthetypeofGoldmantonometry?
a)ApplanationTonometry
b)Dynamiccountourtonometry
c)Reboundtonometry
d)Impressiontonometry
CorrectAnswer-A
Ans.is'a'i.e.,ApplanationTonometry[RefAnatomy&
physiologyofeye2"1/e
p.79]
Measurementofintraocularpressure(10P)
MeasurementofIOPisdoneby:?
A)Manometry:-ItistheonlydirectmeasureofIOP.
B)Tonometry:-ItisanindirectmethodofmeasurementofIOP.
Followingtypesoftonometersarethere:-
1. Indentation(impression)tonometer:-Thesearethemostcommonly
usedtonometers.ExampleisSchiotztonometer.
2. ApplanationtonometerGoldmannapplanationtonometeristhemost
accuratetonometer.Othertypesofapplanationtonometersare
perkin'stonometer,pneumatictonometer,air-pufftonometer,Pulse
airtonometer,Tonopen

963.Spasmofaccommodationmimics
a)Myopia
b)Hypermetropia
c)Amblyopia
d)Presbiopia
CorrectAnswer-A
Ans.is'a'i.e.,Myopia[RefKhuranailth/ep.42]
Pseudomyopia
Spasmofaccommodationoccursduetoexcessivecontractionof
theciliarymuscles.
Thismakesthezonuleslooseandhencethelensbecomesmore
convex.
Thisleadstotheimagebeformedinfrontoftheretinatherebt
mimickingmyopia.
Thisiscalledpseudomyopia.

964.Subretinalhaemorrhageatthemaculain
myopiaisknownas?
a)Lacquercracks
b)FosterFuchsspot
c)Staphyloma
d)Macularretinoschisis
CorrectAnswer-B
Ans.is'b'i.e.,Fosterfuchsspot
Degenerativemyopia:
Refractiveerrormayincreaseupto20-25Dwithdegenerative
changesineye.
Temporalmyopiccrescentisafeatureofpathological/degenerative
myopia.
Itisawhitecrescentatthetemporalborderofthedisc.
Someofthemosttypicalfeaturesofdegenerativemyopiaare:
Vitreousliquefactionandposteriorvitreousdetachment
Peripapillaryatrophyappearingastemporalchoroidalorscleral
crescentsorringsaroundtheopticdisc
Latticedegenerationintheperipheralretina
Tiltingormalinsertionoftheopticdisc,usuallyassociatedwith
myopicconus
Thinningoftheretinalpigmentepitheliumwithresultingatrophic
appearanceofthefundus
Ectasiaofthescleraposteriorly(posteriorstaphyloma)
BreaksinBruch'smembraneandchoriocapillaris,resultinginlines
acrossthefunduscalled"lacquercracks"
FosterFuch'sspotinthemaculararea.

965.Dispersiveprismfunctionsfor
a)Splittinglightintodifferentwavelengths
b)Reflectinglight
c)Polarizinglight
d)None
CorrectAnswer-A
Ans.is'a'i.e.,Splittinglightintodifferentwavelengths[Ref
"TheDiscoveryoftheSpectrumofLight".]
"Prismsmaybeusedforpolarisingandreflectinglightbutdispersive
prismslikethecommonlyusedtriangularprismsplitthelightinto
differentcoloursbasedonwavelengths"
Inoptics,aprismisatransparentopticalelementwithflat,polished
surfacesthatrefractlight.Atleasttwooftheflatsurfacesmusthave
ananglebetweenthem.Theexactanglesbetweenthesurfaces
dependontheapplication.Thetraditionalgeometricalshapeisthat
ofatriangularprismwithatriangularbaseandrectangularsides.
Lightchangesspeedasitmovesfromonemediumtoanother(for
example,fromairintotheglassoftheprism).Thisspeedchange
causesthelighttoberefractedandtoenterthenewmediumata
differentangle(Huygensprinciple).Thedegreeofbendingofthe
light'spathdependsontheanglethattheincidentbeamoflight
makeswiththesurface,andontheratiobetweentherefractive
indicesofthetwomedia(Snell'slaw).Therefractiveindexofmany
materials(suchasglass)varieswiththewavelengthorcolorofthe
lightused,aphenomenonknownasdispersion.Thiscauseslightof
differentcolorstoberefracteddifferentlyandtoleavetheprismat
differentangles,creatinganeffectsimilartoarainbow.

966.Gobletcellsareseenin-
a)Cornea
b)Conjunctiva
c)Retina
d)Vitreous
CorrectAnswer-B
AnswerB.Conjunctiva
Gobletcellswithintheconjunctivalepitheliumare
specializedcellsthatsecretemucinsontothesurfaceofthe
eye.
Histologyofconjunctiva
Theepitheliumisnon-keratinizingandaroundfivecell-layers
deep.Mucin-secretinggobletcellsarelocatedwithintheepithelium,
andtheyaremostdenseinferonasally(nasal>inferior)andin
thefornices.
Thestroma(substantiapropria)consistsofrichlyvascularized,loose
connectivetissue.TheaccessorylacrimalglandsofKrauseand
Wolfringarelocateddeepwithinthestroma.
Conjunctiva-associatedlymphoidtissue(CALT)iscriticalinthe
initiationandregulationofocularsurfaceimmuneresponses.

967.Schwalbe'sringcorrespondsto:
a)Cornealendothelium
b)Descemet'smembrane
c)Schlemm'scanal
d)Ciliarybody
CorrectAnswer-B
Ans.Descemet'smembrane

968.Cornealepithelialrepairincludesallof
thefollowingphasesexcept
a)Cellproliferation
b)Cellmigration
c)Celladhesion
d)Cellfusion
CorrectAnswer-D
Ans.is'd'i.e.,Cellfusion[RefCornealEpithelialwound
healing:BJO1994,78;401-408]
Cornealepithelialrepair
Theprocessesinvolvedinthehealingofcornealepithelialwounds
canbedividedintothreedistinctcomponents:cellmigration,cell
proliferation,andcelladhesion.
Allthreecomponentsarepartofacontinuousprocessbutthe
contributionofeachcanvarydependingonthesizeanddepthofthe
woundandnatureofinjury.

969.Conjunctivalstainingisdonebyall
except
a)Fluoroscein
b)Indiaink
c)RoseBengal
d)Lissamine
CorrectAnswer-B
Ans.is'b'i.e.,Indiaink[RefCanJOphthalmol.2015
Aug;50(4):273-7.doi:10.1016/j.jcjo.2015.05.007.]
Commondyesusedforconjunctivalstaining:RoseBengal,
FluorosceinandLissamine.

970.Treatmentofmooren'sulceris?
a)Cornealgraft
b)Immunosuppressives
c)Topicalsteroids
d)Alloftheabove
CorrectAnswer-D
AlloftheaboveREF:Khurana4thedp.110
MOOREN'SULCER:
Severeinflammatoryperipheralulcerativekeratitis,chronic
serpigenousorrodentulcer
Treatment:
1.Topicalcorticosteroids
2.Immunosuppressiveswithsystemicsteroids,e.g.cyclosporine
3.Softcontactlens
4.Lamellarorfullthicknesscornealgrafting

971.Pseudogerontoxonisseenin
a)Vernalkeratopathy
b)Choroidalmelanoma
c)Trachoma
d)Retinoblastoma
CorrectAnswer-A
Ans.is'a'i.e.,Vernalkeratopathy[RefKanski8'1*Chap.5,p.
138]


972.A50yearoldmalepresentswith
cicatricialentopionofupperandlower
eyelid.Oneversionofuppereyelid,
linearconjunctivalscars-Arltlineare
seen.Whatisthediagnosis?

a)Trachoma
b)Springcatarrah
c)Ligneousconjunctivitis
d)Parinaudoculoglandularsyndrome
CorrectAnswer-A
Ans.is'a'i.e.,Trachoma[RefKanski8th/eChap.5,p.138]
Arltlineisafeatureoftrachoma.
SignsofTrachoma
Conjunctivalsigns:-Congestion,conjunctivalfollicles(boiled
sagograinlike),Papillaryhyperplasia,conjunctivalscarring(Arles
line),concretion.
Cornealsigns:-Superficialkeratitis,Herbertfollicles,Pannus,
Cornealulcer,Herbertpits,Cornealopacity.

973.Painisoutofproportiontosignsin
whichcornealulcer?
a)Herpessimplexkeratits
b)Acanthamoebakeratitis
c)Fungalkeratitis
d)Pneumococcalkeratitis
CorrectAnswer-B
Ans.is'b'i.e.,Acanthamoebakeratitis[RefKanski8thleChap.6,
p.197]
Inhuman,acanthamoebacauses:-(i)Keratitis;(ii)Granulomatous
encephalitis,(iii)Fulminantmeningoencephalitis
Patientpresentswithveryseverepain(whichisoutofproportionto
thedegreeofclinicalsigns),watering,photophobia,blurredvision
andblepherospasm.Onexamination,followingcharacteristic
featuresareseen:?
1. Initiallesions(Epitheliallesions):-InitiallytheAcanthamoeba
keratitisshowstypicalreticularpatternduetoradialkeratoneuritis
(Radialperineuritis).Atthisstageitiscommonlymistakenforherpes
simplexkeratitisbecauseofpseudodentriticepitheliallesion
(dentriticulcermorphology).
2. Advancedcases(Stromalinvolvement):-Overaperiodofweeks
stromalsignsdevelopwithcentralorparacentralringshapedlesion
withstromalinfiltrateandanoverlyingepithelialdefect,ultimately
presentingasringabscess.Theremayberadialperineuritis,
Wessely(inflammatory)ringandhypopyon.

974.Whichofthefollowingisnotafeatureof
fungalcornealulcer?
a)Fixedhypopyon
b)Ulcerwithsloughingmargins
c)Symptomsaremorepronouncedthansigns
d)FungalhyphaeareseenonKOHmount
CorrectAnswer-C
Ans.c.Symptomsaremorepronouncedthansigns

975.Oblateellipsoidappearanceofcorneais
seeninwhichofthefollowing
conditions?

a)PostmyopicLASIKsurgery
b)Withtheruleastigmatism
c)Bi-obliqueastigmatism
d)Obliqueastigmatism
CorrectAnswer-A
Ans.is'a'i.e.,PostmyopicLASIKsurgery[RefQualityof
vision:EssentialOpticsforthecataractandrefractivesurgeon,
chap.3,p.30]
Thecorneaisathreedimensionalprolateellipsoid,likeabulletora
tulip.Itissteeperinthecentreandflatterintheperiphery.
AfterrefractivesurgerieslikemyopicLASIK,corneaisconvertedto
oblateellipsoid,whichissteeperintheperipheryandflatterinthe
centre.

976.Whichofthefollowing
Glycosaminoglycansarenotpresentin
cornea

a)Keratinsulfate
b)Chondroitinsulfate
c)Chondroitin
d)Heparinsulfate
CorrectAnswer-D
Ans.is'd'i.e.,Heparinsulfate[RefKhuranaAnatomyand
PhysiologyofEyes,?die,chapter2,p.26)
GAGs(Glycosaminoglycans)orsocalledacid-
mucopolysaccharidesrepresent4-4.5%ofthedryweightofthe
cornea.
CorneacontainsthreemajorGAGfractionsnamely:Keratansulfate
(50%),chondroitinsulfate(25%)andchondroitin(25%)-present
exclusivelyincornea.
TheGAGarepresentintheinterfibrillarspaceofthecornealstroma
andaccountforthe`stromalswellingpressure'(normal-60mmHg)ie.
Itstendencytoimbibewaterandthusplaysanimportantroleinthe
maintenanceofcornealhydrationlevelandtransparency.An
abnormalaccumulationofGAGoccursinthecornealstromaofthe
patientsaffectedbytheinbornerrorsofGAGmetabolismknownas
mucopolysaccharidosis.

977.Irondepositionlineatedgeofpterygium
oncornealepitheliumisknownas?
a)Stocker'sline
b)KFRing
c)Fleischerring
d)Ferrysline
CorrectAnswer-A
Ans.is'a'i.e.,Stocker'sline[Ref:Khurana4'5/ep.80;Parson
21"/ep.181;Yanoff&DuckerOphthalmologyydlep


978.Geneforeyemorphogenesis
a)Pax-6
b)BMP-4
c)HOX-D13
d)HOX-A13
CorrectAnswer-A
Ans.is'a'i.e.,Pax-6
ThePax-6genelocusisatranscriptionfactorforvariousgenesand
growthfactorsinvolvedineyeformation.Pax-6isamastercontrol
geneforeyemorphogenesisandencodesforPairedboxprotein
Pax-6(alsocalledaniridiatypeIIproteinoroculorhombin).

979.Thepercentageofatropinepresentin
atropinedropsascycloplegicis:
a)0.5%
b)1%
c)4%
d)2%
CorrectAnswer-B
1%

980.Whichorderneuronisopticnerveinthe
visualpathway?
a)Firstorder
b)Secondorder
c)Thirdorder
d)Noneofthese
CorrectAnswer-B
Ans.is'b'i.e.,Secondorder[RefKhurana4th/ep.286-289;
Concisetextbookofphysiology2"/ep.336]
SensoryorgansPhotoreceptors(Rods&cones)
NeuronsoffirstorderAxonofbipolarcells(inRetina)
NeuronsofsecondorderAxonsofganglioniccell(Retinai.e.,
Opticdisc,Opticnerve,Opticchiasma,optictracts)
NeuronsofthirdorderAxonsfromnervecellsinlateralgeniculate
body(opticradiation)

981.Purkinjeimagetestisusedin
a)Keratomoter
b)Retinoscopy
c)Opticalcoherencetomography
d)Pachymeter
CorrectAnswer-D
Ans.is'd'i.e.,Pachymeter[RefElkington'sclinicaloptics3rd/e
chapter14,p.207]
Pachymetryisthemeasurementofcornealthickness.Pachymeters
employeitheropticalorultrasoundprinciples.
OpticalpachymetersusethePurkinje-Sansonimagesformedbythe
anteriorandposteriorsurfacesofthecornea(imagesIandII)to
measurecornealthickness,andthePurkinje-Sansonimagesformed
bytheposteriorsurfaceofthecorneaandtheanteriorsurfaceofthe
lens(imagesIIandIII)tomeasurethedepthoftheanterior
chamber.

982.

Whichofthefollowingisnotapartofuveal
a)Iris
b)Ciliarybody
c)Choroid
d)Retina
CorrectAnswer-D
Ans.is'd'i.e.,Retina[RefKhurana's2"/eChap.3,p.44]
Uvealtissueconstitutesthemiddlevascularcoatoftheeyeball
(Innercoatisretinaandoutercoatiscornea&sclera).Fromanterior
toposterioritcanbedividedintothreeparts:-Iris,ciliarybodyand
choroid.Irisandanteriorpart(2mm)ofciliarybody(parsplicata)are
consideredasanterioruvealtissues.
Posteriorpart(4mm)ofciliarybody(parsplana)andadjacent
choroidareconsideredasintermediateuvealtissue.Choroidis
consideredasposterioruvealtissue.Theentireuvealtractis
developmentally,structurallyandfunctionallyoneindividual
structure.

983.AllexceptonearetrueforScleromalacia
perforans
a)Itisnoninflammatoryscleritis
b)Itaffectsonlymales
c)Perforationoftheglobeisextremelyrare
d)Visionisunaffected
CorrectAnswer-B
Ans.is'b'i.e.,Itaffectsonlymales[RefKanski'sClinical
Ophthalmology8"Vechapter8,p.259]
Scleromalaciaperforans
Scleromalaciaperforans(5%ofscleritis)isaspecifictypeof
progressivescleralthinningwithoutinflammationthattypically
affectselderlywomenwithlongstandingrheumatoidarthritis,buthas
alsobeendescribedinassociationwithothersystemicdisorders.
Despitethenomenclature,perforationoftheglobeisextremelyrare
asintegrityismaintainedbyathinlayeroffibroustissue.Differential
diagnosisisfromtheinnocuousscleralhyalineplaqueandsenile
scleromalacia.

984.TransportofAscorbicacidtolensisdonebywhichofthefollowing?
a)Myoinositol
b)Choline
c)Taurine
d)Na-KATPase
CorrectAnswer-D
Thelensepitheliumistheprincipalsiteofenergyproductionofthelensthatisusedfor
transportofinorganicionsandaminoacidsbyanactiveprocessinvolvingNaandK
activatedATPase.
Ref:HistopathologyofPreclinicalToxicityStudies:InterpretationandRelevance...By
PeterGreaves,PeterGreaves(M.B.,Ch.B.),2007,Page896;TransportofvitaminCinthe
lens,CurrEyeRes.1987Jul;6(7):885-96;Adler'sPhysiologyoftheeye10thEdition,
Page131;AscorbicacidandtheEye,AmJClinNutr1991;54:1198S-1202S.

985.Bluedotcataractiscausedby
a)Diabetes
b)Wilson'sdisease
c)Atopicdermatitis
d)Chalcosis
CorrectAnswer-C
Ans.is'c'i.e.,Atopicdermatitis[RefParson's21"/ep.259]
Bluedotcorticalcataractiscausedbymyotonicdystrophyand
atopicdermatitis.

986.Whichofthefollowingiscomplicationof
prolongeduseofcorticosteroideye
drops?

a)PosteriorSubcapsularcataract
b)Nuclearcataract
c)Capsularcataract
d)Corticalcataract
CorrectAnswer-A
Ans.is'a'i.e.,PosteriorSubcapsularcataract
Causesofposteriorsubcapsularcataract:

Myotoicdystrophy
Down'ssyndrome Ionizingradiation
Wilson'sdisease
Corticosteroids
Galactosemia
DM
Busulfan
Senilecataract
Infrared/heatcataract
Chloroquine
Trauma
(glass-blower'sorglassworker) Atopicdermatitis

987.Falseabouttreatmentofcataractin
childrenis
a)ECCEisthetreatmentofchoice
b)Incaseofbilateralcataractimparingvisionsurgerymustbe
doneby4-6weeksofage
c)Lensectomyisnotoneofthemethodsofextracapsular
extraction
d)Intraocularlensimplantationmustbedoneonlyafter2yearsof
age
CorrectAnswer-C
Ans.is'c'i.e.,Lensectomyisnotoneofthemethodsof
extracapsularextraction[RefTextbookofpediatriccataract
surgeryp.194]
Thecriticalperiodfordevelopingthefixationreflexinbothunilateral
andbilateralvisualdeprivationdisordersisbetween2-4monthsof
age.Anycataractdenseenoughtoimpairvisionmustbedealtwith
beforethisageandtheearliestpossibletimeispreferred.

988.Whatisthenewadvanceincataract
surgery?
a)FemtosecondLaser
b)NeodymiumLaser
c)NanosecondLaser
d)PicosecondLaser
CorrectAnswer-A
Ans.is'a'i.e.,Femtosecondlaser[RefFemtosecondlaser-
assistedcataractsurgery:KendallE.Donaldson,
RosaBraga-Mele,FlorenceCabot,fortheASCRSRefractive
CataractSurgerySubcommittee]
Femtosecondlaser-assistedcataractsurgeryprovidessurgeonsan
excitingnewoptiontopotentiallyimprovepatientoutcomesand
safety.
Inthissurgeryafemtosecondlaserisusedforvariousstepslike
lensfragmentationandcapsulotomy.
Althoughtheresultshavebeengoodthetechnologyiscurrentlyvery
expensive.

989.Whichofthefollowingstepisnotdone
duringphacoemulsificationsurgeryfor
cataract?

a)Irrigationanddrainageofcortex
b)Continuouscurvilinearcapsulorrhexis
c)FoldableIOLimplantation
d)Sclerocornealtunnel
CorrectAnswer-D
Ans.is'd'i.e.,Sclerocornealtunnel[RefPhacoemulsificationby
vajpayee]
Thestepsinphacoemulsificationinclude:(i)Corneoscleralincision,
(ii)Continuouscurvilinearcapsulorrhexis,(iii)Hydrodissectionand
hydrodelineation,(iv)Emulsificationandaspirationofnucleusand
thencortex,(v)FoldableIOLimplantationinposteriorchamber.

990.Phacoemulsificationuses
a)Highfrequencysoundwaves
b)Infraredwaves
c)Ultravioletrays
d)Noneofthese
CorrectAnswer-A
Ans.is'a'i.e.,Highfrequencysoundwaves[RefKanski's8th/e
chapter9,p.281]
InPhacoemulsification,lensnucleusisemulsifiedusinghigh
frequencysoundwaves.

991.Postoperativecomplicationsofcataract
areallexcept?
a)Aftercataract
b)Endophthalmitis
c)Glaucoma
d)Scleritis
CorrectAnswer-D
Ans.is'd'i.e.,Scleritis[RefOphthalmologybyDuker2"d/ep.
484]
Therearesomanycomplicationsofcataractsurgery,Hereare
onlyimportantones:?
Aftercataract(opacificationofcapsule)
Retinaldetachment
Vitreousprolapse&loss
Neovascularglaucoma
Cystoidmacularedema
Anterioruveitis(iridocyclitis)
Endophthalmitis
Irisprolapse
Aphakicglaucoma
Striakeratopathy&Pseudophakicbullouskeratopathy
Fibrous&endothelialgrowth
Cornealendothelialdamage

992.Whichofthefollowingisthemost
importantfactorinthepreventionofthe
endophthalmitisincataractsurgery?

a)Preoperativepreparationwithpovidoneiodine
b)Oneweekantibiotictherapypriortosurgery
c)Trimmingofeyelashes
d)Useofintravitrealantibiotics
CorrectAnswer-A
Ans.Preoperativepreparationwithpovidoneiodine

993.Hemeralopiaisseenin
a)Retinaldetachment
b)Retinitispigmentosa
c)Opticneuritis
d)Subcapsularcataract
CorrectAnswer-D
Ans.is'd'i.e.,Subcapsularcataract[RefKanski8th/eChap.9,
p.270]
Hemeralopiaistheinabilitytoseeclearlyinbrightlight(alsoknown
asdayblindness)andistheexactoppositeofnyctalopia(night
blindness).
Subcapsularcataract
Anteriorsubcapsularcataractliesdirectlyunderthelenscapsule
andisassociatedwithfibrousmetaplasiaofthelensepithelium.
Posteriorsubcapsularopacityliesjustinfrontoftheposterior
capsuleandhasagranularorplaque-likeappearanceonobliqueslit
lampbiomicroscopy,buttypicallyappearsblackandvacuolatedon
retroillumination;thevacuolesareswollenmigratorylensepithelial
cells(bladderorWedl),similartothosecommonlyseen
postoperativelyinposteriorcapsularopacification.Duetoitslocation
atthenodalpointoftheeye,aposteriorsubcapsularopacityoften
hasaparticularlyprofoundeffectonvision.
Patientsarecharacteristicallytroubledbyglare,forinstancefromthe
headlightsofoncomingcars,andsymptomsareincreasedby
miosis,suchasoccursduringnearvisualactivityandinbright
sunlight(dayblindness).

994.Achildhasgotacongenitalcataract
involvingthevisualaxiswhichwas
detectedbytheparentsrightatbirth.
Thischildshouldbeoperated:

a)Immediately
b)At2monthsofage
c)At1yearofagewhentheglobebecomesnormalsized
d)After4yearswhenentireocularandorbitalgrowthbecomes
normal
CorrectAnswer-A
Ans.A[Immediately]
Congenitalcataract-Timingofsurgery
1. Bilateraldense-cataractrequiresearlysurgery(i.e.by6weeksof
age)topreventthedevelopmentofstimulusdeprivationamblyopia
2. Bilateralpartial-cataractmaynotrequiresurgeryuntillaterifatall,
incasesofdoubt,itmaybeprudenttodefersurgerymonitorlens
opacity,andvisualfunctionandintervenelaterifvisiondeteriorates.
3. Unilateraldense-cataractmeritsurgentsurgery(withindays)
followedbyaggressiveanti-amblyopiatherapythecataractis
detectedafter16weeksofagethensurgerycanbedelayedlittle
becauseamblyopiaisrefractory
4.Partialunilateral-cataractcanusuallybeobservedortreated
nonsurgicallywithpupillarydilatationandpossiblypart-time
contralateralocclusiontopreventamblyopia"Thecriticalperiodof
developingthefixationreflexesinbothunilateralandbilateralvisual
deprivationdisordersisbetween2and4monthsofage,any
cataractdenseenoughtoimpairvisionmustbedealtwithbeforethis

ageandtheearliestpossibletimeispreferred"

995.Whichofthefollowingisagooddyeis
usedforlensincataractsurgery?
a)TrypanBlue
b)Fluoroscein
c)Indiaink
d)None
CorrectAnswer-A
Ans.is'A
Trypanbluedyeforanteriorsegmentsurgeries
Trypanbluehasbeenusedasanadjunctforimprovingvisualization
oftheanteriorcapsuleduringphacoemulsificationofmaturewhite
cataractsforthepastfewyears.
Surgeonshavelonguseddyeslikeindocyaninegreen,fluorescein,
andtrypanbluetostaintheanteriorcapsuleinordertofacilitatethe
surgicalprocedure.
studiescomparingthese3dyeshaveconcludedthattrypan
blueprovidessignificantlymoreintensivestainingofthe
anteriorlenscapsulethantheothers
Trypanblueiseasiertousethanindocyaninegreen,becauseit
comesinapremixedsolution,anditisavailableatamore
economicalprice.


996.Whichofthefollowingisalayerbetween
choroidandretina?
a)Bruch'smembrane
b)Descemet'smembrane
c)Photoreceptors
d)Ganglioncelllayer
CorrectAnswer-A
Ans.is'a'i.e.,Bruch'smembrane[RefParson's21"/ep.321]
Bruch'smembraneistheinnermostlayerofthechoroid.Itisalso
calledthevitreouslamina,becauseofitsglassymicroscopic
appearance.Itis2-4thick.Itliesbetweenchoroidandtheretina.

997.100dayglaucomaiscausedby
a)CRVO
b)CRAO
c)Buphthalmos
d)Agerelatedmaculardegeneration
CorrectAnswer-A
Ans.is'a'i.e.,CRVO
100-dayglaucomaistypicallyseeninIschemicCRVO.
Followingcentralretinalveinocclusion,flame-shapedhaemorrhages
developinthenervefiberlayeroftheretina,especiallyaroundthe
opticdisc,asaresultofthehighintravascularpressurethatdilates
theveinsandcollateralvessels.
Edemaoftheopticdiscandretinaoccurbecauseofimpaired
absorptionofinterstitialfluid.
Visionisgenerallypoorbutmayrecoversurprisinglywell,
consideringtheseverityofthefunduscopicchanges.
Intractableclosed-angleglaucoma,withseverepainandrepeated
haemorrhages,commonlyoccurs2to3monthsaftercentralretinal
veinocclusion(100-dayglaucoma';'thromboticglaucoma'),owingto
neovascularizationoftheirisandadhesionsbetweentheirisandthe
anteriorchamberangle(peripheralanteriorsynechiae).

998.KeethWagnerclassificationisfor
a)Hypertensiveretinopathy
b)Diabeticmaculpathy
c)CRVO
d)CRAO
CorrectAnswer-A
Ans.is'a'i.e.,Hypertensiveretinopathy[RefRobartW.Schrier
8"/ep.1377]


999.Allareseeninnon-proliferativediabetic
retinopathyexcept?
a)Microaneurysm
b)Neovascularization
c)Hardexudates
d)Macularedema
CorrectAnswer-B
Ans.is'b'i.e.,Neovascularization
ClassificationofDiabeticretinopathy
Nonproliferative
Proliferative
Backgroundretinopathy
1. Microaneurysm
2. Dotandblothemorrhage(deep Neovasculorizationofthedisc
hemorrhage)
(NVD)
3. Hardexudate
1. Neovasculaizationelsewhereinthe
4. Macularedema
retina(NVE)
B)Preproliferative
2. Viffeoushemorrhage
retinopathy
3. Fibrovascularproliferation
1. Cotton-woolspots(soft
4. Trsctionretinsldetachment
exudates)
5. Irissurfaceneovascularization
2. Venousbeading
(rubeosisiridisor
3. Extensivehemorrhage
neovascularglaacoma)
4. lntraretinalintravascularabn
ormalities(IRMA

1000.Allofthefollowingarethecausesof
exudativeretinaldetachmentexcept:
a)Retinopathyoftoxaemiaofpregnancy
b)Retinopathyofprematurity
c)ExudativeretinopathyofCoats
d)Sympatheticophthalmia
CorrectAnswer-B
Ans.Retinopathyofprematurity

1001.Inwhichofthefollowing,intraocular
pressureisveryhighandinflammation
isminimum?

a)Glaucomatocycliccrisis
b)Acuteiridocyclitis
c)Hypetensiveuveitis
d)Angleclosureglaucoma
CorrectAnswer-A
Ans.is'a'i.e.,Glaucomatocycliccrisis[RefKhurana4th/ep.160]
Glaumatocycliccrisis
Glamatocycliticcrisis(Posner-Schlossmansyndrome)isa
unilateralrecurrentnon-granulomatousiritisthatisassociatedwith
anelevatedocularpressureduringtheattacks.Thisself-limiting
conditiontendstooccurinpersonsduringthethirdtosixthdecade
andthevisualfields,theopticnervehead,andanteriorchamber
anglearenormal.
Amildinflammatoryreactionisveryrarelypresentasevidencedby
afewkeraticprecipitatesontheposteriorsurfaceofthecornea.The
causeoftheglaucomaremainsunknown,butatrabeculitisis
suspected.Manypatients(55%)subsequentlydevelopopenangle
glaucoma

1002.CellbodiesofMuller'sCellsarepresent
inwhichlayerofretina?
a)Innerlimitingmembrane
b)Outernuclearlayer
c)Retinalpigmentepithelium
d)Ganglioncelllayer
CorrectAnswer-A
Ans.is'a'i.e.,Innerlimitingmembrane[RefHistologyofthe
Eye,editedbyWilliamKrause,Dept.PathologyandAnatomical
science,UniversityofMissouriSchoolofMedicine]
Innerlimitingmembrane-basementmembraneelaboratedbyMuller
cells.
Nervefibrelayer-axonsoftheganglioncellnuclei(notethatathin
layerofMullercellfootplatesexistsbetweenthislayerandtheinner
limitingmembrane).
Ganglioncelllayer-containsnucleiofganglioncells,theaxonsof
whichbecometheopticnervefibresformessagesandsome
displacedamacrinecells.
Innerplexiformlayer-containsthesynapsebetweenthebipolarcell
axonsandthedendritesoftheganglionandamacrinecells.
Innernuclearlayer-containsthenucleiandsurroundingcellbodies
(perikarya)oftheamacrinecells,bipolarcellsandhorizontalcells.
Outerplexiformlayer-projectionsofrodsandconesendinginthe
rodspheruleandconepedicle,respectively.Thesemakesynapses
withdendritesofbipolarcells.Inthemacularregion,thisisknownas
theFiberlayerofHenle.
Outernuclearlayer-cellbodiesofrodsandcones.
Externallimitingmembrane-layerthatseparatestheinnersegment

portionsofthephotoreceptorsfromtheircellnucleus.
Layerofrodsandcones-layerofrodcellsandconecells.
Retinalpigmentepithelium-singlelayerofcuboidalcells.Thisis
closesttothechoroid.

1003.Retinaltearsseenmostcommonlyseen
in?
a)Primaryretinaldetachment
b)Secondaryretinaldetachment
c)Tractionalretinaldetachment
d)Exudativeretinaldetachment
CorrectAnswer-A
Ans.is'a'i.e.,Primaryretinaldetachment
Retinaldetachmentisadisorderofeyeinwhichretinapeelsaway
fromitsunderlyinglayerofsupporttissue.Usuallythereis
separationbetweentheneuroepithelium(neurosensoryepithelium)
andthepigmentedlayer.
Theretinalseparationisdividedinto:
1. Primary:-Rheugmatogenousretinaldetachment.
2. Secondary:-Tractionalretinaldetachmentandexudativeretinal
detachment.
Rhegmatogeneousretinaldetachment
Thisisthecommonesttypeofretinaldetachment.Thisisduetoa
retinalbreak/tear/holewhichallowstheliquidvitreoustoseepsinto
thesubretinalspaceandseparatesthesensoryretinafromthe
pigmentaryepithelium.
Predisposingfactorsinclude:(i)Myopia,(ii)Previousintraocular
surgery:cataractextraction,(aphakia)orpseudoaphakia);(iii)
Trauma;(iv)Retinaldegeneration(Latticedegeneration;Snailtrack
degeneration,Senileordegenerativeretinoschisis.
Tractionalretinaldetachment
Itisduetopullingontheretinausuallyfromfibro-vascularbandin
thevitreouscavity,i.e.,vitroretinalband.

Exudativeretinaldetachment(solidretinaldetachment)
Itoccursduetotheretinabeingpushedawaybyaccumulationof
fluidoraneoplasmbeneaththeretina.Thistypeofdetachmentis
causedbyinflammatorydisordersorbytumors.

1004.Anelderlymalewithheartdisease
presentswithsuddenlossofvisionin
oneeye;examinationrevealscherryred
spot;diagnosisis:

a)Centralretinalveinocclusion
b)Centralretinalarteryocclusion
c)Amaurosisfugax
d)Acuteischemicopticneuritis
CorrectAnswer-B
Bi.e.CentralRetinalArteryOcclusion
Centralretinalarteryocclusioncharacteristicallypresentswith
suddenpainlesslossofvision,cherryredspot&cattle-truck
appearanceQ
(ofretinalveinsusually).
Sourceofembolifromcarotidartery&heartdisease,andthrombus
fromarteriosclerosisalongwithhypertension&arteritisare
predisposingfactors.

1005.Whichofthefollowingistrueabout
signsofangleinclosureglaucoma
except

a)Verticalsemidialtedpuil
b)Edematouscornea
c)Multipleirisnodules
d)Edematousandhyperemicopticdisc
CorrectAnswer-C
Ans.is'c'i.e.,Multipleirisnodules[RefParson's21"/ep.290]
Clinicalfeaturesofangleclosureglaucoma
Conjunctivaischemosedandcongested(bothconjunctival&ciliary
vesselsarecongested).
Corneaisoedematousandinsensitive
Anteriorchamberisveryshallow&withaqueousflareAngleof
anteriorchamberisclosed(ongonioscopy)
Irisisdiscoloured
Pupilissemidilated,verticallyovalandfixed.Itisnonreactiveto
bothlightandaccommodation.
IOPismarkedlyraised(40-70mmHg)
Opticdiscisoedematous&hyperemic
Felloweyeshowsshallowanteriorchamberandanarrowangle.

1006.Whichofthefollowingisfalse
statementaboutvitreous?
a)Anatomically,vitreousispresentinanteriorsegment
b)Vitreouslargelycontainwaterandhyaluronicacid
c)Strongestattachmentofvitreousbaseisatoraserrata
d)Vitreousisattachedanteriorlytothelens
CorrectAnswer-A
Ans.is'a'i.e.,Anatomically,vitreousispresentinanterior
segment[RefKanski8thieChap.17,p.722]
Vitreousispresentinposteriorsegmentofeye.
Vitreoushumorisajellylikefluidliquidthatfillsmost(80%)ofthe
eye(fromthelensback,i.e.,intheposteriorsegment).Thevitreous
consistslargelyofwater(99%),anetworkofcollagenfibrils,large
moleculesofhyaluronicacid,peripheralcells(hyalocytes),and
mucopolysaccharides,formingagel-likematerial.
Thereisapotentialspacebetweenvitreousandretina,called
subhyaloidspace.Asweage,vitreouschangesfromageltoaliquid
andgraduallyshrinksseparatingfromtheretina.Thisiswhen
peoplestartseeingfloaters(blackspotsinfrontoftheeye).

1007.Phacodonesisisseeninallexcept
a)Traumaticinjurytotheeye
b)Hypermaturecataract
c)Pseudoexfoliation
d)Diabetesmellitus
CorrectAnswer-D
Ans.is'd'i.e.,Diabetesmellitus[RefKanski8thiep.300]
Phacodonesisisthetremulousnessorvibrationofthelenswitheye
movement.
Itisseenin:
1. Trauma
2. Pseudoexfoliationsyndrome
3. Hypermaturecataract
4. Ectopialentis

1008.Onsetofstereopsisoccursattheage
of:
a)3to5months
b)1to2years
c)5years
d)7years
CorrectAnswer-A
Ans.3to5months

1009.Homonymoushemianopiatypeof
visualfielddefectisseeninallexcept?
a)Lateralgeniculatebody
b)Totalopticradiation
c)Optictract
d)Opticchaisma
CorrectAnswer-D
Ans.is'd'i.e.,Opticchaisma

1010.Normalvisionwithabsenceofdirect&
consensuallightreflex,whichnerveis
involved?

a)Optic
b)Oculomotor
c)Trigeminal
d)Abducens
CorrectAnswer-B
Ans.is'b'i.e.,Oculomotor
Thisquestioncanbesolvedbysimplebasicknowledge:-
Amongthegivenoptionsonlyoiticnerveandocculomotornewes
arerelatedtolightreflex'
aInopticnerveinjuryvisionisalsoimpaired(visionisnormalinthe
question)'
nThusanswerisoculomotornerve
Whenlightisshonetoone(e.g.left)eye.leftopticnervecanies
afferentimpulsetobrainandfrombrainefferent
impulsetoipsiplateral(left)eyecomesthroughipsilateral(left)
occulomotornerye(fordirectlightreflex)andefferent
forcontralatcral(right)eyecomesthroughcontralateral(right)
occulmotornerveforconsensuallightreflex'When
lightisshonetoother(right)eyc,rightopticnervecarriesafferent
impulsetobrainandfrombrain,efferentimpulseto
righteye(fordirectlightreflex)comesthroughrightocculmotor
nerveandeffcrentfbrlefteye(forconsensuallight
reflex)comesthroughleftocculomotornerve'So:-
OpticneweisresponsibtefordirectlightreJlexinipsilateraleyeand

consensuallightreflexforcontralateral
eye.(lnaboveexample,afferentforrightsideddirectandleftsided
consensuallightreflexisthroughrightoptic
nerve;andatl-erentforleftsideddirectandrightsidedconsensual
lightreflexisthroughleftopticnerve)'
Occulomotornerveisresponsiblefordirectandconsensuallight
reflexinthesameeye'(lnaboveexample'
efferentforrightsideddirectasrvellasconsensuallightreflexis
throughrightocculmotornerveandef'ferentfor
leftsideddirectaswellasconsensuallightreflexisthroughleft
occulomotornerve)'
Incompleteopticnervelesionofoneside(Amaurosislight
reflexortotalafferentpupillaryfect)
Theipsilateraldirectreflexislost
Theipsilateralconscnsualreflexisintact
Thecontralateraldirectreflexisintact
Thecontralateralconsensualreflexislost
Inocculomotornervelesionofoneside(efferent
pupillarydefect
Theipsilateraldirectreflexislost
Theipsilateralconsensualreflexislost
Thecontralateraldirectreflexisintact
Thecontralateralconsensualreflexisintact

1011.Whichofthefollowingistrueabout
divergentsquint
a)Itisalsocalledexotropia
b)Itismorecommonthanconvergentsquint
c)Itisafeatureof6thnervepalsy
d)Allofthefollowing
CorrectAnswer-A
Ans.is'a'i.e.,Itisalsocalledexotropia[RefPediatric
strabismus4th/ep.218-224]
Divergentsquintisalsocalledasexotropia.
Itlesscommonthanconvergentsquint(esotropia).
6thnervepalsycausesconvergentsquint.

1012.Crossedeyefixationispositivein-
a)Esotropia
b)Exotropia
c)Hypertropia
d)Hypotropia
CorrectAnswer-A
Ans.is'a'i.e.,Esotropia[RefInternet]
Crossfixationistheuseoftherighteyetoviewtheleftvisualfield
andtheuseofthelefteyetoviewtherightvisualfield.Thisbehavior
isverycommoninchildrenwithinfantileesotropia.Crossfixation
oftencausestheappearanceofnotlookingdirectlyatatargetand
parentsoftenwonderifvisionisreduced.

1013.Partialptosisisoculomotornerveinjury
isduetointact-
a)Supplyfromoppositeoculomotornerve
b)Sympatheticinnervation
c)Parasympatheticinnervation
d)Actionoforbicularisoculi
CorrectAnswer-B
Ans.is'b'i.e.,Sympatheticinnervation[RefTextbookof
ophthalmicreconstructivesurgeriesp.786]
Motornervesupplyoflidisthroughthreesources:?

1. Facialnervesupplyingorbicularisoculi,(closingoflid)
2. Oculomotorsupplyinglevatorpalpebraesuperioris;
3. Sympatheticfibressupplyingmuller'smuscle.
4. BothLPSandMullermusclefunctiontoretractthelid,henceonly
partialptosisisseeninoculomotornervepalsy.

1014.Painfuleyemovementisafeatureof:
a)Iridocyclitis
b)Papilledema
c)Cornealulcer
d)Vernalcatarrh
CorrectAnswer-A
Anterioruveitis,alsoknownasiridocyclitisandiritis,istheinflammationoftheirisand
anteriorchamber.Anywherefromtwo-thirdsto90%ofuveitiscasesareanteriorinlocation.
Injection,photophobia,pain,andblurredvisionusuallyaccompanyiritis(anterioruveitisor
iridocyclitis).
Ref:BravermanR.S.(2012).Chapter16.Eye.InW.W.Hay,Jr.,M.J.Levin,R.R.
Deterding,J.J.Ross,J.M.Sondheimer(Eds),CURRENTDiagnosis&Treatment:
Pediatrics
,21e.

1015.Retinoblastomasshowallofthe
followingexcept
a)Smallroundcells
b)Necrosis
c)PseudorosettesandFleurettes
d)None
CorrectAnswer-D
Ans.'d'isNone(Allareseen)[RefKhurana4thiep.281]
Histologyofretinoblastomas
Thetumorarisesfromsmallroundcellswithlargenuclei,i.e.,itisa
tumorofagroupcalledsmallroundbluecelltumors.
Microscopicfeaturesofawelldifferentiatedtumorinclude
Flexnerwintersteinerrosettes(specificforretinoblastoma),Homer
Wrightrosettes,pseudorosettesandfleurettesformation.

1016.Whichofthefollowingismostcommon
visualdefectinpapilloedema
a)Amourosisfugax
b)Homonymoushemianopia
c)Homonymousquadrantopia
d)Glare
CorrectAnswer-A
Ans.is'a'i.e.,Amourosisfugax[RefClinicalopthalmology
2"d%p.349,350]
InitiallythesymptomsofpapilloedemaareduetoincreasedICT,e.g.
headache,nausea,projectilevomitingandpapilloedema.
Visionisnormalinitially.
In25%ofpatientsvisualsymptomsoccuronlyinadvancedsevere
papilloedema,whenopticatrophysetsin.
Typically,thereisrecurringbriefepisodes(transient)ofvisual
obscurations(Amaurosisfugax)lastinglessthen30seconds,in
whichvisionturnsgreyorblacks,sometimesdecribedasifaveil
hasfallenovertheeyes.
Thesymptomsusuallyaffectbotheyesatonceaspapilloedemais
bilateral.
Centralvisionisaffectedlateinthedisease.
Initiallythereisenlargementofblindspotandprogressive
contractionofthevisualfield(visualfieldbecomessmaller).
Completeblindnesssetsineventually.
Thus,characteristicallythereisgraduallyprogressivepainlessloss
ofvision

1017.Whatisthetreatmentofmeibomianitis
?
a)Cleansingthelidedges
b)Applicationofmoistheat
c)Localantibiotics
d)Alloftheabove
CorrectAnswer-D
Ans.is'd'i.e.,Alloftheabove[RefFosterCS.Theeyeinskin
andmucousmembranedisorders.In:TasmanW,JaegerEA,
eds.Duane'sOphthalmology.15thed.Philadelphia,Pa:
LippincottWilliams&Wilkins;2009:chap27]
Meibomianitis
Meibomianitisisinflammationofthemeibomianglands,agroupof
oil-releasing(sebaceous)glandsintheeyelids.
Causes
Anyconditionthatincreasestheoilysecretionsofthemeibomian
glandswillallowexcessoilstobuildupontheedgesoftheeyelids.
Thisallowsfortheexcessgrowthofbacteriathatarenormally
presentontheskin.
Theseproblemscanbecausedbyallergies,hormonechanges
duringadolescence,orskinconditionssuchasrosaceaandacne.
Meibomianitisisoftenassociatedwithblepharitis,whichcancausea
buildupofadandruff-likesubstanceatthebaseoftheeyelashes.
Insomepeoplewithmeibomianitis,theglandswillbepluggedso
thatthereislessoilbeingmadeforthenormaltearfilm.These
peopleoftenhavesymptomsofdryeye.

1018.Blowoutfractureoforbitleadsto
fracturein
a)Floor
b)PosteriorMedialwall
c)Lateralwall
d)Roofoftheorbit
CorrectAnswer-B
Ans:B
1.Ablowoutfractureoftheorbitalflooristypicallycausedbya
suddenincreaseinorbitalpressurebyastrikingobjectwhichis
greaterthan5cmindiameter,greaterthantheoritalsindiameter
suchasafistortennisball.
2.Thefracturemostfrequentlyinvolvestheflooroftheorbitalong
thethinbonecoveringtheinfraorbitalcanal.
3.Thefloorconsistsofthreebones:Zygomatic,Maxillaryand
Palatine.
4.Theposteromedialportionofthemaxillaryboneisrelatively
weakandmaybeinvolvedina`blowout'fracture.
5.FractureoftheRoofoftheorbitknownas`blowin'fractures.
6.Mostmedialwallorbitalfracturesareassociatedwithfloorfracture

1019.Mostcommonsiteofbasalcell
carcinomaoftheeyeis
a)Eyelid
b)Conjunctiva
c)Cornea
d)Lacrimalapparatus
CorrectAnswer-A
Ans.is'a'i.e.,Eyelid[RefRenuJogi4th/ep.420;Khurana4th/e
p.360]
Basalcellcarcinomaisthecommonestmalignanttumorofthelids
(90%)usuallyseeninelderlypeople.
Itislocallymalignantandinvolvesmostcommonlylowerlid(50%)
followedbymedia!canthus(25%),upperlid(10-15%)andouter
canthus.
"Basalcellcarcinomaisseeninthelowerlidneartheinnercanthus
usually

1020.Vitrectomyshouldbeconsideredifthe
vitreoushaemorrhageisnotabsorbed
within:

a)1month
b)3months
c)6months
d)2months
CorrectAnswer-B
Ans.3months

1021.Ankyloblepheronisdefinedas
a)Adhesionofthelidtotheeyeballs
b)Adhesionofthelidmarginstoeachother
c)Inturnedeyelash
d)Inflammationofthelidmargin
CorrectAnswer-B
Ansis'b'i.e.,Adhesionofthelidmarginstoeachother[Ref:
Kanski8thlep.52]
Disordersoftheeyelids
Blepharitis:-Chronicinflammationoflidmargin.
Blephritisacarica:InflammationoflidmargincausedbyDemodex
folliculorum.Madarosis:-Lossofeyelashes.
Trichiasis:-Misdirectedeyelasheswhichrubagainsttheeyeballin
normalpositionoflidmargin.Distichiasis:-anabnormalextrarowof
ciliatakingplaceofmeibomianglands.
Entropion:-Inwardrollingorinturningoflidmargin.
Ectropion:-Outrollingoroutwardturningoflidmargin.
Symblepharon:-Adhesionofthelidstoeyeball.
Ankyloblepharon:-Adhesionofmarginsoftwoeyelids.
BlepharophimosisPalpebralfissureappearstobecontractedatthe
outercanthus.Lagophthalmos:-Incompleteclosureofthepalpebral
aperturewheneyesareshut.Tylosis:-Thickeningoflidmargin.
Basalcellcarcinoma:-Mostcommonmalignanttumorofeyelid
Clobomaoflid:-Triangulargapinnasalsideofupperlid

1022.Theglobeisdisplacedtowhichsidein
lacrimalglandtumour?
a)Inferotemporal
b)Inferonasal
c)Superotemoral
d)Nasal
CorrectAnswer-B
Ans.is'b'i.e.,Inferonasal[RefBernardiniFP,DevotoMH,
CroxattoJO.Epithelialtumorsofthelacrimalgland:anupdate.
CurrOpinOphthalmol2008;19:409-13]
Alllacrimalglandtumorstypicallysharethefollowingsymptoms:
facialasymmetryduetodisplacementoftheglobe,diplopia,ptosis,
limitedocularmotility,andenlargementofthelacrimalgland.
Epitheliallesionstendtodevelopmostlyintheorbitallobeofthe
lacrimalglandandareprincipallyunilateral.
Theinitialsignisusuallyinferiorandnasaldisplacementoftheglobe
andproptosis,duetothesuperotemporallocationofthelacrimal
glandintheanterioraspectoftheorbit.

1023.'D'shapedpupilisseenin
a)Glaucoma
b)Dislocationoflens
c)Iridodialysis
d)Iridocyclitis
CorrectAnswer-C
Ans.is'c'i.e.,Iridodialysis[RefKhurana4thiep.404]
Iridodialysisisdetachmentofirisfromitsrootattheciliarybody.It
resultsinDshapedpupilandalackbiconvexareaseenatthe
periphery.

1024.Investigationofchoiceforopticneuritis
is?
a)MRIBrainandorbit
b)CtscanBrainandorbit
c)Vitreousbiopsy
d)Electooculogram
CorrectAnswer-A
Ans.is'a'i.e.,MRIBrainandorbit[RefKanski8th/eChap.19,p.
784]
MagneticResonanceImaging(MRI)isfarmoresuperiorforthe
studyofsofttissueandthus,formostneuro-ophthalmicconditions,
MRIistheinvestigationofchoice.

1025.Enophthalmosisseenin?
a)Blowoutfractureoforbitwall
b)Hyperthyroidism
c)RadiationInjuries
d)Diabetesmellitus
CorrectAnswer-A
Ans.is'a'i.e.,Blowoutfractureoforbitwall
Enophthalmosisseeninblowoutfractureoforbitwall.

1026.Ocularfindingsindiabetesareall
except-
a)Retinopathy
b)Earlysenilecataract
c)Neovascularglaucoma
d)Blepharophimosis
CorrectAnswer-D
Ans.is'd'i.e.,Blepharophimosis[RefKanski's8`Vechapter13,
p.520]


1027.Mucinlayertearflilmdeficiencyoccurs
in:
a)Keratoconjunctivitissicca
b)Lacrimalglandremoval
c)Canalicularblock
d)Herpetickeratitis
CorrectAnswer-A
Ai.eKeratoconjunctivitissicca

1028.Whichofthefollowingisalongterm
sideeffectofphakic10Ls
a)Chronicglaucoma
b)Retinaldetachment
c)Opticneuritis
d)None
CorrectAnswer-A
Ans.is'a'i.e.,Chronicglaucoma[RefChenLI,etal.
Metaanalysisofcataractdevelopmentafterphakicintraocular
lenssurgery.JCataractRefractSurg2008;34:1181-200.]
Phakicintraocularlenses(pIOLs)areartificiallensesthatare
insertedeitherontopoftheirisorinbetweentheirisandthenatural
lens.Theyareusedtotreatrefractiveerrorwithoutremovingcornea
tissueorthelens.
LongtermrisksofphakicIOLS
Chronicintraocularpressureelevationcanproduceindolentvision
damagefromangleclosureglaucoma.
Thesurgicalprocedureitselfalsoinducessomeendothelialcell
damageandiftheprostheticisincloseproximitytothecornea,
chroniclosscouldbeinduced.
Finally,chronicinflammationorprosthetic-lenstouchcaninduce
cataractformation

1029.Mostcommonprotozoancausing
keratitisis
a)Plasmodium
b)Acanthamoeba
c)Toxoplasma
d)W.bancrofti
CorrectAnswer-B
Ans.is`b'i.e.,Acanthamoeba[RefKanski8th/ep.197]
"Acanthamoebakeratitisisthemostcommonkeratitiscausedbya
protozoanespeciallyincontactlensusers".

1030.Smokersarepronetowhichlung
infection
a)Mycobacteriumtuberculosis
b)Pneumonia
c)Influenza
d)Alltheabove
CorrectAnswer-D
Answer-D.Alltheabove

1031.Somepatientswithsevereformof
Idiopathicinfantilehypercalcemia,
presntwithphenotypicfeaturessimilar
towhichofthefollowing?

a)Williamssyndrome
b)Potterssyndrome
c)Angelmansyndrome
d)VHLsyndrome
CorrectAnswer-A
Answer-A.Williamssyndrome
Inthesevereform,Prenatalandpostnatalgrowthfailureare
commonandnumberofphenotypicfeaturesofWilliamssyndrome
areobservedinsomeofthepatientsincludecardiovascular
abnormalities(usuallysupravalvularaorticstenoisandperipheral
pulmonicstenosis),latepsychomotordevelopment,selectivemental
deficiency,acharacteristicunusualfaciesandshortstature.
Theserumcalciumlevelsrangebetween12-19mg/dL.

1032.Whichliverdisease/sis/areassociated
withductopenia?
a)Chronicgraftrejection
b)Hepaticsarcoidosis
c)Paraneoplasticsyndromerelatedtohodgkinslymphoma
d)Alltheabove
CorrectAnswer-D
Answer-D.Alltheabove
Lossofbileductsfromtheportaltractsisreferredtoasductopenia.
Ductopeniaismostcommonlyseenrelatedtochronicallograft
rejectionandincludesdrugrelatedinjury,primarybinarycirrhosis,
primarysclerosingcholangitis,chronicgraft-vs-hostdisease,hepatic
sarcoidosis,paraneoplasticsyndromerelatedtoHodgkindisease,
andsyndromicpaucityofbileducts(Alagillesyndrome).

1033.1to2mmhaemorrhagesinskinareknownas:
a)Microbleed
b)Petechiae
c)Purpura
d)Noneoftheabove
CorrectAnswer-B
Minute1-to2-mmhemorrhagesintoskin,mucousmembranes,orserosalsurfacesare
calledpetechiae.
Thesearemostcommonlyassociatedwithlocallyincreasedintravascularpressure,low
plateletcounts(thrombocytopenia),ordefectiveplateletfunction(asinuremia).

Ref:
Robbins8theditionChapter4.

1034.Whichofthefollowingisnottrueabout
metabolicsyndrome?
a)ItisalsocalledSyndromeX
b)Acanthosisandsignsofhyperandrogenismmaybeseen
c)TypeAhasautoantibodiesagainsttheinsulinreceptor
d)Insulinresistanceincreasestheriskoftype2DMinpatients
withPCOS
CorrectAnswer-C
Answer-C.TypeAhasautoantibodiesagainsttheinsulin
receptor
Itisaninsulinresistanceconditionwhichcomprisesaspectrumof
disorders,withhyperglycemiarepresentingoneofthemostreadily
diagnosedfeatures.
Themetabolicsyndrome,theinsulinresistancesyndrome,or
syndromeXaretermsusedtodescribeaconstellationofmetabolic
derangementsthatincludesinsulinresistance,hypertension,
dyslipidemia(decreasedHDLandelevatedtriglycerides),centralor
visceralobesity,type2diabetesorIGT/IFG,andacceleratedcardio-
vasculardisease.
Acanthosisnigricansandsignsofhyperandrogenism
(hirsutism,acne,andoligomenorrheainwomen)arealso
commonphysicalfeatures.
Twodistinctsyndromes-

1. typeA,whichaffectsyoungwomenandischaracterizedbysevere
hyperinsulinemia,obesity,andfeaturesofhyperandrogenism;and
typeAinsulinresistancesyndromehaveanundefineddefectinthe
insulin-signalingpathway.
2. typeB,whichaffectsmiddle-agedwomenandischaracterizedby

severehyperinsulinemia,featuresofhyperandrogenism,and
autoimmunedisorders.


1035.Metabolicsyndromediagnosisinmen
basedonNCEPATPIIIcriteriaincludes
thefollowingexcept

a)Abdominalobesity>40inches
b)HDL<50mg/dL
c)BP>/=130/85mmHg
d)Fastingglucoe>110mg/dL
CorrectAnswer-B
Answer-B.HDL<50mg/dL

1036.Waterhammerpulseseenin
a)>Aorticstenosis
b)>Aorticregurgitation
c)>AorticstenosisandAorticregurgitation
d)>Mitralregurgitation
CorrectAnswer-B
Aorticregurgitation[Ref.Harrison17h/ep1476&166/ep1400
Kundu,bedsideclinicsinmedicine]
Waterhammerpulse
Itisalargeboundingpulse,associatedwithincreasedstroke
volume
oftheleftventricleanddecreaseintheperipheral
resistance,
leadingtoawidepulsepressure.
Thepulsestrikesthepalpatingfingerwitharapid,forcefidjerkand
quicklydisappears.
Itisbestfeltintheradialarterywiththepatientsarmelevated.
ItisseeninAorticregurgitation.
Pathophysiologyofwaterhammerpulse
InAorticregurgitationthestrokevolumeishigh,sothesystolic
pressureishigh
andthisisresponsibleforsharpriseinthepulse.
Thestrokevolumeishighbecausetheleftventriclegetsbloodfrom
twosourcesduringthediastolei.e.bloodleakingfromtheAortaand
theblooditreceivesfromleftatria.
Thecollapseoccursbecause
DiastolicleakofbloodintotheleftventriclefromtheAorta
Rapidrunofftotheperipheryasaresultoflowsystemicvascular
resistance(theincreasedcardiacoutputstimulatesthe
baroreceptorsintheaorticarchandtheresultisreflexvasodilatation
oftheperipheralvesselsintowhichthebloodflowsrapidly).


Wiryarearmselevatedintheexamination?
Whenthearmisraisedthereisfallofbloodcolumnresultingin
vasodilationandthushelpstoreducethediastolicpressuremore,so
thepulsepressurewidens.
Itmaybesothatthearterypalpatedbecomesmoreinthelineof
Aortaafterelevationofthearmandthusallowsthedirectsystolic
ejectionanddiastolicbackwardflow.

1037.FALSEstatementregardingtheECGinacutepericarditisis:
a)TwaveinversiondevelopbeforeSTelevationsreturnto
baseline
b)GlobalSTsegmentelevationisseeninearlypericarditis
c)Sinustachycardiaisacommonfinding
d)PRsegmentdepressionispresentinmajorityofpatients
CorrectAnswer-A
TwaveinversiondevelopafterSTelevationsreturntobaseline.
TherearefourstagesofECGchangesintheevolutionofacutepericarditis.Instage1,
thereiswidespreadelevationoftheSTsegments,oftenwithupwardconcavity,involving
twoorthreestandardlimbleadsandV2toV6,withreciprocaldepressionsonlyinaVRand
sometimesV1,aswellasdepressionofthePRsegmentUsuallytherearenosignificant
changesinQRScomplexes.Instage2,afterseveraldays,theSTsegmentsreturnto
normal,andonlythen,orevenlater,dotheTwavesbecomeinverted(stage3).Ultimately,
weeksormonthsaftertheonsetofacutepericarditis,theECGreturnstonormalinstage4.
Ref:Harrisonsprinciplesofinternalmedicine,18thedition,Page:1971

1038.Ejectionclickofpulmonarystenosisis
betterheardin
a)Inspiration
b)Expiration
c)Patientbendingforward
d)Patientlyinginleftlateralposition
CorrectAnswer-B
Answer-B.Expiration
Itemanatesfromastenoticpulmonaryvalveoradilatedpulmonary
artery.
Itsmostcharacteristicfeatureistodisappearorbecomemarkedly
softerwithinspirationi.e.itisbetterheardinexpiration.
Itislocalizedto2ndand3rdintercostalspaces.
Theymayalsobepresentinpatientswithpulmonaryhypertentionor
theoneswithdilatedpulmonaryartery.


1039.ReciprocalchangesinECGinpatients
withinferiorwallmyocardialinfarction
areseeninwhichleads

a)I
b)II
c)III
d)aVF
CorrectAnswer-A
Answer-A.I

1040.Allarefeaturesofaorticstenosis
except
a)Congestiveheartfailureduetosystolicordiastolicdysfunction
b)Presenceofejectionsystolicmurmur
c)Presenceofpulsustardus
d)Pressureintheaortaisthesameasinleftventricle
CorrectAnswer-D
Answer-D.Pressureintheaortaisthesameasinleftventricle
ClinicalFindings-
Systolicejectionmurmur
Carotidpulsusparvusettardus
Diminishedaorticcomponentof2ndheartsound
Suddendeathinseverestenosisafterexercise
Classicaltriad-
Angina
Syncope
Shortnessofbreath(heartfailure)

1041.Whichofthefollowingisnottrueabout
bicuspidaorticvalve?
a)Usuallyundetectedinearlylife
b)Itismorecommoninfemalesthaninmales
c)Post-stenoticdilatationofascendingaortacanbeseen
d)Diagnosisismadebyechocardiography
CorrectAnswer-B
Answer-B.Itismorecommoninfemalesthaninmales
Thecongenitalbicuspidaorticvalve,whichmayinitiallybe
functionallynormal,isoneofthemostcommoncongenital
malformationsoftheheartandmaygoundetectedinearlylife.
Morefrequentinmales(3:1)
Diagnosisismadebyechocardiography,whichrevealsthe
morphologyoftheaorticvalveandaorticrootandquantitates
severityofstenosisorregurgitation.
Theascendingaortaisoftendilated,misnamed"poststenotic"
dilatation;thisisduetohistologicabnormalitiesoftheaorticmedia
similartothoseinMarfan'ssyndromeandmayresultinaortic
dissection.

1042.Whichofthefollowinghemodynamic
changesisnotevidentincardiac
tamponadeduringdiastole?

a)Rightatrialandventricularcollapse
b)AbsentywaveonJVP
c)Biphasicvenousreturn
d)Elevatedpericardialpressure
CorrectAnswer-C
Answer-C.Biphasicvenousreturn
Duringthediastolethepericardialpressureremainselevated.
Itisgreaterthantheintracavitarypressurethusthereisnofilling
duringthediastole
Theabsenceofvenousreturnduringthediastoleleadstoabsence
of"y"wavesontheJ.V.P.andthe"diastoliccollapse"oftheright
atriaandventricle.


1043.Majorcriteriaforinfectiveendocarditis
includewhichofthefollowing
a)Injectiondruguser
b)Fever
c)Oslersnodes
d)Typicalorganismofinfectiveendocarditisisolatedfromtwo
separatebloodcultures
CorrectAnswer-D
Answer-D.Typicalorganismofinfectiveendocarditisisolated
fromtwoseparatebloodcultures


1044.WhichofthefollowingECGfeaturesare
notseeninpatientswithventricular
tachycardias?

a)BizzareQRScomplexes
b)PresenceofAVdissociation[fusionbeats]
c)ProlongeddurationofQRScomplexes
d)Ppulmonale
CorrectAnswer-D
Answer-D.Ppulmonale
AVdissociation(atrialcapture,fusionbeats)
QRSduration>140msforRBBBtypeV1morphology,VI>160ms
forLBBBtypeVImorphology
Frontalplaneaxis-900to1800
DelayedactivationduringinitialphaseoftheQRScomplex
LBBBpattern-RwaveinV1,V,>40ms
RBBBpattern-onsetofRwavetonadirofS>100ms
BizarreQRSpatternthatdoesnotmimictypicalRBBBorLBBB
QRScomplex.
ConcordanceofQRScomplexinallprecordialleads
RSordominantSinV6forRBBBVTQwaveinV6withLBBBQRS
pattern
MonophasicRorbiphasicqRofR/SinV,withRBBBpattern

1045.Cerebro-occulo-genitalsyndromehas
thefollowingfeaturesexcept
a)Microcephaly
b)Shortstature
c)Agenesisofcorpuscallosum
d)Flaccidquadriplegia
CorrectAnswer-D
Answer-D.Flaccidquadriplegia
Cerebro-occulo-genitalsyndromeisassociatedwithmicrocephaly,
shortstature,microophthalmia,agenesisofcorpuscallosum,
hypospadiasandspasticquadriplegia.

1046.Pulsusbiseferians,whichofthe
followingisnottrue
a)Itisseeninaorticregurgitation
b)Itisbetterfeltinperipheralarteries
c)Ithasonepeakinsystoleandoneindiastole
d)Ithastwopeaks
CorrectAnswer-C
Answer-C.Ithasonepeakinsystoleandoneindiastole
Itischaracterizedbytwosystolicpeaks.
Itisseeninpatientsofaorticregurgitation.
Thepulsewaveupstrokerisesrapidlyandforcefullyproducingthe
firstsystolicpeak(percussionwave).Abriefdeclineinpressureis
followedbyasmallerandsomewhatslowerrisingpositivepulse
wave.
Inbisiferianspulsethesecondriseinsystoleisenhancedby
reflectionfromperipheralarteriesthereforeitismoreprominentin
peripheralpulse.

1047.Themostcommonreentrant
tachycardiaassociatedwithWPW
syndromeis

a)OrthodromicAVreentry
b)AntidromicAVreentry
c)RapidlyconductingAF
d)None
CorrectAnswer-A
AnswerisA(OrthodromicAVreentry)
123.Themostcommonmacro-reentranttachycardiaassociated
WPWsyndromeisorthodromicAVreentry.
Themostcommonmacro-reentranttachycardiaassociatedwith
WPWsyndromeisreferredtoasOrthodromicAVreentry'


1048.SquarewaveseeninECGrecording
denote
a)Atrialdepolarization
b)Ventriculardepolarization
c)Ventricularrepolarization
d)StandardizationofECG
CorrectAnswer-D
Answer-D.StandardizationofECG
EachECGmachinehasaprovisionforStandardization(STD)of
Calibration(CAL).
Thestandardizationleverreleasesacurrentof1mVtothestylusof
ECGmachinethatrecordsashiftof10mmontheECGpaper.
Thestandardizationcurrentgivesrisetoawavepatterncalled
squarewavepatternasdepictedintheimagebelow.

1049.Whichofthefollowingisnottrueabout
Torsadesdepointes?
a)PresenceofprolongedQTintervalonECG
b)PresenceofpolymorphicQRScomplexes
c)Itisatypeofsupraventriculartachycardia
d)QRScomplexesappeartorotatearoundtheisoelectric
baselineofECG
CorrectAnswer-C
Answer-C.Itisatypeofsupraventriculartachycardia
ThesignificanceofthelongQTsyndromeisitsassociationwiththe
developmentofaspecifictypeofventriculartachycardiacalled
Torsadesdepointsorpolymorphicventriculartachycardia
AwidecomplextachyarrythmiawithQRScomplexesofvaryingaxis
andmorphologythatappeartorotatearoundtheisoelectric
baseline.

1050.ProlongedQTintervalisseeninallof
thefollowingexcept
a)Hypokalemia
b)Hypocalcemia
c)Useofmacrolideantibiotitcs
d)Hypernatremia
CorrectAnswer-D
Answer-D.Hypernatremia
Metabolic
Hypokalemia
Hypocalcemia
Hypomagnesemia

1051.Followingaretheclinicalsignsof
widenedpulsepressureseenin
patientsofaorticregurgitationexcept

a)Corriganssign
b)DeMussetssign
c)WaterHammerpulse
d)Diastolicmurmur
CorrectAnswer-D
Answer-D.Diastolicmurmur
Corrigan'spulse:Arapidandforcefuldistensionofthearterialpulse
withaquickcollapse
DeMusset'ssign:Bobbingoftheheadwitheachheartbeat(likea
birdwalking)
Muller'ssign:Visiblepulsationsoftheuvula
Quincke'ssign:Capillarypulsationsseenonlightcompressionofthe
nailbed
Traube'ssign:Systolicanddiastolicsoundsheardoverthefemoral
artery("pistolshots")
Duroziez'ssign:Gradualpressureoverthefemoralarteryleadstoa
systolicanddiastolicbruit
Hill'ssign:Poplitealsystolicbloodpressureexceedingbrachial
systolicbloodpressureby60mmHg(mostsensitivesignforaortic
regurgitation)
Waterhammerpulse

1052.Allofthefollowingarecharacteristic
featuresofTricuspidAtresiaexcept-
a)LeftAxisdeviation
b)Rightventricularhypoplasia
c)Pulmonaryvascularityisdiminished
d)SplittingofS2
CorrectAnswer-D
Ans.is'd'i.e.,SplitingofS2
Atresiaofthetricuspidvalveresultsintheabsenceofa
communicationbetweentherightatriumandrightventricletherefore
therightventricleisunderdevelopedtheinflowportionbeingabsent.
Theonlyexitforsystemicvenousbloodcomingtotherightatriumis
bywayofAtrialSepta!defect.Throughthisthebloodgoestoleft
atriumfromwhereitentersleftventricle.
Aventricularseptaldefectprovidescommunicationbetweentheleft
ventricleandtheoutflowportionoftherightventricle.Theleft
ventriclethereforemaintainboththesystemicaswellasthe
pulmonarycirculation
thusthereishypertrophyoftheleftventricle
whichisreflectedbyleftaxisdeviationinECG.
Thepulmonarybloodflowisdependentonthesizeoftheventricular
defect,
thesmallertheVSD,thelesserthepulmonaryblood
flow.90%patientsofTriscuspidAtresiahavediminishedpulmonary
bloodflow.o
AuscultatoryfindingincaseofTricuspidAtresia
S1-NormalS2-SingleMurmurgradeIItogradeIII/VI

1053.Episodeofstableanginapectoris
typicallylastsfor
a)Lessthan1min
b)2-5mins
c)5-10mins
d)>10mins
CorrectAnswer-B
Answer-B.2-5mins
Anginaisusuallycrescendo-decrescendoinnature,typicallylasts2
to5min,andcanradiatetoeithershoulderandtobotharms
(especiallytheulnarsurfacesoftheforearmandhand).
Italsocanariseinorradiatetotheback,interscapularregion,root
oftheneck,jaw,teeth,andepigastrium.Anginaisrarelylocalized
belowtheumbilicusorabovethemandible.

1054.Levinesignisseenin
a)Stableanginapectoris
b)Acutebronchialasthma
c)Hemolyticanemia
d)Gastroesophagealrefluxdisease
CorrectAnswer-A
Answer-A.Stableanginapectoris
StableAnginaPectoris:
Thisepisodicclinicalsyndromeisduetotransientmyocardial
ischemia
Whenthepatientisaskedtolocalizethesensation,heorshe
typicallyplacesahandoverthesternum,sometimeswithaclenched
fist,toindicateasqueezing,central,substernaldiscomfort(Levine's
sign).

1055.NotTrueaboutPrinzmetal'sangina:
a)Maypresentatrest
b)Occursdueatheroscleroticobstructionofcoronaryarteries
c)Smokingisariskfactor
d)Nitratesareusedfortreatment
CorrectAnswer-B
Answer-B.Occursdueatheroscleroticobstructionofcoronary
arteries
ThissyndromeisduetofocalspasmofanepicardialcoronAry
artery,leadingtoseveremyocardialischemialeadingtosevere
myocardialischemia.
Therightcoronaryarteryisthemostfrequentsite,followedbythe
leftanteriordescendingcoronaryartery.
Acetylcholinereleasedbytheparasympatheticsystematrestwill
simplycausecontractionofthevascularsmoothmuscle.
ItusuallyoccursatrestandisassociatedwithtransientST-segment
elevation.
Etiology
cigarettesmokers

1056.Themostcommontoxincausing
DilatedCardiomyopathyis:
a)Alcohol
b)Chemotherapeuticagents
c)Heavymetal
d)Occupationalexposure
CorrectAnswer-A
AnswerisA(Alcohol)
ChronicAlcoholConsumptionisthemostcommoncauseofToxic
DilatedCardiomyopathy.Alcoholisthemostcommontoxin
implicatedinchronicdilatedcardiomyopathy'
Note:
Dilatedcardiomyopathyisthemostcommontypeofcardiomyopathy
ThemostcommoncauseofdilatedcardiomyopathyisIdiopathic
(Two-Thirds)AlcoholConsumptionisthemostcommoncauseof
ToxicDilatedCardiomyopathy


1057.Obstructiveshockcanbeseenin
a)Pulmonaryembolism
b)Tensionpneumothorax
c)Pericardialtamponade
d)Alltheabove
CorrectAnswer-D
Answer-D.Alltheabove
Causesintraumapatientsincludepulmonaryembolism,pericardial
tamponade,acutecoronarysyndromes,increasedintrathoracic
pressureasintensionpneumothorax,positivepressureventilation
andexcessivePEEP.

1058.Pharmacologicalstressduringstress
myocardialradionucleotideperfusion
imagingcanbeinducedusing

a)Dipyridamole
b)Adenosine
c)Dobutamine
d)Alltheabove
CorrectAnswer-D
Answer-D.Alltheabove
Dipyridamoleoradenosinecanbegiventocreateacoronary"steal"
bytemporarilyincreasingflowinnondiseasedsegmentsofthe
coronaryvasculatureattheexpenseofdiseasedsegments.
Alternatively,agradedincrementalinfusionofdobutaminemaybe
administeredtoincreaseMVO2.
Thedevelopmentofatransientperfusiondefectwithatracersuch
asthallium-201or99m-technetiumsestamibiisusedtodetect
myocardialischemia.

1059.Whiletreatingpatientswithmalignant
hypertentionthemaximumallowed
decreaseinbloodpressureinthefirst2
-6hoursshouldnotexceed%

a)15
b)20
c)25
d)30
CorrectAnswer-C
Answer-C.25
HypertensionwithsystolicBP180mmHganddiastolicBP120mm
Hgisclassifiedas"severehypertension".
Severehypertension(180/120)doesnotnecessarilymean
hypertensiveemergencyormalignanthypertension.?Patientswith
B.P.180/120mayremainasymptomaticwithoutcausingany
complications.
Theinitialaimoftreatmentinmalignanthypertensionand
hypertensiveencephalopathyistolowerdiastolicbloodpressureto
about100to105mmHgwithinminutesto,two-sixhourswiththe
maximumfallinB.P.overthisperiodoftimenotexceeding25%of
theoriginalvalue.

1060.ECGpatternseeninpulmonary
embolismis:
September2009,March2013

a)S3Q3T1
b)S1Q1T3
c)S1Q3T3
d)S3Q3T3.
CorrectAnswer-C
Ans.C:S1Q3T3

1061.Contraindicationforpercutaneous
ballonmitralvalvotomyincludethe
followingexcept

a)Presenceofpulmonaryhypertension
b)Leftatrialthrombus
c)Severemitralregurgitation
d)Commissuralcalcification
CorrectAnswer-A
Answer-A.Presenceofpulmonaryhypertension
Patientswithvalvularcalcification,thickenedfibroticleafletswith
decreasedmobilityandsubvalvularfusion,haveincidenceofacute
complicationsandhigherrateofrestenosisonfollowup.Such
patientsareconsideredacontraindicationfortheprocedure.Other
contraindicationsincludemorethanmoderatemitralregurgitation,
presenceofleftatrialthrombi,andcommissuralcalcification.

1062.Mitralvalvereplacementis
recommendedinpatientswith
a)ModerateMSinNYHAclassII
b)ModerateMSinNYHAclassIII
c)SevereMSinNYHAclassII
d)SevereMSinNYHAclassIII
CorrectAnswer-D
Answer-D.SevereMSinNYHAclassIII
Sincetherearealsolong-termcomplicationsofvalvereplacement,
patientsinwhompreoperativeevaluationsuggeststhepossibility
thatMVRmayberequiredshouldbeoperatedononlyiftheyhave
severeMS--i.e.,anorificearea"1cm2--andareinNYHAClassIII,
i.e.,symptomaticwithordinaryactivitydespiteoptimalmedical
therapy".

1063.HOCMiscommoninwhichagegroup?
a)10-30years
b)20-40years
c)30-50years
d)40-60years
CorrectAnswer-B
Answer-B.20-40years
Hypertrophiccardiomyopathyischaracterizedbymarkedleft
ventricularhypertrophyintheabsenceofothercauses,suchas
hyper-tensionorvalvedisease.
Earliertermedhypertrophicobstructivecardiomyopathy(HOCM)
Hypertrophiccardiomyopathyischaracterizedhemodynamicallyby
diastolicdysfunction,originallyattributedtothehyper?trophy,
fibrosis,andintraventriculargradientwhenpresent.
Hypertrophiccardiomyopathyusuallypresentsbetweentheagesof
20and40years.Dyspneaonexertionisthemostcommon
presentingsymptom,reflectingelevatedintracardiacfillingpressures

1064.Preferredveinforcentralvenous
catheterinsertionis
a)Rightinternaljugularvein
b)Leftinternaljugularvein
c)Rightsubclavianvein
d)Rightantecubitalvein
CorrectAnswer-A
Answer-A.Rightinternaljugularvein
Placementofthecentralvenouscatheterisindicatedforthe
monitoringofthecentralvenouspressureandforprolongeddrug
administrationforparenteralnutrition.
Thepreferredsiteforinsertionofcatheterintothesuperiorvenacava
isfrominternaljugularveinoftheneck.Otherusedsitesarefrom
thesubclavianveinorfromtheperipheralveinintheantecubital
fossa.
Commonlyusedveincannulationsitesforcentralvenous
accessinclude:
Jugularvein
Externaljugularvein
Internaljugularvein(central,posterior,anteriorapproaches)
Subclavianvein(supraclavicular,infraclavicular,axillary
approaches)
Femoralvein
Basilicvein

1065.KerleyBlinesseeninmitralstenosis
whentherestingleftatrialpressure
exceeds

a)10mmHg
b)20mmHg
c)30mmHg
d)40mmHg
CorrectAnswer-B
Answer-B.20mmHg
KerleyBlinesarefine,dense,opaque,horizontallinesthataremost
prominentinthelowerandmid-lungfieldsandthatresultfrom
distentionofinterlobularseptaeandlymphaticswithedemawhen
therestingmeanLApressureexceedsapproximately20mmHg.

1066.Whichofthefollowingisthemost
commonanamolyinpatientswith
fanconi'sanemia?

a)Hyperpigmentationofthetrunk,neckandintertriginousareas
b)Absentradiiandthumb
c)Weakradialpulse
d)Presenceofhorseshoekidneys
CorrectAnswer-A
Answer-A.Hyperpigmentationofthetrunk,neckand
intertriginousareas
Fanconianemia(FA)isprimarilyinheritedinanautosomalrecessive
manner(oneuncommonformisX-linkedrecessive).
ThemostcommonanomalyinFAishyperpigmentationofthetrunk,
neck,andintertriginousareas,aswellascaf?-au-laitspotsand
vitiligo,aloneorincombination.
Anomaliesofthefeet,congenitalhipdislocation,andleg
abnormalitiesareseen.

1067.Patientswithwhichofthefollowing
conditionsareatgreatestriskofpernio
a)Raynaud'sphenomenon
b)Kawasakidisease
c)HenochSchonlenpurpura
d)HepatitisCinfection
CorrectAnswer-A
Answer-A.Raynaud'sphenomenon
Thetwomostcommonnonfreezingperipheralcoldinjuriesare
chilblain(pernio)andimmersion(trench)foot.
Chilblainresultsfromneuronalandendothelialdamageinducedby
repetitiveexposuretodrycold.
Youngfemales,particularlythosewithahistoryofRaynaud's
phenomenon,areatgreatestriskofpernio(chilblain).
Persistentvasospasticityandvasculitiscancauseerythema,mild
edema,andpruritus.Eventuallyplaques,bluenodules,and
ulcerationsdevelop

1068.FEVI/FVCisdecreasein:
a)Asthma
b)Kyphosis
c)Scoliosis
d)Fibrosis
CorrectAnswer-A
AnswerisA(Asthma):
DecreasedFEV1/FVCsuggestsadiagnosisofObstructiveLung
Disease.
AmongsttheoptionsprovidedAsthmaistheonlyconditionthat
leadstoObstructivePatternofLungDiseaseandhenceisthe
answerofchoiceKyphosis,ScoliosisandFibrosisareRestrictive
LungDiseasesthatarecharacterizedbyNormalorElevated
FEVI/FVCratios.


1069.WhatisTiffeneau-Pinelliindex?
a)FEV1/FVCratio
b)Bodymassindex
c)Quetletindex
d)Ventilation/Perfusionratio
CorrectAnswer-A
Answer-A.FEV1/FVCratio

1070.Lightscriteriaisusedfor
a)Pleuraleffusions
b)Pericardialeffusions
c)Ascites
d)Increasedintracranialtension
CorrectAnswer-A
Answer-A.Pleuraleffusions
Light'scritenaforclassificationofunilateralpleuraleffusion
Thepleuralfluidisanexudateifoneormoreofthefollowingciteria
aremet.
Pleuralfluidproteindividedbyserumprotein>0.5
PleuralfluidlacatodehyoctogenaseLDHdividedbyserumLDH>
0.6

1071.Commoncauseofdeathinapatient
withchronicbronchieactasisis
a)Rightsidedheartfail
b)Infection
c)Hemoptysis
d)Carcinoma
CorrectAnswer-A
Answer-A.Rightsidedheartfail
Intodaysworld,rightsidedheartfailureinpatientswithdiffuselong
standingbronchieactasisisacommoncauseofdeathinpatients
withchronicbronchieactasis.
Pneumoniaandhemorrhagearelesscommoncausesofdeath.

1072.Leutrieneinhibitorsareusedinasthma
for
a)Monotherapyforacuteattack
b)Add-ontherapyinpatientsnotcontrolledbylowdoseinhaled
glucocorticoids
c)Statusasthmaticus
d)Noneoftheabove
CorrectAnswer-B
Answer-B.Add-ontherapyinpatientsnotcontrolledbylow
doseinhaledglucocorticoids
Cysteinyl-leukotrienesarepotentbronchoconstrictors,cause
microvascularleakage,andincreaseeosinophilicinflammation
throughtheactivationofcys-LT1-receptors.
Lukotrieneinhibitorssuchasmontelukastandzafirlukast,blockcys-
LT1-receptorsandprovidemodestclinicalbenefitinasthma.
Theyarelesseffectivethaninhaledcorticosteroidsincontrolling
asthmaandhavelesseffectonairwayinflammation,butareuseful
asanadd-ontherapyinsomepatientsnotcontrolledwithlowdoses
ofinhaledcorticosteroids.

1073.Apneahypoapneaindexindicating
obstructivesleepapneais-
a)<1
b)2-5
c)5-8
d)>8
CorrectAnswer-A
Answer-A.<1
Normalpreschoolandschool-agechildrengenerallyhaveatotal
AHIoflessthan1.5(obstructiveAHI<1),andthisisthemostwidely
usedcutoffvalueforObstructiveSleepApneainchildren12yrand
below;inolderadolescents,theadultcutoffofanAHI5isgenerally
used.

1074.Multipleepisodesofacutechest
syndromeareassociatedwith
a)Asthma
b)Bronchieactasis
c)SLE
d)Sjogrenssyndrome
CorrectAnswer-A
Answer-A.Asthma
Patientpresentswithrecurrentepisodesofacutechestsyndrome.
Thecharacteristicsymptomsduringanepisodeofasthmaare
wheezing,dyspnea,andcoughing,whicharevariable,both
spontaneouslyandwiththerapy.
Prodromalsymptomsmayprecedeanattack,withitchingunderthe
chin,discomfortbetweenthescapulae,orinexplicablefear
(impendingdoom).

1075.Triadofskinlesions,mononeuritis
multiplex,eosinophilsseenin
a)Alportssyndrome
b)Churg-Strausssyndrome
c)Cryoglobulinemia
d)Wegenersgranulomatosus
CorrectAnswer-C
Answer-C.Cryoglobulinemia
Churg&Straussischaracterizedbyasthma,eosinophilla,
extravasculargranulomaformation,vasculitis.
ClinicalFeatures-Fever,malaise,anorexia,weightloss.
Mononeuritismultiplexisthesecondmostcommonfeatures.
Allergicrhinitisandsinusitis.
Asthma
Peripheralandtissueeosinophillia,extravascularnecrotizing
granuloma.

1076.Chronicbronchitisissaidtobepresent
whenpatienthaschroniccough
a)3consecutivemonthsinatleasttwoconsecutiveyears
b)2consecutivemonthsfor3consecutiveyears
c)3consecutivemonthsinoneyear
d)1monthinayearfor2consecutiveyears
CorrectAnswer-A
Answer-A.3consecutivemonthsinatleast2consecutive
years.
Cigarettesmokingisthemostimportantriskfactor;airpollutants
alsocontribute.
Thedominantpathologicfeaturesaremucushypersecre-tionand
persistentinflammation.
Histologicexaminationdemonstratesenlargementofmucous-
secretingglands,gobletcellhyperplasia,chronicinflammation,and
bronchiolarwallfibrosis.

1077.Brockssyndromeisduetowhichlobe
oflung?
a)Rightmiddlelobe
b)Rightlowerlobe
c)Leftupperlobe
d)Leftlowerlobe
CorrectAnswer-A
Answer-A.Rightmiddlelobe
Brockssyndromeisduetocollapseofrightmiddlelobeoflung.Itis
seenasanacutecomplicationofpulmonarytuberculosis.Itoccurs
secondarytohilarnodeinvolvement.

1078.PresenceofVelcrocracklesatthelung
baseonauscultationisasignof
a)Scleroderma
b)SystemicLupus
c)WegenersGranulomatosus
d)Polyarteritisnodosa
CorrectAnswer-A
Answer-A.Scleroderma
CLINICALFEATURES-
Skin-sclerodactyly,Ranaud'sphenomenon,calcinosis,
telangiectasia,skinthickening.(inadvancestagefingersbecome
clawlike&facemasklike)
Musculoskeletalfeatures-Arthralgia,flexortenosynovitis
GIT-oesophagitis,dysphagia,malabsorption
Cardiorespiratoryfeatures-pulmonaryfibrosis,pulmonary
hypertension
Renalfeatures-hypertensiverenalcrisis
Malignanthypertension
Physicalexaminationmayreveal"Velcro"cracklesatthelung
bases.

1079.FeatureofAcutesevereAsthmainclude
allofthefollowing,Except:
a)Tachycardia>120/min
b)Pulsusparadoxus
c)Respiratoryacidosis
d)Drowsy
CorrectAnswer-A
AnswerisA.Tachycardia>120/min
Diaphoresis
Bradycardia
Paradoxicalthrocobadominalmovements
PEER<33%
Hypotension
Pulsusparadoxus
Hypercapnea
Silentchest

1080.TypeIVrespiratoryfailureoccursdue
to
a)Alveolarflooding
b)InabilitytoeliminateCO2
c)Lungatelactasis
d)Hypoperfusionofrespiratorymuscles
CorrectAnswer-D
Answer-d.Hypoperfusionofrespiratorymuscles
TypeIVRespiratoryFailure:resultsfromhypoperfusionof
respiratorymusclesinpatientsinshock.
Commonlycausedbycardiogenicshock,septicshockand
hypovolemicshock.

1081.Type3respiratoryfailureoccursdueto
?
a)Post-operativeatelectasis
b)Kyphoscoliosis
c)Flailchest
d)Pulmonaryfibrosis
CorrectAnswer-A
Ans.is'a'i.e.,Post-operativeatelectasis

1082.Tool/swhichobjectivelyassestherisk
ofadverseoutcomesinapatientwith
pneumoniais/are

a)Pneumoniaseverityindex[PSI]
b)CURB-65criteria
c)ApacheeScore
d)Glasgowscale
CorrectAnswer-A:B
Answer-(A)Pneumoniaseverityindex[PSI](B)CURB-65
criteria
Toolsthatobjectivelyassesstheriskofadverseoutcomesarethe
PneumoniaSeverityIndex(PSI),aprognosticmodelusedtoidentify
patientsatlowriskofdying;andtheCURB-65criteria,aseverity-of-
illnessscore.
TheCURB-65criteriaincludefivevariables:confusion(C);urea>7
mmol/L(U);respiratoryrate30/min(R);bloodpressure,systolic"90
mmHgordiastolic"60mmHg(B);andage65years(65).

1083.Empyemanecessitansisdefinedasso
when?
a)Pluralempyemaisunderpressure
b)Pleuralempyemahasrupturedintobronchus
c)Pleuralempyemahasrupturedintothepericardium
d)Pleuralempyemaisshowingextensiontothesubcutaneous
tissue
CorrectAnswer-D
Ans.is'd'i.e.,Pleuralempyemaisshowingextensiontothe
subcutaneoustissue

1084.InICUsettingpatientssufferingfrom
whichrespiratorypathologyareatrisk
ofCO2narcosis?

a)Pneumonia
b)Asthma
c)Emphysema
d)Bronchieactasis
CorrectAnswer-C
Answer-C.Emphysema
Hypoventilationsyndromeoccursmostfrequentlyinpatientswitha
historyofchronicCO,retentionwhoarereceivingoxygentherapy
foremphysemaorchronicpulmonarydisease.
TheelevatedPaco,leadingtoCO,narcosismayhaveadirect
anestheticeffect,andcerebralvasodilationfromincreasedPaco,
canleadtoincreasedICP

1085.Inspiratorysqueaksarethephysical
examinationfindingof
a)Bronchiolitis
b)Pulmonaryhypertension
c)Pneumonia
d)Pulmonaryedema
CorrectAnswer-A
Answer-A.Bronchiolitis
ILDassociatedwithinflammationbutarelesslikelytobeheardin
thegranulomatouslungdiseases.
Cracklesmaybepresentintheabsenceofradiographic
abnormalitiesonthechestradiograph.
Scatteredlateinspiratoryhigh-pitchedrhonchi--so-calledinspiratory
squeaks--areheardinpatientswithbronchiolitis.
Cyanosisandclubbingofthedigitsoccurinsomepatientswith
advanceddisease.

1086.Whichofthefollowingaretheclinical
abnormalitiesofuremia?
a)Hyperphosphatemia
b)Uremicfrost
c)Pepticulcer
d)Alltheabove
CorrectAnswer-D
Answer-D.Alltheabove
Volumeexpasion(I)
Hyperkalemia(I)
Hyponatremia(1)
Hyperphosphatemia(I)



1087.WhichtypeofBartter'ssyndromeis
associatedwithmutationsinbarttin?
a)Type1
b)Type2
c)Type3
d)Type4
CorrectAnswer-D
Answer-D.Type4
Bartter'ssyndromemayresultfrommutationsaffectinganyoffive
iontransportproteinsintheTAL.
Theproteinsaffectedincludetheapicalloopdiuretic-sensitive
sodium-potassium-chlorideco-transporterNKCC2(type1),the
apicalpotassiumchannelROMK(type2),andthebasolateral
chloridechannelC1C-Kb(type3).
Bartter'stype4resultsfrommutationsinbarttin,anessentialsubunit
ofCIC-KaandC1C-Kbthatenablestransportofthechloride
channelstothecellsurface.Barttinisalsoexpressedintheinner
ear;thisaccountsforthedeafnessinvariablyassociatedwith
Bartter'stype4.

1088.Testusedforscreeningforurinarytract
infectionis
a)Nitritetest
b)Nanitroprussidetest
c)PaulBunneltest
d)Fentonstest
CorrectAnswer-A
Answer-A.NitriteTest
Effectiveandrapidmethodusedforscreeningurineforthepresence
ofbacterialinfection.
Testisbasedonthefactthatmostbacteriapresentintheurine,
havethecapacitytoreduceurinenitratetonitrite.

1089.Patientwithnephroticsyndromehas
decreasedamountofwhichantibody
a)IgG
b)IgE
c)IgM
d)IgA
CorrectAnswer-A
Answer-A.IgG
Theimmunologicalabnormalitiesnotedareverypeculiar
IgGantibodyisdecrease
IgEandIgMantibodyincrease
ReducedresponsestoPHAandConA(ConcanavalinA)
Increaseofbetamicroglobulinlevels
ReducedproductionofIL2
Increasedproductionofvascularpermeabilityand
immunosuppressorfactorsbyCD4TcellsandCD8Tcells
respectively.


1090.RIFLEcriteriaisusedfordiagnosisof
a)Acutekidneyinjury
b)Acutesplenicinjury
c)Acuteliverinjury
d)Acutebowelinjury
CorrectAnswer-A
Answer-A.Acutekidneyinjury
TheRIFLEcriteria,definesthreesincreasinglevelsofseverityof
acutekidneyinjuryonthebasisoftheincreaseinserumcreatinine
concentrationordecreaseinurineoutput.

1091.Whatisoliguria-
a)Excretionoflessthan300m1in24hrs
b)Excretionoflessthan500m1in24hrs
c)Excretionoflessthan300ml.in12hrs
d)Excretionoflessthan100ml.in24hrs
CorrectAnswer-B
Ans.is'b'i.e.,Excretionoflessthan500mlin24hrs
(NotethatBailey&Love25/ep1279(24/ep1300&23/ep1168)
writeOliguriatobelessthan300mlin24hrs,butwewouldfollow
Harrison&CMDT;NotethatSmith'sUrology17/ep531(16/e,p538)
writeitto<400ml/day!!whatanutterconfusion.)

1092.Hemodynamicallyimportantlesionsof
renalarterystenosisarepredictedby
renalarteryvelocitiesmorethanon
Dopplerultrasound.

a)100cm/s
b)125cm/s
c)150cm/s
d)200cm/s
CorrectAnswer-D
Answer-D.200cm/s
RenalarteryvelocitiesbyDopplerultrasoundabove200cm/s
generallypredicthemodynamicallyimportantlesions(above60%
vessellumenocclusion),althoughtreatmenttrialsrequirevelocity
above300cm/stoavoidfalsepositives.
Renalresistiveindexhaspredictivevalueregardingtheviabilityof
thekidney.Itremainsoperator-andinstitution-dependent.

1093.Gitelman'ssyndromeresemblesthe
effectsofwhichofthefollowingdrugs?
a)Thiazide
b)Furosemide
c)Spironolactone
d)Amiloride
CorrectAnswer-A
Answer-A.Thiazide
Gitelman'ssyndromeisduetomutationsinthethiazide-sensitive
Na-Clco-transporter,NCCT,inthedistalconvolutedtubule(DCT).
DefectsinNCCTinGitelman'ssyndromeimpairsodiumandchloride
reabsorptionintheDCTandthusresembletheeffectsofthiazide
diuretics.Itremainsunclearhowthisdefectleadstosevere
magnesiumwasting.

1094.Autosomalrecessivepolycystickidneydiseaseischaracterisedbythe
alteredexpressionof:
a)Polycystin
b)Nephrocystin
c)Uromodulin
d)Fibrocystin
CorrectAnswer-D
Fibrocystin(polyductin)genemutationisseeninautosomalrecessivepolycystic
kidneydisease.

Note:
Polycystinmutationisassociatedwithautosomaldominantpolycystickidneydisease.
Nephrocystinisrelatedtonephronophthisis.
Uromodulinmutationisseeninmedullarycystickidneydisease.
Ref:Harrisonsprinciplesofinternalmedicine,18thedition,Page:2356.

1095.Definitionofcomplicatedurinarytract
infectionis,theinfectionwhichfailto
resolveorrecurwithinweek/sof
standardtherapy.

a)1week
b)2weeks
c)3weeks
d)4weeks
CorrectAnswer-B
Answer-B.2weeks
Complicatedurinarytractinfectionsreferstotheinfectionsthatfailto
resolveorrecurwithin2weeksofstandardtherapy.
Theseareassociatedwithbacteremiaorsepsisandareassocated
withperiurethralabscess,obstructionsandpyelonephritis

1096.Apatientpresentswithblunttraumato
abdomen.Oninvestigationspatientis
foundtohavehepaticinjurywhichhas
arupturedsubcapsularhematomawith
activebleeding.Whatisthegradeof
liverinjury?

a)GradeI
b)GradeII
c)GradeIII
d)GradeIV
CorrectAnswer-C
Answer-C.GradeIII



1097.Excellentpredictorofmortalityand
morbidityinpatientsafterhepatectomy
is

a)Serumlactatelevels
b)Serummagnesiumlevel
c)Serumironlevel
d)Serumcopperlevel
CorrectAnswer-A
Answer-A.Serumlactatelevels
Hyperlactemiaandhypophosphatemiaarecommonderangements
inpatientsundergoingliverresection.
Gluconeogenesiscarriedoutbythelivernormallyconsumes40-
60%oflactate.
Whentheliverisdamaged,stressedorresected,itproduceslactate
ratherthanmetabolizingit.
Duetotheadditiveeffectsoflactate-containingintravenoussolution,
non-lactatecontainingsolutionsarerecommendedforpostoperative
useafterhepatectomy.

1098.Portalhypertensionissaidtobe
presentifportalvenouspressureis
morethan:
March2010

a)3-5mmHg
b)5-8mmHg
c)10-12mmHg
d)15-20mmHg
CorrectAnswer-C
Ans.C:10-12mmHg
Normalportalpressureisgenerallydefinedbetween5and10mm
Hg.
Portalhypertensionresultswhentheportalpressurerisesto12mm
Hgorgreaterandcomplicationscanarise,suchasvaricesand
ascites.
Manyconditionsareassociatedwithportalhypertension,ofwhich
cirrhosisisthemostcommoncause

1099.Allofthefollowingdrugsmaybeused
inthetreatmentofulcerativecolitis
Except:

a)Corticosteroids
b)Azathioprine
c)Sulfasalazine
d)Methotrexate
CorrectAnswer-D
AnswerisD(Methotrexate)
Methotrexatehasnotbeenshowntobeeffectivefbrtreatingactive
ulcerativecolitisorformaintainingremission.


1100.AsciticfluidSAAG<1.1whatisthe
diseaseassociatedwith
a)Hepaticfailure
b)Idiopathicportalfibrosis
c)Constrictivepericarditis
d)Peritonealcarcinomatosis
CorrectAnswer-D
Answer-D.Peritonealcarcinomatosis
Lowalbumingradient(SAAG<1.1g/dL)
Peritonealcarcinomatosis
Peritonealtuberculosis
Pancreatitis
Serositis
Nephroticsyndrome

1101.Specificantibodyassociatedwith
primarybiliarycirrhosisis:
a)Anti-myosin
b)Anti-nuclear
c)Anti-mitochondrial
d)Anti-endomysial
CorrectAnswer-C
Ans.C:Anti-mitochondrial
Primarybiliarycirrhosisisstronglyassociatedwiththepresenceof
anti-mitochondrialantibodies(AMA),whicharediagnostic
PBC:
*AssociatedwithCRESTsyndrome,siccasyndrome,auto-immune
thyroiditis,typeIDMandIgAdeficiency
*IgGAMAisdetectedinmorethan90%ofpatientswithPBC
*90%ofwomenarebetween35-60years
*Earliestsymptomispruritis
*Eventuallyhepatocellularfailureandportalhypertensiondevelops
*Labfindings:
-Increasedserumalkalinephosphatase
-Increasedserum5-nucleotidaseactivity
-Increasedgamma-glutamyltranspeptidase
-Serumbilirubinisusuallynormal
-Aminotransferaselevelsminimallyincreased
-Treatment:Ursodiol

1102.Geneassociatedwiththedevelopment
ofPeutz-Jegherssyndromeis
a)STK11
b)PTEN
c)KRAS
d)BRCA1
CorrectAnswer-A
Answer-A.STK11
Germlineheterozygousloss-of-functionmutationsinthegeneSTKil
arepresentinapproximatelyhalfofindividualswithfamilialPeutz-
Jegherssyndromeaswellasasubsetofpatientswithsporadic
Peutz-Jegherssyndrome.

1103. Whichofthefollowingisnottrueabout
amoebiasis?
a)ALAin10%causesintropics
b)Portalsystemefficientfilter
c)Abscessduetosuppuration
d)Abscesswallgoodforculture
CorrectAnswer-C
Ans.is'c'i.e.,Abscessduetosuppuration
Amebicliverabscess
Pusinamoebicliverabscessisnotduetosuppuration,butisa
mixtureofsloughedlivertissueandblood.
Itischocolatebrownincolourandthickinconsistency(anchovy
saucepus)
Inthetropics2-10%oftheindividualsinfectedwithentamoeba
histolyticasufferfromhepaticcomplications.Thetrophozoitesare
carriedfromthelargeintestinetotheliverbyportalvein.Intheliver
thecapillarysystemactsasefficientfilterandholdstheseparasites.
Incaseofamoebicliverabscessthediagnosticaspirationisdone
fromtheabscesswallbecausethetrophozoitesareconfinedtothe
periphery.


1104.Righthanddominantpatientpresents
withnormalcomprehensionbutspeaks
withshortutterancesofafewwordsat
atime,comprisedmostlyofnouns.
Whatisthemostprobablelocationof
thelesion

a)Leftinferiorfrontalgyrus
b)Rightinferiorfrontalgyrus
c)Leftsuperiortemporalgyrus
d)Rightsuperiortemporalgyrus
CorrectAnswer-A
Answer-A.Leftinferiorfrontalgyrus
Patienthasnormalcomprehensionbutspeakswithshortutterances
ofafewwordsatatime,comprisedmostlyofnouns.Thesefindings
aresuggestiveofbrocasaphsia.Itisseeninpatientshaving
affectionoftheinferiorfrontalgyrusofthedominanthemisphere.
Thepatientisrighthandedsothelefthemisphere,willthedominant
one.Thusmostprobablelocationofthelesionisleftinferiorfrontal
gyrus.

1105.Pureworddeafnessisassociatedwith
a)Middlecerebralarterystroke
b)Posteriorcerebralarterystroke
c)Vertebralarteryaneurysm
d)Basilararteryaneurysm
CorrectAnswer-A
Answer-A.Middlecerebralarterystroke
Themostcommoncausesareeitherbilateralorleft-sidedmiddle
cerebralartery(MCA)strokesaffectingthesuperiortemporalgyrus.
Theneteffectoftheunderlyinglesionistointerrupttheflowof
informationfromtheauditoryassociationcortextoWernicke'sarea.

1106.Globalaphasiaisseendueto
a)Strokesinvolvingentiremiddlecerebralarterydistributioninleft
hemisphere
b)Strokesinvolvingentiremiddlecerebralarterydistributionin
righthemisphere
c)Strokesinvolvingentireposteriorcerebralarterydistributionin
lefthemisphere
d)Strokesinvolvingentireposteriorcerebralarterydistributionin
righthemisphere
CorrectAnswer-A
Answer-A.Strokesinvolvingentiremiddlecerebralartery
distributioninlefthemisphere
ThissyndromerepresentsthecombineddysfunctionofBroca'sand
Wernicke'sareasandusuallyresultsfromstrokesthatinvolvethe
entiremiddlecerebralarterydistributioninthelefthemisphere.
Relatedsignsincluderighthemiplegia,hemisensoryloss,and
homonymoushemianopia.

1107.LambertEatonsyndrometrueis
a)Itisaparaneoplasticsyndromeassociatedwithsquamouscell
carcinomaoflung
b)IgMantibodiesagainstligandgatedcalciumchannels
c)Thereisincreaseinreleaseofpresynapticacetylcholine
d)Withcontinuousstimulationthereismarkedincreasein
amplitudeofactionpotentials.
CorrectAnswer-D
Answer-D.Withcontinuousstimulationthereismarked
increaseinamplitudeofactionpotentials.
Itisapara
neoplasticsyndromeassociatedwithcancerparticularly
smallcellCaoflung.
Itisadisorderofneuromuscularjunctiontransmission(Presynaptic)
TheseIgGautoantibodiesagainstthevoltagesensitivecalcium
channelsreducethenumberoffunctioningchannels.
Thiscausesdecreaseinreleaseofpresynapticacetylcholine.
Symptoms
Musclesofthetrunkshouldergirdle,pelvicgirdleandmusclesof
lowerextremities(musclesoftheproximallegarethemost
commonlyinvolvedmuscles)
Oftenthefirstsymptomsaredifficultyinarisingfromachair,
climbingstairsandwalking,theshouldermusclesareaffectedlater
on.

1108.WhichisnotseeninAlzheimers
disease
a)Gradualdevelopmentofforgetfulness
b)Defectivevisuospatialorientation
c)Depression
d)Sequenceofneurologicalabnormalitiesfollowsadescribed
order
CorrectAnswer-D
Answer-D.Sequenceofneurologicalabnormalitiesfollowsa
describedorder
Thesequenceofneurologicaldisabilitiesmaynotfollowany
describedorderandoneoranotherdeficitmaytakepreceedence
butusuallythediseasepreceedesinthefollowingfourobserved
patterns.
1)Korsakoffamnesticstate
2)Dysnomia
3)Visuospatialorientationbecomesdefective
4)Paranoiaandotherpersonalitychanges

1109.Whichlobeisaffectedintheearly
courseofalzheimersdisease
a)Frontallobe
b)Parietallobe
c)Medialtemporallobe
d)Lateraltemporallobe
CorrectAnswer-C
Answer-C.Medialtemporallobe
Structuresofthemedialtemporallobe,includinghippocampus,
entorhinalcortexandamygdala,areinvolvedearlyinthecourseand
areusuallyseverelyatrophiedinthelaterstages.

1110.Whichis/aretheusualfirst
deformity/iestobeseeninCMT
disease?

a)Pescavus
b)Clubhand
c)Mannusvalgus
d)Flexiondeformityofknee
CorrectAnswer-A
Answer-A.Pescavus
Charcot-Marie-Tooth(CMT)diseaseisthemostcommontypeof
hereditaryneuropathy.
Thereisprogressivemuscleweaknessandatrophythatusually
beginsinthefirsttwodecadesoflife.
Thefirstsignsofthediseaseareusuallypescavus,footdeformities
andscoliosis.
Thereisslowlyprogressiveweaknessandwasting,firstofthefeet
andlegsandthenofthehands.
ThemostcommonformofCMTistype1,ademyelinating
neuropathywithautosomaldominantinheritance,mappedmost
commonlytotheshortarmofchromosome17.

1111.Huntingtonsdiseaseiscommonlyseen
inagegroupbetween
a)15-35years
b)25-45years
c)35-55years
d)45-65years
CorrectAnswer-B
Answer-B.25-45years
HDisaprogressive,fatal,highlypenetrantautosomaldominant
disordercharacterizedbymotor,behavioral,andcognitive
dysfunction.
Onsetistypicallybetweentheagesof25and45years(range,3-70
years)withaprevalenceof2-8casesper100,000andanaverage
ageatdeathof60years.
Huntigton'sdiseaseischaracterizedbytriadof
Autosomaldominantinheritence
Choreoathetosis
Dementia

1112.Whichcranialnerveisinvolvedin
Webersyndrome?
a)II
b)III
c)IV
d)V
CorrectAnswer-B
Answer-B.III
Weber'ssyndrome-Midbrain-Oculomotornerve-Ipsilateralthird-
nervepalsy

1113.Violentabnormalflingingmovements
whichareirregularandaffectingone
sidearecalledas-

a)Chorea
b)Athetosis
c)Dystonia
d)Hemiballismus
CorrectAnswer-D
Answer-D.Hemiballismus
Itisdefinedasthedysfunctionintheimplementationofappropriate
targetingandvelocityofintendedmovements,dysfunctionof
postureandabnormalinvoluntarymovement,ortheperformanceof
normalappearingmovementsatinappropriateorunintendedtimes.

1114.Wernickesencephalopathydevelops
secondarytoaccumulationofwhich
substrate?

a)Glutamate
b)Aspartate
c)Lactate
d)Acetate
CorrectAnswer-A
Answer-A.Glutamate
Glutamateaccumulatesowingtoimpairmentofaketoglutamate
dehydrogenaseactivityandincombinationwithenergydeficiency
mayresultinexcitotoxiccelldamage.

1115.Whichofthefollowingclinicaltest
whenpositivesuggestspresenceof
sensoryataxia?

a)Rombergtest
b)Adsontest
c)Stinchfieldtest
d)CrossedSLRtest
CorrectAnswer-A
Answer-A.Rombergtest
TheRombergtestisatestofthebody'ssenseofpositioning
(proprioception),whichrequireshealthyfunctioningofthedorsal
columnsofthespinalcord.
TheRombergtestisusedtoinvestigatethecauseoflossofmotor
coordination(ataxia).
ApositiveRombergtestsuggeststhattheataxiaissensoryin
nature,thatis,dependingonlossofproprioception.

1116.Toddsparalysisisexperiencefollowing
episodeof
a)Focalseizure
b)Generalisedseizure
c)Aftercorrectionofhyponatremia
d)Aftercorrectionofhypokalemia
CorrectAnswer-A
Answer-A.Focalseizure
Focalseizuresarisefromaneuronalnetworkeitherdiscretely
localizedwithinonecerebralhemisphere.
Theroutineinterictal(i.e.,betweenseizures)electroencephalogram
(EEG)inpatientswithfocalseizuresisoftennormalormayshow
briefdischargestermedepileptiformspikes,orsharpwaves.
Second,patientsmayexperiencealocalizedparesis(Todd's
paralysis)forminutestomanyhoursintheinvolvedregionfollowing
theseizure.

1117.Cerebro-occulo-genitalsyndromehas
thefollowingfeaturesexcept
a)Microcephaly
b)Shortstature
c)Agenesisofcorpuscallosum
d)Flaccidquadriplegia
CorrectAnswer-D
Answer-D.Flaccidquadriplegia
Cerebro-occulo-genitalsyndromeisassociatedwithmicrocephaly,
shortstature,microophthalmia,agenesisofcorpuscallosum,
hypospadiasandspasticquadriplegia.

1118.Frontallobesyndromeconsists
a)Euphoria
b)Indifference
c)Irritability
d)Alltheabove
CorrectAnswer-D
Answer-D.Alltheabove
Euphoria,indifference,disinhibition,andirritabilityareconsequences
offrontallobelesions.Theseemotionalandbehavioural
disturbancesareusuallyreferredtoasfrontallobesyndrome.
Otherfeaturesaredecreasedsocialconcern,jocularity,
facetiousness,coarseness,hyperkinesia,disinhibition,lossofsocial
graces,inappropriatesexualadvances,sexualexhibitionism,
impulsiveness,restlessness,andgrandiosedelusions.

1119.Flappingtremorsarenotseenin
a)CO2toxicity
b)Hypomagnesemia
c)Subarachnoidhemorrhage
d)Carbolicacidpoisoning
CorrectAnswer-D
Answer-D.Carbolicacidpoisoning
Itisanimportantclinicalsign
Itisnotpathognomonicofanyconditionbutitgivescluetoserious
underlyingdiseaseprocess.
Flappingtremorisamotordisturbancemarkedbyintermettent
lapsesofanassumedpostureasaresultofintermittencyof
sustainedcontractionofgroupofmuscles.


1120.Featureswhichdifferentiateseizures
fromsyncopeincludethefollowing
except

a)Noimmediateprecipitatingfactorslikestress,valsalva,
orthostatichypotension
b)Immediatetransitiontounconciousness
c)Presenceofcyanosisandfrothingofmouth
d)Presenceofpremonitorysymptomslikediaphoresisand
tunnelingofvision
CorrectAnswer-D
Ans.D.Presenceofpremonitorysymptomslikediaphoresis
andtunnelingofvision


1121.Finetremorsarefoundinwhich
disorder
a)Mercurypoisoning
b)Excesssmoking
c)Hypoglycemia
d)Alltheabove
CorrectAnswer-D
Answer-D.Alltheabove
Finetremorsarenotedwhenalimbisheldinanantigravityposture
Theyarenotedinsituationsofcatecholamineexcesssuchas
anxietystates,thyrotoxicosis,hypoglycemiaandinalcoholismand
excesssmoking.
Theyarealsonotedafteringestionofdrugslikecaffeine,
salbutamol,theophylline,amphetamine,tricyclicantidepressants,
Lithium,valproate,steroids,andinmercurypoisoning.

1122.Trueaboutelectrophoresisinpatients
ofmultiplemyeloma
a)Mcomponentspikeisforthealphaglobulins
b)Monoclonalantibodymustbepresentataconcentrationofat
least10g/L[1.0g/dL]tobeaccuratelyquantitatedby
electrophoresis
c)McomponentisIgMin53%ofthepatients
d)McomponentisIgAin25%ofthepatients
CorrectAnswer-D
Answer-D.McomponentisIgAin25%ofthepatients
Theimmunoglobulinsmoveheterogeneouslyinanelectricfieldand
formabroadpeakinthegammaregion.Theyglobulinregionofthe
electrophoreticpatternisusuallyincreasedintheseraofpatients
withplasmacelltumors.Thereisasharpspikeinthisregioncalled
anMcomponent(Mformonoclonal).
TheserumMcomponentinmultiplemyelomawillbeIgGin53%of
patients,IgAin25%,andIgDin1%;20%ofMietitswillhaveonly
lightchainsinserumandurine.

1123.Allbutoneistrueforbetathalassemia
major
a)Growthanddevelopmentisimpaired
b)Redcellcount<4x10'2/L
c)LevelsofHbA2<3.5%
d)Bonemarrowironisdepleted
CorrectAnswer-D
Answer-D.Bonemarrowironisdepleted
AnaemiaHbgm/dl-<7(severe)
IncreasedHbF,HbA2andabsenceofHbA.
Severityofdisease++++
Growthanddevelopmentimpaired
Splenomegaly++++
Skeletalchanges+++
Thalassemiafacies
B.M.Iron-decreased

1124.Immunethrombocytopenicpuprpura
associatedwith
a)HepatitisAinfection
b)HepatitisBinfection
c)HepatitisCinfection
d)HepatitisDinfection
CorrectAnswer-C
Answer-C.HepatitisCinfection
Immunethrombocytopenicpurpura(ITP;alsotermedidiopathic
thrombocytopenicpurpura)isanacquireddisorderinwhichthereis
immune-mediateddestructionofplateletsandpossiblyinhibitionof
plateletreleasefromthemegakaryocyte.
ITPistermedsecondaryifitisassociatedwithanunderlying
disorder;autoimmunedisorders,particularlysystemiclupus
erythematosus(SLE),andinfections,suchasHIVandhepatitisC,
arecommoncauses.

1125.Whichofthefollowingisnotacauseof
secondaryIdiopathicthrombocytopenic
purpura?

a)Systemiclupuserythmatosus
b)HepatitisCinfection
c)Rheumatoidarthritis
d)HIVinfection
CorrectAnswer-C
Answer-C.Rheumatoidarthritis
ITPistermedsecondaryifitisassociatedwithanunderlying
disorder;autoimmunedisorders,particularlysystemiclupus
erythematosus(SLE),andinfections,suchasHIVandhepatitisC,
arecommoncauses.

1126.Drug/susedinmanagementrelapsed
multiplemyelomais
a)Bortezomib
b)Lenalidomide
c)Doxorubicin
d)Alltheabove
CorrectAnswer-D
Answer-D.Alltheabove
Thecombinationofbortezomibandliposomaldoxorubicinisactive
inrelapsedmyeloma.
Thalidomide,ifnotusedasinitialtherapy,canachieveresponsesin
refractorycases.
High-dosemelphalanandstemcelltransplant,ifnotusedearlier,
alsohaveactivityinpatientswithrefractorydisease.

1127.TreatmentofchronicphaseofCMLin
pregnantwomenis-
a)Imatinib
b)Leukapheresis
c)Spleenectomy
d)Interferontherapy
CorrectAnswer-B
Answer-B.Leukapheresis
Intensiveleukapheresismaycontrolthebloodcountsinchronic-
phaseCML;however,itisexpensiveandcumbersome.
Itisusefulinemergencieswhereleukostasis-relatedcomplications
suchaspulmonaryfailureorcerebrovascularaccidentsarelikely.
SplenectomywasusedinCMLinthepastbecauseofthe
suggestionthatevolutiontotheacutephasemightoccurinthe
spleen.
Splenicradiationisusedrarelytoreducethesizeofthespleen.

1128.Whichofthefollowingdrug/scanbe
usedforimmediateparenteral
anticoagulationinpatientswithvenous
thromboembolism?

a)Unfractionedheparin
b)Lowmolecularweightheparin
c)Fondaparinux
d)Alltheabove
CorrectAnswer-D
Answer-D.Alltheabove
Immediatelyeffectiveanticoagulationisinitiatedwithaparenteral
drug:unfractionatedheparin(UFH),low-molecular-weightheparin
(LMWH),orfondaparinux.
Oneshoulduseadirectthrombininhibitorargatroban,lepirudin,or
bivalirudininpatientswithprovenorsuspectedheparin-induced
thrombocytopenia.
Warfarinrequires5-7daystoachieveatherapeuticeffect.

1129.Apatientpresentswithicterus,but
thereisnoevidenceofbilirubinin
urine.Whatisthemostlikelycauseof
jaundiceinthispatient?

a)Hemolysis
b)Gallstones
c)Carcinomaheadofpancreas
d)Biliaryatresia
CorrectAnswer-A
Answer-A.Hemolysis
Hemolysisandhyperbilirubinemia
Increaseddestructionoferythrocytesleadstoincreasedbilirubin
turnoverandunconjugatedhyperbilirubinemia;the
hyperbilirubinemiaisusuallymodestinthepresenceofnormalliver
function.
Therefore,hemolysisalonecannotresultinasustained
hyperbilirubinemiaofmorethan-68umol/L(4mg/dL).
Whenhemolysisistheonlyabnormalityinanotherwisehealthy
individual,theresultisapurelyunconjugatedhyperbilirubinemia.

1130.Trueaboutsideroblasticanemia
a)SeverityofthediseasedependsontheresidualerythroidALA
synthaseactivity
b)Prussianbluestainingsideroblastsareobserved
c)Pyridoxinesupplementationcanbeusedfortreatment
d)Alltheabove
CorrectAnswer-D
Answer-D.Alltheabove
XLSAresultsfromthedeficientactivityoftheerythroidformofALA-
synthaseandisassociatedwithineffectiveerythropoiesis,
weakness,andpallor.
Typically,maleswithXLSAdeveloprefractoryhemolyticanemia,
pallor,andweaknessduringinfancy.
Peripheralbloodsmearsrevealahypochromic,microcyticanemia
withstrikinganisocytosis,poikilocytosis,andpolychromasia;the
leukocytesandplateletsappearnormal.
AvarietyofPrussianblue-stainingsideroblastsareobserved.Levels
ofurinaryporphyrinprecursorsandofbothurinaryandfecal
porphyrinsarenormal.

1131.Massivetransfusionisdefinedas
transfusionof
a)10/24ormoreredcellproductsinhours.
b)5/12ormoreredcellproductsinhours.
c)10/48ormoreredcellproductsinhours.
d)5/24ormoreredcellproductsinhours.
CorrectAnswer-A
Answer:A10/24ormoreredcellproductsinhours.
Variousdefinitionsofmassivebloodtransfusion(MBT)havebeen
publishedinthemedicalliteraturesuchas:
Replacementofoneentirebloodvolumewithin24h
Transfusionof>10unitsofpackedredbloodcells(PRBCs)in24h
Transfusionof>4unitsofPRBCsin1hwhenon-goingneedis
foreseeable

1132.Whichisthemostcommoncauseof
thrombocytopeniainanICUpatient?
a)Sepsis
b)Bonemarrowfailure
c)ITP
d)Druginduced
CorrectAnswer-A
Answer-A.Sepsis
Thrombocytopeniaisacommonlaboratoryabnormalitythathas
beenassociatedwithadverseoutcomesinICUpatients.
Thrombocytopeniaisdefinedasplateletcount<150X103cells/
mcL.
CommoncausesofthrombocytopeniainICUpatients:
Sepsis
Disseminatedintravascularcoagulation
Consumption(eg,majortrauma,cardiopulmonarybypass)
Dilution(withmassivetransfusion)
Myelosuppressivechemotherapy
Mechanicalcirculatorysupportdevices(eg,intra-aorticballoon
pump)
Lesscommonbutimportantcausesofthrombocytopeniathatshould
notbemissed:
Heparin-inducedthrombocytopenia
Hemophagocyticsyndrome
UncommoncausesofthrombocytopeniathatdevelopduringICU
admission
Drug-inducedthrombocytopenia(otherthanheparinorcytotoxic
chemotherapy)

Leukemia,myelodysplasia,aplasticanemia,etc,unless
abnormalitieswerealreadypresentbeforeICUadmission

1133.WhatpercentageofMultiplemyeloma
patientshavevertebralinvolvement?
a)22%
b)44%
c)66%
d)88%
CorrectAnswer-C
Answer-C.66%
Bonelesionsaremostcommoninthevertebralcolumn.The
followingdistributionwasseeninalargeseriesofcases:
Vertebralcolumn66%
Ribs-44%
Skull4%
Pelvis28%
Femur28%
Clavicle10%

1134.Multiagentchemotherapyinduces
remissioninofthepatientsofacute
myelogenousleukemia

a)65-70
b)75-80
c)85-90
d)95-100
CorrectAnswer-C
Answer-C.85-90
Aggressivemultiagentchemotherapyissuccessfulininducing
remissioninapproximately85-90%ofpatients.
Prognosticfeatures[t(8;21);t(15;17);inv(16);APL]andimproved
outcomewithchemotherapy,matchedsiblingstemcell
transplantationisrecommendedonlyafterarelapse
Matched-siblingbonemarroworstemcelltransplantationafter
remissionachieveslong-termdisease-freesurvivalinabouttwo
thirdsofpatients.

1135.Incaseofhypothyroidismwhich
investigationismostinformativeand
mostcommonlyused?

a)SerumTSHLevel
b)SerumT3,T4Level
c)SerumCalcitoninassay
d)SerumT3level
CorrectAnswer-A
AnsisAieSerumTSHLevel
"AnormalTSHLevelexcludesprimary(butnotsecondary)
hypothyrodism.IftheTSHiselevatedafreeT4levelisneededto
confirmthepresenceofclinicalhypothyrodism,butfreeT4isinferior
toTSHwhenusedasascreeningtest,asitwillnotdetect
subclinicalormildhypothyroidism.CirculatingfreeT3levelsare
normalinabout25%ofthepatients,reflectingadaptiveresposeto
hypothyroidism.T3measurementsarethereforenotindicated."-
Harrison
"Amongthevariousavailableserumthyroidfunctiontests,TSHis
themostusefultoassessglanddysfunction."?
EndocrineSurgeryofHead&Neck,p83.


1136.HbA1ccontrolforhowmuchtime
a)2-3weeks
b)3-6weeks
c)6-8weeks
d)14-18weeks
CorrectAnswer-C
Answer-C.6-8weeks
Thehalf-lifeofanerythrocyteistypically60days,thelevelof
glycatedhemoglobin(HbA1c)reflectsthemeanbloodglucose
concentrationoverthepreceding6-8weeks.

1137.HbA1Ccriteriaforapatienttobe
diagnosedwithdiabetesmellitusis
a)>4.5%
b)>5.5%
c)>6.5%
d)>7.5%
CorrectAnswer-C
Answer-C.>6.5%
Fastingplasmaglucose>(126mg/dl)
Twohourplasmaglucose>(200mg/d1)duringanoralGTT
A/C>6.5%

1138.AllareseeninMENIIAsyndrome
except
a)Medullarycarcinomaofthyroidisseenin100%ofthepatients
b)40-30%patientshavephaeochromocytoms
c)CausedbylossoffunctionmutationinIIRTprotooncogene
d)Primaryhyperparathyroidismisthemostvariablefeatureof
MENIIAsyndrome
CorrectAnswer-C
Answer-C.CausedbylossoffunctionmutationinIIRT
protooncogene
MEN-2AorSipplesyndrome,ischaracterizedby
pheochromocytoma,medullarycarcinomaofthethyroid,and
parathyroidhyperplasia.
Parathyroidhyperplasiaandevidenceofhypercalcemiaorrenal
stones.
MEN-2AisclinicallyandgeneticallydistinctfromMEN-Iandis
causedbygermlinegain-of-functionmutationsinthe
RETproto-oncogeneonchromosomel0qll.2.
40%to50%havepheochromocytomas.
PrimaryhyperparathyroidismisthemostvariablefeatureofMEN2A
syndrome.

1139.Whichofthefollowingisnottrueabout
thedevelopmentofthyroidtumorsin
nodulargoiter?

a)Prevalenceofthyroidcarcinomarangesbetween5-15%inthe
patientswithmultinodulargoiter
b)Papillarycarcinomaisthemostcommoncarcinomadeveloped
inpatientswithnodulargoiter
c)Bothbenignandmalignantneoplasmscanbeseeninpatients
withnodulargoiter
d)Theriskofdevelopmentofcarcinomaisnotcorrelatedwiththe
levelofTSH
CorrectAnswer-D
Answer-D.Theriskofdevelopmentofcarcinomaisnot
correlatedwiththelevelofTSH
Thyroidtumorsbothbenignandmalignantcanbeseenincolloid
goiterwithbothsolitaryandmultiplenodules.
Theprevalenceofthyroidcarcinomarangesfrom5?15%in
multinodulargoiterand8?17%insolitarycolloidnodules.
Theprevalenceinhigherinmencomparedtowomenandusually
occursinolderagegroup.
Themostcommonmalignanttumorarisinginmutinodulargoiteris
papillarycarcinoma.Otherlikefollicularcarcinoma,hurthlecell
carcinomaandmedullarycarcinomaarealsoencountered

1140.Tertiaryhyperparathyroidismis-
a)HighPO4levelwithmetastasis
b)SecondaryhyperparathyroidismwithCRF
c)PrimaryhyperparathyroidismwithlowCa"levels
d)Secondaryhyperparathyroidismwithchiefcelladenoma
CorrectAnswer-D
Ans.is'd'i.e.,Secondaryhyperparathyroidismwithchiefcell
adenoma
Davidsonstates"Inverysmallproportionofcasesofsecondary
hyperparathyroidismcontinuousstimulationoftheparathyroidmay
resultinadenomaformationandautonomousPTHsecretion.Thisis
knownastertiaryhyperparathyroidism".

1141.Indicationforgivingliothyronineas
therapeuticmanagementis
a)Resistantdepression
b)Socialphobia
c)Alzheimersdisease
d)Cataplexy
CorrectAnswer-A
Answer-A.Resistantdepression
Itisthesyntheticlevorotatoryisomeroftriiodothyronine(T3).
Liothyronineisthemostbroadlyusedthyroidhormonefortreatment
ofdepression.
Liothyronineisusedtoacceleratetheresponsetotricyclic
antidepressantsparticularlyinwomen.
Itisknowntoaugmentresponsetoantidepressantsinpatientswith
mooddisorders,inthosewhofailedtorespondtoatricyclic
antidepressanttrialie.Inpatientswithresistantdepression.

1142.Medicalmanagementof
hyperparathyroidismincludeswhichof
thefollowing?

a)Bisphosphonates
b)Calcitonin
c)Plicamycin
d)Alltheabove
CorrectAnswer-D
Answer-D.Alltheabove
Expansionofintravascularvolume,administrationofloopdiuretics,
pharmacotherapywhichreducesosteoclasticboneresorption(like
Bisphosphonates,Calcitonin,andPlicamycin)areusefulinthe
medicalmanagementofhyperparathyroidism.

1143.Whichofthefollowingcausesof
hypercalcemiaisnotassociatedwith
highboneturnover?

a)Hyperthyroidism
b)VitaminAintoxication
c)VitaminDintoxication
d)Thiazides
CorrectAnswer-C
Answer-C.VitaminDintoxication
VitaminDrelated
VitaminDintoxication
Increased1,25(OH)2Deg.Sarcoidosis
Idiopathichypercalcemiaofinfancy
Associatedhighboneturnover
Hyperthyroidism
Immobilization
Thiazides
VitaminAintoxication

1144.Fastinghypoglycemiaiscausedbythe
followingexcept
a)Alcoholintake
b)Pentamidinetherapy
c)Renalinsufficiency
d)Chronicpancreatitis
CorrectAnswer-D
Answer-D.Chronicpancreatitis
Inappropriate(High)InsulinLevel
Insulinreactioninpatientswithdiabetes-Thisisthemostcommon
causeofhypoglycemia,duetoanimbalancebetweeninsulinsupply
andinsulinrequirements.
Insulinsecretagogueoverdoseintype2diabetespatients-Insulin
secretagoguesareoralhypoglycemicagentsthatworkby
stimulatinginsulinreleasefrombetaisletcellsand,herefore,have
thepotentialtocausehypoglycemia.Sulfonylureas
(themostcommonlyprescribedtypeofthesemedications)are
clearedbythekidney,soelderlypatientswithcompromisedrenal
functionareatriskfordevelopinghypoglycemiawhileonthese
agents.
Factitioushypoglycemia(selfinducedorinadvertent)
Autoimmunehypoglycemia
Pentamidine-Pentamidineusedfortreatment/prophylaxisofPCPin
patientswithAIDScancausehypoglycemiabydirectinjurytothe
betaisletcellscausinghyperinsulinemia.
ExcessInsulinSecretion(Insulinoma)

1145.Diabetesinsipidusissaidtobepresent
when
a)>30ml/hrurineoutputin24hrsand<260mosml/Losmolarity
b)>40ml/hrurineoutputin24hrsand<280mosml/Losmolarity
c)>50ml/hrurineoutputin24hrsand<300mosml/Losmolarity
d)>60ml/hrurineoutputin24hrsand<320mosml/Losmolarity
CorrectAnswer-C
Answer-C.>50ml/hrurineoutputin24hrsand<300mosml/L
osmolarity
Decreasedsecretionoractionofargininevasopressinusually
manifestsasdiabetesinsipidus,asyndromecharacterizedbythe
productionofabnormallylargevolumesofdiluteurine.
DImustbedifferentiatedfromotheretiologyofpolyuria.
Thetestshouldbestartedinthemorningwithcarefulsupervisionto
avoiddehydration.
Bodyweight,plasmaosmolality,serumsodium,andurinevolume
andosmolalityshouldbemeasuredhourly.
Thetestshouldbestoppedwhenbodyweightdecreasesby5%or
plasmaosmolality/sodiumexceedtheupperlimitofnormal.
The24-hoururinevolumeis>50ml/kgbodyweightorurine
osmolality
MeasurementofAVPlevelsbeforeandafterfluiddeprivationmay
behelpfultodistinguishcentralandnephrogenicDI.
Occasionally,hypertonicsalineinfusionmayberequirediffluid
deprivationdoesnotachievetherequisitelevelofhypertonic
dehydration,butthisshouldbeadministeredwithcaution.

1146.Chronicadrenalinsufficiencyiscaused
bythefollowingorganismsexcept
a)Mycobacteriumtubercle
b)Histoplasmacapsulatum
c)Coccidioidesimmitis
d)Mycobacteriumbovis
CorrectAnswer-D
Answer-D.Mycobacteriumbovis
Infections,particularlytuberculosisandthoseproducedbyfungi,
causeprimarychronicadrenocorticalinsufficiency.
Whenpresent,tuberculousadrenalitisisusuallyassociatedwith
activeinfectioninothersites,particularlyinthelungsand
genitourinary
AIDSsufferersareatriskfordevelopingadrenalinsufficiencyfrom
severalinfectious(cytomegalovirus,Mycobacterium
aviumintracellulare)andnoninfectious(Kaposisarcoma)
complications.

1147.Whichofthefollowingisnotafeature
ofmyxedemacoma?
a)Reducedlevelofconsciousnessandseizureswithother
featuresofhypothyroidismisseen
b)Hypoventilationleadingtohypoxiaandhypercapnia
c)Levothyroxinecanbegivenviaintravenousandnasogastric
route
d)Levothyroxineshouldnotbeusedinthemanagement
CorrectAnswer-D
Answer-D.Levothyroxineshouldnotbeusedinthe
management
Myxedemacomaisdefinedasseverehypothyroidismleadingto
decreasedmentalstatus,hypothermia,andothersymptomsof
hypothyroidism.
Reducedlevelofconsciousness,sometimesassociatedwith
seizuresmayalsobeseen.
Factorsthatpredisposetomyxedemacomaincludecoldexposure,
trauma,infection,andadministrationofnarcotics.
Therapyformyxedemacomashouldincludelevothyroxine(500g)
asasingleIVbolusfollowedbydailytreatmentwithlevothyroxine
(50?100g/d),alongwithhydrocortisone(50mgevery6h)for
impairedadrenalreserve,ventilatorysupport,spaceblankets,and
treatmentofprecipitatingfactors.

1148.Acuteadrenalinsufficiencycanpresent
as
a)Acuteabdomenwithabdominaltenderness,nausea,vomiting
andfever
b)Neurologicdiseasewithdecreasedresponsivenessprogressing
tostuporandcoma
c)Hypovolemicshock
d)Alloftheabove
CorrectAnswer-D
Answer-D.Alloftheabove
Posturalhypotensionmayprogresstohypovolemicshock.
Adrenalinsufficiencymaymimicfeaturesofacuteabdomenwith
abdominaltenderness,nausea,vomiting,andfever.
Insomecases,theprimarypresentationmayresembleneurologic
disease,withdecreasedresponsiveness,progressingtostuporand
coma.
Anadrenalcrisiscanbetriggeredbyanintercurrentillness,surgical
orotherstress,orincreasedglucocorticoidinactivation(e.g.,
hyperthyroidism).

1149.Thegoldstandardtestfordiagnosisof
Insulinomais:
a)'72hour'fasttest
b)PlasmaGlucoselevels<3mmol/l
c)PlasmaInsulinlevels>6?U/ml
d)C-peptidelevels<50pmol/e
CorrectAnswer-A
TheanswerisA('72hour'fasttest):
TheGoldstandardtestfordiagnosisofInsulinomaisasupervised
'72hourfast'test
Diagnosisofinsulinomarequiresdemonstrationofinappropriately
highlevelsofplasmaInsulin(andC-peptide)inthepresenceof
documentedhypoglycemia(Achievedby72-hourfasttest).
AbsolutevaluesofInsulinorC-peptidearenotreliablein
establishingadiagnosisunlesshypoglycemiaisdocumentedThe
'72hourfasttest'allowsdemonstrationofhypoglycemia,together
willelevatedlevelsofInsulinandC-peptideandthusbecomesthe
mostreliable?goldstandardtestforestablishingadiagnosisof
Insulinoma
DiagnosisofInsulinoma:'72hourfast'test
ThediagnosisofInsulinomarequiresthedemonstrationofan
inappropriatelyelevatedplasmainsulin(andC-peptide)atthetime
ofhypoglycemia.
The'72hourfast'testinvolvessupervisedfastingforupto72hours
oruntilhypoglycemiacanbedocumented.
Thetestisconsideredpositiveifatanytimewhenbloodglucose
levelsdropto<2.2mmo1/1(40mg/dl),theseruminsulinlevelsare
recordedtobegreaterthan6?U/ml.(andC-peptidelevels>100

pmo1/1)
Studiesindicatethat100%ofpatientswithinsulinomawillbe
detectedafterasupervised72hourfastandhencethistestis
consideredthegoldstandardtest.
First24hours:70-80%ofpatientswithinsulinomacanbe
detected
Upto48hours:98%ofpatientswithinsulinomacanbe
detected
By72hours:100%ofpatientswithinsulinomacanhe
detected


1150.Hypotonicsolutiongiventocorrect
a)Dehydrationsecondarytodiuretictherapy
b)Diabeticketoacidosis
c)Hyperosmolar,hyperglycemicnonketoticsyndrome
d)Alltheabove
CorrectAnswer-D
Answer-D.Alltheabove
Hypotonicsolutionhasosmolaritylowerthanseumosmolarity.
Whenapatientreceiveshypotonicsolution,fluidshiftsoutofthe
bloodvesselsandintothecellsandinterstitialspaces,where
osmolarityishigher.
Hypotonicsolutionhydratescellswhilereducingfluidinthe
circulatorysystem.
Indications
Dehydrationsecondarytodiuretictherapy.
Diabeticketoacidosis
Hyperosmolar,hyperglycemicnonketoticsyndrome
Examplesofhypotonicsolutions:halfnormalsaline,0.33%sodium
chloride,dextrose2.5%inwater,dextrose2.5%.

1151.Allofthefollowingarecausesofacute
hyponatremiaexcept
a)GlycineirrigationinTURP
b)Recentinstitutionofthiazidetherapy
c)MDMAingestion
d)Liquoriceingestion
CorrectAnswer-D
Answer-D.Liquoriceingestion
causesofhyponatremia
Iatrogenic
Postoperative:premenopausalwomen
Hypotonicfluidswithcausesof1'vasopressin
Glycineirrigation:TURP,uterinesurgery
Colonoscopypreparation
Recentinstitutionofthiazides
PolydipsiaMDMAingestion
Exercise-induced
Multifactorial,e.g.,thiazideandpolydipsia

1152.Acutehyponatremiabecomes
symptomaticat
a)<135mEq
b)<125mEq
c)<120mEq
d)<110mEq
CorrectAnswer-B
Ans.is'b'i.e.,<125mEq
Serumlevelofsodiumatwhichsymptomsdevelop
Acute<125meq/L
Chronic<120meq/L
Hyponatremiaiscommonlydefinedasaserumsodium<135
mmol/L(<135mEq/L).Neurologicalsymptoms
occuratdifferentlevelsoflowsodium,dependingnotonlyonthe
absolutevaluebutalsoontherateoffall.
Inpatientswithhyponatremiathatdevelopsoverhours,life-
threateningseizuresandcerebraledemamayoccur
atvaluesashighas125mmol/L.
Incontrast,somepatientswithmorechronichyponatremiathathas
slowlydevelopedovermonthstoyearsmaybeasymptomaticeven
withserumlevels<110mmol.
Acuteorhvperacutehvponatremia
Thehyponatremiadevelopedwithintheprevious24hours,itis
called"acute."
Ifthehyponatremiadevelopedoverjustafewhoursduetoa
markedincreaseinwaterintake(self-inducedwaterintoxication,as
maybeseeninmarathonrunners,psychoticpatients,andusersof
ecstasy),itiscalled"hyperacute."

Chronichyponatremia
Ifitisknownthatthehyponatremiahasbeenpresentfbrmorethan
48hours,orifthedurationisunknown(suchasinpatientswho
develophyponatremiaathome),itiscalled"chronic."
Mildtomoderatehyponatremia
Mildhyponatremiaisusuallydefinedasaserumsodium
concentrationbetween130and135meq/L.
Moderatehyponatremiaisoftendefinedasaserumsodium
concentrationbetween121and129meq/L.
Severehvponatremia
Severehyponatremiacanbedefinedasaserumsodiumof120
meq/Lorless.
Symptomsofhvponatremia
Absentsymptoms
Patientswithhyponatremiaarefrequentlyasymptomatic,particularly
ifthehyponatremiaischronicandofmildormoderateseverity(ie,
serumsodium>120meq/L).
However,suchpatientsmayhavesubclinicalimpairmentsin
mentationandgait.
Mildtomoderatesymptoms
Mildtomoderatesymptomsofhyponatremiaarerelatively
nonspecificandincludeheadache,nausea,vomiting,fatigue,gait
disturbances,andconfusion.
Inpatientswithchronichyponatremia(ie,>48hoursduration),these
findingsarenotassociatedwithimpendingherniation;however,in
patientswithmoreacutehyponatremia,suchsymptomsshouldbe
consideredominousandmayevolvewithoutwarningtoseizures,
respiratoryarrest,andherniation.
Severesymptoms
Severesymptomsofhyponatremiainclude
uSeizures
Obtundation
Coma
Respiratoryarrest.

1153.Respiratoryacidosisisrecognized
primarilybyincreasein
a)Pa02
b)PaCO2
c)HCO3
d)Noneoftheabove
CorrectAnswer-B
Answer-B.PaCO2
RespiratoryacidosisoccurswhenthereisaccumulationofCO2due
totypeIIrespiratoryfailure.itcanalsooccurduetosevere
pulmonarydisease,respiratorymusclefatigue,orabnormalitiesin
ventilatorycontrolandisrecognizedbyanincreaseinPaco2and
decreaseinpH
ThisresultsinariseinthePCO2,withacompensatoryincreasein
plasmabicarbonateconcentration,particularlywhenthedisorderis
oflongdurationandthekidneyhasfullydevelopeditscapacityfor
increasedacidexcretion

1154.Whichofthefollowingdrug
administrationisnotassociatedwith
hypomagnesemia?

a)Cisplatin
b)Valproate
c)Foscarnet
d)Cetuximab
CorrectAnswer-B
Answer-B.Valproate
Drugscausinghypomagnesemia
Ethanol
Diuretics(loop,thiazide,osmotic)
Cisplatin
Pentamidine,foscarnetCyclosporine
Aminoglycosides,amphotericinB
Cetuximab

1155.DoseofbenzathainepenicillinGtobe
giveninpatientsoflatentsyphilisin
patientswithoutpenicillinallergyand
normalCSFfindingsis

a)0.6mUIM/weekfor3weeks
b)1.2mUIM/weekfor3weeks
c)2.4mUIM/weekfor3weeks
d)4.8mUIM/weekfor3weeks
CorrectAnswer-C
Answer-C.2.4mUIM/weekfor3weeks
Primary,secondary,orearlylatent-CSFnormalornotexamined:
PenicillinGbenzathine(singledoseof2.4mUIM)
CSFabnormal-Treatasneurosyphilis
Latelatent(orlatentofuncertainduration),cardiovascular,orbenign
tertairy-CSFnormalornotexamined:PenicillinGbenzathine(2.4
mUIMweeklyfor3weeks).
CSFabnormal:Treatasneurosyphilis

1156.SARSinfectioncasefatalityrateof
>50%isobservedinpatientsofwhich
agegroup?

a)<20yrs
b)20-40years
c)40-60years
d)>65years
CorrectAnswer-D
Answer-D.>65years
ThecasefatalityratefromSARS-CoVinfectionduringthe2003
outbreakwas10-17%.Nopediatricdeathswerereported.The
estimatedcasefatalityrateaccordingtoagevariedfrom<1%for
thoseyoungerthan20yearofageto>50%forthoseolderthan65
yrofage.

1157.Whichofthefollowingcorroboratesto
thepresenceofclostridiumdifficle
infectioninpatientstakingantibiotics
foranothercause?

a)Diarrhoeaunformedstoolsper12hfor2dayswithnoother
recognizedcause
b)Diarrhoeaunformedstoolsper24hfor2dayswithnoother
recognizedcause
c)Diarrhoeaunformedstoolsper24hfor3dayswithnoother
recognizedcause
d)Diarrhoeaunformedstoolsper24hfor4dayswithnoother
recognizedcause
CorrectAnswer-B
Answer-B.Diarrhoeaunformedstoolsper24hfor2dayswith
nootherrecognizedcause
Diarrhoeaunformedstoolsper24hfor2dayswithnoother
recognizedcause.
ToxinAorBdetectedinthestoolbyPCRorculture.
Pseudomembranesseenincolonbyendoscopy.

1158.Followingarethefeaturesof
neuropathyassociatedwithvaricella-
zosterinfectionexcept

a)Persistentinfectioninneuronsofsensoryganglia
b)Withreactivationvirustransportedalongnervestoskin
c)Shinglesaredistributedalongmotordermatomes
d)Intranuclearinclusionsarenotfoundinperipheralnervous
system
CorrectAnswer-C
Answer-C.Shinglesaredistributedalongmotordermatomes
Varicella-zosterisoneofthemostcommonviralinfectionsofthe
peripheralnervoussystem.
Followingchickenpox,alatentinfectionpersistswithinneuronsof
sensoryganglia.
Ifthevirusisreactivated,sometimesmanyyearslater,itmaybe
transportedalongthesensorynervestotheskin.
Hereitinfectskeratinocytes,leadingtoapainful,vesicularskin
eruption(shingles)inadistributionthatfollowssensorydermatomes
Mostcommonistheinvolvementofthoracicortrigeminalnerve
dermatomes.

1159.Accordingtothecongenitalrubellasyndromeeradicationprogram,the
firstpriorityforrubellavaccinationisofferedtowhichofthefollowing
group?

a)Allfemalechildrenatoneyear
b)Allnonpregnantwomen
c)Allnonpregnantwomenofage15to34
d)Alladolescentnonpregnantgirls15to24yearsofage
CorrectAnswer-C
Thefirstandforemostpriorityhasbeengiventothegroupofallnon-pregnantwomen
betweentheages15and34forrubellavaccination.Theotherthreeoptionsaresuitable
agegroups.
Ref:Park'sTextbookofPreventiveandSocialMedicine19thedition;pages130-131.

1160.ExtrahepaticManifestationsofHepatitis
CincludeallofthefollowingExcept:
a)LichenPlanus
b)CeliacDisease
c)Glomerulonephritis
d)Cryoglobulinemia
CorrectAnswer-B
AnswerisB(Celiacdisease)
ExtrahepaticmanifestationsinviralhepatitisC:Wepatology'by
Kuntz
Agranulocytosis
Aplasticanaemia
Cornealulceration
Ciyoglobillinaemia
Diabetesmellitus(typeI)
Erythemaexsudativummultiforme
Glomerulonephritis
Guillain-Barresyndrome
Hyperlipasaemia
Lichenplanus
Non-Hodgkinlymphoma
Polyarteritisnodosa
Polyarthritis
Polyneuritis
Porphyriacutaneatarda
Sialadenitis
Sjogren'ssyndrome/Siccasyndrome
Thrombocytopenia

Thyroiditis


1161.AllarefeaturesofSIRSexcept-
a)RR>24&Paco2<22mmhg
b)WBC>11or<4
c)Temperature<36and>38
d)PR>90
CorrectAnswer-A
Answer-A.RR>24&Paco2<22mmhg

1162.Whichofthefollowingisnottrueabout
theepididymo?orchitisofmumps?
a)Itisthemostcommonmanifestationofmumpsinfection
b)Testicularenlargementusuallyresolvesin1week
c)Bilateraltesticularinvolvementseenin10-30%ofcases
d)Sterilityrarelydevelopsinthesepatients
CorrectAnswer-A
Answer-A.Itisthemostcommonmanifestationofmumps
infection
Epididymo-orchitisisthesecondmostcommonmanifestationof
mumps,developingin15?30%ofcasesinpostpubertalmales.
?Orchitis,characterizedbyapainful,tender,feverandenlarged
testis,isbilateralin10?30%ofcasesandresolveswithin1week.
?Oophoritis(manifestedbylowerabdominalpainandvomiting)
occursin~5%ofwomenwithmumps.
?Sterilityinmumpsisrare.

1163.Mostcommonnerveaffectedin
leprosy
a)Posteriortibial
b)Ulnar
c)Median
d)Facial
CorrectAnswer-A
Answer-A.Posteriortibial
Leprosyaffectsperipheralmixednervesandcutaneousnerves.
Themostcommonperipheralnervesaffectedintheorderof
frequencyaretheposteriortibial>ulnar>median>lateralpopliteal>
facial>radial

1164.Dengueshocksyndromeis
characterizedbythefollowingexcept-
a)Hepatomegaly
b)Pleuraleffusion
c)Thrombocytopenia
d)Decreasedhaemoglobin
CorrectAnswer-D
Ans.is'd'i.e.,Decreasedhemoglobin
Denguehemorrhagicfever
FeverMinororMajorhemorrhgic
manifestations
HepatomegalyThrombocytopenia100,000/mm3
HypoalbuminemiaObjectiveevidenceofincreased
capillarypermeability(hematocrit20%.)
Pleuraleffusion(bychestradiograph)[Nelson,17/e,p1093]
CriteriaforDengueshocksyndrome
Itincludesthosefordenguehemorrhagicfeverplus,Hypotensionor
narrowpulse.

1165.Thefollowingstatementsaretrue
regardingbotulismexcept-
a)Infantbotulismiscausedbyingestionofpreformedtoxin
b)ClostridiumbotulinumA,B,CandFcausehumandisease
c)Thegeneforbotulinumtoxinisencodedbyabacteriophage
d)Clostridiumbarattimaycausebotulism
CorrectAnswer-A
Ans.is'a'i.e.,Infantbotulismiscausedbyingestionof
preformedtoxin
Infantbotulismiscausedbyingestionofspores.Sporesare
ingestedinfood,getestablishedinthegutandthereproducethe
toxin.
"SevenmaintypesofC.botulinum,designatedA-G,produce
antigenicallydistincttoxinswithpharmacologicallyidenticalaction.
Alltypescancausehumandisease,buttypeA,BandEaremost
common".
(InHarrison&Ananthanarayan,eighttypesofC.botulinumA,B,CI,
C2,D,E,F,Ghavebeenmentioned).
Toxinproductioninclostridiumbotulinumappearstobedetermined
bypresenceofbacteriophage(atleastintypeC&D).
"Clostridiumbutyricumandclostridiumbarattihavealsobeenfound
toproducetoxin".-Harrison16th/e843-Anystrainproducingtoxin
willobviouslycausebotulism.

1166.Austriansyndromeiscausedbywhich
infection
a)Staphylococcusaureus
b)Streptococcuspneumoniae
c)Staphylococcusepidermidis
d)Streptococcusviridans
CorrectAnswer-B
Answer-B.Streptococcuspneumoniae
AustriansyndromeisamedicalconditionfirstdescribedbyRobert
Austrianin1957.
Theclassicaltriadconsistsofpneumonia,endocarditis,and
meningitis,allcausedbyStreptococcuspneumoniae.
Itisassociatedwithalcoholism,duetothepresenceofhyposplenia
(reducedsplenicfunctioning),andcanbeseeninmalesbetween40
and60yearsold.

1167.Factorscontributingtothe
developmentofcomplicationsin
measlesarethefollowingexcept-

a)Agegroup5-20years
b)Highercasefatalitywithovercrowding
c)Severemalnutrition
d)Lowerserumretinollevels
CorrectAnswer-A
Answer-A.Agegroup5-20years
Complicationsofmeaslesarelargelyattributabletothepathogenic
effectsofthevirusontherespiratorytractandimmunesystem.
Morbidityandmortalityfrommeaslesaregreatestinpatients
youngerthan5yrofage(especially<1yrofage)andolderthan20
yrofage.

1168.TrueaboutVHLsyndromeis
a)Itisanautosomalrecessivecondition
b)Centralnervoussystemisnotinvolved
c)Regularscreeningforclearcellcarcinomaofkidneysis
essential
d)VHLisagrowthpromotergene
CorrectAnswer-C
Answer-C.Regularscreeningforclearcellcarcinomaof
kidneysisessential
VonHippel-Lindaudisease(VHL)isarareautosomaldominant
diseasecharacterizedbyabnormalangiogenesiswithbenignand
malignanttumorsthataffectmultipletissues.
ThediseaseisinheritedasamutationinonealleleoftheVHL
tumor-suppressorgene.
Somaticmutationofthenormalalleleleadstoretinalangiomas,
centralnervoussystem(CNS)hemangioblastomas,
pheochromocytomasandmulticentricclearcellcysts,
hemangiomas,andadenomasofthekidney.
Thehihriskofrenalcellcarcinomamandatesieriodicsurveillance
usuallearlinadultsbCTorMRI.Routinescreeningandawareness
ofthenaturalhistoryoflesionshasenabledrenal-sparing
approachestodiseasemanagement.

1169.Tuberoussclerosisiscausedby
mutationsinthefollowingproteins
a)Hamartin
b)Tuberin
c)Merlin
d)Ankyrin
CorrectAnswer-A:B
Answer-A.Hamartin&B.Tuberin
ItiscausedbymutationsineithertheTSC1gene,whichmapsto
chromosome9q34,andencodesaproteintermedhamartin,or
mutationsintheTSC2gene,whichmapstochromosome16p13.3
andencodesthetuberinprotein.
Hamartinformsacomplexwithtuberin,whichinhibitscellular
signalingthroughthemammaliantargetofrapamycin(mTOR),and
actsasanegativeregulatorofthecellcycle.
Patientswithtuberoussclerosishaveseizures,mentalretardation,
adenomasebaceum(facialangiofibromas),shagreenpatch,
hypomelanoticmacules,periungualfibromas,renal
angiomyolipomas,andcardiacrhabdomyomas.

1170.Whatcharacteristicfindingoftuberous
sclerosisispresentatbirthbutnotlater
inlife?

a)Cardiacrhabdomyosarcoma
b)Facialangiofibroma
c)Periungalfibroma
d)Renalangiomyolipoma
CorrectAnswer-A
Answer-A.Cardiacrhabdomyosarcoma
Patientswithtuberoussclerosishaveseizures,mentalretardation,
adenomasebaceum(facialangiofibromas),shagreenpatch,
hypomelanoticmacules,periungualfibromas,renal
angiomyolipomas,andcardiacrhabdomyomas.
Cardiacrhabdomyosarcomascanbepresentatbirthinupto80%of
theinfantswithtuberoussclerosis.Theseinvoluteinthefirstthree
yearsoflifeandcompletelydisappearbyadulthood

1171.Followingisnottrueaboutepinephrine
a)Haspotentalphaandbetastimulatingproperties
b)Itimprovescoronaryperfusionpressureandmyocardialblood
flow
c)IncreasescerebralbloodflowduringCPR
d)Routineuseofhighdoseepinephrineduringresuscitationis
indicated
CorrectAnswer-D
Answer-D.Routineuseofhighdoseepinephrineduring
resuscitationisindicated
Epinephrine(adrenaline)isanendogenouscatecholaminewith
potenta-and11-adrenergicstimulatingproperties.
Theadrenergicaction(vasoconstriction)increasessystemicand
pulmonaryvascularresistance.Theresultanthigheraorticdiastolic
bloodpressureimprovescoronaryperfusionpressureand
myocardialbloodfloweventhoughitreducesglobalcardiacoutput
duringCPR.
epinephrinealsoincreasescerebralbloodflowdurinooduuali
CPRbecauseperipheralvasoconstrictiondirectsagreater
proportionofflowtothecerebralcirculation.However,epinephrine
candecreaselocalcerebralmicrocirculatorybloodflowatatime
whenglobalcerebralflowisincreased.

1172.Ztracktechniquemustbeusedfor
administrationof
a)InjectionIronDextrandeepIM
b)InjectionHydroxyzinehydrochloridedeepIM
c)InjectionDepomedroxyprogesteroneiv
d)Injectionerythromycin
CorrectAnswer-A:B
Answer-A.InjectionIronDextrandeepIM&B.Injection
HydroxyzinehydrochloridedeepIM
Withintramuscularinjectionsmedicationscanleakupwardintothe
subcutaneoustissuescausingstaining,bruisingandsignificantpain
forseveralweeksorlongerwithsomemedications.
NursesareencouragedtousetheZtracktechnique(causinga
needletrackorpathwayintheshapeofZ)anytimeanintramuscular
injectionisgiven,topreventleakageandassociatedpain.
TheZtracktechniquemustbeusedwheneveradeepintramuscular
injectionofirondextran,andotherirritatingsolutionssuchas
hydroxyzinehydrochlorideandseveralantipsychoticagentsare
given.

1173.Inmanwhatquantityofethylalcohol
consumeddailyfor>10years
increasestherelativeriskof
developmentofalcoholicliverdisease

a)20g/d
b)40g/d
c)60g/d
d)80g/d
CorrectAnswer-D
Answer-D.80g/d
80g/dayx10+yr.

1174.NotafeatureofWernicke'sKorsakoff
Syndrome
a)Ataxia
b)Psychosis
c)Normalpupillaryresponse
d)Opthalmoplegia
CorrectAnswer-C
Ci.e.Normalpupillaryresponse

1175.Whichofthefollowingantineoplastic
agentsisusedinthemanagementof
Hodgkinslymphoma,nonHodgkins
lymphomaandsmallcellcarcinomaof
lung?

a)Cisplatin
b)Bleomycin
c)Paclitaxel
d)Doxorubicin
CorrectAnswer-A
Answer-A.Cisplatin
CisplatinisusedinthemanagementofHodgkinslymphoma,non
Hodgkinslymphomaandsmallcellcarcinomaoflung.

1176.Pierrerobinsyndromefollowingistrue
except
a)Consistsofmicrognathiaandcleftpalate
b)Tongueisofnormalsize
c)Airwayobstructionparticularlyduringexpiration
d)30-50%patientshaveSticklersyndrome
CorrectAnswer-C
AnswerC.Airwayobstructionparticularlyduringexpiration
PierreRobinsyndromeconsistsofmicrognathiausually
accompaniedbyahigharchedorcleftpalate.
Thetongueisusuallyofnormalsize,butthefloorofthemouthis
foreshortened.
Theairpassagescanbecomeobstructed,particularlyoninspiration,
usuallyrequiringtreatmenttopreventsuffocation.
Theinfantshouldbemaintainedinaproneorpartiallyproneposition
sothatthetonguefallsforwardtorelieverespiratoryobstruction.
Somepatientsrequiretra-cheostomy.
Mandibulardistractionproceduresintheneonatecanimprove
mandibularsize,enhancerespiration,andfacilitateoralfeedings.
Sufficientspontaneousmandibulargrowthcantakeplacewithina
fewmonthstorelievethepotentialairwayobstruction.
Oftenthegrowthofthemandibleachievesanormalprofilein4-6
year.

1177.Mantlefieldradiationwasusedfor
managementof-
a)Hodgkinslymphoma
b)Mantlecelllymphoma
c)Multiplemyeloma
d)Cervicalcarcinoma
CorrectAnswer-A
Answer-A.Hodgkinslymphoma
Mantlefieldradiationisatypeofradiationtreatmentusedfor
Hodgkin'slymphoma
Theterm'mantle'isderivedfromthenameofagarment,muchlike
acloak,usedmanyyearsback.Theshapeoftheexposedareathe
radiationfieldhascontoursthatresembletheshieldingcloak.
Thistypeoflargeradiationfieldisnotcommonlyusedtoday.

1178.Alienlimbsyndromeseenin
a)Postneurosurgicalcases
b)Alzheimersdisease
c)Creutzfeldt-Jakobdisease
d)Alltheabove
CorrectAnswer-D
Answer-D.Alltheabove
Alienhandsyndrome(AHS)isaconditioninwhichaperson
experiencestheirlimbsactingseeminglyontheirown,without
controlovertheactions.
Thetermisusedforavarietyofclinicalconditionsandmost
commonlyaffectsthelefthand.
Alienhandsyndromeisbestdocumentedincaseswhereaperson
hashadthetwohemispheresoftheirbrainsurgicallyseparated,a
proceduresometimesusedtorelievethesymptomsofextreme
casesofepilepsy.
Italsooccursinsomecasesafterbrainsurgery,stroke,infection,
tumor,aneurysmandspecificdegenerativebrainconditionssuchas
Alzheimer'sdiseaseandCreutzfeldt-Jakobdisease.

1179.OsmolarityofMilkF-100is
a)399mOsm/L
b)409mOsm/L
c)419mOsm/L
d)429mOsm/L
CorrectAnswer-C
Answer-C.419mOsm/L
UNICEFandWHOpreparedtwoformuladietsbymodificationofthe
cowsmilk-MilkF-75(starter75kcal/100ml)andF-100(followup
100kcal/100ml).

1180.Whichofthefollowingdrugscancause
seizuresexcept?
a)Lithium
b)Phencyclindine
c)INH
d)Ketorolac
CorrectAnswer-D
Answer-D.Ketorolac
Psychotropics
Antidepressants
Antipsychotics
Lithium
DrugsofabuseAmphetamineCocaine
PhencyclidineMethylphenidateFlumazenil

1181.Riboflavindeficiencycauses
a)Cornealvascularization
b)Anemia
c)Personalitychanges
d)Alltheabove
CorrectAnswer-D
Answer-D.Alltheabove
Riboflavindeficiencyismanifestedprincipallybylesionsofthe
mucocutaneoussurfacesofthemouthandskin.Inadditiontothe
mucocutaneouslesions,cornealvascularization,anemia,and
personalitychangeshavebeendescribedwithriboflavindeficiency.

1182.Soretbandinwhichporphyrinsabsorb
lightlieatwhatwavelengthofthe
spectrumoflight?

a)200nm
b)300nm
c)400nm
d)500nm
CorrectAnswer-C
Answer-C.400nm
Duetothisstructureporphyrinsavidlyabsorblightinaregionnear
400nmofthelightspectrum.Thispartofthelightspectrumiscalled
theSoretband.

1183.Whichofthefollowingisnotseenafter
nervetransection?
a)Morphologicpatternofwalleriandegeneration
b)Myelinovoids
c)Painfulneuroma
d)Neuromaincontinuity
CorrectAnswer-D
Answer-D.Neuromaincontinuity
Themorphologichallmarksofaxonalneuropathiesproducedby
cuttingaperipheralnerve,resultsinaprototypicalpatternofinjury
describedasWalleriandegeneration
Withinadayofinjury,thedistalaxonsbegintofragmentandthe
associatedmyelinsheathsunravelanddisintegrateintospherical
structures(myelinovoids).
Afailureoftheoutgrowingaxonstofindtheirdistaltargetcan
producea"pseudotumor"termedtraumaticneuroma--a
nonneoplastichaphazardwhorledproliferationofaxonalprocesses
andassociatedSchwanncellsthatresultsinapainfulnodule.

1184.Reactivenitrogenspeciesforkillingof
microbesaremainlyderivedfrom
a)Elementalnitrogen[N3]
b)NitricOxide[NO]
c)NitrogenDioxide[NO2]
d)NitrousOxide[N20]
CorrectAnswer-B
Answer-B.NitricOxide[NO]
Killingofmicrobesisaccomplishedbyreactiveoxygenspecies
(ROS,alsocalledreactiveoxygenintermediates)andreactive
nitrogenspecies,mainlyderivedfromnitricoxide(NO),andtheseas
wellaslysosomalenzymesdestroyphagocytoseddebris.
Thisisthefinalstepintheeliminationofinfectiousagentsand
necroticcells.

1185.Duringstateofarousalinmen
relaxationofsmoothmuscleincorpus
cavernosumismainlycausedby

a)Acethycholine
b)Nitricoxide
c)Bicarbonateions
d)Calcium
CorrectAnswer-B
Answer-B.Nitricoxide
Erectiledysfunction(ED)referstotheinabilityofmentoattainand
maintainanerectpeniswithsufficientrigiditytoallowsexual
intercourse.
Nitricoxide(NO)releasedparasympatheticnonadrenergic
noncholinergic(NANC)nervesandvascularendotheliumisthemajor
transmittercausingrelaxationofsmoothmuscleincorpus
cavernosumandbloodvesselssupplyingit;AChandPGsalsoplay
arole.

1186.Whichofthefollowinggenesifaffected
willsporadicallycauseJuvenile
myeloidleukemia?

a)NF1
b)PTEN
c)APC
d)SMAD2
CorrectAnswer-A
Answer-A.NF1
NF1-Neurobiastoma,juvenilemyeloidleukemia

1187.ChronicmanifestationsofAspergillosis
arenotevidentinwhichofthefollowing
organs?

a)Skin
b)Brain
c)Lung
d)Eye
CorrectAnswer-D
Answer-D.Eye
Lung,sinus,brain,skin,heart,

1188.Allofthefollowingaretrueaboutincontinentiapigmenti,except:
a)Ocularinvolvementisseeninalmost100%casesandis
typicallyunilateral
b)Avascularityofperipheralretina
c)Primaryskinabnormality
d)X-linkeddominant
CorrectAnswer-A
Ocularinvolvementisseeninabout20-35percentofthecasesofincontinentiapigmenti
butnotin100percentcasesasmentionedintheoption.IncontinentiapigmentiisaX-linked
dominantprimaryskindiseasethatleadstoavascularityoftheretina.
Ref:Rook'sTextbookofDermatology7thEdition,Pages39.20-3.22;TheRetinalAtlasBy
LawrenceA.Yannuzzi,Page38

1189.Oculogyriccrisisisknowntobe
producedbyallofthefollowingdrugs
except

a)Trifluoperazine
b)Atropine
c)Perchlorperazine
d)Perphenazine
CorrectAnswer-B
Answer-B.Atropine
Oculogyriccrisisisoneofthemanifestationsseeninacutedystonic
reaction(acutemusculardystonia).
Othermanifestationsarefacialgrimacing,torticollis,lockedjaw,
abnormalcontractionofspinalmuscles(opisthotonus).
Itoccurswithin1to5daysofantipsychotictherapy.
Trifluperazine,perchlorperazineandperphenazineareantipsychotic

1190.Whichofthefollowingprimarily
governstheuptakeofTc-99mMDPin
body?

a)Amountofosteogenicactivity
b)Amountofiodineuptake
c)Amountofcalciumuptake
d)Amountofcatecholamineactivity
CorrectAnswer-A
Answer-A.Amountofosteogenicactivity
Technetium99-misacommonlyusedradiopharmaceutical.
Technetium99-mmethylenediphosphonate(Tc-99mMDP),
desirableforthegammacarmeraimagingisthecommonlyused
form.
Tc-99mMDPcanbepreparedfromakitcontaining,sodium
pertechnitate(NaTcO4)vial,MDP,stabilizersandstannousion.

1191.Technitium-99mpertechnetatelabelled
methylenediphosphonateis
structurallysimilarto

a)Calciumphosphate
b)Phosphorus
c)Sodiumbicarbonate
d)Magnesiumsulfate
CorrectAnswer-A
Answer-A.Calciumphosphate
TheradionucletideadministeredisTechnitium-99mpertechnetate
labeledmethylenediphosphonateisananalogofcalcium
phosphate.

1192.Laproscopicprocedurepatient
developsshoulderpaindueto
a)Subphrenicabscess
b)Positionalpainduringsurgery
c)Subdiaphragmaticmigrationofgas
d)Injurytoliver
CorrectAnswer-C
Answer-C.Subdiaphragmaticmigrationofgas
Onetypeofpainthatisuniquetolaparoscopyisthepost
laparoscopyshoulderpainduetothephrenicnerveirritationtothe
diaphragmcausedbytheCO,gasthatremainsintheabdomenat
theendoftheprocedure.Whenthepatientsitsup,thegasmoves
upwardstothediaphragmandirritatesitleadingtoreferredpainC3-
C%.

1193.Apatientcomeswithacomplaintof
shoulderpainafterlaparoscopic
surgery.Whatshouldbethenextstep
inmanagement?

a)Oralparacetamolfor2-3days
b)USGofshoulderregion
c)Diagnosticshoulderarthroscopy
d)Intraarticularlignocaineinjection
CorrectAnswer-A
Answer-A.Oralparacetamolfor2-3days
Shouldertippain
Thepatientshouldbewarnedaboutthispreoperativelyandtoldthat
thepainisreferredfromthediaphragmandnotduetoalocal
problemintheshoulders.
Itcanbeatitsworst24hoursaftertheoperation.Itusuallysettles
within2-3daysandisrelievedbysimpleanalgesics,suchas
paracetamol.

1194.Whichofthefollowingshouldbedone
foranacuteonsetpainfulscrotal
swellingin12yearsoldmale?

a)Dopplerstethoscopeevaluation
b)Administeranalgesics
c)Advisebedrest
d)Administerantibiotics
CorrectAnswer-A
Answer-A.Dopplerstethoscopeevaluation
Acuteonsetpainfulscrotalswellingin12yearsoldpreadolescent
maleismostprobablyduetotorsionoftestis.Insuspectedcasesof
torsionoftestisortesticularappendageadopplerstethoscope
shouldbeusedtoevaluatebloodflowtothetesticles.

1195.Earliesthematologicalchangefollowing
spleenectomyis
a)Leukocytosisandthrombocytosis
b)PresenceofHeinzbodies
c)EvidenceofHowellJollybodies
d)Poikilocytosis
CorrectAnswer-A
Answer-A.Leukocytosisandthrombocytosis
Intheimmediatepostsplenectomyperiod,leukocytosis(upto
25,000/4)andthrombocytosis(upto1.106/4)develop,butwithin2-3
weeks,bloodcellcountsandsurvivalofeachcelllineageareusually
normal.

1196.


Followingroadtrafficaccidentpatientsuffers
polytraumaandisevaluatedinthe
emergencysectionofthehospital.Hispulse
rateis116,respiratoryrateis24,blood
pressureof122/78mmofHgandpatientis
mildlyanxious.Whatistheapproximate
bloodlosspatienthasfollowingtrauma?

a)<750ml
b)750-1500ml
c)1500-2000ml
d)>2000ml
CorrectAnswer-B
Answer-B.750-1500ml

1197.Sebaceouscystoccursoccurinallthe
followinglocationsinbodyexcept
a)Palmsandsoles
b)Axilla
c)Back
d)Pubicarea
CorrectAnswer-A
Answer-A.Palmsandsoles
Thescalp,scrotum,shoulders,neck,andbackarethecommon
sites,buttheycanoccurwherevertherearesebaceousglands.
Therearenosebaceousglandsonthepalmsofthehandsandsoles
ofthefeet,thussebaceouscystsarenotfoundintheselocations.

1198.Cortisollevelsremainelevatedforhow
manyweek/sfollowinghemorrhage?
a)1
b)2
c)3
d)4
CorrectAnswer-A
Answer-A.1
Burnpatientshavedemonstratedelevatedcirculatingcortisollevels
forupto4weeks,whilesofttissueinjuryandhemorrhagemay
sustainelevatedcortisollevelsforaslongas1week.

1199.Surgicaltreatmentofcongenital
hydrocelerecommendedifitfailsto
resolvebywhatage?

a)1year
b)2years
c)3years
d)4years
CorrectAnswer-B
Answer-B.2years
Apatentprocessusvaginalisthatistoonarrowtopreventthe
developmentofaninguinalherniamayneverthelessallowperitoneal
fluidtotrackdownaroundthetestistoformacongenitalhydrocele.
Themajorityresolvespontaneouslyastheprocessuscontinuesto
obliteratebutsurgicalligationisrecommendedinboysolderthan2
yearsiftheydonotresolvespontaneously.

1200.Whatpercentageofgallstonesare
radioopaque?
a)10-20
b)30-40
c)50-60
d)70-80
CorrectAnswer-A
Answer-A.10-20
Cholelithiasisisacommoncauseofepigastricorrightupper
quadrantpaininmiddleagedobesefemalepatients.
Only15-20%ofthegallstonesareradio-opaque.
Gallstonesarethemostcommonbiliarypathology.
Itisestimatedthatgallstonesarepresentin10-15%oftheadult
populationintheUSA.

1201.Percentageofrenalstonesthatare
radio-opaque
a)20
b)40
c)60
d)80
CorrectAnswer-D
Answer-D.80
RoutineuseofnoncontastCTScanhascompletelyrevolutionizedto
imagingevaluationofrenalstonedisease,nearlycompletely
replacingplainradigrpahsandXurographyfordiagnosisofacute
ureteralobstructionbyrenalstones.
Nephrolithiasisreferstothepresenceofcalculiintherenalcollecting
system.
Nearly10%ofthepopulationwillformarenalstoneintheirlifetime.
Sufficientcalciumoxalateandphosphateispresentin80%ofthe
renalcalculiforthemtoberadio-opaqueontheplainradiographs.

1202.Indicationsofthoracotomyinblunt
chesttraumaincludeallexcept
a)Initialdrainageof>500mloffreshblood
b)Ruptureofbronchous
c)Continuedbleedingof>200ml/hrfor>/=3hrs
d)Unsuccesfulattemptatdrainageofcardiactamponade
CorrectAnswer-A
Answer-A.Initialdrainageof>500mloffreshblood
Continuedbleedingof>200ml/hrfor>3hrs
Ruptureofbronchus,aorta,esophagusordiaphragm.
Cardiactamponade(ifneedleaspirationunsuccessful).

1203.Visualexaminationisusedas
screeningtestfor
a)Melanoma
b)Breastcancer
c)Thyroidcancer
d)Testicularcancer
CorrectAnswer-A
Answer-A.Melanoma
Visualexaminationistheonlyscreeningtestformelanomaorskin
cancer.
Forlesionswithsuspicionofheadandneckmelanoma,
dermatologistsvisualexaminationscreeningis89-97%sensitive
with35-75%positivepredictivevalue.

1204.Embolisationoftumorsisdoneusing
a)Polyvinylalcohol
b)Autologousbloodclots
c)Absoluteethanol
d)Alltheabove
CorrectAnswer-D
Answer-D.Alltheabove
Thefollowingaretheembolicmaterialscommonlyusedfor
trans-arterialembolization:
Gelatinspongeparticles
Microspheres
Autologousbloodclots
Polyvinylalcohol
n-butylcyanoacrylateglue
Absoluteethanol

1205.

Definitivesurgeryisapartofwhichstageof
damagecontrolsurgery?

a)I
b)II
c)III
d)IV
CorrectAnswer-D
Answer-D.IV
Followingmajorinjury,protractedsurgeryinthephysiologically
unstablepatientwiththe'deadlytriad'-thecombinationof
hypothermia,acidosisandcoagulopathy
'Damagecontrol'or'damagelimitationsurgery'isaconceptthat
originatedfromnavalarchitecture,wherebyashipwasdesignedto
haveareassealedoffinthecaseofdamage,tolimitflooding.
Stages-
1. Patientselection
2. Controlofhaemorrhageandcontrolofcontamination
3. Resuscitationcontinuedintheintensivecareunit
4. Definitivesurgery
5. Abdominalclosure

1206.Breastsurgeryisconsideredunder
whatcategoryofcardiacrisk?
a)>10%
b)5-10%
c)1-5%
d)<1%
CorrectAnswer-D
Answer-D.<1%
Low(reportedcardiarisk<1%)-
Endoscopicprocedures
Superficialproccdure
Cataractsurgery
Breastsurgery
Ambulatorysurgery

1207.Whatistheorderofdisorientation,
whichapersongoesthroughafteran
eventoftraumatohead?

a)Firsttime,thenplacefollowedbyperson
b)Firstplace,thentimefollowedbyperson
c)Firstperson,thentimefollowedbyplace
d)Firsttime,thenpersonfollowedbyplace
CorrectAnswer-A
Answer-A.Firsttime,thenplacefollowedbyperson
Orientationreferstothecleintsrecognitionofperson,placeandtime
-thatisknowingwhoandwhereeorsheisandthecorrectday,
dateandyear.Thisiscommonlydocumentedas"orientedX3".
Absenceofcorrectinformationaboutperson,placeandtimeis
referredtoasdisortientation.

1208.Whichofthefollowingisthepreferred
routeofaccessfortotalparenteral
nutritioninapatientwhorequiresthe
samefor<14daysandthereis
otherwisenoindicationforuseof
centralcatheter?

a)Internaljugularvein
b)Externaljugularvein
c)Periphaeralvein
d)PICCline
CorrectAnswer-C
Answer-C.Periphaeralvein
Administrationofparenteralnutritionviaaperipheralvenous
cathetershouldbeconsideredforpatientswhoarelikelytoneed
short-termparenteralnutrition(lessthan14days)whohavenoneed
forcentralaccessforotherreasons.Careshouldbetakenin
catheterchoice,andinattentiontopH,tonicityandlong-term
compatibilityoftheparenteralnutritionformulationsinordertoavoid
administrationorstabilityproblems.

1209.Whichofthefollowingisnottrueabout
useofgraftinvascularsurgery
a)Autologussaphenousveinachievessuperiorpatencyratesto
prostheticmaterialsespeciallyinfemorodistalbypass
b)ThepatencyofthePTFEgraftsmaybeimprovedby
interpositionofaveincuffatproximalanastomosis
c)Dopplerultrasoundassessmentisthemethodofchoicefor
qualityassuaranceoncompletionofoperativeprocedure
d)Intheabsenceofspecificcontraindicationsaspirinshouldbe
prescribedforallthepatientsofperipheralvasculardisease
CorrectAnswer-B
Answer-B.ThepatencyofthePTFEgraftsmaybeimprovedby
interpositionofaveincuffatproximalanastomosis
Autologoussaphenousveinachievessuperiorpatencyratesto
prostheticmaterials,especiallyinfemorodistalbypass.
Insituandreversedveingraftbypassesperformequallywelland
thechoiceoftechniqueshouldbebaseduponanatomical
considerationsthatarespecifictoindividualpatients.
Intheabsenceofsaphenousvein,noconvencingstudiesexistto
suggestthepreferredprostheticalternative.
ThepatencyofPTFEgraftsmaybeimprovedbyinterpositionofa
veincuffatthedistalanastomosis.
Patencyratesassociatedwithpre-cuffedPTFE,graftsare
comparabletothatobtainedfromstandardPTFEgraftswithan
interpositionveincuff.
Thereisnojustificationfortheroutineuseofadjuvantdistal
arterivenousfistulae.

1210.Intracranialpressureisnotraised
during
a)Hyperventilation
b)Statusepilepticus
c)Headinjury
d)Subduralhematoma
CorrectAnswer-A
Answer-A.Hyperventilation
Hyperventilationisusedasatreatmentwithraisedintracranial
pressure.
HyperventilationcausesdecreasedPaCO2whichsubsequently
leadstoarterialvasoconstrictionthusloweringcerebralbloodflow
(CBF),cerebralbloodvolume,andintracranialpressure

1211.Immediatephysiologicalresponseto
suddendecreaseinbloodvolumeis
a)Releaseofepinephrine
b)Shiftoffluidfromintracellulartointerstitialcompartment
c)Releaseofangiotensin
d)Releaseofthyroxine
CorrectAnswer-A
Answer-A.Releaseofepinephrine
Themajorhemodynamicabnormalityinhypovolemicshockis
decreaseinpreload.
Theimmediatephysiologicalresponseofthebodytothesudden
decreaseinvolume(preload),isareleaseofcatecholamines
(epinephrine,norepinephrine).
Thesubsequentincreaseinheartrateandcontractilityhelpmaintain
cardiacoutput.

1212.Allofthefollowingareindicationsfor
bariatricsurgeryexcept-
a)BMI>40kg/m2
b)BMI>35kg/m2withatleastonecomorbidity
c)BMI>30withlongstandingdiabetes
d)Failureofothermethodsofweightloss
CorrectAnswer-C
Answer-C.BMI>30withlongstandingdiabetes
Indicationsforbariatricswgery
BMI>40kg/m2
BMI>35kg/m2withatleastonecomorbidity
Patientathighriskofobesityassociatedmorbidityandmortality
Failureofothermethodsofwightloss

1213.Slipsignisseenin
a)Lipoma
b)Desmoidtumor
c)Sebaceouscyst
d)Hernia
CorrectAnswer-A
Answer-A.Lipoma
Acharacteristic"slippagesign"maybeelicitedbygentlyslidingthe
fingersofftheedgeofthetumor.Thetumorwillbefelttoslipout
fromunder,asopposedtoasebaceouscystoranabscessthatis
tetheredbysurroundinginduration.

1214.Forreimplantationdigitsarestoredin
a)Icepacks
b)Deepfreeze
c)Coldsaline
d)Plasticbagswithice
CorrectAnswer-D
Answer-D.Plasticbagswithice
Theamputatedpartshouldbewrappedinmoistenedgauzeand
placedinasealedplasticbag.Thisbagshouldthenbeplacedinan
icewaterbath.Donotusedryiceanddonotallowthepartto
contacticedirectly;frostbitecanoccurintheamputatedpart,which
willdecreaseitschanceofsurvivalafterreplantation.Bleeding
shouldbecontrolledintheproximalstumpbyasminimalameans
asnecessary,andthestumpdressedwithanonadherentgauzeand
bulkydressing."

1215.Characteristicsitefordevelopmentof
venouslegulcersis
a)Skinofgaiterregion
b)Behindlateralmalleolus
c)Shinoftibia
d)Medialaspectofknee
CorrectAnswer-A
Answer-A.Skinofgaiterregion
Thevenousulcerofthelegcharacteristicallydevelopsintheskinof
thegaiterregion,theareabetweenthemusclesofthecalfandthe
ankle.ThisistheregionwheremanyoftheCockettperforatorsjoin
theposteriortibialveintothesurfacevein,knownastheposterior
archvein
Themajorityofulcersdeveloponthemedialsideofthecalfbut
ulcersassociatedwithlessersaphenousincompetenceoften
developonthelateralsideoftheleg.
Ulcerscandeveloponanypartofthecalfskininpatientswithpost-
thromboticlegs;however,venousulcersrarelyextendontothefoot
orintotheuppercalfand,ifthereisulcerationatthesesites,other
diagnosesshouldbeseriouslyconsidered.

1216.Whichofthefollowingistrueaboutthe
managementofvenousulceroflower
limb?

a)Initialtreatmentisdebridementandsurgery
b)Compressiondressingsshouldideallybeappliedontwice
weeklybasis
c)Antibioticsdonotspeeduptheulcerhealing
d)Biologicaldressingsdonothavepotentialtoimprovehealing
CorrectAnswer-C
Answer-C.Antibioticsdonotspeeduptheulcerhealing
Patientsareinitiallytreatedbyacompressionbandagingregimen.
Alternativetothesebandagingregimensistoapplyabland
absorbentleak-proofdressingbeneathagraduatedelastic
compressionstocking(classII).
Biopsiesareindicatedifmalignancyissuspectedanditisimportant
torememberthataMarjolin'stypeofulcer(asquamouscellorbasal
cellcarcinoma)candevelopinachroniclongstandingvenousulcer.
Considerationmustbegiventohealingtheulcerbyexcisionand
grafting.
Biologicaldressingshavebeendeveloped,includingfetal
keratinocytesandcollagenmeshes
Pinchgraftsandulcerexcisionwithmeshgraftinghavebeenshown
toprovidegoodearlyhealingwithmoderatelong-termresults

1217.Whichofthefollowingsuturehasmax
tensilestrengthandminimumtissue
reaction

a)Poliglecaprone
b)Polypropylene
c)Polygalctine
d)Polydioxanone
CorrectAnswer-B
Answer-B.Polypropylene
Polypropylenesuturesarenon-absorbableandprovidepermanent
woundsupport.
Polypropylenesuturesarebluecoloredforeasyidentificationduring
surgery.
Polypropylenesutureshaveexcellenttensilestrengthandareused
fororthopaedic,plasticandmicrosurgeries,generalclosureand
cardiovascularsurgeries

1218.Blunttraumaexploratorylaparotomy
donenonexpansileswellingfoundon
mesentericborderofintestine,
management

a)Resectionandanastomosis
b)Ligation
c)Excisionofswelling
d)None
CorrectAnswer-D
Answer-D.None
NonExpansileswellingonmesentericborderofintestineisalmost
alwaysduetomesentericadenitisanditisanincidentalfinding,
hencewarrantingnosurgicalintervention.

1219.InSVCsyndromefornonsmallcell
carcinomaoflungmanagementdone
a)Radiotherapy
b)Immunotherapy
c)Chemotherapy
d)Surgery
CorrectAnswer-A
Answer-A.Radiotherapy
InpatientswithSVCSsecondarytonon?small-cellcarcinomaofthe
lung,radiotherapyistheprimarytreatment.Thelikelihoodofpatients
benefitingfromsuchtherapyishigh,buttheoverallprognosisof
thesepatientsispoor
SmallcellcarcinomaoflungpresentingasSVCsyndrome-
Combinationofchemotherapyandradiotherapy.
NonSmallcellcarcinomaoflungpresentingasSVCsyndrome-
Radiotherpyalone.

1220.Hadfieldsoperationisperformedfor
whichofthefollowingpathology?
a)Ductectasia
b)Fibroadenoma
c)Mondorsdisease
d)Inflammatorybreastcarcinoma
CorrectAnswer-A
Answer-A.Ductectasia
TREATMENT-
Stopsmoking
Hadfield'soperation-excisionofallmajorducts
Antibiotics-amoxiclavandmetronidazole

1221.Microdochotomyistreatmentfor
a)Ductectasia
b)Breastabscess
c)Ductpapilloma
d)DCIS
CorrectAnswer-C
Answer-C.Ductpapilloma
Thefinaldiagnosisismadebyexcisingtheinvolvedduct
(Microdochotomy)andanyunderlyingmassifpresentand
subjectingthemforahistopathologicaldiagnosis

1222.Previousraditiontherapyforwhich
diseaseparticularyincreasestheriskof
breastcarcinomadevelopment?

a)Hodgkinslymphoma
b)Mantlecelllymphoma
c)Nasopharyngealcarcinoma
d)Lungcarcinoma
CorrectAnswer-A
Answer-A.Hodgkinslymphoma
Arealprobleminwomenwhohavebeentreatedwithmantle
radiotherapyaspartofthemanagementofHodgkin'sdisease,in
whichsignificantdosesofradiationtothebreastarereceived.

1223.Whichofthefollowinghistologicaltype
ofbreastcarcinomaworstprognosis?
a)Tubular
b)Colloid
c)Papillary
d)Scirrhous
CorrectAnswer-D
Answer-D.Scirrhous
MCtypeofbreastcarcinomaisinvasiveductalcarcinoma
(schirrhous).
MCformseen(60-75%)
Hardlump,whitishyellow,noncapsulated,irregularwith
cartilaginousconsistency
Retractionofnipple

1224.52yearsfemalepatientpresentswith4
cmdiameterdiagnosedbreastcancer
lesionwithipsilateralaxillaryand
contralateralsupraclavicular
lymphadenopathy.AsperAJCC
system,patientbelongstowhichstage
ofbreastcancer?

a)Ilia
b)None
c)IIIc
d)IV
CorrectAnswer-D
Answer-D.IV



1225.Bestprognosisforcarcinomabreastis
seenwithwhichofthefollowing?
a)<1cmsize,nodes-ye,ER/PR+ve,her2/neu-ye
b)<1cmsize,nodes-ye,ER/PR-ye,her2/neu+ve
c)<2cmsize,nodes-ye,ER/PR+ve,her2/neu-ye
d)<2cmsize,nodes-ye,ER/PR-ye,her2/neu+ve
CorrectAnswer-A
Answer-A.<1cmsize,nodes-ye,ER/PR+ve,her2/neu-ye
Molecularchangesinthetumorarealsouseful.Tumorsthat
overexpresserbB2(HER2/neu)orhaveamutatedp53genehavea
worseprognosis.TumorsthatoverexpresserbB2aremorelikelyto
respondtohigherdosesofdoxorubicin-containingregimensand
predictthosetumorsthatwillrespondtoHER2/neuantibodies
(trastuzumab)(herceptin)andHER2/neukinaseinhibitors.


1226.Favorableprognosiswith>90%5year
survivalrateforcarcinomabreastis
seeninwhichofthefollowing?

a)Screendetectedductalcarcinomainsitu
b)Screendetectedlobularcarcinomainsitu
c)Nodenegativetumorwithfavourablehistology
d)Noneoftheabove
CorrectAnswer-A
Answer-A.Screendetectedductalcarcinomainsitu
Intraductalcarcinoma(malignantmammaryductalepithelialcells)
withoutanyinvasionintobasementmembrane.

1227.Afemaleundergonesurgeryforleft
breastcancer3yrsbacknow
developedbluenoduleonsameside

a)Lymphangiosarcoma
b)Recurrence
c)Hemangioma
d)Cellulitis
CorrectAnswer-A
Answer-A.Lymphangiosarcoma
Angiosarcomaisavasculartumorwhichmayarisedenovointhe
breastorasacomplicationoftheradiationtherapy.

1228.Whatpercentageofreductionindying
frombreastcancercanbeachieved
withannualscreeningafterageof50
yrswithmammography?

a)15-20%
b)20-25%
c)25-30%
d)30-35%
CorrectAnswer-C
Answer-C.25-30%
Meta-analysisexaminingoutcomesfromeveryrandomizedtrialof
mammographyconclusivelyshowsa25-30%reductioninthe
chanceofdyingfrombreastcancerwithannualscreeningafterage
50years.

1229.Whichofthefollowinghormonallevels
influencethedevelopmentofbenign
breastdisease?

a)Estrogen
b)Progesterone
c)Lutenizinghormone
d)Testosterone
CorrectAnswer-A:B
Answer-A.Estrogen&B.Progesterone
Etiology/Pathogenesis
Hormonal:Responsivenessofbreasttissuetomonthlychangesof
estrogenandprogesteroneplayanimportantroleinpathogenesisof
benignbreastdisease.Theymayberelatedtoexcesshormonal
stimulationorhypersensitivityofbreasttissue.

1230.Whichofthefollowingistrueabout
breastreconstructionsurgery
a)Easiestreconstructionisdoneusingsilicongelimplant
b)TRAMflapgivesbettercosmeticresultsthanLDflap
c)Radiotherapyinpostopperioddoesnotinfluencetheoutcome
afterbreastreconstruction
d)Nipplereconstructioncannotbeperformedunderlocal
anesthesia
CorrectAnswer-A
Answer-A.Easiestreconstructionisdoneusingsilicongel
implant
Theeasiesttypeofreconstructionisusingasiliconegelimplant
underthepectoralismajormuscle.
Thismaybecombinedwithpriortissueexpansionusingan
expandablesalineprosthesisfirst(oracombineddevice),which
createssomeptosisofthenewbreast.
Iftheskinatthemastectomysiteispoor(e.g.following
radiotherapy)orifalargervolumeoftissueisrequired,a
musculocutaneousflapcanbeconstructedeitherfromthe
latissimusdorsimuscle(anLDflap)orusingthe
transversusabdominismuscle(aTRAMflapas).Thelattergivesan
excellentcosmeticresultinexperiencedhandsbutisalengthy
procedureandrequirescarefulpatientselection.
Itisnowusuallyperformedasafreetransferusingmicrovascular
anastomosis,althoughthepedicledTRAMfromthecontralateral
sideisstillused.
VariationsontheTRAMflaprequiringlessmuscleharvesting,such
astheDIEPflap(basedondeepinferiorepigastricvessels),are
increasinglybeingused.

increasinglybeingused.
Impedimentstoimmediatereconstructionincludeinsufficienttheatre
timeandalackofexperiencedreconstructivesurgeons.
Inaddition,ifapatientislikelytoneedpostoperativeradiotherapy
thenadelayedreconstructionusingaflapoftengivesabetterresult.
Radiotherapyontoaprosthesisoftenleadstoahighincidenceof
capsularcontractureandunacceptableresults.
Nipplereconstructionisarelativelysimpleprocedurethatcanbe
performedunderalocalanaesthetic.

1231.Treatmentoflargeomphaloceledefect
canbedoneby
a)Primaryclosure
b)StagedclosureusingPTFEmesh
c)Paintingintactsacdailywithantisepticsolution
d)Alltheabove
CorrectAnswer-D
Answer-D.Alltheabove
Largedefectspresentamoresubstantialproblemandfour
techniqueshavebeendescribed:non-operativetherapy,skinflap
closure,stagedclosureandprimaryclosure

1232.Whichofthefollowingisnottrueabout
Pateysmastectomy
a)Itisalsocalledmodifiedradicalmastectomy
b)Intercostobrachialnervesareusuallypreserved
c)Alllymphnodesofaxillaareremoved
d)Pectoralismuscleiseitherdividedorretracted
CorrectAnswer-B
Answer-B.Intercostobrachialnervesareusuallypreserved
Itisalsocalledmodifiedradicalmastectomyandisacommonly
performedprocedure.
Theintercostalbrachialnervesareusuallydividedinthisoperation
andthepatientshouldbewarnedaboutsensationchanges
postoperatively.
Thewoundisdrainedusingawide-boresuctiontube.
Earlymobilisationofthearmisencouragedandphysiotherapyhelps
normalfunctiontoreturnveryquickly

1233.Macroprolactinomaidealtreatmentis
a)Excision
b)Bromocriptine
c)Stereotacticradiosurgery
d)Observation
CorrectAnswer-B
Answer-B
Bromocriptine(BEC)isgenerallyconsideredtobetheagentof
choiceinthetreatmentofprolactinomabecauseofitslongtrack
recordandsafety.

1234.Initialtreatmentformostpatientof
growthhormonesecretingpituitary
adenomais

a)Transphenoidalsurgicalresection
b)Somatostatinanalogs
c)GHrectorantagonists
d)Dopamineagonists
CorrectAnswer-A
Answer-A.Transphenoidalsurgicalresection
Transsphenoidalsurgicalresectionbyanexperiencedsurgeonis
thepreferredprimarytreatmentforbothmicroadenomas(curerate
-70%)andmacroadenomas(<50%cured).

1235.Radioiodinepreferredintreatmentin
a)Youngpatients
b)Pregnancy
c)Recentonsetoftoxicgoiter
d)Postsurgeryforpapillarythyroidcancer
CorrectAnswer-C
Answer-C.Recentonsetoftoxicgoiter
ThemainindicationsforRAItherapyincludethefollowingconditions
1. Hyperthyroidismdueto:
Grave'sdisease
Toxicmultinodulargiotreor
Hyperfunctioningthyroidnodules
2. Non-toxicmultinodulargoitre
3. Thyroidcancer.

1236.Parathyroidadenomasaccountforhow
muchpercentageofpatientswith
primaryhyperpara-thyroidism?

a)50%
b)60%
c)70%
d)80%
CorrectAnswer-D
Answer-D.80%
Solitaryadenomas,Asingleabnormalgland,isthecausein-80%of
patients;theabnormalityintheglandisusuallyabenignneoplasm
oradenomaandrarelyaparathyroidcarcinoma.

1237.MCsiteforthyroglossalcystis:
a)Beneaththeforamencaecum
b)Floorofmouth
c)Abovehyoid
d)Subhyoid
CorrectAnswer-D
Ans.Di.e.Subhyoid
Sitesofthyroglossalcyst
1. Subhyoid:Themostcommontype
2. Atthelevelofthyroidcartilage:2ndcommonsite
3. Suprahyoid:Doublechinappearance
4. Attheforamencaecum:Rare
5. Atthelevelofcricoidcartilage:Rare
6. Inthefloorofthemouth

1238.Investigationofchoiceforhepatic
metastasisformstomachcanceris
a)MRI
b)CECT
c)USG
d)HIDA
CorrectAnswer-B
Answer-B.CECT
CTistheimagingmodalityofchoiceforevaluatinglivermetastases.
Thispreferenceislargelyattributabletotheeffectsofthedualblood
supplyontheenhancementcharacteristicsofmetastases,as
comparedwithnormalliverparenchyma.

1239.Prognosisofsurgeryforliver
secondariesisbestforwhichcancer?
a)Colorectal
b)Neuroendocrine
c)Genitourinary
d)Esophageal
CorrectAnswer-C
Answer-C.Genitourinary
Fornoncolorectal,nonneuroendocrinetumors,metastasesfrom
genitourinaryprimarieshavethebestprognosisfollowinghepatic
metastatectomy.

1240.IndicatonsoflivertransplantinPCM
poisoningareallexcept
a)SGPTincrease
b)PT/INR
c)Highcreatinine
d)Encephalopathy
CorrectAnswer-A
Answer-A.SGPTincrease
Indicationpoorprognosisandhencenecessitateatransplantin
suchpatients-

1. ArterialpH<7.3(takenbysamplingofbloodfromanartery)
2. Allthreeofaninternationalnormalizedratio(INR)ofgreaterthan
6.5,serumcreatinineofgreaterthan300micromolesperlitreand
thepresenceofencephalopathy(ofgradeIIIorIV).Thesethreeare
markersofcoagulopathy,kidneyfunctionandmentalstatus.

1241.Milancriteriaisfor
a)SelectingpatientsforLivertransplantation
b)SelectingpatientsforLungtransplantation
c)SelectingpatientsforKidneytransplantation
d)Selectingpatientsforhearttrnasplantation
CorrectAnswer-A
Answer-A.SelectingpatientsforLivertransplantation
TheMilancriteriastatethatapatientisselectedfor
transplantationwhenheorshehas:

1. Onelesionsmallerthan5cm
2. Upto3lesionssmallerthan3cm
3. Noextrahepaticmanifestations
4. Novascularinvasion

1242.Whatshouldbetheplanof
managementforapatientforwhom
whileundergoingsimple
cholecystectomyT2gallbladder
carcinomaisdiscovered?

a)Resectionof4b-5segement,dissectionoftheN1-2nodesand
excisionofportsites
b)Postoperativeadjuvantchemotherapy
c)Radicalcholecystectomy
d)Whipplesprocedure
CorrectAnswer-A
Answer-A.Resectionof4b-5segement,dissectionoftheN1-2
nodesandexcisionofportsites
StageIandII-simplecholecystectomy
StageIII?cholecystectomy+adjacenthepaticresection(atleast
2cmdepth)+regionallymphadenectomy
Poorprognosis

1243.TheGallstonepainisreferredtothe
shoulderthroughwhichofthefollowing
nerves:

a)C2-C8
b)T1-T4
c)T8-T12
d)C3-05
CorrectAnswer-D
Ansis'd'i.e.C3-05
Gallstonediseasemayreferpaintotherightshouldertip(k/aKehr's
sign).Thisisbecause,aninflamedgallbladderirritatesthe
diaphragmwhichissuppliedbythephrenicnerve(C3-05).These
cervicalnerveroots,alsoprovidesensorysupplytotheright
shoulderthroughsupraclavicularnerves.Hencethegallbladderpain
isreferredtotherightshoulderthroughtheC3-05nerveroots.
Kehr'ssignisaclassicexampleofreferredpain:irritationofthe
diaphragmissignaledbythephrenicnerveaspaininthearea
abovethecollarbone.Thisisbecausethesupraclavicularnerves
havethesamecervicalnervesoriginasthephrenicnerve,C3and
C4.
Boas'signcanalsoindicatestomachandduodenaldisease.When
thetransverseprocessesofthoracicvertebraeT10-T12arepressed
oreffleuragedwiththebottomofthehand,paincanappearatleftof
spinousprocesses(instomach'slessercurvatureulcer)oratright
(inpyloricorduodenalulcer).
Boas'orBoas'ssignishyperaesthesia(increasedoraltered
sensitivity)belowtherightscapulacanbeasymptominacute
cholecystitis(inflammationofthegallbladder).

cholecystitis(inflammationofthegallbladder).

1244.3cmstoneincysticductnearthe
ampullaofvater,TheMethodof
removalisa

a)Transduodenalapproach
b)Supraduodenalapproach
c)Lithotripsy
d)Chemicaldissolution
CorrectAnswer-A
Answer-A.Transduodenalapproach
AccordingtoMaingot's"Themethodsofsurgicaldrainageinclude
transduodenalsphincterotomy,choledochoduodenotomy,and
choledochojejunostomy."

1245.Inwhichcondition,medicaltreatmentof
gallstoneisindicated-
a)Stoneis<15mmsize
b)Radioopaquestone
c)Calciumbilirubinatestone
d)Nonfunctioninggallbladder
CorrectAnswer-A
Ansis'a'isStoneis<15mmsize
Medicaltreatmentsforgallstones,usedaloneorincombination,
includethefollowing-
Oralbilesalttherapy(ursodeoxycholicacid)(particularlyforx-ray-
negativecholesterolgallstonesinpatientswithnormalgallbladder
function)
Extracorporealshockwavelithotripsy(particularlyfornoncalcified
cholesterolgallstonesinpatientswithnormalgallbladderfunction)
Medicalmanagementismoreeffectiveinpatientswithgood
gallbladderfunctionwhohavesmallstones(<1cm)withahigh
cholesterolcontent.Bilesalttherapymayberequiredformorethan
6monthsandhasasuccessrateoflessthan50%.

1246.RegardingCagallbladder-
a)Squamouscellcaisthemostcommon
b)Presentwithjaundice
c)Goodprognosis
d)All
CorrectAnswer-B
Answer(b)Presentswithjaundice
Biliarytractcancerstendtobeslow-growingtumoursthatinvade
locallyandmetastasisetolocallymphnodes.
Distantmetastasestotheperitonealcavity,liverandlungdooccur.
Jaundiceisthemostcommonpresentingfeature.
Abdominalpain,earlysatietyandweightlossarealsocommonly
seen.
Onexamination,jaundiceisevident,cachexiaoftennoticeableanda
palpablegallbladderispresentiftheobstructionisinthedistal
commonbileduct(Courvoisier'ssign).

1247.Whichofthefollowingisnota
prognosticfactorforAcutePancreatitis
a)SerumAmylase
b)SerumCalcium
c)SerumGlucose
d)SerumAST
CorrectAnswer-A
AnswerisA(SerumAmylase):
SerumAmylasedoesnotformanycriteriaforprognosisinAcute
Pancreatitis.
Althoughelevatedserumamylaselevelisimportantforestablishing
diagnosisofacutepancreatitis,itplaysnoroleinpredicting
prognosisorseverity.
"Thereappearstobenodefinitecorrelationbetweenseverityof
pancreatitisandthedegreeofserumamylaseelevation.After48to
72hours,evenwithcontinuingevidenceofpancreatitis,totalserum
amylaselevelstendtoreturntonormal."-Harrison
Hyperglycemia(Glucose),Hypocalcemia(Calcium)andelevated
serumASTareallpoorprognosticfactorsinaccordancewith
Ranson'scriteriaaselaboratedinthepreviousquestion.

1248.Mostsensitiveandspecificforacute
pancreatitisamongstthefollowing
is:
September2008

a)S.amylase
b)S.Alaninetransaminase
c)S.lipase
d)C-reactiveprotein
CorrectAnswer-C
Ans.C:S.lipase
Serummarkersfordiagnosisofacutepancreatitis:
Alaninetransaminase-Associatedwithgallstonepancreatitis;three
foldelevationorgreaterinthepresenceofacutepancreatitishasa
positivepredictivevalueof95percentindiagnosingacutegallstone
pancreatitis
Amylase
Mostaccuratewhenatleasttwicetheupperlimitofnormal;amylase
levelsandsensitivitydecreasewithtimefromonsetofsymptoms
C-reactiveprotein
Latemarker;highlevelsassociatedwithpancreaticnecrosis
Lipase
Increasedsensitivityinalcohol-inducedpancreatitis;morespecific
andsensitivethanamylasefordetectingacutepancreatitis
PhospholipaseA2
Associatedwithdevelopmentofpancreaticnecrosisandpulmonary
failure
Procalcitonin

Earlydetectionofseverity;highconcentrationsininfectednecrosis
Trypsinogenactivationpeptide
Earlymarkerforacutepancreatitisandclosecorrelationtoseverity

1249.Whichenzymeisofdiagnostic
importanceinchronicpancreatitis
a)Amylase
b)Pancreaticpolypeptide
c)Lipase
d)SerumInterleukin6levels
CorrectAnswer-C
Answer-C.Lipase
Elevatedlipaselevelsaremorespecifictothepancreasthan
elevatedamylaselevels.Lipaselevelsremainhighfor12days.In
patientswithchronicpancreatitis(usuallycausedbyalcoholabuse),
lipaselevelsmaybeelevatedinthepresenceofanormalserum
amylaselevel.

1250.Mostcommonscreeningtestforacute
pancreatitis-
a)Serumamylase
b)Serumlipase
c)Urinetrypsinogen
d)Insulin
CorrectAnswer-C
Answer-C.Urinetrypsinogen
Rapidmeasurementofurinarytrypsinogen-2levelisusefulinthe
emergencydepartmentasascreeningtestforacutepancreatitis.

1251.Whichofthefollowingis/arethe
imagingcriteriaforunresectable
carcinomaofpancreas?

a)Metastaticspreadtovertebrae
b)Invasioninduodenalwall
c)Irregularincreaseindensityofomentalfat
d)Alltheabove
CorrectAnswer-D
Answer-D.Alltheabove
Metastaticspredoutsidethepancreasmakestumorirresectable.
Tumorinvasionintoadjacentorganssignificsunresectability.

1252.Drugofchoiceforpalliativetreatment
ofpancreaticcarcinoma
a)Erlotinib
b)Gemcitabine
c)Paclitaxel
d)Cyclophosphomide
CorrectAnswer-B
Answer-B.Gemcitabine
A)Inoperablelocallyadvanceddisease-
Gemcitabineisusedathetreatmentofchoiceforthesepatients.
B)MetastaticDisease
Gemcitabineisthestandardtreatmentwithamediansurvivalof6
monthsanda1-yearsurvivalrateofonly20%.
Capecitabine,anoralfluoropyrimidine,hasbeencombinedwith
gemcitabine(GEM-CAP)inaphaseIIItrialthatshowedan
improvementinresponserateandprogression-freesurvivalover
single-agentgemcitabine,butnosurvivalbenefit.

1253.

Modified-KaushWhippleoperationisdevised
topreservewhichpartofstomach?

a)Fundus
b)Body
c)Lessercurvature
d)Pyloricantrum
CorrectAnswer-D
Answer-D.Pyloricantrum
Thisprocedureisalsocalledpyloruspreserving
pancreaticoduodenectomy(PPPDorpp-KaushWhippleprocedure).
Itistheprocedureofchoiceformostadenocarcinomasofthehead
ofpancreas.
Theoriginalpancreato-duodenectomyasproposedbyWhipple
includedresectionofthegastricantrum.
TheWhippleprocedureisnowreservedforsituationsinwhichthe
entireduodenumhastoberemoved(e.g.inFAP)orwherethe
tumourencroachesonthefirstpartoftheduodenumorthedistal
stomachandaPPPDwouldnotachieveaclearresectionmargin.

1254.Whichofthefollowingcausesofacute
pancreatitiscancauserecurrentbouts
withoutanyobviouspathology

a)SphincterOddidysfunction
b)Pancreasdivisum
c)Hypertriglyceridemia
d)Alltheabove
CorrectAnswer-D
Answer-D.Alltheabove
Etiology-
Gallstones(mostcommon)
Alcoholabuseisthesecondcauseofacutepancreatitis.
Occultdiseaseofthebiliarytreeorpancreaticducts,especially
microlithiasis,sludge.
Hypertriglyceridemia
Pancreasdivisum
Pancreaticcancer
SphincterofOddidysfunction
Cysticfibrosis
Drugs-Steroids,Azathioprine,Valproate,Estrogens,L-
Asparaginase,6-mercaptopurine,Sulfonamides,Tetracycline,Anti-
retroviralagents,Thiazidediuretics
Familialorgenetic
Hyperparathyroidism
Hypercalemia
PostERCP
Mostcommoncausesinchildren:bluntabdominalinjuries,

multisystemdisease(hemolyticuremicsyndromeandinflammatory
boweldisease)biliarystonesormicrolithiasis(sludging),anddrug
toxicity

1255.Pseudocystpancreasisdevelopedinhowmuchdurationfollowingan
attackofacutepancreatitis?
a)Lessthan1week
b)Lessthan2weeks
c)3ormoreweeks
d)4ormoreweeks
CorrectAnswer-D
Answer-D.4ormoreweeks
Pseudocyststypicallyarisefollowinganattackofmildacute
pancreatitis,lieoutsidethepancreas,andrepresentanAPFCthat
hasnotresolvedandmatured.
Formationofapseudocystrequires4weeksormorefromtheonset
ofacutepancreatitis.

1256.Cullen'ssignisseenin:
March2004
a)Acutecholecystitis
b)Acutehepatitis
c)Acutepancreatitis
d)Bluntinjuryabdomen
CorrectAnswer-C
Ans.Ci.e.Acutepancreatitis

1257.Howmuchpercentageoftheblunt
traumainjuriestospleeninadultsare
currentlymanagednonoperatively?

a)30%
b)50%
c)80%
d)90%
CorrectAnswer-C
Answer-C.80%
Spleenistheintra-abdominalorganmostcommonlyinjuredina
blunttraumatotheabdomen.
Upto80%ofbluntsplenicinjuriescanbemanagednon-
operatively.
Itcanbemanagedin2ways:
i)Non-operatively
Thisisnowtheorderofthedayandcurrently>70%ofadultswith
bluntsplenicinjuriesaremanagednon-operatively.
Buttheprimaryrequirementforitis-hemodynamicstability.
ii)Operativemanagement
Patientswhoarehemodynamicallyunstableorarefailingnon-
operativemanagement(eg.requirecontinuingtransfusion)should
undergooperativetreatment.

1258.Mostprevalentsymptominpatientsof
leiomyomaofesophagusis
a)Pain
b)Pyrosis
c)Dysphagia
d)Weightloss
CorrectAnswer-C
Answer-C.Dysphagia
CLINICALFEATURES-
Asymptomatic(<5cm)
Dysphagia,pain
Morecommoninmales
Location-2/3rdofoesophagus

1259.Followingisnottrueabouttraction
diverticulumofesophagus-
a)Producedduetotheextraluminalforces
b)Itisnotatruediverticulum
c)Theoutpouchingisusuallysmallandconical
d)Maydeveloptracheoesophagealfistula
CorrectAnswer-B
Answer-B.Itisnotatruediverticulum
Intractiondiverticulaextraluminalforces(likeinflammed&scarred
peribronchial&mediastinallymphnodes)pullthefullthicknessof
theesophagusout,creatingatruediverticula.
Tractiondiverticulaaremuchlesscommon.Theyaremostlya
consequenceofchronicgranulomatousdiseaseaffectingthe
tracheobronchiallymphnodesduetotuberculosis,atypical
mycobacteriaorhistoplasmosis.
Fibrotichealingofthelymphnodesexertstractiononthe
oesophagealwallandproducesafocaloutpouchingthatisusually
smallandhasaconicalshape

1260.Whichofthefollowingisfalseabout
zenkersdiverticulum
a)Mostpatientsareabove50yearsofage
b)Itisthemostcommonesophagealdiverticulum
c)Mucosaloutpouchingthroughthekillianstriangle
d)Cervicalwebscanbeassociatedwithzenkersdiverticulumin
80%ofthepatients
CorrectAnswer-D
Answer-D.Cervicalwebscanbeassociatedwithzenkers
diverticulumin80%ofthepatients
Zenkerdiverticulumoriginatesfromtheposteriorwallofthe
esophagusinatriangularareaofweakness,limitedinferiorlybythe
cricopharyngeusmuscleandsuperiorlybytheinferiorconstrictor
muscles(ie,theKilliantriangle).
Zenker'sdiverticulum,alsopharyngoesophagealdiverticulum,also
pharyngealpouch,alsohypopharyngealdiverticulum,isa
diverticulumofthemucosaofthepharynx,justabovethe
cricopharyngealmuscle(i.e.abovetheuppersphincterofthe
esophagus).
Itisapseudodiverticulum(notinvolvingalllayersoftheesophageal
wall).
Seenin50yearsold.

1261.Dohlmanprocedurefor-
a)Meckel'sdiverticulum
b)Zenker'sdiverticulum
c)Dermatomyositis
d)Menetrier'sdisease
CorrectAnswer-B
Answer-B(Zenker'sdiverticulum)
Thetreatmentofpharyngealpouches(Zenker'sDiverticulum)may
bebyeitheropensurgicalorendoscopictechniques.The
endoscopicDohlman'sprocedureisanidealtechniqueinthe
elderly.

1262.Trueaboutesophagealcarcinomais
a)Morelethalthanthecolorectalcancers
b)Theseshowanincreasingtrendtowardsthenumberof
squamouscellcarcinomas
c)Smallersizeesophageallesionshavebettersurvival
d)Asymptomaticbenignlesionsshouldbeexcisedimmediately
CorrectAnswer-A
Answer-A.Morelethalthanthecolorectalcancers
Etiology:
alcoholandsmoking:forsquamouscellcarcinomaand
adenocarcinoma
achalasia
asbestosis
Barrettoesophagus:foradenocarcinoma
coeliacdisease
ionisingradiation
causticstricture/lyestricture
Plummer-Vinsonsyndrome
Evenwhendetectedasasmalllesion,esophagealcancerhaspoor
survivalbecauseoftheabundantesophageallymphaticsleadingto
regionallymphnodemetastases.

1263.Patientcomplainsofintermittent
dysphagiawhichisequalbothfor
solidsandliquids,whichofthe
followingisthemostprobable
diagnosis?

a)Achalasiacardia
b)Esophagealstricture
c)Carcinomaesophagus
d)Diffuseesophagealspasm
CorrectAnswer-D
Answer-D.Diffuseesophagealspasm
Clinicalfeatures-
Hypertrophyofcircularmuscles
Dysphagia
Chestpain

1264.Earlyandlatesuspectedinstrumental
perforationofoesophagusshouldfirst
beassessedusing

a)Watersolublecontrastswallow
b)CTScan
c)Dilutebariumswallon
d)MRI
CorrectAnswer-A
Answer-A.Watersolublecontrastswallow
Ifthisisnegative,adilutebariumswallowshouldbeconsidered.
ACTscancanbeusedtoreplaceacontrastswalloworasan
adjuncttoaccuratelydelineatespecificfluidcollections.

1265.Downhillesophagusvaricesdevelopas
aresultofobstructionofseenin
a)Portalvein
b)Hepaticvein
c)Superiorvenacava
d)Inferiorvenacava
CorrectAnswer-C
Answer-C.Superiorvenacava
Downhillvaricesareproducedbytheobstructionofthesuperiorvena
cava,whichresultsincollateraldrainagefromthehead,neckand
upperextremityvenoussystemsintotheveinssurroundingthemid
andupperthoracicesophagusandintotheazygousvein.OnCT
varicesappearasenhancingtubularregionsintheperiesophageal
region.Thisdensityisequaltothevenousbloodpool.

1266.Mostcommonimpactedforeignbodyin
esophagusinchildrenis
a)Coin
b)Foodproduct
c)Krayon
d)Marble
CorrectAnswer-A
Answer-A.Coin
Coinsarethemostcommonlyimpactedforeignbodiesinchildrenin
esophagusandoropharynxfollowedbythefoodproducts

1267.Whichofthefollowingisnottrueabout
medicalmanagementofuncomplicated
GERD?

a)PPIsarethemosteffectivedrugtreatmentforGORD
b)Householdmeasureoftiltingthebedisefficatious
c)LongtermPPItherapyincreasesriskofmalignantchanges
d)AdequatedoseofPPIfor8weeksistherecommended
treatment
CorrectAnswer-C
Answer-C.LongtermPPItherapyincreasesriskofmalignant
changes
Treatment-
1.Lifestylemodificationwithavoidingorcessationofsmoking,tea/
coffee,alcohol.
2.Drugs-
PPI-antisecretorydrugs
Antaacidswithalginate
Prokinetic-itopride(50mgTID)(Cisaprideandmosapridenot
recommendedasitcausescardiacarrhythmia)
LESsphincterisrelaxedbynitrates,atropineandcalciumchannel
blocker.
3.Mucosaprotectiveagents-sucralfatecolloidalbismuth
4.Endotherapy-plexiglassminosphere(PMMA)
5.Surgery-antirefluxsurgery

1268.Singledrugregimenforcarcinoma
esophaguswhichshowssignificant
decreaseintumorsizein15-20%of
patientsincorporateswhichdrug?

a)Cisplatin
b)Bleomycin
c)Doxorubicin
d)Vincristine
CorrectAnswer-A
Answer-A.Cisplatin
Significantreductionsinthesizeofmeasurabletumormasseshave
beenreportedin15-25%ofpatientsgivensingle-agentcisplatin
treatmentandin30-60%ofpatientstreatedwithdrugcombinations
thatincludecisplatin.

1269.MostcommontypeofHiatalherniais
a)Sliding
b)Rolling
c)Mixed
d)Noneoftheabove
CorrectAnswer-A
Answer-A.Sliding
Thereare4typesofhiatalhernia
TypeI:Slidinghernia(mostcommontype-70-80%)
TypeII:Paraesophagealorrollinghernia
TypeIII:Mixed
TypeIV:Teseherniasaredistinguishedbythepresenceofother
abdominalviscerawihinthedefecti.e.omentum,transversecolon.

1270.Incorrosiveinjuryofesophaguscorrect
statementisallexcept
a)Alkalisareusuallyingestedinlargervolumes
b)Alkaliscausemoregasticdamagethanacids
c)Alkalisformfibrousscar
d)Acidsformeschar
CorrectAnswer-B
Answer-B.Alkaliscausemoregasticdamagethanacids
Ingeneral,alkalisarerelativelyodourlessandtasteless,making
themmorelikelytobeingestedinlargevolume.
Alkaliscauseliquefaction,saponificationoffats,dehydrationand
thrombosisofbloodvesselsthatusuallyleadstofibrousscarring.
Acidscausecoagulativenecrosiswithescharformation,andthis
coagulantmaylimitpenetrationtodeeperlayersoftheoesophageal
wall.
Acidsalsocausemoregastricdamagethanalkalisbecauseofthe
inductionofintensepylorospasmwithpoolingintheantrum.

1271.Anindividualofage40yrspresents
withdysphagiatobothsolidsand
liquidswithsymptomsofregurgitation.
Patientalsocomplaintsofchestpain
andweightloss.Whichofthefollowing
isthepreferredinvestigationfor
diagnosisofthispathology?

a)BariumswallowX-ray
b)BariumswallowX-raywithesophagealmanometry
c)BariumswallowX-raywoithesophagealmanometrywith
endoscopy
d)Noneoftheabove
CorrectAnswer-B
Answer-B.BariumswallowX-raywithesophagealmanometry
Bariumswallow-showscucumberoesophagusorbirdsbeakor
penciltipdeformity.
Oesophagealmanometry-
Characteristicsofachalasiacardia
HypertensiveLOS
Aperistalsisinthebodyofoesophagus
Bariumswallowshowsadilatedesophaguswithtaperingnarrowing
intheterminalendofesophagus,describedas`BIRDBEAK'
appearance.

1272.Mostaccuratemethodforthediagnosis
GastroesophagealRefluxDisease
(GERD)is

a)Histologicalstudy
b)Manometry
c)24-hourpHrecordingandelectricalimpedancemeasurement
d)Bariumswallowstudies
e)UpperGIendoscopy
CorrectAnswer-C
Answer-C.24-hourpHrecordingandelectricalimpedance
measurement
ThemostsensitivetestfordiagnosisofGERDis24-hambulatory
pHmonitoring.
Endoscopyisindicatedinpatientswithrefluxsymptomsrefractoryto
antisecretorytherapy;inthosewithalarmingsymptomssuchas
dysphagia,weightloss,orgastrointestinalbleeding;andinthose
withrecurrentdyspepsiaaftertreatmentthatisnotclearlydueto
refluxonclinicalgroundsalone
pHwaslessthan4

1273.Whichofthefollowingtumorsmost
commonlypresentswithupper
gastrointestinalbleeding?

a)Primarygastriccancer
b)Esophagealcarcinoma
c)Metastasestostomach
d)Hepaticadenocarcinoma
CorrectAnswer-A
Answer-A.Primarygastriccancer
Tumorbleedingaccountsforupto5%ofuppergastrointestinal
bleeding(UGIB)cases.


1274.Whichofthefollowingistrueabout
gastriculcerbutnottheduodenal
ulcer?

a)Painrarelyoccursatnight
b)Melenaismorecommonthanhematemesis
c)Usuallyoccursin40-50yrsofage
d)Painisrelievedwithingestionoffood
CorrectAnswer-A
Answer-A.Painrarelyoccursatnight

1275.Themostcommonlypracticed
operativeprocedureforaperforated
duodenalulceris-

a)Vagotomyandpyloroplasty
b)Vagotomyandantrectomy
c)Vagotomyandperforationclosure
d)Graham'somentumpatchrepair
CorrectAnswer-D
Ans.is'd'i.e.,Graham'somentumpatchrepair

1276.Nottrueabouthighlyselective
vagotomy-
a)Itisalsoknownparietalcellvagotomy
b)NervesofLatarjetaresacrifised
c)Recurrenceratesarehigherthanvagotomyanddrainageand
vagotomyandantrestomy.
d)Entiregastricreservoircapacityispreserved
CorrectAnswer-B
Answer-B.NervesofLatarjetaresacrifised
InHighlyselectivevagotomy(alsoknownasparietalcellvagotomy
orproximalgastricvagotomy)thevagalinnervationtotheantrum
andpylorus(nervesofLatarjet)arepreserved,onlythevagalsupply
totheproximaltwo-thirdsofstomach(whereessentiallyallthe
parietalcellsarelocated)iscut.Thispreservesgastricmotility.

1277.Simplestinvestigationtobeperformed
insuspectedcasesofgastriccanceris
-

a)Doublecontrastradiography
b)Plainradiography
c)CTScan
d)Endoscopy
CorrectAnswer-C
Answer-CCtscan
Investigationfordiagnosisofgastriccancer:
*UltrasoundandCTscan
-Toruleoutsecondariesintheliver.
-Tolookforenlargedcoeliacnodes.
-Candetectascites-guidedfluidtapandcellcytology.
-TodetectKrukenbergtumour(pelvicCT).
-Usefulindetectingmetastaticdisease.

1278.Thefeaturesofcrohnsdiseaseareall
except?
a)Lymphoidhyperplasia
b)Skinlesion
c)Transmuralinvolvement
d)CryptAbscess
CorrectAnswer-A
Ans.is'a'i.e.,Lymphoidhyperplasia
Intermittentmilddiarrhea,fever,abdominalpain(MC)
Rightlowerquadrantmass,weightloss,anemia
Sometimesmimicsappendicitisorbowelperforation
Analcomplaints(fissure,fistula,abscess)?frequent
Fat/vitaminmalabsorptionpresent
Recurrenceaftersurgerycommon
Malignancy(mostcommoncause)+withcoloninvolvement
StringsignofKantorisseeninCrohn'sDisease.
CreepingfatisafeatureofCrohn'sDisease.

1279.Trueaboutsmallintestinaltumoris-
a)Lymphomasarethemostcommonsmallintestinaltumors
b)Carcinoidsaremorecommonintheduodenumandjejunum
c)Adenomatouspolypsaremorecommonintheterminalileum
d)Riskofdevelopingsmallboweltumorcorrelatespositivelywith
colorectalcancer
CorrectAnswer-D
Answer-D.Riskofdevelopingsmallboweltumorcorrelates
positivelywithcolorectalcancer
Thesecondmostcommonsmallboweltumoriscarcinoid
accountingfor35%ofallsmallbowelcarcinomas,90%ofwhichare
locatedintheileum.
Lymphomasaccountforthethirdmostcommontumorsofthesmall
bowel.
Adenomatouspolypstendtooccurintheperiampullaryregionand
proximaljejunum,closetotheentranceofbileandpancreatic
secretionsintosmallintestine.

1280.Contraindicationforcolostomy
planningareallexcept-
a)Age>60yrs
b)Stomanearskincreasesandbonyprominences
c)Poorlymotivatedpatientforelectivestomy
d)Stomasthroughpreviousscars
CorrectAnswer-A
Answer-A.Age>60yrs
Thepatientmustbeexplainedtheprocedureandproperlymotivated
fortakingcareofthestoma
Thestomashouldpreferablybethroughrectusabdominis(to
preventprolapseandparastomalhernias)
Itshouldbethroughflatsurfaceofabdomen
Itshouldavoidskincreasesandbonyprominences
Itshouldbeawayfrompreviousscars

1281.Colonoscopyisarenotindicatedin-
a)MEN2B
b)FAP
c)HNPCC
d)Cornkitecanadasyndrome
CorrectAnswer-A
Answer-A.MEN2B
MEN2Bdoesnotpredisposetocoloncancer.
FAP,HNPCC,CronkiteCanadaSyndromepredisposetoColon
Cancerandhencescreeningwithcolonoscopyisneeded

1282.Criticaldiameterofcaecumwhen
perforationisconsideredeminentin
pseudo-obstructionis-

a)>7cm
b)>8cm
c)>9cm
d)>10cm
CorrectAnswer-C
Answer-C.>9cm
Itisdefinedaslargeboweldistentionresultingfromchronic
impairmentofmotility.
Thedegreeofcolonicdistentionmaybesevereenoughtocause
caecalperforation.
Imminenetperforationofcaecumisconsideredwhenthecaecal
diameterexceedsthecriticaldiameterof9cm(>9cm).

1283.ExceptiontoGoodsalsruleis
consideredwhentheanteriorexternal
openingismoresituatedmorethan
cmsfromanalmargin-

a)2
b)3
c)4
d)5
CorrectAnswer-B
Answer-B.3
Goodsall'sRule
Isusedtodeterminethelocationofinternalopening
Accordingtoit:
Fistulaswithexternalopeninganteriortohorizontalimaginaryline
drawnacrossthemidpointofanusconnecttotheinternalopening
byshortstraighttract.
Fistulaswithexternalopeningposteriortothehorizontalline-runa
curvilinearcourseandopeninternallyintotheposteriormidline.

1284.Treatmentofchoiceforlowfistulain
anois-
a)Fistulotomy
b)Intravenousantibiotics
c)Stagedsurgicalresection
d)Noneoftheabove
CorrectAnswer-A
Answer-A.Fistulotomy
Itisdividedinto2types-high&low,accordingtowhethertheir
internalopeningsisbeloworabovetheanorectalring.
Theimportanceofdecidingwhetherafistulaisaloworahightype
isthatalowlevelcanbetreatedbyfistulotomy(openingthetract)
withoutcausingdamagetothesphincter.

1285.Stagingofrectalcarcinomaisbest
doneby-
a)CTScan
b)MRI
c)TRUS
d)Alltheabove
CorrectAnswer-B
Answer-B
90%ofrectalgrowthscanbefeltbyper-rectalexamination
High-resolutionphasedarrayexternalMRIistheinvestigationof
choice
forlocalissuesintheprimarystagingofrectal
cancer(bestinvestigationforstaging)
aswellasforrestaging
afterNACT-RT.Itprovidesthehighestaccuracyforissuesin
pretreatmentlocalstaging
Investigationofchoice?rigidsigmoidoscopyandbiopsy
Toassesslocalspread?TRUS(Endoluminalultrasound)
Forlocalstagingandassessmentofproposedcircumferential
resectionmargin?MRI(CTisnotaccurateinlocalstaging)

1286.Rolledupomentumisseenincasesof-
a)Peritonealtuberculosis
b)Peritonealmetastases
c)Perforationperitonitis
d)Malrotationofgut
CorrectAnswer-A
Answer-A.Peritonealtuberculosis
Theclassicappearanceofgreyishwhitemilitarynodulesscattered
overtheperitoneum.Inadditionfibrousbandsandadhesionsare
common.
Theomentummaybecomethickenedpresentingasatransversely
placedmass(rolledupomentum).
Clinicallythemostfrequentpresentationoftheperitonealdiseaseis
ascites.

1287.Pantaloonherniaisalsocalledas-
a)Dualhernia
b)Saddlebaghernia
c)Bochdalekshernia
d)Retrosternalhernia
CorrectAnswer-B
Answer-B.Saddlebaghernia
Itisalsocalleddualorsaddlebaghernia.
Thistypeofherniaconsistsoftwosacsthatstraddletheinferior
epigastricartery,onesacbeingmedialandtheotherlateraltothis
vessel.

1288.Whichofthefollowingfamous
personalitieshadhydroceleassociated
withhernia,whichprovedtobefatal?

a)EdwardGibbon
b)ThomasEdison
c)MileyWright
d)NeilArmstrong
CorrectAnswer-A
Answer-A.EdwardGibbon
Edwardgibbon(1737-1794),englishhistorianhadlargehydrocele.
Thehydrocelewasassociatedwithalargescrotalherniawhich
probablywaspunctured.

1289.Incaseoffemalecommonestherniais
?
a)Directinguinalhernia
b)Indirectinguinalhernia
c)FemoralHernia
d)Incisionalhernia
CorrectAnswer-B
Ansis'b'ieIndirectInguinalhernia
InIndirectinguinalherniathecontentsoftheabdomenenterthe
deepinguinalringandtraversethewholelengthoftheinguinal
canaltocomeoutthroughthesuperficialinguinalring.
Itisthemostcommonofallformsofhernia.
Itismostcommonintheyoung(cfadirectherniaismostcommon
intheold)
Femoralherniasoccurmostcommonlyinwomenbutlower
incidenceoverallthaninguinalhernias.


1290.40yearsmalepresentswithtranslucent
scrotalswellinginwhichitispossible
togetabovetheswellingon
examination.Whatisthemostprobable
diagnosis?

a)Hydrocele
b)Indirectinguinalhernia
c)Varicocele
d)Sebaceouscyst
CorrectAnswer-A
Answer-A.Hydrocele
Hydrocelesaretypicallytranslucentanditispossibleto'getabove
theswelling'onexaminationofthescrotum.
Ahydroceleisanabnormalcollectionofseriousfluidinapartofthe
processusvaginalis,usuallythetunica.
Encystedhydroceleofthecordisasmoothovalswellingnearthe
spermaticcord,whichisliabletobemistakenforaninguinalhernia.
Theswellingmovesdownwardsandbecomeslessmobileifthe
testisispulledgentlydownwards

1291.Scrotalswellingnonreduciblebut
disappearswhenthechildwakesup
fromsleepismostlikelytobe-

a)Congenitalhydrocele
b)Varicocele
c)Indirectinguinalhernia
d)Noneoftheabove
CorrectAnswer-A
Answer-A.Congenitalhydrocele
Incongenitalhydrocelethecommunicationwithperitonealcavityis
usuallytoosmalltoallowherniationofintra-abdominalcontents.
Usuallyhydroceleisanonreducibleswellingbutcongenital
hydrocelecanpassivelygetdrainedthroughintacttunicsvaginalis
duetogravitywhilesleeping.
Bothvaricoceleandindirectinguinalherniaarereduciblescotal
swelling.
SothemostprobableanswerisCongenitalhydrocele

1292.Treatmentofcongenitalhydrocephalus
is-
a)Ventriculoperitonealshunt
b)Sterestacticradiosurgery
c)Diuretics
d)Radiotherapy
CorrectAnswer-A
Answer-A.Ventriculoperitonealshunt
CongenitalHydrocephalus-TreatmentOptions

1. Ventriculoperitonealshunt
2. Endoscopicthirdventriculostomy
3. Lumbarpuncture(temporarymeasuretillashuntisput)

1293.Trueofumbilicalhernia-
a)Mostcommoncontentislargeintestine
b)Mostoftheumbilicalherniasdisappearspontaneously
c)Malesareaffectedmorethanfemales
d)Uncomplicatedhereniasarerepairedat1yearofagethrough
aninfraumbilicalincision.
CorrectAnswer-B
Answer-B.Mostoftheumbilicalherniasdisappear
spontaneously
Umbilicalherniadevelopsduetoeitherabsenceofumbilicalfascia
orincompleteclosureofumbilicaldefectininfants.
Umbilicalhernia(paraumbilicalhernia)isaprotusionorherniation
throughlineaalbajustaboveorbelowumbilicus.
Weakestpartisumbilicalcicatrix.
Contentsare-greateromentum,smallintestineandtransverse
colon.
CLINICALFEATURES-
Femalesin5thdecade
Swellinghassmoothsurface,distinctedgesresonantwithdragging
pain.
Expansileimpulseonpatientcoughing
Surgicaltreatmentisindicatedif
Herniapersistsat2yearsofageorolder.
Ifthedefectismorthan2cminsize.
Ifitisassociatewithcomplications.
Heriorrhaphy(surgeryforrepairofhernia)isdonethroughan
infraumbilicalincision.Defectisclosedwithinterruptedsuturesafter
ligatingthesac.


1294.A10yearsoldmalepresentsasmooth
swellingnearsuperficialinguinalring,
whichmovesdownwardswhenthe
testicleispulleddownwards.Diagnosis
-

a)Inguinalhernia
b)Congenitalhydrocele
c)Encystedhydroceleofthecord
d)Varicocele
CorrectAnswer-C
Answer-C.Encystedhydroceleofthecord
Ahydrocoeledevelopsinaremnantoftheprocessusvaginalis
somewherealongthecourseofthespermaticcord.Thishydrocoele
alsotransilluminates,andisknownasanencystedhydrocoeleofthe
cord.
Theswellingmovesdownwardsandbecomeslessmobileifthe
testisispulledgentlydownwards.
Infemales,amulticystichydrocoeleofthecanalofNucksometimes
presentsasaswellinginthegroin.

1295.Physiologicaladhesionsbetween
foreskinandglanspenispersistuntil
yearsofage-

a)4
b)5
c)6
d)7
CorrectAnswer-C
Answer-C.6
Phimosisisaconditioninwhichtheforeskinofthepeniscannotbe
pulledbackpasttheglans.
Thephysiologicaladhesionsbetweentheforeskinandtheglans
penismaypersistuntil6yearsofageormore,givingthefalse
impressionthattheprepucewillnotretract.

1296.Commonestsiteofhypospadiasis-
a)Justproximaltoglans
b)Inthemeidofpenis
c)Scrotum
d)Perineum
CorrectAnswer-A
Answer-A.Justproximaltoglans
Hypospadiasisaconditioninwhichtheurethralmeatusopenson
theundersideofpenisortheperineum(i.e.ventralsurfaceofpenis)
proximaltothetipoftheglanspenis.

1297.Besttimeforsurgeryofhypospadiasis
atwhatage-
a)0-4months
b)4-6months
c)6-10months
d)>2years
CorrectAnswer-C
Answer-C.6-10months
Operationsforhypospadiasisareroutinelyperformedwhenthe
patientisbetween6and18monthsofage.
Theinfanthasgoodtolerancetosurgeryandanesthesiabytheage
of6months.Thechildiswellawareofhisgenitaliaandtoilettraining
bytheageof18months.So,themostsuitableagefortheoperation
ofhypospadiasisbetween6and18months.
Thedegreeofhypospadiasdictatestheneedforrepair.Ifthe
openingisglanularorcoronal(85%ofpatients),thepenisisusually
functionalbothformicturitionandprocreationandrepairisdone
primarilyforcosmeticreasons.Openingsthataremoreproximalon
theshaftrequirecorrectiontoallowvoidingwhilestanding,normal
erection,andproperspermdepositionduringintercourse.
Thecommonestprocedurefordistalhypospadiasisthe
`tubularizedincisedplate'urethroplasty
,whiletechniquesthat
utilizetheforeskinarecommonlyusedformoreproximal
hypospadias.
Complicationsofhypospadiassurgery:
urethrocutaneousfistula(mostcommon)
Recurrenceofchordee
Urethralstricture

Meatalstenosis
Infection

1298.Whichoneofthefollowingstatementis
trueofundescendedtestis-
a)Usuallydescendsspontaneouslyatpuberty
b)Orchipexytobedoneifnodescentbypuberty
c)Hasahigherincidenceofmalignancy
d)Maintainsnormalspermproduction
CorrectAnswer-C
Ans.is'c'i.e.,Hasahigherincidenceofmalignancy
TREATMENT-
*Orchidopexydonebefore6monthsofage
*Orchidectomy-patientwithincompletedescendedtestisis
atrophic,pastpubertyandnormaltestis
*Ombredanne'soperationinbilateralcases
*HCGorGnRH-cryptorchidismassociatedwithhypogenitalismand
obesity
*ApproximatelyUDTspontaneouslydescendby3monthsofage
*Secondarysexualcharactersticsarenormal

1299.Mostcommonsiteofpenilecarcinoma
is-
a)Glans
b)Prepuce
c)Shaft
d)Coronalsulcus
CorrectAnswer-A
Answer-A.Glans
Whenitoccursontheglanspenis,itisknownaserythroplasiaof
Queyratandwhenitoccursontheshaftofthepenisitiscalled
Bowen'sdisease.
Carcinomaofthepenisismosttypicallyasquamouscellcarcinoma
arisingintheskinoftheglanspenisortheprepuce.
MCorginatesfromglans>sulcus>prepuce>shaft

1300.Whichofthefollowingarethetreatment
optionsforcystocele?
a)Anteriorcolporrhaphy
b)Transvaginaltape
c)Transobturatortape
d)Alltheabove
CorrectAnswer-D
Answer-D.Alltheabove
Traditionally;ananteriorvaginalwallrepair(anteriorcolporrhaphy)
wasperformedvaginally;nowreplacedbyvaginallyinsertedtape
[transvaginaltape(TVT)ortransobturatortape(TOT)]ormesh
slings.

1301.Traditionallyscrotalcarcinomais
associatedwithwhichofthefollowing
occupations?

a)Chimneysweeps
b)Mulespinners
c)Coalworker
d)Woodworkers
CorrectAnswer-A
Answer-(A)Chimneysweeps
Carcinomaofthescrotum.
Itistraditionallyrecognizedasanoccupationalhazardforchimney
sweepsandmulespinners.
ItwasdescribedbyPotts.
Itwasthefirstcancerlinkedtooccupationalexposurewhen,in
1775,PerivallPottdescribeditinchimneysweepsinEngland.Other
occupationsthathadapreponderanceofthediseaseincluded
peoplewhoworkedwiththedistillatesofcoalandmenexposedto
mineraloil.

1302.Whatisthemainhazardofectopic
testis-
a)Impotence
b)Carcinomadevelopment
c)Liabletoinjury
d)Herniadevelopment
CorrectAnswer-C
Answer-C.Liabletoinjury
Themainhazardisliabilitytoinjury.

1303.40yearsoldmalecomplainsofloin
painsince1month.Patient'scomplaint
ofpainhasseverelyincreasedoverlast
2hoursandpainnowradiatesfromloin
andtogroinandanteriorthighand
patientiswrithinginbedforcomfort.
Whatisthemostprobableetiology?

a)Bladdercalculus
b)Uretericcalculus
c)Vesicouretericreflux
d)Hydronephrosis
CorrectAnswer-B
Answer-B.Uretericcalculus
Thereisapatternofsevereexacerbationonabackgroundof
continuingpain
Radiatestothegroin,penis,scrotumorlabiumasthestone
progressesdowntheureter
Theseverityofpainisnotrelatedtothesizeofthestone
Thepainisalmostinvariablyassociatedwithhaematuria
Theremaybefewphysicalsigns

1304.Treatmentofchoiceforbladderstone
a)Transurethrallitholapaxy
b)Percutaneoussuprapubiclitholapaxy
c)Vesicotomyandstoneretrieval
d)Intravenousantibiotics
CorrectAnswer-A
Answer-A.Transurethrallitholapaxy
TREATMENT-
Atransurethralcystolitholapaxyisthemost
commonprocedureusedtotreatadultswithbladderstones.
Smallstone-Ultrasoundlithotripsy
Largestone-laserlithotripsy
Litholapaxy-cystoscopiclithotrite
Suprapubiccystolithotomy

1305.Followingarethesign/ssuggestiveof
obstructionofurinarytractonCTScan
a)Hyroureter
b)Perinephricstranding
c)Thickeningofthelateroconalfascia
d)Alltheabove
CorrectAnswer-D
Answer-D.Alltheabove
NonenhancedCThasbeenacceptedastheimagingmodalityof
choiceintheradiologicevaluationofpatientssuspectedofhaving
urolithiasis.
CTissuperiortootherimagingmodalitiesinthediagnosticaccuracy
andassessmentoftheircharacteristics.
Themostdirectsignofureterolithiasisisthestonewithintheureteral
lumen,withproximalureteraldilatationandnormaldistalcaliber.
OtherfavorablesecondarysignsatCTare:Hydroureter,
hydronephrosis,perinephricstranding,periureteraledemaand
unilateralrenalenlargement.

1306.Harderrenalstonehavingless
satisfactoryresultswithECWLis-
a)Oxalatestone
b)Phosphatestone
c)Uratestone
d)Cystinestone
CorrectAnswer-D
Answer-D.Cystinestone
ExtracorporealShockwaveLithotripsy(ECWL):
Aurinarycalculushasacrystallinestructure.Bombardedwithshock
wavesofsufficientenergyitdisintegratesintofragments.
TheclearanceofstonefromthekidneyusingESWLwilldepend
upontheconsistencyofthestoneanditssite.
Mostoxalateandphosphatestonesfragmentwell.

1307.

Radicalnephrectomyinvolvesresectionof
thefollowingexcept-

a)Gerotasfascia
b)Ipsilateraladrenalgland
c)Surroundinghilarlymphnodes
d)Proximalpara-aorticlymphnodes
CorrectAnswer-D
Answer-D.Proximalpara-aorticlymphnodes
ThestandardmanagementforstageIorIItumorsandselected
casesofstageIIIdiseaseisradicalnephrectomy.
ThisprocedureinvolvesenblocremovalofGerota'sfasciaandits
contents,includingthekidney,theipsilateraladrenalgland,and
adjacenthilarlymphnodes.

1308.Normalurineflowrateinhealthyadults
inurodynamicstudyis-
a)10ml/sec
b)20ml/sec
c)25ml/sec
d)5ml/sec
CorrectAnswer-B
Answer-B.20ml/sec
Ages14to45--Theaverageflowrateformalesis21mL/sec.The
averageflowrateforfemalesis18mL/sec.
Ages46to65--Theaverageflowrateformalesis12mL/sec.The
averageflowrateforfemalesis18mL/sec.
Ages66to80--Theaverageflowrateformalesis9mL/sec.The
averageflowrateforfemalesis18mL/sec.

1309.Dermoepidermalburniswhatdegreeof
burn-
a)I
b)II
c)III
d)IV
CorrectAnswer-B
Answer-B.II
DermoepidermalburnsareSuperficialIIdegreeburns.

1310.Revascularizationandangiogenesis
processafterskingraftingisseenafter
howmanydaysaftertheprocedure?

a)4
b)5
c)6
d)7
CorrectAnswer-B
Answer-B.5
Revascularizationorangiogenesis
Afterapprox5days,revascularizationoccursandthegraft
demonstratesbotharterialinflowandvenousoutflow.

1311.Themostcommonlyused
myocutaneouspediclegraftforpelvis
surgeriescontainsmusclesegments
from-

a)Rectusabdominismuscle
b)Externalobliquemuscle
c)Internalobliquemuscle
d)Transversusabdominismuscle
CorrectAnswer-A
Answer-A.Rectusabdominismuscle
Themostfrequentlyusedmyocutneouspediclegraftscontain,
musclesegmentsfromtherectusabdominismuscleoftheanterior
abdominalwall,gracialismuscleoftheinnerthigh,bulbocavernosus
muscleofthevulva,thetensorfascialatamuscleofthelateralthigh,
andgluteusmaximusmuscle.

1312.Kernahen'sstriped'Y'classification,
Mainreferencepointis-
a)Incisiveforamen
b)Softpalate
c)Hardpalate
d)Thirdmolar
CorrectAnswer-A
Answer-A.Incisiveforamen
Kernahen'sstriped'Y'classification
Itisusedtoclassifycleftlipandcleftpalate
Theincisiveforamenistakenasreferencepoint


1313.LAHSALcodeisusedtorepresent
congenitalmalformationof-
a)Lip
b)Alveolus
c)Hardandsoftpalate
d)Alltheabove
CorrectAnswer-D
Answer-D.Alltheabove

1314.WhatisthemeaningoftheFrenchword
"Debridement"
a)UnleashorCutOpen
b)Debulk
c)Sanitize
d)Rehydration
CorrectAnswer-A
Answer-A.UnleashorCutOpen
TakenfromtheFrenchmeaningto'unleashorcutopen',
debridementhascometomeanmorethansimplythelayingopenof
tissues.Itplaysacrucialpartinthemanagementoftrauma.

1315.Cardinalrulefordressingofpressure
ulceristo-
a)Keepulcertissuedryandsurroundingintacttissuedry
b)Keepulcertissuedryandsurroundingintacttissuemoist
c)Keepulcertissuemoistandsurroundingintacttissuedry
d)Keeptheulcertissuemoistandsurroundingintacttissuemoist
CorrectAnswer-C
Answer-C.Keepulcertissuemoistandsurroundingintact
tissuedry
Anidealdressingshouldprotectthewound,bebiocompatibleand
provideidealhydration.
Thetypeoftheulcerbedandthedesireddressingfunction
determinethetypeofdressingneeded.
Thecardinalruleistokeeptheulcertissuemoistandthe
surroundingintacttissuedry.

1316.Outcomeofburnsdependson-
a)Extentofburns
b)Typeofresuscitationfluid
c)Maintenanceofairway
d)Skingrafting
CorrectAnswer-A
Answer-A.Extentofburns
Thevariousfactorsinfluencingtheoutcomeofburnsare:
Extentofburns
Depthofburns
TimingoffirstEscharectomy(removesdevitalizedtissueandhence
sourceofinfection)
Ageandmedicalcomorbidities

1317.Allofthefollowingaretrueregarding
fluidresuscitationinburnpatients
except:

a)Considerintravenousresuscitationinchildrenwithburns
greaterthan15%TBSA
b)Oralfluidsmustcontainsalts
c)MostpreferredfluidisRinger'slactate
d)Halfofthecalculatedvolumeoffluidshouldbegiveninfirst8
hours
CorrectAnswer-A
Ans.A:Considerintravenousresuscitationinchildrenwith
burnsgreaterthan15%TBSA
Inchildrenwithburnsover10%TBSAandadultswith15%TBS,
considertheneedforintravenousfluidresuscitation.
Iforalresuscitationistobecommenced,itisimportantthatthe
watergivenisnotsaltfree.
Preferredfluid:LactatedRinger'sSolution,becauseitis:
*Isotonic
*Cheap
*Easilystored
-Resuscitationformulas:Parklandformulamostcommonlyused
-Fluidcalculation:4xweightinkgx%TBSAburn
-Give1/2ofthatvolumeinthefirst8hours.Giveother1/2innext
16hours
-TBSA:Totalburnssurfacearea.

1318.Thebesttreatmentforcystichygroma
is-
a)Surgicalexcision
b)Radiotherapy
c)Sclerotherapy
d)Chemotherapy
CorrectAnswer-A
Ans.is'a'i.e.,SurgicalExcision
Definitivetreatmentiscompleteexcisionofthecystatanearlyage.

1319.Apatientafterroadtrafficaccident
presentedwithtensionpneumothorax.
Whatisthefirstlineofmanagement?

a)Insertwideboreneedlein2ndintercostalspace
b)ImmediatechestX-ray
c)CTscanEmergencythoracotomy
d)Emergencythoracotomy
CorrectAnswer-A
Answer-A.Insertwideboreneedlein2ndintercostalspace
Firstlineofmanagementintensionpneumothorax:Insertwidebore
needlein2ndintercostalspace.

1320.Preferredtreatmentfororaltongue
carcinomawhichinfiltratesthelocal
corticalboneis-

a)Subtotalglossectomy
b)Subtotalglossectomy+selectiveneckdissection
c)Subtotalglossectomy+selectiveneckdissection+
mandibulectomy
d)Totalglossectomy+selectiveneckdissection+
mandibulectomy
CorrectAnswer-C
Answer-C.Subtotalglossectomy+selectiveneckdissection+
mandibulectomy
Themanagementplanforlocallyadvancedtonguecarcinomas
includessubtotalglossectomy+selectiveneckdissection+
mandibulectomy.
Advancedtumors(T3andT4)oftenencroachuponthefloorofthe
mouthand,occasionally,themandible.
Inthesecircumstances,aresectionofthetongueandfloorofthe
mouthandmandibleisrequired.

1321.Forlowerlipcarcinomaof<1cminsize.
Thetreatmentofchoicewillbe-
a)Radiation
b)Chemotherapy
c)Excision
d)Radiationandchemotherapy
CorrectAnswer-C
Answer-C.Excision
Smalltumors(T1&T2)(<2cm)-If1/3rdorlessthan1/3rdlip
involved-VorW-shapedexcision+primaryclosure.
Excisionoflowerlipupto1/3rdcanbesuturedprimarilyin3layers?
mucosa,muscle,andskinkeepingvermillionborderinproper
apposition.

1322.

UptoLevelIIIlymphnodedissectionisdone
forwhichnodalstatusoforopharyngeal
cancer?

a)N1
b)N2
c)N3
d)N4
CorrectAnswer-A
Answer-A.N1
Managementofneck
NOSelectiveneckdissection
OralCavity atleastlevelsl-lll
Oropharynx atleastlevelsll-lV
N1-N2a-c
Selectiveorcomprehensiveneck
LevelVl
Subglotticlaryngealcancers

1323.Hoarsenessofvoiceinlungcarcinoma
isduetoinvasionofwhichstructure?
a)Recurrentlaryngealnerve
b)Internallaryngealnerve
c)Glossopharyngealnerve
d)Vagusnerve
CorrectAnswer-A
Answer-A.Recurrentlaryngealnerve
Hoarseness-Recurrentlaryngealnerveinvasion

1324.Acrallentiginestypeofmalignant
melanomaoccursin-
a)Face
b)Napeofneck
c)Mucosa
d)Sunexposedareas
CorrectAnswer-C
Answer-C.Mucosa
Acrallentiginous-
Leastcommonwithworstprognosis
MCsite-sole,mucosa

1325.Whichofthefollowingprovides
excellentdetailsaboutthe
chemodectomas?

a)Xray
b)CTangiography
c)MRI
d)PETSCAN
CorrectAnswer-C
Answer-C.MRI
MRIscanningprovidesexcellentdetailinmostcasesof
chemodectomas.

1326.Hemorrhagecommonlyseenwithtrivial
traumainelderly-
a)Subduralhemorrhage
b)Extraduralhemorrhage
c)Subarachnoidhemorrhage
d)Intraparenchymalhemorrhage
CorrectAnswer-A
Answer-A.Subduralhemorrhage
Subduralhemorrhagefrequentlyoccursinolderadults,after
apparentlytrivialtraumaandisoftenrelatedtoafallinwhichthereis
nodirecttraumatothehead.
Recurrenttrivialtraumainelderlyisthemostcommoncauseof
subduralhemorrhagedeveloping.

1327.Mostcommonorganinjuredin
penetratinginjuryoftheabdomen:
a)Liver
b)Spleen
c)Smallbowel
d)noneofthese
CorrectAnswer-C
Ans-C-SmallBowel
Themostcommoncauseisastaborgunshot.
Themostcommonorgansinjuredarethesmallbowel(50%),large
bowel(40%),liver(30%),andintra-abdominalvascular(25%).
Whentheinjuryiscloserange,thereismorekineticenergythan
thoseinjuriessustainedfromadistance.Eventhoughmostgunshot
woundstypicallyhavealinearprojection,thehigh-energywounds
areassociatedwithunpredictableinjuries.
Theremayalsobesecondarymissileinjuriesfromboneorbullet
fragments.Stabwoundsthatpenetratetheabdominalwall
aredifficulttoassess.

1328.Indicationforsurgicalcompartment
releaseincompartmentSyndromein
anycompartmentisabsolutepressure
greaterthan?

a)15mmHg
b)20mmHg
c)30mmHg
d)Variesfromcompartmenttocompartment
CorrectAnswer-C
30mmHgREF:Withtext
"Settingthethresholdforfasciotomyataperfusionpressureof30
mmHgcanbeconsideredsafe,butstillmayleadtoovertreatmentif
usedroutinely"REF:Evidence-basedOrthopedics-MohitBhandari
Page634
Differentauthorsconsidersurgicalinterventionif:(REF:TiwariA,
HaqAI,MyintF,HamiltonG.Acutecompartmentsyndromes.BrJ
Surg
2002;89:397-412.)
1. AbsoluteICPgreaterthan30mmHg
2. DifferencebetweendiastolicpressureandICPgreaterthan30
mmHg
3. DifferencebetweenmeanarterialpressureandICPgreaterthan40
mmHg
"Intracompartmentalpressuremaybemeasuredbythewick
catheterinpatientssuspectedtohavecompartmentsyndrome.By
suchmethodapressureof30mmHgormoresustainedfor6-8
hoursormoreisalikelyindicationfordecompressivefasciotomy"
REF:SkeletalinjuryinthechildbyJohnAnthonyOgdenPage317
Intracompartmental Comments

Intracompartmental Comments
pressure
Normalcompartmentpressureoflower
<15mmHg
limbs
Venousdrainagefromclosedmyofascial
>25mmHg
spacesisimpaired.
>30mmHg
Completevenouscollapse
>60mmFig
Neuromuscularischemia
REF:Masteryofvascularandendovascularsurgery-GeraldB.
Zelenock,ThomasS.Huber,LouisM.MessinaPage507

1329.Achildswallowedawatchbattery
containingalkalinecontent.Whatnext-
a)ImmediateX-raymeasurements
b)Removesurgicallyimmediately
c)CTabdomen
d)Laxatives
CorrectAnswer-A
Answer-A.ImmediateX-raymeasurements
Alkalinedrycellbatteriescontain:
Sodiumhydroxide
Potassiumhydroxide

1330.Burrholeisdonefor-
a)ChronicSDH
b)EDH
c)SAH
d)Contusion
CorrectAnswer-A
Answer-A.ChronicSDH
LiquefiedSDHsarecommonlytreatedwithdrainagethroughoneor
twoburrholesplacedoverthethickestaspectsofthehematoma.
Manysurgeonsplacefrontalandparietalburrholesthatlatercanbe
incorporatedintoafrontotemporoparietalcraniotomy,ifneeded.

1331.Typicalofrectussheathhematomais-
a)Severetenderness
b)Bluishdiscoloration
c)Firmpainfulmass
d)Ecchymosis
CorrectAnswer-C
Answer-C.Firmpainfulmass
Clinicalfeaturesofrectussheathhematoma
A)Symptoms
Commonhistoricalfeaturesofrectussheathhematoma(RSH)
includeacuteabdominalpain,fever,nausea,andvomiting.
B)Signs
i)Vitalsigns
Alow-gradefeveriscommoninrectussheathhematoma.The
hematomacanbelargeenoughtocompromiseintravascular
volume,withresultantsignsofhypovolemicshockincluding
hypotension,tachycardia,andtachypnea.
ii)Abdominalexamination
Typically,theabdominalexaminationrevealsapalpable,painful,
firm,nonpulsatileabdominalmasscorrespondingtotherectus
sheath.Themassmaybebilobarwithacentralgroove.Themass
doesnotmovewithrespiration.Becausethehematomaisdeepto
thesubcutaneoustissueandrectusmuscles,themassisnotalways
palpable,particularlyinobesepatients.

1332.Percuteneouschemicallumbar
sympathectomyispractisedusing-
a)Phenol
b)Ethanol
c)Formalin
d)Aceticacid
CorrectAnswer-A
Answer-A.Phenol
Chemicalsympathectomyrequirestheinjectionofsmallquantitiesof
diluteaqueousphenolintothelumbarsympatheticchainunder
radiographiccontrol.

1333.Whichofthefollowingissparedin
lumbarsympathectomy:
September2009

a)L1
b)L2
c)L3
d)L4
CorrectAnswer-A
Ans.A:Ll
Topreservesexualfunctions,Llispreserved.

1334.Mostcommonlyperformedand
acceptablemethodofbariatricsurgery
is:

a)Biliopancreaticdiversion
b)Biliopancreaticdiversionwithilcostomy
c)Laparoscopicgastricbanding
d)Roux-en-Ygastricbypass.
CorrectAnswer-D
Ans:D.Roux-en-Ygastricbypass.
(Ref.Sabiston20/ep1187,19/ep363;Schwartz10/ep1112,9/952;
Harrison19/p2398).
Roux-en-Ygastricbypass:
Mostcommonlyperformed&acceptablemethodofbariatricsurgery.
3restrictive-malabsorptivebypassprocedures:
Combineelementsofgastricrestriction&selectivemalabsorption.
ProceduresincludeRoux-en-Ygastricbypass,biliopancreatic
diversion&biliopancreaticdiversionwithduodenalswitch.
Roux-en-Y:
Mostcommonlyundertaken&mostacceptedbypassprocedure.
Performedwithanopenincisionorbylaparoscopy.



1335.Whichofthefollowingelectrolyte
abnormalitiescanbeseenafterbrain
injury?

a)Hyponatremia
b)Hyperkalemia
c)Hypomagnesemia
d)Alltheabove
CorrectAnswer-A
Answer-A
Electrolyteabnormalitiesoccurin60%ofthepatientswithhead
injury.
Patientswithbraininjuryareatahighriskforthedevelopment
ofelectrolyteimbalanceincludinghyponatremia,hypocalemia,
hypophosphatemiaaswellashypokalemiaand(toalesserdegree)
Hypomagnesemia.
Sotheappropriateanswerwillbehyponatremia.

1336.Neuroimagingfeaturesof
neurocysticercosisareconsideredas
criteriafordefinitivediagnosis-

a)Major
b)Minor
c)Probable
d)Absolute
CorrectAnswer-A
Answer-A.Major
Evidenceoflesionshighlysuggestiveofneurocysticercosison
neuroimagingstudies
Positiveserumimmunoblotforthedetectionofanticysticercal
antibodies
Resolutionofintracranialcysticlesionsaftertherapywith
albendazoleorpraziquantel
Spontaneousresolutionofsmallsingleenhancinglesions.

1337.Featuresofmarasmusareallexcept:
a)Absenceofanasarca
b)Increasedappetite
c)Excessivecatabolismofadiposetissueandmuscleprotein
d)UncompensatedphaseofPEM
CorrectAnswer-D
Answer-D.UncompensatedphaseofPEM
Itisduetoprolongdeficiencyofcaloriesandproteins.Thusthereis
exessivecatabolismofadiposetissueandmuscleprotein.
Itischaracterizedbygrosswastingofmuscleandsubcutaneous
tissuesresultinginemaciationandmarkedstunting.
Childmayshowvoraciousappetite.
MarasmusrepresentsthecompensatedphaseofPEM.

1338.Allofthefollowingarecausesof
pseudoparalysisexcept
a)Osteomyelitis
b)Scurvy
c)Septicarthritis
d)Polio
CorrectAnswer-D
Answer-D.Polio
Causesofpseudoparalysis

1. Scurvy(vitaminCdeficiency)
2. Osteomyelitis
3. Septic(arthritis)
4. Congenitalsyphilis

1339.Whichofthefollowingcanleadto
regressionofdevelopmental
milestones

a)Rett'ssyndrome
b)Autism
c)Neuromusculardiseases
d)Alloftheabove
CorrectAnswer-D
Answer-D.Alloftheabove
Thehallmarkofmanydegenerativedisordersisneurological
regression.
Lossofonlylanguageskills?autism?suspected.
RegressionofbothlanguageandmotormilestonesRett's
syndrome.

1340.AllofthefollowingarefeaturesofRett's
syndromeexcept
a)Microcephaly
b)Regressionofmilestones
c)Cardiaarrhythmias
d)FocalConvulsions
CorrectAnswer-D
Answer-D.FocalConvulsions
Thisisthecharacteristicfeatures,thattheybegintoloosetheir
acquiredskills,e.g.,cognitiveandheadgrowthisnormalduring
earlyperiodafterwhichthereisanarrestofgrowth.
Acquiredmicrocephaly(Declerationofheadgrowthdueto
significantlyreducedbrainweight).
Mostchildrendeveloppeculiarsighingrespirationswithintermittent
periodsofapneathatmaybeassociatedwithcyanosis-4Breath
holdingspells.

1341.Childwhileplayinghassuddenlossof
consciousnessandappearspale.There
isnosignificantmedicalhistoryandthe
childwasotherwisehealthy.Whichof
thefollowingisthemostprobable
diagnosis?

a)Attentiondeficithyperkineticdisorder
b)Breathholdingspell
c)Autism
d)Rett'ssyndrome
CorrectAnswer-B
Answer-B.Breathholdingspell
Pallidform(Pallidspells)
Theseareinitiatedbypainfulexperience,e.gfallingandstrikingthe
head.
Pallidspellsareduetoexcessivecentralparasympetheticactivity.
Clinicalfeaturesincludepallor,apnea,lossofconsciousness,
hypotonia,seizuresandbradycarda.
Treatmentincludessupportandreassuranceofparents.Atropine
maybeusedinrefractorycases

1342.Infantilebodyproportioninadultsis
seeninallexcept-
a)Achondroplasia
b)Hypothyroidism
c)Klinefelter'ssyndrome
d)Cretinism
CorrectAnswer-C
Answer-C.Klinefelter'ssyndrome
Infantiletypebodyporportion
Achondroplasia
Juvenilemyxedema(hypothyroidism)
Cretinism

1343.Ifchronologicalage>skeletalagewith
normalgrowthvelocity,thenthefinal
heightthatisexpectedtobeachieved
is

a)Normal
b)Lessbecauseofsmallbones
c)Morethanexpected
d)Lessbecauseofepiphysealclosureduetoacceleratedgrowth
velocity
CorrectAnswer-A
Answer-A.Normal
Ifthegrowthvelocityisnormalbutthechronologicalageismore
thantheboneage,thenthediagnosisisConstitutionaldelayin
growth.
Constitutionaldelayingrowth
Itisthemostcommoncauseofshortstatureinmidchildhoodperiod
buttheultimateheightisnormal.
Theirbirthweightandheightarenormal.Strongfamilyhistoryof
parentshavingshortstatureinchildhoodwithdelayinonsetof
pubertyisusuallypresent.

1344.Whichofthefollowingabouthormone
levelsinamalnourishedchildistrue?
a)Increasedinsulinlevels
b)Decreasedcortisollevels
c)Increasedgrowthhormone
d)Alloftheabove
CorrectAnswer-C
Answer-C.Increasedgrowthhormone
changesinPEM
Decreasedinsulinlevels
Increasedcortisol
Increasedgrowthhormone

1345.A9montholdchildwithrespiratory
rate53/minandpresenceofcoughis
classifiedas:

a)SIRS
b)Respiratorydistress
c)Tachypnoea
d)ARDS
CorrectAnswer-C
Answer-C.Tachypnoea
Tachypnea(fastbreathing):Fastbreathingisdefinedas:

1. lessthan2monthsofage->60breathsperminute
2. Childaged2monthsupto12months-50breathsperminute
3. Childaged12monthsupto5years-40breathsperminute

1346.Anewbornafterprolongedlabourisnot
breathingwellandafter30secondsof
receiving100%oxygenbybagand
mask,heartrateis88beatspermin,
whatisthenextstepinmanagement?

a)Discontinueoxygenandventilation
b)Discontinueoxygen,continueventilation
c)Continueoxygenandventilation
d)Startchestcompressions
CorrectAnswer-C
Answer-C.Continueoxygenandventilation
Aftertheinfanthasreceived30secondsofventilationwith
100%oxygenbybagandmask,evaluationofheartrateshould
bedone-
HR>100Discontinueventilationifspontaneousrespirationis
present.
HR60to100Continueventilation
Below60Continueventilation+chestcompressions
After30secondsofchestcompressions,theheartrateis
checked.
HR<60Continuechestcompressionandbag&maskventilation
+initiatemedications.
HR>60Discontinuechestcompressionbutcontinuebag&
maskventilationuntiltheheartrateisabove100.
[Ref:O.P.Ghai7th/ep.98]

1347.Howarechestcompressionsgivenina
newborn?
a)Usingpalmonthelowerthirdofsternum
b)Usingtwofingersonthemiddlethirdofsternum
c)Usingthetwothumbsonthelowerthirdofsternum
d)Usingthreefingersonthelowerthirdofsternum
CorrectAnswer-C
Answer-C.Usingthetwothumbsonthelowerthirdofsternum
Two-fingertechnique
Thetipsofthemiddlefingerandeithertheindexfingerorringfinger
ofonehandareusedtocompressthesternum.
Theotherhandisusedtosupporttheinfant'sback,unlesstheinfant
isonaveryfirmsurface.

1348.Contraindicationofbagandmask
ventilationareallofthefollowing
except:
March2009

a)Tracheo-esophagealfistula
b)Hiatushernia
c)Pregnancy
d)Emptystomach
CorrectAnswer-D
Ans.D:EmptyStomach
Bag-maskventilationcanproducegastricinflationwith
complications,includingregurgitation,aspiration,andpneumonia.
Conditionspredisposingtoaspirationare:
Fullstomachpatients
Hiatushernia,pregnancy
Intestinalobstruction
Tracheo-esophagealfistula
Meconiumaspirationsyndrome
Gastricinflationcanelevatethediaphragm,restrictlungmovement,
anddecreaserespiratorysystemcompliance

1349.Graspreflexdevelopsby-
a)20weeks
b)24weeks
c)28weeks
d)32weeks
CorrectAnswer-C
Ans.is'c'i.e.,28weeks
Reflex-Ageofappearance32-Ageofdisappearance(afterbirth)

1350.Asymmetrictonicneckreflex
disappearsatwhatage?
a)2months
b)3months
c)6months
d)8months
CorrectAnswer-C
Answer-C.6months
Assymmetictonicneck-wksofgestation4-66-7months

1351.Trueabouttonicneckreflexis
a)Extensionofarmonipsilateralside,flexiononcontralateralside
b)Extensionofarmoncontralateralside,flexiononipsilateralside
c)Extensionofarmsonbothsides
d)Flexionofarmsonbothsides
CorrectAnswer-A
Answer-A.Extensionofarmonipsilateralside,flexionon
contralateralside
Thetonicneckreflexisproducedbymanuallyrotatingtheinfant's
headto1sideandobservingforthecharacteristicfencingposture
(extensionofthearmonthesidetowhichthefaceisrotatedand
flexionofthecontralateralarm).
Anobligatorytonicneckresponse,inwhichtheinfantbecomes
"stuck"inthefencingposture,isalwaysabnormalandimpliesa
CNSdisorder.

1352.Withdrawalreflexisanexampleofwhichofthefollowing?
a)Monosynapticreflex
b)Polysynapticreflex
c)BothAandBoftheabove
d)Noneoftheabove
CorrectAnswer-B
Thewithdrawalreflexisatypicalpolysynapticreflexthatoccursin
responsetoausuallypainfulstimulationoftheskinorsubcutaneous
tissuesandmuscle.
Whenareflexarcconsistsofonlytwoneuronsinananimal(one
sensoryneuron,andonemotorneuron),itisdefinedas
monosynaptic.Monosynapticreferstothepresenceofasingle
chemicalsynapse.Inthecaseofperipheralmusclereflexes(patellar
reflex,achillesreflex),briefstimulationtothemusclespindleresults
incontractionoftheagonistoreffectormuscle.
Inpolysynapticreflexpathways,oneormoreinterneuronsconnect
afferent(sensory)andefferent(motor)signals.Allbutthemost
simplereflexesarepolysynaptic,allowingprocessingorinhibitionof
polysynapticreflexeswithinthebrain.
Ref:Ganong'sReviewofMedicalPhysiology23rdedition,Chapter
9.

1353.Closureofpatentductusarteriosusis
stimulatedby?
a)ProstaglandinF2a
b)Cycloxygenase
c)Increasein02tensionatbirth
d)Hypercarbia
CorrectAnswer-C
Answer-C.Increasein02tensionatbirth
Themechanismproducingtheinitialconstrictionofductusarteriosus
isnotcompletelyunderstood,buttheincreaseinarterial02tension
playsanimportantrole.
Onemorefactorwhichhelpsinclosureoftheductusarteriosusis
thedecreaseinconcentrationofprostaglandinsatthetimeofbirth.

1354.FalseaboutPDAis
a)Morecommoninfemales
b)Anatomicalclosuretakes21hoursafterbirth
c)PGEmaintainspatencyofductus
d)Dilatationofascendingaorta
CorrectAnswer-B
Answer-B.Anatomicalclosuretakes21hoursafterbirth
Patentductusarteriosusisacommunicationbetweenthepulmonary
arteryandaorta.
'Functionalclosure'takesplacewithin15hoursofbirth.'Anatomic
closure'ofductusarteriosusoccurs10-21daysafterbirth.
Prostaglandinsmaintainthepatencyofductus.
Thepersistenceoffunctionofductusarteriosusbeyond24hours
afterbirthisconsideredasPDAintermneonate,i.e.iffunctional
closuredoesnottakeplacein24hoursafterbirth,itisconsidered
asPDA.

1355.Whichofthefollowingisnotseenin
patentductusarteriosus?
a)Leftatrialhypertrophy
b)Leftventricularenlargement
c)Continuousmurmur
d)AttenuatedSI
CorrectAnswer-D
Answer-D.AttenuatedSI
Increasedflowafterpassingthroughlungreachestheleftatriumand
causesvolumeoverloadLeftatrialdilatationandhypertrophy.
Increasedbloodvolumepassesfromleftatriumtoleftventricle
throughmitralvalve,i.e.,increasedflowthroughmitral
valveAccentuationofS1anddelayeddiastolicmurmur.
Leftventriclereceiveslargeramountofbloodthatresultsinvolume
overloadLeftventricleenlargement.

1356.Whichofthefollowingcongenital
anomaliesleadstoheartfailureat
birth?

a)Totalanomalouspulmonaryvenousconnection
b)Transpositionofgreatarteries
c)Pulmonaryatresia
d)Coarctationofaorta
CorrectAnswer-C
Answer-C.Pulmonaryatresia
TimingofCHFincongenitalheartdiseases-Pulmonary,mitraland
aorticatresias
Hypoplasticleftandrightheartsyndromes,transpositionand
malpositionofgreatateries.

1357.Thefollowingfeaturesaretruefor
tetralogyofFallot,except-
a)Ventricularseptaldefect
b)Rightventricularhypertrophy
c)Atrialseptaldefect
d)Pulmonarystenosis
CorrectAnswer-C
Ans.is'c'i.e.,ASD
TetralogyofFallot
*Theclassicalexampleofcyanoticpatientswithpulmonicstenosis
istetralogyoffallot.
*TOFisthecommonestcongenitalheartdisease.
*ConstituentsofTOF
VentricularseptaldefectOverridingordextroposedaorta
PulmonicstenosisRightventricularhypertrophy.

1358.Mostimportantprognosticmarkerof
tetralogyoffallot
a)VSD
b)Pulmonarystenosis
c)Overridingofaorta
d)Rightventricularhypertrophy
CorrectAnswer-B
Answer-B.Pulmonarystenosis
Tetrologyoffallothas4components:

1. Obstructiontorightventricularoutflow(pulmonarystenosis),
2. Amal-alignmenttypeofventricularsepta]defect(VSD),
3. Dextropositionoftheaortasothatitoverridestheventricular
septum,and
4. Rightventricularhypertrophy
[RefNelson20thlep.2211]

1359.Microcephalyiscommoninchildrenof
motherswithallexcept
a)Alcoholintake
b)Warfarinintake
c)Warfarinintake
d)Varicella
CorrectAnswer-B
Answer-B.Warfarinintake
Secondary
Structuraldefects:Neuraltubedefects(anencephaly,
encephalocele).
Metabolicdisorders:Rhenylketonuria,citrullinemia,methylmalonic
aciduria.
Congenitalinfections:Rubella,CMV,HSV,toxoplasmosis,syphilis,
varicella.
Teratogens:Alcohol,tobacco,cocaine,heroin.
Others:Maternaldiabetes,maternalphenylketonuria,
hypothyroidism,hypopituitrism,adrenalinsufficiency.

1360.Whichofthefollowingstatementistrueforphysiologicaljaundicein
neonate?
a)Occursinthefirst6hoursofdelivery
b)Neurologicalsequelaearecommon
c)Besttreatedbyphototherapy
d)Startson2nddayoflife
CorrectAnswer-D
Mostneonatesdevelopvisiblejaundiceduetoelevationof
unconjugatedbilirubinconcentrationduringtheirfirstweek.
ThiscommonconditioniscalledPhysiologicaljaundice.
Itlastsfor5daysinterminfants&7daysinpreterminfants.
Itdoesnotrequireanytreatment&disappersspontaneously.In
pathologicaljaundiceclinicaljaundicewillappearinthefirst24hrsof
life.
Ref:Nelson,18thEdition,Pages760-761;OPGhai,6thEdition,
Pages170-171.

1361.Whatisthecapacityofstomachat
birth
a)5ml
b)25ml
c)50ml
d)100ml
CorrectAnswer-C
Answer-C.50ml
Day15-7ml.Sizeofcherry
Day322-27ml.Sizeofwalnut
Oneweek45-60ml.Sizeofanapricot
Onemonth80-150ml.Sizeofalargeegg

1362.Colourofstoolsinbreastfednewborn
is-
a)Red
b)Green
c)Black
d)Golden
CorrectAnswer-D
Answer-D.Golden
Colourofstoolsinneonate
Meconium(firststool)ispassedwithin24hours.Afterthat
meconiumstools(blacktarry)canbepassedupto3days.
On4th-5thdaystransitionalstools(greenish)arepassed.After5
daysregularmilkstools(goldenyellow)arepassed.
Thereisgoldendiscolorationofstool.

1363.Trigonocephalyisduetopremature
closureofwhichsuture?
a)Sagittalsuture
b)Metopicsuture
c)Lambdoidsuture
d)Coronalsuture
CorrectAnswer-B
Answer-B.Metopicsuture
Trigonocephal-Metopicsuture
Keelshapedforehead
Hypotelorism
Abnormalitiesofforebrain

1364.Whichofthefollowingisamarkerforneuraltubedefects?
a)Phosphatidylesterase
b)Pseudocholinesterase
c)Acetylcholinesterase
d)Butyrylcholinesterase
CorrectAnswer-C
Neuraltubedefectsareassociatedwithhighlevelsof
Acetylcholinesterase.
Ref:GeneticdisordersandFetus,4thEdition,Page673;Ian
Donald'sPracticalObstetricProblemByRenuMisra,6thEdition,
Page44
*Inintraembryoniclife,neuraltubeisopenatbothendandfreely
communicatewithamnioticcavity.Failureofclosureofneuraltube
resultsinpersistentofthiscommunication.Thisallowsexcretionof
followingfetalsubstancesintoamnioticcavity?
-Alpha-fetoprotein
-Acetylcholinesterase
*TheseserveasbiochemicalmarkersforNTDsforprenatal
diagnosis.

1365.Meconiumileusisassociatedwith:
a)Cysticfibrosis
b)Infantofdiabeticmother
c)Hypothyroidism
d)Noneoftheabove
CorrectAnswer-A
Infantswithcysticfibrosishavecharacteristicpancreaticenzyme
deficienciesandabnormalchloridesecretionintheintestinethat
resultintheproductionofviscous,water-poormeconium.
Meconiumileusoccurswhenthisthick,highlyviscousmeconium
becomesimpactedintheileumandleadstohigh-gradeintestinal
obstruction.
Ref:Schwartz'sprincipleofsurgery9thedition,chapter39.

1366.RDAofzincinachild?
a)10mg
b)20mg
c)6-8mg
d)4-5mg
CorrectAnswer-C
Ans.is'c'i.e.,6-8mg
RDAofZincinchildren
1-3years-->3mg
4-8years-55mg
9yearsandabove(male)-->8-11mg
9yearsandabove(female)-->8mg

1367.Whichofthefollowingisnotafeature
ofMinimalchangedisease?
a)Hypertension
b)Edema
c)Proteinuria
d)Responsivetosteroidtherapy
CorrectAnswer-A
Answer-A.Hypertension
MinimalChangeDiseaseisthemostcommoncauseofNephrotic
syndromeinchildren.
EdemaandSelectiveproteinuriaarefeaturesofnephrotic
syndrome.Fevermaybepresentonaccountofincreased
susceptibilitytoinfection.
Minimalchangediseaseprsentswithinsidiousonsetofnephrotic
syndromeinchildrenbelow6yearsofage.
Hypertensionisnotafeatureofnephroticsyndromeandisrarein
Minimalchangedisease.
Hematuria(afindingofnephriticsyndrome)isalsorare.

1368.Whichofthefollowingdrugsisusefulintheprophylaxisofmigraine?
a)Propranolol
b)Sumatriptan
c)Domperidone
d)Ergotamine
CorrectAnswer-A
Drugssuchastopiramate,valproate,propanolol,timolol,candesartan,verapamiland
amitryptillineareindicatedinmigraineprophylaxis.
Migraineprophylaxisisindicatedwhenmigraineheadachesoccurmorethantwoorthree
timesamonthorwhenitisassociatedwithsignificantdisability.Afterinitiationoftherapy,it
shouldbecontinuedforseveralmonths.Oncethepatientremainsheadachefree,thedose
istaperedandthedrugiseventuallywithdrawn.BotulinumtoxintypeAwasapprovedby
theUSFoodandDrugAdministration(FDA)formigrainepreventioninlate2010.
Ref:CurrentMedicalDiagnosisandtreatment2012,Chapter24

1369.Drugofchoiceforinfantilespasmis?
a)Vigabatrin
b)Adrenocorticotropichormone(ACTH)
c)Ethosuximide
d)Carbamazepine
CorrectAnswer-A
Ans.'a'i.e.,Vigabatrin
Vigabatrin(drugofchoice),ACTH(2ndchoice)and
corticosteroids
areusedfortreatment.

1370.WhatistherateofCSFformationin
children?
a)0.3ml/min
b)1ml/min
c)3ml/min
d)20ml/min
CorrectAnswer-A
Answer-A.0.3ml/min
TherateofCSFformationinchildrenandadultsis:-
0.3to0.4ml/minOR
18to20ml/hour

1371.Mostcommoncauseofcranial
irradiationinchildrenis
a)SmallcelllungCa
b)ALL
c)AML
d)Craniopharyngioma
CorrectAnswer-B
Answer-B.ALL
ALLandsmallcelllungCaaretwomajorindicationsforcranial
irradiation,evenprophylacticallytopreventbrainmetastasis.
Inchildren,ALListhemostcommoncause.

1372.Mostcommoncauseofsevere
hematemesisinachildis-
a)Portalhypetension
b)Pepticulcer
c)Malloryweisssyndrome
d)Noneoftheabove
CorrectAnswer-A
Ans.is'a'i.e.,Portalhypertension
"Massivehematemesisinachildisalmostalwaysduetovariceal
bleeding".
Varicealbleedingisduetoportalhypertension.

1373.Whichofthefollowingisnotafeature
ofphysiologicalanaemiaofinfancy?
a)Terminfanthemoglobin7gm%
b)Preterminfanthemoglobin7gm%
c)Terminfanthemoglobin9gm%
d)Preterminfanthemoglobin9gm%
CorrectAnswer-A
Answer-A.Terminfanthemoglobin7gm%
PhysiologicAnemiaofInfancy

1. Hemoglobindropstolowpointatage6to8weeks
2. ErythropoietinnadirdropsHemoglobin
3. TermInfants:Hemoglobindropsto9-11g/dl
4. PretermInfants:Hemoglobindropsto7-9g/dl
[RefAnemiaininfancy,pediatricinreviewAmericanacademyof
pediatrics2012]

1374.Meanhemoglobinina1yearoldchild
is
a)18.5g/dl
b)16.5g/d1
c)14g/c11
d)12g/dl
CorrectAnswer-D
Answer-D.12g/dl
Hblevel
Prevalence(%)
(g/L)
Age
Mildanemia
ModeratetoSevere
N
Meam SE
(mo)
(Hb<110g/L)
anemia(Hb<80g/L)
0-5
56 9.8
0.21 78.1
5.2
6-11
88 10.08 0.17 75.3
5.8
12-23 128 10.04 0.21 63.3
11.9
24-60 251 10.18 0.13 68.3
8.1
Totel 523 10.09 0.09 69.3
8.3
Hb,hemoglobin
a.Meansandfrequenciesareweighted.

1375.WhichofthefollowingbiochemicaltestisusedtodiagnoseDubin
Johnsonsyndrome?
a)Serumtransaminases
b)Bromsulphaleintest(BSP)
c)Hippuratetest
d)Gammaglutamyltransferaselevel
CorrectAnswer-B
Bromsulphaleintest(BSP)isthediagnostictestforDubin-JohnsonSyndrome.Biliary
excretionofnumerousanioniccompoundssuchasBromsulphalein(BSP)iscompromised
inDubin-JohnsonSyndrome(DJS).Inthistest,BSPisadministeredasIVbolusandits
clearancefromplasmaisdetermined.BSPlevelsshowacharacteristicriseinpatientswith
DJSafter90minutesofinjection,duetorefluxofconjugatedBSPintothecirculationfrom
thehepatocyte.
Ref:Davidson'sprinciplesandpracticeofMedicine,20thEdition,Chapter23,Page
945;Harrison'sPrinciplesofInternalMedicine,16thEdition,Page1821;Digestive
DiseasesandSciencesVol/17numbers6.

1376.Albinismisduetodeficiencyofthefollowingenzyme?
a)Phenylalaninehydroxylase
b)Homogentisicacidoxidase
c)Tyrosinase
d)Decarboxylase
CorrectAnswer-C
Themostcommoncauseofalbinismisadefectintyrosinase,theenzymemostresponsible
forthesynthesisofmelanin.Albinismisaninbornerrorduetolackofsynthesisofmelanin.
Itisanautosomalrecessivedisorderwithafrequencyof1in20,000.
Ref:TextbookofBiochemistryandHumanBiologybyG.P.Talwar,3rdEd,Page452

1377.Aminoacidmetabolismisimplicatedin
whichdisease?
a)Maplesyrupurinedisease
b)Reye'ssyndrome
c)VonGierke'sdisease
d)McArdle'sdisease
CorrectAnswer-A
Answer-A.Maplesyrupurinedisease
Itisduetodeficiencyofenzymethatcatalyzesthesecondreaction
intheseaminoacidsmetabolismi.e.branchedchain-aketoacid
dehydrogenasewhichcatalysesdecarboxylationofbranchedchain
aminoacids.

1378.A3monthsoldchildwasstartedon
supplementalfoodsalongwith
breastmilk.Thechildwasfedwithfruit
pulpandsweetenedcereals.Soonthe
childdevelopedbloatingofabdomen,
vomiting,lethargy,irritability.On
investigation,therewas
hyperbilirunemiaandelevated
transaminaselevels.Thechildis
sufferingfromwhichofthefollowing
enzymedeficiencies?

a)Fructokinase
b)AldolaseB
c)Galactokinase
d)Galactose-1-phosphateuridyltransferase
CorrectAnswer-B
Answer-B.AldolaseB
Symptomsoccurwhenfoodsorformulascontainingthesesugars
areintroducedintothediet.
Clinicalmanifestationsresemblegalactosemiaandincludejaundice,
hepatomegaly,vomiting,lethargy,irritability,andconvulsions.
Laboratoryfindingsincludeaprolongedclottingtime,hypo-
albuminemia,elevationofbilirubinandtransaminaselevels,and
proximaltubulardysfunction.


1379.EarliestsymptomofTaysachdisease
a)Exaggeratedstartleresponse
b)Bonedeformation
c)Hepatomegaly
d)Excessivebleeding
CorrectAnswer-A
Ans.is'a'i.e.,Exaggeratedstartleresponse[RefIllustrated
medicalbiochemistryp.330]
Tay-Sachdisease
Clinicalsymptomsareusuallyevidentinthefirstyearoflife
Initialsignsarenotdramaticandpresentasenfeeblement,spasticity
andslowdevelopment.
Anexaggeratedstartleresponsetosoundmaybethemost
significantearlysignofwhichaparentisaware.
Otherfeaturesarementalretardation,deteriorationofvisionand
earlydeath.
"Affectedinfantsusuallydevelopnormallyuntil4-5monthsofage
whendecreasedeyecontactandanexaggeratedstartleresponseto
noise(hyperacusis)arenoted."---Nelson.
PatientswiththeinfantileformofTay-Sachsdiseasehaveclinical
manifestationsininfancyincludinglossofmotorskills,increased
startlereaction,andmacularpallorandretinalcherry-redspots.

1380.Allaretrueaboutsacrococcygeal
teratomaexcept
a)Notassociatedwithincreasedserummarkers
b)Inmostcasesisnotvisibleexternally
c)Ifassociatedwithhydrops,shouldberesectedantenatally
d)Mostcommontumoroffetus
CorrectAnswer-B
Answer-B.Inmostcasesisnotvisibleexternally
Uretersmaybepartiallyobstructedresultinginhydro-ureterand
hydronephrosis.
SacrococcygealTeratoma(SCT)isthemostcommonneoplasmin
thefetusandnewborn
Mostcommontumorinfetusandneonatesacrococcygealteratoma
Mostcommontumorininfancyneuroblastoma.
Theyarenotassociatedwithelevatedmarkersunlessmalignancyis
present.
Sacrococcygealteratomawithhydrops:-Treatment-Inutero
resectionorcatheterdirectedvesselobliteration

1381.MostcommonsignofLRTI[Lower
respiratorytractinfection]inchildren
is

a)Chestindrawing
b)Tachypnea
c)Nasalflaring
d)Failuretofeedwell
CorrectAnswer-B
Answer-B.Tachypnea
Tachypneaisthemostconsistentmanifestationofpneumonia.
Pneumoniaisaninflammationoftheparenchymaoflungs,and
mostlycausedbybacterialorviralinfection.
Mostcommoncauseofpaediatricpneumoniaisrespiratorysyncytial
virus(RSV).Othervirusescausingpneumoniaareinfluenzavirus
(2"?mostcommonvirus),adenovirus,rhinovirus,andparainfluenza
virus.

1382.Mostcommonintraabdominalsolid
organtumorinchildis?
a)Neuroblastoma
b)Rhabdomyoblastoma
c)Wilm'stumor
d)Hypernephroma
CorrectAnswer-A
Ans.is'a'i.e.,Neuroblastoma
Mostcommonabdominalcancerofchildhood.
Mostcommoncancerofinfancy.
Mostcommonextracranialsolidtumorofchildhood(mostcommon
solidtumorofchildhoodisbraintumor).

1383.Whichofthefollowingisnotasignof
severedehydration?
a)Tachycardia
b)Anuria
c)Increasedthirst
d)Delayedcapillaryrefill[>3sec]
CorrectAnswer-C
Answer-C.Increasedthirst
Peripheralpulseseitherrapidand
weakorabsent
Decreasedbloodpressure
Nourineoutput
Verysunkeneyesandfontanel
Notears
Parchedmucousmembrane
Delayedelasticity(poorskinturgor)
Verydelayedcapillaryrefill(>3sec)
Coldandmottled
Limp
Depressedconsciousness

1384.Whatisthegradeofdehydrationifa
childdemonstratesexcessivethirstand
decreasedurineoutput?

a)Nodehydration
b)Milddehydration
c)Moderatedehydration
d)Severedehydration
CorrectAnswer-B
Answer-B.Milddehydration
Normalorincreasedpulse
Decreasedurineoutput
Thirsty
Normalphysicalfindings

1385.ContentofNa'inringerlactateismeq/1-
a)154
b)12
c)130
d)144
CorrectAnswer-C
Ans.is'c'i.e.,130
1. 5%Dextrose/10%DxNil
2. N/2saline77meq
3. N/5saline30meq
4. 3%salime513meq
5. Ringerlactate130meq
6. IsolyteP.26meq

1386.Whichofthefollowingcannotbeused
todetectHIVstatusinearlyinfancy?
a)DNA-PCR
b)HIVculture
c)ELISA
d)P-24antigenassay
CorrectAnswer-C
Answer-C.ELISA
ELISAorWesternblottestarenotasreliableinyounginfants.
Inolderinfants(>6months),detectionofanti-HIVIgAantibodiesby
ELISAisdiagnostic.
Inchildren(>18months)demonstrationofanti-HIVIgGantibodies
byELISAisused.

1387.Teratologyisastudyof
a)Congenitalheartdefect
b)Congenitalabnormalities
c)Woundsandinjuries
d)Noneoftheabove
CorrectAnswer-B
Answer-B.Congenitalabnormalities
StudyofCongenitalheartdefectsisapartofthebroadspectrumof
congenitalabnormalities,thestudyofwhichisknownasTeratology

1388.Russellsilversyndromeisassociated
withwhichofthefollowing?
a)Autosomalinheritance
b)X-linkedinheritance
c)Sporadicgenemutation
d)Uniparentaldisomy
CorrectAnswer-D
Answer-D.Uniparentaldisomy
Angelmansyndrome
PraderWillisyndrome
PseudohypoparathyroidismIb
Transientneonataldiabetesmellitus
Beckwith-Wiedemannsyndrome
Russellsilversyndrome
Wangsyndrome
Templesyndrome

1389.Whichofthefollowingisnotafeature
ofDown'ssyndrome?
a)Hypotonia
b)Infections
c)Femaleinfertility
d)EarlyonsetAlzheimer'sdisease
CorrectAnswer-C
Answer-C.Femaleinfertility
GIT:-Analatresia,Duodenalatresia,Hirschsprungdisease,annular
pancreas.
Increasedincidenceofleukemia(1%).LeukemiascommonareALL
(mostcommon),AML(M7-AML)transientmyeloproliferative
disorders,andJuvenilCML.
Others:EarlyonsetofAlzheimer'sdisease,Decreasedimmunity
withrecurrentinfections,obesity,DM,Hypothyroidism(most
commonendocrineabnormality).

1390.Whichofthefollowingisnotafeature
ofTurner'ssyndrome?
a)Cubitusvalgus
b)Cryptorchidism
c)Shortfourthmetacarpal
d)Shieldchest
CorrectAnswer-B
Answer-B.Cryptorchidism
Clinicalfeaturesinadolescentsareshortstature,webbedneck,low
posteriorhairline,widelyspacednippleswithbroadchest(shield
chest),hypertelorism,epicanthus,slantedpalpebralfissure,ptosis,
micrognathia,cubitusvalgus(increaredcarryingangle),
sensorineuralhearingloss,shortfourthmetacarpal,hypothyroidism,
streakovaries,andsexualinfantilism.Turnersyndromeisthemost
importantcauseofprimaryamenorrhea.

1391.InTurner'ssyndromewhichofthe
followingisNOTseen:
a)Shortstature
b)Widelyspacednipple
c)Webbedneck
d)Mentalretardation
CorrectAnswer-D
AnswerisD(Mentalretardation):
MentalretardationisseeninDown&Klinefelter'ssyndromebutnot
inTurner'ssyndrome.Allotherfeaturesmentionedasoptionsmay
beseeninTurner'ssyndrome.


1392.Whichofthefollowingistrueregarding
Turner'ssyndrome?
a)Cubitusvalgus
b)Autosomaldominant
c)Monosomyofchromosome12
d)Sensorineuralhearingloss
CorrectAnswer-A
Answer-A.Cubitusvalgus
Turnersyndromeisamonosomyofsexchromosome(not
autosomaldominant).CubitusvalgusisafeatureofTurner
syndrome.SNHLisnotafeature.

1393.Singlegenedisorderwhichdoesnot
followmendelianinheritance-
a)Sicklecellanemia
b)Downsyndrome
c)FragileX-syndrome
d)Retinoblastoma
CorrectAnswer-C
Ans.is'c'i.e.,FragileX-Synd.
Transmissionofcertainsingle-genedisordersdoesnotfollow
classicalmendelianprinciples.
Thisgroupofdisordercanbeclassifiedintofollowing
categories?

1. Diseasescausedbytriplet-repeatmutation,e.g.fragile-X
syndrome,Huntingtondisease,&others.
2. Disorderscausedbymutationinmitochondrialgenes,e.g.laber
heriditaryopticneuropathy.
3. Disorderassociatedwithgenomicimprinting.
4. Disordersassociatedwithgonadalmosaicism.

1394.Mostcommoncauseofshockinchild
a)Septicshock
b)Hypovolemicshock
c)Cardiogenicshock
d)Anaphylacticshock
CorrectAnswer-B
Answer-B.Hypovolemicshock
Hypovolemiaisthemostcommoncauseofshockinchildren.
The2ndmostcommoncause-Septicordistributiveshock.
3rdmostcommon-Cardiogenicshock

1395.Whichistheprognosticscoringsystem
forheadinjuryinchildren?
a)CCS
b)AUDIT
c)Injuryseverityscore
d)PediatricTraumaScore
CorrectAnswer-A
Answer-A.CCS
Table1:ChildrenComaScore(CCS)<2years"
OcularResponse
4
Pursuit
3
Extraocularmusclesintactreactivepupils
2
FixedpupilsandEOMimpaired
1
FixedpupilsandEOMparalyzed
Verbalresponse
3
Cries
2
Spontaneousrespiration
1
Apneic
Motorresponses
4
Flexesandextends
3
Withdrawsfrompainfulstimuli
2
Hypertonic
1
Flaccid
TotalMax.Score 11
TotalMinScore

1396.Whichofthefollowingisacriteriafor
clinicalStageIIofAIDSinchildren?
a)Lymphadenopathy
b)Oralcandidiasis
c)Hepatosplenomegaly
d)Oesophagealcandidiasis
CorrectAnswer-C
Answer-C.Hepatosplenomegaly
ClinicalStage2
Hepatosplenomegaly
Papularpruriticeruptions
Seborrhoeicdermatitis
Extensivehumanpapillomavirusinfection
Extensivemolluscumcontagiosum
Fungalnailinfections
Recurrentoralulcerations
Linealgingivalerythema(LGE)
Angularcheilitis
Parotidenlargement
Herpeszoster

1397.Whichofthefollowingisclosedat
birth?
a)Foramenovale
b)Posteriorfontanelle
c)Ductusvenosus
d)Anteriorfontanelle
CorrectAnswer-B
Answer-B.Posteriorfontanelle
Posteriorfontanelle
Posteriorfontanellegenerallyclosesby2-4monthsafterbirth.
Posteriorfontanelleusuallyclosesbytheageof1-4months.But
sometimeitmaybeossified(closes)atbirth.Thus,itisthebest
answeramongthegivenchoices.

1398.Harlequinskinchangeisseendueto
mutationofwhichgene?
a)ABCA12
b)FAD
c)Keratin1
d)ALOXE3
CorrectAnswer-A
Answer-A.ABCA12
Harlequinichthyosis(HI)iscausedbymutationsintheABCAl2
gene.
MutationinthegeneleadstodefectivelipidtransportandABCAl2
activityisrequiredforthegenerationoflong-chainceramidesthat
areessentialforthedevelopmentofthenormalskinbarrier.Itis
inheritedbyautosomalrecessivemodeofinheritance.

1399.Ritterdiseaseisadiseasecausedby-
a)Infection
b)Autoimmune
c)Genetic
d)Metabolicdisorder
CorrectAnswer-A
Answer-A.Infection
Staphylococcalscaldedskinsyndromeiscausedpredominantlyby
phagegroup2staphylococci,particularlystrains71and55,which
arepresentatlocalizedsitesofinfection.

1400.PinkcolorintheIMNCIchartis
suggestiveof
a)Normalzoneofweightforage
b)Undernutrition(Upto-2SD)
c)Severelyunderweightzone(Upto-3SD)
d)Veryseverelyundernourished(Upto-5SD)
CorrectAnswer-C
Answer-C.Severelyunderweightzone(Upto-3SD)
Green-Normalzoneofweightforage
Yellow-Undernutrition(upto-2SD)
Pink-Severelyunderweightzone(upto-3SD)

1401.5DHTisnecessaryfordevelopmentof
whichofthefollowing?
a)Externalgenitalia
b)Internalgenitalia
c)Mullerianstructures
d)Wollfianstructures
CorrectAnswer-A
Answer-A.Externalgenitalia
Virilizationofthewolffianductiscausedbytheactionof
testosteroneitself.
Masculinizationoftheurogenitalsinusandexternalgenitals
dependsontheactionofDHT(Dehydrotestosterone)duringthe
criticalperiodoffetalmasculinization.

1402.TrueaboutAspergersyndrome:
a)Morecommoningirl
b)Repetitiveactivitypattern
c)Subnormalintelligenceisconsistentfeature
d)Severelanguageimpairmentsischaracteristic
e)All
CorrectAnswer-B
Ans.b.Repetitiveactivitypattern
Aspergersyndrome:
Itisfourtimesmorelikelytooccurinmalesthaninfemalesand
usuallyisfirstdiagnosedinchildrenbetweentheagesof2and6.
Thecommoncharacteristicsincludeaverageoraboveaverage
intelligence"
Thereisnoclinicallysignificantgeneraldelayinspokenorreceptive
languageorcognitivedevelopment.Self'helpskills,adaptive
behaviour,andcuriosityabouttheenvironmentduringthefirst3
yearsshouldbeatalevelconsistentwithnormalintellectual
development

1403.Definitionofchildhoodisunderwhat
age?
a)8years
b)10years
c)12years
d)16years
CorrectAnswer-C
12yearsREF:NelsonTextbookofPaediatrics17theditiondifferent
pages
Infancy
0-1yr
Toddlerhood
1-3yr
Earlychildhood(toddlerhoodandsome
1-4yr
timeafterwards)
MiddleChildhood(SchoolAge)
6-12yr
Preschool
2-5yr
Adolescence(onsetofpubertyto
12-20yr
maturity)
Adulthood(fullphysicalandintellectual
20-21yr
maturity)
onwards

1404.Prevalenceofomphaloceleatbirthis
a)1in100livebirths
b)1in2000livebirths
c)1in4000livebirths
d)1in10,000livebirths
CorrectAnswer-C
Answer-C.1in4000livebirths
Incidenceofomphaloceleat11-14weeksgestation-1in1100
pregnancies.
Prevalenceatbirth-1in4000-6000livebirths.
Thisindicatessuddenmortalitymostlikelyduetoinuterofetal
demisefromassociatedchromosomalanomaliesaswellaselective
terminationafterthediagnosis

1405.Pectusexcavatumis
a)Protrusionofsternum
b)Sternaldepression
c)Sternalcleft
d)Lateraldisplacementofsternum
CorrectAnswer-B
Answer-B.Sternaldepression
Pectusexcavatum(funnelchest)ismidlinenarrowingofthoracic
cavityduetosternaldepression.
Mayoccurinisolationormaybeassociatedwithaconnectivetissue
disorder,MarfanorEhlers-Danlossyndrome.Secondarytochronic
lungdisease,neuromusculardisease,ortrauma.

1406.Achildaged7yearshashowmany
teeth
a)15
b)20
c)26
d)32
CorrectAnswer-C
Answer-C.26
Permanentteeththatappear:
1stmolars:-4
Centralincisors:-2
Temporaryteeth:-20(sincemolarsaresuperaddedpermanent
teethandcentralincisorsarereplaced).
Soinall26teeth(Range24to26)-at7yearsofage.

1407.Whatistheageofthechildwhodraws
acircleandbuildstowerof7cubes?
a)1year
b)2years
c)21/2years
d)3years
CorrectAnswer-D
Answer-D.3years
Age
Milestone
Age
Numberof
cubesoftower

12-24
Triestoscribble
12months
2
spontaneously
3


months
2years
Drawsaverticalor
15months
4
horizontalline
3years
Drawsacircle
18months
6
4years
Drawsacross(plus
21months
7
sign)anddrawsa
24months
rectangle
5years
Drawsatriangle
30months
9


36months
10

1408.Bywhatageisthemilestoneof
climbingstepswithalternatefeet
achieved?

a)2years
b)3years
c)4years
d)5years
CorrectAnswer-B
Answer-B.3years
Walkindependently-1year
unwell,climbingupstairsandgoingdownstairswithonestepata
time-2years
Ridetricycle;climbingupstairswithalternatefeet-3years
Hopping;goingdownstairswithalternatefeet-4years
Skipping-5years

1409.Maturefingergripcomesatwhatage?
a)5months
b)7months
c)9months
d)1year
CorrectAnswer-C
Answer-C.9months
6monthsDropsoneobjectwhenanotherisoffered
7monthsTransfersobject&unidextrousapproach
9months-pPincergrasp
12--13monthsCastingappear,mouthingdisappear
15monthsFeedshimselfwithcup,slightspillage

1410.Socialsmileisattainedatwhatage?
a)2months
b)5months
c)9months
d)1year
CorrectAnswer-A
Answer-A.2months
Socialsmiledevelopsat2months.

1411.Arrangethefollowingmilestonesinthe
correctorderoftheirattainment
I.Buildtowerof4cubes
II.Makesimplesentences
III.Drawingacircle
IV.Drawingarectangle

a)IIIIIIVI
b)IIIIIIIV
c)IIIIIIIV
d)IIIIVIII
CorrectAnswer-B
Answer-B.IIIIIIIV
Theageofattainmentofthemilestonesis:-
Buildsatowerof4cubes18months
Makessimplesentences2years
Drawingacircle3years
Drawingarectangle4years

1412.Bywhatagecananewbornrecognize
mother?
a)2months
b)3months
c)6months
d)7months
CorrectAnswer-B
Answer-B.3months
Headcontrol
Startscooing
Recognizesmother
Canfollowanobjectupto180?
Onpullingthechildtosit,headlagspartially(between2-3months).
After3monthsheadcontroldevelops.

1413.Whichofthefollowingcanbedoneby
an18monthsoldbaby?
a)Makingtowerof9cubes
b)Canuse10wordswithmeaning
c)Ridetricycle
d)Turnpagesofbookoneatatime
CorrectAnswer-B
Answer-B.Canuse10wordswithmeaning
At18months,thechildcanuse10wordswithmeaning.

1414.Vocabularyof1.5yearoldchildis-
a)1-10words
b)10-20words
c)20-30words
d)30-40words
CorrectAnswer-B
Ans.is'b'i.e.,10-20words
At18months,thechildcanuse10wordswithmeaning.
Othermilestonesaskedinthequestionareachievedinchildren
olderthan18months:?
Makingatowerof9cubes-30months
Turnpagesofbookoneatatime-2years
Ridetricycle-3years

1415.Achildof5yearscanusesentencesof
aroundhowmanywords?
a)6words
b)10words
c)100words
d)250words
CorrectAnswer-B
Answer-B.10words
Althoughachildhasavocabularyof250wordsat3years,thechild
canusesentenceof10wordsat5years.

1416.Milestonesachievedbya10monthsold
childareallexcept-
a)Pincergrasp
b)Wavingbye-bye
c)Standingwithoutsupport
d)Playsapeek-a-boogame
CorrectAnswer-C
Answer-C.Standingwithoutsupport
Babystandswithoutsupportby1yearofage.
Pincergrasp9months
Wavingbye-bye9months
Playsapeek-a-boogame10months

1417.Whatistheageofachildwhocan
identify4coloursanddrawatriangle?
a)21/2years
b)3years
c)4years
d)5years
CorrectAnswer-D
Answer-D.5years
Achildlearnstodrawatriangleattheageof5years.
Identificationoffourcoloursisattainedattheageof4years.

1418.Weightofaninfantdoublesbywhat
age?
a)6months
b)1year
c)2years
d)3years
CorrectAnswer-C
Answer-C.2years
Weightofaninfantdoublesby5monthsandquadruplesby2years
ofage.

1419.Growthofheadcircumferencein1st3
monthsoflifeisby
a)2cm
b)3cm
c)5cm
d)10cm
CorrectAnswer-C
Answer-C.5cm
Headcircumferenceismeasuredfromtheoccipitalprotuberanceto
thesupraorbitalridgeofforeheadwhichisthemaximumoccipito
frontaldiameterofskull.Theheadcircumferenceinuterogrowsby
0.5cminfirst2weeks,0.75cmin3rdweekandafterthat1
cm/weektillbirth.

1420.Delayederuptionisfailureofteethto
appearby
a)6months
b)13months
c)25months
d)37months
CorrectAnswer-B
Answer-B.13months
Delayederuptionisusuallyconsideredwhentherearenoteethby
approximately13monthsofage(mean+3SD).
Commoncausesofdelayederuptionofteethinclude:-
Idiopathic(Mostcommon).
Hypothyroid
Hypoparathyroid
Familial

1421.Breastmilkprotectsfrominfectionsas
itcontainsallofthefollowingexcept:
a)IgE
b)Lactoferrin
c)Bifidusfactor
d)PABA
CorrectAnswer-A
Answer-A.IgE
Breastmilkcontainsseveralantiinfectivefactors

1. Antibodies-3secretoryIgA,IgM
2. Lysozyme
3. Antistaphylococcalfactor
4. Specificinhibitorysubstancesagainstviralinfections.
5. LactoferrinInhibitsgrowthofE.coli.
6. BilestimulatedlipasekillsentamoebahistolyticaandGiardia
lamblia.
7. BifidusfactorInhibitsgrowthofE.coli
8. Para-amino-benzoicacid(PABA)Providesprotectionagainst
malariaix)Phagocyticmacrophagesandlymphoidcells

1422.Whichofthefollowingistrueregarding
prematuremilkascomparedtomature
milk?

a)Lesslactose
b)Lessiron
c)Lessimmunoglobulins
d)Lesssodium
CorrectAnswer-A
Answer-A.Lesslactose
Pretermmilk
Themilkofmotherwhodeliversprematurelydiffersfromthemilkof
amotherwhodeliversatterm.
Pretermmilkcontains:Lesslactose(incomparisontotermmilk).
ContainsmoreproteinS,sodium,iron,immunoglobinsandcalories
astheyareneededbythepretermbaby.

1423.Allofthefollowingaretrueregarding
breastmilkascomparedtocow'smilk
except

a)Containsmorelactose
b)Moreamountofproteins
c)Lessamountoffatcontent
d)Mineralsandsaltsisless
CorrectAnswer-B
Answer-B.Moreamountofproteins
Incomparisontocowmilk,humanmilkcontainslessamountof:
Proteins(1gm/100ml),salts(sodium,chloride,potassium),fat(3.4
gm/100ml),andminerals(calcium,phosphate)&more:Lactose
(7g/100mlor7%).

1424.Whichofthefollowingisthebest
referenceforgrowthmonitoringin
children?

a)ICMR
b)NCHS
c)Boston
d)IAP
CorrectAnswer-B
Answer-B.NCHS
WHOreferencevalues(NCHSstandards)
Thesearemostcommonlyusedandbestavailablereferencevalues
forinternationaluse.
ThesevaluesarebasedonthedataassembledbyUnitedStates
NationalCentreforHealthStatistics(NCHS).
ClassificationofPEMisbasedonthesestandards

1425.Kwashiorkarisduetodeficiencyof
a)Calories
b)Minerals
c)Vitamins
d)Zinc
CorrectAnswer-A
Answer-A.Calories
Marasmusandkwashiorkorareduetodeficiencyofproteinsand
calories.
Itischaracterizedbyclassical'triad'ofedema(Dueto
hypoalbuminemia),markedlyretardedgrowthandpsychomotor
(mental)changes.

1426.

Whichofthefollowingisthemost
metabolicallyactivepartoflongbone?

a)Epiphysis
b)Metaphysis
c)Diaphysis
d)Physis
CorrectAnswer-D
Answer-D.Physis
Thegrowthplate(physis)andtheadjacentterminaldiaphysis
representthemostmetabolicallyactivesegmentofthelongbone.
Thispartchangesdramaticallyduringdevelopmentandhenceitis
calledthemetaphysis

1427.Iliotibialbandcontractureinpatientsof
poliomyelitiswillleadto
a)Flexionathipandknee
b)Flexionathip,extentionatknee
c)Extentionathipflexionatknee
d)Extentionathipandknee
CorrectAnswer-A
Answer-A.Flexionathipandknee
Deformities:Iliotibialbandcontracturecanleadto:
Flexion,abductionandexternalrotationdeformityathip(most
common).
Flexonandvalgusatlcneeorsometimestripledeformityatknee
(flexion,posteriorsubluxationandexternalroutationoftibiaon
femur).
Equinovarusatankleandfoot.
Lumbarscoliosisandpelvicobliquityatspineandpelvis
respectively.

1428.Jumpersknee
a)Apophysitisofpatellartendonasitinsertsinpatella
b)Apophysitisofpatellartendonasitinsertsintibia
c)Apophysitisofquadricepstendonasitinsertsinpatella
d)Apophysitisofhamstringtendonasitinsertsintibia
CorrectAnswer-A
Answer-A.Apophysitisofpatellartendonasitinsertsinpatella
ItisalsocalledPatellartendinitis
Thisisanapophysitis(inflammation)ofthepatellartendonasit
insertsintothepatella.
Itisassociatedwithpain,swellingandcrepitus.

1429.Whichofthefollowingisnottrueabout
Galeazzifracturedislocation?
a)Fractureofdistalthirdofradiusanddislocationofdistalradio-
ulnarjoint
b)Resultsfromfallonoutstretchedhand
c)Thedistalendofulnadislocatesvolarlyafterdisruptionofdistal
radio-ulnarjoint
d)Radiusisangulatedmediallyandanteriorly
CorrectAnswer-C
Answer-C.Thedistalendofulnadislocatesvolarlyafter
disruptionofdistalradio-ulnarjoint
Fractureofthedistalthirdoftheradiuswithdislocationofthedistal
radioulnarjoint
ThisinjuryisthecounterpartoftheMonteggiafracture-dislocation.
Itcommonlyresultsfromafallonanoutstretchedhand.
Theradiusfractureisangulatedmediallyandanteriorly.Thedistal
radioulnarjointisdisrupted,resultingindorsaldislocationofthe
distalendoftheulna
Malunionoccursbecauseofdisplacementofthefragment.Itresults
indeformityandlimitationofsupinationandpronation.

1430.InMonteggiafracture,whichistrue
aboutulnarfractureandheadofradius
a)Bothulnarfractureandheadofradiusisdisplacedposteriorly
b)Bothulnarfractureandheadofradiusisdisplacedanteriorly
c)Ulnarfracturesisposteriorlyandheadofradiusisdisplaced
anteriorly
d)Ulnarfractureisanteriorlyandheadofradiusisdisplaced
posteriorly
CorrectAnswer-B
Ans:Bi.e.Bothulnarfractureandheadofradiusisdisplaced
anteriorly
*Thisisafractureoftheupper-thirdoftheulnawithdislocationof
theheadoftheradius.Itiscausedbyafallonanoutstretchedhand.
Itmayalsoresultfromadirectblowonthebackoftheupper
forearm.
-Thesefallintotwomaincategoriesdependinguponthe
angulationoftheulnafracture-extensionandflexiontype.The
extensiontype,isthecommonerofthetwo,wheretheulnafracture
angulatesanteriorly(extends)andtheradialheaddislocates
anteriorly.Theflexiontypeiswheretheulnafractureangulates
posteriorly(flexes)andtheradialheaddislocatesposteriorly.

1431.Whichofthefollowingisnottrueabout
Jeffersonsfracture?
a)Itisaburstfractureoftheringofatlasvertebra
b)Itisthemostcommontypeofatlasfracture
c)FracturedefinintionisparticularlyclearonCTScanimage
d)Itisassociatedwithinjuryelsewhereinspinein25%ofthe
cases
CorrectAnswer-D
Answer-D.Itisassociatedwithinjuryelsewhereinspinein
25%ofthecases
Suddensevereloadonthetopoftheheadmaycausea'bursting'
forcewhichfracturestheringoftheatlas(Jefferson'sfracture).Thus
itisatypeofaxialcompressionforce.
ItisthemostcommontypeoffractureoftheAtlas.
Thereisnoencroachmentontheneuralcanalandusuallyno
neurologicaldamage.
Withtheexceptionofpainorlossofsensationinthegreater
occipitalnervedistribution,neurologicalsequelaeareuncommon
andmorelikelytoberelatedtoassociatedinjuries.
Fractureofatlasareassociatedwithinjuryelsewhereincervical
spineinupto50%ofcases;odontoidfracturesandhangman's
fractureinparticularshouldbeexcluded

1432.Whichofthefollowingshoulder
movementsis/areweakinpatientsof
supraspinatustear?

a)Abduction
b)Adduction
c)Externalrotation
d)Internalrotation
CorrectAnswer-A
Answer-A.Abduction
Patientsmaytearthesupraspinatustendonacutelybyfallingonan
outstretchedarmorliftingaheavyobject.
Symptomsarepainalongwithweaknessofabductionandexternal
rotationoftheshoulder.
Atrophyofthesupraspinatusmusclesdevelops.

1433.Whichofthefollowingisnottrueabout
impingementsyndrome?
a)Itisthetendinitiscausedbyinflammationoftherotatorcuff
tendons
b)Supraspinatustendonismostofteninvolved
c)Shoulderabductioninthearcof60-120degreesisparticularly
painful
d)Surgicaldecompressionofthesubacromialspaceisfrequently
indicated
CorrectAnswer-D
Answer-D.Surgicaldecompressionofthesubacromialspace
isfrequentlyindicated
Impingementsyndrome(painfularesyndrome)ismostcommonly
duetendinitsofsupraspinatuscomponentofrotatorcuffandis
characterizedbypainin60?-120"ofabduction.Treatmentisinitialy
conservative.Surgicaldecompressionisrequiredinfailedcases.

1434.VascularsignofNarathisnoticedin
a)Fractureneckoffemur
b)Perthesdisease
c)Posteriordislocationofhip
d)Alloftheabove
CorrectAnswer-C
Ci.e.Posteriordislocationofhip
Vascularsignofnarathispositiveinposteriordislocationofhipjoint.
Duetoposteriordislocation,thehipjointfallsonthefemoralartery,
andthiscausesfeebleorabsentfemoralpulse.

1435.Mostdangeroustypeofodontoid
fractureasperAndersonandD'Alonzo
classificationanditsrespective
managementis

a)TypeI-immobilizationinrigidcollar
b)TypeII-screwfixation
c)TypeIII-halovestimmmobilization
d)TypeIV-opereductioninternalfixation
CorrectAnswer-B
Answer-B.TypeII-screwfixation
OdontoidfractureshavebeenclassifiedbyAndersonand
D'Alonzo(1974)asfollows:

1. TypeI-Anavulsionfractureofthetipoftheodontoidprocessdueto
tractionbythealarligaments.Thefractureisstable(abovethe
transverseligament)anduniteswithoutdifficulty.
2. TypeII-Afractureatthejunctionoftheodontoidprocessandthe
bodyoftheaxis.Thisisthemostcommon(andpotentiallythemost
dangerous)type.Thefractureisunstableandpronetonon-union.It
requiresfixationbyscrew.
3. TypeIII-Afracturethroughthebodyoftheaxis.Thefractureis
stableandalmostalwaysuniteswithimmobilization.

1436.IncreasedQanglepredisposesto
a)Medialpatellarsubluxation
b)Lateralpatellarsubluxation
c)Superiorpatellarsubluxation
d)Inferiorpatellarsubluxation
CorrectAnswer-B
Answer-B.Lateralpatellarsubluxation
PatellaralignmentcanbeassessedbymeasuringtheQ-angle
(quadricepsangle).
Thisistheanglesubtendedbyalinedrawnfromtheanterior
superioriliacspinetothecentreofthepatellaandanotherfromthe
centreofthepatellatothetibialtubercle.
Itnormallyaveragesabout14degreesinmenand17degreesin
women.
Patellofemoralstabilityismaintainedbyacombinationofthe
articularsurfacegeometryandsofttissuerestraints.

1437.Seatbeltinjuryis
a)Teardropfracture
b)Wedgefracture
c)Chancefracture
d)Whiplashinjury
CorrectAnswer-C
Ci.e.Chancefracture
Ahorizontalfractureofthevertebraextendingfrombodytothe
posteriorelement,causedbyastrongdistractionforce

1438.Palpablefemurheadonperrectalexam
isafeatureofwhichofthefollowing
conditions?

a)Posteriorhipdislocation
b)Anteriorhipdislocation
c)Centralhipdislocation
d)Inferiorhipdislocation
CorrectAnswer-C
Answer-C.Centralhipdislocation
Incentralfracture-dislocationofthehipthefemoralheadisdriven
throughthefloor(medialwall)oftheacetabulumtowardsthepelvic
cavity.
Itoccursduetofallontheside,orablowoverthegreatertrochater.

1439.Themostcommoncomplicationof
intracapsularfractureneckoffemuris
a)Malunion
b)Osteoarthritis
c)Non-Union
d)Shortening
CorrectAnswer-C
Ci.e.Nonunion
Bothintracapsularneckfractureandextracapsularfracture
(basicervicalandITfracture)havesamedeformities,i.e.external
rotationandshortening.
Thesedisplacementsaremoremarkedinextracapsularfractures,
becauseinanintracapsularfracture,thecapsuleofthehipjointis
attachedtothedistalfragmentandpreventsextremerotationand
displacementofthedistalfragment(andwithit,thelimb).
Inextracapsularfracture,distalfragmentbeingoutsidethecapsule
isdisplacedmoremarkedly.

1440.Poplitealarteryinjuryiscommonly
seeninwhichtypeoftraumaticknee
dislocation?

a)Anterior
b)Posterior
c)Medial
d)Lateral
CorrectAnswer-B
Answer-B.Posterior
Poplitealarteryinjuryiscommonwithbothanteriorandposterior
dislocations.
Posteriordislocationsmorelikelytoresultindirectinjuryandeven
ruptureofpoplitealartery(isolatedtransection).
Anteriordislocationscausestretchingofpoplitealarterywhichmay
leadtointimaldisruptionandthrombosis(damageisoveralonger
segmentofartery).

1441.Kochermanoeuverisusedfor
a)Shoulderreduction
b)Elbowreduction
c)Ankledislocation
d)Kneedislocation
CorrectAnswer-A
Answer-A.Shoulderreduction
Kocher'smanoeuvre:
Thisisthemostcommonlyusedmethod.
Thestepsareasfollows:
1. Traction--withtheelbowflexedtoarightanglesteadytractionis
appliedalongthelongaxisofthehumerus;
2. Externalrotation--thearmisrotatedexternally;
3. Adduction--theexternallyrotatedarmisadductedbycarryingthe
elbowacrossthebodytowardsthemidline;and
4. Internalrotation?thearmisrotatedinternallysothatthehandfalls
acrosstotheoppositeshoulder.

1442.Anklesprainduetoforcedinversionof
aplantarflexedfootisduetoinjuryto
a)Anteriortalofibularligament
b)Posteriortalofibularligament
c)Calcaneofibularligament
d)Posteriorfibresofdeltoid
CorrectAnswer-A
Ans.a.Anteriortalofibularligament
*Structuresdamagedduetoinversioninjury.
-Peronealtendoninjury.
-Avulsionfractureoftipoflateralmalleolus.
-Avulsionfractureofanterolateralsurfaceoftalus&calcaneum
(sustentaculumtali).
-Fractureofbaseof5thmetatarsal.
-Lateralcollateralligamentinjury(anteriortalofibular
>calcaneofibular>posterior-talofibularligament).
-Medialmalleolusfracture.

1443.Treatmentofscaphoidfracture
a)Conservative
b)CompressionScrews
c)CompressionPlating
d)Traction
CorrectAnswer-A
Answer-A.Conservative
Thetreatmentofascaphoidisessentiallyconservative.
Thehandisimmobilizedinascaphoidcastwithwristinlittle
dorsiflexionandradialdeviation(glassholdingposition).

1444.Allofthefollowingaretrueregarding
fractureoflateralcondyleofhumerus
except

a)Usuallyseenat6-10yearsofage
b)ResultsinGunstockdeformity
c)Cubitusvalgusoccurs
d)Tardyulnarnervepalsyisseen
CorrectAnswer-B
Answer-B.ResultsinGunstockdeformity
FractureLateralCondyleHumerus/Jupiterfracture
Itisacommonfractureinchildren.Thelateralcondylar(orcapitular)
epiphysisbeginstoossifyduringthefirstyearoflifeandfuseswith
shaftat12-16years.Betweentheseagesitmaybeshearedoffor
avulsedbyforcefultraction.Themaximumchancesofinjuryis
between6-10years.

1445.Mostcommoncomplicationoffracture
oftibia
a)Infection
b)Compartmentsyndrome
c)Delayedunion
d)Vascularinjury
CorrectAnswer-C
Answer-C.Delayedunion
Thetibiahassomecharacteristicfeatureswhichareresponsiblefor
delayedunionornon-unionoftibiafractures.
Thedistalthirdoftibiaisparticularlypronefordelayedunionand
nonunionbecauseofitsprecariousbloodsupply.

1446.Whichofthefollowingisnottrueabout
ACLinjury?
a)Itisacomponentofthe0'Donoghuetriad
b)ACLisintrasynovial
c)ACLisimportantforproprioceptivefunction
d)Anteriordrawertestisthemostsensitivetest
CorrectAnswer-D
Answer-D.Anteriordrawertestisthemostsensitivetest
Lachman'stestisthemostsensitivetestforanteriorcruciate
ligamenttears.Itisdonewiththekneeflexedat20degrees.Soit
canbedoneinacuteaswellaschronicinjuries.(becauseinacute
caseswithhemarthrosismoreflexionisusuallynotpossibleso
performinganteriordrawertestisdifficult).
ACLisintrasynovial&hasproprioceptivefunction.

1447.Continuousfixedtractionisprovided
by-
a)Thomassplint
b)BBsplint
c)HamiltonRussel
d)Gallows
CorrectAnswer-A
Answer-A.Thomassplint
Combinedtraction-IfaThomas'splintisused,thetapesaretiedto
theendofthesplintandtheentiresplintisthensuspended,asin
balancedtraction

1448.Cobraheadplateisusedfor
a)Hiparthrodesis
b)Kneearthrodesis
c)Elbowarthrodesis
d)Anklearthrodesis
CorrectAnswer-A
Answer-A.Hiparthrodesis
Specialimplants
SPnail-plants
Dynamichipscrew(DHS)-Intertrochantericfracture
Condylarblade-plate-Interochantericfracture
T-plate-Condylarfractureoffemur
Spoonplatetibia-Condyarfractureoftibia
Cobraplate-FractureofloweraendofHiparthrodesis

1449.Lockingcompressionplatingforis
commonlyindicatedinthefollowing
fracturetypes

a)Periarticularfractures
b)Transverseorobliquefracturesoflongbones
c)Intertrochantericfractures
d)Fractureoflongbones
CorrectAnswer-A
Answer-A.Periarticularfractures
Lockingcompressionplatesarecommonlyusedinperiarticular
fractures

1450.Dunloptractionisatypeoftraction
usedinmanagementof
a)Fracturehumerus
b)Fractureradius
c)Fracturefemur
d)FractureTibia
CorrectAnswer-A
Answer-A.Fracturehumerus
Itisusedinmanagementoffracturehumerus.
Itisaskintractionappliedtothearmwiththechildsupine.
Tractionstrapsareappliedtotheforearmwiththearmsupinated.
Acounterweightishungfromtheupperpartofthearmtohelppull
theproximalfragmentofthehumerusposteriorly,toapproximatethe
distalfragment.
Longitudinaltractionisthenappliedtothesupinatedforearmwith
theelbowflexedtoabout45degrees

1451.Anklereflexisaffectedinprolapsed
intervertebraldiscatwhatlevel?
a)L3-L4
b)L4-L5
c)L5-S1
d)S1-S2
CorrectAnswer-C
Answer-C.L5-S1
S1root-Weaknessofplantarflexorsofroot-Overlateralsideoffoot

1452.CTEVshoetrueis
a)Itisthesameasnormalshoe
b)Ithasstraightmedialborder
c)Ithasmedicalshoeraise
d)Ithasheelwithextralength
CorrectAnswer-B
Answer-B.Ithasstraightmedialborder
Ithasstraightinner(medial)borderwhichhelpspreventforefoot
adduction.
Ithasoutershoeraisewhichhelpspreventforefootinversion.
Thereisnoheelwhichhelpspreventequinus.

1453.Allarefeaturesofinflammatoryarthritis
except?
a)Morningstiffness
b)X-rayshowingsclerosis
c)ElevatedESR
d)Weightgain
CorrectAnswer-B
Answer-B.X-rayshowingsclerosis
X-rayfeatureofinflammatoryarthritisshowsrarefactionwhilex-ray
featuresinnon-inflammatoryarthritisrevealssclerosis.

1454.Whichofthefollowingisnottrueabout
thetestsforhipinstabilityinneonates?
a)Ortolanistesthastwoparts
b)Theyareperformedat2-3daysofbirth
c)Inortolanistestexaminersfingersrestonthegreatertrochanter
d)Inbarlowstestexaminersthumbisplacedinthegroin
CorrectAnswer-A
Answer-A.Ortolanistesthastwoparts
InOrtolani'stest,thebaby'sthighsareheldwiththethumbsmedially
andthefingersrestingonthegreatertrochanters;thehipsareflexed
to90degreesandgentlyabducted.Normallythereissmooth
abductiontoalmost90degrees.
Incongenitaldislocationthemovementisusuallyimpeded,butif
pressureisappliedtothegreatertrochanterthereisasoft'clunk'as
thedislocationreduces,andthenthehipabductsfully(the'jerkof
entry').
Ifabductionstopshalfwayandthereisnojerkofentry,theremaybe
anirreducibledislocation.

1455.Ortolanitestispositivewhenthe
examinerhearsthe?
a)Clunkofentryonabductionandflexionofhip
b)Clunkofentryonextensionandadductionofhip
c)Clickofexitonabductionandflexionofhip
d)Clickofexitonextensionandadductionofhip
CorrectAnswer-A
Ans.is'a'i.e.,Clunkofentryonabductionandflexionofhip
ClinicaltestsforCDH/DDH
Ininfancytwotestsareused.
Barlow'stest
Thistestisdonewithin2-3daysofbirth.
Thetesthastwoparts:?
1. Part1:-Infantisinsupinepositionwithhipandkneein90?of
flexion,Thehipisslowlyadducted&pushedtodislocatethehipand
onecanhearaclunckofexitoffemoralheadoutoftheacetabulum.
2. Part2:-Nowthehipisgentelyabductedandpulledtoreducethe
hip.Thiswillcause'clunk'indicatingreductionofhip.
Itisquiteobviousthatpart1canbedoneonlydislocatablehip;but
notinalreadydislocatedhipastheheadisalreadyoutofthe
acetabulum.
Ortolani'stest
Thistestissimilarto2ndpartofBarlow'stest,i.e.slowabductionof
hipinflexedpositionofhip&kneetoreducethehip.

1456.Oncogenicosteomalaciaismediatedby
a)Phosphatonin
b)Calcitonin
c)Interleukin2
d)Interleukin6
CorrectAnswer-A
Answer-A.Phosphatonin
Oncogenicosteomalaciaismediatedbyphosphatoninincertain
tumors,particularlyvasculartumourslikehemangiopericytomasand
alsofibrohistocyticlesions

1457.Whichofthefollowingistrueabout
CTEV
a)Itismorecommoninfemales
b)Rightfootisusuallymoreaffectedthantheleft
c)Talusisdisplacedmedialandplantarwards
d)Tibiausuallyshowslateraltorsion
CorrectAnswer-C
Answer-C.Talusisdisplacedmedialandplantarwards
CTEVisthecommonestandmostimportantcongenitaldeformityof
thefoot.CTEVismorecommonmalesinthaninfemales(malesto
femaleratio2.5:1).InhalfofthecasesCTEVisbilateral.Rightand
leftfootareaffectedequally.

1458.Thelastdeformitytobecorrectedby
Ponseti'smethodforCTEVis-
a)HeelVarus
b)Equinus
c)FootAdduction
d)Cavus
CorrectAnswer-B
Answer-B.Equinus
Ponseti'stechnique
Thisinvolvesfirstcorrectingthecavusdeformirythenthe
adductionandheelvarusandfinallytheequinusdeformity.
ThistechniqueisnowmostlyacceptedtechniqueforCTEV
correctionasitisbasedonbetterunderstandingofthe
pathoanatomyofthedeformedfoot.
Thesuccessofreductionis90-98Percent.

1459.Whichofthefollowingisnottrueabout
themanipulationmethodstocorrect
CTEV?

a)Involvesserialcastingandbelowkneeplastercasting
b)Inkitesmethoddeformitiesarecorrectedsequentially
adductioninversionequinus
c)Ponseti'stechniquehassuccessrateof90-98%
d)Ponseti'smethodofcorrectioninvolvescavus-->adduction-->
heelvarus-->equinus
CorrectAnswer-A
Answer-A.Involvesserialcastingandbelowkneeplaster
casting
Serialmanipulationandabovekneeplastercastingisdoneweekly
forthefirst6weeks.
Otheroptionsarecorrect.

1460.Whichofthefollowingisthe
managementforneglectedcaseof
CTEVinapatient>10yearsofage?

a)Triplearthrodesis
b)Anklearthrodesis
c)Jessfixation
d)Ponseticasting
CorrectAnswer-A
Answer-A.Triplearthrodesis
*Alltheserequiresurgicalcorrectionandthesurgerydependsupon
theageofthepatient.
-<4yearsofage:-Postero-medialsofttissuerelease
->4years:-Postero-medialsofttissuereleasewithabony
procedure:-
*4-8years:-Dilwyn-Evansprocedure(PMR+Calcaneo-cuboid
fusion)
*8-10years:-PMR+Wedgetarsectomy
*>10years:-Triplearthrodesis(subtalar,calcaneo-cuboid,and
talonavicularjoints)

1461.Osteoporosisisseeninallthefollowing
except
a)Thyrotoxicosis
b)Rheumatoidarthritis
c)Hypoparathyroidism
d)Steroidtherapy
CorrectAnswer-C
AnswerisC(Hypoparathyroidism):
Osteoporosisasassociatedwithlzyperparathyroidism(not
hypoparathyroidism).
DISEASESASSOCIATEDWITHANINCREASEDRISKOF
GENERALIZEDOSTEOPOROSISINADULTS:

Nutritionalandgastrointestinal
HypogonadalStates
disorders
TurnerSyndrome
Malnutrition
Klinefeltersyndrome
Parenteralnutrition
Anorexianervosa
Malabsorptionsyndromes
Hypothalmicamenorrhea
Gastrectomy
Otherprimaryorsecondary
Severeliverdisease,especially
hypogonadalstates
biliarycirrhosis
Perniciousanemia
Hematologicdisorders/
Endocrinedisorders
Malignancy
Cushing'ssyndrome
Multipledisorders/malignancy
Hyperparathyroidism
Lymphomaandleukemia
Malignancy-associatedparathyroid
Thyrotoxicosis
hormone-related(PTHrP)
Insulin-dependentdiabetes

Insulin-dependentdiabetes
production
mellitus
Acromegaly
Mastocytosis
Adrenalinsufficiency
Hemophilia
Thalassemia
Selectedinherited&
Miscellaneous
Rheumatologicdisorders
OsteogenesisimperfectaQ
ImmobilizationQ
Chronicobstructivepulmonary
MarfansyndromeQ
disease
Hemochromatosis
Pregnancyandlactation
HypophosphatasiaQ
Scoliosis
Glycogenstoragediseases
Multiplesclerosis
HomocystinuriaQ
Sarcoidosis
Ehlers-Danlossyndrome
Amyloidosis
Porphyria
Alcoholism
Menkes'syndrome
Epidermolysisbullosa
Rheumatoidarthritis
DRUGSASSOCIATEDWITHANINCREASEDRISKOF
GENERALIZEDOSTEOPOROSISINADULTS:

Glucocorticoids2
Excessivethyroxin
CyclosporineQ
Aluminium
Cytotoxicdrugs
Gonadotropin-releasing
Anticonvulsants
hormoneagonistsQ
Excessivealcohol
Heparin
Lithium

1462.Whichofthefollowingisnottrueabout
SCFE?
a)Malesareaffectedmorefrequently
b)Extensionisrestricted
c)Commonlyoccursduringadolescence
d)Varus,adductionandexternalrotationdeformitiesarepresent
CorrectAnswer-B
Answer-B.Extensionisrestricted
SCFEistheseparationofproximalfemoralcapitalepiphysis(head)
atgrowthplate.
SCFEoccursduetoweaknessofgrowthplateandoccursduring
adolescentperiod(11-15years)whenthegrowthplateisweakdue
toacceleratedgrowth.
Malesareaffectedmorefrequentlythanfemales.
Movementsarerestrictedparticularlyabductionandinternalrotation.
Flexionisalsorestrictedandextensionisincreased.Andthisclinical
featuredifferentiatesSCFEwithallotherhippathologiesbecause
mostofthehippathologieshaveoppositedeformity,i.e.flexion
deformitywithrestrictedextension.
Varus,adductionandexternalrotationdeformitiesarepresent.
Externalrotationisincreasedalongwithextension.
Whenhipisflexeditgoesintoexternalrotation,i.e.obligatory
externalrotation.ItisaveryimportantsignofSCFE.
Waddlinggaitispresent.
Trendelenburg'stestispositive.

1463.Genurecurvatumisseenin-
a)Rheumatoidarthritis
b)Poliomyelitis
c)Rickets
d)Alltheabove
CorrectAnswer-D
Answer-D.Alltheabove
a)Congenitalrecurvatum
Thismaybeduetoabnormalintra-uterineposture;itusually
recoversspontaneously.Rarely,grosshyperextensionisthe
precursoroftruecongenitaldislocationoftheknee.
b)Laxligaments
Normalpeoplewithgeneralizedjointlaxitytendtostandwiththeir
kneesbackset.Prolongedtraction,especiallyonaframe,orholding
thekneehyperextendedinplaster,mayoverstretchligaments,
leadingtopermanenthyperextensiondeformity.Ligamentsmayalso
becomeoverstretchedfollowingchronicorrecurrentsynovitis
(especiallyinrheumatoidarthritis),thehypotoniaofrickets,the
flailnessofpoliomyelitisortheinsensitivityofCharcot'sdisease.

1464.Psoriaticarthritismostcommonly
involveswhichjoint-
a)Distalinterphalangealjoint
b)Proximalinterphalangealjoint
c)Wristjoint
d)Metacarpophalangealjoint
CorrectAnswer-A
Answer-A.Distalinterphalangealjoint
Theseinclude:

1. Arthritisofdistalinterphalangeal(DIP)joints
2. Assymetricaloligoarthritis:Mostcommonpattern
3. SymmetricalpolyarthritissimilartoRA
4. Axialinvolvement(sacroiliacandspine)similartoankylosing
spondylitis
5. Arthritismultilans

1465.Whichofthefollowingis/arefeature/s
ofsprengelsdeformity?
a)Elevatedshoulderonaffectedside
b)Smallerthanusualscapula
c)Shortneck
d)Alltheabove
CorrectAnswer-D
Answer-D.Alltheabove
Sprengel'sdeformity:Deformityistheonlysymptomanditmaybe
noticedatbirth.Theshoulderontheaffectedsideiselevated;the
scapulalooksandfeelsabnormallyhigh,smallerthanusualand
somewhatprominent;occasionallybothscapulaeareaffected.

1466.Tinelsignisseenin?
a)Nervedegeneration
b)Nerveregeneration
c)Muscledegeneration
d)Muscleregeneration
CorrectAnswer-B
Ans.is'b'i.e.,Nerveregeneration
Pathologicalchangesafternerveinjury
Afternerveinjury,nervefirstdegeneratesandthentriesto
regenerate.
Nervedegeneration
Thepartoftheneuronedistaltothepointofinjuryundergoes
secondaryorWalleriandegeneration;theproximalpartundergoes
primaryorretrogradedegenerationuptoasinglenode.
Nerveregeneration
Asregenerationbegins,theaxonalstumpfromtheproximal
segmentbeginstogrowdistally.Iftheendoneuraltubewithits
containedSchwanncellsisintact,theaxonalsproutmayreadily
passalongitsprimarycourseandreinnervatetheend-organ.The
rateofrecoveryofaxonis1mmperday.
Themusclesnearestto
thesiteofinjuryrecoversfirst,followedbyothersasthenerve
reinnervatesmusclesfromproximaltodistal,theso-calledmotor
march.
Whentheskinoverthenerveispercussedgentlyfromdistalto
proximal,thepatientgetsatinglingsensationifthenerveis
recovering.ThisiscalledTinel'ssignandisasignofrecovery.

1467.AllaretrueaboutMarie-strumpell
diseaseexcept:
a)Mostcommonlyinvolvesthesacro-iliacjoints
b)Enhesitisiscommon
c)Morecommoninmales
d)Roentgenogramisthemostsensitiveinvestigation
CorrectAnswer-D
Answer-D.Roentgenogramisthemostsensitiveinvestigation
Marie--StrumpelldiseaseisalsoknownasAnkylosingSpondylitis
Intheearlydiseaseprocess,plainx-rays(Roentgenogram)maybe
readasnormal-->notverysensitive.
Ankylosingspondylitisprimarilyaffectsaxialskeleton.Thedisease
usuallybeginsinthesacro-iliacjointsandusuallyextendsupwards
toinvolvethelumbar,thoracic,andoftencervicalspine.Intheworst
casesthehipsorshouldersarealsoaffected.Hipjointisthemost
commonlyaffectedperipheraljoint.
Itismorecommoninmales(maletofemaleratio2-3:1)

1468.Mostcommonlylesionassociatedwith
pathologicalfractureinhandis-
a)Enchondroma
b)Metastases
c)Osteoidosteoma
d)Osteochondroma
CorrectAnswer-A
Answer-A.Enchondroma
Inhandpathologicalfracturesaremostcommonlyassociatedwith
benignbonetumors.
About23%ofthebonetumorsinhandpresentwithpathological
fractures.
Theaverageageofpresentationis37years.
Themostcommonboneaffectedistheproximalphalynx,thefifth
raywasinvolvedin44%ofthepatients.
Majority(approximately88%)ofthepathologicalfracturesare
causedbyenchondromas.

1469.Mostcommonprimarybonetumourofhandis:
a)Osteoma
b)Osteochondroma
c)Enchondroma
d)Noneoftheabove
CorrectAnswer-C
Enchondromasarisefromcartilageandarethemostcommonprimarybonetumorsofthe
hand.Theselesionsaccountfor>90%ofbonetumorsseeninthehand.
Ref:Schwartz'sprincipleofsurgery9thedition,chapter44.
Commonestbonemalignancies->Secondaries
(metastases)
Commonestprimarymalignanttumor>Multiple
myeloma
Commonestprimarymalignanttumorolongbones
Osteosarcoma
Commonestbenigntumorofbonne-
Osteochondroma
(Osteochondromaisnottureneoplasmsinceitsgrowthstopswith
cessationofgrowthattheepiphysealplate)
Commonesttruebenigntumorofbone->Osteroid
osteoma
Commonestbenigntumorofhand-Enchondroma

1470.Mostcommonmalignantbone
tumor-
a)Osteogenicsarcoma
b)Secondaries
c)Osteoma
d)Enchondroma
CorrectAnswer-B
Ans.is'b'i.e.,Secondaries
oSecondariesaremostcommonmalignantbonetumor.
oOsteogenicsarcomaisthemostcommonprimarymalignantbone
tumor.oOsteomaismostcommonbenignbonetumor.

1471.Agegroupaffectedbyosteosarcoma-
a)Upto10years
b)10-20years
c)30-40years
d)Olderthan45years
CorrectAnswer-B
Answer-B.10-20years
Osteogenicsarcomaisthemostcommonprimarymalignanttumor
ofboneinchildren.
Thisispredominantlyatumorofchildhoodoradolescence,occuring
mostcommonlyinthe10-25years.
Themostcommonsiteofinvolvementismetaphysisoflongbone
aroundknee:-
Lowerendoffemur(45%)
UpperendofTibia(25%)

1472.Whichisintramendullarytumoramong
carcinomaofbone-
a)ClassicalOsteosarcoma
b)Parostealosteosarcoma
c)Periostealosteosarcoma
d)Noneoftheabove
CorrectAnswer-A
Answer-A.ClassicalOsteosarcoma
Initsclassic(intramedullary)form,osteosarcomaisahighly
malignanttumorarisingwithintheboneandspreadingrapidly
outwardstotheperiosteumandsurroundingsofttissue.Thetumor
mostcommonlybeginsinthemetaphysis.Osteosarcomaarises
fromprimitivebone-formingcells.Tumordestroysthebone
structureandeventuallyburstsintothesurroundingsofttissues.

1473.33yroldfemalepresentswithaslow
growingbonymassalongthedistal
femurcortexinthemetaphysealregion
withanappreciablegapbetweenthe
cortexandtumorwithoutanycortical
invasion.Whatistheusualtreatment
forthesame?

a)Localresection
b)Amputaion
c)Chemotherapy
d)Radiotherapy
CorrectAnswer-A
Answer-A.Localresection
Slowgrowingbonymassalongthedistalfemurcortexinthe
metaphysealregionwithanappreciablegapbetweenthecortexand
tumorwithoutanycorticalinvasioninanindividualinthe3rdorLIth
decadeoflifeissuggestiveof-parostealosteosarcoma.Local
resectionofthelesionistheusualtreatmentforparosteal
osteosarcoma.

1474.Firstradiologicalsignforactive
tuberculararthritisis-
a)Localizedosteoporosis
b)Sclerosis
c)Jointspacereduction
d)Osteophytes
CorrectAnswer-A
Answer-A.Localizedosteoporosis
Intuberculararthritis,localizedosteoporosisisthefirstradiological
signofactivedisease.

1475.Whichofthefollowingisnottrueabout
themanagementofpottsparaplegia?
a)Chemotherapyisthemainstayofconservativemanagement
b)Paraplegianotimprovingwithconservativetreatmentevenafter
3-6monthsisanindicationforoperativeintervention
c)Decompressionviaanterolateralapproachismostpreferred
d)Posteriorfusionandinstrumentationcanbeusedtocorrectthe
deformity
CorrectAnswer-C
Answer-C.Decompressionviaanterolateralapproachismost
preferred
ThefollowingmeasuresareadoptedinthetreatmentofPott's
paraplegia:
1)Conservativetreatment
Chemotherapy(ATTs)isthemainstayofconservativetreatment.
Immobilizationbytraction(incervicalspine)orbrace(indorsal
region).
Physiotherapyofparalysedlimb.
2)Surgicaltreatment
1. Followingarethemainindicationsforsurgery.
2. Failedconservativetreatment:-Paraplegiadoesnotshow
improvementbyconservativetreatmentevenafter3-6months.
3. Patientdevelopsparaplegiawhileonconservativetreatment.
4. Paraplegiagettingworsedespiteadequateconservativetreatment.
5. Indoubtfuldiagnosis.
6. Rapidonsetparaplegia
7. Recurrenceofparaplegiaafterimprovementinitialy.
OperativeprocedureforPott'sparaplegia

Therearevariousprocedures,themostcommonlyusedprocedure
isanteriordecompressionbysurgicaldebridement(removaldead,
necrotic&caseousmaterial)followedbyautogenousstrutgrafting.
Thelogiciswellunderstood;thecompressionisfromanteriorside
mostofthetimebecausetuberculosisoccursinvertebralbody
whichliesanteriortothespinalcord.So,anteriordecompressionis
thebestprocedure.
Anteriordecompressioncanbecausedby:
1. Anteriorapproach:-Calledanteriordecompression.Itisthemost
preferredprocedure.
2. Anterolateralapproach:-Calledanterolateraldecompression.
Othersurgicalprocedures(otherthananteriordecompression)
are:

1. Costo-transversectomy
2. Posteriorfusionandinstrumentationtocorrectkyphoticdeformity.
3. Laminectomy

1476.Tuberculosisofspinemostcommonly
affectswhichvertebralsegment?
a)Upperdorsal
b)Lowerdorsal
c)Lumbar
d)Cervical
CorrectAnswer-B
Answer-B.Lowerdorsal
Thespineisthecommonestsiteofboneandjointtuberculosis.The
mostcommonsiteisDorsolumbarregion.Lowerdorsal(thoracic)
regionisthemostcommonsegmentinvolvedfollowedbylumbar
segment.Thetuberculosisofspineisalsocalledpott'sdiseaseor
tubercularspondylytis.

1477.A20yearoldmalepresentswithhistory
ofgradualonsetpainandswellingin
leftkneesince6months.Nowsincelast
1monthpatienthasstartedlimping
whilewalkingandalsohasflexion
deformityofknee.Ultrasonography
showspresenceofsynovialthickening.
Whatisthemostprobablediagnosis?

a)Tuberculosisofknee
b)Pigmentedvillonodularsynovitis
c)Synovialsarcoma
d)Hemarthrosis
CorrectAnswer-A
Answer-A.Tuberculosisofknee
Unilateralmonoarticularkneeinvolvementwithgradualonsetpain
swellingandflexiondeformity,withsynovialthickeningismost
probablysuggestiveoftuberculosisofknee.
Presentingcomplaints:
Thepatient,usuallyintheagegroupof10-25years,presentswith
complaintsofpainandswellingintheknee
OnExamination:
1. Swelling:Thejointisswollen,whichmaybeduetosynovial
hypertrophyoreffusion.
2. Muscleatrophy
3. Coldabscess

4. Sinus
5. Deformity
6. Themovementsatthejointarelimited

1478.X-rayappearanceofsequestrumis-
a)Unnaturalradiodensefragments
b)Osteopenicfragment
c)Fragmentwithhoneycombloculatedappearance
d)Radiolucentareawithspeckledcalcification
CorrectAnswer-A
Answer-A.Unnaturalradiodensefragments
Sequestrumisapieceofdeadbone,surroundedbyinfected
granulationtissuetryingtoeatthesequestrumaway.
Onx-ray,sequestrashowupasunnaturaldensefragments,in
contrasttothesurroundingosteopenicbone

1479.Cubitusvalgusdevelopsas
complicationof-
a)Jupitorfracture
b)Smithsfracture
c)Malgaignefracture
d)Staddlefracture
CorrectAnswer-A
Answer-A.Jupitorfracture
Fractureoflateralcondyleofhumerus(Jupitorfracture)

1480.Mostcommonsitefortheosteoporotic
vertebralfractureis?
a)Dorsolumbarspine
b)Cervicalspine
c)Lumbosacralspine
d)Dorsalspine
CorrectAnswer-A
Ans.is'a'i.e.,Dorsolumbarspine
Osteoporosisisanasymptomaticdisorderunlesscomplications
(predominantlyfractures)occur.
Mostcommonsymptomofosteoporosisisbackpainsecondaryto
vertebralcompressionfracture.
Dorso-lumbarspineisthemostfrequentsite.
Othercommonsitesoffracturearelowerendradius(Colle's
fracture)andfractureneckfemur.
Osteoporoticfracture(fragilefractures)are:(i)Fracturevertebrae
(mostcommon),
(ii)Colle'sfracture,(iii)Fractureneckfemur.
Serumcalcium,phosphateandalkalinephosphatasearenormalin
osteoporosis.

1481.Apatientpresentswithwristtrauma.
Oninvestigationspatientisdiagnosed
tohaveasprainedwrist,withoutany
evidenceoffracture.Thereis
tendernessinanatomicalsnuffbox.
Whichligamentiscommonlyinvolved-

a)Scapholunateligament
b)Radialcollateralligament
c)Lunotriquetralligament
d)Ulnarcollateralligament
CorrectAnswer-A
Answer-A.Scapholunateligament
Lunateandscapholunateligmante-Tenderinlunatefractureand
scapholunatedissociation

1482.Trendelenburgtestispositivedueto
injurytowhichofthefollowingnerve?
a)Obturator
b)Sciatic
c)SuperiorGluteal
d)InferiorGluteal
CorrectAnswer-C
Ans.C.SuperiorGluteal
ApositiveTrendelenburgisrelativelynon-specificandmay
indicate:
Pain(e.g.duetoosteoarthritisofthehipjoint)
Weakhipabductors(gluteusmedius,gluteusminimus)
Shortfemoralneck/fractureofneck
Dislocationorsubluxationofthehip
Neuropathy
GluteusmediusandminimusaresuppliedbySuperiorGluteal
nerve.
Trendelenburgtest
Normallywhenapersonismadetostandononeleg,thehip
abductorsoftheipsilateralsideraisetheoppositeandthe
unsupportedsideofthepelvis.Iftheabductormechanismis
defective,theunsupportedsideofthepelvisdropsandthisisknown
aspositiveTrendelenburgtest.

1483.Falseaboutosteoarthritisis?
a)Involvessynovialjoints
b)Progressivesofteningofthearticularcartilage
c)Itisaninflammatoryarthritis
d)Marginalosteophytesareproduced
CorrectAnswer-C
Ans.is'c'i.e.,Itisaninflammatoryarthritis
Osteoarthritis(OA)isachronicdisorderofsynovialjointsinwhich
thereisprogressivesofteninganddisintegrationofarticularcartilage
accompaniedbynewgrowthofcartilageandboneatthejoint
margins(osteophytes),cystformationandsclerosisinthe
subchondralbone,mildsynovitisandcapsularfibrosis.
Thetermosteoarthritisisamisnomerasitisanon-inflammatory
condition.
Therighttermisosteoarthrosisordegenerativejointdisorder
becauseitisadegenerativewear-and-tearprocessoccuringin
joints.

1484.ShentonslineisseeninXrayof-
a)Antero-posteriorpelviswithbothhips
b)Antero-posteriorshoulder
c)Lateralcervicalspine
d)Laterallumbosacralspine
CorrectAnswer-A
Answer-A.Antero-posteriorpelviswithbothhips
WithanormalhipShenton'sline,whichcontinuesfromtheinferior
borderofthefemoralnecktotheinferiorborderofthepubicramus,
lookscontinuous;anyinterruptioninthelinesuggestsanabnormal
positionofthefemoralhead.
Narrowingofthejoint'space'isasignofarticularcartilageloss,a
featureofbothinflammatoryandnon-inflammatoryarthritis.

1485.Falseaboutosteogenesisimperfectais
-
a)Defectivecollagenformation
b)Associatedwithcataractformation
c)Autosomaldominant
d)KnownasBrittlebonedisease
CorrectAnswer-B
Answer-B.Associatedwithcataractformation
Osteogenesisimperfecta,alsoknownasbrittlebonediseaseor
Lobsteinsyndrome,isahereditaryconditioncharacterizedby
fragilityofbones,deafness,bluesclera,laxityofjointsandtendency
toimprovewithage.
Itisadiseaseofdefectivecollagenformation.Therefore,collagen-
containingtissuesareaffected,e.g.bone,teeth,skin,tendonsand
ligaments.
Primarydefectinboneisdefectiveosteoidformation.
Itisusuallytransmittedasanautosomaldominant,butinasevere
variantofthediseasetheparentsarenormalandafreshgene
mutationorautosomalrecessiveinheritanceispostulated.
Radiologyshowswormianbonesintheskull.

1486.Mostmobilesegmentofvertebral
columnis-
a)Cervical
b)Thoracic
c)Lumbar
d)Sacral
CorrectAnswer-A
Answer-A.Cervical
Thespinalcolumncanbedividedintothreemobile(cervical,
thoracic,andlumbarregions)andtwofused(sacrumandcoccyx).
Cervicalregion:Itisthemostmobileregionofthespinalcolumnwith
rangeofmotionofapproximately80-90degreesofflexion,70
degreesofextention,20-45degreesoflateralflexion,andupto90
degreesofrotationtobothsides

1487.Chauffeurfractureis-
a)Extra-articularfractureofstyloidprocess
b)Intra-articularfractureofstyloidprocess
c)Intra-articularfractureofbaseof1stmetacarpal
d)Extra-articularfractureofbaseof1stmetacarpal
CorrectAnswer-B
Answer-B.Intra-articularfractureofstyloidprocess
Anintra-articularobliquefractureofthestyloidprocessoftheradius.

1488.TreatmentofchoiceinAcutemyositis
ossificansis:
a)Immobilizationofelbow
b)Shortwavediathermy
c)Passivemovementsofarm
d)Activeexercises
CorrectAnswer-A
Ai.e.Immobilizationofelbow
*Restoftheaffectedpartduringtheperiodwhiletheprocessis
activeisthebasicprincipleoftreatment(TN91).
*Nomassageshouldbegiven.
*Ifanyphysicaltherapyisgoingon,itshouldbediscontinued
immediately.
*Allpassivemovementsandvigorousexercisemustbestopped.
Verygentleexercisemustbestarted.
*Indomethacinandradiotherapypreventcalcification.However,
radiotherapyshouldbeavoidedinchildren.
-Surgicalexcisionisindicatedifmyositisossificansinterfereswith
function.Itisworthnotingthatsurgicalexcision
iscontraindicatedduringactivephasewhencalcification
matures.
*Surgicalexcision,ifrequired,shouldbedoneonlyafter
completematurationofossification.


1489.Whatisthetreatmentforpatientwith
hypertrophicnonunionwithdeformity
atfracturesite?

a)Notreatmentrequired
b)Fixationonly
c)Bonegraftingonly
d)Fixationwithbonegrafting
CorrectAnswer-B
Answer-B.Fixationonly
Patientswithhypertrophicnonunionhavesufficientvascularityto
healbutstabilityislackingornormalaxialalignmenthasnotbeen
restored.Thusinacaseofhypertrophicnonunionwithoutdeformity,
fixationalonewillprovidestabilityandwillleadtohealing.

1490.Proximaltibialepiphysisfusesat-
a)12-14years
b)14-16years
c)16-18years
d)18-20years
CorrectAnswer-C
Answer-C.16-18years
Thetibiaossifiesfromthreecentres,oneintheshaftandonein
eachepiphysis.Ossificationbeginsinmidshaftatabouttheseventh
intrauterineweek.
Theproximalepiphysialcentreisusuallypresentatbirth:at
approximately10yearsathinanteriorprocessfromthecentre
descendstoformthesmoothpartofthetibialtuberosity.

1491.Managementofdisplacednon
comminutedintercondylarhumerus
fractureis-

a)Openreductioninternalfixation
b)Aboveelbowplasterslabapplication
c)Olecranonpintraction
d)Externalfixation
CorrectAnswer-A
Answer-A.Openreductioninternalfixation
Itdependsuponthedisplacement.
Anundisplacedfractureneedssupportinanabove-elbowplaster
slabfor3-4weeks,followedbyexercises.
Adisplacedfractureistreatedgenerallybyopenreductionand
internalfixation.
Incaseswithseverecomminution,olecranonpintractionisgivento
reducethefractureandmaintainthereduction.

1492.MCcauseofPrimaryAmenorrheais?
a)Constitutional/idiopathic
b)RMKHsyndrome
c)Ovariandysgenesis
d)Noneoftheabove
CorrectAnswer-C
Ans,C.Ovariandysgenesis
MCcauseofprimaryamenorrheaisovariandysgenesis/Turner
syndrome.

1493.Aninfertilewomanhasbilateraltubal
blockatcornuadiagnosedon
hysterosalpingography.Nexttreatment
ofchoiceis?

a)IVF
b)Laparoscopyandhysteroscopy
c)Tuboplasty
d)Hydrotubation
CorrectAnswer-B
Ans.B.Laparoscopyandhysteroscopy
Laparoscopy(withchromopertubationwithmethyleneblue
dye):
Bestinvestigationfortubalpatency,astubalpatencycanbe
confirmedundervision,andbesides,anypathologycan
simultaneouslybecorrectedwithoperativelaparoscoPy.

1494.Fetalkaryotypingcanbedonebyall,
EXCEPT?
a)Cordocentesis
b)Amniocentesis
c)CVS
d)Fetalskinbiopsy
CorrectAnswer-D
Ans.D.Fetalskinbiopsy
Percutaneousumbilicalbloodsampling(PUBS)isalsoknownas
cordocentesisisperformedafter16weeks'gestation'underUSG
guidanceaneedleisinsertedintoumbilicalvein.Thistechnique
apartfromkaryotypingisalsousefulforevaluatingfetalmetabolism
andhematologicabnormalities.

1495.55yearoldladycomplaintsofmassin
thevagina,whichisreducible&
increasesondefecation.Thediagnosis
is?

a)Rectalprolapse
b)Uterineprolapse
c)Cervicalfibroid
d)Vaginalcancer
CorrectAnswer-B
Ans.B.Uterineprolapse
Prolapseisdefinedasthedisplacementofanorganfromitsnormal
anatomicalposition.
GenitalprolapseoccursduetoweaknessofthesuPPorts.

1496.Moschowit'ssurgeryisdonefor-
a)Cervicalcancer
b)Fundalfibroids
c)Uterineprolapse
d)Enteroceleprevention
CorrectAnswer-D
Ans.D.Enteroceleprevention
EnteroceleformatiolcanbepreventedbyMoschowit'sorHalban's
surgeries,inboththePODisobliterated.

1497.
Whichofthefollowingisusedin
quantifyinghirsutism?
a)Bishopscore
b)Rotterdamcriteria
c)Ferriman-Gallweyscore
d)Alloftheabove
CorrectAnswer-C
Ans.C.Ferriman-Gallweyscore
TheFerriman-Gallweyscoreisamethodofevaluatingand
quantifyinghirsutisminwomen.Themethodwasoriginallypublished
inl96lbyD.FerrimanandJ.D.GallweyintheJournalofClinical
Endocrinology.

1498.MCheartdiseaseinpregnancyis?
a)MS
b)AS
c)MR
d)WPWsyndrome
CorrectAnswer-A
Ans.A.MS
MitralstenosisistheMCvalvularheartdiseaseinpregnancy.

1499.Rateofcervicaldilatationin
primigravidais?
a)0.8cm/hour
b)1.2cm/hour
c)1.5cm/hour
d)2cm/hour
CorrectAnswer-A
Ans.A.0.8cm/hour
Thenormalrateofcervicaldilatationinactivephaseis1.2cm/hour
inprimigravidaeand1.5cm/hourinmultiparae.

1500.Complicationsofpreeclampsiaareall
except?
a)Postdatism
b)DIC
c)Blindness
d)Noneoftheabove
CorrectAnswer-A
Ans.A.Postdatism

1501.Whatismonitoredinapatientofpre
eclamsia?
a)Uricacid
b)Plateletcount
c)LFT
d)Alloftheabove
CorrectAnswer-D
Ans.D.Alloftheabove

1502.Notanabsolutecontraindicationfor
methylergometrineuseis?
a)Eclampsia
b)Heartdisease
c)Rhincompatibility
d)Afterdeliveryoffirstbabyintwinpregnancy
CorrectAnswer-C
Ans.C.Rhincompatibility
Methylergometrine(Methergin)canbeusedintheprevention
andtreatmentofPPH.Absolutecontraindicationstotheuseof
Metherginare:

1. Chronichypertension/preeclampsia/eclampsia
2. Heartdiseaseinpregnancy
3. Afterthedeliveryofthefirstbabyofthetwins.

1503.Aladywith35weeksofpregnancyis
admittedinviewoffirstepisodeof
painlessboutofbleedingyesterday.On
examinationHb10g%,BP120/70
mmHg,uterusrelaxed,andcephalic
floating.FHSregular.Nextlineof
managementis?

a)Cesareansection
b)Inductionoflabor
c)Waitandwatch
d)Bloodtransfusion
CorrectAnswer-C
Ans.C.Waitandwatch
Thisisacaseofplacentaprevia(painlessbleeding,relaxeduterus,
andfloatingheadallpointtoplacentaprevia).
Inthiscase,allthecriteriaforconservativemanagementarefulfilled
andthereforetheansweriswaitandwatchforfetallungmaturity.

1504.G3P2L2withprevious2LSCSwith
anteriorplacentapreviahasgotavery
highriskofwhichcomplication?

a)Placentaacreta
b)Vasaprevia
c)Abruption
d)Noneoftheabove
CorrectAnswer-A
Ans.A.Placentaacreta
Thetermplacentaaccretaisusedtodescribeanyplacental
implantationinwhichthereisabnormallyfirmadherencetothe
uterinewall.
Theincidenceofplacentaaccreta,increta,andpercretahas
increased,mostlikelybecauseoftheincreasedcesareandelivery
rate.

1505.B-Lynchsutureisappliedon?
a)Cervix
b)Uterus
c)Fallopiantubes
d)Ovaries
CorrectAnswer-B
Ans.B.Uterus
DescribedfirstbyChristopher,B-Lynchisacompressionsuture
placedonuterusinthemanagementofatonicPPHwhenthe
medicalmethodsfail.

1506.A28-year-oldprimigravidawith33
weeksofpregnancysuddenly
complainsofheadache,oliguria,and
blurredvision.HerBPis180/110and
urinealbuminis+3.Thelineoffurther
managementis?

a)Waitandwatch
b)LSCS
c)Inductionoflabor
d)Anticonvulsant+antihypertensivetherapy
CorrectAnswer-D
Ans.D.Anticonvulsant+antihypertensivetherapy
Thepatientisacaseofseverepreeclampsia,withimpending
eclampsia.
Magnesiumsulfateisthedrugofchoiceforeclampsiaandalsofor
impendingeclampsia.

1507.Whichofthefollowingstatements
concerningabdominalpregnancyis
correct?

a)Gastrointestinalsymptomsarequiteoftenverysevere
b)Fetalsurvivalisapproximately80%
c)Aggressiveattemptsshouldbemadetoremovetheplacentaat
thetimeofinitialsurgery
d)Placentacanbeleftinsituatthetimeofsurgery
CorrectAnswer-D
Ans.D.Placentacanbeleftinsituatthetimeofsurgery
Althoughleavingtheplacentaintheabdomenfollowingsurgical
deliverypredisposestoriskesofpostoperativeinfections,theriskis
muchlessseverethanthehemorrhageassociatedwithattemptsof
removalofplacentaatthetimeofprimarysurgery.

1508.Completevesicularmoleisassociated
Thecaleutincsytsinwhatpercentage
ofcases?

a)<5
b)5-15
c)20-40
d)60-70
CorrectAnswer-C
Ans.C.20-40
Theca-luteincysts:Inmanycasesofhydatidiformmole,theovaries
containmultipletheca-luteincysts.

1509.Patientwith3monthsamenorrhoea,c/o
hyperemesisandvaginalbleedingsince
onemonth.O/E=uterus16weekswith
absentfetalheartsound.Thediagnosis
is?

a)Vesicularmole
b)Ectopicpregnancy
c)IUFD
d)Abruptioplacentae
CorrectAnswer-A
Ans.A.Vesicularmole
Vesicularmoleisanabnormalconditionofplacentawherethereis
hydropicdegenerationandproliferativechangesintheyoung
chorionicvilli.Itisabenignconditionwithmalignantpotential.
Uterinebleedingisalmostuniversalandmayvaryfromspottingto
profusehemorrhage.
ItistheMCpresentingfeature.
Thedischargehas'whitecurrantinredcurrantjuice'appearance.

1510.Patientwithchoriocarcinoma&
jaundice,treatmentofchoiceis?
a)Methotreaxate
b)ActinomycinD
c)Suctionevacuation
d)Combinationofall
CorrectAnswer-B
Ans.B.ActinomycinD

1511.EMACOregimeisfor?
a)Cacervix
b)Caendometrium
c)Caovary
d)Choriocarcinoma
CorrectAnswer-D
Ans.D.Choriocarcinoma

1512.Stagelbcervicalcancerisdiagnosedin
ayoungwoman.Assumingthatthe
cancerisconfirmedtothecervixand
thatintraoperativebiopsiesare
negative,whichofthefollowing
structurewouldnotberemovedduring
theradicalhysterectomy?

a)Uterosacralanduterovesicalligaments
b)Pelvicnodes
c)Theentireparametriumonbothsidesofthecervix
d)Bothovaries
CorrectAnswer-D
Ans.D.Bothovaries
Preservationoftheovariesisgenerallyacceptable,particularlyin
youngerwomen.

1513.Invaginalhysterectomy,thefirstclamp
includes:
a)Uterineartery
b)Fallopiantube&roundligament
c)Uterosacralligament
d)Noneoftheabove
CorrectAnswer-C
Ans.C.Uterosacralligament
Vaginalhysterectomywithpelvicfloorrepairisdonefor
vaginouterineprolapseinperi/postmenopausalwomen.
Firstclampincludesuteroscaral&Macenrodt'sligament.
Secondclampincludesuterineartery.
Thirdclampincludescornualstructures.

1514.Therapeuticconizationisindicatedin?
a)Microinvasivecarcinomacervixstagelal
b)CINIII
c)Unsatisfactorycolposcopywithcervicaldysplasia
d)Cervicalmetaplasia
CorrectAnswer-A
Ans.A.Microinvasivecarcinomacervixstagelal
Instage1A1,thereisnolymphnodeinvolvement.
Therapeuticconizationisthesurgeryofchoiceforstage1A1in
youngpatientswhoaredesirousoffuturechildbearing

1515.CervicalcancerIIIBtreatmentis?
a)Wertheim'soperation
b)Radiotherapy
c)Chemotherapy
d)Chemoradiation
CorrectAnswer-D
Ans.D.Chemoradiation
CisplatinisgivenbeforeRTasaradiosensltizer,hencethepreferred
terminologyisCTRT(concurrentchemoandradiotherapyalso
knownaschemoradiation).

1516.Wertheim'shysterectomyisdonefor?
a)1A1cervicalcancer
b)IBcervicalcancer
c)Germcellovariancancer
d)Alloftheabove
CorrectAnswer-B
Ans.B.IBcervicalcancer
StagesofCacervixthatareoperable(radical/Wertheim's
hysterectomy)are1A2,IB,andIIA.

1517.RadioisotopeusedinCacervix
Brachytherapy?
a)Cobalt
b)Iridium
c)Cesium
d)Alloftheabove
CorrectAnswer-D
Ans.D.Alloftheabove

1518.Cavulvaspreadstoallexcept?
a)Urethra
b)Deepinguinalnodes
c)Superficialinguinalnodes
d)Paraaorticnodes
CorrectAnswer-D
Ans.D.Paraaorticnodes
Modesofspreadofca.vulva

1. Direct:Urethra,Vagina,Rectum,pelvicbones
2. Lymphatic:MCrouteofspread.
3. Lymphaticsoflabiatosuperficial,thendeepinguinalnodes&then
pelvicnodes
4. Hematogenousrouterare&mayoccurinadvancedcases

1519.Whichovariantumourcanpresentwith
menorrhagia?
a)Demoidcyst
b)Epithelialovariancancer
c)Granulosacelltumour
d)Yolksactumour
CorrectAnswer-C
Ans.C.Granulosacelltumour
Granulosacelltumours(orgranulosa-thecacelltumours)are
tumoursthatarisefromgranulosacells.Thesetumoursarepartofthe
sexcordgonadalstromaltumourornon-epithelialgroupoftumours.

1520.Allareriskfactorsforectopic
pregnancyexcept?
a)Pasthistory
b)Tuballigationfailure
c)IVF
d)LNGIUCD
CorrectAnswer-D
Ans.D.LNGIUCD
IUCD:ThemoderncopperIIJDdoesnotincreasetherkkofectopic
pregnancy.However,thereisarelativeincreaseintubalpregnancy
(7timesmore)shouldpregnancyoccurwithIUCDinsitu.
Studieshavedemonstratedthatuptol%ofpregnanciesachieved
throughIVForGIFTcanresultinaheterotopicgestation.

1521.A21-year-oldfemalepresentsto
emergencywardwith2monthsof
amenorrheawithpaininabdomenand
shock.BP90/60mmHgandHb6gm%.
Urinepregnancytestisfoundpositive.
Nextimmediatelineoftreatmentis?

a)Laparotomy
b)IVfluids&crossmatch
c)Medicalmanagement
d)Laparoscopy
CorrectAnswer-B
Ans.B.IVfluids&crossmatch
Thisisacaseofrupturedectopicpregnancy.PositiveUrine
PregnancyTestindicatesthattheamenorrheaisduetopregnancy.
Painandshockinearlypregnancyaremostlyalwaysdueto
rupturedectopic.

1522.DOCformedicalmanagementof
ectopicpegnancy?
a)ActinomycinD
b)IntramuscularMethotrexate
c)IntramuscularMethotrexate
d)PGf2alpha
CorrectAnswer-B
Ans.B.IntramuscularMethotrexate

1523.WhatisnotusedinPCOS?
a)OCpills
b)Cyclicalprogesterones
c)Myoinositol
d)Danazol
CorrectAnswer-D
Ans.D.Danazol
Insulinsensitizersarealsousedtotackleinsulinresisitance.
DanazolhasnoroleinPCOS.

1524.Pearlnecklaceappearanceis
characteristicof?
a)Ectopicpregnancy
b)PCOS
c)Endometriosis
d)PID
CorrectAnswer-B
Ans.B.PCOS
USGfeaturesofpolycysticovariansyndrome(PCOS)
Greaterthan12folliclesmeasuringbetween2mmand9mmin
diameterlocatedperipherally,resultinginapearlnecklace
appearance.
Increasedechogenicityofovarianstromaand/orovarianvolume
greaterthan10ml.

1525.Violinstringadhesion[violentstring
sign]isseenin?
a)PCOS
b)Endometriosis
c)Fitz-high-curtissyndrome
d)Rupturedectopicpregnancy
CorrectAnswer-C
Ans.C.Fitz-high-curtissyndrome
Laparoscopyisperformed,thelivercapsulewillappearinflamed,
withclassicviolinstringadhesionsintheperietalperitoneum
beneaththediaphragm.
Fivepercentto10%ofwomenwithacutePIDdevelopsymptomsof
perihepaticinflammation,theFitz-High-Curtissyndrome.

1526.Dysgerminomaspreadsmainlyvia?
a)Hematogenousroute
b)Lymphaticroute
c)Directspread
d)Doesnotspread
CorrectAnswer-B
Ans.B.Lymphaticroute
Dysgerminomasarethemostcommonmalignantgermcelltumors,
accountingforabout30%to40%ofallovariancancersofgermcell
origin.
Inthe25%ofpatientswhopresentwithmetastaticdisease,the
tumormostcommonlyspreadsviathelymphatics.

1527.Triadofsymptomsofendometriosisare
allexcept?
a)Infertility
b)Dysmenorrhea
c)Dyspareunia
d)Cyclicalhematuria
CorrectAnswer-D
Ans.D.Cyclicalhematuria
Endometriosisisdefinedasthepresenceofnormalfunctional
endometrialmucosa(glandsandstroma)abnormallyimplantedin
locationsotherthantheuterinecavity.ItwasfirstdescribedbyVon
RokitanskyAboutonethirdofwomenwithendometriosisremain
asymptomatic.

1528.ComplicationofBenignOvarianCysts
is?
a)Torsion
b)Intracystichemorrhage
c)Pseudomyxomaperotonei
d)Alloftheabove
CorrectAnswer-D
Ans.D.Alloftheabove

1529.Level1supportofuterus&vaginais?
a)levatorani
b)Perinealbody
c)Uterosacralligaments
d)Alloftheabove
CorrectAnswer-C
Ans.C.Uterosacralligaments
Delancey'sthreelevelsofpelvic(uterus,vagina)support=
Level1:Theuterosacral-cardinalligamentcomplexprovides
attachmentoftheuterusandvaginalvaulttothesacrum.Uterine
prolapseoccurswhenthisligamentcomplexbreaksorisattenuated.

1530.MCpresentingsymptomoffibroidis?
a)Menorrhagia
b)Infertility
c)Lump
d)Compression
CorrectAnswer-A
Ans.A.Menorrhagia

1531.Medicalmanagementoffibroidsiswith
allexcept-
a)Progesterone
b)Mifepristone
c)Ulipristalacetate
d)Misoprostol
CorrectAnswer-D
Ans.D.Misoprostol

1532.Endometriosisis?
a)Endometriumwithinthemyometrium
b)Functionalendometriumoutsidetheuterus
c)Myometriumwithintheendometrium
d)RaresquamousvarietyofCAendometrium
CorrectAnswer-B
Ans.B.Functionalendometriumoutsidetheuterus
Endometriosis
Definition:Presenceoffunctionalendometriumatplacesotherthan
uterus(ectopicendometrialtissue)

1533. Samson'stheoryforDevelopmentof
Endometriosisis?
a)Celomicmetaplasia
b)Hematogenousspread
c)Lymphaticspread
d)Retrogrademenstruation
CorrectAnswer-D
Ans.D.Retrogrademenstruation
TheoriesforDevelopmentofEndometriosis

1. Samson'stheoryofretrogrademenstruation:themostaccEpted
theory
2. IvanoffandMeyer:Celomicmetaplasia
3. HematogenoussPread
4. Lymphaticspread(Halban'stheory)
5. Directimplantation.

1534.Examinationofa26yearsoldobese
infertilefemalereveals.Fixed
retroverteduterus&Nodularityofthe
uterosacralligaments.Themostlikely
diagnosisis?

a)PCOS
b)Endometriosis
c)Adenomyosis
d)TB
CorrectAnswer-B
Ans.B.Endometriosis
Pelvicexaminationofendometriosis:Maybenormalormay
revealthefollowing:
Fixedretroverteduterus
Pelvictenderness
NodulesinthePOD
Nodularityoftheuterosaoalligaments
Unilateralorbilateraladnexalmass
Speculumexaminationmayrevealbluishnodulesinposteriorfornix.

1535.Investigationofchoicefor
endometriosis?
a)USG
b)CA125
c)MRI
d)Laparoscopy
CorrectAnswer-D
Ans.D.Laparoscopy
LaparoscopyistheInvestigationofChoice.

1536.Powderburntlesionseenin?
a)PID
b)PCOS
c)Endometriosis
d)Alloftheabove
CorrectAnswer-C
Ans.C.Endometriosis
Laparoscopyfindingsinendometriosisare:
Chocolatecysts
Blueberrylesion
Red/flamelesion
PowderburnsPots
Red/purpleraspberrylesion
Subovarianadhesions
MatchstickburntsPots
Whitelesion.

1537.Treatmentofchoiceinpatientwith
infertility&endometriosis?
a)IUI
b)Surgery
c)Danozol
d)Ovulationinduction
CorrectAnswer-B
Ans.B.Surgery
SurgicalManagement

1. Patientswithinfertility:laparoscopicovariancystectomy,
adhesiolysis,andelectrocoagulationofendometrioticimplants.
2. Ifthefamilyiscompleteandthepatienthasseverepainor
menstrualcomplaints:hysterectomywithbilateral
salpingooophorectomy.Generallycombinedapproachisadopted
wherelaparoscopicsurgeryisfollowedbyGnRHa.

1538.Surgeryofchoicein42yearoldP3L3
withdiffuseendometriosisis?
a)Ovariancystectomy&adhesiolysis
b)Hysterectomy
c)HysterectomywithBSOwithresectionofendometrialimplants
d)Ovariancystectomy&adhesiolysis&resectionofimplants
CorrectAnswer-C
Ans.C.HysterectomywithBSOwithresectionofendometrial
implants
Ifthefamilykcompleteandthepatienthasseverepainormenstrual
complaints:Hystetectomywithbilateralsalpingo-oophorectomywith
resectionofallendometrioticimplants.

1539.ThemostcommonMulleriananomaly
is?
a)Mullerianagenesis(RMKH)
b)Unicornuateuterus
c)Bicornuateuterus
d)Septateuterus
CorrectAnswer-D
Ans.D.Septateuterus
SeptateuterusistheMCMulleriananomaly.

1540.SEAMusedinDUBis?
a)Clomiphene
b)Raloxifene
c)Ormiloxifene
d)Mifepristone
CorrectAnswer-C
Ans.,C.Ormiloxifene
OrmeloxifeneisathirdgenerationbenzopyranSERMwhichblocks
thecytosolreceptorsbyitscompetitivebindingandselectivelyacts
onestrogenrecePtorsasagonistandantagonistindifferent
reproductivetissues.

1541.Choiceofadjuvanttreatmentfor
endometrialcarcinomastageIA,gradeI
is?

a)Radiotherapy
b)Chemotherapy
c)Chemotherapyplusradiotherapy
d)Notreatment
CorrectAnswer-D
Ans.D.Notreatment
ManagementofCaendometrium
A)Stagel=1:
Surgery(totalabdominalhysterectomywithbilateralsalpingo-
oophorectomywithlymphnodesampling),followedbyradiotherapy.
Onlypatientswithstage1A,grades1and2donotrequire
postoperativeradiotherapy.
B)Stage2:
Modifiedradicalhysterectomy,bilateralsalpingo-oophorectomywith
lymphnodedissection,followedbyradiotherapy.
C)Stages3and4:
Debulkingsurgeryfollowedbyradiotherapy.

1542.A46-year-oldP3L3complainsof
menorrhagiasince3months.Nextline
ofmanagementis?

a)D&C
b)Progesteronex6months
c)OCpillsx6months
d)Hysterectomy
CorrectAnswer-A
Ans.A.D&C
InPatientswithmenorrhagiainperimenopausalagegroup(40+),
alwaysmakethediagnosisfirstbeforeproceedingwithany
treatment.
Itisnecessarytoruleoutendometrialhyperplasiaandcancerinthis
agegroup.Hence,histopathologicalexaminationofendometriumis
required,andthereforeD6Cshouldbedonefirst.

1543.Simplehyperplasiawithatypiawill
progresstocaendometriumin%of
cases?

a)1-2
b)3-4
c)8-9
d)20
CorrectAnswer-C
Ans.C.8-9

1544.Grade1Caendometrium,thereis
presenceof%nonsqoumaousgrowth
?

a)<5
b)6-25
c)25-50
d)>50
CorrectAnswer-A
Ans.A.<5
FIGOGradingofEndometrialCarcinoma
Histopathologicdegreeofdifferentiation:
Gl:<5%nonsquamousornonmorulargrowthpattern.

1545.Definitivetreatmentofadenomyosisis
?
a)OCpills
b)NSAIDS
c)Endometrialablation
d)Hysterectomy
CorrectAnswer-D
Ans,D.Hysterectomy
Hysterectomy
Theonlywaytocompletelycurethisconditionistohavea
hysterectomy.

1546.MCcauseforhysterectomyis?
a)Prolapse
b)Fibroids
c)Caendometrium
d)AcutePID
CorrectAnswer-B
Ans.B.Fibroids
Ahysterectomyisanoperationtoremovetheuterus.
Awomanmayhaveahysterectomyfordifferentreasons,
including:
Uterinefibroidsthatcausepain,bleeding,orotherproblems.

1547.Prolongedsurgerytimeofvaginal
hysterectomywouldleadtodamageto
whichnerve?

a)Obturator
b)Pudendal
c)Peroneal
d)Sural
CorrectAnswer-C
Ans.C.Peroneal
Mostcommonlyinjuredlowerextremitynerveinpatientsundergoing
surgeryinlithotomypositionisthecommonparonealnerve(LA-S2).

1548.

WhichofthefollowingisnotapartofPID-
a)Endometritis
b)Cervicitis
c)Tuboovarianabcess
d)Peritonitis
CorrectAnswer-B
Ans.B.Cervicitis
PIDisaspectrumofinfection&inflammationofuppergenitaltract
organsinvolvinguterus,fallopiantubes,ovaries,pelvic
peritoneum&parametrium.
Cervicitisisnotincluded.

1549.TriadforclinicaldiagnosisPIDincludes
allexcept?
a)Fever
b)Lowerabdominalpain
c)Cervicalmotiontenderness
d)Bilateraladnexaltenderness
CorrectAnswer-A
Ans.A.Fever
DiagrosisofPIDisoftendifficult.
The"goldstandard"fordiagnosisreliesonthelaparoscopic
appearanceofFallopiantubeinflammationbutcostandlimited
availabilityofthetechniqueoftenprecludeitsuse.
Intheabsenceoflaparoscopy,thetriadoflowerabdominalpain,
cervicalmotiontenderness,andbilateraladnexaltendernesshas
beenadvocatedastheminimalcriterionforclinicaldiagnosisofPID.

1550.Acutesalpingitisismostcommonly
causedby?
a)N.gonorrhoeae
b)Chlamydiatrachomatis
c)Mycoplasma
d)Staphylococcus
CorrectAnswer-B
Ans,B.Chlamydiatrachomatis
OptionBisnowadaysslightlymorecommonerthanoptiona)&
hencethebestoptiontomark
Ifpolymicrobial/mixedinfection,isintheoption,thenthatisthe
answer.

1551. Tumormarkerforgermcellmalignancy
areallexcept?
a)LDH
b)Alkalinephosphatase
c)AFP
d)CA-125
CorrectAnswer-D
Ans.D.CA-125

1552.Acutepelvicpaincouldbedueto?
a)Ectopicpregnancy
b)PID
c)Corpusluteumhematoma
d)Alloftheabove
CorrectAnswer-D
Ans.D.Alloftheabove

1553.Nugentscoreincludesallexcept-
a)Lactobacillus
b)Gardnerella
c)Mobiluncus
d)Gonococcus
CorrectAnswer-D
Ans.D.Gonococcus
Nugent'scriteriafordiagnosisofbacterialvaginosis
ManyuseNugent'scriteriatoquantifyorgradebacteriaviaGram
stainofvaginalsamples.
Inbrief,Nugent'scriteriaevaluated3typesofbacteriaviaGram
stain:Lactobacillus,Bacteroides/Gardnerella,andMobiluncus.

1554.Cluecellsareseenin?
a)Bacterialvaginosis
b)Candidiasis
c)Chlamydiasis
d)Trichomoniasis
CorrectAnswer-A
Ans.A.Bacterialvaginosis
Bacterialvaginosis/vaginitis(BV)isacommonvaginalinfection.
Cluecells(vaginalepithelialcellscoveredwithcoccobacilliandthe
cellsappearasstippledorgranular).CIuecellsarediagnosticofBV.

1555.HSGfindingssuggestiveofgenital
koch?
a)Beadedtubes
b)Honeycombuterus
c)Golfclubtube
d)Alloftheabove
CorrectAnswer-D
Ans.D.Alloftheabove
Inactivetuberculosis,HSGiscontraindicated.

1556.Apregnantladypresentswithgenital
warts.Thebestmanagementforheris
?

a)Imiquimod
b)Trichloroaceticacid
c)Podophyllin
d)Cryotherapy
CorrectAnswer-D
Ans,.D.Cryotherapy
Forreasonsunknowngenitalwartsincreaseinsizeandnumber
duringpregnancy.
Treatmentoptionsduringpregnancyincludecryotherapyand
trichloroaceticacid(TCA).
Outofthetwo,cryosurgeryismoreeffectivethanTCAandhenceis
prefnred.

1557.28yearoldfemalecomplaintsoffoul
smellingyellowishurethraldischarge
since4days.Historyofburning
micturation.Historyofsexualcontact
withmultiplepartners2daysbeforethe
onsetofsymptoms.Mostlikely
diagnosisis?

a)Syphilis
b)Chancroid
c)Gonococcalurethritis
d)Non-infectiveurethritis
CorrectAnswer-C
Ans.C.Gonococcalurethritis
Historyoffoulsmellingmucopurulentdischarge,&shortincubation
periodwithhighriskexposureclinchesthediagnosis

1558.Twin-peaksignisseenin?
a)AllMonozygotictwins
b)Monochorionictwins
c)Dichorionictwins
d)Siamesetwins
CorrectAnswer-C
Ans.C.Dichorionictwins

1559.IncaCervixtreatment,pointAreceives
?
a)3000cGy
b)5000cGy
c)7000cGy
d)10,000cGy
CorrectAnswer-C
Ans.C.7000cGy
PointAandPointBareinrelationtoradiotherapyforCaCervix.

1560.Ashermanssyndromeischaracterized
by?
a)Amenorrhea
b)Menorrhagia
c)Polymenorrhea
d)Alloftheabove
CorrectAnswer-A
Ans.A.Amenorrhea
Intra-uterineadhesions
Ashermansyndromewasidentifiedin1948asuterinesynechiae.
Theseintra-uterineadhesions(IUA)areoftenassociatedwith
amenorrheaorinfertility.

1561.GARDASILvaccineisfor-
a)HPV16,18
b)HSV
c)HPV6,11,16,18
d)HepatitisB
CorrectAnswer-C
Ans.C.HPV6,11,16,18

1562.LNGcontentofMirenais-
a)20gms
b)20mg
c)52gms
d)52mgs
CorrectAnswer-D
Ans.D.52mgs
Mirena/LNGIUD/LNG2O/levonova/LNGIUS
Mirenacontainsatotalof52mglevonorgestrel(LNG).LNGisreleasedintotheuterinecavity
atarateofapproximtely20pg/day.

1563.Whichofthefollowingdecreasesthe
riskofPelvicInflammatoryDisease?
a)CuT
b)Spermicidalagents
c)O.C.pills
d)Todayvaginalsponge
CorrectAnswer-C
Ans.C.O.C.pills
SeveralstudieshaveshownthatregularO.C.pillusersare
protectedfromPIDstotheextentof50%.

1564.Patientwith45XO,whatHRTtobe
given?
a)Growthhormone+E+P
b)Estrogene
c)NoHRTNeeded
d)HRTonlyafter45years
CorrectAnswer-A
Ans.A.Growthhormone+E+P
Growthhormone(GH)therapyhasbecomethestandardofcarefor
girlswithturnersyndromeandshouldbeconsideredassoonas
decreasedlineargrowthvelocityisapparent.
Initialhormonereplacementinvolveslowdose
estrogenmonotherapy.
Progestagenreplacementisgenerallyadded1-2yearsafterstarting
estrogenoruponbreakthroughbleeding.

1565.Tamoxifendecreasestheriskofwhich
cancer?
a)Breast
b)Endometrium
c)Ovary
d)Alloftheabove
CorrectAnswer-A
Ans.A.Breast
Tamoxifenhasbeenusedformorethan30yearstotreatpatients
withbreastcancer.
Tamoxifenworksagainstbreastcancer,inpart,byinterferingwith
theactivityofestrogen,afemalehormonethatpromotesthegrowth
ofbreastcancercells.
InOctober1998,theU.S.FoodandDrugAdministration(FDA)
approvedtheuseoftamoxifentoreducetheincidenceofbreast
cancerinwomenatincreasedriskofthedisease.

1566.Raloxifenedecreasestheriskofwhich
cancer?
a)Breast
b)Cervix
c)Ovary
d)Alloftheabove
CorrectAnswer-A
Ans.A.Breast
Afteranaverageof81months,raloxifenereducesriskofinvasive
breastcancerbyabout38percentcomparedtotamoxifenreducing
breastcancerbyabout50percent.

1567.Leastfailurerate?
a)CuT
b)MIRENA
c)DMPA
d)O.C.PILLS
CorrectAnswer-B
Ans.B.MIRENA

1568.Nuvaringcontains?
a)EE+etonogestrel
b)LNG+EE
c)LNG
d)EE+drosperinone
CorrectAnswer-A
Ans.A.EE+etonogestrel
Contraceptiverings
NuvaRing:Itisasoftvaginalringthatreleases15microgramEE
and120microgramENG,etonogestrel,theactivemetaboliteof
desogestrel,perdayasacontrolleddeliverysystem.

1569.Whichdrugusedforendometriosiscan
causeincreaseinhepaticenzyme&
adverselipidprofile-

a)0.C.pills
b)GnRhanalogues
c)Bothoftheabove
d)Noneoftheabove
CorrectAnswer-A
Ans.A.0.C.pills
O.C.pills(progesteronecomponent)area/wincreaseinLDL&
decreaseinHDLcholesterolbutestrogenshaveoppositeeffect.
Cholestasis&cholestaticjaundiceareoccasionalsideeffectsofO.
C.pills.

1570.
NorgestimateinOCpillshasthe
followingadvantage?
a)Reducesvenousthrombosis
b)IscheaperthanstandardOCpills
c)Reducesacneandhirsutism
d)Usefulinheartdisease
CorrectAnswer-C
Ans.C.Reducesacneandhirsutism
Threenewerprogestogens,namelydesogestrel,gestodene,and
norgestimatecandecreasetheameanilhirsutismascomparedto
olderprogesterones,whichactuallycancauseoilyskinandacne.

1571.Whichofthefollowingisnotanideal
candidateforIUCDinsertion?
a)PreviousLSCS
b)Lactatingmother
c)AcutePID
d)Alloftheabove
CorrectAnswer-C
Ans.C.AcutePID

1572.Betamethasonegiventopretermpatient
forallexcept?
a)Fetallungmaturity
b)Decreaseintraventricularhemorrhage
c)Preventsperiventricularleukomalacia
d)PreventPPH
CorrectAnswer-D
Ans.D.PreventPPH
Steroids(dexamethasoneorbetamethasone)aregiventoenhance
fetallungmaturityandtheyalsodecreasetheincidence
intraventricularhemorrhage.

1573.A32-year-oldfemalewithmild
hypertension.Twodaysafternormal
delivery,shedevelopseizures,
headache.Noproteinuriawasthere.On
imagingshewasfoundtohave
parasagittalinfarctionandhematoma
3x2cm.Themostprobablecauseis?

a)Eclampsia
b)Superiorsagittalsinusthrombosis
c)Pituitaryapoplexy
d)Subarachnoidhemorrhage
CorrectAnswer-B
Ans.B.Superiorsagittalsinusthrombosis
Thevariousetiologiesforduralsinusthrombosisare:

1. Thrombophilia(factorVLeidenmutation,prothrombingene
mutation20210,deficienciesofantithrombin,proteinCandproteinS,
APLAsyndrome,hyperhomorysteinemia)
2. Pregnancy
3. Postpartumstate

1574.Penicillamineuseinpregnancyis
associatedwiththisfetalcomplication
?

a)Conradisyndrome
b)Renalanomalies
c)Thymushypolplasia
d)Cutislaxa
CorrectAnswer-D
Ans.D.Cutislaxa
Pencillamineinterfereswithsynthesisofcollagen&elastin&can
cause:elastosisperforansserpiginosaandlocalizedcutislaxa.

1575.Theprostaglandinmostcommonly
usedattermforinductionoflaboris?
a)PGI2
b)PGE1
c)PGE2
d)PGF2a
CorrectAnswer-C
Ans.C.PGE2
PGE2ismostcommonlyusedattermforinductionoflabor.

1576.HCGlevelsatwhichExpectant
managementofEctopicpregnancycan
bedone:

a)10000IU/L
b)1000IU/L
c)2500IU/L
d)5000IU/L
CorrectAnswer-B
Ans.B.1000IU/L
InitialHCGlevels<1000IU/l&subsequentlevelsarefalling.

1577.Outcomesofoccipitoposterior
position?
a)Deeptransversearrest
b)Occipitosacralarrest
c)Facetopubisdelivery
d)Alloftheabove
CorrectAnswer-D
Ans,D.Alloftheabove

1578.RMPcanperformMTPinfirsttrimester
ifhehasassistedinMTPs-
a)5
b)15
c)25
d)50
CorrectAnswer-C
Ans,C.25
Aregisteredmedicalpractitionershallhaveoneormoreofthe
followingexperienceortrainingingynecologyandobstetricsnamely
=
Ifhehasassistedaregisteredmedicalpractitionerinthe
performanceoftwenty-fivecasesofmedicaltermination
ofpregnancyofwhichatleastfivehavebeenperformed
independently,

1579.Cryingoffetusinuteroiscalledas?
a)Vagitusuterinus
b)Firstcry
c)Uterovaginalis
d)Vagitusvagina
CorrectAnswer-A
Ans,A.Vagitusuterinus

1580.Trueaboutcephalhematomais:
a)Crossesthesuturelines
b)Alwayspresentatbirth
c)Ventousedeliveryisariskfactor
d)Alloftheabove
CorrectAnswer-C
Ans.C.Ventousedeliveryisariskfactor
Theusualcausesofacephalohematomaareaprolongedsecond
stageoflabororinstrumentaldelivery,particularlyventouse.

1581.Innercellmassdifferentiatesinto?
a)Chorion
b)Trophoectoderm
c)Embryo
d)Alloftheabove
CorrectAnswer-C
ANs.C.Embryo
Blastocystenlarges&thezonapellucidaundergoeslysis,thisis
calledzonahatching.
Thecellsontheoutersidebecometrohoectodermwhich
differentiatesintochorion.
Thecellsontheinnersideforminnercellmasswhichdifferentiates
intoembrvo.

1582.Embryoiscalled"fetus"afterhow
manyweekspost
fertilization/conception?

a)6
b)8
c)10
d)12
CorrectAnswer-B
Ans.B.8
Embryonicperiodbeginsat3rdweekfollowingovulation/fertilization
&extendsupto8weekspostconception(10weeksfromLMP).
Fetalperiodbeginsafter8weekspostconception(10weeksfrom
LMP)&endsindelivery.

1583.Utero-placentalcirculationis
establisheddaysafterfertilization?
a)5
b)10
c)15
d)20
CorrectAnswer-B
Ans.B.10
TheuteroPlacentalcirculationisestablished9-10daysafter
fertilization.
Fetoplacentalcirculationisestablished2ldayspostfertilization.

1584.Pregnancyiscontraindicatedinallof
thefollowingexcept-
a)PrimaryPulmonaryHypertension
b)Eisenmenger'ssyndrome
c)Marfan'swithaorticrootdilation
d)WPWsyndrome
CorrectAnswer-D
Ans,D.WPWsyndrome

1585.Whichheartdiseasehastheworst
prognosis/maximummortalityin
pregnancy?

a)MS
b)AS
c)PDA
d)Eisenmenger'ssyndrome
CorrectAnswer-D
Ans.D.Eisenmenger'ssyndrome

1586.Ovariancyclecanbecorrelatedwithall
except?
a)Endometrialsampling
b)Vaginalcytology
c)Bloodhormonallevels
d)Estrouscycle
CorrectAnswer-D
Ans.D.Estrouscycle
Estrouscycledoesnotoccurinhumanbeings.

1587.Endometrialbiopsytodetectovulation
isdoneonwhichdayofthemenstual
cycle?

a)Day8-9
b)Day13-15
c)Day21-23
d)Day3-5
CorrectAnswer-C
Ans.C.Day21-23
EndometrialBiopsy:Rarelydonenowadaysforthepurposeof
iletectingovulation.

1588.Theuterinebloodflowattermis-
a)50mL/min
b)100-150mL/min
c)350-375mL/min
d)500-750mL/min
CorrectAnswer-D
Ans.D.500-750mL/min
Uteroplacentalbloodflowincreasesprogressivelyduringpregnancy
andrangesfrom5N)-800mL/minatterm.

1589.Godell'ssignis?
a)Duskyhueofthevestibule
b)Softeningofthecervix
c)Increasedpulsationsfeltthroughthelateralfornices
d)Regularandrhythmiccontractionsduringbimanualexamination
CorrectAnswer-B
Ans.B.Softeningofthecervix

1590.Infetus,insulinproductionbeginat
weeksofgestation-
a)4-6
b)8-12
c)14-18
d)24-28
CorrectAnswer-B
Ans.B.8-12
Reaching8to10weeksintodevelopment,thepancreasstarts
producinginsulin,glucagon,somatostatin,andpancreatic
polypeptide.

1591.Limbbudappearatwhatweeksof
gestation?
a)3
b)4
c)6
d)9
CorrectAnswer-B
Ans.B.4
Theupperextremityisfirstdiscretelyvisibleasabulgeorlimbbud
thatdevelopsontheventrolateralwalloftheembryoonday26(4-
mmcrown-to-rumplength).

1592.Featuresofnonsevere/mildpre
ecampsiaareallecxept-
a)DiastolicBP<100mmHg
b)SystolicBP<160mmHg
c)MildIUGR
d)Nopremonitorysymptoms
CorrectAnswer-C
Ans.,C.MildIUGR

1593.FERNINGisdueto?
a)Estogen&sodiumchloride
b)Progesterone&sodiumchloride
c)HCG
d)Alloftheabove
CorrectAnswer-A
Ans,A,Estogen&sodiumchloride
Cervicalmucusisrelativelyrichinsodiumchloridewhenestrogen
(butnotprogesterone)isbeingproduced.

1594.Secondwaveoftrophoblasticinvasion
occursatweeksofgestation?
a)8-11
b)10-12
c)12-15
d)16-20
CorrectAnswer-C
Ans,C.12-15
Thetimingofthedevelopmentoftheuteroplacentalvesselshas
beendescribedinwaves,orstages,overthecourseofgestation.
Thefirstwaveoccursbefore12weekspost-fertilizationandconsists
ofinvasionandmodificationofthespiralarteriesofthedecidua.
Between12and16weekspos-fertilization,thesecondworeoccurs.
Thisinvolvesinvasionoftheintramyometrialpartsofthespiral
arteries,convertingnarrowlumen,muscularspiralarteriesinto
dilated,low-resistanceuteroplacentalvessels.

1595.Abstinenceperiodbeforesemen
analysisis?
a)1-2days
b)3-5days
c)5-7days
d)7-9days
CorrectAnswer-B
Ans.B.3-5days
Theidealspecimenforexaminationisafter3-5daysofabstinence.

1596.Engagingdiameterinfacepresentation
is-
a)Suboccipitobregmatic
b)Mentovertical
c)Submentobregmatic
d)Occipitofrontal
CorrectAnswer-C
ANs,C.Submentobregmatic

1597.Leastlikelytocausedysmenorrhea-
a)Endometriosis
b)Adenomyosis
c)Uterinepolyp
d)Cervicalpolyp
CorrectAnswer-D
Ans,D.Cervicalpolyp

1598.LHsurgeisdueto?
a)Progesterone
b)Estrogen
c)AMH
d)Alloftheabove
CorrectAnswer-B
Ans,B.Estrogen
3-4daysbeforetheovulation,estrogenlevelcrossesacertainlimit
(thresholdlevel).

1599.Menopauseisdefinedas?
a)Presenceofhotflushes
b)Cessationofmensesfor1year
c)Cessationofmensesfor6months
d)Cessationofmensesfor2years
CorrectAnswer-B
Ans,B.Cessationofmensesfor1year
Menopauseisdefinedasthepermanentcessationofmensesfor7
yearandisphysiologicallycorrelatedwiththedeclineinestrogen
secretionresultingfromthelossoffollicular/ovarianfunction.

1600.Symptomsofmenopauseareallexcept
?
a)Hotflushes
b)Nightsweats
c)Decreaselibido
d)Intermittenthypotension
CorrectAnswer-D
Ans,D.Intermittenthypotension
Hotflushes
TheclassicsymPtomassociatedwithestrogendeficiencyisthehot
flash,alsoknownashotflush
Thissymptomisdescribedas'recurrent,transientperiodsof
flushing,sweatingandasensationofheat,oftenaccompaniedby
palpations,feelingofanxietyandsometimesfollowedbychills".

1601.MENOPAUSEisdiagnosedby?
a)Estradiol<20pg/ml
b)Progesterone<40ng/dl
c)FSH>40IU/L
d)LH>20IU/L
CorrectAnswer-C
Ans,C.FSH>40IU/L
AspertheAmericanAssociationofclinicalEndocrinologists,the
diagnosisofmenopauseisconfirmedbyFSHlevels>40IU/L.

1602.Thevelocityofspermis?
a)1-2cm/hr
b)2-4cm/min
c)1-4mm/min
d)1-4mm/hr
CorrectAnswer-C
Ans.C.1-4mm/min
Thevelocityofasperminfluidmediumisusually1-4mm/min.
Thisallowsthespermtomovetowardsanovuminordertofertilize
it.

1603.Afterejaculationsemenliquefiesin?
a)10minutes
b)30minutes
c)75minutes
d)120minutes
CorrectAnswer-B
Ans.B.30minutes
Followingejaculation,thesemenformsagelwhichprovides
protectionforthespermfromtheacidicenvironmentofthevagina.
Thegelisliquefiedwithin20-30minutebyenzymesfromtheprostate
gland.

1604.Duringpregnancy,truestatementabout
CVSis?
a)Cardiacoutputdecreases
b)Rightaxisdeviation
c)Increaseinleftventricularenddiastolicdiameter
d)Alloftheabove
CorrectAnswer-C
Ans,C.Increaseinleftventricularenddiastolicdiameter
2Decho:Increaseinleftventricularenddiastolicdiameter.
Increaseinleftandrightatrialdiameters.

1605.MCsiteofimplantationis?
a)Fallopiantubeamullarypart
b)Fallopiantubeisthmus
c)Fundusofuterus
d)Cornuofuterus
CorrectAnswer-C
Ans,C.Fundusofuterus
Implantationoccursintheendometriumontheanteriororposterior
wallofthebodynearthefundusonthesixthdayfollowing
fertilization(correspondingtothe20thdayofthemenstrualcycle).

1606.AlphasubunitofhCGissimilarto?
a)FSH
b)LH
c)TSH
d)Alloftheabove
CorrectAnswer-D
Ans,D.Alloftheabove
HumanchorionicGonadotropin(hcG)isstructurallyrelatedtothree
otherglycoproteinhormonesLH,FSH,andTSH.

1607.Placentalhormonewithhighest
carbohydratecontentis-
a)HCG
b)Humanpregnancyspecificbetaglycoprotein
c)HPL
d)Relaxin
CorrectAnswer-A
Ans,A.HCG
HCGhasthehighestcarbohydratecontentofanyhumanhormone-
30%.

1608. Thepelvicinletusuallyisconsideredto
becontractedifitsshortest
anteroposteriordiameterislessthan-
a)12cm
b)10cm
c)8cm
d)14cm
CorrectAnswer-B
Ans.B.10cm
Thepelvicinletusuallyisconsideredtobecontractedifitsshortest
anteroposteriordiameterislessthan10cmorifthegreatest
transversediameterislessthan12cm

1609.AzoospermiawithnormalFSHwould
indicate?
a)Hypothalamicfailure
b)Testicularfailure
c)Obstructionofvasdefrens
d)Alloftheabove
CorrectAnswer-C
Ans,C,Obstructionofvasdefrens

1610.Causesofmaleinfertility?
a)Idiopathic
b)Varicocele
c)Yq11microdeletion
d)Alloftheabove
CorrectAnswer-D
Ans,D.Alloftheabove

1611.Fetalthyroidglandisabletosynthesize
hormonesbyweeksofgestation?
a)6-7
b)7-8
c)10-12
d)12-14
CorrectAnswer-C
Ans,C.10-12
Fetalthyroidglandisabletosynthesizehormonesby10-12
weeksofgestation.

1612.PoorprognosisinfirsttrimesterUSGis
?
a)Nofetalpoleat5weeks
b)Nocardiacactivityat5weeks
c)Nogestationalsacat4weeks
d)Nocardiacactivityat8weeksofgestation
CorrectAnswer-D
Ans,D.Nocardiacactivityat8weeksofgestation
Absenceofembryowithheartbeat22weeksafterascanthat
showedagestationalsacwithoutayolksac

1613.Allaretrueaboutpostpartum
depressionexcept-
a)Symptomsresolvein10-12days
b)Affectsbothsexes
c)SSRIsareeffective
d)Noneoftheabove
CorrectAnswer-A
Ans,A.Symptomsresolvein10-12days

1614.NerveinjuredinMcRobertsmaneuveris
?
a)Lumbosacraltrunk
b)Obturatornerve
c)Femoralnerve
d)Pudendalnerve
CorrectAnswer-C
Ans,C.Femoralnerve
McRobertsmaneuveristobedoneincasesofshoulderdystocia.
Whenthematernalthighsaremarkedlyflexedandabducted,
pressurefromtheoverlyinginguinaliigamentmayleadtofemoral
nerveinjury.

1615.Mostconclusiveclinicalsignof
pregnancyis?
a)Uterineenlargement
b)Cervicalsoftening
c)Amenorrhea
d)Fetalheatsoundauscultation
CorrectAnswer-D
Ans.D.Fetalheatsoundauscultation
FHSauscultationisthemostconclusiveclinicalsignofpregnancy.

1616.CRLwhencardiacactivitycanbe
detectedearliestbyTVS-
a)1-4mm
b)1cm
c)6-7mm
d)2-4cm
CorrectAnswer-A
Ans,A.1-4mm
Fetalheartbeatcanbedetectedasearlyasjustunder6weeks
gestationongoodquality,highfrequencytransvaginalultrasound,as
acrownrumplength(CRL)ofaslittleas1-2mm.

1617.Kamla,30yearsold,P2L2with3.2x4.1
cmfibroiduterus,complainsof
menorrhagiaandisonsymptomatic
treatmentsince6months.Thepatient
refusessurgery.Nextlineof
managementis?

a)GnRHanalogs
b)Danazol
c)Myomectomy
d)Uterinearteryembolization
CorrectAnswer-D
Ans,D.Uterinearteryembolization
UAEcanbeusedasatherapyforsymptomaticpatientswhorefuse
orwanttoavoidsurgery.
Afterembolization,thereis60-65%decreaseinsizeoffibroidsover
aperiodof6-9months,andsothepatient'ssymPtomsmay
decreaseordisaPPear.
Ifthepatientisstillsymptomaticafter1year,thensurgeryshouldbe
considered.

1618.Livingligatureoftheuterusis?
a)Endometrium
b)Middlelayerofmyometrium
c)Innerlayerofmyometrium
d)Parametrium
CorrectAnswer-B
Ans,B.Middlelayerofmyometrium
Middlecrisscrossfibresactaslivingligatureduringinvolutionofthe
uterusandpreventbloodloss.

1619.Inpartogramsrecommendedby'WHO'
thedistancebetweenthealertand
actionlinesis?

a)1hour
b)2hours
c)4hours
d)5hours
CorrectAnswer-C
Ans.C.4hours
Theconceptofalertline'and`actionline'wasintroducedbyPhilpott
andCastlein1972.
Theactionlinecanbeplacedat2?4hoursintervaltotherightand
paralleltoalertline.
Inpartogramsrecommendedby'WH0'thedistancebetweenthealert
andactionlinesis4hours.

1620.Surgicalexcisionofcorpusluteum
beforeweeksofgestation,resultsin
miscarriage

a)6-7
b)9-11
c)11-12
d)12-14
CorrectAnswer-A
Ans,A,6-7
Surgicalexcisionofcorpusluteum(luteoctomy)before7weeksof
gestation,uniformlyprecipitatedanabruptdecreaseinserum
progesteroneconcentrationfollowedbymiscarriage

1621.Factorsresponsiblefordevelopmentof
OHSSinclude?
a)Histamine
b)Cytokines
c)VascularEndothelialGrowthFactor
d)Alloftheabove
CorrectAnswer-D
Ans,D.Alloftheabove
VEGFisconsideredtobethemostimportant.

1622.MVAsyringeisusedfor?
a)FirsttrimesterMTP
b)2ndtrimesterMTP
c)Vacuumdelivery
d)Alloftheabove
CorrectAnswer-A
Ans,A,FirsttrimesterMTP
Manualvacuumaspiration(MvA)isasafeandeffectivemethodof
abortionthatinvolvesevacuationoftheuterinecontentsbytheuse
ofahand-heldplasticaspirator.
Itisappropriatefortreatmentofincompleteabortionforuterinesizes
upto12weeksfromthelastmenstrualperiod(including
miscarriage,spontaneousabortionandremovalofretainedproducts
fromaninducedabortion),first-trimesterMTPandendometrial
biopsy.

1623.Diihrssenincisionistakenon-
a)Fallopiantube
b)Ovary
c)Incompletelydilatedcervix
d)Fullydilatedcervix
CorrectAnswer-C
Ans,C,Incompletelydilatedcervix
Occasionallnespeciallywithsmallpretermfetuses,theincompletely
dilatedcervixwillnotallowvaginaldeliveryoftheafter-cominghead
ofthebreech.
Insuchcases,Duhrssenincisionsareusuallynecessary(cutthe
cervixatl0and2o'clockpositions).

1624.A27-year-oldfemalewithplacenta
previahadseverebleeding.Whatisthe
mostlikelyoutcomepostdelivery?

a)Galactorrhea
b)Diabetes
c)Absenceofmenstrualcycle
d)Cushingsyndrome
CorrectAnswer-C
Ans.c.Absenceofmenstrualcycle
Sheehansyndrome,alsoknownaspostpartumhypopituitarismor
postpartumpituitarynecrosis,ishypopituitarismcausedbynecrosis
duetobloodlossandhypovolemicshockduringandafterchildbirth.
MostcommoninitialsymptomsofSheehansyndromeare
agalactorrhea(absenceoflactation)underdifficultieswithlactation.
Manywomenalsoreportamenorrheaoroligomenorrheaafter
delivery.

1625.Womanhas100mlbloodlossevery30
days.Thisiscalledas?
a)Menorrhagia
b)Polymenorrhea
c)Hypomenorrhea
d)Normalmenses
CorrectAnswer-A
Ans.A.Menorrhagia
Normalbloodlossduringmensesisaround35ml(20-80ml)
Bloodlossmorethan80mlismenorrhagia

1626.Menometrorrhagiais?
a)Heavyperiods
b)Intermenstrualbleeding
c)Heavy&irregularbleeding
d)Uterinebleedingoccurringatregularintervalsoflessthan21
days
CorrectAnswer-C
Ans.C.Heavy&irregularbleeding

1627.Prevalenceofbreechpresentationat
fulltermis?
a)10%
b)6-7%
c)3-4%
d)1-2%
CorrectAnswer-C
Ans.C.3-4%

1628.Theshortestconjugateis?
a)Trueconjugate
b)Obstetricconjugate
c)Diagonalconjugate
d)Anatomicalconjugate
CorrectAnswer-B
Ans.B.Obstetricconjugate
Obstetricconjugate(10cm)
DistancebetweenmidpointofsacralPromontorytotheprominent
bonyprojectioninthemidlineoninnersurfaceofsymphysispubis.

1629.Shortesttransversediameteris?
a)BPD
b)Bitemporaldiameter
c)Bimastoiddiameter
d)Allareequalinlength
CorrectAnswer-C
Ans.C.Bimastoiddiameter
Bimastoiddiameter=7.5cm
Occipitofrontaldiameter11.5cm

1630.Withreferencetofetalheartrate,a
nonstresstestisconsideredreactive
when?

a)Twofetalheartrateaccelerationsarenotedin20minutes
b)Onefetalheartrateaccelerationisnotedin20minutes
c)Twofetalheartrateaccelerationsarenotedin10minutes
d)Threefetalheartrateaccelerationsarenotedin30minutes
CorrectAnswer-A
Ans.A.Twofetalheartrateaccelerationsarenotedin20
minutes
Reactive(Reassuring)NST
Twoormoreaccelerationsof>15beats/minuteabovethebaseline,
lastingfor>75secondsarcpresentin20-4Ominutesobservation
period.

1631.Variabledecelerationisseenin?
a)Headcompression
b)Uteroplacentalinsufficiency
c)Cordcompression
d)Noneoftheabove
CorrectAnswer-C
Ans,C.Cordcompression
Variabledecelerationsareduetocordcompression
(oligohydramniosinlabor)

1632.ModifiedBIOPHYSICALPROFILEis?
a)NST+FETALTONE
b)FETALTONE+AFI
c)NST+AFI
d)NST+FETALTONE+AFI
CorrectAnswer-C
Ans,C.NST+AFI
ModifiedBPP=NST&AFI
BPPhas5components

1633.Iffetusishavinghypoxia,whichofthe
BPPparameterwillbeaffectedlast?
a)Fetaltone
b)Fetalbreathingmovement
c)Fetalmovements
d)NST
CorrectAnswer-A
Ans.A.Fetaltone
Firstactivitytoappear,FetalToneatabout7.5-8.5weeks,isalso
presumablythelastactivitytodisappearwithprogressively
worseninghypoxia.

1634. Testusedtodetectgeneticabnormality
inembryo,beforetransferringittothe
uterusinIVFis?
a)Embryocellbiopsy
b)CVS
c)ICSI
d)Alloftheabove
CorrectAnswer-A
Ans,A.Embryocellbiopsy
PGD,involvesremovingacellfromanIVFembryototestitfora
specificgeneticcondition(cysticfibrosis'forexample)before
transferringtheembryototheuterus.

1635.InIVF,embryosaretransferredbackto
uterinecavityatcellsstage?
a)2
b)2-4
c)4-8
d)8-16
CorrectAnswer-C
Ans,C.4-8
Typicallyembryosatetransferredatthecleavagestage(Day2or3
afteroocyteretrieval).
Daythreeembryosatecalledcleavagestageembryosandhave
approximately4-8cells.

1636.Azoospermicpatientcanbeafatherof
achild,bywhichofthefollowing?
a)IUI
b)ZIFT
c)ICSI
d)Notpossible&counselregardingadoption
CorrectAnswer-C
Ans.C.ICSI
PESA=percutaneousepididymalspermaspiration
MESA=microscopicepididymalspermaspiration
TESA=testicularspermasPiration
TESE=testicularspermextraction(testicularbiopsy)

1637.Aprimigravidawith36weeksof
pregnancyisinlaborwith3cm
dilatationandminimaluterine
contraction.Onruptureofmembranes,
freshbleedingisnotedwithlatefetal
decelerationupto50beats/min.The
patientwastakenforLSCSbutfetus
couldnobesaved.Noabruptioor
placentapreviawasseen.Thelikely
diagnosisis?

a)Placentaprevia
b)Revealedabruptio
c)Circumvallateplacenta
d)Vasaprevia
CorrectAnswer-D
Ans.D.Vasaprevia
Vasaprevia(l:2500)isarareconditioninwhichfetalbloodvessels
areinfrontofthepresentingpartandcrossthecervix.
Theconditionhasahighfetalmortalityrate(50-95%).Thisis
attributedtorapidfetalexsanguination,resultingfromthevessels
tearingwhenthecervixdilates,membraneruPture.

1638.AllarecomponentsofActive
ManagementoftheThirdStageof
Laborexcept?

a)Uterotonicagentwithin1minuteofbirth
b)Massageofuterusbeforecontrolcordtraction
c)Controlcordtraction
d)Noneoftheabove
CorrectAnswer-B
Ans.B.Massageofuterusbeforecontrolcordtraction
Administeranuterotonicdrugatthedeliveryoftheanteriorshoulder
orafterwards,withinoneminuteofthebaby'sbirth.
BeforeperformingAMTSL,gentlypalpatethewoman'sabdomento
ruleoutthepresenceofanotherbaby.Atthispoint,donotmassage
theuterus.
Performcontrolledcordtraction
Massagetheuterusimmediately

1639.Withwhichofthefollowingevents,the
fetomaternalhaemorrhageriskisthe
least?

a)Amniocentesis
b)Cordocentrsis
c)Chorionicvillussampling
d)Abruption
CorrectAnswer-D
Ans.D.Abruption

1640.Infertilityisdefinedas?
a)Inabilitytoconceiveafter1yearofregularunprotected
intercourse
b)Inabilitytoconceiveafter1yearofmarriage
c)Inabilitytoconceiveafter2yearsofmarriage
d)Inabilitytoconceiveinspiteof2yearsofregularunprotected
intercourse
CorrectAnswer-A
ANS.A.Inabilitytoconceiveafter1yearofregularunprotected
intercourse
Infertilityisdefinedasaninabilitytoconceiveinspiteof1yearof
regularunprotectedintercourse

1641.Cordprolapseisleastlikelywith-
a)Transverselie
b)Footlingbreech
c)Oligohydroamnios
d)Floatinghead
CorrectAnswer-C
Ans.C.Oligohydroamnios
Cordprolapsehasbeendefinedasthedescentoftheumbilicalcord
throughthecervixalongside(occult)orpastthepresentingpart
(overt)inthevaginaoroutsidethevulvainthepresenceofruptured
membranes

1642. IdealtimetodoGlucosechallengetest
inpregnancyis?
a)12-16weeks
b)20-24weeks
c)24-28weeks
d)30-34weeks
CorrectAnswer-C
Ans.C.24-28weeks
O'sullivanBloodSugarScreeningTest(GlucoseChallengeTest)
Theidealtimetodothistestis24-28weeksofgestation(asinsulin
resistanceinpregnancyismaximumat28weeksofgestation)

1643.AfterIUFD,whendoesthemother
developDIC-
a)48hours
b)1-2weeks
c)3-4weeks
d)6weeks
CorrectAnswer-C
Ans.C.3-4weeks
Thromboplastinfromthedeadfetuscanenterthematernalsystem
andcauseDIC.
Thisonlyhappenswhenthedeadfetusisretainedinsidefor3-4
weeks.

1644.Folicacidrequiredinfirsttrimesterof
normalpregnancy-
a)100microgram
b)400-500microgram
c)4mg
d)5mg
CorrectAnswer-B
Ans,B,400-500microgram
SomeNTDsareassociatedwithaspecificmutationinthe
methylenetetrahydrofolatereductasegene,theadverseeffectsof
whichcanbelargelyovercomebypericonceptionalfolicacid
supplementation.
MorethanhalfofNTDscouldbepreventedwithdailyintakeof400
microgramoffolicacidthroughoutthepericonceptionalperiod.
Awomanwithapriorpregnancycomplicatedbyaneuraltubedefect
canreducethe23%recurrenceriskbymorethan70%ifshetakes4
mgoffolicacidforthemonthbeforeconceptionandforthefirst
trimesterofpregnancy.

1645.A30-year-oldis14weekspregnant.She
hadtwopainlessdeliveriesat16weeks
earlier.Nextlineofmanagementis?

a)Cervicalencerclage
b)Evaluationfordiabetesmellitusandthyroiddisorders
c)Cervicallengthassessment
d)Tocolytics
CorrectAnswer-C
Ans,C.Cervicallengthassessment
Thepatienthadtwopainlessabortionsat16weeksinthepast,so
mostlyitisacaseofincompetentos.
Nextlineofmanagementinthesepatientsisfrequentcervicallength
assessment:clinicallyorbyUSG.
Thepatientisevaluatedmorefrequentlyandifthecervixisshort
(lessthan2.5cm)thancervicalencerclagehastobedone.
Cervicalencerclageisthesurgeryofchoiceforincompetentos,but
thesurgeryitselfcanleadtocomplicationssuchasuterine
contractions,abortions,andPROM.
Sothesurgeryisonlytobedoneifitisindicated.

1646.Uppertwo-thirdanteriorvaginalwall
prolapseis?
a)Cystocele
b)Urethrocele
c)Rectocele
d)Enterocele
CorrectAnswer-A
Ans.A.Cystocele

1647.EarliestsignafterIUFDis?
a)Overlappingofskullbones
b)Hyperflexionofspine
c)Gasingreatvessel
d)Overcrowdingofribs
CorrectAnswer-C
Ans.C.Gasingreatvessel
Robertsign(gasingreatvessels)-12hrsafterdeath.

1648.TrueaboutGartnerscystis?
a)RetentioncystinremnantsofWolffianduct
b)Arisesfrommullerianduct
c)Commonlyarisesfromcervix
d)Impulseoncoughing
CorrectAnswer-A
Ans,A.RetentioncystinremnantsofWolffianduct

1649.Iftheanalspincterisinjured,itiswhich
degreeofPerinealTear?
a)First
b)Second
c)Third
d)Fourth
CorrectAnswer-C
Ans,C.Third
PerinealTearsareclassifiedintofourcategories
First-degreetear:lacerationislimitedtothefourchetteand
superficialperinealskinorvaginalmucosa.
Second-degreetear:lacerationextendsbeyondfourchette,
perinealskinandvaginalmucosatoperinealmusclesandfascia,but
nottheanalsphincter.
Third-degreetear:fourchette,perinealskin,vaginalmucosa,
muscles,andanalsphincteraretorn

1650.Iftherectalmucosaisinjured,itis
whichdegreeofPerinealTear?
a)First
b)Second
c)Third
d)Fourth
CorrectAnswer-D
Ans,D.Fourth
Rectalmucosaltearisfourthdegreetear.

1651.Cryptomenorrheaoccursin?
a)Fibroids
b)PCOS
c)Imperforatehymen
d)Alloftheabove
CorrectAnswer-C
Ans,C.Imperforatehymen
Congenital
Imperforatehymen:Itisduetofailureofdisintegrationofthecentral
cellsofMullerianeminencethatprojectintourogenitalsinus
TransversevaginalsePtum
Atresiaofvagina,cervix.

1652.Bestindicatorforovarianreserveis?
a)AMH
b)LH/FSHratio
c)FSH
d)Estradiol
CorrectAnswer-A
Ans.A.AMH
AMHbloodlevelsarethoughttoreflectthesizeoftheremainingegg
supplyor"ovarianreserve"

1653.HPLhasactivitysimilartowhich
hormone?
a)Oxytocin
b)Growthhormone
c)Insulin
d)Alloftheabove
CorrectAnswer-B
Ans.B.Growthhormone
Humanplacentalactogen(hPL)wasnamedso,becauseofits
potentlactogenic&growthhormonelikebioactivityaswellas
immunochemicalresemblancetohumangrowthhormone'

1654.Whichofthefollowingisnotasoft
tissuemarkerofDownsyndromeon
USG?

a)IncreaseNT
b)Absentnasalbone
c)Exomphalos
d)Polydactyly
CorrectAnswer-D
Ans.D.Polydactyly

1655.Inacaseofrecurrentspontaneous
abortionthefollowinginvestigationis
unwanted?

a)Hysteroscopy
b)Testingforantiphospholipidantibodies
c)TestingforTORCHinfections
d)Thyroidfunctiontests
CorrectAnswer-C
Ans.C.TestingforTORCHinfections
TestingforTORCHinfectionsisnowthoughttobeunwarranted.

1656.Dilatation&evacuationisdoneforall
ecxept?
a)Inevitableabortion
b)Incompleteabortion
c)Threatenedabortion
d)Noneoftheabove
CorrectAnswer-C
Ans.C.Threatenedabortion
Inevitableabortionmeanstheprocessofexpulsionofproducts
ofconceptionhasbecomeirreversible.Theexpulsionofproductsof
conceptionhasnotoccurredbutitisboundtohappenandnothing
canbedonetostopthisprocess.
Whentheentireproductsarenotexpelled,partofitisleftinsidethe
uterinecavity,itiscalledincompleteabortion.

1657.WHOnormalHbvalueforanon
pregnantadultfemaleis?
a)10gm/di
b)11gm/dl
c)12gm/dl
d)13gm/dl
CorrectAnswer-C
Ans.C.12gm/dl

1658.Whichofthefollowingepidermallayer
isdeadlayer?
a)Stratumbasale
b)Stratumspinosum
c)Stratumcorneum
d)Stratumgranulosum
CorrectAnswer-C
Ans.C.Stratumcorneum
Stratumcorneum(Hornylayer):
Thisisthemostsuperficiallayerofepidermisandskin.
Cellsarefullykeratinizedandendupasanucleatedeadcells.
Therefore,stratumcorneumisdeadlayer.
Stratumcorneumislasttodevelop4Thereforeinpremature
newbornitisabsent.

1659.Langerhanscellareseeninwhichlayer
ofskin?
a)Stratumbasal
b)Stratumcarneum
c)Stratumgranulosum
d)Stratumspinosum
CorrectAnswer-D
Ans.D.Stratumspinosum
[RefVenkataram151/ep.2]
Langerhanscells
Thesecellsarefoundinstratumspinosumandfunctionas
epidermalmacrophages(Antigenpresentingcells).
Thesecellscontaincharacteristictennisracquetshapedgranules
(Birbeckgranules).

1660.Increaseinthethicknessoftheprickle
celllayeroftheepidermisiscalled?
a)Spongiosis
b)Acanthosis
c)Hypergranulosis
d)Hyperkeratosis
CorrectAnswer-B
Ans.B.Acanthosis
[RefRook's7th/ep.7.36]
Importantterminologyrelatedtoepidermallayers
Separationofkeratinocytesduetolossofintracellularbridges-
Acantholysis
IntracellularedemaofkeratinocytesBallooning

1661.Substancecommoninskinandhairis?
a)Keratin
b)Laminin
c)Nectin
d)Vimentin
CorrectAnswer-A
Ans.A.Keratin
[Ref:IADVL3rd/ep.12]
Keratinisafamilyoffibrousstructuralproteins.
Keratinistheproteinthatprotectsepithelialcellsfromdamageor
stress.
Keratinisthekeycomponentofourskin,hairandnails.
Itsproteinbuildingblockshaveaminoacidchains,coiled,cross-
linkedandclassifiedaseitherhardorsoft.

1662.Whichofthefollowingisamelanising
agent?
a)Methoxsalen
b)Dapsone
c)Minocycline
d)Kojicacid
CorrectAnswer-A
Ans.A.Methoxsalen
[RefInternet&IADVL3'/ep.756]
Melanizingagentsaredrugsthatincreasesensitivitytosolar
radiationandpromotere-pigmentationofde-pigmentedareasof
skin.
Melanizingagentssensitizetheskintosunlight.Asaresult,
erythema,inflammationandpigmentationoccurs.

1663.Fordycespotsinvolve?
a)Penis
b)Tongue
c)Fingers
d)Nails
CorrectAnswer-A
Ans.A.Penis
[RefIADVL3"1/ep.1779]
Fordycespots(FordycegranulesorFordycedisease):
Fordyce'sspotrepresentsectopicsebaceousglandsonlips(most
commonsite)andoralmucosa.
Theymayalsoappearonvulvaandpenis,wheretheyarecalled
Tyson'sgland,i.e.,ectopicsebaceousglandatpenis(prepuce)and
vulvaiscalledtyson'sgland.
Theseglandshavehistopathologysimilartonormalsebaceous
glands,despitetheirectopiclocation.

1664.Mostcommonetiologyoferythema
multiformeis?
a)Idiopathic
b)Drugs
c)HSV
d)TB
CorrectAnswer-A
Ans.A.Idiopathic
[Ref:NeenaKhannaYalep.61,63]
Erythemamultiforme
Mostofthecasesoferythemamultiformeareidiopathic,but
amongstthecausativeagentsHerpessimplexvirusisthemost
importantcause.
Erythemamultiformeisanacute,oftenselflimitederutption
characterizedbyadistinctiveclinicaleruption,thehallmarkofwhich
istheTargetlesion(IrislesionorBulle'seyelesion).

1665.Hypopigmentedmaculesarefoundin?
a)Addison'sdisease
b)Porphyria
c)Cutaneousmastocytosis
d)Tuberoussclerosis
CorrectAnswer-D
Ans.D.Tuberoussclerosis
[RefRook's7th/ep.17.37-17,39.58-39.59;Behl10thiep.154]
Causesoflocalizedhypopigmentation(Macule/Patch):
PrimaryCutaneousdisorders:
Vitiligo
Pityriasisversicolor
Pityriasisalba
Postinflammatory
Nevusdepigmentosus(achromicus)
Nevusanemicus
Piebaldism
Chemicalleukoderma
IdiopathicguttateHypomelanosis
Systemicdiseases:
Scleroderma
Tuberoussclerosis
Sarcoidosis
CutaneousT-celllymphoma
Leprosy(tuberculoid&Indeterminate)
Onchocerciasis
HypomelanosisofIto
Incontinetiapigmenti(StageIV)

Vogt-Koyangi-Haradadisease

1666.Whichofthefollowingdiseaseis
closelyrelatedtoenetropathy?
a)LinearIgAdisease
b)Pemphigusfoliaceous
c)Dermatitisherpetiformis
d)Erythemamultiforme
CorrectAnswer-C
Ans.C.Dermatitisherpetiformis
[Ref:Behl10m/ep.293]
Dermatitisherpetiformisisadiseaseoftheskincausedbythe
depositionofIgAinpapillarydermisandalongtheepidermal
basementmembranezone(Dermoepidermaljunction).
Almostallpatientsofdermatitisherpetiformishaveanassociated
glutensensitiveenteropathy.

1667.ResidualHypopigmentationfollowinga
drugreaction,isbestknownas?
a)Vitiligo
b)Chemicalleukoderma
c)Postinflammatoryhypomelanosis
d)Piebaldism
CorrectAnswer-C
Ans.C.Postinflammatoryhypomelanosis
[RefIADVL3"1/ep.747-748]
Amonggivenoptions,postinflammatoryhypomelanosisisthebest
answer.
understandletushavealookatthedefinitionfewimportantterms.
Vitiligoisalsoanacquiredconditionwithlossofpigmentationbut
mostcasesareidiopathic.
Itismainlyconsideredtobeanautoimmunecondition.
Precipitatedbyhormonalchanges,acuteemotionaltraumaorstress
oranyconditionleadingtoimmuneimbalance.
So,anyhypomelanosisresultingafterinflammationisknownaspost
inflammatoryhypomelanosis(includingleucoderma).
Sometimesboththetermsareusedsynonymously.
Whereastheterm'chemicalleucoderma'isusedonlywhenthere
repeatedexposure(contact)tosomespecificchemical.
Thereforeamonggivenoptions,postinflammatoryhypomelanosisis
thebestanswerasdrugreactionisaninflammatorycondition
leadingtoresidualhypopigmentation.

1668.Mostcommonpatternof
onychomycosisis?
a)Distalandlateralsubungual
b)Proximalsubungual
c)Whitesuperficial
d)Totaldystrophic
CorrectAnswer-A
Ans.A.Distalandlateralsubungual
[RefIADVL3fli/ep.266]
Distalandlateralsubungual:
DSOisthemostcommonform(90%)ofonychomycosis.
Characterizedbyinvasionofthenailbedandundersideofthenail
platebeginningatthehyponychiumandmigratesproximallythrough
theunderlyingnailmatrix
DSOisusuallycausedbythedermatophyteT.rubrum.
Infectionofthetoenailsbeingmuchmorecommonthaninfectionof
thefingernails

1669.Whichofthefollowingisnotafeature
ofdermatomyositis?
a)'V'sign
b)Holstersign
c)Pokiloderma
d)Groovesign
CorrectAnswer-D
Ans.D.Groovesign
[Ref:Rooks7th/ep.127-38;IADVL3'/ep.1236]
Groovesign(adepressionalongthecourseofaveinorbetween
musclegroups)isseenindeepmorphea.
Cutaneoussignsoddermatomyositis
1. Gottron'spapules:lilacorviolaceouspapulesonknucklesand
dorsaofhands
2. Gottron'ssign:violaceouserythemawithedemaovershoulders,
armsandforearms
3. Heliotropesign:violaceouserythemawithedemaovereyelids,
periorbitalregion
4. Poikiloderma:atrophyofskin,hypopigmentation,dialaltedblood
vesselsovertrunk
5. Mechanichand:symmetricalhyperkeratosisonulnaraspectof
thumbandradialaspectoffingers
6. Shawlsign:violaceouserythemaextendingfromdorsolateral
aspectofhands,forearmsandarmstoshouldersandneck.
7. 'V'sign:violaceouserythemainaVshapeddistributionover
anteriorneckandchest.
8. Holstersign:b/1symmetrical,patchymacularviolaceouserythema
displayibgareticuloidorlevidoidarrayoverthelateralaspectof

upperthighandhips.
9. Calcinosiscutis:calciumdepositsinskin(injuvenilevariant)
10. Miscellaneoussigns:Photosensitivity,vasculitis,panniculitis,Nail-
foldtelangiectasia

1670.Notafeatureofcandidalintertrigois?
a)Obesityisariskfactor
b)C.albicansisthemostcommoncausativespecies
c)Centralscaling
d)Satellitelesions
CorrectAnswer-C
Ans.C.Centralscaling
[RefNeenaKhanna's4th/ep.294]
Candidalintertrigo
Infectionofskinfoldsiscalledcandidalintertrigo.Itischaracterized
byreddened(erythematous)plaques,withsatellitepustules
(peripheralpustules).Theremaybefinescalesatperiphery.
Overtheregionscoveredwiththickstratumcorneum,i.e.,toewebs
andFingerwebs,thelesionappearasmoist,whitecoloredplaques.
Virtuallyanybodyfoldmaybeaffected.Groins,axillae,
submammaryfoldsinfemales,toes&fingerwebsarecommonsites
ofaffection.
Lesionspersistingforalongtimeleadtothedevelopmentof
superficialpainfulerosionsthattakealongerperiodtoheal.Thisis
called"erythemaoflacquer.

1671.Scalingisnotfeatureofwhichofthe
following?
a)Tinea
b)Lichenplanus
c)Herpeszoster
d)Reiter'sdisease
CorrectAnswer-C
Ans.C.Herpeszoster
[RefBeh110th/ep.254-268;NeenaKhanna3"Yep.37]
Inherpesthereiscrustingbutnoscaling.
Papulo-squamousdisordershavefollowingtwocharacteristics:-
Papule(solidelevatedskinlesion<1cms)orplaques(solid
elevatedlesion>1cm).ii)Scales(visibleexfoliationoftheskin
whichrepresntsvisiblesheddingofskin).
"Papulosquamousdisordersmanifestpapulessurmountedby
scales".

1672.A40yearsoldmalepatientpresents
withmultipleerythematousannular
lesionswithperipheralscalesarranged
predominantlyontrunk.Treatmentof
choiceis?

a)Topicalsteroids
b)Systemicsteroids
c)SystemicAzathioprine
d)Topicalantifungal
CorrectAnswer-A
Ans.A.Topicalsteroids
[Ref:Beh110`Vep.263]
Pityriasisrubrapilaris
Clinicalpresentationofthepatientinquestionsuggeststhe
diagnosisofPRPforwhichtreatmentofchoiceistopicalsteroids
andsalicylicacid.
Treatment
Localizedlesions-Topicalcorticosteroids+Keratolytics(Salicylic
acid,urea)
Erythroderma-VitaminA,Acitretin(Retinoids),oralmethotrexate.

1673.Woronoff'sringisafeatureof?
a)Psoriasis
b)Lichenplanus
c)Pityriasisrosea
d)Pemphigus
CorrectAnswer-A
Ans.A.Psoriasis
[RefRooks7"Vep.35.1-35.63]
ClinicalfeaturesofPsoriasis(psoriasisvulgaris)
Psoriasisoccursatallages,mostpatientsareyoungormiddleaged
adults.
Thetypicallesionisnummularroundplaquewhichhas
followingcharacteristicfeatures:
Welldefined
Profuse,silverywhite,powderyscales(Candledropscales)-Losely
adherentandeasilydrops.
Brightrederythmatousbase.
PlaqueisoftensurroundedbyahypopigmentedhaloRingof
Woronoff.

1674.Mostimportantfactorincausationof
Ingrowntoenailis?
a)Fungalinfection
b)Illfittingshoes
c)Geneticpredisposition
d)Nutritionaldeficiency
CorrectAnswer-B
Ans.B.Illfittingshoes
Ill-fittingshoes:
Ingrowntoenail(unguisincarnates/onychocryptosis)
Itisacommonpainfulconditioninwhichthenailgrowssothatit
cutsintooneorbothsidesoftheparonychiumornailbed.
Whileingrownnailscanoccurinthenailsofboththehandsandthe
feet,theyoccurmostcommonlywiththetoenails.
Presentswithpain,tenderness,rednessandswellingalongoneor
bothsidesoftheaffectednail.

1675.Whichofthefollowingisnotafeature
oflichenplanus?
a)Pterygium
b)Spontaneoushealing
c)Scarringalopecia
d)Notpremalignant
CorrectAnswer-D
Ans.D.Notpremalignant
[RefBehl/ep.265;Rooks7th/ep.5.13]
Veryrarelychroniculcerativelesionsmaydevelopmalignant
changes,i.e.,squamouscellcarcinoma.Clinicalfeaturesoflichen
planus.

1676.Whichofthefollowingisnottruefor
scabies?
a)Wristiscommonsiteinchildren
b)Burrowsareintradermallesions
c)Papulesandpustulesareduetohypersensitivitytomite
d)Itchinggeneralized
CorrectAnswer-B
Ans.B.Burrowsareintradermallesions
[RefBehl10th/ep.179]
Burrowistheserpentine(S-shaped),threadlikegray-brownline
whichrepresenttheintraepidermaltunnelcreatedbythemoving
femalemiteinstratumcorneum.
BurrowisPathognomicofscabies.
Burrowsaredifficulttodemonstrateininfant.
Clinicalfeaturesofscabies
1.Severeitchingisthemostprominentsymptom,andhasfollowing
characteristicfeatures:
Worseatnight
Generalized
Affectingseveralfamilymembers
2.InScabies,severeitchingtypicallyworsenatnight,mostnotably
alongthewebspacesoffingers,wrists,elbows,axillaeandgroin
area-AreasknowascircleofHebra.
3.Papulesandpapulovesiclesduetohypersensitivitytomite.
4.Pustulescanoccurduetosecondaryinfection.
5.Excoriationandscratchmarks.
6.Historyofinvolvementoffamilymember

1677.Scabiesiscausedby?
a)Mite
b)Tic
c)Virus
d)Fungus
CorrectAnswer-A
Ans.A.Mite
[RefBehl10thiep.179]
ScabiesisanintenslypruriticskininfestationcausedbySarcoptes
scabiei,anacarus(mite).
Scabiesusuallyaffectschildrenbutcanoccuratanyage.
Morecommoninlowsocioeconomicstrataasovercrowdingand
poorhygienefacilitatetransmission.
Themostimportantmeansoftransmissionisviadirectcontactwith
aninfectedindividual.
Scabesisawaterwasheddiseasewhichoccursduetoinadequate
useofwaterorimproperhygiene.

1678.Seleniumsulphideisusedinthe
treatmentof?
a)Scabies
b)T.versicolor
c)T.cruris
d)Cutaneousleishmaniasis
CorrectAnswer-B
Ans.B.T.versicolor
[RefHarrison17th/ep.318]
Seleniumsulphide(2.5%)indetergentbaseisappliedalloverbody
belowneck(sparingthegenitalia),leftovernightandwashedofnext
morning.
Twotothreeapplicationsappliedonceortwiceaweekusually
clearstheinfection.
TreatmentofP.versicolor
1. Systemicagents:-azolessuchasketoconazole,fluconazoleor
itraconazole.
2. Topicalagents:
3. Azoles-clotrimazole,econazole,miconazole,ketoconazole.
4. Others-seleniumsulfide(2.5%),sodiumthiosulphate(20%),
whitfield'sointment(3%Salicylicacid+6%Benzoicacid),zinc
pyrithione(1%),tolnaftate,ciclopiroxolamine.

1679.Firstgenerationtopicalretinoidis?
a)Retinoicacid
b)Adapalene
c)Tazarotene
d)Acitretin
CorrectAnswer-A
Ans.A.Retinoicacid
[RefComprehensivedermatologicaldrugtherapyp.254&internet]
Retinoids
Theretinoidscompriseaclassofchemicalcompoundsthatare
vitamersofvitaminAorarechemicallyrelatedtoit.
Retinoidshavefounduseinmedicinewheretheyregulateepithelial
cellgrowth.
Retinoidshavemanyimportantfunctionsthroughoutthebody
includingrolesinvision,regulationofcellproliferationand
differentiation,growthofbonetissue,immunefunction,and
activationoftumorsuppressorgenes.

1680.Treatmentforimpetigoincludesall
except?
a)Topicalmupirocin
b)Systemicerythromycin
c)Topicalgentamycin
d)Systemiccephalosporins
CorrectAnswer-C
Ans.C.Topicalgentamycin
[RefIADVL3rd/ep.235]
*"Topicaluseofgentamycinshouldbeavoidedasgentamycin
resistancecandevelopanditcanbetransferredbetweendifferent
speciesandstrainsofstaphylococcus."
Treatmentofimpetigo
*Impetigocontagiosum
-Localized4Topicalantibioticslikefusidicacidormupirocin
-Extensive-Systemicantibiotics(erythromycingrouptocover
staphylococcusandstreptococcus).
-Ifresponseispoor,oxacillin-Clavulanicacidorcephalexincanbe
tried.
BullousImpetigo
*LocalizedTopicalfusidicacidormupirocin
*Extensive4Systemicantistaphylococcalantibiotics(flucloxacillin,
amoxicillin-clavulanicacid,methicillinorerythromycin)

1681.Jockitchiscausedby?
a)Epidermophytonfloccosum
b)Candidaalbicans
c)Trichophytontonsurans
d)Malasseziafurfur
CorrectAnswer-A
Ans.A.Epidermophytonfloccosum
[RefNeenaKhanna3rd/ep.242-244]
Tineacruris(Jockitch):
ItisalsoknownasDhobi'sitch.
Tineacrusisisdermatophyticinfectionofthegroinandadjacent
skin.
Theclassicalappearanceoftineacrurisisred(erythematous)scaly
lesionwithclearcentre.
Marginsarewelldefinedwithraisedborders.
Itchingisveryprominent.
Themostcommonsitesofinvolvementaregenitalareaandmedial
aspectofupperthigh(Mostcommonsiteofdermatophytosisin
males).
Tcrurisusuallyaffectyoungadultmale.
InIndiatrichophytonrubrumisthemostcommoncauseandin
westerncountriesepidermophytonfloccosumisthemostcommon
cause.

1682.Resorcinolisusedinthetreatmentof?
a)Lichenplanus
b)Acne
c)Vitiligo
d)Scabies
CorrectAnswer-B
Ans.B.Acne
[RefContactandoccupationaldermatologyp.195]
Resorcinolisincludedinmanyantisepticandkeratolytictopical
medications.
Usesincludepsoriasis,hidradenitissuppurativa,eczema,acne,
seborrhea,corns,calluses,warts,andotherskindisorders.

1683.Hertoghe'ssignisseenin?
a)Atopicdermatitis
b)CutaneousTB
c)Lichenplanus
d)Psoriasis
CorrectAnswer-A
Ans.A.Atopicdermatitis
[RefIndianDermatolOnlineJ.2012Sep-Dec;3(3):159-165.doi:
10.4103/2229-5178.101810]
TheSignofHertogheorQueenAnne'ssignisathinningorlossof
theouterthirdoftheeyebrows(superciliarymadarosis).
Itisaclassicalsignofhypothyroidismoratopicdermatitis.
Itcanalsobeseeninleprosy,myxedema,follicularmucinosis,
trichotillomania,ectodermaldysplasia,discoidlupuserythematosus,
alopeciaareata,syphilis,ulerythemaophryogenes,systemic
sclerosisandHIVinfection.

1684.Preferredconcentrationofminoxidilfor
femaleandrogeneticalopeciais?
a)2%
b)5%
c)8%
d)10%
CorrectAnswer-A
Ans.A.2%
[RefIADVL3'/ep.891]
Infemaleandrogenicalopeciaresultsaresimilarwith2%and5%
minoxidilbutsideeffectsaremorewith5%solution.
Whereasinmales5%ismoreefficacious.
Thereforeinafemalepatient2%minoxidilispreferredwhereasin
males5%solutionisthepreferredchoice.

1685.Normalepidermalturnovertimeis?
a)1week
b)2weeks
c)3weeks
d)4weeks
CorrectAnswer-C
Ans.C.3weeks
[Ref:NeenaKhanna3rdlep.297;Roxburgh's7th/ep.62]
Treatmentofscabiesinchildren
Topicalpermethrin(5%cream)isasafeandeffectivescabicidein
children.Itisrecommendedasafirst-linetherapyforpatientsolder
than2monthsofage.
Becausetherearetheoreticalconcernsregardingpercutaneous
absorptionofpermethrinininfantsyoungerthan2monthsofage,
guidelinesrecommend7%sulfurpreparationinsteadofpermethrin.

1686.Treatmentofchoiceforscabiesinan
infant<6monthsis?
a)BHC
b)Ivermactin
c)Permathrin
d)Crotomiton
CorrectAnswer-D
Ans.D.Crotomiton
[Ref:NeenaKhanna3rdlep.297]
Ivermectinistheonlyoraldrug,availableforscabiestreatment.

1687.Oraltreatmentofchoiceforscabiesis?
a)Albendazole
b)Itraconazole
c)Sulphur
d)Ivermactin
CorrectAnswer-D
Ans.D.Ivermactin
[RefBehl10th/ep.406;NeenaKhanna3rd/ep.50,51]
Pityriasisrosea:
P.roseaisselflimitingdisease,subsideswith6-12weeks.
P.roseaisacommonscalydisorder,occuringusuallyinchildren
andyoungadults(10-35years).
Characterizedbyround/ovalpinkbrownpatcheswithasuperficial,
centrifugalscale,distributedovertrunkinaChristmastreepattern.
Theexactetiologyisnotknown,butitisconsideredtobeaviral
disease;
HumanHerpesvirus6(HHV6)andHumanHerpesvirus7(HHV7)
mayplayarole.

1688.Pityriasisroseaclearswithin?
a)1-2weeks
b)2-4weeks
c)4-8weeks
d)6-12weeks
CorrectAnswer-B
Ans.B.2-4weeks
[RefBehl10th/ep.406;NeenaKhanna3rd/ep.50,51]
Pityriasisrosea:
P.roseaisselflimitingdisease,subsideswith6-12weeks.
P.roseaisacommonscalydisorder,occuringusuallyinchildren
andyoungadults(10-35years).
Characterizedbyround/ovalpinkbrownpatcheswithasuperficial,
centrifugalscale,distributedovertrunkinaChristmastreepattern.
Theexactetiologyisnotknown,butitisconsideredtobeaviral
disease;HumanHerpesvirus6(HHV6)andHumanHerpesvirus7
(HHV7)mayplayarole.
ClinicalmanifestationsofP.rosea:
Thediseasestartswithanupperrespiratoryprodromeoramildflu.
After1-2weeks,annularerythematousplaqueappearsontrunkthat
isreferredtoasmotherpatchorheraldpatch.
Overthenext1-2weeks,freshpatchappearalloverthetrunk,ina
ChristmastreeconfigurationorFirtreeConfiguration.

1689.Aknowncaseofdiabetesdevelops
annularorangeskinlesions,which
disappearafterbiopsy.This
phenomenonisknownas?

a)Koebner'sphenomenon
b)ReverseKoebner'sphenomenon
c)AsboeHensonsign
d)Isotopicphenomenon
CorrectAnswer-B
Ans.B.ReverseKoebner'sphenomenon
[Ref
https://www.researchgate.net/publication/268343496_Proposed_
classification_for_koebner_wolf
isotopic_renbok_koebner_nonreaction_isotopic_nonreaction_other_relatedphenomen]
Annularorangeskinlesionsinadiabeticindicatetowardsgranuloma
annulare.
Therehavebeenfewreportsofdisappearanceoflesionsafter
biopsyingranulomaannulare.
ThisphenomenonisknownasreverseKoebner'sphenomenon.

1690.Alopeciauniversalisis?
a)Lossofallthescalphair
b)Lossofallbodyhair
c)Losshairatthescalpmargin
d)Malepatternhairloss
CorrectAnswer-B
Ans.B.Lossofallbodyhair
[RefFitzpatrick]
Inalopeciaaerata,whenthereistotallossofscalphairitiscalled
Alopeciatotalis.
WhenthereislossoftotalbodyhairitisreferredasAlopecia
universalis.
Alopeciaalongthescalpmarginiscalledophiasis.
Aninverseophiasispattern(sisaphio)iswhenitsparesoccipital
regionandaffectsrestofthescalp.

1691.Treatmentofchoiceforerytherodermic
psoriasisis?
a)Prednisolone
b)Hydroxyurea
c)Acitretin
d)Ciclosporin
CorrectAnswer-C
Ans.C.Acitretin
[Ref:NeenaKhanna3rdlep.46]
MethotrexateistheDOCforErythrodermicpsoriasis.
Acitretinisanalternative.

1692.Multiplepsoriaticlesionsonhands.
Treatmentofchoiceis?
a)NBUVB
b)Systemicmethotrexate
c)Topicalsteroidsandsalicylicacid
d)Systemicsteroids
CorrectAnswer-C
Ans.C.Topicalsteroidsandsalicylicacid
[RefNeenaKhanna3rdlep.46]
Preferredtreatmentforlocalizedpsoriasisistopicalcoaltarorshort
contact,dithroanol.
Alternativeistopicalsteroids+Salicylicacid.

1693.FalseaboutLanger'slinesis?
a)Remainconstantthroughoutlifetimeofaperson
b)Correspondtothecollagenfibersindermis
c)Incisionalongtheselinesproducesbetterscar
d)Skinalongtheselinesisleastflexible
CorrectAnswer-A
Ans.A.Remainconstantthroughoutlifetimeofaperson
[Ref:IADVLtextbookofdermatology3rdlep.175&Internet]
Langer'slines
Alsocalledcleavagelines,isatermusedtodefinethedirection
withinthehumanskinalongwhichtheskinhastheleastflexibility.
Theselinescorrespondtothealignmentofcollagenfiberswithinthe
dermis.
Usually,asurgicalcutiscarriedoutinthedirectionoflanger'slines,
andincisionsmadeparalleltolanger'slinesgenerallyhealbetter
andproducelessscarring.
DirectionalchangesofLanger'slineshavebeenknowntooccur
withinthecourseofaperson'slifetime.
Sometimestheexactdirectionoftheselinesareunknown,because
insomeregionsofthebodytherearedifferencesbetweendifferent
individuals.

1694.Mostcommontypeofcutaneous
mastocytosisis?
a)Solitarymastocytoma
b)Urticariapigmentosa
c)Telangiectasiamaculariseruptivaperstans
d)Diffuseerythrodermic
CorrectAnswer-B
Ans.B.Urticariapigmentosa
[RefIADVLtextbookofdermatology3rdep.146
Urticariapigmentosa:
(Generalizederuptionofcutaneousmastocytosis(childhoodtype).
Itisthemostcommonformofcutaneousmastocytosis.
Urticariapigmentosaismostcommoninchildren.
Itcanalsooccurinadults.
Itisafamilialcutaneousdisordercharacterisedbygeneralised
distributionofredbrownmacules.
Eachlesionrepresentsacollectionofmastcellsinthedermiswith
hyperpigmentationofoverlyingepidermis.
Themostcharacteristicfeaturesisthattheselesionsurticateon
scratching.

1695.Volcanosignisseenin?
a)Leprosy
b)Leishmaniasis
c)Lupusvulgaris
d)DLE
CorrectAnswer-B
Ans.B.Leishmaniasis
[RefIndianDermatolOnlineJ.2012Sep-Dec;3(3):159-165.doi:
10.4103/2229-5178.101810]
Volcanosign:
DescriptivetermforthemorphologicfeatureofOldWorldcutaneous
leishmaniasis.
Thelesionstartsasasmallnontenderpapule,whichenlargesin
sizeandulceratesinthecentre.
Theborderofthecrustedulceroftenhasanerythematousrimandis
calledas"Volcanosign".

1696.Treatmentofchoicefororalcandidiasis
is?
a)Terbinafin
b)Nystatin
c)Griesofulvin
d)Seleniumsulphide
CorrectAnswer-B
Ans.B.Nystatin
[RefIADVL3rdlep283]
Treatmentofcutaneouscandidiasis
Topical(TOCforuncomplicatedcases):-nystatinsuspension,
clotrimazoletroches,gentionviolet,chlorhexidine,ketoocnazole,
amphotericingel.
Systemic(reinfection,unresponsive&chroniccases):-oral
fluconazole,itraconazoleorketoconazoleareneeded.

1697.Mostcommonmetalresponsiblefor
contactdermatitisis?
a)Gold
b)Silver
c)Nickel
d)Mercury
CorrectAnswer-C
Ans.C.Nickel
[RefAndrew'sdiseasesofskinE-bookp99&IADVL3rd/ep.559]
Contactdermatitisismainlyoftwotypes:
A.Irritantcontactdermatitis
Duetodirectirritantactionofthemateriale.g.Solvents,Alkalis,
Detergents.Mostcommonsitesarehands&forearms.
B.Allergiccontactdermatitis
Itisduetodelayedhypersensitivity(typeIVhypersensitivity)toa
particularantigeninasensitizedindividual.
Themostcommonallergenscausingallergiccontactdermatitis
arepollenandmetals:

1. Parthenium(Congressgrass)
2. Nickel

1698.Whichofthefollowingistrueabout
xerodermapigmentosa?
a)Autosomaldominant
b)Goodlongtermprognosis
c)Purinedimmers
d)DNArepairdefect
CorrectAnswer-D
Ans.D.DNArepairdefect
[RefRook's4thiep.78.12]
Xerodermapigmentosa
Moleculardefect:
ExposuretoUVraysdamagesDNAduetoproductionofconvalent
linkagesbetweenadjacentpyrimidines(pyrimidinedimmers).
NormallythedamagedDNAisrepairedbyexcisionandrepair.
InxerodermapigmentosatherepairofUVdamagedDNAis
defective.
Itisanautosomalrecessivedisorder.

1699.Trueabouterythematoxicumis?
a)Commoninpre-termbaby
b)Lesionscontainmanylymphocytes
c)Canbelifethreateningcondition
d)Occursmainlyinneonatalperiod
CorrectAnswer-D
Ans.D.Occursmainlyinneonatalperiod
[RefIADVL.3rdlep.163]
*Erythematoxicumneonatorum(ETN/babyacne/toxicerythemaof
thenewborn)
-Itisabenign,asymptomaticskinconditionthatischaracterized
bysmall,erythematouspapules,vesicles,and,occasionally,
pustules.
-Thelesionsareusuallysurroundedbyadistinctivediffuse,
blotchy,erythematoushalo
-Itismorecommoninbabiesbornatfullterm(between37and40
weeksofgestation)comparedwithprematurebabies.
-Onlyoccursduringtheneonatalperiod,usuallybetweenday2-5
afterbirthandtypicallyresolveswithinfirsttwoweeksoflife.
-Thefluidfromerythematoxicumlesionswillshowmany
eosinophils

1700.Nevussimplexcommonlypresentsat?
a)Face
b)Trunk
c)Legs
d)Hand
CorrectAnswer-A
Ans.A.Face
[RefIADVL3rdiep.201]
Nevussimplex(Nevusflammeusnuchae/storkbite/Salmonpatch)
Itisacongenitalcapillarymalformationpresentsin30percentof
newbornbabies.
Astorkbiteisduetodilationofbloodvesselsandmaybecome
darkerwhenthechildcriesorstrains.
Theyaremostcommonontheforehead,eyelids,upperlip,between
theeyebrows,andthebackoftheneck,thesemarksfadeasthe
infantgrows.

1701.Inmechanicalventilation,peak
pressureininspirationdenotes?
a)Complianceoflung
b)Capacityofinspiratorymuscles
c)Airwayresistance
d)Alloftheabove
CorrectAnswer-C
Ans.C.Airwayresistance
[Ref:Essentialsofanaesthesia-786]
Pressuresinmechanicalventilation
Peakpressure
Peakpressureapplieswhenthereisairflowinthecircuit,i.e.,during
inspiration.
Peakpressuredeterminesairwayresistance.

1702.Propofolvial,onceopened,shouldbe
usedwithin?
a)2hours
b)4hours
c)6hours
d)8hours
CorrectAnswer-C
ANs.C.6hours
[Ref:Lee13th/ep.158-160;Morgan4thlep.200-202]
Propofolisoilbasedpreparationcontainingsoybeanoil,egglecithin,
andglycerol.Thecolorofsolutionismilkywhite.
Solutionshouldbeusedwithin6hoursafteropeningthevial
becausetherehavebeendeathreportsfollowingtheuseof
contaminatedsolutionasegglecithinisagoodmediumforbacterial
growth.
Topreventthisproblemrecentlyavailablepropofolpreparations
havedisodiumedetateorsodiummetabisulfiteasantimicrobial
agent.

1703.Relativecontraindicationof
neuraxial/regionalanesthesiais?
a)Hypertension
b)Renaldisease
c)Sepsis
d)Diabetes
CorrectAnswer-C
Ans.C.Sepsis
[RefMorgan4th/ep.299]
Contraindicationstoneuraxialblock(spinal&epidural)
Absolute:-Infectionatthesiteofinjection,patientrefusal,
coagulopathyorotherbleedingdisorder,severehypovolemia,
increasedICT,severeaorticormitralstenosis.
Relative:-Sepsis,unto-operativepatient,pre-existingneurological
deficit,demyelinatinglesions,severespinaldeformity,stenotic
valvularheartdisease.

1704.CNSaffectionofalocalanesthetic
agentleadsto?
a)Convulsion
b)Perioralnumbness
c)Depression
d)Alloftheabove
CorrectAnswer-D
Ans.D.Alloftheabove
[RefMorgan4thlep.270]
Manifestationsoflocalanesthetictoxicity
Earlysymptoms(Prodrome)Circumoralnumbness,dizziness,
tongueparesthesia,restlesness,tinnitus,agitation.
CNSsymptoms:-Thesearebiphasici.e.excitation(convulsions,
restlessness,agitation&tinnitus),followedbydepression
(drowsiness,disorientation,respiratorydepression,
unconciousness).
Cardiovascularmanifestations:-hypotension,cardiacarrest,coma

1705.Americananestheticassociationsays
thatclopidogrelshouldbewithheldhow
manydaysbeforesurgery?

a)1day
b)1week
c)3weeks
d)4weeks
CorrectAnswer-B
Ans.B.1week
[RefAjayYadav3'/ep.46]
Preoperativemodificationsofpre-existingdrugs
Drugswhichcanbecontinued
Drugswhichcanbestopped
tillthedayofsurgery
Conventionaldoseaspirin&
Antianginal(exceptaspirin)
clopidogrel(antiplatelets)-I
weekbeforesurgery
Antiepileptics
Oralanticoagulants-4days
before&switchtoheparin,
Antihypertensives(exceptAT-II
whichisstopped12hoursprior antagoists)Levodopa
tosurgery
Oralhypoglycemic(metformin) Digitalis
-48hoursbeforeurgery
andswitchtoinsulin
TCAs
AT-IIantagonists(losartan,
Lowdoseaspirin
valsarton)-1dayprior
Lithium-48-72hoursbefore
surgery

MAOinhibitors-3weeks
beforesurgery
Note:Antiplateletdrugslikeclopidogrelandconventionaldoseof
aspirinshouldbestopped7dayspriortosurgery.
Butlowdoseaspirincanbecontinuedtillthedayofsurgery.

1706.Whichofthefollowinginhalational
anestheticagentmosteasilycrosses
bloodbrainbarrier?

a)Methoxyflurane
b)Sevoflurane
c)Desflurane
d)Nitrousoxide
CorrectAnswer-A
Ans.A.Methoxyflurane
[RefAjayYadav4th/ep.63]
Oil:gaspartitioncoefficientmeasuresthelipidsolubilityoftheagent
andthereforesolubilityinthefat-richtissuesoftheCNS(abilityto
crossBBB).
Methoxyfluranehasmaximumoil:Gaspartitioncoefficient-->Most
lipidsoluble&mosteasilycrossesBBB.
1120hasminimumoil:Gaspartitioncoefficient-pLeastlipidsoluble
&thereforelikelytocrossBBB.

1707.Whichofthefollowinganestheticagent
mostislipidsoluble?
a)Nitrousoxide
b)Methoxyflurane
c)Isoflurane
d)Halothane
CorrectAnswer-B
Ans.B.Methoxyflurane
Oil:gaspartitioncoefficientmeasuresthelipidsolubilityoftheagent
andthereforesolubilityinthefat-richtissuesoftheCNS(abilityto
crossBBB).
Methoxyfluranehasmaximumoil:Gaspartitioncoefficient-->Most
lipidsoluble&mosteasilycrossesBBB.
1120hasminimumoil:Gaspartitioncoefficient-pLeastlipidsoluble
&thereforelikelytocrossBBB.

1708.Etomidateisnotusedforlongterm
infusionbecause?
a)Resultsinadrenalsuppression
b)Maycausevasospasm
c)Resultsincardiacarrhythmias
d)MaycauseincreaseinICT
CorrectAnswer-A
Ans.A.Resultsinadrenalsuppression
[RefMorgan4th/ep.200]
Etomidatesuppressescorticosteroidsynthesisintheadrenalcortex
byreversiblyinhibiting1113-hydroxylase,whichcoverts11-
deoxycortisoltocortisolandbyarelativeminoreffecton17a-
hydroxylase.
Usingacontinuousetomidateinfusionforsedationofcriticallyill
traumapatientsinintensivecareunitshasbeenassociatedwith
increasedmortalityduetoadrenalsuppression.
Themortalityofpatientsexposedtoacontinuousinfusionof
etomidateformorethan5daysincreasedfrom25%to44%,mainly
duetoinfectiouscausessuchaspneumonia.

1709.Amongthefollowingagents,maximum
boilingpointisassociatedwith?
a)Sevoflurane
b)Isofurane
c)Methoxyflurane
d)Desflurane
CorrectAnswer-C
Ans.C.Methoxyflurane
[RefAjayYadav4th/ep.63]
Boilingpointofmethoxyfluraneismorethanwater(104?C)

1710.Apatientwithnormalsuccinylcholine
metabolismwillhaveDibucainenumber
between?

a)20-30
b)40-45
c)50-60
d)70-80
CorrectAnswer-D
Ans.D.70-80
Dibucainnumber:Dibucain(alocalanaesthetic)inhibits80%of
normalpseudocholinesteraseand20%ofatypical(non-functional)
pseudocholinesterase.Thereforenormaldibucainnumberis70-
80%.

1711.Benzocaineisusedinwhichtypeof
anesthesia?
a)Topical
b)Spinal
c)Epidural
d)Alloftheabove
CorrectAnswer-A
Ans.A.Topical
[RefMorgan4th/ep.270]
Benzocaineandcocaineareusedonlyintopicalanesthesia.

1712.Succinylcholineiscontraindicatedin?
a)Hyperkalemia
b)Hypokalemia
c)Hypercalcemia
d)Hypocalcemia
CorrectAnswer-A
Ans.A.Hyperkalemia
[RefMorganPiep.214]
Schcancausedangeroushyperkalemiainandiscontraindicatedin:
Burn,massivetrauma,crushinjury,Severeintraabdominalinfection
(sepsis).

1713."TriangleofPetit"isalandmarkfor
whichblock?
a)Spinalblock
b)Bier'sblock
c)TAPblock
d)Epiduralblock
CorrectAnswer-C
Ans.C.TAPblock
[RefMiller'sanesthesiaE-bookp.1735]
*Transverseabdominisplane(TAP)block
-Itisaperipheralnerveblockdesignedtoanesthetizethenerves
supplyingtheanteriorabdominalwall(T6toL1).
-ThepointofentryfortheblindTAPblockisthelumbartriangleof
Petit.
-Thisissituatedbetweenthelowercostalmarginandiliaccrest.
-Itisboundanteriorlybytheexternalobliquemuscleand
posteriorlybythelatissmisdorsi.

1714.Whatistheintubationdoseof
pancuronium?
a)0.1mg/kg
b)1mg/kg
c)10mg/kg
d)20mg/kg
CorrectAnswer-A
Ans.A.0.1mg/kg

1715.Whichofthefollowingagentis
associatedwithmaximumhistamine
release?

a)d-Tubocurarine
b)Cisatracurium
c)Pancronium
d)Rocuronium
CorrectAnswer-A
Ans.A.d-Tubocurarine
HistaminereleaseiscausedbyD-TC(maximumtendency),
succinylcholine,mivacurium,doxacurium,atracurium,tubocurarine
cancausebronchoconstriction.

1716.Hypotensionfollowingspinal
anesthesiacanbebestpreventedby?
a)Preloadingwithcolloids
b)Usingsmallsizeneedle
c)Preloadingwithcrystalloids
d)Alloftheabove
CorrectAnswer-A
Ans.A.Preloadingwithcolloids
[RefLee13th/ep.509,510;Morgan4th/ep.297]
Hypotensionisthemostcommoncomplicationofspinalanesthesia.
Itarisesduetoblockingofsympatheticrootfibersandisusually
accompaniedbybradycardia(Preganglionicblockofthe
sympatheticnervestoheartT1-T4)andnausea.
Hypotensioncanbepreventedbypreloadingthepatientwith
colloids,Preloadingwithcrystalloiddoesnotpreventhypotension
becauselargevolumesofcrystalloidsquicklyredistributefrom
intravasculartoextravascularspace.
Beachchairpositionalsopreventshypotension.

1717.Intra-arterialthiopentoneinjection
leadsto?
a)Ischemia
b)Vasodilatation
c)Vomiting
d)Hypertension
CorrectAnswer-A
Ans.A.Ischemia
[RefAjayYadav4th/ep.80;Lee13thiep.155]
Ischemia
Inadvertentintra-arterialinjectionofthiopentoneisaverydreadful
complication.
Itproducesthrombosis,vasospasm,ischemia,necrosisandfinally
gangrene.
Thefirstsymptomisburningpain.Thefirstsignisblanchingofthe
handduetovasospasm.

1718.SizeofLMAfora15kgchildis?
a)1
b)2
c)3
d)4
CorrectAnswer-B
Ans.B.2
LMAaccordingtopatient'sweightandage:
10-20kgs,infants&children

1719.Anestheticagentleadingtobradycardia
is?
a)Pancuronium
b)Vecuronium
c)Atracurium
d)Propofol
CorrectAnswer-D
Ans.D.Propofol
Bradycardiacausinganestheticagents:
Succinylcholine
Propofol
Opioidsanesthetics(fentanylanditscongers)

1720.TypeEcircuitisusedfor?
a)Spontaneousventilation
b)Controlledventilation
c)Children
d)Anusedforalloftheaboveindications
CorrectAnswer-C
Ans.C.Children
[RefAjayYadav4th/ep.29,30]
Circuitofchoiceforspontaneousventilationinadult-4MaplesonA
CircuitofchoiceforcontrolledventilationinadultMaplesonD(Bain
circuit)
Circuitofchoiceforchildren-TypeF,i.e.Jackson-Rees(first
choice)andtypeE,i.e.Ayre'sTpiece(secondchoice)

1721.Propofolshowsfollowingeffecton
EEG?
a)Activation
b)Depression
c)Depressioninlowdosesandactivationinhighdoses
d)Noneoftheabove
CorrectAnswer-B
Ans.B.Depression
EEGchangesduringanesthesia:
EEGDepressioncausedby,
Inhalationalagents(1-2MAC)
Barbiturates
Opioids
Etomidate
Propofol
Hypocapnia
Markedhypercapnia
Hypothermia
Latehypoxia,ischemia

1722.Pinindexsystemisasafetyfeature
adoptedinanesthesiamachinesto
prevent?

a)Incorrectattachmentofanesthesiamachines
b)Incorrectattachmentofanesthesiafacemasks
c)Incorrectinhalationagentdelivery
d)Incorrectgascylinderattachment
CorrectAnswer-D
Ans.D.Incorrectgascylinderattachment
[RefLee13th/ep.85]
Pinindexsystem:-Thisisthesafetymechanismsothatone
cylindercannotbefittedattheother'sposition.

1723.Anestheticagentofchoiceinasthma
patientis?
a)Thiopentone
b)Methexitone
c)Ketamine
d)Propofol
CorrectAnswer-C
Ans.C.Ketamine
[RefMiller's7"lep.744-746]
Ketamineisapotentbronchodilator;thereforeitistheanesthetic
agentofchoiceinbronchialasthmapatients.
Halothaneistheinhalationalagentofchoiceinasthmatics.

1724.Mallampatti'sclassificationisfor?
a)Mobilityofcervicalspine
b)Mobilityofatlantoaxialjoint
c)Assessmentoffreerotationofneckbeforeintubation
d)Inspectionoforalcavitybeforeintubation
CorrectAnswer-D
Ans.D.Inspectionoforalcavitybeforeintubation
[RefMorgan4thiep.113]
Mallampatiscore(Mallampatioropharyngealscale):
Mallampatigradingisusedtoevaluatethevisibilityoftonsiland
tonsillarfossawhichinturnassesstheadequatemouthopening
dependinguponthegrade.
IndicationsofMallampatiscoreareOralcavityassessmenttorule
outdifficultintubation(Inspectionoforalcavitybeforeintubation)&
Sleepapneaevaluation.

1725.Thicknessofleadaprontoprevent
radiation:
a)1mm
b)3mm
c)0.5mm
d)7mm
CorrectAnswer-C
Ans.C.0.5mm
"Itisrecommendedthatforgeneralpurposeradiographytheminimal
thicknessofleadequivalentintheprotectiveapparelshouldbe
0.5mm."
-TextbookofRadiologyPhysicsp.39
Leadapronof0.5mmthicknessreduceintensityofscatteredX-rays
byover90%.

1726.Radiationprotectionshieldsaremade
upof:
a)Copper
b)Silver
c)Lead
d)Tin
CorrectAnswer-C
Ans.Lead

1727.SoapbubbleappearanceonMRIbrain
ischaracteristicof?
a)Tubercularmeningitis
b)Neurocysticercosis
c)Cryptococcalcysts
d)Ependymoma
CorrectAnswer-C
Answer-C.Cryptococcalcysts
Cryptococcomas
T1:lowsignal
T2/FLAIR:highsignal
T1C+(Gd):variable,rangingfromnoenhancementtoperipheral
nodularenhancement.
GelatinouspseudocystscausedbyCryptococcustendtogivea
"soapbubble"appearanceonMRI.MRIfindingsinneural
cryptococcosis.

1728.T2imageinMRIis-
a)Goodtodetectpathology
b)Goodtodetectanatomy
c)Goodforboth
d)Goodfornone
CorrectAnswer-A
Answer-A.Goodtodetectpathology
Manypulse-sequencetechniquesareusedinMRI,butmostclassic
areT1andT2weightedimages.Asageneralrule,TI-weighted
imagesaregoodforviewinganatomy,andT2weightedimagesare
goodfordetectingpathology.
[RefFundamentalsofdiagnosticradiologyp.18]

1729.CSFonMRIappears:
a)HyperintenseonT1weighedimageandhypointenseonT2
weighedimage
b)HypointenseonT1weighedimageandhyperintenseonT2
weighedimage
c)HyperintenseonT1andT2weighedimages
d)HypointenseonT1andT2weighedimages
CorrectAnswer-B
Ans.HypointenseonT1weighedimageandhyperintenseonT2
weighedimage
Fluid-Edema,Urine,Bile,CSF-T1weightedsignallow&T2
weightedsignalhigh

1730.Keyholesignonultrasoundisseenin-
a)Polycystickidney
b)Hydronephrosis
c)Chronicpyelonephritis
d)Posteriorurethralvalves
CorrectAnswer-D
Answer-D.Posteriorurethralvalves
Thekeyholesignisanultrasonographicsignseeninboyswith
posteriorurethralvalves.ltreferstotheappearanceoftheproximal
urethra(whichisdilated)andassociatedthickwalleddistended
bladderwhichonultrasoundmayresembleakeyhole.

1731.MIBGdoseis-
a)40-80MBq
b)80-100MBq
c)100-120MBq
d)120-150MBq
CorrectAnswer-A
Answer-A.40-80MBq
MIBGscanisascintigraphicstudythatuses
metaiodobenzylguanidinelabeledtoIodine-123orIodine-131.
Theactivityadministeredtoadultsshouldbe:for131I-mIBG:40-80
MBq(1.2-2.2mCi);for123ImIBG:400MBq(10.8mCi).
Forminimumandmaximumrecommendedactivitiesinchildrenone
shouldconsulttheGuidelinesforRadioiodinatedMIBG
ScintigraphyinChildren(minimumactivity20MBqfor123I-mIBG
and35MBqfor131I-nIBG;maximumactivity400MBqfor123I-mIBG
and80MBqfor131I-mIBG).

1732.Bariummealfollowthroughishelpfulin
diagnosing-
a)Colonicstricture
b)Ilealstricture
c)Rectalstricture
d)Esophagealstricture
CorrectAnswer-B
Answer-B.Ilealstricture
Followingabnormalitiesofsmallintestinecanbeidentified:-

1. Malabsorption
2. BD(CD&UC)
3. Tumorsofsmallintestine
4. Smallbowelobstruction
5. Intestinalstrtcture

1733.Themaximumpermissiblelevelof
occupationalexposuretoradiation
is...peryear-

a)5rad
b)2rad.
c)10rad
d)50rad
CorrectAnswer-A
Ans.is'a'i.e.,5Rad
Theamountofradiationreceivedfromouterspaceandbackground
radiationhasbeenestimatedtobe0.1radayear.Apparently,this
doesnotatpresentconstituteahazard.Theadditionalpermissible
dosefrommanmadesourcesshouldnotexceed5radayear.

1734.Bananaandlemonsignseeninwhich
fetalnomalies:
a)Neuraltubedefect
b)Hydropsfetalis
c)Twins
d)IUD
CorrectAnswer-A
Ans.isai.e.Neuraltubedefect
SignsofSpinabifidaonUltrasound
Smallbiparietaldiameter.
Ventriculomegaly.
Frontalbonescallopingorthesocalledlemonsign.
Elongationanddownwarddisplacementofthecerebellum-theso
calledbananasign.
Effacementorobliterationofthecisternamegna.

1735.Radiographicsigncharacteristicof
pulmonaryedemais-
a)Westermark'ssign
b)Hampton'shump
c)Pallasign
d)Batwingsign
CorrectAnswer-D
Answer-D.Batwingsign
Acutepulmonaryedema:-Acutepulmonaryedemaischaracterized
bycentrallylocatedalveolarsign,withhazyorfluffyincreased
densityinperihilardistribution,creatingabat-wingorangel-wing
paftern.Thereisrelativesparingofthemoreperipheralzonesof
lungfields.Airbronchogrambecomeevidentasedemabecomes
moreopaque.

1736.Gammaknifeutilizes-
a)Strontium89
b)1-131
c)Cobalt-60
d)P-32
CorrectAnswer-C
Answer-C.Cobalt-60
GammaknifecontainsCobalt-60sourcesofapproximately30curis
placedincirculararrayinaheavilyshieldedunit.Theunitdirectsthe
gammarirystothetarget.

1737.SnowstormappearanceonchestX-ray
isseenin-
a)Anthracosis
b)Byssinosis
c)Silicosis
d)Bagassosis
CorrectAnswer-C
Answer-C.Silicosis
Simpleformz-Multiplesmallrounded(nodular)opacitiesinthelung
parenchyna(snowstonnappeoraace).
Thesenodulestendtobelocatedpredominantlyinthemiddleand
upperlungfieldswithrelativesparingoflowerlungfields(Though
Iowerlungfieldscanalsobeinvolvedlaterindiseaseprocess).
Thereisbilateralhilarlymphadenopathywithcharacteristiceggshell
calcification.

1738."Drooplilysign"isseenin-
a)"Drooplilysign"isseenin
b)Duplicatedcollectingsystem
c)Chronicpyelonephritis
d)Hypernephroma
CorrectAnswer-B
Answer-B.Duplicatedcollectingsystem
Thedroopinglilysrgl(droppingflowersign)isaurographicsignin
somepatientswithaduplicatedcollectingsystem.Itreferstothe
inferolateraldisplacementoftheopacifiedlowerpolemoietydueto
anobstructed(andunopacified)upperpolemoeity.

1739.Sonographicappearanceofhydatid
cystis-
a)Hyperechoicaseptatelesion
b)Hypoechoicaseptatelesion
c)Hyperechoicseptatelesion
d)Hypoechoicseptatelesion
CorrectAnswer-D
Answer-D.Hypoechoicseptatelesion
TypeI(Simplecyst):-Single(solitary)hnrcechoic/anechoicaseptate
(withoutsepta)lesion.Thediagnosisofhydatidcystmaybe
consideredwhenfocalthickeningofwallispresentorwhen
hlperechoicspots,duetohydatidsand,appearinthedependent
areas.

1740.Roentgenistheunitof:
March2010
a)Radioactivity
b)Radiationexposure
c)Absorbeddose
d)Noneoftheabove
CorrectAnswer-B
Ans.B:RadiationExposure
Thecurie,namedafterscientistMarieCurie,isaunitof
measurementusedtomeasurehowradioactiveanobjectis,orhow
muchradiationitproduces.
Thisisdonebyexamininghowfastitsatomsdisintegrateand
measuringtheirdisintegrationpersecond.
Roentgen,ontheotherhand,isaradiationunitthatindicates
howmuchradiationispresentintheairofaspecific
environment.
Thisisusedtoshowhowmuchradiationmaybeabsorbedby
standinginaparticularplaceforacertainamountoftime.One
roentgenofgamma-orX-rayexposureproducesapproximately1
rad(0.01gray)tissuedose.
Morecommonthanthesetwoaretheradandtherem.
Thesetwounitscanmeasureanytypeofionizingradiation,
includingalpha,beta,neutron,gammaand"X,"anddealwithhow
muchradiationisabsorbedbyobjects.
Radstandsfor"radiationabsorbeddose."
Oneradequals100ergs(anenergyunit)absorbedby1gof
material.
Radsareusedtoshowhowmuchradiationanyobject,

especiallythingslikemetalandstone,hasabsorbed.Rem(dose
equivalent)isastrictlybiologicalmeasurement,andstandsfor
"roentgenequivalentman,"meaningthatitisthesameessential
measurementasaroentgen,onlyappliedtothehumanbody,
althoughthisworksonlywithgammaand"X"typesofradiation.
Remisusedtodefinelimitsofexposureforpeoplewhoworkin
nuclearpowerplants.
Remisoftendividedinmilliremsandassignedalengthoftime,such
asmilliremsperhour.
Curie/becquerelistheunitofradioactivity.

1741.SIunitofradioactivityis:
March2013(c,f)
a)Rem
b)Rad
c)Becuerel
d)Curie
e)None
CorrectAnswer-C
Ans.Ci.e.Becquerel
OldunitofradioactivityisCurieandnewunit(SI)isBecquerel.


1742.SIunitofabsorbeddoseis-
a)Becquerel
b)Columb/cm
c)Gray
d)Sievert[Sv]
CorrectAnswer-C
Answer-C.Gray
OldunitofabsorbeddoseisRadandnewunit(SI)isGray.

1743.Stenver'sviewisusedfor-
a)Superiororbitalforamen
b)Inferiororbitalforamen
c)Internalauditorycanal
d)Sellaturcica
CorrectAnswer-C
Answer-C.Internalauditorycanal
Skulltrauma(sellaturcica)(pituitaryfossa)-Lateralview
Internalauditoryview(bothside)-Stenver'sview

1744.Acutemyocarditisschintigraphyis
donewith-
a)Thallium
b)Technetium
c)Gallium
d)None
CorrectAnswer-C
Answer-C.Gallium
Gallium-67uptakeisincreasedininflamedmyocardium.Gallium-67
citrateinjectedintravenouslybindstotransferrin,anditis
incorporatedintothetransferrinreceptorofinflammatorycellsor
malignanttumorcells.
Gallium-67scintigraphyisusefulforexaminationofheartdisease
includingcardiacsarcoidosisandacutemyocarditis

1745.Radiationusedmostcommonlyforpain
managementduetobonemetastasisis
-

a)Co60
b)Iridium192
c)Tritium
d)Tin-117
CorrectAnswer-A
Answer-A.Co60
Radiopharmaceuticals(radioactiveisotopes)usedformetastatic
bonepainarestrontium(Sr89),Samarium(Sm153),rhenium
(Re186),Phosphorus-32andTin-117(Sn-177).

1746.Minimumradiationdosewhichmay
leadtooligospermiais-
a)<1Gy
b)2-3Gy
c)7-10Gy
d)15Gy
CorrectAnswer-A
Answer-A.<1Gy
Fractionateddoses0.7-0.9Gyleadtooligospermia/azoospermiabut
withfrequentrecoveryat1-1.5years.Permanentazoospermiamay
occurafterfractionateddosesaslowas1.2Gy,andislikely>2Gy.

1747.HalflifeofTechnetium99is:
a)2hours
b)6hours
c)12hours
d)24hours
CorrectAnswer-B
Ans.6hours
Tc-99-Technitium-6hours
Halflife(tin)ofradium(Ra226)is1602-1626years(longest)Q;
Cesium(137Cs)is30yearsQ;
Cobalt(60Co)
is5.2yearsQ;iridium(1921r)is74.5daysQ;Iodine(P31)is8
daysQ;1123
is13hoursQ;technitium(Tc99)is6hoursQ;and
1132is2.3hoursQ.


1748.Halflifeoftritiumis-
a)10.2years
b)12.3years
c)15.5years
d)20.7years
CorrectAnswer-B
Answer-B.12.3years
Tritium(hydrogen-3)isaradioactiveisotopeofhydrogen.The
nucleusoftritium(sometimescalledatriton)containsoneproton
andtwoneutrons.
Whereasthenucleusofprotiumcontainsoneprotonandno
neutrons.
Tritiumhasahalf-lifeof12.3years

1749.Stereotacticradiosurgeryisdonefor-
a)Glioblastomamultiforme
b)Medulloblastomaspinalcord
c)Ependymoma
d)AVmalformationofbrain
CorrectAnswer-D
Ans.is'D'i.e.,AVmalformationofbrain
Itisalsousedfor-
1. Solitarycerebralmetastasis
2. Arteriovenousmalformation
3. Smallmeningiomas
4. Schwannomas
5. Pituitaryadenomas

1750.ABonemarrowtransplantrecipient
patientdevelopedchestinfection.On
HRCT'Treeinbudappearance'isseen.
Mostlikelycausativeagentis:

a)Klebsiella
b)Pneumocystis
c)TB
d)RSV
CorrectAnswer-B
Pneumocystis[Ref:HarrisonI7/ep843;Article'Treeinbudsign'in
Journal'Radiology'and'Radiographics7
Tree-in-budisasignseeninHRCT,mostcommonlyseenwith
endobronchialspreadofTuberculosis,butcanbeseenwithawide
varietyofconditions,mostcommonlyinfections.(Describedaheadin
detail).
Theoreticallyalltheoptionscancause'Tree-in-bud'sign(although
Klebsiellahasnotbeenmentionedinthelist).
SothenextclueisBonemarrowtransplant.
Bonemarrowtransplantcausesatransientstateofimmunological
deficiencyleadingtoawidevarietyofopportunisticinfections.
Amongthegivenoptions,accordingtothetableandtextofHarrison
(17/echapter'InfectionsinTransplantRecipients')Pneumocystis
andRSVcancausepneumoniaaftertransplant:butPneumocystisis
amuchmorecommonthanRSV.
CommonSourcesofInfectionsafterHematopoieticSteinCell
Transplantation

PeriodafterTransplantation

Early(<1
Middle(1-4
InfectionSite
Late(>6Months)
Month)
months)
Disseminated
Aerobicgram- Nocardia
Encapsulated
negative,
Candida,
bacteria
grain-positive Aspergillus
(Streptococcus
bacteria
pneumoniae,
Haemophilus
influenzae,
Neisseria
meningitidis)

Skinand
HSV-
HHV-6
VZV
mucous
membranes
Lungs

Candida,
CMV,
Pneumocystis
Aspergillus
seasonal
HSV
respiratory
viruses
Pneumocystis
Toxoplasma

Gastrointestinal
CMV
tract
BKvirus,
Kidney
BKvirus
adenovirus
Brain
HHV-6
Toxoplasma
Toxoplasma
JCvirus

Bonemarrow
HHV-6
Tree-in-budsign
Thetree-in-budsignisafindingseenonthin-sectioncomputed
tomographicimagesofthelung(HRCT).(NotseenonX-rays)
Peripheral,small,centrilobularnodulesareconnectedtolinear,
branchingopacitiesthathavemorethanonecontiguousbranching
site,thusresemblingabuddingor,fruitingtree:thisisknownas
tree-in-bud-pattern.
Itrepresentsdilatedandimpacted(mucusorpus-filled)centrilobular
bronchioles.
Thepresenceoftree-in-budisindicativeofsmallairway
disease.


Itismostcommonlyassociatedwithendobronchialspreadof
Mycobacteriumtuberculosis.Butitcanalsobeseeninalarge
numberofconditions.
Pulmonaryinfectiousdisordersinvolvingthesmallairwaysarethe
mostcommoncausesofthetree-in-budsign.Anyinfectious
organism,includingbacterial,mycobacterial,viral,parasitic,and
fungalagents,caninvolvethesmallairwaysandcauseatree-in-bud
pattern.
CausesofTree-in-budappearance
Congenital
disorders
Peripheralairwaydisease Cysticfibrosis
Connectivetissue
Infection
Kartagener
disorders
Bacterial
syndrome
Rheumatoidarthritis
Mycobacterium
Idiopathic
Sjo-grensyndrome
tuberculosis
disorders
Peripheral
Mavium-intracellulare
Obliterative
pulmonaryvascular
complex
bronchiolitis
disease
Staphylococcusaureus
Diffuse
Neoplasms
Haemophilus
panbronchiolitis
Primarypulmonary
influenzae
A
lymphoma
Fungal
spiration
Neoplastic
Aspergillus
Inhalation
pulmonaryemboli
Pneumocystiscarinii,
Toxicfumesand
Gastriccancer
(renamedPneumocystis
gases
Breastcancer
jiroveci)Viral
Immunologic
Ewingsarcoma
Cytomegalovirus
disorders
Renalcancer
Respiratorysyncytialvirus Allergic
bronchopulmonary
aspergillosis

1751.Safelightinradiographicdarkroomis
ideallyshouldbeoffollowingcolor-
a)Red
b)Yellow
c)Purple
d)Blue
CorrectAnswer-A
Answer-A.Red
Red-Somebluesensitivematerials,mostphototypesetting
materials,mostblueandmostgreensensitivemedicalx-rayfilms
(usedindarkroom).
Darkamber-Colornegativepapersandmaterials,panchromatic
blackandwhitepapers.
Amber-Colornegativepapers,panchromaticblackandwhitepapers

1752.Chainoflakesappearanceisseenin?
a)Chronicpancreatitis
b)Acutepancreatitis
c)Gallstoneileus
d)Sub-acuteintestinalobstruction
CorrectAnswer-A
ChronicpancreatitisREF:Sutton'sRadiology7theditionvolume1
page798,Sabistontextbookofsurgery18thedchapter5
Chronicpancreatitisischaracterizedbyirregularitiesofthe
pancreaticducts,ductalstrictures,andareasofductdilation.The
majoraswellastheside-branchductsmaybeinvolved.For
unexplainedreasons,somepatientswithchronicpancreatitis
developdilatedmainpancreaticducts(largeductdisease),whereas
othersretainductsofnormalorevensmallerthannormalcaliber
(smallductdisease).Somepatientswithchronicpancreatitiscanbe
showntohavemajorductsthathavetheappearanceofa"chainof
lakes"ora"stringofpearls"thatistheresultofsegmentsofdilated
ductseparatedbyareasofductalstricture

1753.Commonlyusedtypeofradiationin
radiotherapyis:
a)Alpharays
b)Betarays
c)Gammarays
d)X-rays
CorrectAnswer-C
Ans.Gammarays
Radioiodinegeneratesbothbetaandgammaraysbutpredominantly
betarays.

1754.ThephotosensitivematerialusedinX-
raysfilmsconsistof:
a)Cellulose
b)Silverbromide
c)Zincsulphide
d)Cadmiumtungstate
CorrectAnswer-B
Ans.Silverbromide
Aftertheimagehasbeendeveloped,theresultantimageisthen
fixedbyafixer(Hypo-sodiumthiosulphate)whichremovesunused
silverhalide,whichwouldmakethefilmappearmilkyorcloudy.X-
rayfilmshouldbedevelopedindarkroom,otherwiselightwillspoil
thefilm(x-rayfilmhasphotosensitivesilverbromide).Blueand
greenlightaremostsensitivewhereasyellowandredlightareleast.

1755.Leastpenetratingpoweramong
followingmentionedraysisin-
a)Alpharays
b)Betarays
c)Gammarays
d)X-ray
CorrectAnswer-A
Answer-A.Alpharays
Penetrationpower:Gammarays>Xrays>Betaparticle>Alpha
particle(orheliumion)
Ionizing&damagingpower:Alphaparticle(orheliumion)>Beta
particle>Xray>Gammaray
Alphaparticles(Heliumnuclei)havehighestionizingpowerbecause
theyhavealargecharge.
Alphaparticleshavethehighestdamagingpowerastheyare
relativelyslowandheavy.

1756.Non-ionizingradiationamongthe
followingis-
a)MRI
b)CTScan
c)X-ray
d)Positionemissionscintigraphy
CorrectAnswer-A
Answer-A.MRI
Non-ionizingradiation-
USG
MRI
Thermography(infraredrays)
UVrays
Radiofrequencywaves
Microwaves

1757.Confabulationis?
a)Astateofconfusionwherepatientisnotabletodescribethe
details
b)Purposefullyfabricatingstoriestoprojectacertainimage
c)Fillinguptogapsbyfabricationtocoverlapsesinmemory
d)Afeelingofstrangenesstofamiliarsituationsorevents.
CorrectAnswer-C
Ans.C.Fillinguptogapsbyfabricationtocoverlapsesin
memory
[RefKaplan&Sadock's10th/ep.275]
Confabulation
ItisatypeofParamnesia(Distortedorfalsifiedrecallofeventsin
relationtodetailsortheirtemporalrelationships)
Unintentionalfillingofgapsofmemorywithmaterialwhichare
untrueandfanciful.

1758.Alcoholwithdrawalisnotassociated
with?
a)Seizure
b)Amnesia
c)Tremers
d)Delirium
CorrectAnswer-B
Ans.B.Amnesia
[Ref:NirajAhuja6th/ep.41]
Amnesiaoccursduringacuteintoxication(notduringwithdrawal).

1759.A40yearoldmarriedmalethinksthat
heismultitalentedandisalways
overconfident.Heneverlistenstohis
familyorfriends.Infactwhenever
anyonegiveshimanyadvice,hethinks
thattheyhavesomemotiveagainst
him.Heisalwayssuspiciousofhis
wife.
Allthesearefeatureof?

a)Borderlinepersonalitydisorder
b)Schizoidpersonalitydisorder
c)Paranoidpersonalitydisorder
d)Histrionicpersonalitydisorder
CorrectAnswer-C
Ans.C.Paranoidpersonalitydisorder
[RefNamboodiri3rdlep.303;NirajAhujaelep.123]
Paranoidpersonalitydisorder:
Itischaracterizedbygeneralizedmistrustandsuspiciousness
aboutthemotivesandactionsofothersandatendencytointerpret
themasmalevolent.Thepatientbelievesthat:
1. Othersareexploitingordeceivingtheperson.
2. Friendsareuntrustworthyandnotloyal.
3. Thespouse/partnerisunfaithful.
4. Thereishiddenmeaninginneutralorfriendlyremarks.
5. Manypatientshavefeelingofself-importanceandthinktheyare

unusuallytalented.

1760.Characteristicofhistrionicpersonality
disorderis?
a)Violationofrulesofsociety
b)Attention-seekingbehavior
c)Unstableinterpersonalrelationship
d)Grandiosebehavior
CorrectAnswer-B
Ans.B.Attention-seekingbehavior
[RefNirajAhuja6'"/ep.122]
Patientswithhistrionicpersonalitydisorderdisplayexcessive
emotionalityandattention-seekingbehavior.Otheroptions
Violationofrulesofsocietyantisocialpersonalitydisorder
Unstableinterpersonalrelationshipborderlinepersonalitydisorder
Grandiosebehaviornarcissisticpersonalitydisorder

1761.Personalitytypeseeninschizophrenia
is?
a)Schizoid
b)Paranoid
c)Borderline
d)Alloftheabove
CorrectAnswer-D
Ans.D.Alloftheabove
[RefNirajAhuja6th/ep.125&Internet]
Schizophreniaisassociatedwith3typesofpersonalitydisordersi.e.
schizoid,borderlineandparanoid.

1762.Irresistibleurgetodrinkalcoholis
called?
a)Kleptomania
b)Pyromania
c)Dipsomania
d)Trichotillomania
CorrectAnswer-C
Ans.C.Dipsomania
[RefPeculiaritiesofbehavior]
Dipsomaniaischaracterizedbyperiodicboutsofuncontrollable
cravingforalcohol.

1763.A39yearsoldmalepatientpresents
withwaxyflexibility,negativismand
rigidity.Mostprobablediagnosisis?

a)Excitatorycatatonia
b)Stuporouscatatonia
c)Paranoidschizophrenia
d)None
CorrectAnswer-B
Ans.B.Stuporouscatatonia
[RefNeerajAhuja&hiep.62,63]
Stuporous(retarded)catatonia:
Characterizedbyextremeretardationofpsychomotorfunction,
whichincludesmutism,rigidity,negativism,posturing,echolalia,
Echopraxia,Catalepsy(waxyflexibility),ambitendency,gegenhalten,
streotypies,stupor,Mannerism,Grimicing,Automaticobedience,
andverbigeration.

1764.Mostappropriatetestforchild
psychologisttoevaluatetheintellectual
abilityofa3yearoldis?

a)StanfordBinetscale
b)Denverdevelopmentscale
c)Alexander'spassalongtest
d)Rorschachinkblottest
CorrectAnswer-A
Ans.A.StanfordBinetscale
[RefPsychologicaltestingp.101]
Stanford-BinetIntelligenceScale
TheStanford-BinetIntelligenceScaleisanindividuallyadministered
standardizedtestthatmeasuresintelligenceandcognitiveabilitiesin
childrenandadults,fromagetwothroughmatureadulthood.
TheStanford-BinetIntelligenceScaleisnowinitsfifthedition(SB5)
andwasreleasedin2003.Itisacognitiveabilityandintelligence
testthatisusedtodiagnosedevelopmentalorintellectual
deficienciesinyoungchildren.

1765.Treatmentofchoiceforakathesiais?
a)Phenytoin
b)Propranolol
c)Dantrolene
d)Lithium
CorrectAnswer-B
Ans.B.Propranolol
[RefKaplan&Saddock's10thlep.1020]
"Thefirstlinedrugforakathisiaismostcommonlyalpha-blocker".

1766.Cardinalelementofbehaviortherapy
is?
a)Modeling
b)Learning
c)Conditioning
d)Guidance
CorrectAnswer-B
Ans.B.Learning
[RefNirajAhuja6'1*p.220]
Behaviortherapyisbasedontheassumptionthatallbehaviors
(normalorabnormal)arelearningresponse.Normalandabnormal
behaviorsaresubjecttothelawsoflearningandthesamelawscan
beusedtochangethem.Behaviortherapyisbasedontheoriesof
learningandaimsatchangingthemaladaptivebehaviorand
substitutingitwithadaptivebehavior.

1767.Toleranceisseenin?
a)Alcoholdependentsyndrome
b)Schizophrenia
c)OCD
d)Alloftheabove
CorrectAnswer-A
Ans.A.Alcoholdependentsyndrome
[RefKaplan&Saddock's10thlep.382]
Alcoholdependentsyndrome
Alcoholdependentsyndromeusesthesamecriteriafordependence
forothersubstances,i.e.,threeormoreofthefollowing:-
1. Tolerance
2. Withdrawalsymptoms
3. Alcoholistakeninlargeramountorforlongerperiod.
4. Persistentdesireorsenseofcompulsiontotakealcohol.
5. Agreatdealoftimespenttoobtainalcohol,tousealcoholorto
recoverfromitseffect.
6. Neglectofimportantsocial,occupationalandrecreationalactivities.

1768.Treatmentofchoiceforgeneralized
anxietydisorderis?
a)Benzodiazepines
b)Neuroleptics
c)Betablockers
d)Barbiturates
CorrectAnswer-A
Ans.A.Benzodiazepines
[RefHarrison17th/ep.2712;Kaplan&Saddock's10th/ep.626]
Treatmentofgeneralizedanxietydisorder
Benzodiazepinesarethedrugofchoice.Drugsinthisgroupare
diazepam,Lorazepam,Alprazolam,Oxazepam,chlordiazepoxide.

1769.Generalizedanxietydisorderis
diagnosedwhenanxietyandworry
continuesforatleast?

a)2months
b)4months
c)6months
d)8months
CorrectAnswer-C
Ans.C.6months
[RefHarrison17"/ep.2712;Kaplan&Saddock's10th/ep.626]
GENERALIZEDANXIETYDISORDER
Thisischaracterizedbyexcessiveanxietyandworrywhichare
persistent&generalizedandnotrestrictedtoanyspecificsituation
orobject.
Excessiveanxietyworryoccurforatleast6months.

1770.Riskfactorforsuicideis?
a)Increasedserotonin
b)Drugabuse
c)Femalesex
d)Marriedperson
CorrectAnswer-B
Ans.B.Drugabuse
[Ref:NirajAhujaelep.236,237;Essentialsofpsychiatry4th/ep.
734]
Causes/Riskfactorsforsuicide
Psychiatricdisorders:-Depression(mostcommon),alcoholism(2nd
mc),Drug/Substancedependence,Schizophrenia,Dementia.
Physicalillness:-Cancer,AIDS,Multiplesclerosis,Headtrauma.
Psychosocialfactors:-Failureinlove,maritaldifficulties,family
dispute,illegitimatepregnancy.
Biologicalfactors:-Decreaseinserotonin
Other-Malesex,Age>40years,Single(Unmarried,divorcedor
widowed),previoussuicideattempt,socialisolation.

1771.A25yearsoldmalec/orecurrent
abdominalpainbutbiochemicalassays
andultrasoundabdomenisnormal.He
alsocomplainsofconstantheadaches.
Hesuddenlycomplainsoflossofvision
ofbilateraleyes.Ophthalmologistfinds
nothingonexamination.Symptomsare
mostprobablydueto

a)Bilateralopticneuritis
b)Posteriorinferiorcerebellararteryinfarct
c)Malingering
d)Factitiousdisorder
CorrectAnswer-D
Ans.D.Factitiousdisorder
Factitiousdisorder
ItisalsoknownasHospitaladdiction,hospitalhoboes,or
Professionalpatient.
ThetermMunchausensyndromeisusedforthosepatientswho
repeatedlysimulateorfakediseases(intentionally)forthesole
purposeofobtainingmedicalattention.Thereisnoother
recognizablemotive(incontrasttomalingering).
ThetypicalpresentationofMunchausensyndromeischaracterized
byarestlessjourneyfromdoctortodoctorandhospitaltohospital,
anever-changinglistofcomplaintsandsymptoms.
Thepatienttriestomaintainthesickroletoobtainmedicalattention.

Theremaybeevidenceofearliertreatmentusuallysurgical
procedure,forexample,multiplesurgicalscars(gridironabdomen).

1772.Mostimportantreceptorsinvolvedwith
schizophreniaare?
a)GABAA
b)GABA,,
c)DZd
d)5-HT
CorrectAnswer-C
Ans.C.DZd
[RefKaplan&Saddock's10thlep.470]
Dopaminehypothesisisthemostacceptedhypothesisfor
schizophrenia.Thereishyperactivityofdopaminergicsystem.This
hypothesisissupportedby:
1. Amphetamineandcocainewhichreleasedopamineincentral
synapsesinduceschizophrenialikesymptoms;and
2. Antipsychoticdrugscontroltheschizophrenicsymptomsbyblocking
dopamine(D2)receptors.

1773.Functionalsomaticdisorderis?
a)Somatizationdisorder
b)Chronicfatiguesyndrome
c)Hypochondriasis
d)Bodydysmorphicdisorder
CorrectAnswer-B
Ans.B.Chronicfatiguesyndrome
Functionalsomaticsyndromes:
Theyarecharacterizedmorebysymptoms,sufferinganddisability
thanbydiseasespecific,demonstrableabnormalitiesofstructureor
function,i.e.Thereisreportingofsomaticsymptomsandresultant
disabilityratherthanontheevidenceofunderlyingconvention
diseaseprocess
Threemostcommonfunctionalsomaticsyndromesarefibromyalgia,
Irritablebowelsyndromeandchronicfatiguesyndrome.

1774.DHATsyndromeis?
a)Passageofbloodinurine
b)Passageofsemeninurine
c)Passageofpusinurine
d)None
CorrectAnswer-B
Ans.B.Passageofsemeninurine
[RefEncyclopediaofmulticulturalpsychologyp.135]
Dhatsyndromeisaculture-boundsyndromeprevalentinIndian
subcontinentinwhichmalepatientsreportthattheysufferfrom
prematureejaculationorimpotence,andbelievethattheyare
passageofsemen(dhat)inurine.
Theconditionhasnoknownorganiccause.

1775.SuiciderateinIndiais?
a)10.5/100,000
b)12.5/100,000
c)14.5/100,000
d)18/100,000
CorrectAnswer-A
Ans.A.10.5/100,000
[RefInternet;Indian1Psychiatry.2012Oct-Dec;54(4):304-
319.doi:10.4103/0019-5545.104793]
Indiaranks43rdindescendingorderofratesofsuicidewitharateof
10.6/100,000reportedin2009(WHOsuiciderates).
InthemostrecentNationalCrimeRecordsBureau(NCRB;Ministry
ofHomeAffairs)reporttheratein2010roseto11.4per100,000
population.
Themale:femalesuicideratiowas1.78inIndiain2008and2009.

1776.Whichofthefollowingisnotaculture
boundsyndrome?
a)Amok
b)Latah
c)Dhat
d)Von-Gogh
CorrectAnswer-D
Ans.D.Von-Gogh
[RefNirajAhuja6'/ep.65]
Importantculture-boundsyndromes
Ataquedenervios
Dhatsyndrome
Khyalcap
Ghostsickness
Kufungisisa
Maladimoun
Nervios
Shenjingshuairuo
Taijinkyofusho
Susto
Amok
Koro
Latah
Windigo

1777.A25yearsoldmaleisnothappywith
itsgenderandisalwaysindistressdue
tothis.Hewantstochangesexand
havevagina.Itcomesunder?

a)Transsexualism
b)Dualroletransvestism
c)Genderdysphoria
d)Sexualmaturation
CorrectAnswer-C
Ans.C.Genderdysphoria
[RefEncyclopediaofrelationshipsacrossthelifespanp.191]
*Genderdysphoria(formerlyGenderIdentityDisorder)
-Itisdefinedbystrong,persistentfeelingsofidentificationwiththe
oppositegenderanddiscomfortswithone'sownassignedsexthat
resultsinsignificantdistressorimpairment.
-Inthesecases,theassignedsexandgenderdonotmatchthe
person'sgenderidentity,andthepersonistransgender.

1778.Scatologiais?
a)Eatingdisorder
b)Sleepdisorder
c)Paraphilia
d)Defensemechanism
CorrectAnswer-C
Ans.C.Paraphilia
[Ref:NirajAhuja6th/ep.133,134]
Scatologia,alsocalledCoprolalia,isadeviantsexualpracticein
whichsexualpleasureisobtainedthroughthecompulsiveuseof
obscenelanguage.
Theaffectedpersoncommonlysatisfieshisdesiresthroughobscene
telephonecalls,usuallytostrangers.

1779.DrugofchoiceforTourettesyndrome
is?
a)Haloperidol
b)Amantidine
c)Propanolol
d)Diazepam
CorrectAnswer-A
Ans.A.Haloperidol
[RefKaplan&Saddock10`Vep.557]
Medicationforticsuppression(Tourretesyndrome)
Neuroleptics-Pimozide,alongwithhaloperidolandfluphenazineare
themedicationswiththemostprovenefficacyincontrollingtics.

1780.Loadingdoseofdiazepamforalcohol
withdrawalis?
a)80mg
b)50mg
c)40mg
d)20mg
CorrectAnswer-D
Ans.D.20mg
Rigid:
10mgfourtimesdaily
Flexible:
10mgevery4-6hoursasneededbasedonsymptomstoa
maximumof60mg/day
Frontloading:
20mgevery2-4hoursuntilsedationisachieved;then10mgevery
4-6hoursasneededtoamaximumof60mg/day

1781. TCAsarecontraindicatedinallofthe
followingexcept?
a)Narrowangleglaucoma
b)Prostatehypertrophy
c)ApatientonMOAinhibitors
d)Impairedrenalfunction
CorrectAnswer-D
Ans.D.Impairedrenalfunction
[RefLippincottp.81]
CommonContraindicationsofTCAs
1. Hypersensitivitytothemedicines
2. Cardiacconductionabnormalities
3. Within14daysofMAOinhibitors
4. Urinaryretention
5. Narrowangleglaucoma
6. Prostateenlargement
7. Shouldbeusedcautiouslyinsuicidaltendency,schizophrenia,
seizuredisorders,paranoia,impairedliverfunctions.
8. Safetyinnotestablishedinpregnancyandlactation.Not
recommendedforchildrenless12yearsofage.

1782.DOCforschizophrenicpatientwith
poororalabsorptionis?
a)Clozapine
b)Fluphenazine
c)Sulpride
d)Penfluridol
CorrectAnswer-B
Ans.B.Fluphenazine
[Ref:TherAdvPsychopharmacol.2014Oct;4(5):198-219.doi:
10.1177/2045125314540297]
Long-actinginjectable(LAI)antipsychotics(APs)(LAIAPs)have
provedeffectiveinschizophreniaandotherseverepsychotic
disordersbecausetheyassurestablebloodlevels,leadingtoa
reductionoftheriskofrelapse.
LAIsbypasstheinitialdeactivatingprocessbyavoidingfirst-pass
metabolismintheliver.

1783.
Whichofthefollowingisnotaside
effectofquetiapine?
a)Drymouth
b)Hairloss
c)Suddencardiacdeath
d)Dyspepsia
CorrectAnswer-B
Ans.B.Hairloss
[RefGoodman&Gilmanli'Vep.463-466]
Quetiapineisanatypicalantipsychoticusedforthetreatmentof
schizophrenia,bipolardisorder,andmajordepressivedisorder.
VeryCommon Lesscommon
Rare
Drymouth
Highbloodpressure
ProlongedQTinterval
Orthostatic
Dizziness
Suddencardiacdeath
hypotension
Headache
Highbloodcholesterol Syncope
Elevatedserum
Somnolence
Diabeticketoacidosis
triglycerides
Nausea
Abdominalpain
Restlesslegssyndrome
Hyponatraemia,lowblood
Vomiting
Constipation
sodium.
Increased
Increasedappetite
Jaundice
appetite
Increasedliver
Sorethroat
Pancreatitis
enzymes
Trouble
Backache
Agranulocytosis
moving
Rapid

Rapid
Insomnia
Seizure
heartbeat
Weakness
Fatigue
Cardiomyopathy
Pain
Suicidalideation
Dyspepsia
Priapism
(Indigestion)
Neurolepticmalignant
Peripheraledema
syndrome
Dysphagia
TardiveDyskinesia.
Weightgain
invalidquestionid

This post was last modified on 30 July 2021